{ "Tag": [ "AMC", "AMC 10", "AMC 10 A", "symmetry", "blogs", "number theory", "least common multiple" ], "Problem": "Here's a topic for discussing problems on the 2005 AMC10A. I'll start off.\r\n\r\nEmbarrassingly, I spent 20 minutes on #24 by misreading the instructions and not noticing that n>1. Luckily, there was no choice for 2, because that's what I got first. :blush:", "Solution_1": "#17 might have been the most interesting problem, since it can be solved with a symmetry approach like the ones discussed in [url=http://www.artofproblemsolving.com/Forum/weblog_entry.php?e=60]Mr. Rusczyk's blog[/url]. Well, sort of. Since we have an arithmetic sequence,\r\n\r\nAB + BC + CD + DE + EA = 5(middle segment)\r\n\r\nBut AB + BC + CD + DE + EA = 2(A+B+C+D+E) = 2(3+5+6+7+9) = 60\r\n\r\nSo middle segment = 12", "Solution_2": "Man, I was so disappointed after missing #22 about the sets of multiples of 4's and 6's. You had to look for the LCM of those numbers, and my mind ignorantly thought for a moment that LCM (4,6) = 24, so I picked 333, which I thought should have been 334. Right after the test, I realized my mistake. :(", "Solution_3": "[quote=\"keta\"]Man, I was so disappointed after missing #22 about the sets of multiples of 4's and 6's. You had to look for the LCM of those numbers, and my mind ignorantly thought for a moment that LCM (4,6) = 24, so I picked 333, which I thought should have been 334. Right after the test, I realized my mistake. :([/quote]\r\n\r\ndude lcm of 4 and 6 is 12", "Solution_4": "Yeah, he said he realized his mistake right after the test. Anybody else think #24 was a really nice problem? The whole difference of squares...and number 8 was just the diagram used to prove the Pythagorean Theorem, which I'm sure most of us are familiar with.", "Solution_5": "Most of them were great. I agree that 17 and 24 were cool. Was there any way to do 25 besides $A=\\frac 12 ab\\sin\\theta$? I would have gotten that one had I known the formula before taking the test. Instead I omitted it and our proctor explained the formula afterwards.", "Solution_6": "For number 25, you don't need the sine formula... just draw a couple of lines.\r\nConnecting DC, the areas of triangles ADC and DBC have the ratio 19/25, (same height)\r\nand similarly, connecting ED, the areas of triangles AED and EDC have the ratio 14/42.\r\nI almost left 24 blank, but got it in the last 3 minutes... is trial and error the only way to do it?", "Solution_7": "[quote=\"sky9073\"]For number 25, you don't need the sine formula... just draw a couple of lines.\nConnecting DC, the areas of triangles ADC and DBC have the ratio 19/25, (same height)\nand similarly, connecting ED, the areas of triangles AED and EDC have the ratio 14/42.\nI almost left 24 blank, but got it in the last 3 minutes... is trial and error the only way to do it?[/quote]\r\n\r\nWell, we know that both n and n+48 have to be squares of prime numbers, and 48 can be expressed as 23+25 or 9+11+13+15 or 3+5+7+9+11+13, so there's only 3 possibilities for n and n+48 being squares. Check all 3 and you'll find that only n=121 and n+48=169 works.", "Solution_8": "i personally like the [i]trefoi[/i]l problem\r\nalso the one where we had an arithmetic sequence.\r\nhated octagon problem (forgot that the sqrt(2) was divided by 2... D'OH!) hence I got 7 intead of 7/2", "Solution_9": "What I didn't like about 25 was that my friend who isn't really good at math assumed the the triangles were right in 25, and got it right, but I checked, and found that they weren't, and didn't answer it.", "Solution_10": "I'm really annoyed at myself about 25 now. I knew the 1/2ab(sinC) formula but I only looked at it in the last 2 minutes because I thought: Wow, #25, must be too hard, let's check my answers first. :(\r\n\r\nKudos to you who got it :D", "Solution_11": "i screwed up on reading 21 and thought it was the other way around....\r\ni can't believe I missed 12 for number 15\r\nalso, i marked number 23 wrong even though I had the right answer...\r\nand I couldn't really get an \"actual\" solution to 19 w/o guessing", "Solution_12": "does anyone know why that soccer problem is 1/5?? my intuition would tell me it would be 1/2, but apparently not....", "Solution_13": "Intuition is often not good enough my friend :P \r\nThere were 4 cases where B was 2nd and A won.\r\nOne of them consisted of 4 games (ABAA I think).\r\nlet p be the probability of a 5 game occurence with A winning and B winning the second game\r\nso we can write 2p (for the ABAA game) + 3p = 1\r\nso 5p = 1\r\nprobability of a 5 game win that fits the requirements is 1/5\r\n(I put 1/4 on the test...)", "Solution_14": "hmm okay.. maybe i dont understand it well b/c i dont really know the prerequisities.... (i didnt take the AMC A, so this is based upon what other ppl told me)\r\n\r\n5 games; Team B won game 2 and you want to find the probability that Team A wins the series?", "Solution_15": "no, team B wins game 2, team A wins the series.\r\n\r\nFind probability that team B wins game 1 also.", "Solution_16": "[quote=\"yif man12\"][quote=\"sky9073\"]For number 25, you don't need the sine formula... just draw a couple of lines.\nConnecting DC, the areas of triangles ADC and DBC have the ratio 19/25, (same height)\nand similarly, connecting ED, the areas of triangles AED and EDC have the ratio 14/42.\nI almost left 24 blank, but got it in the last 3 minutes... is trial and error the only way to do it?[/quote]\n\nWell, we know that both n and n+48 have to be squares of prime numbers, and 48 can be expressed as 23+25 or 9+11+13+15 or 3+5+7+9+11+13, so there's only 3 possibilities for n and n+48 being squares. Check all 3 and you'll find that only n=121 and n+48=169 works.[/quote]\r\n\r\nAlternatively, let n = k^2 and n+48 = l^2. Then l^2-k^2 = 48, so (l+k,l-k) = (1,48), (2,24),... (48,1). Of these, only the pairs where k = 1, 4, or 11 work, so n = 1, 16, or 121. But n > 1, and 16 isn't the square of a prime, so only n=121 works.", "Solution_17": "[quote=\"pkothari13\"]does anyone know why that soccer problem is 1/5?? my intuition would tell me it would be 1/2, but apparently not....[/quote]\r\n\r\nAs for your intuition, it is true that team B has a 1/2 chance of winning game 1. However, that's not the question - the question is actually 'out of all the cases where team B wins game 2 but team A wins the series, what is the probability that team B will win the first game'. Since the set of cases where team B wins game 2 but team A wins the series is a different set from the set of all games, it's no longer 1/2 - we have additional conditions that we must satisfy.\r\n\r\nThe four possible outcomes, and the probability that they occur, is\r\nABAA = (1/2)^4 = 1/16\r\nABABA = (1/2)^5 = 1/32\r\nABBAA = 1/32\r\nBBAAA = 1/32\r\n\r\nWe are given that one of these occured (since B won game 2, A won series). One of these occurs 1/16+1/32+1/32+1/32 = 5/32 of the time, so now we need to multiply all of the probabilities by 32/5 so that they add up to 1 (since we know that one of them must occur):\r\n\r\nABAA => 2/5\r\nABABA => 1/5\r\nABBAA => 1/5\r\nBBAAA => 1/5\r\n\r\nAnd there you have it.", "Solution_18": "Even though a \"best of 5\" series usually stops when one team wins 3 games, we can imagine that the series continues for the full 5 games. For example, the event ABAA splits into two outcomes ABAAA and ABAAB. That way, it is clear that there are 5 equally-likely outcomes, and we don't have to worry about some outcomes being weighted more than others.\r\n\r\nAdding such fictitious games to \"best of\" series often makes such problems a lot easier. I recall using that trick in the past, but I can't think of the examples now." } { "Tag": [ "algebra unsolved", "algebra" ], "Problem": "find integer $a$ such that:$x^2-x+a|x^{13}+x+90$", "Solution_1": "[quote=\"amir2\"]find integer $a$ such that:$x^2-x+a|x^{13}+x+90$[/quote]\r\n$x^2-x+a|x^{13}+x+90$\r\nPut $x=0$, we get $a|90$\r\nPut $x=1$, we get $a|92$\r\nPut $x=-1$, we get $a+2|88$\r\nSo $a$ can only be $-1$ or $2$.\r\nWhen $a=-1$, let $x^{13}+x+90=(x^2-x-1)Q(x)$. But when $x^2-x-1=0$, $x^{13}+x+90\\neq 0$. So $a=-1$ does not satisfy the condition.\r\nWhen $a=2$, let $u$ be a root of $x^2-x+2=0$. Then\r\n$u^2=u-2$\r\n$u^3=u^2-2u=-u-2$\r\n$u^6=u^2+4u+4=5u+2$\r\n$u^{12}=25u^2+20u+4=45u-46$\r\n$u^{13}=45u^2-46u=-u-90$ or $u^{13}+u+90=0$.\r\nTherefore $a=2$." } { "Tag": [ "algebra", "polynomial", "LaTeX", "superior algebra", "superior algebra unsolved" ], "Problem": "Prove that sum and product of 2 algebric integers is an algebric integer.\r\n\r\nProof:Suppose a,b are algebric integers then a satisfies a minimal monic polynomial with integer coefficients i.e. let\r\n$a^n +a_1a^{n-1}+...+a_n=0$.\r\n So, $a^n =-(a_1a^{n-1}+...+a_n)$ \u2026\u2026\u2026\u2026..(i)\r\nThus all the higher powers of a can be written in the form of linear combinations of 1,a,\u2026.a^n-1 \r\nSo this set generates Z[a]. Similarly if b satisfies a minimal monic polynomial of degree m, then 1,b,\u2026,b^m-1 generates Z[b].\r\nNow we claim that Z[a,b] is generated by elements of the form aibj where I=0(1)n-1\r\nJ=0(1)m-1.\r\nTo see this take any element p belonging to Z[a,b]. then p is of the form \r\n $P= \u00e5\u00e5x_ij a_ib_j$ . Since there are only a finitely many terms in the expression, starting from the 1st term check the power of a. If it is less than n, we leave the term unchanged and go to the next term. If the power is greater than n we reduce it by the help of (i).After reducing the power of a from each term we then reduce the power of b by the same method. By these operations each term becomes a product of polynomials where the degree of a & b are atmost n-1 and m-1 respectively. Multiplying all the terms and rearranging them we can write p as a linear combination of aibj . Thus aibj generate Z[a,b] as claimed and as the no of such generators is finite we have Z[a,b] a finitely generated Z-module.\r\n\r\nIs my proof correct?It certainly is easy for a double starred herstein problem", "Solution_1": "Please use $\\LaTeX$ correctly (e.g. use a^{n-1} to get $a^{n-1}$, not as you do a^(n-1) ).", "Solution_2": "I am very sorry I dont know latex. :blush: \r\nI have a lot of problems with the summation sign.I know its difficult for you to read but plz tell me if my soln is correct or not.", "Solution_3": "[quote=\"the game\"]Prove that sum and product of $2$ algebric integers is an algebric integer.\n\nProof:Suppose $a,b$ are algebric integers then a satisfies a minimal monic polynomial with integer coefficients i.e. let\n$a^n +a_1a^{n-1}+...+a_n=0$.\n So, $a^n =-(a_1a^{n-1}+...+a_n)$ \u2026\u2026\u2026\u2026..(i)\nThus all the higher powers of a can be written in the form of linear combinations of $1,a,....a^{n-1}$ \nSo this set generates $\\mathbb{Z}[a]$. Similarly if $b$ satisfies a minimal monic polynomial of degree $m$, then $1,b,...,b^{m-1}$ generates $\\mathbb{Z}[b]$.\nNow we claim that $\\mathbb{Z}[a,b]$ is generated by elements of the form $a^i b^j$ where $i=0...n-1$\n$J=0...m-1$.\nTo see this take any element $p$ belonging to $\\mathbb{Z}[a,b]$. then $p$ is of the form \n $p= \\sum_{i,j} x_{i,j} a^ib^j$ . Since there are only a finitely many terms in the expression, starting from the 1st term check the power of $a$. If it is less than $n$, we leave the term unchanged and go to the next term. If the power is greater than $n$ we reduce it by the help of (i).After reducing the power of $a$ from each term we then reduce the power of $b$ by the same method. By these operations each term becomes a product of polynomials where the degree of $a$ & $b$ are atmost $n-1$ and $m-1$ respectively. Multiplying all the terms and rearranging them we can write $p$ as a linear combination of $a^ib^j$ . Thus $a^ib^j$ generate $\\mathbb{Z}[a,b]$ as claimed and as the no of such generators is finite we have $\\mathbb{Z}[a,b]$ a finitely generated $\\mathbb{Z}$-module.\n\nIs my proof correct?It certainly is easy for a double starred herstein problem[/quote]\r\nThe above should be what you want :) (click on quote to see the plain text for the formulas)\r\n\r\nTo the prove:\r\nThe part \"finetely generated $\\implies$ algebraic integers\" is missing.\r\nThe rest seems ok." } { "Tag": [], "Problem": "Can 10th grader apply RSI?\r\nThanks!", "Solution_1": "From what I've heard, you should only apply to RSI in 10th grade if you intend to graduate in 11th grade." } { "Tag": [ "invariant", "linear algebra", "linear algebra unsolved" ], "Problem": "This is just beautiful: suppose that $A,B$ are invertible complex matrices which commute to their commutator. Then all eigenvalues of the commutator are roots of the unity.", "Solution_1": "I suppose the commutator here is $ABA^{-1}B^{-1}$ and not $AB-BA$.", "Solution_2": "Yes, sorry.", "Solution_3": "Let $\\lambda$ be an eigenvalue of $[A,B]=ABA^{-1}B^{-1}$. because $A,B$ commute with $[A,B]$, the $\\lambda$-eigenspace of $[A,B]$ is invariant under both $A$ and $B$. By restricting attention to this eigenspace, we may as well assume that $[A,B]=\\lambda$. \r\n\r\nWe now have $AB=\\lambda BA$. Let $\\sigma$ be an eigenvalue of $A$ and let $x$ be a $\\sigma$-eigenvector of $A$. We then have $ABx=\\lambda BAx=\\lambda\\sigma Bx$, i.e. $Bx$ is a $\\lambda\\sigma$-eigenvector of $A$. We repeat this procedure to conclude that for all positive integers $n,\\ \\lambda^{n}\\sigma$ is an eigenvalue of $A$. Since $A$ has finitely many eigenvalues, this can only happen if $\\lambda$ is a root of unity." } { "Tag": [ "LaTeX" ], "Problem": "I'm hoping to get a summation symbol like this:\r\n\r\n$ \\displaystyle\\sum_{k\\equal{}0}^n$\r\n\r\nin a fraction like this: $ \\displaystyle\\frac{\\sum_{k\\equal{}0}^n}{\\sum_{j\\equal{}0}^u}$.\r\n\r\nBut I can't figure out how to get the summation sign to render stylistically in a fraction.\r\n\r\nHelp appreciated. Thanks!", "Solution_1": "Use \\sum\\limits... \r\n$ \\frac{\\sum\\limits_{k\\equal{}0}^{n}}{\\sum\\limits_{j\\equal{}0}^{u}}$" } { "Tag": [ "calculus", "integration", "function", "trigonometry", "logarithms", "complex numbers", "calculus computations" ], "Problem": "Compute the \"antiderivative\" of the function $ln(sinx)$", "Solution_1": "There is no elementary antiderivative. The best we can do is to evaluate some definite integrals for that function.", "Solution_2": "Starting with the argument I just posted [url=http://www.artofproblemsolving.com/Forum/viewtopic.php?t=152690]here[/url] but taking the real part instead of the imaginary part, we get this result:\r\n\r\n$\\frac1n\\ln|2\\sin2\\theta|=\\ln|1-e^{i\\theta}|=\\sum_{n=1}^{\\infty}\\frac{\\cos n\\theta}{n}.$\r\n\r\nIntegrating, we get\r\n\r\n$\\int_{0}^{x}\\frac12\\ln|2\\sin2\\theta|\\,d\\theta=\\sum_{n=1}^{\\infty}\\frac{\\sin nx}{n^{2}}.$\r\n\r\nOne could, with sufficient patience, use this to calculate certain definite integrals, such as\r\n\r\n$\\int_{0}^{\\pi}\\ln\\sin x\\,dx.$ Carcul claimed as much." } { "Tag": [ "function", "real analysis", "real analysis solved" ], "Problem": "Does there exists a real function f such that for all real x the set of solutions of the equation f(t)=x is in bijection with the reals?", "Solution_1": "Of course. f(0,abcdef...)=0,ace..., where 0,abcde.. is a decimal representation of a number.", "Solution_2": "What a simple example! Thank you. I start to ask some very easy questions, but I like them. I don't know why. \r\n Another (maybe stupid question): does there exist a continuous function with those properties?", "Solution_3": "Yes, it exists, There exists a continious function f which maps the segment to a square. If f(t)=(x(t),y(t)), then x(t) is such a function." } { "Tag": [ "algorithm", "analytic geometry", "graphing lines", "slope", "trigonometry", "ratio" ], "Problem": "I am looking for some justification as to why when you have a line in Ax+By+C=0 form you divide by :pm: :sqrt: A :^2: +B :^2: to get it into normal form and then the coefficents represent cos :phi: x + sin :phi: y - p =0. \r\n\r\nDoes anyone know where I can find a link that would tell me this?\r\n\r\nI have been trying to do this myself but cannot find justification for the C term.\r\n\r\nThanks!", "Solution_1": "What do you mean that you cannot find a justification for the C term? You need to explain more what you're asking about for us (well, me at least) to understand and help.", "Solution_2": "Okay well I am trying to find a justification as to why one preforms a certain algorithim to get from general form of an equation of a line, Ax+By+C=0 to Normal form, xcos( :phi: )+ysin( :phi: )-p=0.\r\n\r\nThe algorithim is that you find sqrt(A^2+B^2), and give it a sign that is opposite of C's sign. Then you divide all terms of Ax+By+C=0 by this quantity, and now the line is in normal form.\r\n\r\nI have figured out why this is true for the A/ :pm: :sqrt: A :^2: +B :^2: and B/:pm: :sqrt: A :^2: +B :^2: but I can't figure out why it works w/the C term. (FYI I did this by changing Ax+By+C=0 to y=(-A/B)x+(-C/B), since the slope of this line is -A/B I constructed a triangle w/ A*n, B*n, and n*sqrt(A^2+B^2) leg lengths, n is a constant, and formed trig ratios w/ 90+ :phi: , to find out why the coefficents were in the normal form now.)", "Solution_3": "[quote=\"mrblueskies\"]I have figured out why [b]this[/b] is true ...[/quote]\r\n(emphasis mine)\r\n\r\nWhat is the \"this\" you are refering to? I understand everything you've written. I understand the operation you're carrying out. What I don't understand is what you think is happening to specific coefficients -- what does the \"this\" in that sentence refer to? \"Normal form\" is a property of the equation of the line; it isn't a property of the individual coefficients.", "Solution_4": "\"this\" as in the algorithim, I found out why this algorithim transformes the line so the coefficents of x and y represent cos phi and sine phi. I cannot however find out why the new C term after preforming the algorithim represents p.", "Solution_5": "Hey,\r\n\r\nWell...if you have the line Ax + By + C = 0, dividing by :pm: :sqrt: (A^2 + B^2) will give you the hypotenuse of a right triangle with sides A and B. In our case, since the slope is -A/B, say n = arctanA/B. Dividing gives us that sin(n)x + cos(n)y - C/root(A^2 + B^2) = 0. Is that the p you are trying to \"justify\"?", "Solution_6": "I don't understand what you think p is -- it's just a number, which is the result of dividing C by :sqrt:(A2 + B2). What significance are you attributing to it? All it's doing is telling you that there is some constant left over.", "Solution_7": "p is the perpendicular distance of the line from the origin.\r\n\r\nIf the line is at an angle :theta: to the x-axis (so that tan( :theta: )= -A/B) then the perpendicular distance p of the line from the origin is given by\r\n\r\np = ycos( :theta: ) - xsin( :theta: )\r\n\r\nif the line crosses the y-axis above the origin, or\r\n\r\np = xsin( :theta: ) - ycos( :theta: )\r\n\r\nif the line crosses the y-axis below the origin, where (x,y) is any point on the line.\r\n\r\nIn the first case, set :phi: = :theta: + :pi: /2 and in the second case set :phi: = :theta: - :pi: /2, and then in both cases you have\r\n\r\np = xcos( :phi: ) + ysin( :phi: )\r\n\r\nand, also in both cases, tan( :phi: ) = -cot( :theta: ) = B/A" } { "Tag": [ "integration", "number theory unsolved", "number theory" ], "Problem": ":roll: What is the product $1 \\cdot 2 \\cdot 3 \\cdot 4 \\cdot ... \\cdot n$?", "Solution_1": "$1 \\cdot 2 \\cdot 3 \\cdot ... \\cdot n = n!$ ~ $\\sqrt{2\\pi n} e^{n \\ln(n)-n+\\frac{1}{12n}-...}$", "Solution_2": "$\\Gamma(n+1) = \\int_{0}^{\\infty} t^n e^{-t} \\, dt$\r\n\r\nAlternately:\r\n\r\nhttp://mathworld.wolfram.com/StirlingsSeries.html", "Solution_3": "For completness sake:\r\n$n! = \\Gamma ( n + 1 )$ for all positive integer $n$.", "Solution_4": "[quote=\"sagar1331\"]:roll: What is the product 1*2*3*4*.........n?[/quote]\r\nMy friend says:\r\nEvgeny: $1*2*...*n=n*(n-1)*...*1$.\r\nRoman: It's very silly question. ;)", "Solution_5": "I agree with Roman.. :P \r\n\r\n(Sorry for the useless post)\r\n\r\nOh, well, just to make it more valuable : http://mathworld.wolfram.com/Factorial.html\r\n\r\n;)" } { "Tag": [ "inequalities", "AMC", "USA(J)MO", "USAMO", "inequalities solved" ], "Problem": "given positive reals a1,a2,... a5 satisfy \\sum 1/(1+a)=1\r\nProve that \\sum a/(4+a 2 ) \\leq 1.", "Solution_1": "you joined CWMO?\r\nI got a silver this yr..\r\n\r\nThe official solution used a substitution.\r\nanyone wants me to post it?", "Solution_2": "West Math Olympiad?", "Solution_3": "[quote=\"liyi\"]West Math Olympiad?[/quote]\r\n\r\nyep, something like Western China Mathematical Olympiad", "Solution_4": "Very beautiful and quite challenging. Do you see anything similar with Usamo 2003 or Japan 1997?\r\n Let 1/(1+a_i)=x_i. Then a_1=(x_2+x_3+x_4+x_5)/x_1 and so on. The inequality becomes sum x_1*(x_2+x_3+x_4+x_5)/(4x_1^2+(x_2+x_3+x_4+x_5)^2)<=1. We prove that this is true for all x_i, not necessarily with sum 1. Since it is homogenuous, we can assume that sum x_i=5. It becomes sum x_i*(5-x_i)/(4+(x_i-1)^2)<=5. It is obvious that LHS is at most sum x_i(5-x_i)/4 and from now on everything is clear since sum x_i=5 and we can apply Cauchy ( rest in peace; beautiful inequality had he discovered).", "Solution_5": "For $x>0$, we have $\\displaystyle \\frac{x}{4+x^2} \\leq \\frac{1}{20}\\left(1+\\frac{15}{1+x}\\right)$. In fact, this is equivalent to $(x-4)^2(x+4)\\geq0$.\r\n\r\nAdding, we get $\\displaystyle \\sum_{i=1}^5\\frac{a_i}{4+a_i^2} \\leq \\frac{1}{20}\\sum_{i=1}^5 \\left(1+\\frac{15}{1+a_i}\\right) = 1$.", "Solution_6": "[quote=\"fuzzylogic\"]For $x>0$, we have $\\displaystyle \\frac{x}{4+x^2} \\leq \\frac{1}{20}\\left(1+\\frac{15}{1+x}\\right)$. [/quote]\n\nSomebody asked me how I came up with this inequality. I have to give credit to [b]hxtung[/b]. I learned this technique from his post [url=http://www.artofproblemsolving.com/Forum/viewtopic.php?t=180]Poland 1996 inequality[/url]. This technique can be applied to Japan 1997 and USAMO 2003 as well. \n\nFor this particular problem, I expect that $\\displaystyle \\frac{x}{4+x^2} \\leq \\frac{ax+b}{1+x}$ for some $a,b$. This is equivalent to \n$f(x) = (ax+b)(4+x^2) - x (1+x) \\geq 0$, where $f(x)$ is a cubic polynomial.\n\nSo how do we find $a,b$? I expect the equality holds when all $a_i$ are equal to 4 (from the given condition). So I expect $f(4)=0$. Further, I expect $(x-4)^2$ divides $f(x)$, otherwise $f(x)\\geq0$ can't be true. So $f'(4)=0$ too. From $f(4)=f'(4)=0$, I found that $a=\\frac{1}{20}$ and $b=\\frac{4}{5}$. The rest is just verify that the inequality is true.\n\n\nThis problem can be extended to $n$ variables as follows:\n[quote]Let $a_1,a_2,\\ldots, a_n > 0$ satisfying $\\displaystyle \\sum_{i=1}^n \\frac{1}{1+a_i}=1$. Prove that\n\\[\n\\sum_{i=1}^n \\frac{a_i}{(n-1)+a_i^2} \\leq 1.\n\\][/quote]\r\n\r\nCheers! :D :D" } { "Tag": [ "ratio", "combinatorics unsolved", "combinatorics" ], "Problem": "Numbers $ 1,2,...,64$ are written on a $ 8\\times 8$ board. For every two numbers $ a,b$ with $ a>b$ in the same row or column, the ratio $ \\frac{a}{b}$ is calculated. The characteristic of the board is defined as the least of these ratios. Find the greatest possible value of a characteristic.", "Solution_1": "Is the answer 64/7 or am I being stupid?", "Solution_2": "It has to be at most $ \\frac{64}{14}$ (there are only 13 numbers less than 14, so 14 appears in the denominator of some ratio).", "Solution_3": "Ah, sorry - somehow I completely missed \"or column\" when I read that!\r\n\r\n64/14 is indeed right.", "Solution_4": "[hide=\"Ponderings\"]Let the characteristic be $ r$. In the row containing 1, note that the next smallest entry is at least $ r$. The next after that is at least $ r^2$; otherwise, the ratio between the two would be less than $ r$, meaning that $ r$ would not be the characteristic. Then by similar reasoning, the largest entry in the row is at least $ r^7$. To maximize $ r$, you want the largest entry in this row to be as high as possible. Therefore, $ r<\\sqrt[7]{64}$.\n\nYou can do better by considering the maximum among the least entries in each row. This maximum is at least 8, so $ 8r^7<64$.\n\nFurther note that since the entries are integers, you will always be considering the ceiling of the $ r$ powers at each step, so that deflates it even more. As an example, the second smallest entry in the row containing 1 is at least 2. The maximum among second smallest entries in each row is at least 16. Then we have $ 16r^6<64$.\n\nI'm pretty sure this continues to the 7th smallest (or second largest) entry in the row, suggesting the maximum characteristic is less than $ \\frac{64}{56}$.[/hide]\r\n\r\nThis method doesn't lend itself to determine whether characteristics can actually be achieved, though." } { "Tag": [ "calculus", "integration", "algebra", "function", "domain", "Ring Theory", "algorithm" ], "Problem": "Let [b]F [/b]be a field. State under what circumstances, if any, [b]F[/b] is respectively an Euclidean Domain, Principal Ideal Domain, Uniqe Factorization Domain. Justify your answers\r\n\r\nThank you very much in advanced", "Solution_1": "What do you mean? If you have a certain field which satisfies the criteria for those domains, it'll be that.\r\n\r\nFor instance construct a norm over your field as follows \r\n$ N(a) \\equal{} 1$ for any $ a \\not \\equal{} 0$\r\n$ N(0) \\equal{} 0$\r\nThen your field is an Euclidean Domain.\r\n\r\nTo be a PID, its really easy to see, since every element in a field is a unit. Hence with the Euclidean Algorithm there won't be any non-zero remainders...", "Solution_2": "There's no need to define a Euclidean valuation since a field is vacuously Euclidean: there are no nonzero remainders upon division. Since Euclidean => PID => UFD all else follows. But, of course, it's trivial to directly verify the other claims too." } { "Tag": [], "Problem": "Roslyn has ten boxes. Six of the boxes contain pencils, three of\nthe boxes contain pens, and two of the boxes contain both pens\nand pencils. How many boxes contain neither pens nor pencils?", "Solution_1": "Call the number of boxes with neither pens nor pencils $ x$. Using [url=http://www.artofproblemsolving.com/Wiki/index.php/PIE]PIE[/url], we know that $ 6\\plus{}3\\minus{}2\\plus{}x\\equal{}10\\implies x\\equal{}\\boxed{3}$." } { "Tag": [ "combinatorics proposed", "combinatorics", "Bulgaria" ], "Problem": "Find the greatest positive integer $n$ such that we can choose $2007$ different positive integers from $[2\\cdot 10^{n-1},10^{n})$ such that for each two $1\\leq i 2d.\r\n\r\nI have an algebraic solution, but I am looking for a solution using Erd\u00f6s Mordell", "Solution_1": "Let distances from $ K$ to the $ B^{'}C^{'},C^'A^',A^'B^',BC,CA,AB$ be $ x,y,z,a,b,c$. Then we have $ x^2 = bc,y^2 = ca,z^2 = ab$.(this can be proved as follow: Let $ P,Q,R$ be projection of $ K$ on $ AB,B^'C^',CA. \\angle KQP = \\angle KC^'P = \\angle KB^'C^' = \\angle KRQ$ and similarly, $ \\angle KPQ = \\angle KQR$. Hence triangle $ KQP$ and triangle $ KRQ$ are simialr, and we obtain $ x^2 = bc$). By Erd\u00f6s Mordell theorem, $ KA + KB + KC\\geq 2(a + b + c)\\geq 2(\\sqrt {bc} + \\sqrt {ca} + \\sqrt {ab}) = 2(x + y + z) = 2d$. Obviously, Equal isn't hold. Hence $ KA + KB + KC > 2d$.", "Solution_2": "Thanks Euler. :)", "Solution_3": "Sorry but I can't contain my curiosity:\r\n\r\nWhat are F1 and F2 in your avatar (I am supposing that O and N are the circuncircle and the Nagel point) :oops:", "Solution_4": "[quote=\"m.candales\"]Sorry but I can't contain my curiosity:\n\nWhat are F1 and F2 in your avatar (I am supposing that O and N are the circuncircle and the Nagel point) :oops:[/quote]\r\n$ F_1,F_2$ is two Fermat point and $ N$ is nine-point center. My avatar is picture of Lester theorem( http://www.mathlinks.ro/viewtopic.php?search_id=298908544&t=16494)" } { "Tag": [ "function", "calculus", "calculus computations" ], "Problem": "$f: R\\to R$\r\n$fx)= x+m$, for x<=1\r\n $2mx-1$, for x>1\r\nIf the function is surjective, what is the $m$'s interval ?", "Solution_1": "$f$ automatically takes all values in $(-\\infty,m+1]$. We then need that $2mx-1$ for $x>1$ take values in the rest of $\\mathbb{R}$. So the minimum of $2mx-1$ must be $m+1$ or less. When $x\\to1$, $2mx-1\\to 2m-1$, so:\r\n\r\n$2m-1\\leq m+1$\r\n\r\nyielding,\r\n\r\n$m\\leq2$\r\n\r\nHowever, it's obvious that $2mx-1$ must go to $\\infty$ for some $x$ also, which means that $m>0$. Hence,\r\n\r\n$m\\in(0,2]$" } { "Tag": [ "linear algebra", "matrix", "combinatorial geometry", "linear algebra unsolved" ], "Problem": "Let $ A \\in \\mathcal{M}_n(\\mathbb{C})$, $ H(A)\\equal{}\\{ X^*AX \\quad | \\quad X \\in \\mathbb{C}^n, X^*X\\equal{}1 \\}$ ($ X^*$ denotes transpose conjugate).\r\nFor which $ n\\in \\mathbb{N}^*$, do we have the following statement:\r\n\r\nIf $ H(A)$ is the convex hull of the eigenvalues of $ A$, then $ A$ is normal.", "Solution_1": "Answer : For $ n\\leq 4$ \r\n This year it's really a nice problem at ENS , especially in compare with the other one ,but unfortunately I didn't succeed ... :(", "Solution_2": "I have a partial proof: with Facts 1-2 we can prove that the answer is YES if $ n\\leq3$. Unfortunately we cannot conclude if $ n\\equal{}4$.\r\n\r\nThere exists $ P$ unitary matrix s.t. $ P^{\\minus{}1}AP$ is triangular. Thus we may assume that $ A\\equal{}[a_{ij}]$ is upper triangular with $ a_{ii}\\equal{}\\lambda_i$. $ X^*AX\\equal{}\\sum_i{\\lambda_i|x_i|^2}\\plus{}\\sum_{j>i}a_{ij}\\bar{x_i}x_j$ where $ X\\equal{}[x_i]$ satisfies $ \\sum_i{|x_i|^2}\\equal{}1$. Let $ \\mathcal{C}$ be the convex hull of $ (\\lambda_i)$ and $ \\mathcal{B}$ its edge.\r\nThe question is: must $ A$ be a diagonal matrix ?\r\nFact 1: let $ i,j$ s.t. $ \\lambda_i\\not\\equal{}\\lambda_j$ and $ [\\lambda_i,\\lambda_j]\\subset\\mathcal{B}$. \r\nLet $ X$ s.t. $ x_k\\equal{}0$ except $ x_i\\equal{}\\alpha_i/\\sqrt{2},x_j\\equal{}\\alpha_j/\\sqrt{2}$ where $ |\\alpha_i|\\equal{}|\\alpha_j|\\equal{}1$. $ X^*AX\\equal{}\\lambda_i/2\\plus{}\\lambda_j/2\\plus{}\\bar{\\alpha_i}\\alpha_j{a_{ij}}/2$. $ X^*AX\\in\\mathcal{C}$ and $ \\lambda_i/2\\plus{}\\lambda_j/2\\in\\mathcal{B}$. Then the complex number $ \\bar{\\alpha_i}\\alpha_j{a_{ij}}$ is always in the same half plane for all $ \\alpha_i,\\alpha_j$. This fact implies that $ a_{ij}\\equal{}0$.\r\nFact 2: let distinct $ i,j$ s.t. $ \\lambda_i\\equal{}\\lambda_j$. \r\nIf $ \\mathcal{C}\\not\\equal{}\\{\\lambda_i\\}$ then $ \\bar{\\alpha_i}\\alpha_j{a_{ij}}$ is in a fixed angular sector (eventually in a half plane) for all $ \\alpha_i,\\alpha_j$. This implies that $ a_{ij}\\equal{}0$. \r\nIf $ \\mathcal{C}\\equal{}\\{\\lambda_i\\}$ then $ \\bar{\\alpha_i}\\alpha_j{a_{ij}}\\equal{}0$ and $ a_{ij}\\equal{}0$." } { "Tag": [ "abstract algebra", "function", "algebra", "polynomial", "topology", "Ring Theory", "superior algebra" ], "Problem": "At the site http://www.math.uga.edu/~bensondj/ I found this remarkable remark:\r\n\r\n[b]Foundations sometimes matter\nThe projective dimension of $\\mathbb C(x,y,z)$ as a module over $\\mathbb C[x,y,z]$ is:\ntwo if the continuum hypothesis holds and three otherwise.[/b]\r\n\r\nI assume C(x,y,z) is a function field over the complex, and C[x,y,z] is a polynomial ring, so I understand that the former is a module over its subring. What is the \"projective dimension\" here?\r\n\r\nLiMa", "Solution_1": "Hi Lima ;-),\r\n\r\nis the general definition unclear?\r\n \r\nhttp://planetmath.org/encyclopedia/ProjectiveDimension.html\r\n \r\nI guess that results of homological algebra may involve foundations by the recursive construction of homotopy equivalences of arbitrary projective resolutions of a module. But I can't imagine what does this matter in the case of finite resolutions :maybe:. Well, perhaps CH admits the construction of curious modules, which may shorten the projective dimension. :-)", "Solution_2": "... after a short google-session ;-)\r\n\r\nCH plays even often a role dealing with projective dimensions. For an example, see\r\n \r\nmpcs.usip.edu/faculty/vas/homepage_sa_UMDja/Semisimple_abstract.pdf\r\n \r\nBut this seems to be quite difficult to understand :/" } { "Tag": [ "function", "logarithms", "real analysis", "real analysis unsolved" ], "Problem": "Can anyone give an example of a function $ f \\in L^p(\\Bbb R)$ such that $ f \\notin L^h(\\Bbb R)$ for any $ h > p$? ($ p$ is finite)", "Solution_1": "Try $ f(x)\\equal{}|x\\ln^2x|^{\\minus{}1/p}$ for $ x\\in (0,1/2)$ and zero otherwise." } { "Tag": [ "calculus", "integration", "trigonometry", "function", "absolute value", "complex analysis" ], "Problem": "MAIN QUESTION:\r\n\r\n\r\nDoes $ |(cos(R^2 cos(2 \\theta)) \\plus{} i sin(R^2 cos(2 \\theta)))| \\equal{} 1$ UNIFORMLY even though \r\nthe absolute value of $ e^{i R^2 e^{i 2 \\theta}}$ is NOT equal to 1?\r\n\r\nAnd if so, IN WHAT CASES THEN DO A NUMBER OF THE FORM $ e^{i \\theta}$ actually have an absolute magnitude of 1? A theta value of $ R^2 cos(2 \\theta)$ seems to work (correct me if I'm wrong), but a theta value of $ R^2 e^{i 2 \\theta}$ doesn't seem to work. I'd like to know why (besides the \"you need to simplify it\" explanation - which does not help with my intuition here). \r\n==\r\n\r\nSo I want to evaluate $ \\int_0^{\\infty} e^{i z^2}$. One of the central steps is to evaluate the integral around the contour line $ z \\equal{} R e^{i \\theta}$. So I substitute in $ z \\equal{} R e^{i \\theta}$ so that I get $ e^{i R^2 e^{i 2 \\theta}} \\equal{} e^{i R^2 cos(2 \\theta) \\minus{} R^2 sin(2 \\theta)}.$. I have to show convergence to 0. This equals $ e^{i R^2 cos(2 \\theta)} e^{ \\minus{} R^2 sin(2 \\theta)} \\equal{} (cos(R^2 cos(2 \\theta)) \\plus{} i sin(R^2 cos(2 \\theta))) e^{ \\minus{} R^2 sin(2 \\theta)}$. So my sources say that the absolute value of this becomes $ e^{ \\minus{} R^2 sin(2 \\theta)},$ implying that $ |(cos(R^2 cos(2 \\theta)) \\plus{} i sin(R^2 cos(2 \\theta)))| \\equal{} 1$ throughout the entire interval. Of course, the absolute value of $ e^{i \\theta}$ is 1 as long as the number is of this form. But then, why can't the absolute value of $ e^{i R^2 e^{i 2 \\theta}}$ also be 1 throughout the range? What properties make a the absolute value of a number of the form $ e^{i \\theta}$ NOT converge to 1 for ANY value of $ \\theta$?\r\n\r\n====\r\n\r\nAnother question: Since I have to evaluate $ \\int_0^{\\infty} e^{i z^2} dz$, where $ dz \\equal{} i e^{i \\theta} d \\theta$ - I have to multiply this equation by that factor. I probably have to do this for the final solution. But since this is bounded by i, and my only objective is to show that this integral converges to 0, this factor of dz should be effectively negligible, right?\r\n\r\n==", "Solution_1": "On your first question- of course not. That absolute value relation only holds when the argument of the sines and cosines is pure real. We do have that $ \\sin^2 t\\plus{}\\cos^2 t\\equal{}1$ everywhere, but that's different.\r\n\r\nThe integral you're trying to find is along the $ x$-axis, so there are no extra multiples- it's just $ \\int_0^{\\infty} e^{iz^2}\\,dz$ there.\r\n\r\nOn the circular arc $ z\\equal{}Re^{i\\theta}$, we can estimate\r\n$ \\left|\\int_{Re^{i\\alpha}}^{Re^{i\\beta}}e^{iz^2}\\,dz\\right|\\le \\int_{Re^{i\\alpha}}^{Re^{i\\beta}}\\left|e^{iz^2}\\right|\\,|dz|\\equal{} \\int_{\\alpha}^{\\beta}\\left|\\exp(iR^2e^{2i\\theta})\\right|\\cdot \\left|iRe^{i\\theta}\\right|\\,d\\theta$\r\n$ \\left|\\int_{Re^{i\\alpha}}^{Re^{i\\beta}}e^{iz^2}\\,dz\\right|\\le \\int_{\\alpha}^{\\beta}\\exp\\left(\\minus{}R^2\\cdot\\text{Im}(e^{2i\\theta})\\right)\\cdot R\\,d\\theta$\r\n\r\nNow, that $ \\sin 2\\theta$ comes from the imaginary part. The $ \\alpha$ and $ \\beta$ you want are zero and $ \\frac{\\pi}{2}$- the first quadrant piece.\r\nProving this integral goes to zero is tricky, because the function we're integrating only goes to zero away from the endpoints. Integration by parts is the most elementary way of dealing with this.", "Solution_2": "[quote=\"Simfish\"]MAIN QUESTION:\n\n\nDoes $ |(cos(R^2 cos(2 \\theta)) + i sin(R^2 cos(2 \\theta)))| = 1$ UNIFORMLY even though \nthe absolute value of $ e^{i R^2 e^{i 2 \\theta}}$ is NOT equal to 1?\n\nAnd if so, IN WHAT CASES THEN DO A NUMBER OF THE FORM $ e^{i \\theta}$ actually have an absolute magnitude of 1? A theta value of $ R^2 cos(2 \\theta)$ seems to work (correct me if I'm wrong), but a theta value of $ R^2 e^{i 2 \\theta}$ doesn't seem to work. I'd like to know why (besides the \"you need to simplify it\" explanation - which does not help with my intuition here). \n[/quote]\r\n\r\nYou forgot the $ e^{ - R^2Sin(2\\theta)}$ part:\r\n\r\n$ e^{iR^2e^{i2\\theta}} = e^{iR^2[Cos(2\\theta) + iSin(2\\theta)]}$\r\n\r\n$ = e^{iR^2Cos(2\\theta) - R^2Sin(2\\theta)}$\r\n\r\n$ = e^{ - R^2Sin(2\\theta)}\\left[Cos(R^2Cos2\\theta) + iSin(R^2Cos2\\theta)\\right]$\r\n\r\nso the absolute value of $ e^{iR^2e^{i2\\theta}}$ is $ |e^{ - R^2Sin(2\\theta)}|$ and that's going to be all kinds of values depending on R and $ \\theta$.\r\n\r\nNow, $ |e^{iw}| = |Cos[w] + iSin[w]|$ which is 1 when $ w$ is real but when w is a complex number it's:\r\n\r\n$ e^{i(x + iy)} = e^{ - y + ix}$ and so:\r\n\r\n$ |e^{i(x + iy)}| = |e^{ - y}|$\r\n\r\nSo now you see when the complex component of w is zero, then the absolute value of $ e^{iw}$ is 1. In your case, \r\n\r\n$ w = R^2e^{i2\\theta} = R^2[Cos(2\\theta) + iSin(2\\theta)]$\r\n\r\nso the complex component is in general not zero so the absolute value of your expression won't in general be 1.", "Solution_3": "[quote=\"jmerry\"]\nProving this integral goes to zero . . . [/quote]\r\n\r\nFirst I'd make the substitution $ \\phi\\equal{}2\\theta$ and obtain:\r\n\r\n$ \\frac{R}{2}\\int_0^{\\pi} Exp[\\minus{}R^2 Sin(\\phi)]d\\phi$\r\n\r\nNow we wish to find an upper bound for this integral. Note the plot below:\r\n\r\n[img]http://img87.imageshack.us/img87/3495/plotaik6.jpg[/img]\r\n \r\n\r\nOn Blue: $ Sin(\\phi)\\geq \\frac{2}{\\pi} \\phi$\r\n\r\nOn Green: $ Sin(\\phi)\\geq \\minus{}\\frac{2}{\\pi} \\phi\\plus{}2$\r\n\r\nThen I can write:\r\n\r\n$ \\left|\\frac{R}{2}\\int_0^{\\pi} Exp[\\minus{}R^2 Sin(\\phi) d\\phi\\right|\r\n\\leq \\frac{R}{2}\\left(\\left|\\int_0^{\\pi/2} Exp[\\minus{}R^2\\frac{2\\phi}{\\pi}]d\\phi\\right|\\plus{}\r\n\\left|\\int_{\\pi/2}^{\\pi}Exp[\\minus{}R^2(\\minus{}\\frac{2}{\\pi}\\phi\\plus{}2)]d\\phi\\right|\\right)$\r\n\r\nevaluating the integrals I obtain:\r\n\r\n$ \\left|\\int_0^{\\pi/2}R Exp[\\minus{}R^2 Sin(2\\theta)]d\\theta\\right|\\leq \\frac{R}{2}\\left[\\frac{\\pi}{2R^2}(1\\minus{}e^{\\minus{}R^2})\\plus{}\\frac{\\pi}{2R^2}(1\\minus{}e^{\\minus{}R^2})\\right]$\r\n\r\nor:\r\n\r\n$ \\left|\\int_0^{\\pi/2}R Exp[\\minus{}R^2 Sin(2\\theta)]d\\theta\\right|\\leq \\frac{\\pi}{R}\\left(1\\minus{}e^{\\minus{}R^2}\\right)$\r\n\r\nwhich goes to zero as $ R\\to\\infty$" } { "Tag": [ "Putnam", "Harvard", "college", "college contests" ], "Problem": "As I know, of the 5 fellows, there'll be one receiving the putnam scholarship to Harvard, but I can't find past scholars. Anyone knows? and how would they decide who'll receive the scholarship?", "Solution_1": "(At least long ago, when I was Putnam scholar)\r\nThe advisors/professors of the 5 Putnam Fellows write recommendations, and one is selected based on that.\r\n\r\nI think admission to graduate study at Harvard is in practice never denied to that winner. Even before he completes his bachelor's degree: when I was at Harvard, another Putnam scholar began there immediately without completing his bachelor's degree.\r\n\r\nIn fact the fellowship (one year tuition plus generous allowance for living and expenses at Harvard) need not be used for graduate study in mathematics. It has sometimes been used for undergraduate study (by a student already at Harvard), and it has at least once been used for Harvard Medical School.\r\n\r\nThe volumes published by the MAA on the Putnam have listings of the winners at the back, and the holders of the Putnam Fellowship are noted in the first volume (1938--1964) but not in the later ones.", "Solution_2": "[quote=\"transseries\"](At least long ago, when I was Putnam scholar)\nI think admission to graduate study at Harvard is in practice never denied to that winner. Even before he completes his bachelor's degree: when I was at Harvard, another Putnam scholar began there immediately without completing his bachelor's degree.[/quote]\r\n\r\nWhat?! You can do that? Can you go to grad school in math without completing a bachelor's degree first?", "Solution_3": "Yes, it occasionally happens, but for obvious reasons it's quite rare.", "Solution_4": "[quote=\"sexynsmartjenny\"]What?! You can do that? Can you go to grad school in math without completing a bachelor's degree first?[/quote]\r\n\r\nYou can if the graduate school admits you.", "Solution_5": "[quote=\"transseries\"](At least long ago, when I was Putnam scholar)\nThe advisors/professors of the 5 Putnam Fellows write recommendations, and one is selected based on that.\n\nI think admission to graduate study at Harvard is in practice never denied to that winner. Even before he completes his bachelor's degree: when I was at Harvard, another Putnam scholar began there immediately without completing his bachelor's degree.\n\nIn fact the fellowship (one year tuition plus generous allowance for living and expenses at Harvard) need not be used for graduate study in mathematics. It has sometimes been used for undergraduate study (by a student already at Harvard), and it has at least once been used for Harvard Medical School.\n\nThe volumes published by the MAA on the Putnam have listings of the winners at the back, and the holders of the Putnam Fellowship are noted in the first volume (1938--1964) but not in the later ones.[/quote]\r\n\r\nThank you for the info! I ask about this since I am quite confident to become a fellow this year or next year, and going to harvard is my dream. getting the scholarship helps my admission to harvard much easier.", "Solution_6": "[quote=\"ohhotgirl\"][quote=\"transseries\"](At least long ago, when I was Putnam scholar)\nThe advisors/professors of the 5 Putnam Fellows write recommendations, and one is selected based on that.\n\nI think admission to graduate study at Harvard is in practice never denied to that winner. Even before he completes his bachelor's degree: when I was at Harvard, another Putnam scholar began there immediately without completing his bachelor's degree.\n\nIn fact the fellowship (one year tuition plus generous allowance for living and expenses at Harvard) need not be used for graduate study in mathematics. It has sometimes been used for undergraduate study (by a student already at Harvard), and it has at least once been used for Harvard Medical School.\n\nThe volumes published by the MAA on the Putnam have listings of the winners at the back, and the holders of the Putnam Fellowship are noted in the first volume (1938--1964) but not in the later ones.[/quote]\n\nThank you for the info! I ask about this since I am quite confident to become a fellow this year or next year, and going to harvard is my dream. getting the scholarship helps my admission to harvard much easier.[/quote]\r\n\r\n\r\n\r\nwow, what is your name?\r\n\r\nIf you have the ability to become a fellow, I don't think there is any difficulty for you to get into Harvard.\r\n\r\nPutnam fellows never need to worry about grad admissions", "Solution_7": "[quote=\"EulerFish\"]Putnam fellows never need to worry about grad admissions[/quote] What basis do you have for making this statement? (My guess: none whatsoever.)", "Solution_8": "[quote=\"JBL\"][quote=\"EulerFish\"]Putnam fellows never need to worry about grad admissions[/quote] What basis do you have for making this statement? (My guess: none whatsoever.)[/quote]\r\n\r\nI don't mean that the title \"putnam fellow\" can guarantee admissions. \r\n\r\nWhat I tried to say was that if someone has the ability to be a putnam fellow, he should have made a background strong enough to earn admissions from top grad schools without much difficulty.", "Solution_9": "No, it doesn't work that way. The Putnam is a very strong positive factor, but the top schools will care if you screw up otherwise- and people do. Like me.", "Solution_10": "[quote=\"jmerry\"]No, it doesn't work that way. The Putnam is a very strong positive factor, but the top schools will care if you screw up otherwise- and people do. Like me.[/quote]\r\nHow did you \"screw up\"?", "Solution_11": "[quote=\"ohhotgirl\"]\n\nThank you for the info! I ask about this since I am quite confident to become a fellow this year or next year, and going to harvard is my dream. getting the scholarship helps my admission to harvard much easier.[/quote] \r\n\r\n\r\nDid you take putnam last Saturday?\r\n\r\nIt is amazing to heard someone saying \"confident to become a fellow\"." } { "Tag": [ "number theory unsolved", "number theory" ], "Problem": "$m, m+1 , ... , m+n$ are consecutive natural numbers and sum of numbers on digits $m$ and $m+n$ are divisible 8, but sum of numbers on digits other natural numbers are not divisible 8. Find maximum value of $n$.", "Solution_1": "$n\\le 15$\r\nLet $d(n)$ be the sum of the digits of n. \r\nNote that if the number last digit of $k$ isn't $9$ we have that \r\n$d(k)+1=d(k+1)(\\mod 8)$.\r\nIf the last $t$ digits of $k$ are $9$ we have that:\r\n$d(k)+1-t\\equiv d(k+1) \\left(\\mod 8\\right)$ because $9=1 (\\mod 8)$\r\nIf the last digit of $m$ is $0$ or $1$ we have that\r\n$d(m)= d(m+8) (\\mod 8)$\r\nIf the last digit of $m$ is $a\\not=0,1$ then $n \\le 10-a+7 \\le 10-2+7 = 15$ Because the first digit $n+k$ that ends at $0$ is at least $1$ modulo $8$ and one number between $d(n+k), d(n+k+1)...,d(n+k+7)$ is $0$ modulo $8$.\r\nNow we note that if $m=9999992$ the secuence $d(m),d(m+1),...,d(m+15)$ modulo 8 is:\r\n$0,1,2,3,4,5,6,7,1,2,3,4,5,6,7,0$ and we are done." } { "Tag": [ "function", "algebra proposed", "algebra" ], "Problem": "Find all continuous functions $f : \\mathbb{R}\\to\\mathbb{R}$ such that for all $x \\in\\mathbb{ R}$,\r\n\\[f (f (x)) = f (x)+x.\\]", "Solution_1": "[url=http://www.mathlinks.ro/Forum/viewtopic.php?t=128763]Done[/url] ($a=b=1$ satisfy $a\\not =0,b>0,a+b\\not =1$)", "Solution_2": "Reviewing old problems, any solutions here???", "Solution_3": "[quote=N.T.TUAN]Find all continuous functions $f : \\mathbb{R}\\to\\mathbb{R}$ such that for all $x \\in\\mathbb{ R}$,\n\\[f (f (x)) = f (x)+x.\\][/quote]\n\nSee http://artofproblemsolving.com/community/c6h1344617p7316804 subcase 7.1.1\n", "Solution_4": "Thank you!! " } { "Tag": [ "inequalities", "inequalities proposed" ], "Problem": "Let $ a, b, c$ be positive real numbers. Prove that: $ \\frac{a^2}{2a^2\\plus{}ab\\plus{}c^2}\\plus{}\\frac{b^2}{2b^2\\plus{}bc\\plus{}a^2}\\plus{}\\frac{c^2}{2c^2\\plus{}ca\\plus{}b^2} \\le \\frac{3}{4}$ \r\n :maybe:", "Solution_1": "[quote=\"nguoivn\"]Let $ a, b, c$ be positive real numbers. Prove that: $ \\frac {a^2}{2a^2 \\plus{} ab \\plus{} c^2} \\plus{} \\frac {b^2}{2b^2 \\plus{} bc \\plus{} a^2} \\plus{} \\frac {c^2}{2c^2 \\plus{} ca \\plus{} b^2} \\le \\frac {3}{4}$ \n :maybe:[/quote]\r\n\r\n\r\nassume $ a \\equal{} max{(a,b,c)}$\r\n\r\nThe following inequality is equivalent to \r\n\r\n$ (2b^2 \\plus{} 7bc \\plus{} 2ca)(a \\minus{} b)^2(a \\minus{} c)^2 \\plus{} ((12acb^2 \\plus{} 10ac^2b\\minus{} 22b^2c^2)$$ \\plus{} ac^3 \\plus{} b^4 \\plus{} 8b^3c \\plus{} 7ab^3 \\plus{} 4bc^3 \\plus{} 3c^4)(a \\minus{} b)(a \\minus{} c)$\r\n\r\n$ \\plus{} ((3a \\minus{} c)b^3 \\plus{} (15ca \\minus{} 10c^2)b^2 \\plus{} (15ac^2 \\minus{} c^3)b \\plus{} 3ac^3)(a \\minus{} b)^2\\geq 0$\r\n\r\nobvious true." } { "Tag": [], "Problem": "What is the units digit of the prime number $ 2^{44,497}\\minus{}1$?", "Solution_1": "The units digit of powers of 2 repeats as follows:\r\n\r\n2,4,8,6,2,4,...\r\n\r\n44497=1 mod 4, so the answer is 2-1=1." } { "Tag": [ "geometry", "geometric transformation", "rotation", "ratio", "symmetry", "reflection", "similar triangles" ], "Problem": "This is Tournament of the Towns 2004 problem (8 points)\r\nBut I think it is easier than that one with geometrical locus.\r\n\r\nAngle AOB and COD are combined with a rotation such that\r\nray OA is combined with a ray OC, OB with OD.\r\nIn them are inscribed two circles which intersect in points E and F.\r\nProve that angle AOE is equal to the angle DOF.", "Solution_1": "Just wondering, had all the countries finished Tournament of Towns yet? I'm not sure how this contest is conducted internationally.", "Solution_2": "This is a sketch. Some details are left out.\r\n\r\nI'll assume WLOG (because I don't want to name other points) that $A,B$ are the points of tangency between $OA,OB$ and the circle inscribed in $\\angle AOB$, and the same for $C,D$. In this case we replace the word \"rotation\" in the text with \"spiral similarity\". I'm sure you won't mind :).\r\n\r\nLet $\\mathcal C_1$ be the circle inscribed in $\\angle AOB$ and $\\mathcal C_2$ the other one, and let $r_1,r_2$ be their respective radii, and $p_1,p_2$ the powers of $O$ wrt these two circles (respectively). Also, let $P$ be the second pt of intersection between $OE$ and $\\mathcal C_2$, while $Q$ is the second pt of intersection between $OF$ and $\\mathcal C_1$. Assume for now that we have shown $\\frac{OE}{OQ}=\\frac{OP}{OF}=\\frac{r_1}{r_2}\\ (*)$. In this case we find the triangles $QCF,EBP$ to be similar, and we\u2019re pretty much done.\r\n\r\nIn order to prove $(*)$, let $X$ be the intersection between the tangent at $E$ to $\\mathcal C_1$ and the tangent at $F$ to $\\mathcal C_2$. Let $Y$ be the second intersection between $XF$ and $\\mathcal C_1$, and $Z$ the second intersection between $XE$ and $\\mathcal C_2$. Try to show that $(1)\\ \\frac{XE}{XF}=\\frac{r_1}{r_2}$ and $(2)\\ XEF$ is similar to both $OEQ$ and $OPF$.", "Solution_3": "Another solution:\r\n\r\nAgain assume that $A, B, C, D$ are points of the relavant tangencies. Let $O_1, O_2$ be the centres of the two circles respectively ($O_1$ for the circle in $\\angle AOB$).\r\n\r\nSince $\\angle AOB= \\angle COD$, we have $\\angle AOO_1 = DOO_2$. Similar triangles give $\\frac{O_1O}{O_2O} = \\frac{r_1}{r_2}$ (the $r$'s are the radii of the circles)\r\n\r\nLet the internal angle bisector of $\\angle O_1OO_2$ touch $O_1O_2$ at $X$. Then all four ratios\r\n\r\n\\[\r\n\\frac{O_1O}{O_2O}, \\frac{O_1E}{O_2E}, \\frac{O_1X}{O_2X}, \\frac{O_1F}{O_2F}\r\n\\]\r\nare equal to $r_1/r_2$. By the Apollonius circle, the points $O, E, X, F$ are concyclic (or collinear in the trivial case $r_1=r_2$.) By symmetry, $XE=XF$, so $\\angle XOE = \\angle XOF$. After knowing this, the rest is just trivial angle calculations about the point $O$.", "Solution_4": "Wow, billzhao you have a very pretty solution, nobody from our country\r\ndidn't solve it in a such way! Very good idea with Apollonius circle.\r\n\r\n Actually all who solved it used the same idea that Grobber used,\r\nFor example I just explained solution using inversion and symmetry \r\n- similar to Grobber spiral similarity.", "Solution_5": "This might come as a shock to you, but I for one was very much thinking about an Apollonius circle :).", "Solution_6": "I also solved it wth apollonius circle during the test! gteat billzhao!", "Solution_7": "Here's my solution, in the test. I found it quite easy for an 8 points problem. \r\n Let the circles C1 and C2 be tangent at A,B,C and D to the lines. Then consider the inversion with center O and ratio OA.OD . Clearly it takes the circle C1 to a circle equal to C2 but reflected across the angle bisector of AOD, and the same with C2 to C2' equal to C1 but reflected. Suposse E maps to E' . It is obvious now to reflect the inverted figure across the angle bisector of AOD. C2' maps to C2'', C1' maps to C1'' and E' to E'' . Compare the obtained figure with the original one. It is obvious that C1'' = C2 , C2'' = C1, therefore E'' = F, and we know that the line OE'' (the same as OF) is the reflection of the line OE' (the same as OE), so OF is the reflection across the angle bisector of AOD of the line OE, and the conclussion follows.\r\nhope u could understand what i did.." } { "Tag": [ "geometry", "3D geometry" ], "Problem": "Hi,\r\nI have a deadLine untill Thursday to find out the solutions to the following Questions.\r\nCan Anyone Help me??? please??\r\n\r\n1) A cylinder with Relative Density=0.5 and height=10m is holding vertical with the bottom base touching the surface of water.\r\nIf we leave the cylinder to move vertical towards water, Find the maximum depth in which the upper base of cylinder will reach.\r\n\r\n2) Flow of Water in an experimental tube with diametre 100mm has pressure fall 10.000 Pascal with n(a coefficient)=0.01. Find the length of the tube when Turbulent appears with Re=1000000\r\n\r\n3) Water is flowing in a Venturi Meter of 400mm X 100mm with discharge 72000lit/h and pipe difference 250mm.\r\nIf the 'narrowing' is 200mm X 50 mm, \r\nfind the coefficient z of the meter.\r\n\r\n4) An Open reservoir fill with water has at it's side walls and in 2meters depth from the surface, Orifice(hole) with diametre=25mm.\r\na) Find the Velocity of Water at the Exit of the Orifice\r\nb) At the Orifice we Adjust a Pipe with diametr=25mm and length=9meters. Find the velocity at the exit of the Pipe.\r\nc) How much % has the Velocity increase or Decrease??\r\nGenerally, take the water flow normal.\r\n\r\n5) A cube with edge of 10 cm and Relative Density p=1.2 is equalising in 2 non-mixable fluids of relative density a=0.6 and b=1.6. \r\nFind the parts of the Cube's Edge that are Immersed for each case.\r\n\r\n\r\nPLEEEEEAAAASSSSEEEEEEE!!!!!", "Solution_1": "YO new member.\r\n\r\nFOr the first question,i'm not really getting the wordings.Could u be more clear?\r\n\r\nI think the fourth one has something to do with Toricelli's equation.Directly substitute the values i guess.", "Solution_2": "[quote=\"shadysaysurspammed\"]YO new member.\r\n\r\nFOr the first question,i'm not really getting the wordings.Could u be more clear?\r\nquote]\r\n\r\nWell, Excuse my English,\r\nWhat The Problem means is that We have a Cylinder that we are holding above the surface of water (the down part of cylinder touches the water) and the question is :\r\nIf we leave it to fall towards the water , the upper part of cylinder in what depth will reach (I suppose before changing it's vertical direction to some angle). \r\nSo, We have it's 'Relative Density' =0.5 and the cylinder height=10meters.\r\n\r\nFor All I know, the 'Relative Density' =( Weight of Object / weight of moved water )\r\n\r\nI don't know if this is the right Solution but I think something like :\r\n\r\nWeight of Cylinder = Weight of Moved Water ==>\r\n9,81(gravity) X 0.5 (Relative Density) X 1000 = 9810 X(10m X h) [where h=what we are looking for].\r\n\r\nso solving this , should be something like : h= 0.05 m\r\n\r\nBut I guess this is wrong cause the Result (h=0.05m) seems too small as a number. If I could guess I'd say h=50meters but who knows!\r\n\r\nThat's why I want your help.!!\r\n\r\nPlease, I've LOST all my notes and books in a fire!!! [and I need answers till Thursday]\r\n\r\nThanks again.", "Solution_3": "[quote=\"shadysaysurspammed\"] What starts with \"f\" and ends in \"uck\"? \n\nanswer-Firetruck!!(heheh,i know what u were thinkin:)) [/quote]Really? I thought it was \"Friar Tuck,\" but then that must be because I just watched \"Robin Hood (Prince of Thieves)\" the other night on TV :)" } { "Tag": [ "geometry", "inradius", "inequalities", "circumcircle", "trigonometry", "function", "incenter" ], "Problem": "In any triangle ABC proove [tex] \\frac{m_a}{a}+\\frac{m_b}{b}+\\frac{m_c}{c}\\geq\\frac{p}{2r} [/tex]\r\n\r\n[i]Edited by Myth[/i]", "Solution_1": "What is $s$?", "Solution_2": "Semiperimeter , I suppose... :?", "Solution_3": "Notation \"p\" is generally accepted for semiperimeter. Am I right?", "Solution_4": "yes , s is semiperimeter", "Solution_5": "I'm more accustom to having $s$ as the semiperimeter and $p$ as the perimeter.\r\n\r\nBut that goes along with Darij's [url=http://www.mathlinks.ro/Forum/viewtopic.php?p=138794#138794]comment[/url].", "Solution_6": "[quote=\"nickolas\"]In any triangle ABC proove [tex] \\frac{m_a}{a}+\\frac{m_b}{b}+\\frac{m_c}{c}\\geq\\frac{p}{2r} [/tex]\n\n[i]Edited by Myth[/i][/quote]\r\n\r\nMyth, you hardly made the problem easier to understand. I have always been used to denoting the semiperimeter by s (if my memory doesn't cheat me, this notation goes back to Euler). Could you better just write explicitely what is meant by $m_a$, ..., p, r, as I proposed in another thread? Thanks.\r\n\r\n Darij", "Solution_7": "I hear that $s$ is a semiprimetr for the first time. In all russian books I saw notation $p$ is used (see geometr's bible Prasolov's \"Planimetry\").", "Solution_8": "[quote=\"nickolas\"]In any triangle ABC proove [tex] \\frac{m_a}{a}+\\frac{m_b}{b}+\\frac{m_c}{c}\\geq\\frac{p}{2r} [/tex][/quote]\r\n\r\nAt first, the $\\geq$ sign should actually be an $\\leq$ sign here, and also, I rewrite the problem using the notations I prefer:\r\n\r\n[color=blue]In any triangle ABC with semiperimeter $s=\\frac{a+b+c}{2}$, inradius r and medians $m_a$, $m_b$, $m_c$, we have $\\frac{m_a}{a}+\\frac{m_b}{b}+\\frac{m_c}{c}\\leq\\frac{s}{2r}$.[/color]\r\n\r\n[i]Proof.[/i]\r\n\r\n[color=red][b]EDIT:[/b] The proof below is WRONG. Thanks to Mak for pointing this out. In fact, the \"well-known inequalities\" $m_a\\leq\\sqrt{s\\left(s-a\\right)}$, $m_b\\leq\\sqrt{s\\left(s-b\\right)}$, $m_c\\leq\\sqrt{s\\left(s-c\\right)}$ which I used are incorrect; actually, one has $m_a\\geq\\sqrt{s\\left(s-a\\right)}$, $m_b\\geq\\sqrt{s\\left(s-b\\right)}$, $m_c\\geq\\sqrt{s\\left(s-c\\right)}$.\n\nSorry to everybody for the wrong proof. I will try to find a correct one.[/color]\r\n\r\nUsing the well-known inequalities $m_a\\leq\\sqrt{s\\left(s-a\\right)}$, $m_b\\leq\\sqrt{s\\left(s-b\\right)}$, $m_c\\leq\\sqrt{s\\left(s-c\\right)}$, we find\r\n\r\n$\\frac{m_a}{a}+\\frac{m_b}{b}+\\frac{m_c}{c}\\leq\\frac{\\sqrt{s\\left(s-a\\right)}}{a}+\\frac{\\sqrt{s\\left(s-b\\right)}}{b}+\\frac{\\sqrt{s\\left(s-c\\right)}}{c}$.\r\n\r\nHence, in order to prove the inequality\r\n\r\n$\\frac{m_a}{a}+\\frac{m_b}{b}+\\frac{m_c}{c}\\leq\\frac{s}{2r}$,\r\n\r\nit will be enough to establish the sharper inequality\r\n\r\n$\\frac{\\sqrt{s\\left(s-a\\right)}}{a}+\\frac{\\sqrt{s\\left(s-b\\right)}}{b}+\\frac{\\sqrt{s\\left(s-c\\right)}}{c}\\leq\\frac{s}{2r}$.\r\n\r\nUpon division by $\\sqrt{s}$, this inequality becomes\r\n\r\n$\\frac{\\sqrt{s-a}}{a}+\\frac{\\sqrt{s-b}}{b}+\\frac{\\sqrt{s-c}}{c}\\leq\\frac{s}{2r\\sqrt{s}}$.\r\n\r\nNow, let's transform the right hand side of this inequality: On the one hand,\r\n\r\n(s - a) + (s - b) + (s - c) = 3s - (a + b + c) = 3s - 2s = s.\r\n\r\nOn the other hand, by a well-known formula for the inradius of a triangle,\r\n\r\n$r=\\sqrt{\\frac{\\left(s-a\\right)\\left(s-b\\right)\\left(s-c\\right)}{s}}$,\r\n\r\nso that\r\n\r\n$r\\sqrt{s}=\\sqrt{\\frac{\\left(s-a\\right)\\left(s-b\\right)\\left(s-c\\right)}{s}}\\sqrt{s}=\\sqrt{\\left(s-a\\right)\\left(s-b\\right)\\left(s-c\\right)}$.\r\n\r\nHence, the fraction on the right hand side of our inequality can be transformed as follows:\r\n\r\n$\\frac{s}{2r\\sqrt{s}}=\\frac{\\left(s-a\\right)+\\left(s-b\\right)+\\left(s-c\\right)}{2\\sqrt{\\left(s-a\\right)\\left(s-b\\right)\\left(s-c\\right)}}$\r\n$=\\frac{\\left(s-a\\right)}{2\\sqrt{\\left(s-a\\right)\\left(s-b\\right)\\left(s-c\\right)}}+\\frac{\\left(s-b\\right)}{2\\sqrt{\\left(s-a\\right)\\left(s-b\\right)\\left(s-c\\right)}}+\\frac{\\left(s-c\\right)}{2\\sqrt{\\left(s-a\\right)\\left(s-b\\right)\\left(s-c\\right)}}$\r\n$=\\frac{\\sqrt{s-a}}{2\\sqrt{\\left(s-b\\right)\\left(s-c\\right)}}+\\frac{\\sqrt{s-b}}{2\\sqrt{\\left(s-c\\right)\\left(s-a\\right)}}+\\frac{\\sqrt{s-c}}{2\\sqrt{\\left(s-a\\right)\\left(s-b\\right)}}$.\r\n\r\nHence, the inequality that we must prove rewrites as\r\n\r\n$\\frac{\\sqrt{s-a}}{a}+\\frac{\\sqrt{s-b}}{b}+\\frac{\\sqrt{s-c}}{c}$\r\n$\\leq\\frac{\\sqrt{s-a}}{2\\sqrt{\\left(s-b\\right)\\left(s-c\\right)}}+\\frac{\\sqrt{s-b}}{2\\sqrt{\\left(s-c\\right)\\left(s-a\\right)}}+\\frac{\\sqrt{s-c}}{2\\sqrt{\\left(s-a\\right)\\left(s-b\\right)}}$.\r\n\r\nBut this follows from the following three inequalities:\r\n\r\n$\\frac{\\sqrt{s-a}}{a}\\leq\\frac{\\sqrt{s-a}}{2\\sqrt{\\left(s-b\\right)\\left(s-c\\right)}}$;\r\n$\\frac{\\sqrt{s-b}}{b}\\leq\\frac{\\sqrt{s-b}}{2\\sqrt{\\left(s-c\\right)\\left(s-a\\right)}}$;\r\n$\\frac{\\sqrt{s-c}}{c}\\leq\\frac{\\sqrt{s-c}}{2\\sqrt{\\left(s-a\\right)\\left(s-b\\right)}}$.\r\n\r\nSo, to complete the proof, it remains to verify these three inequalities. We will only verify the first inequality,\r\n\r\n$\\frac{\\sqrt{s-a}}{a}\\leq\\frac{\\sqrt{s-a}}{2\\sqrt{\\left(s-b\\right)\\left(s-c\\right)}}$;\r\n\r\nthe other two are just its cyclic permutations.\r\n\r\nThe proof of the inequality $\\frac{\\sqrt{s-a}}{a}\\leq\\frac{\\sqrt{s-a}}{2\\sqrt{\\left(s-b\\right)\\left(s-c\\right)}}$ is very easy: First, we divide it by $\\sqrt{s-a}$ and obtain\r\n\r\n$\\frac{1}{a}\\leq\\frac{1}{2\\sqrt{\\left(s-b\\right)\\left(s-c\\right)}}$;\r\n\r\nthis is obiviously equivalent to\r\n\r\n$a\\geq 2\\sqrt{\\left(s-b\\right)\\left(s-c\\right)}$.\r\n\r\nNow we square this:\r\n\r\n$a^2\\geq 4\\left(s-b\\right)\\left(s-c\\right)$.\r\n\r\nBut\r\n\r\n$4\\left(s-b\\right)\\left(s-c\\right)=\\left(2\\left(s-b\\right)\\right)\\cdot\\left(2\\left(s-c\\right)\\right)=\\left(c+a-b\\right)\\cdot\\left(a+b-c\\right)$\r\n$=\\left(a-\\left(b-c\\right)\\right)\\cdot\\left(a+\\left(b-c\\right)\\right)=a^2-\\left(b-c\\right)^2$,\r\n\r\nso that $a^2\\geq 4\\left(s-b\\right)\\left(s-c\\right)$ rewrites as $a^2\\geq a^2-\\left(b-c\\right)^2$, what is trivial since $\\left(b-c\\right)^2\\geq 0$.\r\n\r\nProof complete.\r\n\r\nNote that in the initial inequality $\\frac{m_a}{a}+\\frac{m_b}{b}+\\frac{m_c}{c}\\leq\\frac{s}{2r}$, equality holds if and only if the triangle ABC is equilateral. This is easy to see from the above proof.\r\n\r\n Darij", "Solution_9": "Thank you very much Darij!", "Solution_10": "Hey, but the proof is wrong!!\r\n\r\n darij", "Solution_11": "In fact , this is a prove for the ineq\r\n\r\n[tex] \\frac{w_a}{a}+\\frac{w_b}{b}+\\frac{w_c}{c}\\leq \\frac{s}{2r} [/tex]\r\nwhere [tex] w_a , w_b , w_c [/tex] are angle bisectors and [tex] s [/tex] is semiperimeter", "Solution_12": "[quote=\"darij grinberg\"]Hey, but the proof is wrong!![/quote]\r\nAt least, it is a good edification for long non-conceptual proofs. :?", "Solution_13": "One man asked me some days ago to prove $\\frac{m_a}{a}+\\frac{m_b}{b}+\\frac{m_c}{c}\\geq\\frac{3\\sqrt 3}{2}$.\r\n\r\nI proved it. And now I can't find the topic, which contain this inequality, though I remember we discussed it. I want to know what solution was proposed here.\r\nDoes anybody know?", "Solution_14": "[quote=\"Myth\"]One man asked me some days ago to prove $\\frac{m_a}{a}+\\frac{m_b}{b}+\\frac{m_c}{c}\\geq\\frac{3\\sqrt 3}{2}$.\n\nI proved it. And now I can't find the topic, which contain this inequality, though I remember we discussed it. I want to know what solution was proposed here.[/quote]\r\n\r\nHmm, I don't remember whether there was a separate topic for this inequality, but it is a special case of [url=http://www.mathlinks.ro/Forum/viewtopic.php?t=21016]the general geometric inequality[/url] stating that $\\frac{MA}{a}+\\frac{MB}{b}+\\frac{MC}{c}\\geq\\sqrt3$ for every point M in the plane of the triangle ABC (in your case, one has just to take M = centroid of triangle ABC).\r\n\r\nBy the way, are there any advances on the initial inequality $\\frac{m_a}{a}+\\frac{m_b}{b}+\\frac{m_c}{c}\\leq\\frac{s}{2r}$? I have showed it for all acute-angled triangles ABC, but I can't prove it for obtuse-angled ones...\r\n\r\n darij", "Solution_15": "[quote=\"darij grinberg\"][color=blue]In any triangle ABC with semiperimeter $s=\\frac{a+b+c}{2}$, inradius r and medians $m_a$, $m_b$, $m_c$, we have $\\frac{m_a}{a}+\\frac{m_b}{b}+\\frac{m_c}{c}\\leq\\frac{s}{2r}$.[/color][/quote]\r\n\r\nAfter giving a wrong proof of this inequality in post #9, I have now succeeded to find a correct one, although I find it quite ugly. Anyway, here it goes:\r\n\r\nFirst let's consider the case when the triangle ABC is acute-angled or right-angled. Let D be the midpoint of its side BC. Then, the segment AD is the A-median of triangle ABC and thus has the length $AD=m_a$. On the other hand, if O is the circumcenter of triangle ABC, then $OD\\perp BC$ (since the point O, being the circumcenter of triangle ABC, must lie on the perpendicular bisector of its side BC). Thus, in the right-angled triangle ODC, we have $OD=OC\\cdot\\cos\\measuredangle DOC$. But since the point O is the circumcenter of triangle ABC, we have < BOC = 2 < BAC = 2A by the central angle theorem, and also we have OA = OB = OC = R, where R is the circumradius of triangle ABC. Thus, the triangle BOC is isosceles with OB = OC, and hence the line OD, being the altitude from its apex O (since $OD\\perp BC$), must bisect its angle < BOC. Hence, $\\measuredangle DOC=\\frac{\\measuredangle BOC}{2}=\\frac{2A}{2}=A$. Thus, $OD=OC\\cdot\\cos\\measuredangle DOC=R\\cdot\\cos A$.\r\n\r\nNow, by the triangle inequality in the (possibly degenerate) triangle AOD, we have\r\n\r\n$AD\\leq OA+OD=R+R\\cdot\\cos A=R\\cdot\\left(1+\\cos A\\right)=R\\cdot 2\\cos^2\\frac{A}{2}=2R\\cos^2\\frac{A}{2}$.\r\n\r\nBy the extended law of sines, $2R=\\frac{a}{\\sin A}$; thus,\r\n\r\n$AD\\leq 2R\\cos^2\\frac{A}{2}=\\frac{a}{\\sin A}\\cdot \\cos^2\\frac{A}{2}=a\\frac{\\cos^2\\frac{A}{2}}{\\sin A}$\r\n$=a\\frac{\\cos^2\\frac{A}{2}}{2\\sin\\frac{A}{2}\\cos\\frac{A}{2}}=\\frac{a}{2}\\cdot\\frac{\\cos\\frac{A}{2}}{\\sin\\frac{A}{2}}=\\frac{a}{2}\\cdot\\cot\\frac{A}{2}$.\r\n\r\nSince $AD=m_a$, this becomes $m_a\\leq\\frac{a}{2}\\cdot\\cot\\frac{A}{2}$. Dividing this by a, we get $\\frac{m_a}{a}\\leq\\frac12\\cdot\\cot\\frac{A}{2}$. By the half-angle laws, $\\tan\\frac{A}{2}=\\frac{r}{s-a}$, so that\r\n\r\n$\\frac{m_a}{a}\\leq\\frac12\\cdot\\cot\\frac{A}{2}=\\frac12:\\tan\\frac{A}{2}=\\frac12:\\frac{r}{s-a}=\\frac{s-a}{2r}$.\r\n\r\nSimilarly, $\\frac{m_b}{b}\\leq\\frac{s-b}{2r}$ and $\\frac{m_c}{c}\\leq\\frac{s-c}{2r}$. Thus,\r\n\r\n$\\frac{m_a}{a}+\\frac{m_b}{b}+\\frac{m_c}{c}\\leq\\frac{s-a}{2r}+\\frac{s-b}{2r}+\\frac{s-c}{2r}=\\frac{3s-\\left(a+b+c\\right)}{2r}=\\frac{3s-2s}{2r}=\\frac{s}{2r}$.\r\n\r\nThus, the inequality in question is proved for the case of an acute-angled or right-angled triangle ABC.\r\n\r\nThe case when the triangle ABC is obtuse-angled is much more tricky. In fact, assume WLOG that A is the obtuse angle. Then, we have $OD=-R\\cdot\\cos A$ rather than $OD=R\\cdot\\cos A$, and the inequality $m_a\\leq\\frac{a}{2}\\cdot\\cot\\frac{A}{2}$ doesn't hold. But rather rough estimates can be applied:\r\n\r\nFirst, we have A > 90\u00b0; in other words, $\\measuredangle CAB>90^{\\circ}$. Thus, the point A lies inside the circle with diameter BC. The center of this circle is the midpoint D of the segment BC, and the radius of this circle is $\\frac{BC}{2}=\\frac{a}{2}$. Thus, $AD<\\frac{a}{2}$. Since $AD=m_a$, this becomes $m_a<\\frac{a}{2}$. Hence, $\\frac{m_a}{a}<\\frac12$.\r\n\r\nFor the next estimate, we will use vectors. If E is the midpoint of the segment CA, then $\\overrightarrow{BE}=\\frac{\\overrightarrow{BC}+\\overrightarrow{BA}}{2}$, so that\r\n\r\n$BE=\\left|\\overrightarrow{BE}\\right|=\\left|\\frac{\\overrightarrow{BC}+\\overrightarrow{BA}}{2}\\right|=\\frac12\\cdot\\left|\\overrightarrow{BC}+\\overrightarrow{BA}\\right|\\leq\\frac12\\cdot\\left(\\left|\\overrightarrow{BC}\\right|+\\left|\\overrightarrow{BA}\\right|\\right)$ (by the triangle inequality)\r\n$=\\frac12\\cdot\\left(BC+BA\\right)=\\frac12\\cdot\\left(a+c\\right)$.\r\n\r\nSince the point E is the midpoint of the side CA of triangle ABC, the segment BE is the B-median of this triangle; thus, its length is $BE=m_b$, and hence this inequality becomes $m_b\\leq\\frac12\\cdot\\left(a+c\\right)$. Similarly, $m_c\\leq\\frac12\\cdot\\left(b+a\\right)$.\r\n\r\nThus,\r\n\r\n$\\frac{m_a}{a}+\\frac{m_b}{b}+\\frac{m_c}{c}<\\frac12+\\frac{\\frac12\\cdot\\left(a+c\\right)}{b}+\\frac{\\frac12\\cdot\\left(b+a\\right)}{c}$\r\n$=\\frac12\\cdot\\left(1+\\frac{a+c}{b}+\\frac{b+a}{c}\\right)=\\frac12\\cdot\\left(1+\\left(\\frac{a+b+c}{b}-1\\right)+\\left(\\frac{a+b+c}{c}-1\\right)\\right)$\r\n$=\\frac12\\cdot\\left(\\frac{a+b+c}{b}+\\frac{a+b+c}{c}-1\\right)=\\frac12\\cdot\\left(\\frac{2s}{b}+\\frac{2s}{c}-1\\right)=\\frac{s}{b}+\\frac{s}{c}-\\frac12$.\r\n\r\nLet F be the area of triangle ABC; then, by a formula for the area of a triangle, $F=\\frac12bc\\sin A$. Since the angle A is obtuse, sin A < 1, and thus this becomes $F<\\frac12bc$. On the other hand, by another formula for the area of a triangle, F = rs. Thus, $rs<\\frac12bc$. In other words, 2rs < bc. This yields $\\frac{s}{b}<\\frac{c}{2r}$ and $\\frac{s}{c}<\\frac{b}{2r}$. Thus,\r\n\r\n$\\frac{m_a}{a}+\\frac{m_b}{b}+\\frac{m_c}{c}<\\frac{s}{b}+\\frac{s}{c}-\\frac12<\\frac{c}{2r}+\\frac{b}{2r}-\\frac12=\\frac{c+b}{2r}-\\frac12$\r\n$=\\frac{\\left(a+b+c\\right)-a}{2r}-\\frac12=\\frac{2s-a}{2r}-\\frac12=\\frac{s+\\left(s-a\\right)}{2r}-\\frac12=\\frac{s}{2r}+\\frac{s-a}{2r}-\\frac12$.\r\n\r\nBut since A > 90\u00b0, we have $\\frac{A}{2}>\\frac{90^{\\circ}}{2}=45^{\\circ}$, and thus, since the tangens function is monotonically increasing on the interval [0\u00b0; 90\u00b0[, we conclude that $\\tan\\frac{A}{2}>\\tan 45^{\\circ}=1$. But by the half-angle laws, $\\tan\\frac{A}{2}=\\frac{r}{s-a}$. Hence, $\\frac{r}{s-a}>1$, so that s - a < r. Thus,\r\n\r\n$\\frac{m_a}{a}+\\frac{m_b}{b}+\\frac{m_c}{c}<\\frac{s}{2r}+\\frac{s-a}{2r}-\\frac12<\\frac{s}{2r}+\\frac{r}{2r}-\\frac12=\\frac{s}{2r}+\\frac12-\\frac12=\\frac{s}{2r}$.\r\n\r\nSo the required inequality is proven for the case of an obtuse-angled triangle ABC, too. This completes the proof of the inequality.\r\n\r\nFinally, let's remark that the inequality we just proved,\r\n\r\n$\\frac{m_a}{a}+\\frac{m_b}{b}+\\frac{m_c}{c}\\leq\\frac{s}{2r}$,\r\n\r\ncan be rewritten in the form\r\n\r\n$m_ah_a+m_bh_b+m_ch_c\\leq s^2$,\r\n\r\nwhere $h_a$, $h_b$, $h_c$ are the three altitudes of triangle ABC. In fact, this is an equivalent transformation, since the area F of the triangle ABC satisfies $F=\\frac12ah_a=\\frac12bh_b=\\frac12ch_c=rs$, so that $a=2F:h_a$, $b=2F:h_b$, $c=2F:h_c$ and $r=F:s$, and thus\r\n\r\n$\\frac{m_a}{a}+\\frac{m_b}{b}+\\frac{m_c}{c}\\leq\\frac{s}{2r}$\r\n$\\Longleftrightarrow\\ \\ \\ \\ \\ \\frac{m_a}{2F:h_a}+\\frac{m_b}{2F:h_b}+\\frac{m_c}{2F:h_c}\\leq\\frac{s}{2F:s}$\r\n$\\Longleftrightarrow\\ \\ \\ \\ \\ \\frac{m_ah_a}{2F}+\\frac{m_bh_b}{2F}+\\frac{m_ch_c}{2F}\\leq\\frac{s^2}{2F}$\r\n$\\Longleftrightarrow\\ \\ \\ \\ \\ m_ah_a+m_bh_b+m_ch_c\\leq s^2$.\r\n\r\nIn the form $m_ah_a+m_bh_b+m_ch_c\\leq s^2$, the inequality was posted by Manlio at http://www.mathlinks.ro/Forum/viewtopic.php?t=5208 .\r\n\r\n Darij", "Solution_16": "I read this post this noon and have a solution which is considerably longer than Darij 's. However, as this solution is replete with properties, so I post it here.\r\n In order to prove this, let 's first say a little bit about about a bijection $f$ that maps from the set of all triangles $X$ to itself (the set $X$ doesn't have any two congruent triangles in it ). Let 's map a triangle with sides $a, b, c$ to the triangle with side $\\frac{2}{\\sqrt{3}}m_{a}, \\frac{2}{\\sqrt{3}}m_{b}, \\frac{2}{\\sqrt{3}}m_{c}$. Then of course $f = f^{-1}$. Notice that $f$ preserves the area of triangles.\r\n Now let me introduce some lemmas that are very useful :\r\n [b]Lemma 1[/b] Let $ABC$ be a triangle. Then there exists a triangle $XYZ$ such that $sin^{2}{X}= \\frac{1}{2}sinA(\\sum tg{\\frac{A}{2}}, ...$\r\n [b]Lemma 2 [/b] Let $XYZ$ be a triangle and $x, y, z$ be real numbers. Then $\\sum yz sin^{2}{X}\\leq \\frac{1}{4}(x+y+z)^{2}$\r\n From lemma 1, it follows that \r\n [b]Lemma 3 [/b] Let $XYZ$ be a triangle and $x, y, z$ be real numbers. Then $\\sum yz sin{X}\\leq \\frac{(x+y+z)^{2}}{2.(\\sum \\tg \\frac{A}{2})}$\r\n Let 's stop there and progress our proof for a moment. Call $R'$ the circumradius of the image triangle $XYZ$ of $ABC$ under $f$. Then $\\frac{2}{\\sqrt{3}}m_{a}= 2R' sin{X}, ....$. By applying lemma 3 we obtain \r\n ${m_{a}.bc = \\sqrt{3}R'(\\sum bc.sinX) \\leq \\frac{\\frac{\\sqrt{3}}{2}}R'(a+b+c)^{2}}{\\sum tg{\\frac{X}{2}}}= \\frac{2\\sqrt{3}R'p^{2}}{\\sum tg{\\frac{X}{2}}}$.\r\n We need to show that \r\n $\\frac{2\\sqrt{3}R'p^{2}}{\\sum tg{\\frac{X}{2}}}\\leq abc.\\frac{p}{2r}$\r\n Notice that $abc = 4RS$, and thus it is equivalent to proving $R' \\leq \\frac{\\sum{tg\\frac{X}{2}}}{\\sqrt{3}}.R$\r\n Carrying out the bijection $f$, this inequality is equivalent to \r\n $R \\leq \\frac{\\sum{tg \\frac{A}{2}}}{\\sqrt{3}}.R'$ [color=blue](*) [/color]\r\n We now go on introducing the lemmas\r\n [b]Lemma 4 [/b] $4Rm_{a}\\geq b^{2}+c^{2}$\r\n It is easy to prove if one compares the diameter and the chord formed by the $A$ median of the circumcircle. Therefore it follows that\r\n [b]Lemma 5[/b] $2R(m_{a}+m_{b}+m_{c})\\geq a^{2}+b^{2}+c^{2}$\r\n Lemma 6 is a direct corollary of this inequality if we carry out the bijection $f$\r\n [b]Lemma 6[/b] $\\frac{m_{a}.m_{b}.m_{c}}{m_{a}^{2}+m_{b}^{2}+m_{c}^{2}}\\geq r$, here $r$ is the inradius of $ABC$.\r\n [b]Lemma 7[/b] $a+2+b^{2}+c^{2}= 2(p^{2}-r^{2}-4Rr)$\r\n [b]Lemma 8[/b] $\\sum tg{\\frac{A}{2}}= \\frac{4R+r}{p}$\r\n By replacing $R' = \\frac{2m_{a}.m_{b}.m_{c}}{3\\sqrt{3}S}\\geq \\frac{2(m_{a}^{2}+m_{b}^{2}+m_{c}^{2}). r}{3\\sqrt{3}S}= \\frac{(a^{2}+b^{2}+c^{2}).r}{2\\sqrt{3}S}= \\frac{(p^{2}-r^{2}-4Rr).r}{\\sqrt{3}S}$ and using lemma 4 to the inequality [color=blue](*)[/color], we obtain we need to prove\r\n $p^{2}(R+r) \\geq 16R^{2}r+r^{3}+8Rr^{2}$ [color=blue](**)[/color]\r\n And here is the final lemma (which I don't prove here, as a special case of Leibnitz 's theorem)\r\n [b]Lemma 9 [/b] Let $I, G$ be the incenter and the centroid of $ABC$. Then $IG^{2}= p^{2}+5r^{2}-16Rr \\geq 0$\r\n [color=blue](**)[/color] follows immediately as $R \\geq 2r$.\r\n PS This may be the longest post I ever reply in ML. :lol:" } { "Tag": [ "2nd edition" ], "Problem": "k is a range?", "Solution_1": "hmm I remember saying no questions about the problems :D I belive that the questions are pretty well put :) read the questions carefully and take your time :)", "Solution_2": "suppose I think the question 3 is wrong (the one with BMN isosceles)? could you please double check that question? :D", "Solution_3": "I've checked the problem and I don't think there is anything wrong with it! :) anyway it's good that you asked :D", "Solution_4": "[quote=\"Valentin Vornicu\"]I've checked the problem and I don't think there is anything wrong with it! :) anyway it's good that you asked :D[/quote]\r\n\r\nthen i must be really stupid because I still cannot draw a proper diagram to that question for x /= 1 :( :( :(", "Solution_5": "... I will answer this question next Monday :D - anyway the problem's fine, I can assure you of that." } { "Tag": [ "videos", "Princeton", "college" ], "Problem": "Before I start, let me just say I'm not necessarily bad at grammar, or overly obsessed with it (or the SAT, for that matter). There just always seems to be ONE question I get wrong in the writing section every time, and I get frustrated with 48C, 1W, 0O test after test. So here's one more (and hopefully the last):\r\n\r\nA major cause of stress in school is [u]where seniors must manage not only academic requirements and sports schedules, but also[/u] standardized testing and college applications, during the first semester.\r\n\r\n(A) where seniors must manage not only academic requirements and sports schedules, but also\r\n(B) seniors need to manage not only academic requirements and sports schedules, but also\r\n(C) where seniors must manage not only academic requirements and sports schedules, and also\r\n(D) when seniors must manage both academic requirements and sports schedules, but also\r\n(E) the management by seniors of not only academic requirements and sports schedules, but also\r\n\r\n\r\n[hide=\"My reasoning...\"]\nOkay, so I know the correct format is \"not only... but also\", so I can eliminate (C), which says \"and also\".\nFurthermore, \"a major cause\" cannot be a \"where\" or \"when\", so eliminate (A) and (D). I'm down to (B) and (E).\nNow take out all the fillers and look at the skeleton sentence:\n(B) A cause is seniors need to manage not only requirements but also testing.\n(E) A cause is the management of not only requirements but also testing.\nThe management causes the stress, not the seniors, so the answer must be (E)![/hide]\n\n\n[hide=\"But I was wrong!\"]\nThe answer is (B). Can someone please explain?[/hide]", "Solution_1": "I think you're parsing (E) wrong. Your interpretation is that \"the management causes the stress, not the seniors.\" But if we strip (E) down to a basic copular sentence, we get \"A cause of stress is [the] management\" which is certainly OK.\r\n\r\nMy best guess as to why (E) is wrong is that it's in the passive voice-- except actually it's not, it just looks suspiciously like it.\r\n\r\nI find the lack of an overt complementizer in (B) pretty jarring, but maybe that's just me.", "Solution_2": "[quote=\"Osud\"]I think you're parsing (E) wrong. Your interpretation is that \"the management causes the stress, not the seniors.\" But if we strip (E) down to a basic copular sentence, we get \"A cause of stress is [the] management\" which is certainly OK.[/quote]\n\nI'm not sure what you mean, I never said (E) was [i]not[/i] okay...\n\n[quote=\"Osud\"]My best guess as to why (E) is wrong is that it's in the passive voice-- except actually it's not, it just looks suspiciously like it.[/quote]\r\n\r\nThe book says that (E) is a \"very awkward passive phrase\". But is (B) any less awkward?", "Solution_3": "[quote]The book says that (E) is a \"very awkward passive phrase\".[/quote]\n\nWhat your book says is completely wrong. (E) is emphatically [b]not[/b] a passive sentence. \n\nHere's how the passive works. In converting a sentence from active to passive, a few things happen:\n(1) The active verb is converted to a form of \"to be\" plus a participle;\n(2) The direct object of the active verb is promoted to subject position;\n(3) The subject of the active verb is demoted to an optional \"by\" phrase.\n\nSo, if your active example is \"seniors manage academic requirements and sports schedules,\" then the passive form of the sentence will be \"academic requirements and sports schedules are managed by seniors.\" We've converted the active verb \"manage\" to the passive \"are managed,\" moved \"academic requirements and sports schedules,\" the direct object of the \"manage,\" to subject position, and moved the subject of the active verb \"manage\" to an optional \"by\" phrase, \"by seniors.\"\n\n\"The management by seniors of academic requirements and sports schedules\" is not a passive phrase. This is a noun phrase headed by the noun \"management.\" The fact that this noun phrase has a \"by\" phrase containing what would have been the subject had \"management\" been an active verb does nothing to change the fact that nouns cannot be either active or passive and that the real verb of the sentence, \"is,\" is active, not passive.\n\nAgreed that that construction is very awkward, though.\n\n[quote]I'm not sure what you mean, I never said (E) was not okay... [/quote]\r\nSorry, I wasn't clear about that. You said in your earlier post that you read the sentence to mean that \"The management causes the stress, not the seniors,\" I responded with an alternate meaning, and by \"OK\" I meant that my parse of the sentence was in line with what you thought the sense of the sentence should have been. That make sense?", "Solution_4": "[quote=\"Osud\"]\n[quote]I'm not sure what you mean, I never said (E) was not okay... [/quote]\nSorry, I wasn't clear about that. You said in your earlier post that you read the sentence to mean that \"The management causes the stress, not the seniors,\" I responded with an alternate meaning, and by \"OK\" I meant that my parse of the sentence was in line with what you thought the sense of the sentence should have been. That make sense?[/quote]\r\n\r\nThanks for your explanation, makes sense now!\r\n\r\nAs for the passive stuff, yeah, I see what you mean.. with management as the noun, the phrase is not really passive at all. But do you agree with them that (B) is the correct answer? And if it is, is awkwardness the only thing wrong with (E)? And [i]if it is[/i], isn't (B) awkward too??? :lol:", "Solution_5": "I would actually say that (B) is worse than (E), and here's why. Imagine you have read only this much of the sentence:\r\n\r\n[quote]A major cause of stress in school is[/quote]\n\nWhat do you naturally expect to come next? A noun phrase, of course. So then you hit the next word:\n\n[quote]seniors[/quote]\n\nAnd, since it's a noun, you naturally interpret it as the noun phrase that's the complement of the verb. But wait! There's more!\n\n[quote]need to manage not only academic requirements[/quote]\r\n\r\nYou start interpreting that as part of a noun phrase headed by \"seniors,\" but that's ungrammatical if all of that stuff goes in the noun phrase. So you have to go back and interpret the whole thing starting with \"seniors\" as a complementizer phrase introduced by a silent complementizer. Usually in English silent complementizers are used when omitting an overt complementizer won't create a garden-path sentence (so-called because it leads you down a garden path instead of the 'real' interpretation). \r\n\r\nEven if grammar books have nothing against garden-path sentences, it is very bad form to write a sentence which many people will parse wrong on the first reading. \r\n\r\nIf it were up to me, I would pick (D). The \"both...but also\" isn't great, but at least it is easy to read and it's immediately obvious what the sentence means. \r\n\r\nActually, if it were me, I'd just say \"Managing standardized testing and college applications, on top of academic requirements and sports schedules, is a major cause of stress in school for seniors.\" Or, if you really wanted to put \"a major cause of stress in school\" at the front of the sentence, I would say, \"It is a major cause of stress for seniors to manage standardized testing and college applications on top of academic requirements and sports schedules.\" Just delete the prepositional phrase \"in school,\" it's not really adding anything here. What else would it be, a major source of stress in church? In video gaming? In Sub-Saharan Africa? :lol:", "Solution_6": "I agree. When I first read the question, I really thought none of the choices made good sentences.\r\n\r\nBut thanks a lot for your input, I feel better about the question now that we've thrown it through a grinder. :)", "Solution_7": "Which book was this question from?", "Solution_8": "Cracking the SAT 2009 Edition. The Princeton Review.", "Solution_9": "For the SAT, I wouldn't trust a test-prep book besides the College Board's own Study Guide. You can use the other books if you have run out of other material, but don't be surprised to see errors. In math, I have caught numerous errors by the outside companies.\r\n\r\nDisclosure: I work for a tutoring company.", "Solution_10": "If you add \"that\" to (B), it becomes an obvious right choice. \"That\" can be sometimes omitted but here the omission creates a terribly awkward sentence, much worse than (E), so I suspect it was just a typographical error. (E) changes the meaning of the sentence: the cause is not \"management by seniors\" but the \"necessity for seniors to manage\" if you want that type of construction.", "Solution_11": "Fedja wrote:\r\n[quote]If you add \"that\" to (B), it becomes an obvious right choice. \"That\" can be sometimes omitted but here the omission creates a terribly awkward sentence, much worse than (E), so I suspect it was just a typographical error.[/quote]\n\nThat's what I was trying to get at with the business about silent vs. overt complementizers, but didn't think to give an example with an overt complementizer... thanks, fedja! :blush: \n\nbuzzer11 wrote:\n[quote]I agree. When I first read the question, I really thought none of the choices made good sentences.[/quote]\n\nI think your instincts are spot-on, it's just that the SAT requires rote application of a set of prescriptive rules instead of a more holistic approach to style, grammar, and readability. This is what I meant earlier when I said not to take the stuff too seriously-- if you puzzle over something for a while and all of the answers seem a bit off to you, then stick with your instinct. The SAT is a big deal at the time you take it, but you have a whole life of writing ahead of you. You shouldn't try to permanently override your instincts about writing and language for the sake of one test.\n\n[quote]But thanks a lot for your input, I feel better about the question now that we've thrown it through a grinder. [/quote]\r\n\r\nSure! Glad I could help. (I'd never thought of myself as a grinder before... :gleam:)", "Solution_12": "A, C, and D are automatically eliminated, because you know it shouldn't begin with \"where\" or \"when\".\r\n\r\nThe management is not the cause of stress. The cause of stress is either the things that the seniors have to manage or the fact that they have to manage them.\r\n\r\nHence, the best answer is B.\r\n\r\n\r\nHaving said that, I don't really like Princeton Review. Their practice problems are usually significantly easier than the actual test, and are sometimes weird. Kaplan has the right difficulty, but is riddled with technical errors and typos. Barrons is a bit harder than the real thing. For subject tests in science, Barrons introduces more material than is required for the actual tests, and the questions are not as understanding-of-theory type as the real ones, but perhaps provide the best practice.\r\n\r\nAlso, I don't particularly agree with the above poster. When I took the SAT's, I went off gut feeling for every single writing question, and it worked just fine. In the past, when I tried consciously invoking particular grammar rules, I got bogged up and confused, and ended up not only getting questions wrong, but also not finishing on time.", "Solution_13": "Why can't management be the cause of the stress? When I read the question that was what I thought the question was implying. The seniors were stressed because they have to [i]manage[/i] a lot.\r\n\r\nIn terms of the \"gut feeling\" idea, I usually go with strict grammar rules and not my gut and I do fine most of the time. I guess it depends on the person.", "Solution_14": "I've reduced B and E by eliminating some adjectives and unnecessary prepositional phrases. I have placed in parentheses what is communicated as the cause of stress.\r\n\r\n(B) A cause of stress is ([implicit*: the fact that] seniors need to manage various tasks).\r\n\r\n(E) A cause of stress is (the management) by seniors of various tasks.\r\n\r\nThe management is not a cause of stress -- management is generally a good thing. Causes of stress might be the tasks themselves, the fact that they have to manage these tasks (as in part B), or inability to manage.\r\n\r\nI totally agree that the assumption of (*) should not be necessary. However, in the SAT's, the first thing you must do is see if the sentence means what it wants you to mean, and then see if it is properly expressed -- it is not just about finding a sentence that is gramatically correct! Sometimes they will try to trick you by making a few gramatically correct responses, only one of which has the desired meaning.", "Solution_15": "Oh. When I read (B) I just read \"the cause of stress is --> seniors\" and so I eliminated that one. :(", "Solution_16": "And that [i][b]is[/b][/i] a bad mistake ;)", "Solution_17": "[quote=\"TZF\"](B) A cause of stress is ([implicit*: the fact that] seniors need to manage various tasks). [/quote]\n\n[quote=\"fedja\"]And that is a bad mistake :wink: [/quote]\r\n\r\nThen how do we know to interpret (B) implying an object?" } { "Tag": [ "calculus", "integration", "trigonometry", "algebra", "binomial theorem", "calculus computations" ], "Problem": "I need help on getting something straightened out.\r\n\r\nFor the integral $ \\int_0^\\frac {1}{2}\\cfrac{\\sqrt {x}}{\\sqrt {1 - x}}\\: dx$, if I make the substitution $ \\sqrt {x} = \\sin{y}$, I get this:\r\n\r\n$ x = \\sin^2y$\r\n$ dx = 2\\sin{y}\\cos{y}$\r\n\r\nand for\r\n\r\n$ \\sqrt {0} = 0 = \\sin{y}$ and $ \\sqrt {\\cfrac{1}{2}} = \\cfrac{1}{\\sqrt {2}} = \\sin{y}$ to hold true,\r\n\r\n$ y = 0$ and $ y = \\cfrac{\\pi}{4}$ respectively.\r\n\r\n\r\nso:\r\n\r\n\\begin{align*} & \\int_0^\\frac {1}{2}\\cfrac{\\sqrt {x}}{\\sqrt {1 - x}}\\: dx \\\\\r\n& = \\int_0^\\frac {\\pi}{4}\\cfrac{\\sin{y}}{\\sqrt {1 - \\sin^2y}}\\cdot{}2\\sin{y}\\cos{y}\\: dy \\\\\r\n& = 2\\int_0^\\frac {\\pi}{4}\\sin^2y\\: dy \\end{align*}\r\n\r\n\r\n\r\nwas the above procedure correct? In particular, did I change the bounds of integration in the correct step?\r\n\r\nMy grasp of this is still kinda shaky.. What is this called anyway? Replacing bounds? It'd be great if someone could tell me under what section of a calculus course I could find more information on this. Thanks! :)", "Solution_1": "Seems fine to me :) Usually the corresponding section in Calculus textbooks is called either \"Change or variable\" or \"Finding integrals by substitution\". Just look at the table of contents and see what looks like it.", "Solution_2": "we can't find any mistakes in your calculation, undefined 117. :) \r\n\r\nHow about this one?\r\n\r\nQ. 1 $ \\int_0^1 x\\sqrt {1 \\minus{} x}\\ dx$.\r\n\r\nQ.2 Let $ x \\equal{} \\cos ^3 t,\\ y \\equal{} \\sin ^ 3 t\\ \\left(0\\leq t\\leq \\frac {\\pi}{2}\\right)$.\r\n\r\n$ \\int_0^1 y\\ dx$.", "Solution_3": "Thanks fedja - I found [url=http://en.wikipedia.org/wiki/Integration_by_substitution]this Wikipedia article[/url] :D\r\n\r\n@kunny: hehe I spent 40 minutes filling up two pages trying to solve that first one by substituting $ x = \\sin^2\\theta$ :rotfl:\r\n\r\nhmm idk is there a way to do it that way? I get $ \\int_0^\\frac {\\pi}{2}2\\sin^3\\theta\\cos^3\\theta\\: d\\theta$, which looks a bit like Q. 2\r\n\r\nokay so\r\n\r\n[hide=\"Q. 1\"]\n$ \\int_0^1x\\sqrt {1 - x}\\: dx$\n\n\n$ u = 1 - x$, so $ x = 1 - u$ and $ du = - 1\\: dx$.\n\n\n\\begin{align*} & - \\int_1^0(1 - u)\\sqrt {u}\\: du \\\\\n& = - \\int_1^0(u^\\frac {1}{2} - u^\\frac {3}{2})\\: du \\\\\n& = - \\int_1^0u^\\frac {1}{2}\\: du + \\int_1^0u^\\frac {3}{2}\\: du \\\\\n& = - \\left[\\cfrac{2}{3}u^\\frac {3}{2}\\right]_1^0 + \\left[\\cfrac{2}{5}u^\\frac {5}{2}\\right]_1^0 \\\\\n& = \\boxed{\\cfrac{4}{15}} \\end{align*}\n\nw00t! :lol:\n[/hide]\r\nI'll need to think a little more about Q. 2 :|", "Solution_4": "O.K. :lol: Q1 is right.", "Solution_5": "Well, You could do the q.1 using that substitution ($ x = sin^2\\theta$), but you won't get the integral you've written. You'll get $ \\int_{0}^{\\pi/2}2\\sin^3\\theta\\cos^2\\theta d\\theta$. And now, you just need to transform it into the form suitable for integration. Of course, it is hard to do it (and quite unnecessary, as your first solution is elegant enough), but here's the form:\r\n\r\n$ \\frac {1}{8} (2\\sin \\theta + \\sin 3\\theta - \\sin 5\\theta)$\r\n\r\nwhich gives you the same answer: $ \\frac {4}{15}$.", "Solution_6": "mk I think I'll need a hint for Q. 2 :D\r\n[hide=\"Q. 2\"]\n\\begin{align*} \\int_0^1y\\: dx & = \\int_0^\\frac {\\pi}{2}\\sin^3t\\cdot( - 3\\cos^2t\\sin{t})\\: dt \\\\\n& = - 3\\int_0^\\frac {\\pi}{2}\\sin^4t\\cos^2t\\: dt \\\\\n& = - 3\\int_0^\\frac {\\pi}{2}\\sin^4t(1 - \\sin^2t)\\: dt \\\\\n& = - 3\\left(\\int_0^\\frac {\\pi}{2}\\sin^4t\\: dt - \\int_0^\\frac {\\pi}{2}\\sin^6t\\: dt\\right) \\end{align*}\nerr..\n\\begin{align*} & - 3\\int_0^\\frac {\\pi}{2}(\\sin^2t\\cos{t})^2\\: dt \\\\\n& = - 3\\int_0^\\frac {\\pi}{2}(\\cos{t} - \\cos^3t)^2\\: dt \\end{align*}\nmeh. :|\n[/hide]", "Solution_7": "Well, let's stop here:\r\n\r\n$ \\minus{} 3\\left(\\int_0^\\frac {\\pi}{2}\\sin^4t\\: dt \\minus{} \\int_0^\\frac {\\pi}{2}\\sin^6t\\: dt\\right)$\r\n\r\nThese two can be easily evaluated if you know:\r\n\r\n$ \\int\\sin^n {ax}\\;dx \\equal{} \\minus{}\\frac{\\sin^{n\\minus{}1} ax\\cos ax}{na} \\plus{} \\frac{n\\minus{}1}{n}\\int\\sin^{n\\minus{}2} ax\\;dx$\r\n\r\n\r\nBut you can also use this nice result \r\n\r\n[url]http://www.mathlinks.ro/viewtopic.php?t=214671[/url]", "Solution_8": "whoa.. that stuff looks crazy beastly, but thanks for reminding me about integration by parts.\r\n\r\nhmm..\r\n\r\nFor $ \\int\\sin^3t( - 3\\cos^2t\\sin{t})\\: dt$\r\n\r\n$ u = \\sin^3t$ and $ dv = - 3\\cos^2t\\sin{t}$\r\n$ du = 3\\sin^2t\\cos{t}$ and $ v = \\cos^3t$\r\n\r\nso\r\n\r\n\\begin{align*} \\int\\sin^3t( - 3\\cos^2t\\sin{t})\\: dt & = \\sin^3t\\cos^3t + 3\\int\\cos^4t\\sin^2t\\: dt \\\\\r\n0 & = (\\sin{t}\\cos{t})^3 + 3\\left(\\int\\cos^4t\\sin^2t\\: dt + \\int\\sin^4t\\cos^2t\\: dt\\right) \\\\\r\n0 & = (\\sin{t}\\cos{t})^3 + 3\\int\\sin^2t\\cos^2t(\\cos^2t + \\sin^2t)\\: dt \\\\\r\n0 & = (\\sin{t}\\cos{t})^3 + 3\\int\\sin^2t\\cos^2t\\: dt \\end{align*}\r\n\r\nheh.. I'm probably going way off-course.", "Solution_9": "No need really for this kind of calculations, but I can't guarantee that you won't solve the problem this way :lol: I thought a bit and found two more ways (well, the first one was the first I used, and the second one is more a showing off than a method):\r\n\r\n1. $ \\sin x\\equal{}\\frac{e^{ix}\\minus{}e^{\\minus{}ix}}{2i}$. Now, if you plug it in those two sine integrals, using binomial theorem reduces you to the easily integrable form shown in 2.\r\n\r\n2. Theoretically, after using addition (multiple angle) trig formulae, it is possible to obtain: \r\n\r\n$ \\frac{1}{32}(2 \\minus{} \\cos2t \\minus{} 2 \\cos4t \\plus{}\\cos6t)$ which is the same you'll get in Method 1 using binomial theorem and proper grouping of elements.", "Solution_10": "I think I've seen that form $ \\sin{x}\\equal{}\\cfrac{e^{ix}\\minus{}e^{\\minus{}ix}}{2i}$ before, but sorry I'm not familiar with it. What's it called? Hopefully I can look it up and familiarize myself :)", "Solution_11": "Well, I call it the complex form (or, even better-complex definition) of sine. $ x$ doesn't have to be real $ (x\\in\\mathbb{C})$.", "Solution_12": "ooh I found this page: [url]http://en.wikipedia.org/wiki/De_Moivre%27s_formula[/url]\r\n\r\nthat explains it in sorta easy terms..\r\n\r\nI'll just need to eventually convince myself that $ e^{ix} = \\cos{x} + i\\sin{x}$\r\n\r\nBut yes! It works! :lol:\r\n\\[ - 3\\left(\\int_0^\\frac {\\pi}{2}\\sin^4t\\: dt - \\int_0^\\frac {\\pi}{2}\\sin^6t\\: dt\\right)\r\n\\]\r\n[hide=\"First integrand\"]\n\\begin{align*} \\sin^4t & = \\left(\\cfrac{e^{ix} - e^{ - ix}}{2i}\\right)^4 \\\\\n& = \\cfrac{1}{8}\\cdot\\cfrac{e^{4ix} + e^{ - 4ix}}{2} - \\cfrac{4}{8}\\cdot\\cfrac{e^{2ix} + e^{ - 2ix}}{2} + \\cfrac{6}{16} \\\\\n& = \\cfrac{1}{8}\\cos{4x} - \\cfrac{1}{2}\\cos{2x} + \\cfrac{3}{8} \\end{align*}\n[/hide]\n[hide=\"Second integrand\"]\nI spent a bit more time on this one because I kept messing up pos/neg signs and the fractions. :P\n\\begin{align*} \\sin^6t & = \\left(\\cfrac{e^{ix} - e^{ - ix}}{2i}\\right)^6 \\\\\n& = - \\cfrac{1}{32}\\cdot\\cfrac{e^{6ix} + e^{ - 6ix}}{2} + \\cfrac{6}{32}\\cdot\\cfrac{e^{4ix} + e^{ - 4ix}}{2} - \\cfrac{15}{32}\\cdot\\cfrac{e^{2ix} + e^{ - 2ix}}{2} + \\cfrac{20}{64} \\\\\n& = - \\cfrac{1}{32}\\cos{6x} + \\cfrac{3}{16}\\cos{4x} - \\cfrac{15}{32}\\cos{2x} + \\cfrac{5}{16} \\end{align*}\n[/hide]\r\n\r\nSo combining the two parts we see that the original expression equals:\r\n\\[ - 3\\left[\\int_0^\\frac {\\pi}{2}\\left(\\cfrac{1}{8}\\cos{4x} - \\cfrac{1}{2}\\cos{2x} + \\cfrac{3}{8}\\right)\\: dx - \\int_0^\\frac {\\pi}{2}\\left( - \\cfrac{1}{32}\\cos{6x} + \\cfrac{3}{16}\\cos{4x} - \\cfrac{15}{32}\\cos{2x} + \\cfrac{5}{16}\\right)\\: dx\\right]\r\n\\]\r\nwhich, as hsiljak said, equals\r\n\\[ - 3\\left[\\int_0^\\frac {\\pi}{2}\\cfrac{1}{32}(2 - \\cos{2x} - 2\\cos{4x} + \\cos{6x})\\: dx\\right]\r\n\\]\r\nThen simply:\r\n\\begin{align*} & - \\cfrac{3}{32}\\left(\\int_0^\\frac {\\pi}{2}2\\: dx - \\int_0^\\frac {\\pi}{2}\\cos{2x}\\: dx - 2\\int_0^\\frac {\\pi}{2}\\cos{4x}\\: dx + \\int_0^\\frac {\\pi}{2}\\cos{6x}\\right) \\\\\r\n& = - \\cfrac{3}{32}\\left(\\left[\\cfrac{}{}2x\\right]_0^\\frac {\\pi}{2} - \\left[\\cfrac{1}{2}\\sin{2x}\\right]_0^\\frac {\\pi}{2} - \\left[\\cfrac{2}{4}\\sin{2x}\\right]_0^\\frac {\\pi}{2} + \\left[\\cfrac{1}{6}\\sin{6x}\\right]_0^\\frac {\\pi}{2}\\right) \\\\\r\n& = \\boxed{ - \\cfrac{3}{32}\\pi} \\end{align*}\r\nyeah! :w00t:\r\n\r\nThanks for the help, hsiljak, and thanks for the problems kunny!\r\n\r\nEDIT: oops, er I accidentally switched variables along the way, but you knew what I meant. :lol:" } { "Tag": [ "LaTeX", "function" ], "Problem": "There are a few minor changes in the way the LaTeX code is rendered on the site. For those of you unfamiliar with LaTeX, read the [url=http://www.artofproblemsolving.com/LaTeX/AoPS_L_About.php]LaTeX guide[/url] provided by this site to learn more about typesetting mathematics. \r\n\r\nFirst of all, all the formulas are rendered with the \\displaystyle command from default. You need not add that command anymore. The \\displaystyle command makes fractions, summations, etc., render more legibly, as shown below:\r\n\r\nWith \\displaystyle: $ \\sum_{k \\equal{} 0}^n \\frac 1{k}$ \r\n\r\nWithout \\displaystyle: $ \\textstyle \\sum_{k \\equal{} 0}^n \\frac 1{k}$. \r\n\r\nTo force your posts to be without \\displaystyle you can use the \\textstyle \r\n command (start your LaTeX code with \\textstyle). \r\n\r\nSecond, we encourage all users to use the normal LaTeX syntax, i.e. the dollar signs, instead of the [tex ] and [/tex ] tags (without spaces there). \r\n\r\nThe [tex ] button will still be available and can still be used as before, but after rendering the text, the forum will automatically transform the [tex ] tags into equivalent dollar signs. \r\n\r\nIf you do not edit your post, you will \"feel nothing\". If you do, the [tex ] tags that you had used will be replaced by dollar signs. \r\n\r\nPS I do not know if everyone knows this already, so I will mention it here again: on most browsers the latex code used to produce a block of math text (the whole latex code, including the dollar signs) can be viewed by holding the mouse a couple of seconds still above the picture.", "Solution_1": "[quote=\"RC-7th\"]Is there a command for the thing that makes this $\\textstyle \\sum\\limits_{k=0}^n \\frac 1{k}$ different from this \\[\\textstyle \\sum\\limits_{k=0}^n \\frac 1{k}\\] ?[/quote] You probably didn't carefully read the text above. You are using the \\limits command, which automatically acts as a \\displaystyle. Without the \\limits command (which is not necessary!), the code above would be \r\n$ \\textstyle \\sum^n_{k=0} \\frac 1k $.", "Solution_2": "The \\limits thing is just a habit. I was wondering if there was a command equivalent of using double dollar signs [code]($$)[/code] on the ends.", "Solution_3": "next time, if you want to display dollars use [code ] (dollars) [/code ] (without spaces), and use all the time the function preview, to see if there are any errors in your messages ;) \r\n\r\nas for your question, if you had carefully read the post above, you would have discovered that \"we encourage all users to use the normal LaTeX syntax\", that means any known LaTeX syntax, including double dollars as\r\n\r\n\\[ \\frac 12 \\cdot \\frac 34 = \\frac 38 \\] does. \r\n\r\nPlease read carefully the general annoucements, as they usually contain all the answers you need. :) \r\n\r\nGood luck \"tExING\" :D", "Solution_4": "The feature of some web browsers to show LaTeX code when scrolled over that v.v. remarked about on the original post does not work in Mozilla.", "Solution_5": "Or Opera..", "Solution_6": "Sorry guys, I have just tested it on IE Explore, and Semaca Mobile browser for SE (where it works!). \r\n\r\nIt is curious why it does not work in Opera, or in Mozzila. I will try to see what is wrong ... :maybe: \r\n\r\nIf anyone knows more about these browsers, you can check the source code yourselves and tell me what exactly is wrong :) ;)", "Solution_7": "We might be able to add netscape to that list (or maybe my version is simply too old :? )", "Solution_8": "Ok, I've identified the problem. It simply seems that Opera, Mozzila and Netscape just do not accept the alt tag. Thus, there is only one solution. Tell me if it works ;)", "Solution_9": "Works fine here (Opera speaking :)).", "Solution_10": "Works with Firefox now. Thanks.", "Solution_11": "Lemme try in Firefox:\r\n\r\n\\[\r\n\\frac{1}{2}! = \\frac{1}{2}\\Gamma{}\\left(\\frac{1}{2}\\right) = \\frac{\\sqrt{\\pi}}{2}\r\n\\]", "Solution_12": "if you type $$ and $$ at the ends of a sentence, does the code always come out as text in the center of a line and always occupy the whole line?\r\nFor example if you type \"I have $$3$$ dollars\", it will come out as \r\n\r\nI have\r\n 3\r\ndollars. \r\n\r\n\r\nHow can you aviod this?", "Solution_13": "[quote=\"henryyangrui\"]if you type \\[ and \\] at the ends of a sentence, does the code always come out as text in the center of a line and always occupy the whole line?\nFor example if you type \"I have \\[3\\] dollars\", it will come out as \n\nI have\n 3\ndollars. \n\n\nHow can you aviod this?[/quote]What do you mean? This is how normal $\\LaTeX$ works. If you want inline equations, use a single dolar sign at the beginning and the end of a formula: $\\$ \\frac 12 $\\$ will reveal $ \\frac 12 $ ;)", "Solution_14": "Oh, i used double $ instead. Thx." } { "Tag": [ "geometry", "trapezoid", "LaTeX" ], "Problem": "Hey..ive been bustin my brains with this problem. It might be easy but im still having troubles\r\n\r\nA equivalent triangle (A,B,C) with 10 cm as sides thus all angles are 60 degrees.\r\nA half-circle passes trough point A and B passing trough the interior of the triangle, yet there is a small portion of the circle outside the triangle. Find the area of that portion.\r\n\r\nIf you dont understand the question, just tell me. I'll try to do a drawing", "Solution_1": "BTW...\r\nall i need is a guideline of how to do it. \r\nI dont need the solution\r\nthx in advance", "Solution_2": "edit: oops.", "Solution_3": "hmm. i guess this is the picture, right??\r\n[hide=\"hint\"]\nangle $AOM=120$ because $\\triangle MOB$ is equalateral. with side length of $5$.\nnow find $S_{\\widehat {AOM}+\\triangle MOB}$ and subtract if from the area of the half circle[/hide]", "Solution_4": "sorry...i seem to have miss explainedthe question.\r\nfirst of all, it is a half circle we are talking about...\r\nsecond of all, i forgot to mention that the diameter of the circle is a side of the triangle so a portion of the circle would not be in the triangle....\r\n\r\n\r\ni could do an image, but we're can i host it???\r\n\r\nEDIT...\r\ni am in grade 10 math, dont over complicate plz", "Solution_5": "here's a picture i drew...\r\nfind the area of the shaded part\r\n\r\nBTW...each side is 10 cm", "Solution_6": "[hide=\"tostart you off..\"]\ni dont know if i made a correct assumption but anyways:\ndraw a line at top of circle to form a trapezoid at bottom of the equiliateral triangle (in other words, 3 smaller equilateral triangles of sidelength 5. \n[/hide]\n[hide=\"stilldontgetit?\"]\nThe area of an equilateral triangle is \nS^2*sqrt3/4 where s is sidelength, so the area of the three triangles is 75sqrt3/4 \nSince we know the diameter of the circle to be 10 we know that half the circle would be 25pi/2\nthe area o fthe two shaded things and thing at top of circle is 25pi/2-75sqrt3/4 <--\nim assuming they are all the same so u multiply this value ------------------------------| by 2/3\nim not gonna bother to actually calculate it but u get the idea :D[/hide]\r\n\r\nEdit: sry i havent bothered to learn latex yet so i hope you can still read it :P", "Solution_7": "I agree with sheepwarrior, the answer is \\[\\frac{2}{3}\\left(\\frac{25\\pi}{2}-\\frac{75\\sqrt{3}}{4}\\right).\\] Here's a picture:" } { "Tag": [ "geometry proposed", "geometry" ], "Problem": "[b]Let ABCD be a convex quadrilateral inscribed in the circle[/b] $w$.[b] I note the [/b][b]intersections[/b]\n\n$S\\in AC\\cap BD,\\ P\\in BB\\cap DD,\\ R\\in AA\\cap CC$. [b]Prove that:[/b]\n\n$1.\\ AB.CD=AD.BC\\Longleftrightarrow R\\in BD\\Longleftrightarrow P\\in AC$.[b] ($XX$ is the[/b] [b]tangent in the point[/b] $X\\in w$[b]).[/b]\n\n[b]2. The point S is the foot of a simedian-line at least in the one from the triangles ABC, ABD, ACD, BCD.[/b]", "Solution_1": "P = BB intersect DD ??? I think I am seeing deja vu... :( \r\n\r\ncould you explain?", "Solution_2": "Did you read the last sentence, al.M.V.? There, levi explains what he means by $XX$, in general. The post isn't that long, man, you could have read the whole thing.. :)", "Solution_3": "al.m.v ,i think levi mean :tangent ;)", "Solution_4": "Yes" } { "Tag": [ "geometry", "number theory", "relatively prime" ], "Problem": "Prove or give a counterexample that a right angled triangle with integer sides can't have area as a perfect square.", "Solution_1": "because one leg is odd the other is even.....and the odd cant be a perfect square without the even containing an odd multiple of it too", "Solution_2": "[quote=\"usaha\"]because one leg is odd the other is even.....and the odd cant be a perfect square without the even containing an odd multiple of it too[/quote]\n\n[quote]$\\cdots$ and the odd cant be a perfect square without the even containing an odd multiple of it too[/quote]\r\ncan you elborate more?", "Solution_3": "I welcome criticism of this proof as I am trying to become better at this sort of thing.\r\n\r\nProof:\r\n\r\n[hide]\nIt can be shown that all primitive pythagorean triples (primitive in that there is no factor common to the lengths of all three sides) can be described for positive integers $a$ and $b$, where $a>b$, such that the two legs are $a^{2}-b^{2}$ and $2ab$ and the hypotnuse is $a^{2}+b^{2}$. \n\nThus the area of any such triangle is equal to half base times height or:\n$A=\\frac{1}{2}(a^{2}-b^{2})(2ab)=(a+b)(a-b)ab$\n\nConsider non-primitive cases; all such cases can be described as a primitive case with each side multiplied by a common positive integer factor $k$. For such a non-primitive case, the area would be $A=k^{2}(a+b)(a-b)ab$ and for A to be a perfect square, the $k^{2}$ factors and the product $(a+b)(a-b)ab$ must still be a perfect square for the area to be a perfect square. \n\nThus, all we must do is show that the product $(a+b)(a-b)ab$ cannot be a perfect square to cover all primitive and non-primitive cases.\n\nThus the only cases to consider are:\n1) $(a-b)=(a+b)ab$\n2) $(a+b)=(a-b)ab$\n\nSince a and b are both positive integers and a>b, case 1 is clearly rediculous.\n\nManipulating case 2 gives:\n$a-b=\\frac{1}{a}+\\frac{1}{b}$\nSince a and b are positive integers, this could only be possible if a-b=1. Thus:\n$1=\\frac{1}{b+1}+\\frac{1}{b}$\nAnd this clearly does not work for any positive integer b. \n\nThus the product $(a+b)(a-b)ab$ for relatively prime a and b cannot ever be a perfect square, and thus the area of a right triangle with integer sides cannot ever be a perfect square.[/hide]", "Solution_4": "[quote=\"usaha\"]and the odd cant be a perfect square without the even containing an odd multiple of it too[/quote]\n\nNot true. Consider the triple $9, 40, 41$. \n\n[quote=\"bthings_2000\"]Thus, all we must do is show that the product $(a+b)(a-b)ab$ cannot be a perfect square to cover all primitive and non-primitive cases.\n\nThus the only cases to consider are:\n1) $(a-b)=(a+b)ab$\n2) $(a+b)=(a-b)ab$[/quote]\r\n\r\nI have a problem with this narrowing down to two cases; what about the case where $a+b, a-b, a, b$ are all simultaneously square?", "Solution_5": "Hmm, good point t0rajir0u, and I've messed around with that for a while and not figured it out... input from anyone welcome.", "Solution_6": "Yea toujorou, you're right.\r\n\r\nWhat i was trying to say is that (this is like a guess) if a leg is a perfect square, the other side must contain an odd power of an odd multiple (so 9, 40, 41 the 40 has only 1 multiple of 5), so when you do the area, it cant be a perfect square. Also, for like generic cases like 5,12,13, im guessing that since one is even the other is odd, that when you multiply it up there will be an odd number power of an odd multiple, AI dont know what i just said.", "Solution_7": "Well, we can reduce it to the case of a primitive triangle (since dividing each side by $g$ divides area by $g^{2}$, so it remains a perfect square), and then one leg is twice a square and the other leg is a square. So we're looking at\r\n\\[4a^{4}+b^{4}=c^{2}\\]\r\nin positive integers.\r\n\r\nProbably factoring the left hand side or using primitive Pythagorean triple formula could help here.", "Solution_8": "some treatment available here... external link.\r\n[hide][url]http://mcraefamily.com/MathHelp/PythagTriangleAreaNotSquare.htm[/url][/hide]" } { "Tag": [ "function", "inequalities", "logarithms", "induction", "rearrangement inequality", "algebra unsolved", "algebra" ], "Problem": "Let $f(x)=ax^{2}+bx+x$ $(a,b,c>0)$; $a+b+c=1$, $x_{1}>0,x_{2}>0,...,x_{n}>0$, $x_{1}x_{2}...x_{n}=1$\r\nProve that:\r\n$f(x_{1})f(x_{2})f(x_{3})...f(x_{n}) \\geq 1$", "Solution_1": "$f(x)f(1/x)=a^{2}+b^{2}+c^{2}+ab(x+1/x)+ac(x^{2}+1/x^{2})+bc(x+1/x)=1+(ab+bc)\\frac{(x-1)^{2}}{x}+ac\\frac{(x^{2}-1)^{2}}{x^{2}}\\ge 1$.\r\nWe have $f'(x)\\ge 0$ and $f(1)=1$. It give your result.", "Solution_2": "It's not clear to me how you get the result from what you've written.\r\n\r\nHow about the following:\r\n\r\nFor $x < 1 < y$, \r\n$f(x)f(y)=a^{2}+b^{2}+c^{2}+ab(x^{2}y+x y^{2})+ac(x^{2}+y^{2})+bc(x+y) \\geq a^{2}+b^{2}+c^{2}+ab(x^{2}y^{2}+x y)+ac(x^{2}y^{2}+1)+bc(xy+1) = f(xy) f(1)$,\r\n\r\nusing the rearrangement inequality for each term. So if the $x_{i}$ are not identically 1, we can convert a non-1 value to 1 while decreasing the product of the $f(x_{i})$. Eventually all the $x_{i}$ are equal to 1, and the product of the $f(x_{i})$ is exactly 1.", "Solution_3": "$f(xy)f(1)-f(x)f(y)=(abxy+bc)(x-1)(y-1)+ac(x^{2}-1)(y^{2}-1)$.\r\nLet $x_{1}\\le x_{2}\\le ....x_{k}\\le 1\\le x_{k+1}\\le...\\le x_{n}$. Let $\\sigma(1)=1,y_{1}=x_{1},\\sigma(k+1)=n+1-\\sigma(k), \\ if \\ y_{k}<1, \\ sigma(k+1)=\\sigma(k)-1 \\ else \\ y_{k+1}=y_{k}*x_{\\sigma(k+1)}.$ Then\r\n$f(y_{k+1})\\lef(y_{k})f(x_{\\sigma(k)})$. Therefore\r\n$1\\le \\pmod_{k}f(x_{k}).$", "Solution_4": "I'm afraid I'm not following this either.\r\n\r\nFor example, if $y_{1}> 1$, then $\\sigma(2) = 0$.", "Solution_5": "If $y_{k}>1$ we find little $x_{m}$ don't used before. If $y_{k}<1$ we find biggest $x_{m}$ don't used before. Define $m=\\sigma (k+1)$. \r\nIt give $(y_{k+1}=y_{k}x_{\\sigma (k+1)}, \\ (y_{k}-1)(x_{\\sigma (k+1)}-1)\\le 0 \\ \\forall k .$", "Solution_6": "is there any solutions else? I dont like Rust's Solution :wink:", "Solution_7": "I just did this problem in a practice test (1990 All-Soviet Olympiad, I think?)\r\n\r\nSay $e^{a_{i}}= x_{i}$. Let $g(x) = \\log (a e^{2x}+b e^{x}+c)$. Then $g''(x) > 0$ for $x > 0$ (this is easy to confirm, unless I made an algebra error). Then we have $\\sum a_{i}= 0$. We need to show $\\sum g(a_{i}) = 0$. But Karimata gives $\\sum g(a_{i}) \\geq \\sum g(0)$, and since $a+b+c = 1$, $g(0) = \\log(1) = 0$, and we have the desired result.\r\n\r\nDeedlit: Maybe I am missing something, but you appear to be using rearrangement backwards? Certainly $x+y \\leq x y+1$ is not a true statement.", "Solution_8": "[quote=\"hieuchuoi@\"]is there any solutions else? I dont like Rust's Solution :wink:[/quote]\n\nWas there a problem with mine?\n\n[quote]Deedlit: Maybe I am missing something, but you appear to be using rearrangement backwards? Certainly x+y \\leq x y+1 is not a true statement.[/quote]\r\n\r\nWhoops, yes the inequality is the wrong way. I'll fix it.", "Solution_9": "A nice solution :D \r\nUse this lemma:\r\n$f(x).f(y) \\geq f(\\sqrt{xy})^{2}$\r\nThe rest is for you :roll: !", "Solution_10": "It is nice. But we need $f(x_{1})f(x_{2})...f(x_{n})\\ge(f((x_{1}...x_{n})^{1/n}))^{n}.$\r\nFor example consider $n=3.$", "Solution_11": "Yes, we easy have it by induction", "Solution_12": "Explain it for $n=3$.", "Solution_13": "sorry, i was wrong, and this is two new solutions\r\n1) you can apply Holder ineq :D \r\n2) you can prove with n is odd, n is even and we have conclusion\r\nif n is odd, we add $a_{n+1}=1$ :D", "Solution_14": "It is easy to prove that $f(x)f(y)\\geq (f(\\sqrt{xy}))^{2}$\r\nNow suppose that $f(x_{1})...f(x_{n})\\geq (f(\\sqrt[n]{x_{1}..x_{n}}))^{n}$ \r\n\r\nis true for $n=2^{k}$ and prove it for $n=2^{k+1}$ :wink:", "Solution_15": "no, it's true for any positive integer $n$ :wink:", "Solution_16": "I can prove it forall n. But it is not trivial for $n\\not =2^{k}$, for example for n=3.", "Solution_17": "Well, the easy way would be to take the logarithm and apply Jensen's inequality, which would make it essentially the same as 101101101's proof.", "Solution_18": "[quote=\"hieuchuoi@\"]no, it's true for any positive integer $n$ :wink:[/quote]\r\n\r\nWho said that ? :wink: \r\nAfter the proof for $n=2^{k}$ suppose $n$ is arbitary and $x_{1}..x_{n}=1$ . Let $k$ a positive integer such that $2^{k-1}0$. Let $a=a_0,a_1a_2a_3...$ and $b=b_0,b_1b_2b_3...$ (binary representation).\r\nChoose large $n$ s.t. $2^na>10$ and $2^{2n}b>10$. Then $[2^na]\\cdot [2^{2n}b]=\\overline{a_0a_1...a_n}\\cdot \\overline{b_0b_1...b_{2n}}$ and $2^{3n}ab>\\overline{a_0a_1...a_n}\\cdot \\overline{b_0b_1...b_{2n}}+10\\cdot 0,a_{n+1}a_{n+2}...+10\\cdot 0,b_{2n+1}b_{2n+2}...$. It follows that $10\\cdot 0,a_{n+1}a_{n+2}...<1$ and $10\\cdot 0,b_{2n+1}a_{2n+2}...<1$, i.e. $a_{n+1}=0$ and $b_{2n+1}=b_{2n+2}=0$, so $a_n=b_n=0$ for all large $n$. And we can choose arbitrary base instead of 2. Therefore $a$ and $b$ are integer.\r\n\r\nI applied the same analysis for cases $(a>0,b<0)$, $(a<0,b>0)$ and $(a<0,b<0)$. However, to write it here is to long." } { "Tag": [], "Problem": "Exista vreun sir real marginit $\\left( x_{n}\\right)_{n \\geq 1}$ astfel incat $\\left| \\left( x_{i}-x_{j}\\right) \\left( i-j \\right) \\right| \\geq 1, \\, \\forall i \\neq j$?", "Solution_1": "Asta e problema 6 de la olimpiada sovietica 1978.\r\nUn exemplu de astfel de \u015fir este $x_{n}=4\\{n\\sqrt{2}\\}.$" } { "Tag": [ "algebra solved", "algebra" ], "Problem": "if a+b+c=2 then prove that:\r\n \r\n3(a^3+b^3+c^3)+10(ab+bc+ca)>=16", "Solution_1": "after homogenisation it becomes $a^3+b^3+c^3+3abc\\geq ab(a+b)+bc(b+c)+ac(a+c)$. \r\nYou may suppose $a\\geq b\\geq c$.\r\nIf $c\\geq 0$, it is true by Schur.\r\nIf $c<0$; put x=a+b. Then the conclusion follows rapidly from x>2" } { "Tag": [ "group theory", "abstract algebra", "superior algebra", "superior algebra unsolved" ], "Problem": "Let $ G$ be a finite group of order $ n.$ Suppose $ G$ has a simple subgroup of order $ >n/3$ and also has a proper normal subgroup of order $ 3.$ Then $ n\\equal{}6.$", "Solution_1": "Suppose that $ H\\leq G$ is simple, $ N\\lhd G$, $ |N|\\equal{}3$, $ |H| > \\frac{n}{3}$.\r\n\r\nIf $ H \\equal{} G$, then $ G$ is simple so $ N\\equal{}G$ but then $ N$ is not a proper subgroup, contradiction.\r\n\r\nSo $ |H|\\equal{}\\frac{n}{2}$. Let $ N \\equal{} \\langle x \\rangle$, then the cosets $ H$, $ xH$, $ x^2 H$ cannot be distinct, so we have $ x\\in H$. Hence $ N \\lhd H$, so $ N\\equal{}H$ and $ 3 \\equal{} \\frac{n}{2}$." } { "Tag": [], "Problem": "I'm having trouble with this problem. Can someone help me, not necessarily giving me the answer but hints would be nice. :) thanks!\r\n[b]\nFor each of n=84 and n=88, find the smallest integer multiple of n whose base 10 representation consists entirely of 6's and 7's.[/b]", "Solution_1": "[hide]\nWell, you should prime factorize them, and then look at each factor by itself.\nFor example, 88 is 11*8, so you should see how you can arrange the last 3 digits to make it divisible by 8, then see which of these arrangements you can add a digit or two to to make it divisible by 11 and have it be the smallest.\n\nAnd, 84 is 7 * 4 * 3, so you look at arrangements of the last 2 digits, and you see it must be 76. So, obviously there are have to be 3 7s. (to be divisible by 3), so see how you can add 2 more 7's and as few 6s as possible to the left of the 76.\n[/hide]", "Solution_2": "wow! that makes a lot of sense, i never would have thought of that...thanks a lot klebian!!! :)" } { "Tag": [ "algebra", "polynomial", "linear algebra", "linear algebra unsolved" ], "Problem": "For each $n$, denote \r\n\\[ P_n(X)=nX^{n-1}+(n-1)X^{n-2}+...+2X+1. \\]\r\nFind all $n$ such that the polynomial $P_n(X)$ is irreducible over $Q$.", "Solution_1": "I can prove that it's prime for $n$ prime... Valentin Vornicu has a similar problem, for $n=2004$.\r\n\r\nhttp://www.mathlinks.ro/Forum/viewtopic.php?t=75053&postorder=asc", "Solution_2": "As I assume everyone is well aware $P_n(X)=(X+X^2+...+X^n)'$ so I made a few computations and got that $P_n(X)=\\frac{X^{n+1}-2X^n+1}{(X-1)^2}$ (assuming I made the computations correct)... now what?!... does this help?", "Solution_3": "[quote=\"spix\"]As I assume everyone is well aware $P_n(X)=(X+X^2+...+X^n)'$ so I made a few computations and got that $P_n(X)=\\frac{X^{n+1}-2X^n+1}{(X-1)^2}$ (assuming I made the computations correct)... now what?!... does this help?[/quote]\r\nMaybe, but I thinks that sense is difficult to finish.", "Solution_4": "Guys, don't kid yourself, the problem is still open as far as I know.", "Solution_5": "[quote=\"harazi\"]Guys, don't kid yourself, the problem is still open as far as I know.[/quote]\r\nI want to discuss this problem, and if any has solutions for some n, please write them here.\r\nfor n<20000, it has been done by computer aid.", "Solution_6": "for $n$ prime it's not hard... Use the lemma posted here\r\nhttp://www.mathlinks.ro/Forum/viewtopic.php?p=440913#p440913" } { "Tag": [ "calculus", "integration", "calculus computations" ], "Problem": "A particel is moved along the x-axis by a force that measures 10/(x+1)^2 pounds at a point x feet from the origin. Find the work done in moving the particle from the origin to a distance of 9 feet.\r\n\r\ncan you give me the example and answer, here's my work\r\n\r\ni antidifferentiated 10/(1+x)^2 into U^-1 or 10U^-1 if you want to leave the 10 their, then i put in 1+x into the u,\r\n\r\n1/2 integral x=0, x=9, 10(1+9)^-1= 1 1 times 1/2 equals 1/2\r\n\r\nthen i plugged in 0\r\n\r\nand i got 5, then i subtracted 1/2 - 5=and this equals -9/2 \r\nand the answer is 9 ft-lb\r\n\r\nso what did i do wrong, and can u show me how u got the answer", "Solution_1": "[quote=\"afcweswarrior\"]i antidifferentiated 10/(1+x)^2 into U^-1 or 10U^-1[/quote] Which should have been $-10U^{-1}$ or $-10(1+x)^{-1}$. \n\n[quote=\"afcweswarrior\"]1/2 integral[/quote] Why did you put $1/2$ here?", "Solution_2": "when i substituted (\r\n1+x)\r\n\r\noh man, that's what i did wrong i substituted 1+x^2 \r\ni was suppost to let u=1+x" } { "Tag": [ "calculus", "Olimpiada de matematicas" ], "Problem": "Este problema esta sencillo, pero bueno\r\n\r\nHalle todas las soluciones enteras al sistema de ecuaciones\r\n$a+b+c=1$\r\n$a^3+b^3+c^2=1$", "Solution_1": "Bonito problema. Tengo dos soluciones, pero prefiero no publicarlas todavia. Ahora, doy una \"pista\" bastante evidente que puede ayudar:\r\n\r\n[hide=\"Pista\"]\n1. Sirve aplicar productos especiales, reacomodando los terminos y dividiendo en casos. No son tantos.\n[/hide]", "Solution_2": "Quien quiere colocar su solucion?", "Solution_3": "Ok, coloco mi solucion \r\n\r\n[hide=\"\"Solucion\"\"] Primero, si c=1, obtenemos que \n$a+b=0\\Rightarrow a^3+b^3=(a+b)(a^2-ab+b^2=0=1-c^2$\nEntonces todas las triplas (a,b,c)=(x,-x, 1) cumplen. Ahora si $c\\not=1$ podemos escribir la segunda ecuacion como como\n$(a+b)(a^2-ab+b^2)=(1-c)(1+c)$\nComo $c-1\\not=0$ podemos dividir a ambos lados por $c-1=a+b$ y nos queda\n$a^2-ab+b^2=1+c$\nAdemas, sabemos que \n$a^2+2ab+b^2=(1-c)^2$\nRestabdi ka segunda menos la primera, obtenemos que\n$3ab=c(c-3)$\nEntonces vemos que $3|c$, de donde $c=3k$ y queda que\n$ab=3k(k-1)$\n$a+b=1-3k$\nEntonces se tiene que $a,b$ son las raices de la ecuacion\n$X^2-(1-3k)X+3k(k-1)=0$\nDonde $\\Delta=-3k^2+6k+1$. Queremos que $\\Delta>0$, y con simples calculos obtenemos que $0\\leq k\\leq 2$. Por ultimo vemos que en cada uno de estos casos, $\\Delta$ es un cuadrado perfecto, entonces resolvemos y queda\nSi $k=0$, entonces $\\{a,b\\}=\\{0,1\\}$ y obtenemos las triplas $(a,b,c)=(0,1,0);(1,0,0)$. Si $k=1$, $\\{a,b\\}=\\{-2,0\\}$ de donde se tienen las triplas $(a,b,c)=(-2,0,3);(0,-2,3)$. Por ultimo, cuando $k=2$, obtenemos $\\{a,b\\}=\\{-3,-2\\}$, dandonos las triplas $(a,b,c)=(-3,-2,6);(-2,-3,6)$. Con esto se tienen todas las triplas y se sigue lo pedido.[/hide]" } { "Tag": [ "inequalities proposed", "inequalities" ], "Problem": "let a,b,c are the sides of a triangle.prove that:\r\n\\[ \\sum_{a,b,c}(\\frac{\\sqrt{b}\\plus{}\\sqrt{c}\\minus{}\\sqrt{c}}{\\sqrt{b\\plus{}c\\minus{}a}})\\leq3\\]", "Solution_1": "[quote=\"stvs_f\"]let a,b,c are the sides of a triangle.prove that:\n\\[ \\sum_{a,b,c}(\\frac {\\sqrt {b} \\plus{} \\sqrt {c} \\minus{} \\sqrt {c}}{\\sqrt {b \\plus{} c \\minus{} a}})\\leq3\n\\]\n[/quote]\r\nSee here:\r\nhttp://www.artofproblemsolving.com/Forum/viewtopic.php?t=130812" } { "Tag": [ "integration" ], "Problem": "Show that \\[\\sum_{i = 1}^{n + 1} \\frac{2^i}{i} \\cdot {n \\choose {i - 1}} = \\frac{3^{n + 1} - 1}{n + 1}.\\]", "Solution_1": "Hi,\r\n\r\n\r\nMay be proved by noting that \\[{{n+1} \\choose {i}} = \\frac{n+1}{i} {{n} \\choose {i-1}}\\]\r\n\r\n(or by integrating binomial sum)", "Solution_2": "$\\sum_{i=1}^{n+1} \\frac{2^i}{i}\\ _nC_{i-1}=\\sum_{r=0}^n \\frac{2^{r+1}}{r+1}\\ _nC_r$ where $r=i-1$\r\n\r\n$=\\sum_{r=0}^n 2^{r+1} \\int_0^1 x^r dx\\cdot\\ _nC_r $\r\n\r\n$=2\\int_0^1 \\sum_{r=0}^n \\ _nC_r (2x)^r dx$\r\n\r\n$=2\\int_0^1 (1+2x)^n dx$\r\n\r\n$=2\\left[\\frac{(1+2x)^{n+1}}{2(n+1)}\\right]_0^1 =\\frac{3^{n+1}-1}{n+1}$", "Solution_3": "Here is a standard solution. \r\n\r\n$\\sum_{i=1}^{n+1} \\frac{2^i}{i} \\ _nC_{i-1}$\r\n\r\n$=\\sum_{i=1}^{n+1} \\frac{2^i}{i} \\frac{n!}{(i-1)!(n-i+1)!}$\r\n\r\n$=\\sum_{i=1}^{n+1} 2^i\\cdot \\frac{(n+1)!}{i!(n-i+1)!}\\cdot\\frac{1}{n+1}$\r\n\r\n$=\\frac{1}{n+1} \\sum_{i=1}^{n+1}\\ _{n+1}C_i\\ 2^i$\r\n\r\n$=\\frac{1}{n+1}\\left(\\sum_{i=0}^{n+1}\\ _{n+1}C_i\\ 2^i - \\ _{n+1}C_0\\ 2^0\\right)$\r\n\r\n$=\\frac{1}{n+1}\\{(1+2)^{n+1}-1\\}=\\frac{3^{n+1}-1}{n+1}$" } { "Tag": [ "linear algebra", "matrix", "LaTeX" ], "Problem": "For each of following systems of lin.equations Ax = B, factor A as LU, solve LC = B for c using forward substitution, solve UX = C be back substitution. \r\n\r\n\r\n1) 2x + y = -3\r\n 6x + 4y = -4\r\n\r\n\r\n2) x + 2y + 0z = 3\r\n 2x + 6y -z =7\r\n 5x - 4y + 10z =4\r\n\r\n\r\nThank you", "Solution_1": "Can someone show me how to forward and backward substitute?", "Solution_2": "So do you mean elimination and back substitution? I am just learning so even if that is not so, may I try to solve?\r\n\r\nFOR #1 I think:\r\n\r\n1) 2x + y = -3 \r\n6x + 4y = -4 \r\n\r\nthe matrix is(sorry i don't have latex)\r\n\r\n2 1 -3\r\n6 4 -4\r\n\r\nso we mult. by -3 and add to row 2(note that 2 & 4 are pivots..):\r\n\r\n2 1 -3\r\n0 1 5\r\n\r\nand thus we have\r\n\r\n2x+y=-3\r\ny=5\r\nso \r\nx=-4\r\n\r\nIf you are still on this site please tell me if this is what you were looking for and I can show you #2.", "Solution_3": "First you need to perform the LU factorization. For small systems with nice numbers you can usually do this in your head:\r\n\r\nFactorization:\r\n$A = LU \\implies \\begin{bmatrix}2 & 1 \\\\ 6 & 4 \\end{bmatrix}= \\begin{bmatrix}1 & 0 \\\\ 3 & 1 \\end{bmatrix}\\begin{bmatrix}2 & 1 \\\\ 0 & 1 \\end{bmatrix}$\r\n\r\nForward substitution goes from the top row to the bottom row. Each row you solve gives you just enough values to solve for the next row:\r\n$L\\widehat{c}= B \\implies \\begin{bmatrix}1 & 0 \\\\ 3 & 1 \\end{bmatrix}\\begin{bmatrix}c_{x}\\\\ c_{y}\\end{bmatrix}= \\begin{bmatrix}-3 \\\\-4 \\end{bmatrix}\\implies \\begin{bmatrix}c_{x}\\\\ c_{y}\\end{bmatrix}= \\begin{bmatrix}-3 \\\\ 5 \\end{bmatrix}$\r\n\r\nBackward substitution is like forward substitution, except you start from the bottom row and work your way up:\r\n$U\\widehat{x}= C \\implies \\begin{bmatrix}2 & 1 \\\\ 0 & 1 \\end{bmatrix}\\begin{bmatrix}x \\\\ y \\end{bmatrix}= \\begin{bmatrix}-3 \\\\ 5 \\end{bmatrix}\\implies \\begin{bmatrix}x \\\\ y \\end{bmatrix}= \\begin{bmatrix}-4 \\\\ 5 \\end{bmatrix}$\r\n\r\nYou solve the second problem the same way you solve the first:\r\n$A = LU \\implies \\begin{bmatrix}1 & 2 & 0 \\\\ 2 & 6 &-1 \\\\ 5 &-4 & 10 \\end{bmatrix}= \\begin{bmatrix}1 & 0 & 0 \\\\ 2 & 1 & 0 \\\\ 5 &-7 & 1 \\end{bmatrix}\\begin{bmatrix}1 & 2 & 0 \\\\ 0 & 2 &-1 \\\\ 0 & 0 & 3 \\end{bmatrix}$\r\n\r\nI've leave the rest up to you.", "Solution_4": "I like the way you do it! I'm going to remember it..." } { "Tag": [ "number theory unsolved", "number theory" ], "Problem": "Determine all natural numbers $ n$ and $ k\\geq 1$ such that $ k$ divides each of the numbers\r\n\r\n$ \\binom{n}{1},\\binom{n}{2},...,\\binom{n}{n\\minus{}1}$", "Solution_1": "Let $ p$ is prime divisor of k. Then $ p|\\binom{n}{1}\\equal{}n$. Let $ p^m|n, p^{m\\plus{}1}\\not |n$.\r\nIf $ n\\not \\equal{}p^m$ we consider $ p\\not |\\binom{n}{p^m}$. Therefore $ n\\equal{}p^m$. It give , that k had only one prime divisor p.\r\nIf $ k\\equal{}p^l,l>1$ we consider $ p^l\\not |\\binom{p^m}{p^{m\\minus{}1}}$. It give $ k\\equal{}p,n\\equal{}p^m$." } { "Tag": [ "inequalities", "inequalities proposed" ], "Problem": "[color=purple] Let a,b,c are three positive real numbers satisfy $ a \\plus{} b \\plus{} c \\equal{} 1$. Prove that :\n $ (1 \\plus{} a)\\sqrt {\\frac {1 \\minus{} a}{a}} \\plus{} (1 \\plus{} b)\\sqrt {\\frac {1 \\minus{} b}{b}} \\plus{} (1 \\plus{} c)\\sqrt {\\frac {1 \\plus{} c}{c}}\\ge\\frac {3\\sqrt {3}}{4}.\\frac {(1 \\plus{} x)(1 \\plus{} y)(1 \\plus{} z)}{\\sqrt {(1 \\minus{} x)(1 \\minus{} y)(1 \\minus{} z)}}$\n It's of Walther Janous welcome hungkhtn, arqady, can_hang2007, Vacs, hope you can help me to prove this problem :) [/color]", "Solution_1": "The left expression has 3 variables a, b, c but in the right expression has 3 variables x, y, z.\r\nCan you write exactly the problem?", "Solution_2": "[quote=\"evarist\"][color=purple] Let a,b,c are three positive real numbers satisfy $ a \\plus{} b \\plus{} c \\equal{} 1$. Prove that :\n $ (1 \\plus{} a)\\sqrt {\\frac {1 \\minus{} a}{a}} \\plus{} (1 \\plus{} b)\\sqrt {\\frac {1 \\minus{} b}{b}} \\plus{} (1 \\plus{} c)\\sqrt {\\frac {1 \\plus{} c}{c}}\\ge\\frac {3\\sqrt {3}}{4}.\\frac {(1 \\plus{} x)(1 \\plus{} y)(1 \\plus{} z)}{\\sqrt {(1 \\minus{} x)(1 \\minus{} y)(1 \\minus{} z)}}$\n It's of Walther Janous welcome hungkhtn, arqady, can_hang2007, Vacs, hope you can help me to prove this problem :) [/color][/quote]\r\n\r\nI think correct problem is:\r\n$ (1 \\plus{} a)\\sqrt {\\frac {1 \\minus{} a}{a}} \\plus{} (1 \\plus{} b)\\sqrt {\\frac {1 \\minus{} b}{b}} \\plus{} (1 \\plus{} c)\\sqrt {\\frac {1 \\plus{} c}{c}}\\ge\\frac {3\\sqrt {3}}{4}.\\frac {(1 \\plus{} a)(1 \\plus{} b)(1 \\plus{} c)}{\\sqrt {(1 \\minus{} a)(1 \\minus{} b)(1 \\minus{} c)}}$", "Solution_3": "[quote=\"NguyenDungTN\"][quote=\"evarist\"][color=purple] Let a,b,c are three positive real numbers satisfy $ a \\plus{} b \\plus{} c \\equal{} 1$. Prove that :\n $ (1 \\plus{} a)\\sqrt {\\frac {1 \\minus{} a}{a}} \\plus{} (1 \\plus{} b)\\sqrt {\\frac {1 \\minus{} b}{b}} \\plus{} (1 \\plus{} c)\\sqrt {\\frac {1 \\plus{} c}{c}}\\ge\\frac {3\\sqrt {3}}{4}.\\frac {(1 \\plus{} x)(1 \\plus{} y)(1 \\plus{} z)}{\\sqrt {(1 \\minus{} x)(1 \\minus{} y)(1 \\minus{} z)}}$\n It's of Walther Janous welcome hungkhtn, arqady, can_hang2007, Vacs, hope you can help me to prove this problem :) [/color][/quote]\n\nI think correct problem is:\n$ (1 \\plus{} a)\\sqrt {\\frac {1 \\minus{} a}{a}} \\plus{} (1 \\plus{} b)\\sqrt {\\frac {1 \\minus{} b}{b}} \\plus{} (1 \\plus{} c)\\sqrt {\\frac {1 \\plus{} c}{c}}\\ge\\frac {3\\sqrt {3}}{4}.\\frac {(1 \\plus{} a)(1 \\plus{} b)(1 \\plus{} c)}{\\sqrt {(1 \\minus{} a)(1 \\minus{} b)(1 \\minus{} c)}}$[/quote]\r\n\r\nIt's $ (1 \\plus{} a)\\sqrt {\\frac {1 \\minus{} a}{a}} \\plus{} (1 \\plus{} b)\\sqrt {\\frac {1 \\minus{} b}{b}} \\plus{} (1 \\plus{} c)\\sqrt {\\frac {1 \\plus{} c}{c}}\\ge\\frac {3\\sqrt {3}}{4}.\\frac {(1 \\plus{} a)(1 \\plus{} b)(1 \\plus{} c)}{\\sqrt {(1 \\minus{} a)(1 \\minus{} b)(1 \\minus{} c)}}$\r\n\r\nor $ (1 \\plus{} a)\\sqrt {\\frac {1 \\minus{} a}{a}} \\plus{} (1 \\plus{} b)\\sqrt {\\frac {1 \\minus{} b}{b}} \\plus{} (1 \\plus{} c)\\sqrt {\\frac {1 \\minus{} c}{c}}\\ge\\frac {3\\sqrt {3}}{4}.\\frac {(1 \\plus{} a)(1 \\plus{} b)(1 \\plus{} c)}{\\sqrt {(1 \\minus{} a)(1 \\minus{} b)(1 \\minus{} c)}}$ ?\r\n\r\nI think the second is problem which evarist wants to post :wink:", "Solution_4": "[color=purple] thank you Dung and doduclam the corriction problem must be as you posted. Sorry ervery one :) [/color]" } { "Tag": [ "inequalities unsolved", "inequalities" ], "Problem": "Let $ a,b,c,d$ be reals such that $ a^2\\plus{}b^2\\plus{}c^2\\plus{}d^2\\equal{}1$.\r\n\r\nProve that\r\n\r\n$ \\frac{1}{1\\minus{}ab}\\plus{}\\frac{1}{1\\minus{}ac}\\plus{}\\frac{1}{1\\minus{}ad}\\plus{}\\frac{1}{1\\minus{}bc}\\plus{}\\frac{1}{1\\minus{}bd}\\plus{}\\frac{1}{1\\minus{}cd}\\leq 8$", "Solution_1": "[u]Solution[/u] \r\n\\[ S \\equal{} \\frac {1}{1 \\minus{} ab} \\plus{} \\frac {1}{1 \\minus{} bc} \\plus{} \\frac {1}{1 \\minus{} cd} \\plus{} \\frac {1}{1 \\minus{} ca} \\plus{} \\frac {1}{1 \\minus{} bd} \\plus{} \\frac {1}{1 \\minus{} da}\r\n\\]\r\n\r\n\\[ \\therefore S \\minus{} 6 \\equal{} \\frac {ab}{1 \\minus{} ab} \\plus{} \\frac {bc}{1 \\minus{} bc} \\plus{} \\frac {cd}{1 \\minus{} cd} \\plus{} \\frac {ca}{1 \\minus{} ca} \\plus{} \\frac {bd}{1 \\minus{} bd} \\plus{} \\frac {da}{1 \\minus{} da}\r\n\\]\r\n\r\n\\[ \\equal{} \\sum_{cyc}\\frac {2ab}{2 \\minus{} 2ab}\r\n\\]\r\n\r\n\\[ \\equal{} \\sum_{cyc}\\frac {2ab}{2a^2 \\plus{} 2b^2 \\plus{} 2c^2 \\plus{} 2d^2 \\minus{} 2ab}\r\n\\]\r\n\r\n\\[ \\leq \\sum_{cyc}\\frac {2ab}{a^2 \\plus{} b^2 \\plus{} 2c^2 \\plus{} 2d^2}\r\n\\]\r\n\r\n\\[ \\equal{} \\sum_{cyc}\\frac {2ab}{(a^2 \\plus{} c^2 \\plus{} d^2) \\plus{} (b^2 \\plus{} c^2 \\plus{} d^2)}\r\n\\]\r\n\r\n\\[ \\leq \\frac {1}{2}\\sum_{cyc}(\\frac {a^2}{a^2 \\plus{} c^2 \\plus{} d^2} \\plus{} \\frac {b^2}{b^2 \\plus{} c^2 \\plus{} d^2}) \\equal{} 2\r\n\\]\r\nQED." } { "Tag": [], "Problem": "There is a secret message in this post. First one to guess the secret message gets a prize. \r\n\r\n :D \r\n[hide=\" :D \"] The secret message is: The elephant ate my balloon. [/hide] \r\n :D", "Solution_1": "The elephant ate my balloon.", "Solution_2": "There is a secret message in this post. First one to decode it gets absolutely nothing.\r\n\r\n[color=white]zngu92, gung jnf ernyyl rnfl.[/color]", "Solution_3": "math92, that was really easy.", "Solution_4": "The secret message is \r\nzngu92, gung jnf ernyyl rnfl.\r\n\r\nYay, I get absolutely nothing! :lol: Hey, wait a sec.", "Solution_5": "Haha, it is halfway almost very entertaining. A secret encoded crytopuzzle resides elegantly throughout my e-missive, so solve :P. A great encryption truly, only occasionally. Doesn't really always turn just while done... \r\n\r\n:D", "Solution_6": "to mathnerd314 or watever:\r\nzngu92, gung jnf ernyyl rnfl.", "Solution_7": "alan and ra, you guys are wrong\r\nyour things can be deciphered more :)", "Solution_8": "[quote=\"ra5249\"]to mathnerd314 or watever:\nzngu92, gung jnf ernyyl rnfl.[/quote]\r\n\r\nMath92, that was really easy...\r\n\r\nNobody's solved mine yet :P", "Solution_9": "[hide=\"Solution\"]\nh i i h a v e a s e c r e t m e s s a g e t o o d r a t j w d\nThe conclusion follow immediately. :P \n[/hide]", "Solution_10": "\"Hi, I have a secret message too. Dratjwd.\" What's with the last part?", "Solution_11": "[quote=\"bookaholic\"]\"Hi, I have a secret message too. Dratjwd.\" What's with the last part?[/quote]\r\n\r\nHi, I have a secret message too. Drat. JWD", "Solution_12": "I used to have this one for a signature, though as far as I know, no one has decrypted it. It's easy though if you just see the little trick... two words and you'll find it trivial.\r\n\r\n**N**0**1**7**P**Y**R**C**N**3**Y**7**5**4**N**5**1**H**7**3**3**5**U**N**4**C**", "Solution_13": "[quote=\"Peter VDD\"]**N**0**1**7**P**Y**R**C**N**3**Y**7**5**4**N**5**1**H**7**3**3**5**U**N**4**C**[/quote]\r\n[hide=\"My response to that, which leads to an obvious answer\"]Yes, I can see that nasty encryption.[/hide]", "Solution_14": "Huh, I used to wonder about that, and then I looked at again today, and got it without even looking at hint...", "Solution_15": "Okay, there's a secret message hidden in this post too. Though I kinda stole it from somebody else.\r\n\r\nEDIT: Wait, I forgot to put the actual message in.[hide=\" :blush: \"][color=white][size=9]This is the message.[/size][/color][/hide]\r\n\r\n[color=white]This is not the message. Keep looking.[/color]", "Solution_16": "OK, to miyomiyo, it is \"This is the message\".\r\nAnd to PeterVDD, I still can't figure out ur code... :blush:", "Solution_17": "@ra5429\r\n\r\nthink 1337speak", "Solution_18": "Aargh, I finally get it!\r\n\r\n(For the record: I didn't see the post by sapphyre571)", "Solution_19": "[quote=\"math92\"]There is a secret message in this post. First one to guess the secret message gets a prize. \n\n :D \n[hide=\" :D \"] The secret message is: The elephant ate my balloon. [/hide] \n :D[/quote]\n\nLol, math92. I get it...It took me a whole day, but I get it... :rotfl: \n\nIn case someone wants one: [hide=\" A hint \"]Try to reply with a quote...[/hide]\n\n[hide=\" The answer \"] The elephant ate my balloon. ra5249 wasn't kidding. :wink: [/hide]", "Solution_20": "Uh...good job guys on decoding the post! You guys should all be cryptologists for the CIA.", "Solution_21": "[quote=\"math92\"]Uh...good job guys on decoding the post! You guys should all be cryptologists for the CIA.[/quote]\r\n\r\n:roll:\r\n\r\nFgiweoru bjerm qwivk iowp jmvegf.", "Solution_22": "[quote=\"KrazyNerd\"]ra5249 wasn't kidding.[/quote]Who claimed he was, huh? :huh: \r\n\r\nAs for peter's quote, I had tried leetspeek for some time now, even before seeing the hints, though why I had always started at the front will now remain a mystery. :)", "Solution_23": "[hide=\"decode this\"]\npublic void paint (Graphics g)\n\t{\n\t\tg.setColor(Color.YELLOW);\n\t\tPolygon ears=new Polygon();\n\t\tears.addPoint(33,7);\n\t\tears.addPoint(77,33);\n\t\tears.addPoint(118,32);\n\t\tears.addPoint(174,26);\n\t\tears.addPoint(133,47);\n\t\tears.addPoint(63,45);\n\t\tg.fillPolygon(ears);\n\t\t\n\t\tg.setColor(Color.BLACK);\n\t\tPolygon blackEars=new Polygon();\n\t\tblackEars.addPoint(43,14);\n\t\tblackEars.addPoint(48,28);\n\t\tblackEars.addPoint(34,8);\n\t\tg.fillPolygon(blackEars);\n\t\t\n\t\tPolygon blackEars2=new Polygon();\n\t\tblackEars2.addPoint(161,28);\n\t\tblackEars2.addPoint(174,26);\n\t\tblackEars2.addPoint(155,37);\n\t\tg.fillPolygon(blackEars2);\n\t\t\n\t\t//face\n\t\tg.setColor(Color.YELLOW);\n\t\tg.fillOval(119,41,20,49);\n\t\tg.fillOval(59,40,20,49);\n\t\tg.fillRect(67,40,61,49);\n\t\t\n\t\t//eyes\n\t\tg.setColor(Color.BLACK);\n\t\tg.fillOval(71,56,12,12);\n\t\tg.fillOval(117,55,12,12);\n\t\tg.setColor(Color.WHITE);\n\t\tg.fillOval(73,57,6,6);\n\t\tg.fillOval(119,56,6,6);\n\t\t\n\t\t//cheeks\n\t\tg.setColor(Color.RED);\n\t\tg.fillOval(63,70,12,12);\n\t\tg.fillOval(125,70,12,12);\n\t\t\n\t\tg.setColor(Color.BLACK);\n\t\tPolygon nose=new Polygon();\n\t\tnose.addPoint(100,73);\n\t\tnose.addPoint(97,70);\n\t\tnose.addPoint(103,70);\n\t\tg.fillPolygon(nose);\n\t\t\n\t\t//mouth\n\t\tg.drawArc(82,62,20,20,250,70);\n\t\tg.drawArc(98,62,20,20,220,70);\n\t\t\n\t\t//body\n\t\tg.setColor(Color.YELLOW);\n\t\tg.fillOval(58,87,32,68);\n\t\tg.fillOval(111,87,32,68);\n\t\tg.fillRect(74,87,53,68);\n\t\t\n\t\t//arms\n\t\tg.setColor(Color.BLACK);\n\t\tg.drawLine(75,102,87,129);\n\t\tg.drawLine(87,129,83,128);\n\t\tg.drawLine(83,129,83,131);\n\t\tg.drawLine(83,131,80,129);\n\t\tg.drawLine(80,129,80,134);\n\t\tg.drawLine(80,134,64,122);\n\t\tg.drawLine(121,106,114,127);\n\t\tg.drawLine(114,127,118,128);\n\t\tg.drawLine(118,127,119,130);\n\t\tg.drawLine(119,130,121,129);\n\t\tg.drawLine(121,129,123,131);\n\t\tg.drawLine(123,131,137,113);\n\t\t\n\t\t//tail\n\t\tColor brown=new Color(102,0,0);\n\t\tg.setColor(Color.YELLOW);\n\t\tPolygon tail=new Polygon();\n\t\ttail.addPoint(143,120);\n\t\ttail.addPoint(148,115);\n\t\ttail.addPoint(143,107);\n\t\ttail.addPoint(154,95);\n\t\ttail.addPoint(142,82);\n\t\ttail.addPoint(186,36);\n\t\ttail.addPoint(191,68);\n\t\ttail.addPoint(167,88);\n\t\ttail.addPoint(173,96);\n\t\ttail.addPoint(155,109);\n\t\ttail.addPoint(159,117);\n\t\ttail.addPoint(149,125);\n\t\ttail.addPoint(150,130);\n\t\ttail.addPoint(143,134);\n\t\tg.fillPolygon(tail);\n\t\t\n\t\tg.setColor(brown);\n\t\tPolygon brownTail=new Polygon();\n\t\tbrownTail.addPoint(143,120);\n\t\tbrownTail.addPoint(148,115);\n\t\tbrownTail.addPoint(156,112);\n\t\tbrownTail.addPoint(159,117);\n\t\tbrownTail.addPoint(149,125);\n\t\tbrownTail.addPoint(150,130);\n\t\tbrownTail.addPoint(143,134);\n\t\tg.fillPolygon(brownTail);\n\t\t\n\t\t//feet\n\t\tg.setColor(Color.YELLOW);\n\t\tPolygon leftFoot=new Polygon();\n\t\tleftFoot.addPoint(77,155);\n\t\tleftFoot.addPoint(72,160);\n\t\tleftFoot.addPoint(76,160);\n\t\tleftFoot.addPoint(77,160);\n\t\tleftFoot.addPoint(78,161);\n\t\tleftFoot.addPoint(81,163);\n\t\tleftFoot.addPoint(87,155);\n\t\tg.fillPolygon(leftFoot);\n\t\t\n\t\tPolygon rightFoot=new Polygon();\n\t\trightFoot.addPoint(127,155);\n\t\trightFoot.addPoint(135,159);\n\t\trightFoot.addPoint(131,159);\n\t\trightFoot.addPoint(132,160);\n\t\trightFoot.addPoint(129,160);\n\t\trightFoot.addPoint(128,162);\n\t\trightFoot.addPoint(118,155);\n\t\tg.fillPolygon(rightFoot);\n[/hide]", "Solution_24": "Oh, no...I don't have the program to run that, but I have a bad feeling...\r\n\r\nIt's not a pikachu, is it?", "Solution_25": "[quote=\"bubka\"][quote=\"KrazyNerd\"]ra5249 wasn't kidding.[/quote]Who claimed he was, huh? :huh: \n\nAs for peter's quote, I had tried leetspeek for some time now, even before seeing the hints, though why I had always started at the front will now remain a mystery. :)[/quote]\r\ni don't get it \r\nwhat is the message?", "Solution_26": "[quote=\"KrazyNerd\"]Oh, no...I don't have the program to run that, but I have a bad feeling...\n\nIt's not a pikachu, is it?[/quote]you win", "Solution_27": "@iamagenius: read backwards (obviously).\r\n\r\nI haven't got mathnerd's yet. It is not a Caesar shift, it may be a transposition.", "Solution_28": "[quote=\"bubka\"]I haven't got mathnerd's yet. It is not a Caesar shift, it may be a transposition.[/quote]\r\n\r\nIt's unbreakable.\r\n\r\n[hide]I just banged on the keyboard randomly and made it look convincingly like a code. Sorry about that.[/hide]", "Solution_29": "I have one! No one will find this one!! :D :D", "Solution_30": "[quote=\"laughinghead505\"]I have one! No one will find this one!! :D :D[/quote]Because there is none...", "Solution_31": "[quote=\"laughinghead505\"]I have one! No one will find this one!! :D :D[/quote]Hmm, by trial and error, it's not by clicking the sixth box....", "Solution_32": "bnel aerl aq[] 2l3lkaso?", "Solution_33": "I have a code. \r\nit is:\r\n010000110110000101101110001000000111100101101111011101010010000001110010011001010110000101100100001000000111010001101000011010010111001100111111", "Solution_34": "C 4 N U 5 3 3 7 H 1 5 N 4 5 7 Y 3 N C R Y P 7 1 0 N\r\n\r\ncan you decipher ......... cryption...?\r\n\r\ni don't really get it?", "Solution_35": "Key: 13375P34K.", "Solution_36": "no one can de[code] this message[/code]", "Solution_37": "[quote=\"iamagenius\"]C 4 N U 5 3 3 7 H 1 5 N 4 5 7 Y 3 N C R Y P 7 1 0 N\n\ncan you decipher ......... cryption...?\n\ni don't really get it?[/quote]C4N = can\r\nU = you\r\n533 = see\r\n7H15 = this\r\nN457Y = nasty\r\n3NCRYP710N = encryption\r\n\r\n--> Can you see this nasty encryption?", "Solution_38": "[quote=\"sapphyre571\"]@ra5429\n\nthink 1337speak[/quote]\r\nlol\r\ni just figured it out :blush:", "Solution_39": "[quote=\"hunter34\"]I have a code. \nit is:\n010000110110000101101110001000000111100101101111011101010010000001110010011001010110000101100100001000000111010001101000011010010111001100111111[/quote]\r\n\r\nDid you use [url=http://nickciske.com/tools/binary.php]this site[/url] to get binary from ASCII?\r\n\r\n[hide=\"Answer\"]Can you read this??[/hide]" } { "Tag": [ "superior algebra", "superior algebra solved" ], "Problem": "Hello\r\n\r\nHow can you prove mathematically that a complete bipatite graph\r\ncan only have a even number of edges?\r\n\r\nI can explain this by words or intuition but Im having trouble formulating\r\nthis is methematical forms.\r\n\r\nThanks!\r\nX", "Solution_1": "The complete bipartite graph $B_{1,1}$ has only one edge...", "Solution_2": "great I've found the answer:\r\nhttp://www.artofproblemsolving.com/Forum/viewtopic.php?t=15325", "Solution_3": "[quote=\"eXKoR\"]great I've found the answer:\nhttp://www.artofproblemsolving.com/Forum/viewtopic.php?t=15325[/quote]\r\n\r\nanswer to what?", "Solution_4": "To the problem given there, what he wrote is just nonsense." } { "Tag": [ "geometry", "rectangle", "calculus", "integration", "real analysis", "invariant", "geometric transformation" ], "Problem": "I have no idea where to put this, so I will put this here:\r\n\r\nIf a finite collection of points has an area of 0, and sometimes even an infinite collection of points (i.e. a line) has an area of 0, what justifies that a different infinite collection of points (like a rectangle) to have an area that is not 0?", "Solution_1": "Definition of area. If a rectangle has area=0, then what is area?", "Solution_2": "Infinity is not always the same. It's sort of weird but it's true. Consider this example: The set of rational numbers, $ \\mathbb{Q}$ is obvioulsy infinte, as is the set of irrational numbers. However, the set of irrational numbers is much, much larger. Check this link out. Interesting stuff.\r\n\r\nhttp://en.wikipedia.org/wiki/Countable_sets", "Solution_3": "Good question.\r\n\r\nNotions such as length, area, and volume (known as [u]measure[/u]) are not the same as the notion of [u]cardinality[/u]. They're both notions of \"amount\", but measure typically tells you more about the relative stucture/geometry of a set than it's cardinality.\r\n\r\nSee [url=http://en.wikipedia.org/wiki/Measure_Theory]Measure Theory[/url]. (Note: this is an abstract presentation of the idea of measure - something you wouldn't normally learn about until well into College.)", "Solution_4": "Actually, I was reading this in Apostol (the single variable calculus volume) where he defines area (for defining integration) firstly if a set is \"measurable.\" Perhaps my question would be better satisfied if someone also explained what the important of this is.", "Solution_5": "Here's the importance: You're reading about integration, and integrals (at their basic level) are based on limits of sums of areas. Well... what is the definition of area? This is a complicated question and a very important question.\r\n\r\nThe full blown answer to the question begins with the definition of a measurable set with respect to a measure (usually the Lebesgue measure). It turns out that there are some sets which are not (Lebesgue) measurable - basically meaning that they don't have a well-defined area in any usual sense.\r\n\r\nIf you think that area is an obvious concept that doesn't need definition, ask yourself the question, what exactly does it mean for the area of a circle to be $ \\pi r^2$ [i]square units[/i]?\r\nWhat is the area of the set $ S \\equal{} \\{(x,y) | x$ has no 2's in its decimal representation and $ y$ has at least one 2 in it's decimal representation$ \\} \\cap [0,1]X[0,1]$?", "Solution_6": "I know area is not an obvious concept and hence needs axioms to describe it, which is why I did not have an obvious answer to my own question. :) When I said importance, I meant the importance of measure and being measurable in this definition of area that he is describing, not the importance of defining area.\r\n\r\nAfter reading through those Wikipedia entries, it seems that measure is a $ \\sigma$-algebra, and hence its unions must countable. Thus the union of infinite points with area zero is not zero. Am I right? Thus area must be defined in a different manner...\r\n\r\nActually, someone can explain it to me anyway...", "Solution_7": "If you try to find a measure (volume) with \"expected\" properties (like: a rectangle/cuboid/etc has the canonical volume, it's invariant under rotation and translation, the volume of a disjoint union of the sum of the individual measures) for all sets of $ n$-dimensional space, then you get problems. The probably best known example is the Banach-Tarski-Pardoxon ( http://en.wikipedia.org/wiki/Banach-Tarski ).\r\nThe solution is simple: just don't try to measure everything.\r\nOne still want's to measure \"most\" sets, e.g. all open or closed sets. Then one can have the mentioned properties, finally leading to the Borel- and the Lebesgue-measure (or in more general manner to general measure theory). See http://en.wikipedia.org/wiki/Measure_(mathematics) .\r\nThen this has lots of good properties, and one can build integration theory solely on this (without much more analysis). This kind of integration has much more properties than \"standard\" integration.", "Solution_8": "I will say that right now you shouldn't worry about measurable sets and all that. That will come later when you study Real Analysis. Just be aware for now that there are some crazy subsets of $ \\mathbb{R}$ and $ \\mathbb{R}^2$ that don't have well-defined \"length\" or \"area\".\r\n\r\nSince you're studying integration, here are some not-too-theoretical, but fun, integration questions:\r\n\r\n1. What is the average distance of all the points in a circle from the center?\r\n2. What is the average distance of all the points in a sphere from the center?\r\n3. Prove that the gravity inside a hollow sphere of mass M equals 0 (physics knowledge required).", "Solution_9": "Question 3 is true by the shell theorem and can be shown for electrostatics as well. Another interesting piece on area is the Dirac Delta function. This is a function such that it is infinitesimally thin, yet has an area of 1.\r\n\r\nhttp://en.wikipedia.org/wiki/Dirac_delta_function", "Solution_10": "It isn't a function at all (and thus has no \"area\"). It's more a set of functions, the real limit function has area $ 0$ (or better: should not be considered).", "Solution_11": "[i]\n1. What is the average distance of all the points in a circle from the center?\n2. What is the average distance of all the points in a sphere from the center? [/i]\r\n\r\n[hide]I got 2/3 the radius for the first question and 3/4 the radius for the second question. What I did was look at all the concentric circles or spheres inside the given circle or sphere, multiply each concentric circle or sphere by its radius, add up all these values, and then divide by the area for #1 and the volume for #2.[/hide]", "Solution_12": "The dirac delta is only treated as a function for convenience in physics, it fits in better as an elementary example of a generalised function (or as they are called, a distribution).\r\n\r\nArea is notoriously difficult to define in elementary terms (the best we usually get away with is by defining axiomatically that a unit area exists and then describing the notion of area by a set of its properties), and better left till you're exposed to a rigorous theory of measures.", "Solution_13": "i know this may be somewhat off-topic, but you may be interested in looking up the Koch star. Similar to Gabriel's trumpet, it has the property of finite area with infinite perimeter. \r\n\r\nGabriels trumpet is a solid of revolution generated by rotating the graph of y=1/x about the x axis, and considering it from x=1-infinity. it has finite volume,(of pi if i remember correctly?), while it has infinite surface area.\r\n\r\nUnlike these two objects, Can any object have infinite volume(area), while still having finite surface area(perimeter)?", "Solution_14": "What is the definition of infinite volume/area? It means that at least one of its dimensions must stretch indefinitely (in informal terms). Thus its surface area/perimeter is always...", "Solution_15": "[quote=\"mathking123\"]i know this may be somewhat off-topic, but you may be interested in looking up the Koch star. Similar to Gabriel's trumpet, it has the property of finite area with infinite perimeter. \n\nGabriels trumpet is a solid of revolution generated by rotating the graph of y=1/x about the x axis, and considering it from x=1-infinity. it has finite volume,(of pi if i remember correctly?), while it has infinite surface area.\n\nUnlike these two objects, Can any object have infinite volume(area), while still having finite surface area(perimeter)?[/quote]\r\n\r\n\r\nHow about (and I'm speaking loosely here) rotating the Dirac Delta 'function' about the x-axis? The resulting surface area would be infinite (basically a disc of infinite radius) but the volume would be finite I think.\r\n\r\nBut I'm having trouble thinking of a standard function to fit this description." } { "Tag": [ "blogs", "\\/closed" ], "Problem": "How can you create a second blog?\r\nI believe you could...", "Solution_1": "I am fairly certain that each user can only create one blog. You may, however, change the name of your original blog and then delete all the entries and shouts, so it would be a new blog, sort of.", "Solution_2": "See here:\r\n[url]http://www.artofproblemsolving.com/Forum/weblog.php?w=967[/url]\r\n[url]http://www.artofproblemsolving.com/Forum/weblog.php?w=893[/url]", "Solution_3": "I'm confused - Nerd_of_the_Ages apparently has two avatars.", "Solution_4": "[quote=\"isabella2296\"]I'm confused - Nerd_of_the_Ages apparently has two avatars.[/quote]\r\n\r\nThis was an issue that was dealt with before your time.", "Solution_5": "But this blog seems to have 'orginated' relatively recently...", "Solution_6": "Its a clone", "Solution_7": "[quote=\"#H34N1\"]Its a clone[/quote]\r\n\r\nThanks, I was trying not to give them any clever ideas. :dry:" } { "Tag": [], "Problem": "When two lines are drawn in a plane, they can create four non-overlapping regions. If five straight lines are drawn in the plane, what is the largest number of non-overlapping regions they can create?", "Solution_1": "The formula for maximum intersections of $ n$ lines in a plane is $ 1$ more than the $ n$th triangular number. \r\n\r\nThis formula can be written mathematically as $ \\frac {n(n \\plus{} 1)}{2} \\plus{} 1$.\r\n\r\nSo we plug in for $ n \\equal{} 5$.\r\n\r\n$ \\frac {5(5 \\plus{} 1)}{2} \\plus{} 1$\r\n\r\n$ \\frac {30}{2} \\plus{} 1$\r\n\r\n$ 15 \\plus{} 1$\r\n\r\nThe answer is $ \\boxed{16}$ intersections.", "Solution_2": "[quote=\"Textangle\"]The formula for maximum intersections of $ n$ lines in a plane is $ 1$ more than the $ n$th triangular number. \n...\nThe answer is $ \\boxed{16}$ intersections.[/quote]\r\n\r\nErr... this question isn't asking anything about intersections of lines, it's talking about regions of the plane.\r\n\r\nI'm also curious where you found this formula... please let me know, thanks!\r\n\r\nThe most regions I've been able to cook up is 15 :/ EDIT: and then like 30 seconds later, 16", "Solution_3": "i'm guessing text was solving for regions, but put intersections by mistake (because that's the right answer)", "Solution_4": "Wouldn't the maximum intersection of lines be the same as the number of regions?", "Solution_5": "You revived! :mad: \n\nAnyway, no it isn't. The $n$th triangular number is the maximum number of intersections created by $n$ lines, however, the number of regions created is the $n$th triangular number plus $1$ (as said by Textangle).\n\nEdit: Facepalm.", "Solution_6": "[quote=\"$LaTeX$\"]You revived! :mad: [/quote]\n\nwhat do you mean?", "Solution_7": "Usually, you don't want to post in a thread that hasn't been posted on in three years. Or four months, for that matter..." } { "Tag": [], "Problem": "A huge-room has $ 1000$ doors, each with a lock provided and numbered $ 1$ to $ 1000$. A mad-man enters and at the first round, he unlocks all doors. At the second round, he locks only doors that have numbers the multiples of $ 2$. At the third round, he operates locks of doors which have a multiple of three as their number-he unlocks locked doors and locks doors which are unlocked. And this goes on for long. At his $ 1000^{th}$ round, how many doors are locked?", "Solution_1": "it does not really belong this sub-forum, does it ?", "Solution_2": "Every door changes it's state as many times, as many divisors it has. So, the unlocked doors have odd number of divisors, so they are perfect squares.", "Solution_3": "i'm confused...what happens in round 1000?" } { "Tag": [], "Problem": "Is it bad to take, say, SAT II Physics after only taking physics honors? is it necessary to learn teh AP material? Thanks.", "Solution_1": "i have no idea what [i]your[/i] school's honors physics class's curriculum is, but the SAT II tests are made for testing high school learning, not freshman college learning, which is what the AP tests (are supposed to) do. So no matter what physics class you took, as long as you took a high school course, you should be fine with some reviewing on the SAT IIs.", "Solution_2": "If I were you I would try to get a SAT II Physics review book, either from the library or purchase one, and see how much you know of it. I agree with khan-it depends on the quality of your schools honors program.", "Solution_3": "I cant speak for Physics, but I took first year chemistry at my high school, got an A for the final grade, and I took the SAT IIs in June and got a 760.", "Solution_4": "don't take it unless you are going to study very HARD by yourself, very hard" } { "Tag": [ "function" ], "Problem": "If $f(x+1)=x^2+3x+5$, then find $f(x)$", "Solution_1": "[quote=\"4everwise\"]If $f(x+1)=x^2+3x+5$, then find $f(x)$[/quote]\r\n[hide]\n$f(x)=(x-1)^2+3(x-1)+5$\n\n$=x^2-2x+1+3x-3+5$\n\n$=x^2-x-3$[/hide]\r\n\r\nI hope that's right. :(", "Solution_2": "[quote=\"Drunken_Math\"][hide]\n$f(x)=(x-1)^2+3(x-1)+5$[/hide]\n[/quote]\n\n[hide=\"clarification\"]\nHe just substituted $(x-1)$ for $(x+1)$ to get $f((x-1)+1)=f(x)$\n[/hide]" } { "Tag": [ "articles", "geometry", "geometric transformation", "reflection" ], "Problem": "The cover edition of the latest edition of business week is found [url=http://www.businessweek.com/magazine/content/06_04/b3968001.htm]here[/url]. \r\n\r\nIt's an interesting article talking about more opportunities for math in the world, particularly in business and analyzing data.\r\n\r\nAny comments?", "Solution_1": "I notice the small numbers and the fact that we are finally having sufficient computer power and information available to make this a potential future area of great interest. No market driven company like Ford or P&G can avoid the potential to be on the ground floor for the very small cost it takes to form such a group.\r\n\r\nIt may well be like those modeling the planet's climate -- the model is at least several orders of magnitude too simple to have meaningful results. I'm hopefull, but expect this to be a dead end for at least a while. Some areas, limited in scope and time, will probably work, but there will be large mistakes (model doesn't reflect real market). For example with the test markets the army of MBAs unleached on businesses -- Polaroid almost lost the company when its test market for SX-70 failed to note that some folks who did not live in the test markets bought cameras anyway and the factory was built ten times too large.\r\n\r\nIt'll be great to look back 30 years from now and see how mislead we were in discussing this potential. I'm sure we will be wrong -- I just don't know which way." } { "Tag": [ "floor function", "number theory unsolved", "number theory" ], "Problem": "The sum of $ k$ different even and $ l$ different odd natural numbers equals\r\n1997. Find the maximum possible value of $ 3k + 4l$ .", "Solution_1": "The sum of the odd numbers is at least $k^2$ (the sum of the first $k$ odd numbers), and the sum of the even numbers is at least $l^2+l$ (the sum of the first $l$ even numbers), so $3l+4k\\le3l+4\\lfloor\\sqrt{1997-l^2-l}\\rfloor\\le221$ for $0\\le l\\le44$ ($l<45$ since $45^2+45=2060>1997$).\r\n\r\nThis bound is achievable, taking all the even numbers less than $56$, all the odd numbers less than $67$, and the number $85$ (for instance).", "Solution_2": "[quote=\"Diarmuid\"] $3l+4k\\le3l+4\\lfloor\\sqrt{1997-l^2-l}\\rfloor\\le221$ for $0\\le l\\le44$ ($l<45$ since $45^2+45=2060>1997$).\n\nThis bound is achievable, taking all the even numbers less than $56$, all the odd numbers less than $67$, and the number $85$ (for instance).[/quote]\r\nSo the maximum value is 436?And an another how do you find the number 221?", "Solution_3": "$436$? No, the maximum value is $221$.\r\n\r\nI found it by making a table of the relevant values, and looking (I did it in Excel, but it's small enough that you could make it by hand in an exam). I don't know of any smarter way of doing it.", "Solution_4": "[quote=\"Diarmuid\"]\nThis bound is achievable, taking all the even numbers less than $56$, all the odd numbers less than $67$, and the number $85$ (for instance).[/quote]\r\nHow do you find this?", "Solution_5": "Oops - that should be $54$, not $56$. The set chosen is the first $27$ even numbers, the first $34$ odd numbers, and $85$ as the remainder necessary to make up the sum.\r\n\r\nThe bound I found was that the maximum value of $3k+4l$ was $221$. This can be achieved with $k=27$, $l=35$ (or with $k=23$, $l=38$, but we need an odd $l$ to have an odd sum). So we take the first $27$ even numbers, and the first $35$ odd numbers, and add them up - but this is only $1981$, so we need another $16$ - I got this by adding $16$ to one of the numbers (in particular, $69$)." } { "Tag": [ "ratio", "geometry", "trigonometry", "area of a triangle" ], "Problem": "Triangle ABC is equilateral Extend $ AC$ through $ C$ to a point $ D$ such that $ AC \\equal{} DC$. Let $ E$ be the midpoint of $ BC$. Draw the segment from $ D$ to $ E$. Extend $ DE$ to hit side $ AB$ at $ F$. Find the ratio of $ BF$ to $ FA$.", "Solution_1": "[hide]\nFirst, just set a side of $ ABC$ to be 2 (for the sake of ease) and $ BF$ to be $ x$.\nNow, we will find the ratio of the area of $ AFD$ to $ AED$ in two different ways.\n\nFirst, the altitude from F to AD has length $ \\frac{(2\\minus{}x)\\sqrt{3}}{2}$ since it is the longer leg of a 30-60-90 triangle.\nThe altitude from E to AD is $ \\frac{\\sqrt{3}}{2}$ since it is the shorter leg of a 30-60-90 triangle.\nNow, since $ AFD$ and $ AED$ have the same base of 4, the ratio of the areas is just $ 2\\minus{}x$.\n\nNext, a different way to find this ratio:\n$ [AFE] \\equal{} \\frac{2\\minus{}x}{4}[AED]$ so $ \\frac{[AFD]}{[AED]} \\equal{} \\frac{6\\minus{}x}{4}$.\n\nEquating the 2 ratios and solving, we have $ x \\equal{} \\frac{2}{3}$ so $ \\frac{BF}{FA} \\equal{} \\frac{1}{2}$.[/hide]\r\n\r\nPretty complicated for a MC problem...", "Solution_2": "[quote=\"alkjash\"]\n\nNext, a different way to find this ratio:\n$ [AFE] \\equal{} \\frac {2 \\minus{} x}{4}[AED]$ so $ \\frac {[AFD]}{[AED]} \\equal{} \\frac {6 \\minus{} x}{4}$.\n\nPretty complicated for a MC problem...[/quote]\r\n\r\nI think you should probably explain this. Or I will =)... So $ \\angle EAD\\equal{}\\angle EAB$. Since the area of a triangle can be computed by $ \\frac{1}{2}ab\\sin\\alpha$, the ratio of the two areas is simply the ratio of the two corresponding sides. Since $ AE$ is a common side, we simply need $ \\frac{AB}{AD}\\equal{}\\frac{2\\minus{}x}{4}$.\r\n\r\nNice solution by the way. I made the problem up xD. Use menelaus if you know it. Gives you the answer in one step.", "Solution_3": "Ahh... right looked something like that..\r\nHaven't gotten around to memorizing Menelaus. :blush: \r\nDoesn't have as nice a pattern as Ceva, you know?", "Solution_4": "Ceva's is found using menelaus :wink:", "Solution_5": "Ummm...\r\nNot necessarily...\r\nAt least the way I saw it proved..." } { "Tag": [ "function", "inequalities", "algebra unsolved", "algebra" ], "Problem": "Let $f$ be a function from $R$ to $R$ sastisfying $\\exists f''(x)\\ \\ \\ \\forall x\\in R$\r\nAssume that : $|f(x)|\\le a$ \r\n$|f'(x)|\\le b$\r\n$|f''(x)|\\le c$\r\nWhere $a,b,c$ are constant \r\nProve the inequality : $b\\le \\sqrt{2ac}$", "Solution_1": "http://www.mathlinks.ro/Forum/viewtopic.php?t=44225" } { "Tag": [ "inequalities unsolved", "inequalities" ], "Problem": "prove that in a triangle we have\r\nsinA*sinB+sinB*sinC+sinC*sinA<=cos(A/2)*cos(B/2)+cos(B/2)*cos(C/2)+cos(C/2)*cos(A/2);\r\nDo you have a strongert ineq.?", "Solution_1": "$2\\sum{sinAsinB}=\\sum(cosC+cos(A-B))$\r\n$2\\sum{cos(A/2)cos(B/2)}=\\sum(sin(C/2)+cos\\frac{A-B}{2})$\r\nBut $\\sum cosC \\le \\sum sin(C/2)$\r\n$\\sum cos(A-B) \\le \\sum cos\\frac{A-B}{2}$\r\n\r\nAm I wrong?" } { "Tag": [ "limit", "calculus", "calculus computations" ], "Problem": "if $ A_n$ is not = 0 and $ {[A_n]}^{\\infty}_1$ Converges $ \\lim_{n \\rightarrow \\infty} \\frac{A_{n\\plus{}1}}{A_n} \\equal{} L \\Rightarrow |L| \\leq 1$\r\nhow can i prove it", "Solution_1": "Compare it with some geometric series." } { "Tag": [ "function", "trigonometry", "limit", "integration", "calculus", "calculus computations" ], "Problem": "Let $f,g\\;: \\mathbb R\\rightarrow\\mathbb R$ with $g(x)=\\sqrt{\\{x\\}-\\{x\\}^2}\\qquad$ and $\\qquad f(x)=\\arccos(\\cos x)$; $\\{x\\}$ is the usual fractional part of x.\r\nI) Prove that the two functions are integrable in every conpact set of $\\mathbb R$\r\nII) Compute\r\n\\[ \\lim_{x\\to \\infty}\\frac{\\int_0^xf(t)\\,dt}{\\int_0^xg(t)\\,dt} \\]", "Solution_1": "You can easily compute the upper integral for $x=k\\pi$, and the lower one for $x$ positive integer. Since the numerator goes to infinity as $x \\rightarrow + \\infty$, you can compute the quotient and you'll find $4$." } { "Tag": [ "function", "topology" ], "Problem": "Define a function f from the interval I = [0, 2\u03c0) of R to R2 by setting f(t) = (cos t, sin t).\r\nShow that the inverse function from f(I) to I is not continuous.", "Solution_1": "f : I -> R^2 has image f(I) = S^1 which is a compact subset of R^2; but the inverse map from f(I) to R maps S^1 to [0, 2 \\pi ) which is not compact in R. therefore as the inverse image of a compact set is not compact, f inverse is not continuous. :maybe:", "Solution_2": "[quote=\"buddhu\"]f : I -> R^2 has image f(I) = S^1 which is a compact subset of R^2; but the inverse map from f(I) to R maps S^1 to [0, 2 \\pi ) which is not compact in R. therefore as the inverse image of a compact set is not compact, f inverse is not continuous. :maybe:[/quote]\r\n\r\nThis works in this case since we are showing that the [i]inverse[/i] map is not continuous, but I just want to note the following: the continuous image of a compact set is compact (this is the result we used to show $ f^{\\minus{}1}$ is not continuous), but the preimage of a compact set under a continuous map is not necessarily compact. If the preimage of every compact set under a continuous map is always compact, then the map is called a proper map.\r\n\r\nIn other words, this exercise asks to show that $ f$ is not a proper map. I think buddhu understood this already, but I just wanted to make sure diophantine interprets the solution correctly." } { "Tag": [ "function", "floor function", "trigonometry", "ceiling function", "logarithms", "calculus", "integration" ], "Problem": "Alright, so I'm looking for a way to rewrite the floor function as $x-$ (some composition with trigonometric functions)\r\n\r\nOne idea I have is using Arcsin(sin(x)) or something of that nature somehow. Any help in constructing an equivilent to [x] would be appreciated :)", "Solution_1": "What's the problem with the current definition?", "Solution_2": "Nothing, I just want to see if floor can be written using trigonometric functions.", "Solution_3": "Well you're not going to be able to write it with continuous functions or any composition of such, because it's discontinuous at infinitely many points.", "Solution_4": "Alright, so I've made a bit of progress, but now I'm stuck:\r\n\r\nSo far I have:\r\n\r\n$\\lfloor x\\rfloor=x-\\sqrt{\\left(\\frac{\\sin^{-1}(\\cos (\\pi x))}{\\pi}-\\frac{\\sin^{-1}(\\sin (\\pi x))}{2\\sqrt{(\\sin^{-1}(\\sin (\\pi x)))^{2}}}\\right)^{2}}$\r\n\r\nHere's the thing: it works for all non-integers, but whenever I put in an integer, it encounters an indeterminate from the $\\sin (\\pi x)$.\r\n\r\nThe only reason that whole thing is there is to take care of the fact that the $\\frac12$ is either positive or negative depending on the value of $\\sin (\\pi x)$...", "Solution_5": "I was wondering if it was possible to define the ceiling function in terms of $\\lfloor x\\rfloor$ and $[x]$, since I don't think there's a ceiling function on my graphing calculator.", "Solution_6": "[quote=\"lotrgreengrapes7926\"]I was wondering if it was possible to define the ceiling function in terms of $\\lfloor x\\rfloor$ and $[x]$, since I don't think there's a ceiling function on my graphing calculator.[/quote]\r\n$\\lceil x\\rceil =-\\lfloor-x \\rfloor$\r\nI like to imagine it as a composition of reflections.", "Solution_7": "I'm not sure I understand exactly what you want. Can you tell me what's wrong with simply $\\lfloor x \\rfloor=x-\\frac{sin^{-1}(sin(2\\pi x))}{2\\pi}$", "Solution_8": "Because its not, since Arcsin(sin(x)) still increases and decreases.\r\n\r\nSo your suggestion would resemble the attached:", "Solution_9": "$\\frac{\\tan \\pi x/2 }{|\\tan \\pi x/2 |}= (-1)^{\\lfloor x \\rfloor}$, and then you just have to take a logarithm base $-1$ ... :P \r\n\r\n[quote]...more \"accurately\" defined functions. [/quote]\r\n\r\nThere is no sense in which $\\sin$, $\\sin^{-1}$ or any other trigonometric function is \"more accurately defined\" then the plain old floor function. (There is still, of course, the interest in doing something for the sake of doing it.)", "Solution_10": "I guess its more of that, just trying to do it for the sake of doing it.\r\n\r\nSee, the thing about your tan thing is, tangent function becomes zero at some integral values, so you'd get an indeterminate (which is what's wrong with\r\n$\\lfloor x\\rfloor=x-\\sqrt{\\left(\\frac{\\sin^{-1}(\\cos (\\pi x))}{\\pi}-\\frac{\\sin^{-1}(\\sin (\\pi x))}{2\\sqrt{(\\sin^{-1}(\\sin (\\pi x)))^{2}}}\\right)^{2}}$\r\nas well...", "Solution_11": "Do you just want a [i]finite[/i] function? Because $x-\\lfloor x \\rfloor$ has a very simple Fourier series.", "Solution_12": "[quote=\"lotrgreengrapes7926\"]I was wondering if it was possible to define the ceiling function in terms of $\\lfloor x\\rfloor$ and $[x]$, since I don't think there's a ceiling function on my graphing calculator.[/quote]\r\n$\\lceil x\\rceil=\\lfloor x\\rfloor+1-\\left\\lfloor 1-x+\\lfloor x\\rfloor\\right\\rfloor$.", "Solution_13": "[quote=\"rnwang2\"][quote=\"lotrgreengrapes7926\"]I was wondering if it was possible to define the ceiling function in terms of $\\lfloor x\\rfloor$ and $[x]$, since I don't think there's a ceiling function on my graphing calculator.[/quote]\n$\\lceil x\\rceil=\\lfloor x\\rfloor+1-\\left\\lfloor 1-x+\\lfloor x\\rfloor\\right\\rfloor$.[/quote]\n\nBictor's answer\n\n[quote=\"Bictor717\"] $\\lceil x \\rceil =-\\lfloor-x \\rfloor$ [/quote]\r\n\r\nseems a little nicer to me. :P", "Solution_14": "[quote=\"t0rajir0u\"][quote=\"rnwang2\"][quote=\"lotrgreengrapes7926\"]I was wondering if it was possible to define the ceiling function in terms of $\\lfloor x\\rfloor$ and $[x]$, since I don't think there's a ceiling function on my graphing calculator.[/quote]\n$\\lceil x\\rceil=\\lfloor x\\rfloor+1-\\left\\lfloor 1-x+\\lfloor x\\rfloor\\right\\rfloor$.[/quote]\n\nBictor's answer\n\n[quote=\"Bictor717\"] $\\lceil x \\rceil =-\\lfloor-x \\rfloor$ [/quote]\n\nseems a little nicer to me. :P[/quote]\r\n :rotfl:" } { "Tag": [ "geometry", "circumcircle", "incenter", "Euler", "power of a point", "radical axis", "geometry unsolved" ], "Problem": "Let P be the intersection of the diagonals AC and BD of the convex quadrilateral ABCD for which AB=CA=BD.\r\nLet O and I be the circumcenter and incenter of the triangle ABP.\r\nProve that if $ O \\neq I$, then $ OI \\perp CD$.\r\n :help: please~", "Solution_1": "If $ a \\equal{} BP, b \\equal{} PA, p \\equal{} AB$ are side lengths of the $ \\triangle ABP,$ $ \\overline{CP} \\equal{} p \\minus{} b, \\overline{DP} \\equal{} p \\minus{} a.$ External bisectors of the angles $ \\angle PAB, \\angle ABP$ cut $ BP, AP$ at $ X, Y$ and $ \\frac {\\overline{XP}}{\\overline{XB}} \\equal{} \\frac {b}{p},$ $ \\frac {\\overline{YP}}{\\overline{YA}} \\equal{} \\frac {a}{p}$ $ \\Longrightarrow$ $ \\overline{XP} \\equal{} \\frac {ab}{p \\minus{} b},$ $ \\overline{YP} \\equal{} \\frac {ab}{p \\minus{} a}$ $ \\Longrightarrow$ $ \\frac {\\overline{DP}}{\\overline{CP}} \\equal{} \\frac {p \\minus{} a}{p \\minus{} b} \\equal{} \\frac {\\overline{XP}}{\\overline{YP}}$ $ \\Longrightarrow$ $ YX \\parallel CD.$\r\n\r\nIf $ I_a, I_b, I_p$ are excenters of the $ \\triangle ABP,$ $ A, B, P$ are the altitude feet of the excentral $ \\triangle I_aI_bI_p,$ $ I$ is its orthocenter, $ (O)$ its 9-point circle, $ OI$ its Euler line. The circumcenter $ O'$ of the $ \\triangle I_aI_bI_p$ is on $ OI.$ $ API_aI_p$ and $ BPI_bI_p$ are cyclic because of the right angles $ \\angle I_pAI_a, \\angle I_pPI_a$ and $ \\angle I_pBI_b, \\angle I_pPI_b$ $ \\Longrightarrow$ $ \\overline {YP} \\cdot \\overline {YA} \\equal{} \\overline {YI_a} \\cdot \\overline {YI_p}$ and $ \\overline {XP} \\cdot \\overline {XB} \\equal{} \\overline {XI_b} \\cdot \\overline {XI_p}$ $ \\Longrightarrow$ $ YX$ is the radical axis of $ (O), (O')$ perpendicular to the center line $ O'O$ identical with $ OI$ $ \\Longrightarrow$ $ CD \\perp OI.$", "Solution_2": "Thanks a lot ~ I am happy for your solution.\r\n:coolspeak:" } { "Tag": [ "MATHCOUNTS", "AMC", "USA(J)MO", "USAMO" ], "Problem": "This is the thread that mock Mathcounts that I'll make would be posted.\r\n\r\nI'm not quite confident whether I'll be able to hold any timed contest (since I have other stuff to do), here is what I'll do.\r\n\r\nI'll post the questions on PDF.\r\n\r\nAfter a day or two, I'll post the solution packet on another post (PDF also).\r\n\r\nI'll probably include all sprint, target, team, countdown (I'm thinking of around 40 questions? Is that sound okay? :) ), and if I have good idea, maybe a master round (I have All-Time Greatest Mathcounts Problems so I might be able to know what they require in there) and like always, those solutions are what I think, not what you should have. :lol: There are always more than one way to solve the problems.\r\n\r\nBefore the answer packet is posted, you can discuss the questions or whatever you prefer to do.\r\n\r\nIf you're okay or happy with this idea, please post here. \r\n\r\nThanks and Good luck on your National MC.\r\n\r\nSilverfalcon", "Solution_1": "Hmm, this sounds like fun. :D I need practice so I can even GO to mathcounts next year.", "Solution_2": "cool!", "Solution_3": "Should we follow regular time rules (40 min sprint 24 target 20 team)?", "Solution_4": "It depends on you.\r\n\r\nIf you want to be real, do it that way.\r\n\r\nIf you just want to see how it goes, do whatever you prefer.", "Solution_5": "I think this would be great! Old national problems are so hard to access . . . I need more problems to practice on . . ..\r\n\r\nAround what difficulty will this test be?", "Solution_6": "sounds good!", "Solution_7": "i'm in", "Solution_8": "I'll do it", "Solution_9": "I'm in.", "Solution_10": "Okay.", "Solution_11": "[quote=\"DarkKnight\"]Hmm, this sounds like fun. :D I need practice so I can even GO to mathcounts next year.[/quote]\r\n\r\nAren't you in high school?", "Solution_12": "I thought he was in 6th grade--I could be wrong.", "Solution_13": "I'll be in it\r\n\r\nKaran Batra", "Solution_14": "[quote=\"Max300\"][quote=\"DarkKnight\"]Hmm, this sounds like fun. :D I need practice so I can even GO to mathcounts next year.[/quote]\n\nAren't you in high school?[/quote]\r\n\r\nUhh? What? I'm in 6th grade. How'd you get the idea I was in high school?", "Solution_15": "[quote=\"Max300\"][quote=\"DarkKnight\"]Hmm, this sounds like fun. :D I need practice so I can even GO to mathcounts next year.[/quote]\n\nAren't you in high school?[/quote]\r\n\r\nlol, akash, i think you have the wrong andrew...", "Solution_16": "[quote=\"DarkKnight\"][quote=\"Max300\"][quote=\"DarkKnight\"]Hmm, this sounds like fun. :D I need practice so I can even GO to mathcounts next year.[/quote]\n\nAren't you in high school?[/quote]\n\nUhh? What? I'm in 6th grade. How'd you get the idea I was in high school?[/quote]\r\n\r\nthere was a guy who got 5th in state last year named Andrew Wang (at least in texas)", "Solution_17": "Yeah Andrew qualified for the USAMO.", "Solution_18": "[quote=\"tarquin\"]Yeah Andrew qualified for the USAMO.[/quote]\r\n\r\nYeah. I was looking at the USAMO qualifiers and that guy had the same name as me. :P I'm only in 6th grade.", "Solution_19": "i guess it's an easy mix up, but last year, andrew and mark were the two to beat all year in mathcounts, they both went to sartartia and always got like 1 and 2 with a 46 and 45, now mark moved to fort settlement and has dennis on his team instead...lol, hard to imagine mark zhang getting a non-perfect in mathcounts" } { "Tag": [], "Problem": "Posts related to Inorganic and Analytical Chemistry.", "Solution_1": "[url=http://www.mathlinks.ro/viewtopic.php?p=999973#999973]identify A,B...E[/url]" } { "Tag": [ "geometry", "3D geometry", "FTW" ], "Problem": "The surface of a $ 4'' \\times 5'' \\times 6''$ block is painted yellow. If the block is separated into one-inch cubes, what is the number of cubes with exactly one yellow face?", "Solution_1": "Uhh.... isnt this one just (2*3+3*4+2*4)*2=52????\r\n\r\nThe answer on FTW was 26, and i don't see what i did wrong.", "Solution_2": "I don't see what you did wrong either. Maybe the answer-typer forgot to multiply by 2?", "Solution_3": "That's probably it.", "Solution_4": "I just reported it.", "Solution_5": "It is still not fixed.", "Solution_6": "It has been fixed. " } { "Tag": [ "inequalities", "puzzles" ], "Problem": "Determine the maximum value that is obtained by multiplying together a set of positive integers which are all different and whose sum is 70.", "Solution_1": "[hide=\" I think it helps..\"]AM - GM gives us for the set of positive integers $ \\{a_1, a_2, ...,a_n\\}$\n\n$ \\frac{a_1 \\plus{} a_2 \\plus{} ... \\plus{} a_n}{n} \\geq \\sqrt[n]{a_1a_2...a_n}$\n\n$ \\frac{70}{n} \\geq \\sqrt[n] {a_1a_2...a_n}$\n\n$ \\left( \\frac{70}{n} \\right) ^n \\geq a_1a_2...a_n$\n\nMaximum occurs when there is equality. So now the problem is just how to optimize $ \\left( \\frac{70}{n} \\right) ^n$.[/hide]", "Solution_2": "[hide=\"comment\"]Nice methodology. However in terms of the given conditions the set of positive integers must be all different, and thus a(1), a(2),...., a(n) cannot be equal.[/hide]", "Solution_3": "Solution:\r\n[hide]Mark the numbers we choose for the set like this:\n_, 2, 3, 4, 5, _, _, _, 9, 10, 11, 12, _, 14\n\nThen if we ever have a blank after a number (like the 5) and then a different blank before a bigger number (like the 9), we can move those two numbers closer together (in this example changing them to 6 and 8). This will increase the product by AM/GM or Jensen's or whatever inequality you want to use.\n\nWe can perform this operation whenever there are two different blanks somewhere inside our set of numbers.\n\nThe set with maximal product, therefore, can have at most one blank inside it, so it must be a sequence of consecutive numbers with one of the numbers missing.\n\nAlso our set must have at least one of 2 and 3; otherwise we could just put 2 into the set and decrease the lowest odd number by 2, which increases the product. So, the string of consecutive numbers starts with either a 2 or a 3.\n\nFrom there getting the solution is easy:\n2, 3, 4, 5, 6, _, 8, 9, 10, 11, 12[/hide]", "Solution_4": "The AM-GM approach would still work if done in cases. If we want $ k$ numbers that multiply to a maximal value, we can make them as close as possible to $ \\frac{70}{k}$ while still being distinct. I'll have to check the cases on that though...", "Solution_5": "[quote=\"cyberspace\"]If we want $ k$ numbers that multiply to a maximal value, we can make them as close as possible to $ \\frac {70}{k}$ while still being distinct.[/quote]\r\nIn that case, the obvious thing to do is to assume that the numbers are consecutive. Suppose the numbers are $ n,\\,n \\plus{} 1,\\,n \\plus{} 2,\\,\\ldots,\\,n \\plus{} k \\minus{} 1.$ The sum of these numbers is\r\n\r\n[list]$ \\frac {k(n \\plus{} n \\plus{} k \\minus{} 1)}2\\ \\equal{} \\ 70$[/list]\ngiving $ k(2n \\plus{} k \\minus{} 1) \\equal{} 140.$ So $ k$ is a divisor of 140. Combining this with the given conditions, we find that the following are the only possibilities:\n\n[list]$ k \\equal{} 4,\\ n \\equal{} 16 \\\\\n[2mm] k \\equal{} 5,\\ n \\equal{} 12 \\\\\n[2mm] k \\equal{} 7,\\ n \\equal{} 7$[/list]\nHence the maximum product for a selection of consecutive integers is\n\n[list]$ 7\\times8\\times9\\times10\\times11\\times12\\times13 \\equal{} 8\\,648\\,640.$[/list]\r\n\r\nSo, are there non-consecutive integers whose product is higher than this (or can we prove that there aren\u2019t)? ;)" } { "Tag": [ "inequalities", "inequalities unsolved" ], "Problem": "Where x,y,z are positive reals, show that $x^{2}(x-y)(x-2y)+y^{2}(y-z)(y-2z)+z^{2}(z-x)(z-2x)\\geq 0$\r\n\r\nCan this be generalised Schur-style to $x^{r}(x-y)(x-2y)+y^{r}(y-z)(y-2z)+z^{r}(z-x)(z-2x)\\geq 0$?\r\n\r\nSorry if either of these questions is obvious, I have only just started learning inequalities properly.", "Solution_1": "the first ineq is a very strong ineq proposed by vasc.", "Solution_2": "Yeah sorry, I should have said the source of the first is Vasc (via the Hojoo Lee sheet). The second is my own hypothesis based on the first.\r\n\r\nAnyone got a proof?", "Solution_3": "http://www.mathlinks.ro/Forum/viewtopic.php?p=251877#p251877\r\nThere are three proofs of the first ineq there, one by Namdung, one by Vasc, oen by Stefan V. (posted by darij)", "Solution_4": "For the record, the generalisation is false. :(" } { "Tag": [ "AMC" ], "Problem": "1. Find positive integers k, n such that kn! = (((3!)!)!/3! and n is as large as possible. \n\n\n\n\n\nI got this off Kalva, but I think this question is bugged....\n\n\n\nAnswer and my solution in spoiler:\n\n\n\n[hide]\n\nI don't think Kalva had this problem correct.\n\n\n\n(((3!)!)!)/3!=720!/3!... So the largest n would be 720 while k would be 6.... I dunno where I went wrong.\n\n[/hide]", "Solution_1": "[quote=\"Ragingg\"]1. Find positive integers k, n such that kn! = (((3!)!)!/3! and n is as large as possible. \n[/quote]\r\n\r\nk*n!=720!/6\r\nk*n!=120*719!\r\n\r\nSo:\r\nk=120\r\nn=719", "Solution_2": "Erm.\r\n\r\n720!*6 is NOT equal to 720!/6.", "Solution_3": "Oh... Hmm I read the problem wrong. Woops! Wow... That was a stupid mistake. :oops:", "Solution_4": "The question is indeed bugged. The original question (I hope AMC doesn't get mad at me for copyright issues) was\r\n1. Given that ((3!)!)!/(3!)=k*n!, where k and n are positive integers and n is as large as possible, find k+n.", "Solution_5": "[quote=\"captcha000\"]The question is indeed bugged. The original question (I hope AMC doesn't get mad at me for copyright issues) was\n1. Given that ((3!)!)!/(3!)=k*n!, where k and n are positive integers and n is as large as possible, find k+n.[/quote]\r\n\r\nI don't think the question was bugged; he just misinterpreted it for a second.", "Solution_6": "Yeah theres nothing wrong with the question.. thats exactly the same thing that you posted captcha :)" } { "Tag": [], "Problem": "What is the largest possible product of a set of positive integers whose sum is 20?", "Solution_1": "[quote=\"happyme\"]What is the largest possible product of a set of positive integers whose sum is 20?[/quote]\r\n\r\nEdit: hmm... \r\nMoldytape pointed that out...\r\nI should think about it more ...", "Solution_2": "[hide]Actually, 2^7x3^2=1152 is bigger and 2x7+3x2=20[/hide]", "Solution_3": "[hide]\n$3^{6}\\cdot2=1458$[/hide]", "Solution_4": "[hide]3s optimize, so put in as many 3s as possible (note that 2 2s is preferable to a 3 and a 1).[/hide]", "Solution_5": "bpms and DarkKnight are both right, while you can get 2^10 and 4^5, they both equal 1024. So 3^6*2=1458. :D", "Solution_6": "[quote=\"happyme\"]What is the largest possible product of a set of positive integers whose sum is 20?[/quote]\r\n\r\nCan you please be a bit more specific on 'set'?" } { "Tag": [], "Problem": "How many integer solutions does $ xy \\plus{} 9(x \\plus{} y) \\equal{} 2006$ have?", "Solution_1": "Rewrite as $ (x \\plus{} 9)(y \\plus{} 9) \\equal{} 1925 \\equal{} 7*11*25$.\r\n\r\n$ x \\plus{} 9$ has to be one of the positive or negative factors of this number. Since there are 8 positive factors, there are 8 negative factors, making 16 solutions.", "Solution_2": "[quote=\"rofler\"]Rewrite as $ (x \\plus{} 9)(y \\plus{} 9) \\equal{} 1925 \\equal{} 7*11*25$.\n\n$ x \\plus{} 9$ has to be one of the positive or negative factors of this number. Since there are 8 positive factors, there are 8 negative factors, making 16 solutions.[/quote] You have to add $ 81$ not subtract it. So,$ (x \\plus{} 9)(y \\plus{} 9) \\equal{} 2087$\r\nAnd $ 2087$ is a prime. Which implies that $ 2$ positive and $ 2$ negative solutions and $ 4$ solutions in total.\r\nMoreover,I think that $ 1925 \\equal{} 7.11.5^2$ has $ 12$ positive divisors. BTW nice try. :)" } { "Tag": [ "factorial" ], "Problem": "Find the smallest integer between 100 and 999 which is equal to the sum of the factorials of its digits.", "Solution_1": "[hide]I used guess and check and I got 145.[/hide]", "Solution_2": "If only I can simplify this: $100a+10b+c=a!+b!+c!$....., which is kind of hard...\r\n\r\nP.S. Robinhe, how did you just guess and check?", "Solution_3": "Well, I knew 5! was 120, so I thought that the number might be close to 120. So, I tried 125, 135, and I came to 145, which is my answer. I just got lucky.", "Solution_4": "I think what Robinhe did is what I did. You want the lowest three digit one, and to get into the hundreds you need a 5, which would best go in the ones place to make it smallest. Since 5! is 120, and you need two more digits before the five, you try 1_5 and it works once you get to 145. Does this make sense? It's not the most elegant solution. Any better ideas anyone?\r\n[b]EDIT[/b] Beaten to it yet again.", "Solution_5": "Okay I understand, but still there should be a way to simplify it enough to deduce an answer....", "Solution_6": "If you start with 5 as your highest digit, you can make a table of factorial values to help you find an answer. \r\n\r\n5!+1!+2!=123\r\n5!+1!+3!=127\r\n...\r\n\r\nTHere's only like 10 possibilities." } { "Tag": [ "real analysis", "function", "integration", "limit", "advanced fields", "advanced fields unsolved" ], "Problem": "I am just learning about distributions on my own for an interesting problem of how generally I may apply some idea. Anyway, my personal story is not of much importance to this question. \r\n\r\nIn all that follows I am thinking of tempered distributions in particular. \r\n\r\nI have just read that we cannot in general define pointwise product or convolution for distributions. In fact we can define it when one of the distributions has compact support. For example, the Dirac distribution is the identity operator for convolution, so why shouldn't that hold true when we convolve it with a distribution instead of a function? From this it follows we can only define pointwise product for distributions if the Fourier transform of one of them has compact support. \r\n\r\nI am told that Schwartz proved this ages ago. But I don't really see why it is so. What problem do we encounter? Thanks...", "Solution_1": "[quote=\"Xevarion\"]What problem do we encounter? Thanks...[/quote]\r\nI give you two versions of the same problem.\r\n\r\n1. Consider the Dirac delta, $\\delta.$ What is the pointwise product of $\\delta$ with itself?\r\n\r\nIf $f$ is a smooth function, then the pointwise product $f\\cdot \\delta$ is simply $f(0)\\delta.$ But since $\\delta(0)$ makes no sense, we clearly can't do that.\r\n\r\n2. Let $1$ be the function on $\\mathbb{R}$ whose constant value is $1.$ How can you possibly define $1*1?$\r\n\r\nYou can define convolution when one of the distributions is compactly supported - and that makes $\\delta$ the identity element for convolution even with distributions. Actually, with tempered distributions, you can go a little weaker than compactly supported, if you can make some sense out of what \"vanishes rapidly at infinity\" should mean for a distribution.\r\n\r\nYou can define pointwise multiplications when one of the distributions is repesented by a (tempered) smooth function - and that makes $1$ the identity element for pointwise multiplication.", "Solution_2": "Thanks, Professor Merryfield. \r\n\r\nOkay, so I guess the idea is that if I apply a distribution to a test function, it is no longer necessarily a test function. So then if we try to apply the second distribution, it doesn't make sense. \r\n\r\nOk, but how about this? Suppose we make a sequence of functions (nice functions, maybe $L^{2}$) that converges to the distribution. Then we can do stuff like pointwise products or convolutions with impunity. So what happens when we look at the limit of the pointwise product, inner product-ed with a test function? If the pointwise product makes sense for the distribution, will this converge to what we want, but when it doesn't, there is no limit?", "Solution_3": "Sure, let's do that. The only kind of limits that make sense are weak limits - that is, limits in the sense of distributions.\r\n\r\nLet $\\varphi$ be a test function such that $\\int\\varphi=1$ and $\\varphi(0)=1.$ We should be able to manufacture something like that. $\\varphi*\\varphi=\\psi$ is also a test function with $\\int\\psi=1.$\r\n\r\nNow let $f_{n}(x)=n\\varphi(nx)$ and $g_{n}(x)=\\varphi\\left(\\frac{x}{n}\\right).$ Take the limits: $f_{n}\\to\\delta$ and $g_{n}\\to1.$\r\n\r\nThe pointwise product $f_{n}\\cdot f_{n}$ is the function $n^{2}\\varphi^{2}(nx).$ If $\\eta$ is any test function such that $\\eta(0)\\ne0,$ then \r\n\r\n$\\lim\\int n^{2}\\varphi^{2}(nx)\\eta(x)\\,dx=\\lim n\\int\\varphi^{2}(u) \\eta(u/n)\\,du$ does not exist.\r\n\r\nThus collapses our effort to use such limits to find $\\delta\\cdot\\delta.$\r\n\r\nWe can compute that $g_{n}*g_{n}(x)=n\\psi(x/n).$ That is not going to have a weak limit, which collapses our effort to use such limits to find $1*1.$\r\n\r\nThat's the easy part. The question you're really asking is this: had those limits existed, would they be well-defined? Let me think about that." } { "Tag": [ "function", "trigonometry", "algebra unsolved", "algebra" ], "Problem": "Find the maximum value of the function:\r\n\r\n$ f(x)\\equal{}\\frac{x}{x^2\\plus{}9}\\plus{}\\frac{1}{x^2\\minus{}6x\\plus{}21}\\plus{}\\cos {2 \\pi x}, x>0.$", "Solution_1": "$ f(x) \\equal{} \\frac {x}{x^2 \\plus{} 9} \\plus{} \\frac {1}{x^2 \\minus{} 6x \\plus{} 21} \\plus{} \\cos {2 \\pi x}\\ (x > 0).$\r\n\r\n$ \\leq \\frac {x}{(x \\minus{} 3)^2 \\plus{} 6x} \\plus{} \\frac {1}{(x \\minus{} 3)^2 \\plus{} 12} \\plus{} \\cos 2\\pi x\\leq \\boxed{\\frac {5}{4}}$ when $ x \\equal{} 3.$\r\n\r\nYou can also use AM-GM." } { "Tag": [], "Problem": "hi everyone, i just joined to your forum :)", "Solution_1": "Welcome. \r\n\r\nWie geht's?", "Solution_2": "dankeschn, gut, und selbst ? :D" } { "Tag": [ "geometry unsolved", "geometry" ], "Problem": "Find the point P in triangle ABC,to let the triangle PBC \u3001PAC and PAB have the same radius.\r\n :( My English is so poor!!!", "Solution_1": "No one to help me... :(", "Solution_2": "I think that P is not constructible with ruler-compass since it must satisfy\n\n$ \\frac {[\\triangle APC]}{PC \\plus{} PA \\plus{} AC} \\equal{} \\frac {[\\triangle BPC]}{PB \\plus{} PC \\plus{} BC} \\ , \\ \\frac {[\\triangle APB]}{PA \\plus{} PB \\plus{} AB} \\equal{} \\frac {[\\triangle BPC]}{PB \\plus{} PC \\plus{} BC}$\n\nBoth loci are not constructible by R & C.", "Solution_3": "I got this problem from a math magazine....I guess the point maybe the internal point...\r\nThank you all the same..." } { "Tag": [ "inequalities" ], "Problem": "Inegalitatea Medilor \r\n\r\n\\[(\\sum{x_i}\\frac{1}{n})^n\\geq{\\prod{x_i}}\\]\r\n\r\nInegalitatea Cauchy\r\n\r\n\\[\\sum{x^2_i}\\sum{y^2_i}\\geq(\\sum{x_iy_i})^2\\]\r\n\r\nInegalitatea Cebasev\r\n\r\n\\[n\\sum{x_iy_i}\\geq\\sum{x_i}\\sum{y_i} \\\\ unde \\ x = \\ ( \\ x_1 \\ , \\ x_2 \\ , \\ ..... , \\ x_n ) \\ \\ si \\ \\ y = \\ ( \\ y_1 \\ , \\ y_2 \\ , \\ ... \\ y_n ) \\ sunt \\\\ n-uple \\ la \\ fel \\ ordonate \\ \\\\ daca \\ \\ \\ \\ n-uplele \\ sunt \\ invers \\ ordonate \\ atunci \\ avem : \\\\n\\sum{x_iy_i}\\leq\\sum{x_i}\\sum{y_i}\\]\r\n\r\nInegalitatea Minkovsky\r\n\r\n\\[ \\sqrt{\\sum( x_i \\ + \\ y_i \\ )^2}\\leq\\sqrt\\ \\sum(x_i)^2} +\\sqrt\\ \\sum(y_i)^2 \\]\r\n\r\nInegalitatea Schur\r\n\r\n\\[ a^r(a-b)(a-c)+b^r(b-a)(b-c)+c^r(c-a)(c-b)\\ge 0 \\]\r\n\r\nInegalitatea Holder\r\n\r\n\\[ \\sum(p_ix_iy_i)\\leq(\\sum(p_ix_i^\\alpha)^\\frac{1}{\\alpha})(\\sum(p_iy_i^\\beta)^\\frac{1}{\\beta})\\\\\\ unde \\frac{1}{\\alpha}+\\frac{1}{\\beta}=1\\]\r\n\r\nInegalitatea Jensen\r\n\r\n\\[ f( \\sum(p_ix_i))\\leq\\ \\sum(p_if(x_i)) \\ \\ unde \\ f \\ este \\ convexa \\ pe \\ intervalul \\ I \\\\ \\sum(p_i)=1 , p_1,p_2,....,p_n\\geq0 \\ \\ x_1,x_2,....,x_n\\ \\in \\ I \\]", "Solution_1": "Daca tot vrei sa faci o lista, vezi si [url=http://www.animath.fr/cours/inegalites.pdf]materialul asta.[/url]", "Solution_2": "[quote=\"rockymarcianocosmin99\"]Inegalitatea Medilor \n\n\\[ (\\sum{x_i}\\frac{1}{n})^n\\geq{\\prod{x_i} }\\]\n\nInegalitatea Cauchy\n\n\\[ \\sum{x^2_i}\\sum{y^2_i}\\geq(\\sum{x_iy_i})^2 \\]\n\nInegalitatea Cebasev\n\n\\[ n\\sum{x_iy_i}\\geq\\sum{x_i}\\sum{y_i} \\\\ unde \\ x = \\ ( \\ x_1 \\ , \\ x_2 \\ , \\ ..... , \\ x_n ) \\ \\ si \\ \\ y = \\ ( \\ y_1 \\ , \\ y_2 \\ , \\ ... \\ y_n ) \\ sunt \\\\ n-uple \\ la \\ fel \\ ordonate \\ \\\\ daca \\ \\ \\ \\ n-uplele \\ sunt \\ invers \\ ordonate \\ atunci \\ avem : \\\\n\\sum{x_iy_i}\\leq\\sum{x_i}\\sum{y_i} \\]\n\nInegalitatea Minkovsky\n\n\\[ \\sqrt{\\sum( x_i \\ + \\ y_i \\ )^2}\\leq\\sqrt\\ \\sum(x_i)^2} +\\sqrt\\ \\sum(y_i)^2 \\]\n\nInegalitatea Schur\n\n\\[ a^r(a-b)(a-c)+b^r(b-a)(b-c)+c^r(c-a)(c-b)\\ge 0 \\]\n\nInegalitatea Holder\n\n\\[ \\sum(p_ix_iy_i)\\leq(\\sum(p_ix_i^\\alpha)^\\frac{1}{\\alpha})(\\sum(p_iy_i^\\beta)^\\frac{1}{\\beta})\\\\\\ unde \\frac{1}{\\alpha}+\\frac{1}{\\beta}=1 \\]\n\nInegalitatea Jensen\n\n\\[ f( \\sum(p_ix_i))\\leq\\ \\sum(p_if(x_i)) \\ \\ unde \\ f \\ este \\ convexa \\ pe \\ intervalul \\ I \\\\ \\sum(p_i)=1 , p_1,p_2,....,p_n\\geq0 \\ \\ x_1,x_2,....,x_n\\ \\in \\ I \\][/quote]\r\n\r\n Ai uitat Inegalitatea lui Bergstrom! :mad: Este des utilizata!" } { "Tag": [ "Support" ], "Problem": "The war has to be brought speedily to a successful conclusion and the destruction of Japanese cities by means of atomic bombs may very well be an effective method of warfare. We feel, however, that such an attack on Japan could not be justified in the present circumstances. We believe that the United States ought not to resort to the use of atomic bombs in the present phase of the war, at least not unless the terms which will be imposed upon Japan after the war are publicly announced and subsequently Japan is given an opportunity to surrender.\r\nIf such public announcement gave assurance to the Japanese that they could look forward to a life devoted to peaceful pursuits in their homeland and if Japan still refused to surrender, our nation would then be faced with a situation which might require a re-examination of her position with respect to the use of atomic bombs in the war.\r\nAtomic bombs are primarily a means for the ruthless annihilation of cities. Once they were introduced as an instrument of war it would be difficult to resist for long the temptation of putting them to such use. Both sides possess certain weapon capabilities (for example, poison gas), which, though not expressly outlawed, are understandably forbidden. Every nation is held responsible for resisting the use of these tempting, yet inhumane, weapons. If one nation was to commit a wartime atrocity, then all other nations would be justified in the\r\nThere are several alternates to the atomic bomb, which would be just as effective or perhaps more effective. \r\nThe last few years show a marked tendency toward increasing ruthlessness. At present our Air Forces, striking at the Japanese cities, are using the same methods of warfare which were condemned by American public opinion only a few years ago when applied by the Germans to the cities of England. Our use of atomic bombs in this war would carry the world a long way further on this path of ruthlessness.\r\nAtomic power will provide the nations with new means of destruction. The atomic bombs at our disposal represent only the first step in this direction and there is almost no limit to the destructive power which will become available in the course of this development. Thus a nation which sets the precedent of using these newly liberated forces of nature for purposes of destruction may have to bear the responsibility of opening the door to an era of devastation on an unimaginable scale.\r\n\r\nWhat do you guys think?", "Solution_1": "I think that you should read the book of Revelation in the bible which will tell you what will happen in the (possibly near) future.", "Solution_2": "Do not think that anybody actually supports the use of atomic bombs or similar WMD. But the way things are moving, a nuclear war may become inevitable - it may even come in our lifetime!\r\nSo enjoy life and do your good deeds while you have time. If anything can persuade the beasts in the US Administration to stay away from the inhumane deeds, it won't be our innocent opinions. It will be more advisable to concentreate on games and maths!", "Solution_3": "I think this belongs in Round Table.", "Solution_4": "If a nuclear war breakes out, I hope you all are saved through Jesus! Otherwise it would be a dark day in your life. Eventually someone is going to be completely defeated through normal warfare, and in desperation destroy the world with bombs, bialogical diseases, and the like. I think it was the right descision for us to bomb Japan, because if we had not done so, they would have continued fighting until every last one of them had been killed off (Examples at the many islands in the Pacific Ocean, the Japanese totally fought until they were all gone) That is my opinion on the matter.", "Solution_5": "filletwho, I have an interest in history too, but I really hope a moderator moves this to other topics->round table, this is more the place to chill and talk about games, and ask random questions\r\n\r\nUS officials sometimes argued that if the war had continued without the use of the bombs, more japanse would have died\r\n\r\nI can't estimate whether or not this is true.\r\n\r\nPoint is, the bomb was developed because of fear the germans were developing one too. Later inquiries after the liberation of western europe apparently pointed out the germans were never that close. On the other hand, some historians claim that germany actually tested a small nuclear device during the war.\r\nAnyway, after huge spending in the manhattan project, I think they needed to show that it had paid off actually. Also it was a good way of demonstrating superiority towards the russians.\r\n\r\n\r\nOne question, maybe a bit morbid, don't you think the russians actually saved thousands, perhaps millions of lives, by developing nuclear weapons too. During the Korean war, general McArthur proposed bombing several chinese cities (he was later replaced because he appeared out of control). The plan was rejected, partially because of fear of intervention of the Soviet Union, at that time already a nuclear power.\r\n\r\nCome to think of it, it must have been pretty scary for the soviet union to see the usa becoming the ultimate superpower, as the only nations with such weapons, even though it was only for a couple of years. \r\n\r\nOne thing bothers me, the USA is the first to point fingers at nations developing nuclear weapons, and at the same time it is the only nation to have ever bombed another country. And as despicable as you may find nuclear weapons, it is a fact that nations like Pakistan or North Korea are less likely to be attacked, because they DO have nukes. (On the other hand, the conflict at the end of sixties between Soviet Union and China shows that two nuclear nations clashing does not immediately imply a nuclear war)\r\n\r\nI am interested in your thoughts about this too." } { "Tag": [ "LaTeX" ], "Problem": "i send you the second problem.\r\ni wish you'll help me\r\nHere the problem,\r\nSolve equation:\r\n$ \\sqrt{1\\plus{}\\sqrt{1\\minus{}x^2}}\\equal{}x(1\\plus{}2\\sqrt{1\\minus{}x})$\r\n\r\nMany thanks to you!\r\nBest, Luan.", "Solution_1": "[hide]\nWell $ x\\equal{}0$ $ x\\equal{}1$ are solutions.\nI believe there are two others.[/hide]", "Solution_2": "Fraenkel, $ x\\equal{}0$ is an extraneous root.", "Solution_3": "[quote=\"10000th User\"]Fraenkel, $ x \\equal{} 0$ is an extraneous root.[/quote]\r\n\r\nlol my bad.\r\ni was seeing the wrong equation when i said that.", "Solution_4": "Thanks to interest the problem!\r\nBut i saw that, this equation has a root $ x\\equal{}1$. i can't find any other roots.can you see again and help me solve it.\r\nBest, Luan.", "Solution_5": "do you mind doing the algebra for your solution in latex for us?", "Solution_6": "Wait... did you just \"guess and check\" or actually do algebra to get $ x\\equal{}1$ ?\r\n\r\nBecause I saw that $ 1$ works, but it's too tedious to do it the long way... :D", "Solution_7": "lol i just looked at the equation and said x=1 is a solution.\r\n\r\nI think there is another real root. Having trouble finding it." } { "Tag": [ "inequalities unsolved", "inequalities" ], "Problem": "Is it true that if $a_{i}\\in[-1;1]$ and $\\sum_{i=1}^{2n}a_{i}^{3}=0$ then $\\sum_{i=1}^{2n}a_{i}\\leq 0?$", "Solution_1": "[quote=\"bilarev\"]Is it true that if $a_{i}\\in[-1;1]$ and $\\sum_{i=1}^{2n}a_{i}^{3}=0$ then $\\sum_{i=1}^{2n}a_{i}\\leq 0?$[/quote]\r\n\r\nIsn't this absurd? We can replace all $a_{i}$ by $-a_{i}$, the first condition remains true but the second changes its sign (and it's easy to see that equality can't always hold)?\r\n\r\n darij", "Solution_2": "Yeah, I saw it after I posted the problem. It was stupid.\r\nThanks Darij :roll:", "Solution_3": "Let $a_{i}$=sin $x_{i}$\r\nWith $\\sum_{i=1}^{n}a_{i}^{3}$=0 \r\nWe can prove |$\\sum_{i=1}^{n}a_{i}$| $\\leq$ $\\frac{1}{3}$\r\n\r\nIt's simple,use sin3x=3sinx-4sin$x^{3}$ only", "Solution_4": "you should replace $\\frac{1}{3}$ with $\\frac{n}{3}$ :wink:" } { "Tag": [ "inequalities proposed", "inequalities" ], "Problem": "$a, b, c \\ge 0$.\r\n$x=a^{2}+bc$\r\n$y=b^{2}+ca$\r\n$z=c^{2}+ab$\r\nprove that\r\n\\[x^{3}+y^{3}+z^{3}+(6\\sqrt{2}-3)xyz\\ge \\sqrt{2}\\sum_{sym}x^{2}y \\]\r\n.", "Solution_1": "The inequalit is equivalent to\r\n\\[\\sum (x+y+z)(x-y)^{2}\\ge 2\\sqrt{2}\\sum z(x-y)^{2}\\]\r\nOr\r\n\\[\\sum (x-y)^{2}(x+y+(1-2\\sqrt{2})z) \\ge 0 \\]\r\nNote that $x-y=(a-b)(a+b+c)$, then it is equivalent to\r\n\\[(a+b+c)^{2}\\sum (a-b)^{2}(a^{2}+b^{2}+(1-2\\sqrt{2})c^{2}+ac+bc+(1-2\\sqrt{2})ab) \\ge 0 \\]\r\nor\r\n\\[\\sum (a-b)^{2}(a^{2}+b^{2}+(1-2\\sqrt{2})c^{2}+ac+bc+(1-2\\sqrt{2})ab) \\ge 0 \\]\r\n\\[\\sum S_{c}(a-b)^{2}\\ge 0 \\]\r\nwhere\r\n\\[S_{a}=(1-2\\sqrt{2})a^{2}+b^{2}+c^{2}+(1-2\\sqrt{2})bc+ab+ac \\]\r\n\\[S_{b}=a^{2}+(1-2\\sqrt{2})b^{2}+c^{2}+ab+(1-2\\sqrt{2})ac+bc \\]\r\n\\[S_{c}=a^{2}+b^{2}+(1-2\\sqrt{2})c^{2}+ac+bc+(1-2\\sqrt{2})ab \\]\r\nWLOG, assume $a \\ge b \\ge c \\ge 0$, we have\r\n\\[S_{b}\\ge (3-2\\sqrt{2})b^{2}+2(1-\\sqrt{2})bc+c^{2}=((1-\\sqrt{2})b+c)^{2}\\ge 0 \\]\r\nSimilarly $S_{c}\\ge 0$. Furthermore,\r\n\\[a^{2}S_{b}+b^{2}S_{a}\\ge 0 \\]\r\nThen we are done. :)", "Solution_2": "$x-y=(a-b)(a+b-c)$, not $(a-b)(a+b+c)$.\r\nSo your solution is wrong. :(", "Solution_3": "Sorry, that's a big mistake. :( I will try it again.", "Solution_4": "In case no one's figured out yet, its false.\r\n[img]8631[/img]\r\nand that would have been an amazing proof, by the way :P", "Solution_5": "I think you are wrong.\r\nWhen $a=0.08, b=1, c=1$ -> $LHS-RHS=0.000605067 >0$.\r\nPlease check your calculation.\r\nIf $a=b$ -> $LHS-RHS = (3-2\\sqrt{2})(a-c)^{2}c^{2}((1+\\sqrt{2})c-a)^{2}>0$.\r\nI checked my solution, But I can't find any mistakes.", "Solution_6": "I'm not sure how it could be wrong, I'll double check the file but I'm pretty sure there is no mistake...", "Solution_7": "I think your program is not precise.\r\nI used Mathematica to calculate it.\r\nWhat program did you use?", "Solution_8": "[quote=\"me@home\"]In case no one's figured out yet, its false.\n[img]8631[/img]\nand that would have been an amazing proof, by the way :P[/quote]\r\n\r\nif $a=0.08$ $b=1$ $c=1$ then $x=1.0064$ and not $x=1.01$\r\n\r\nand moreover $LHS = 9.977726743....$ and $RHS = 9.977121676....$\r\n\r\nso $LHS-RHS=6.0506742*10{}^{-}{}^{4}$", "Solution_9": "Sorry for posting an irrelevant subject.\r\nI am planning to buy computer software for math.\r\nIs Mathematica better or Maple better?\r\nThey seem to be very expensive, so I'm thinking about it." } { "Tag": [ "trigonometry", "geometry", "geometric transformation", "reflection", "power of a point", "Pythagorean Theorem" ], "Problem": "$AB$ is a diameter of circle $O$. $AB$ is extended to $C$ so that $BC=2$. ($A$, $B$ and $C$ are collinear in this order.) $D$ is a point on circle $O$ so that $CD$ is tangent to the circle. If $AB=6$, find the length of $BD$.", "Solution_1": "[hide=\"Hint 1\"] Power-of-a-Point Theorem :) [hide=\"Then, a second hint...\"] [hide=\"Only if you really cannot figure out what to do next :P \"] Stewart's Theorem. :wink: [/hide] [/hide] [/hide]", "Solution_2": "Yeah, Stewart's Theorem works, but you don't even need to use it. There's an easier way :wink:", "Solution_3": "Well you could be lazy and use [hide=\"a piece of pie\"] Law of Cosines[/hide] also.", "Solution_4": "[hide]\nCD=4\nLet BD=a\nLet AD=b\n$a^{2}+b^{2}=36$\nFrom stewart's, we see that \n$96+2b^{2}=96+8a^{2}$\n$b^{2}=4a^{2}$\n$b=2a$\n$a^{2}+4a^{2}=36$\n$a=\\frac{6\\sqrt{5}}{5}$\n$b=\\frac{12\\sqrt{5}}{5}$[/hide]", "Solution_5": "[quote=\"cincodemayo5590\"]Well you could be lazy and use [hide=\"a piece of pie\"] Law of Cosines[/hide] also.[/quote]\r\n\r\nlol sam...you cannot use that if your solution sucks...random words are reserved for arnav/alex zhai who totally pwn\r\n\r\ni guess the fastest i see is using pythagorean theorem to get $AD^{2}+BD^{2}=36$ and $\\triangle ADC\\sim\\triangle DBC$ to get $DC=4$ and $\\frac{AD}{DB}=2$...then solve from there\r\n\r\ni suppose since i feel like showing off random geometry...here are two more solutions...\r\n\r\nreflect $B$ and $C$ over $AD$ to $B'$ and $C'$. Let $C'D$ intersect $AC$ at $E$. By menelaus' theorem, $\\frac{AE}{EB}*\\frac{BD}{DB'}*\\frac{B'C'}{C'A}=1$, $\\frac{AE}{EB}=4$...and $AE+EB=6$, so $AE=\\frac{24}{5}$.\r\n\r\nwe also have $\\angle BDC=\\angle B'DC'=\\angle EDB=\\angle DAB$, so $\\triangle DAB\\sim\\triangle EDB$, so $DB^{2}=EB*BA=6*6*4*\\frac{1}{5}$ so $DB=\\frac{12\\sqrt{5}}{5}$\r\n\r\n--\r\n\r\nor drop a perpendicular from D to AC, say D'...then AD'BC are in a harmonic range (consider other tangent from C, etc) $\\frac{AD'}{D'B}*\\frac{BC}{CA}=1$, $\\frac{AD'+D'B}{D'B}=5$, and $AD'+D'B=6$, so $AD'=\\frac{24}{5}$...using similarity, $\\triangle ADB\\sim\\triangle DD'B$, so $DB^{2}=D'B*AB$...same answer...", "Solution_6": "Um i dont know where i am going wrong but here\r\n\r\n[hide]you know that $OD=3$ and that $CD=4$. Now drop an altitude to $OC$ which has length $\\frac{12}{5}$. Call the foot of this altitude $N$. Now you know that $NC=5-\\frac{9}{5}=\\frac{16}{5}$ Now you know that since $BC=2$ that $NB=\\frac{16}{5}-2=\\frac{6}{5}$ now you have a right triangle $DNB$ with legs $\\frac{6}{5}$ and $\\frac{12}{5}$ so then $DB=\\frac{6\\sqrt5}{5}$[/hide]", "Solution_7": "[hide=\"I am just going to use Stewart for this...\"]$c=3$\n\n$b=4$\n\n$m=3$\n\n$n=2$\n\n$cnc+bmb=dad+man$\n\n$18+48 = d^{2}5+30$\n\n$d^{2}= \\frac{36}{5}$\n\n$d=\\frac{6\\sqrt{5}}{5}$\n[/hide]\r\nWHAT??? I'm getting the same answer as sonny.", "Solution_8": "i probably made a mistake...w/e" } { "Tag": [ "limit", "function" ], "Problem": "If x is greater than 0, how large must x be so that sqrt(x^2+x)-x shall differ from 1/2 by less than 0.02?", "Solution_1": "We solve the equation $ \\sqrt{x^{2}+x}-x=.48$, as it is apparent that $ \\sqrt{x^{2}+x}$ is always greater than $ x=\\sqrt{x^{2}}$.\r\n\r\n$ \\sqrt{x^{2}+x}-x=.48$\r\n$ x^{2}+x=(x+.48)^{2}$\r\n$ .04x=.2304$\r\n$ x=5.76$\r\n\r\nSo $ x$ must be greater than $ 5.76$ for the expression above to differ from $ \\frac{1}{2}$ by less than $ 0.02$.", "Solution_2": "", "Solution_3": "Firstly, we need to show that $ \\lim_{x \\to+\\infty}({\\sqrt{x^{2}+x}-x}) = \\frac{1}{2}$.\r\n\r\nLet ${ y = \\sqrt{x^{2}+x}-x}$\r\n\r\n$ \\Longrightarrow y+x ={\\sqrt{x^{2}+x}}$\r\n$ \\Longrightarrow y^{2}+2xy+x^{2}= x^{2}+x$\r\n$ \\Longrightarrow y^{2}+2xy = x$\r\n$ \\Longrightarrow x(1-2y) = y^{2}$\r\n$ \\Longrightarrow x = \\frac{y^{2}}{1-2y}$\r\n\r\nAs $ y \\to \\frac{1}{2}$ from below, $ x \\to+\\infty \\Longleftrightarrow$ as $ x \\to+\\infty$, $ y \\to \\frac{1}{2}$ from below.\r\n\r\nHaving shown that $ \\lim_{x \\to+\\infty}(\\sqrt{x^{2}+x}-x) = \\frac{1}{2}$, we now require the solution to $ {\\sqrt{x^{2}+x}-x}> 0.5-0.02 = 0.48$\r\n\r\n$ \\Longrightarrow x > \\frac{0.48^{2}}{1-2{\\cdot}0.48}$\r\n$ \\Longrightarrow x > 5.76$", "Solution_4": "[quote=\"sludgethrower\"]Firstly, we need to show that $ \\lim_{x \\to+\\infty}({\\sqrt{x^{2}+x}-x}) = \\frac{1}{2}$.[/quote]\n\nThis isn't actually necessary to address the stated problem, although it is the result that the problem suggests. \n\n[hide=\"One line\"] $ \\sqrt{x^{2}+x}-x = \\frac{x}{ \\sqrt{x^{2}+x}+x}= \\frac{1}{ \\sqrt{1+\\frac{1}{x}}+1}$ [/hide]\n\n[quote=\"davidyko\"]We solve the equation $ \\sqrt{x^{2}+x}-x=.48$, as it is apparent that $ \\sqrt{x^{2}+x}$ is always greater than $ x=\\sqrt{x^{2}}$.\n\n$ \\sqrt{x^{2}+x}-x=.48$\n$ x^{2}+x=(x+.48)^{2}$\n$ .04x=.2304$\n$ x=5.76$\n\nSo $ x$ must be greater than $ 5.76$ for the expression above to differ from $ \\frac{1}{2}$ by less than $ 0.02$.[/quote]\r\n\r\nThe problem with this is that you assumed $ f(x) = \\sqrt{x^{2}+x}-x$ was an increasing function; without this knowledge, you also need to solve $ \\sqrt{x^{2}+x}-x = 0.52$, and even then you're assuming that $ f(x)$ is monotonic.", "Solution_5": "[quote=\"t0rajir0u\"][hide=\"One line\"] $ \\sqrt{x^{2}+x}-x = \\frac{x}{ \\sqrt{x^{2}+x}+x}= \\frac{1}{ \\sqrt{1+\\frac{1}{x}}+1}$ [/hide][/quote]\r\n\r\nDammit, it's solutions like these that make me want to learn more.\r\n\r\n\r\nAnd I suppose you could calc bash it into submission, as you can easily get the fact that it's monotonically increasing by taking the first derivative.", "Solution_6": "Fortunately, the fact that it's monotonic follows pretty easily from the rewrite I gave, which is perfectly elementary. :) \r\n\r\nAnd if you're wondering about the motivation, I've seen many problems use that conjugation concept; for example, to telescope a summation.", "Solution_7": "\"Take the conjugate\" is a title of one of the chapters of [i]Math Olympiad Treasures[/i] by Titu Andreescu. Consequently, it is a technique that you should be able to recognize quickly." } { "Tag": [ "function", "AMC" ], "Problem": "Which of the following is equivalent to $ \\displaystyle \\sqrt {\\frac {x}{1 \\minus{} \\frac {x \\minus{} 1}{x}}}$ when $ x < 0$?\r\n\r\n$ \\textbf{(A) } \\minus{} x \\qquad \\textbf{(B) } x \\qquad \\textbf{(C) } 1 \\qquad \\textbf{(D) } \\sqrt {\\frac x2} \\qquad \\textbf{(E) } x\\sqrt { \\minus{} 1}$", "Solution_1": "[hide=\"Answer\"]Algebraically simplifying, we have $\\sqrt{x^2}=x$. However, this is for $x<0$, and the square root function gives the positive root, so our answer is $\\boxed{A}$.[/hide]", "Solution_2": "i got A, but some of my freinds messed up with the sign and got B. which is really suprising considering it's only number 7.", "Solution_3": "I don't mean to revive, but I have a question:\n\nFor $ \\sqrt{x^{2}}=x $, doesn't squaring a negative make it positive? Then the square root of a positive is still positive, so I got $B$.... what's wrong with that?", "Solution_4": "Yes, but if $\\sqrt{(-x)^2}=x$, the answer is not $x$, it's $-x$.", "Solution_5": "But x is already a negative number, so -(-x) = positive x", "Solution_6": "But $x$ is negative. Think about it, $\\sqrt{ (-4)^2}=4=-(-4)$. It doesn't make sense for $\\sqrt{x^2}=x$ when $x$ is negative because square roots are always greater than or equal to $0$.", "Solution_7": "So if it was to be negative, it would actually be $E$? I'm confused right now....", "Solution_8": "Can you rephrase on what you are confused with?\nThe definition of square root is\n$\\sqrt{x^2}=| x |$\nIf $x\\ge 0$, then $\\sqrt{x^2}=x$\nIf $x<0$, then $\\sqrt{x^2}=-x$\n\n$\\sqrt{-x^2}=x\\sqrt{-1}$", "Solution_9": "Right so the answer isn't $A$, it's $E$?", "Solution_10": "If you square (E) you get $16*-1=-16$, not 16. \n\nJust think about it: a positive is the negative of a negative number. \n1=-(-1), doesn't it?\n\nLet $x=-1$\nSo $\\sqrt{x^2}=\\sqrt{(-1)^2}=\\sqrt{1}$\nWe always take the positive root (it's a definition that when we see the square root of something it is positive), so $\\sqrt{1}$ is equal to 1. \n1 does equal $-x$, as we said before.", "Solution_11": "$ \\sqrt{\\frac{x}{1-\\frac{x-1}{x}}}=\\sqrt{x^2} $ NOT $\\sqrt{-x^2}$\n\nLook at the diference\n$ \\sqrt{\\frac{x}{1-\\frac{x-1}{x}}}=\\sqrt{x^2}=| x |$ This would be $-x$ if $x<0$\n$ \\sqrt{\\frac{x}{\\frac{x-1}{x}-1}} =\\sqrt{-x^2}=|x|\\sqrt{-1}$ This would be $-x\\sqrt{-1}$ if $x<0$", "Solution_12": "No, the answer is A. The expression simplifies to $\\sqrt{x^2}$. Since $x=-k$ is negative, we have $\\sqrt{(-k)^2}=k=-x$.", "Solution_13": "Technically, couldn't it be both A and B?", "Solution_14": "No since $x<0$", "Solution_15": "Remember, it's the principle square root, so it's positive. And since $x$ is negative, we want $-x$ which is positive.", "Solution_16": "\\begin{align*}\n\\sqrt{\\frac{x}{1-\\frac{x-1}{x}}} &= \\sqrt{\\frac{x}{\\frac{1}{x}}} \\\\\n&= \\sqrt{x^2} \\\\\n&= |x| \\\\\n&= \\boxed{-x}\n\\end{align*}" } { "Tag": [ "inequalities", "inequalities unsolved" ], "Problem": "If $x,y,z > \\frac{1}{2}$ and $xyz \\geq 8$ prove that:\r\n\r\n$xy+yz+zx + \\frac{1}{\\sqrt{2}}(yz\\sqrt{x}+zx\\sqrt{y}+xy\\sqrt{z}) \\geq 3xyz$\r\n\r\n :) :) :)", "Solution_1": "[quote=\"gemath\"]If $x,y,z > \\frac{1}{2}$ and $xyz \\geq \\frac{1}{8}$[/quote]\r\n\r\nIsn't there any redundancy? :?", "Solution_2": "[quote=\"spider_boy\"][quote=\"gemath\"]If $x,y,z > \\frac{1}{2}$ and $xyz \\geq \\frac{1}{8}$[/quote]\n\nIsn't there any redundancy? :?[/quote]\r\n\r\nOK OK!! I have fixed it Thank you!! :)", "Solution_3": "Why noboby help me?? :? or can you show me is it wrong ??" } { "Tag": [ "inequalities solved", "inequalities" ], "Problem": "Let $a,b,c \\in (0,1)$ and $a+b+c=2$. Prove that $4(a^{2}+b^{2}+c^{2}) \\geq 8-9abc$.\r\nWhen do we have equality?", "Solution_1": "homogenize and it's equivalent to schur's\r\n :D", "Solution_2": "What's about this problem ?\r\n\r\nLet 0< a, b ,c < 1 . a + b + c = 2 .\r\n Prove that :\r\n a 2 + b 2 + c 2 \\leq 2 - 2abc", "Solution_3": "Homogenize, then put x = m+n, y = n+p, z = p+m, m, n, p>0 (why we can?) and expand both sides, we come to an trivial inequality.\r\n\r\nNamdung" } { "Tag": [ "invariant", "linear algebra", "matrix", "linear algebra unsolved" ], "Problem": "Consider $K$ an arbitrary field and let $\\sigma_1, \\sigma_2$ two permutations in $S_n$ and $P(\\sigma_1), P(\\sigma_2)$ the permutation matrices associated to these permutations. Prove that $\\sigma_1, \\sigma_2$ are conjugated in $S_n$ if and only if the two matrices are similar. It is not that easy problem in any field.", "Solution_1": "The non-trivial implication is showing that $\\sigma_1,\\sigma_2$ are conjugates of one another if $P(\\sigma_i)$ are similar. \r\n\r\nWe can work in the algebraic closure of $K$ to conclude that the matrices have the same Jordan form. However, I think it's not that hard to see that for each $i\\in\\overline{1,n}$ the number of Jordan blocks of dimension $i$ in the Jordan form of $P(\\pi)$ is precisely the number of cycles of length $i$ in $\\pi$, so the conclusion follows, since two permutations are conjugate iff they have the same cycle structure.", "Solution_2": "Grobber, could you please be a little bit more explicit? I'm not familiar with Jordan form and I cannot prove your statement about the number of cycles. The solution I have is more difficult, but elementary. Your approach looks very nice, but I don't understand very well. Sorry for being dumb. :(", "Solution_3": "I've erased my previous response.\r\n\r\nThe whole space can be decomposed in subspaces invariant to the transformation represented by the matrix in a unique way s.t. the subspaces involved are indecomposable, i.e. cannot be further decomposed in invariant subspaces. The dimensions of these spaces are precisely the dimensions of the Jordan blocks of a matrix in Jordan form. \r\n\r\nIt's pretty clear now that for each cycle of length $i$ in the permutation, there is an indecomposable invariant subspace of dimension $i$, and that these spaces can be joined as a direct sum to form the large space, so the cycles correspond to the Jordan blocks.", "Solution_4": "Ok, thanks. :)" } { "Tag": [ "vector", "linear algebra", "matrix", "superior algebra", "superior algebra unsolved" ], "Problem": "Prove that the vectors {a_1,a_2,....,a_k} satisfying the $\\xi_1 a_1+\\xi_2 a_2+...+\\xi_n a_n=0 \\text{ where } \\xi_i\\ne 0 \\forall i$ make the linear (n-1)th-space :blush: \r\nsorry i dont know really how this problem should be writen in english...", "Solution_1": "Did you mean by this that $\\textrm{rank}(a_1,a_2,\\dots,a_n)=n-1$?", "Solution_2": "i have found a solution involving the $rank(\\xi_1,...,\\xi_n)=1$so the fundamental system of solution have the rank=n-1 as a corrallary from the theorem that if the matrix of the coefficients $a_{ij}, i,j=1 \\text{ to } n$ has the rank=k, then the rank of the vectors in the fundamental system of solutions( the system of solutions,such that any other solution is the linear combination of this solution) equals to n-r .\r\n but i want to know is there another solution that doesn't ******* to this theorem" } { "Tag": [ "real analysis", "topology", "advanced fields", "advanced fields unsolved" ], "Problem": "Is this result true?\r\n\r\nLet $ (\\Omega,\\mathcal{B})$ be a Borel-measure space and let $ \\mu$ be a $ \\sigma$-finite measure defined on it. If $ B\\in\\mathcal{B}$ such that $ \\mu(B)> 0$ then there exists a non-empty open set $ V$ that is contained in $ B$.", "Solution_1": "It is false. Just take $ \\mathbb R\\setminus\\mathbb Q$ and the Lebesgue measure on the line. It is even more apparently false if you take the $ \\delta$-measure for $ \\mu$ and the one point set it is supported by for $ B$.", "Solution_2": "A result which is true: $ B\\minus{}B$ contains an open set.", "Solution_3": "@ZetaX: could you explain that a bit? I didn't get what is $ B\\minus{}B$\r\n\r\nand what if $ \\Omega$ is locally compact and Hausdorff?", "Solution_4": "$ \\Omega \\equal{} \\mathbb R \\backslash \\mathbb Q$ is still a counterexample. You always get counterexamples if there is a countable basis for the topology.\r\n\r\nWith $ B \\minus{} B$ I meant the Minkowski-difference, meaning: $ B \\minus{} B \\equal{} \\{b_1 \\minus{} b_2 | b_i \\in B\\}$." } { "Tag": [ "function", "algebra", "functional equation" ], "Problem": "Solve the functional equation $ f(x\\plus{}t)\\minus{}f(x\\minus{}t)\\equal{}4xt$\r\n\r\nhow do you [i]solve[/i] something like this?", "Solution_1": "Consider in particular the case $ x \\equal{} t$, then $ f(2x) \\equal{} 4x^2 \\plus{} f(0)$, which is satisfied only if $ f(x) \\equal{} x^2 \\plus{} c$ for some constant $ c$.", "Solution_2": "and what if t is something else? and hwo do u knwo its asking for f(x)?", "Solution_3": "The 'solution' to a functional equation is, by definition, an expression for the function in terms of its parameters. So in this case, we are looking for $ f(x)$ in terms of $ x$. Since the functional equation satisfies the equality for [i]any[/i] $ x$ and $ t$ it must satisfy it, in particular, for the case when $ x\\equal{}t$. Thus, any $ f$ that satisfies your functional equation must necessarily satisfy $ f(x)\\equal{}x^2\\plus{}c$ for some constant $ c$. We say, then, that $ f(x)\\equal{}x^2\\plus{}c$ is [i]the[/i] family of solutions to the functional equation.", "Solution_4": "[quote=\"JoeBlow\"]The 'solution' to a functional equation is, by definition, an expression for the function in terms of its parameters.[/quote]\r\n\r\noh...that helps a lot, wish i knew that earlier\r\n\r\n(no sarcasm)\r\n\r\nthanks\r\n\r\n :lol:", "Solution_5": "JoeBlow, can you show why that is the only solution?", "Solution_6": "He just did, in the post above. :wink: (see [url=http://www.artofproblemsolving.com/Forum/viewtopic.php?t=191785]here[/url] also)." } { "Tag": [ "puzzles" ], "Problem": "What is next in the series:\r\n13,17,31,37,71,73,79,107,____", "Solution_1": "[hide]113,149,157,167[/hide]", "Solution_2": "jmadsen, what is the formation of the sequence?????", "Solution_3": "Primes everywhere, but I don't know how he got them.", "Solution_4": "[hide]Try reversing the digits.[/hide]", "Solution_5": "i dont understand... maybe it is like if you reverse it is still prime????", "Solution_6": "[quote] [b]Jmadsen wrote:[/b]\n113,149,157,167\nand\nTry reversing the digits\n[/quote]\r\n\r\n\r\n\r\nThat's right. The series contain primes whose reversal of digits is a different prime in ascending order.", "Solution_7": "You missed 97.", "Solution_8": "Well, that is a good one!\r\n:first:", "Solution_9": "[quote]\n[b]JBL wrote:[/b]\nYou missed 97[/quote]\r\n\r\nOops! Sorry for that." } { "Tag": [ "calculus", "geometry" ], "Problem": "EVERYONE\r\n\r\nconway may seem pretty boring at first\r\nthis is because it's the first lecture and you aren't really used to getting lectured so much\r\nbut he really knows what he's talking about\r\nit's also pretty confusing at times, he goes on tangents often, but they're always related and relevant\r\n\r\nMAKE SURE YOU LISTEN IN HIS LECTURES", "Solution_1": "hey wait a minute are you allowed to record lectures?", "Solution_2": "no one can really stop you...", "Solution_3": "ahem\r\n\r\nalways relevant?\r\n\r\nwhy was he talking about like the earth or whatever when he was supposed to be explaining rainbows?\r\n\r\nAND SINCE WHEN WAS RAINBOWS THAT IMPORTANT TO MATH?!", "Solution_4": "well, rainbows is pretty physics-y\r\n\r\nlike I'm pretty sure on one of my physics tests two years ago, one of the questions asked us to explain rainbows\r\n\r\nand physics is kinda math?\r\n\r\nlike it's based on snell's law, I suppose\r\n\r\nand you could derive snell's law based on fermat's principle\r\n\r\nusing calculus of variations\r\n\r\nAND HA THAT'S MATH", "Solution_5": "[quote=\"Ubemaya\"]ahem\n\nalways relevant?\n\nwhy was he talking about like the earth or whatever when he was supposed to be explaining rainbows?\n\nAND SINCE WHEN WAS RAINBOWS THAT IMPORTANT TO MATH?![/quote]\r\n\r\n\r\nwell yeah lol i believe i fell asleep during that lecture\r\nBUT THAT WAS THE ONLY TIME DURING CAMP I SWEAR\r\nI SAT THROUGH ALL OF BERNDT'S LECUTRES (\"cue (q) cue cubed sub infinity, cue cubed cue cubed sub zero, cue squared cue cubed sub n cubed...\")\r\n\r\nbut yeah he was pretty funny in that lecture\r\n\"i'm one of only two people who know how rainbows work\"\r\n\r\nand yeah watch out for the \"i have one fault, i'm too modest\" thing, he'll say that at least once\r\n\r\nhis other lectures were pretty good though (bernoulli numbers, n-D geometry, fibonacci/lucas/similar sequences, infinite numbers), it would have been pretty cool if he had done cellular automata but w/e", "Solution_6": "In 07 he talked about pinecones :D", "Solution_7": "pinecone bombs? those are pretty awesome. my friends made one.", "Solution_8": "no, just relating them to Fibonacci Numbers", "Solution_9": "pft. ok :rotfl:" } { "Tag": [], "Problem": "\u03a0\u03a1\u039f\u03a4\u0391\u03a3\u0397 3. \u0394\u03af\u03b4\u03b5\u03c4\u03b1\u03b9 \u03c4\u03c1\u03af\u03b3\u03c9\u03bd\u03bf \u0391\u0392\u0393 \u03ba\u03b1\u03b9 \u03ad\u03c3\u03c4\u03c9 \u0394 \u03c4\u03c5\u03c7\u03cc\u03bd \u03c3\u03b7\u03bc\u03b5\u03af\u03bf \u03b5\u03c0\u03af \u03c4\u03b7\u03c2 \u03c0\u03bb\u03b5\u03c5\u03c1\u03ac\u03c2 \u0392\u0393. \u03a0\u03c1\u03bf\u03c3\u03b4\u03b9\u03bf\u03c1\u03af\u03c3\u03c4\u03b5 \u03c3\u03b7\u03bc\u03b5\u03af\u03bf \u039c \u03b5\u03c0\u03af \u03c4\u03b7\u03c2 \u03b5\u03c5\u03b8\u03b5\u03af\u03b1\u03c2 \u0391\u0394, \u03ad\u03c4\u03c3\u03b9 \u03ce\u03c3\u03c4\u03b5 \u03c4\u03b1 \u03c4\u03c1\u03af\u03b3\u03c9\u03bd\u03b1 \u0391\u039c\u0392, \u0391\u039c\u0393, \u03bd\u03b1 \u03ad\u03c7\u03bf\u03c5\u03bd \u03af\u03c3\u03bf\u03c5\u03c2 \u03c0\u03b5\u03c1\u03b9\u03b3\u03b5\u03b3\u03c1\u03b1\u03bc\u03bc\u03ad\u03bd\u03bf\u03c5\u03c2 \u03ba\u03cd\u03ba\u03bb\u03bf\u03c5\u03c2.", "Solution_1": "\u039f\u03c5\u03c3\u03b9\u03b1\u03c3\u03c4\u03b9\u03ba\u03ac \u03b5\u03af\u03bd\u03b1\u03b9 \u03b9\u03c3\u03bf\u03b4\u03cd\u03bd\u03b1\u03bc\u03b7 \u03bc\u03b5 \u03c4\u03b7\u03bd \u03c0\u03c1\u03cc\u03c4\u03b1\u03c3\u03b7 (1) :wink:", "Solution_2": "\u03a3\u03c4\u03b9\u03c2 \u03c3\u03b7\u03bc\u03b5\u03b9\u03ce\u03c3\u03b5\u03b9\u03c2 \u03bc\u03bf\u03c5 \u03bf\u03b9 \u03c0\u03c1\u03bf\u03c4\u03ac\u03c3\u03b5\u03b9\u03c2 (1), (3) \u03b5\u03af\u03bd\u03b1\u03b9 \u03b5\u03bd\u03bf\u03c0\u03bf\u03b9\u03b7\u03bc\u03ad\u03bd\u03b5\u03c2 \u03c3\u03b5 \u03bc\u03af\u03b1 \u03c0\u03c1\u03cc\u03c4\u03b1\u03c3\u03b7, \u03bb\u03cc\u03b3\u03c9 \u03b1\u03c5\u03c4\u03ae\u03c2 \u03b1\u03ba\u03c1\u03b9\u03b2\u03ce\u03c2 \u03c4\u03b7\u03c2 \u03c0\u03b1\u03c1\u03b1\u03c4\u03ae\u03c1\u03b7\u03c3\u03b7\u03c2. \u03a4\u03b9\u03c2 \u03c0\u03b1\u03c1\u03bf\u03c5\u03c3\u03af\u03b1\u03c3\u03b1 \u03b5\u03b4\u03ce \u03be\u03b5\u03c7\u03c9\u03c1\u03b9\u03c3\u03bc\u03ad\u03bd\u03b5\u03c2, \u03b3\u03b9\u03b1\u03c4\u03af \u03ad\u03c4\u03c5\u03c7\u03b5 \u03ba\u03b1\u03b9 \u03b5\u03b3\u03ce \u03bd\u03b1 \u03b1\u03c3\u03c7\u03bf\u03bb\u03b7\u03b8\u03ce \u03bc\u03b5 \u03b1\u03c5\u03c4\u03ad\u03c2 \u03be\u03b5\u03c7\u03c9\u03c1\u03b9\u03c3\u03c4\u03ac. \u0397 (3) \u03bc\u03c0\u03bf\u03c1\u03b5\u03af \u03bd\u03b1 \u03c3\u03c4\u03b1\u03b8\u03b5\u03af \u03bc\u03cc\u03bd\u03b7 \u03c4\u03b7\u03c2 \u03c3\u03b1\u03bd \u03c0\u03c1\u03cc\u03c4\u03b1\u03c3\u03b7 \u03ba\u03b1\u03b9 \u03b7 (1) \u03b5\u03af\u03bd\u03b1\u03b9 \u03bf\u03c5\u03c3\u03b9\u03b1\u03c3\u03c4\u03b9\u03ba\u03ac \u03c0\u03cc\u03c1\u03b9\u03c3\u03bc\u03b1 \u03c4\u03b7\u03c2 (3).", "Solution_3": "\u039b\u03bf\u03b9\u03c0\u03cc\u03bd \u03b1\u03c6\u03bf\u03cd \u03bc\u03bf\u03c5 \u03b6\u03b7\u03c4\u03ae\u03b8\u03b7\u03ba\u03b5 \u03bd\u03b1 \u03b2\u03ac\u03bb\u03bb\u03c9 \u03c4\u03b7 \u03bb\u03cd\u03c3\u03b7 \u03bc\u03bf\u03c5 here it is :wink: \r\n\r\n\u0388\u03c3\u03c4\u03c9 \u0393' \u03c4\u03bf \u03c3\u03c5\u03bc\u03bc\u03b5\u03c4\u03c1\u03b9\u03ba\u03cc \u03c4\u03bf\u03c5 \u0393 \u03c9\u03c2 \u03c0\u03c1\u03bf\u03c2 \u03c4\u03b7\u03bd \u0391\u0394 . \u03a4\u03cc\u03c4\u03b5 \u03ad\u03c7\u03bf\u03c5\u03bc\u03b5 <\u0391\u0393\u039c = <\u0391\u0393'\u039c = <\u0391\u0392\u039c \r\n\u039a\u03b1\u03b9 \u03c4\u03cc\u03c4\u03b5 \u03c4\u03b1 \u0393', \u0392 \u03b2\u03c1\u03af\u03c3\u03ba\u03bf\u03bd\u03c4\u03b1\u03b9 \u03c3\u03c4\u03bf \u03af\u03b4\u03b9\u03bf \u03bc\u03ad\u03c1\u03bf\u03c2 \u03b5\u03c0\u03af\u03c0\u03b5\u03b4\u03bf\u03c5 \u03c0\u03bf\u03c5 \u03c7\u03c9\u03c1\u03af\u03b6\u03b5\u03b9 \u03b7 \u0391\u0394 \u03ac\u03c1\u03b1 \u03c4\u03bf \u03b6\u03b7\u03c4\u03bf\u03cd\u03bc\u03b5\u03bd\u03bf \u03c3\u03b7\u03bc\u03b5\u03af\u03bf \u03b5\u03af\u03bd\u03b1\u03b9 \u03c4\u03bf \u03c3\u03b7\u03bc\u03b5\u03af\u03bf \u03c4\u03bf\u03bc\u03ae\u03c2 \u03c4\u03bf\u03c5 \u03c0\u03b5\u03c1\u03b9\u03b3\u03b3\u03b5\u03b3\u03c1\u03b1\u03bc\u03bc\u03ad\u03bd\u03bf\u03c5 \u03ba\u03cd\u03ba\u03bb\u03bf\u03c5 \u03c4\u03bf\u03c5 \u0391\u0392\u0393' \u03c4\u03c1\u03b9\u03b3\u03ce\u03bd\u03bf\u03c5 \u03bc\u03b5 \u03c4\u03b7\u03bd \u0391\u0394 . :)", "Solution_4": "\u0388\u03c4\u03c3\u03b9 \u03ad\u03c7\u03bf\u03c5\u03bd \u03c4\u03b1 \u03c0\u03c1\u03ac\u03b3\u03bc\u03b1\u03c4\u03b1, \u03c4\u03b1 \u03c3\u03c5\u03bc\u03bc\u03b5\u03c4\u03c1\u03b9\u03ba\u03ac \u03c4\u03c9\u03bd \u0392, \u0393, \u03c9\u03c2 \u03c0\u03c1\u03bf\u03c2 \u03c4\u03b7\u03bd \u0391\u0394, \u03ba\u03b5\u03af\u03c4\u03b1\u03b9 \u03b5\u03c0\u03af \u03c4\u03c9\u03bd \u03af\u03c3\u03c9\u03bd \u03ba\u03cd\u03ba\u03bb\u03c9\u03bd ( \u03bc\u03af\u03b1 \u03bc\u03b9\u03ba\u03c1\u03ae \u03bc\u03cc\u03bd\u03bf \u03c4\u03c5\u03c0\u03bf\u03b3\u03c1\u03b1\u03c6\u03b9\u03ba\u03ae \u03b4\u03b9\u03cc\u03c1\u03b8\u03c9\u03c3\u03b7: \u03ad\u03c7\u03bf\u03c5\u03bc\u03b5 <\u0391\u0393\u039c = <\u0391\u0393'\u039c = <\u0391\u0392\u039c ). \r\n\r\n \u0391\u03bd \u03ba\u03b1\u03b9 \u03b3\u03b5\u03bd\u03b9\u03ba\u03ac \u03bf\u03b9 \u03ba\u03b1\u03c4\u03b1\u03c3\u03ba\u03b5\u03c5\u03ad\u03c2 ( \u03c0\u03b9\u03b8\u03b1\u03bd\u03cc\u03bd ) \u03b4\u03b5\u03bd \u03ad\u03c7\u03bf\u03c5\u03bd \u03c4\u03b7\u03bd \u03b1\u03af\u03b3\u03bb\u03b7 \u03c4\u03c9\u03bd \u03b1\u03c0\u03bf\u03b4\u03b5\u03b9\u03ba\u03c4\u03b9\u03ba\u03ce\u03bd \u03c0\u03c1\u03bf\u03c4\u03ac\u03c3\u03b5\u03c9\u03bd, \u03cc\u03c4\u03b1\u03bd \u03b6\u03b7\u03c4\u03b5\u03af\u03c4\u03b1\u03b9 \u03bf \u03c0\u03c1\u03bf\u03c3\u03b4\u03b9\u03bf\u03c1\u03b9\u03c3\u03bc\u03cc\u03c2 \u03c4\u03b7\u03c2 \u03b8\u03ad\u03c3\u03b7\u03c2 \u03b5\u03bd\u03cc\u03c2 \u03c3\u03b7\u03bc\u03b5\u03af\u03bf\u03c5, \u03ce\u03c3\u03c4\u03b5 \u03bd\u03b1 \u03b9\u03ba\u03b1\u03bd\u03bf\u03c0\u03bf\u03b9\u03b5\u03af\u03c4\u03b1\u03b9 \u03ba\u03ac\u03c0\u03bf\u03b9\u03b1 \u03c3\u03c5\u03bd\u03b8\u03ae\u03ba\u03b7, \u03c3\u03c4\u03b7\u03bd \u03bf\u03c5\u03c3\u03af\u03b1 \u03ad\u03c7\u03bf\u03c5\u03bc\u03b5 \u03c0\u03ac\u03bb\u03b9 \u03bc\u03af\u03b1 \u03b1\u03c0\u03bf\u03b4\u03b5\u03b9\u03ba\u03c4\u03b9\u03ba\u03ae \u03c0\u03c1\u03cc\u03c4\u03b1\u03c3\u03b7.\r\n\r\n \u03a3\u03b5 \u03b5\u03c5\u03c7\u03b1\u03c1\u03b9\u03c3\u03c4\u03ce \u03b3\u03b9\u03b1 \u03c4\u03bf \u03b5\u03bd\u03b4\u03b9\u03b1\u03c6\u03ad\u03c1\u03bf\u03bd \u03c3\u03bf\u03c5.\r\n\r\n \u039a\u03ce\u03c3\u03c4\u03b1\u03c2 \u0392\u03ae\u03c4\u03c4\u03b1\u03c2.", "Solution_5": "\u0395\u03b3\u03ce \u03b5\u03c5\u03c7\u03b1\u03c1\u03b9\u03c3\u03c4\u03ce \u03b3\u03b9\u03b1 \u03c4\u03b9\u03c2 \u03c9\u03c1\u03b1\u03af\u03b5\u03c2 \u03b1\u03c3\u03ba\u03ae\u03c3\u03b5\u03b9\u03c2 . \r\n\u039d\u03b1 \u03c3\u03b7\u03bc\u03b5\u03b9\u03ce\u03c3\u03c9 \u03b5\u03c0\u03af\u03c3\u03b7\u03c2 \u03cc\u03c4\u03b9 \u03b1\u03c6\u03bf\u03cd \u03ba\u03b1\u03c4\u03b1\u03c3\u03ba\u03b5\u03c5\u03ac\u03c3\u03c4\u03b7\u03ba\u03b5 \u03b1\u03c5\u03c4\u03ae \u03ac\u03bc\u03bc\u03b5\u03c3\u03b1 \u03ba\u03b1\u03c4\u03b1\u03c3\u03ba\u03b5\u03c5\u03ac\u03c3\u03c4\u03b7\u03ba\u03b5 \u03ba\u03b1\u03b9 \u03b7 \u03c0\u03c1\u03cc\u03c4\u03b1\u03c3\u03b7 (1) \u03cc\u03c0\u03c9\u03c2 \u03b5\u03af\u03c7\u03b1\u03bc\u03b5 \u03c0\u03c1\u03bf\u03b1\u03bd\u03b1\u03c6\u03ad\u03c1\u03b5\u03b9 :wink:" } { "Tag": [ "linear algebra", "matrix", "search" ], "Problem": "Let $A, B \\in Mat_{n}(K)$. Prove that:\r\n1) $|rankA-rankB| \\leq\\ rank(A+B) \\leq\\ rank(A)+rank(B)$\r\n2) $rankA+rankB-n \\leq\\ rank(AB) \\leq\\ min (rank(A), rank(B))$", "Solution_1": "[quote=\"kingkong\"]Let $A, B \\in Mat_{n}(K)$. Prove that:\n1) $|rankA-rankB| \\leq\\ rank(A+B) \\leq\\ rank(A)+rank(B)$\n2) $rankA+rankB-n \\leq\\ rank(AB) \\leq\\ min (rank(A), rank(B))$[/quote]\r\n\r\nNo one can prove!!??!!", "Solution_2": "You should consider using the Search function.\r\nRelated to the second part: http://www.mathlinks.ro/Forum/viewtopic.php?search_id=252804477&t=43691" } { "Tag": [ "Support" ], "Problem": "what exactly is the spam policy?", "Solution_1": "Right, I think some serious action needs to be taken over something as stupid as this.. perhaps a banning for a couple of weeks? And then a permanent one if it continues? Thats usually what happens in other forums..", "Solution_2": "yes...he's just trying to annoy me...he kept doing the same in Fun & Games, but I deleted them all. He started here because I can't get to them...", "Solution_3": "i must be missing something...", "Solution_4": "apparently... me too...\r\n\r\nwell...\r\n\r\ndo some contests, like the Getting Started Contest.\r\n\r\n(yes, that was an ad, lol)", "Solution_5": "Ah you must have only seen this after it was edited.. mystic just came through posting about 3 separate messages with their subjects and contents just the word \"spam\".\r\n\r\nOh by the way.. as for the edited topic.. I don't think this should be in fun and games, either \"support\" or \"advanced problems\" would be very suitable for your spam problem..", "Solution_6": "lol!", "Solution_7": "If you spam in Advanced Problems or Other Problems Solving Topics, your spams will quickly be removed. I shall be sure to enforce that. I think TripleM will enforce it in Advanced Problems as well if he gets to it before I do.", "Solution_8": "I pledge my support. I will try to remove any spam from AMC if it appears there as soon as I see it. \r\n\r\nMaybe we should have a treaty of moderators or something." } { "Tag": [ "geometry", "geometry theorems" ], "Problem": "Brahmagupta's formula gives the area of cyclic quadrilaterals. Is there a general formula which gives the area of cyclic polygons?", "Solution_1": ":? this is the same as me... I've asked my self this and asked my self about what about the product of diagonals of a incribed 2n-gon (you know if n=4 ptolomey, if n=6 furhmann)", "Solution_2": "thx for the response... I would like to know the furhmann formula...." } { "Tag": [ "inequalities", "inequalities open" ], "Problem": "If $ a,b,c,d,k \\ge 0$ prove that\r\n\\[ \\sum_{cyc} \\frac {a^k}{(a \\plus{} b \\plus{} c)^k} \\le \\max \\{ 2, \\frac {4}{3^k}\\}\r\n\\]\r\nI have solution only for $ k \\ge 1$ and $ k \\le \\frac {1}{2}$. :(", "Solution_1": "I think there is an inequality \r\n\r\nFor $ a,b,c \\geq 0 , s>r>0$\r\n$ (\\sum_{cyc} a^s)^ {\\frac {1}{s}} < (\\sum_{cyc} a^r)^ {\\frac {1}{r}}$ (Is it Jensen?)\r\n\r\nFor $ k \\geq 1$ we have $ \\frac {\\sum_{cyc} a^k}{(a+b+c)^k} \\leq 1 \\leq RS$ \r\n\r\nFor $ k<1$ use power-mean\r\n\r\n${ \\frac {\\sum_{cyc} a^k}{(a+b+c)^k} \\leq 3^{(1-k)}}$\r\nObviosly $ 3^{(1-k)} \\leq \\frac {4}{3^k} \\leq RS$", "Solution_2": "i will try to come up with another solution if i can" } { "Tag": [ "algebra unsolved", "algebra" ], "Problem": "$\\begin{gathered} f: N \\to N,p,k \\in N. \\hfill \\\\ f^{(1)} (n) = f(n),f^{(m + 1)} (n) = f^{(m)} (f(n)). \\hfill \\\\ \\end{gathered}$\r\nIf $f^{(p)} (n) = n + k$.Prove $p|k$.", "Solution_1": "It'a cute problem :) Let us define also $f^{(0)}(n)=n$. Now we have $f^{(p)}(n)=n+k$ Thus: \\[ f(n+k)=f^{(p+1)}(n)=f^{(p)}(f(n))=f(n)+k \\] so we've obtained $f(n+k)=f(n)+k$. Our first equation means that $f^{(p)}\\equiv f^{(0)}mod (k)$ It can be shown ( I can show it later if you want) that in that case all numbers $f^{(0)}(n),f^{(1)}(n),...,f^{(p-1)}(n),$ have different residues modulo $k$. One can also show that if for some $o,l,m,n$ $f^{(l)}(n) \\not\\equiv f^{(o)}(m)$ then for any $x,y$ $f^{(x)}(n) \\not\\equiv f^{(y)}(m)$. Let's take numbers $f^{(0)}(1),f^{(1)}(1),...,f^{(p-1)}(1)$ modulo $k$ these are $p$ diferent numbers. Now let's take the smallest residue modulo $k$ which wasn't chosen earlier. Let it be residue $r$. Then numbers $f^{(0)}(r),f^{(1)}(r),...,f^{(p-1)}(r)$ are different and so we have now chosen $2p$ distinct residues modulo $k$....and so on ...I'm sure you can see this :) we are always able to chose $p$ different residues, different to all residues we've chosen earlier. And as we have in total $k$ residues then $p|k$. Iknow this proof was not clear so I if you want me to write sth. more precisely just ask :)", "Solution_2": "[quote=\"jastrzab\"]It can be shown ( I can show it later if you want) that in that case all numbers $f^{(0)}(n),f^{(1)}(n),...,f^{(p-1)}(n),$ have different residues modulo $k$. One can also show that if for some $o,l,m,n$ $f^{(l)}(n) \\not\\equiv f^{(o)}(m)$ then for any $x,y$ $f^{(x)}(n) \\not\\equiv f^{(y)}(m)$. [/quote]\r\nWould you please show it more particularly?\r\nIt is not so easy to catch for me. ;) ;)", "Solution_3": "Sorry....I've totally forgotten about this post :) Well first of all if for some $x,y$ we have $f^{(x)}(n )\\equiv f^{(y)}(m) \\; mod \\; k$ then for every $w$ we have $f^{(x+w)}(n )\\equiv f^{(y+w)}(m) \\; mod \\; k \\; (*)$ Suppose that in the set $f^{(0)}(n), f^{(1)}(n),..., f^{(p-1)}(n)$ there are two numbers with the same residue $mod \\; k$ From this fact, there exists such indice $rn+k$ so we have a contradiction which shows that in set \r\n$f^{(0)}(n), f^{(1)}(n),..., f^{(p-1)}(n)$ there are exactly $p$ different residues $mod \\; k$. Now the second part of this what was unclear...well actually I'think I haven't written precisely what I wanted to write...Maybe now I'll do it more clear :) I want to show that if for some $l,o,m,n$ we have $f^{(l)}(n)\\equiv f^{(o)}(m)$ then thet ser of residues $f^{(0)}(n), f^{(1)}(n),..., f^{(p-1)}(n)$ is the same as set $f^{(0)}(m), f^{(1)}(m),..., f^{(p-1)}(m)$of course terms may be in different order, but sets are the same. This result comes straightforward from $(*)$ Now we are able to choose this disjoints sets of residues as we made in final part of this unclear proof", "Solution_4": "thank you very much!!\r\ntwo more questions:\r\n(1)why $f^{(0)}(n )\\equiv f^{(r)}(n)$\r\n(2)why $f^{(p)}(n)=n+ak(\\frac{p}{r})>n+k$\r\nforgive my foolishness :P", "Solution_5": "Well when we assume that there exist two different indices $i,j$ and $ii with i from 1 to p, s.t. xixi+1 is the same no matter what i we choose (the indices are considered modulo p). Show that 2p | n.", "Solution_1": "Looks the same like this one:\r\nhttp://www.mathlinks.ro/Forum/viewtopic.php?t=35507" } { "Tag": [ "puzzles" ], "Problem": "You're given 9 empty boxes and the digits from 1 to 9. Each digit may only be used once and only one digit can be used in one box. Find some permutation of the digits such that \\[\\frac{\\square}{\\square}\\times \\square+\\frac{\\square \\times \\square \\times \\square}{\\square}+\\square \\times \\square =100.\\] Have fun. :D\r\n\r\n[hide=\"Answer\"]$\\frac{6*4}{8}+\\frac{2*5*7}{1}+3*9$[/hide]", "Solution_1": "Is that the only way to do it?\r\n\r\nHow would you arrive to that answer other than doing guess and check or maybe brute force?\r\n\r\n :ninja: :ninja:", "Solution_2": "Nah, I've found 1 more. \r\n\r\n[hide=\"Approach\"]I considered the parity in two cases. You have\n\neven+even+even=even or\neven+odd+odd=even \n\n :) \n[/hide]" } { "Tag": [ "algebra", "polynomial", "absolute value", "algebra unsolved" ], "Problem": "Let $ a,b_1,b_2,\\cdots,b_n,c_1,c_2,\\cdots,c_n$ be real numbers such that\r\n\r\n$ x^{2n}\\plus{}ax^{2n\\minus{}1}\\plus{}ax^{2n\\minus{}2}\\plus{}\\cdots\\plus{}ax\\plus{}1\\equal{}(x^2\\plus{}b_1x\\plus{}c_1)(x^2\\plus{}b_2x\\plus{}c_2)\\cdots(x^2\\plus{}b_nx\\plus{}c_n)$\r\n\r\nfor all real numbers $ x$. Prove that $ c_1\\equal{}c_2\\equal{}\\cdots\\equal{}c_n\\equal{}1$.", "Solution_1": "A partial proof:\r\nLet $ p(x)$ be the studied polynomial. $ f(x)\\equal{}p(x)(1\\minus{}x)\\equal{}\\minus{}x^{2n\\plus{}1}\\plus{}x^{2n}(1\\minus{}a)\\plus{}x(a\\minus{}1)\\plus{}1$. \r\n$ f''(x)\\equal{}x^{2n\\minus{}2}(\\minus{}\\alpha{x}\\plus{}\\beta)$ with $ \\alpha>0$. Thus $ f'$ is increasing then decreasing. Therefore $ f$ has at most 3 (with multiplicity) real roots and $ p$ has at most 2 (with multiplicity) real roots in the form $ u,1/u$. The $ p$- decomposition in irreducible over $ \\mathbb{R}$ is in one of the following forms: $ (x^2\\plus{}b_1x\\plus{}c_1)\\cdots(x^2\\plus{}b_{n\\minus{}1}x\\plus{}c_{n\\minus{}1})(x\\minus{}u)(x\\minus{}1/u)$ or $ (x^2\\plus{}b_1x\\plus{}c_1)\\cdots(x^2\\plus{}b_{n}x\\plus{}c_{n})$. It remains to prove that if $ z$ is a non real root of $ p$ then $ |z|\\equal{}1$.", "Solution_2": "Let $ z\\equal{}e^{2i\\alpha}.$ Then $ \\frac{z^{2n}\\plus{}1}{z^{2n\\minus{}1}\\plus{}...\\plus{}z}\\equal{}\\frac{\\sin(2n\\plus{}1)\\alpha}{\\sin(2n\\minus{}1)\\alpha}\\minus{}1.$ (Prove)\r\nNow consider $ f(\\alpha)\\equal{}\\sin(2n\\plus{}1)\\alpha \\plus{} (a\\minus{}1)\\sin(2n\\minus{}1)\\alpha\\equal{}0.$ Prove that it has $ n\\minus{}1$ roots in $ (0,\\pi /2).$ Thus the given polynomial has at least $ n\\minus{}1$ roots on the unit circle with arguments in $ (0, \\pi /2).$ Since $ p(x)\\in\\mathbb{R}[x]$ it has at least $ 2n\\minus{}2$ roots on the unit circle which yields to the statement from the problem.", "Solution_3": "[quote=\"loup blanc\"]and $ p$ has at most 2 (with multiplicity) real roots in the form $ u,1/u$[/quote]\r\nOnly if $ p(x)$ has a real root $ u\\not \\equal{}{1\\over u}$. \r\nOtherwise you need to show, that $ 1$ and $ \\minus{}1$ can't be both roots pf $ p(x)$.\r\nAlso if $ z$ is a non-real root of $ p(x)$ of absolute value $ \\not \\equal{}1$,\r\nthen $ z,\\bar{z},{1\\over z}$ are $ 3$ different such roots, which contradicts bilarev's findings.\r\n\r\nIncredible work, loup blanc and bilarev!", "Solution_4": "Hi olorin,\r\nyou are right; we must prove that $ \\pm1$ can't be both roots of $ p$; it's easy because else $ a\\equal{}2$ and $ n\\equal{}0$." } { "Tag": [ "calculus", "integration", "function", "LaTeX", "real analysis", "real analysis unsolved" ], "Problem": "Here are two problems for those who are interrested:\r\n \r\n $\\bb First Problem $\r\n\r\n Let $f:[0,1]\\to\\mathbb{R}$ a continuous function such that $\\int_{0}^{1} f(x)dx=0$.Prove that there is $c\\in (0,1)$ such that $\\int_{0}^{c} f(x)dx=f(c)$.\r\n\r\n $\\bb Second Problem$\r\n\r\n Let $f:[a,b]\\to\\mathbb{R}$ a continuous function such that $\\int_{a}^{b} f(x)dx=0$.Prove that there is $c\\in (a,b)$ such that $\\int_{a}^{c} f(x)dx=c\\cdot f(c)$", "Solution_1": "Still nobody? :(", "Solution_2": "Be patient. You only posted them 3 hours ago. :)", "Solution_3": "For the first problem consider the function $g(x)=e^{-x} \\cdot \\int_{0}^{x} f(t)dt$ \r\nObserve that $g(0)=g(1)=0$ and apply the Rolle's theorem.\r\n\r\nIs this ok?? :) :)\r\n\r\nThe second is solved by the same way: taking the auxiliary function $h(x)=\\frac{\\int_{a}^{x} f(t)dt}{x}$\r\n\r\nBy the way, could you give me a latex form for the power that has base $e$ and exponent $-x$", "Solution_4": "At first glance,Socrates,I think it is correct. ;)", "Solution_5": "Ok.\r\nThey are quite clever problems from analysis!!! :D ;) :)" } { "Tag": [ "algebra unsolved", "algebra" ], "Problem": "Let $ x,y,z\\in R, x, y, z \\neq 0$ and $ \\frac {1}{x} \\plus{} \\frac {1}{y} \\plus{} \\frac {1}{z} \\equal{} \\frac {1}{x \\plus{} y \\plus{} z}$\r\nProve that: $ \\frac {1}{x^{2009}} \\plus{} \\frac {1}{y^{2009}} \\plus{} \\frac {1}{z^{2009}} \\equal{} \\frac {1}{x^{2009} \\plus{} y^{2009} \\plus{} z^{2009}}$\r\n\r\n\r\n--------------------------------------------------------------\r\nMy school site: http://www.chuyenquangtrung.com.vn", "Solution_1": "If $ x\\plus{}y\\ge 0, y\\plus{}z\\ge 0, z\\plus{}x\\ge 0$, then only one of $ x,y,z$ may be negative. From $ xyz>0$ we get, that $ x>0,y>0,z>0$, therefore $ \\frac 1x\\plus{}\\frac 1y \\plus{}\\frac 1z >\\frac{1}{x\\plus{}y\\plus{}z}$. :lol:", "Solution_2": "Thank Rust. I changed it, so.", "Solution_3": "[quote=\"thanhnam2902\"]Let $ x,y,z\\in R, x, y, z \\neq 0$ and $ \\frac {1}{x} \\plus{} \\frac {1}{y} \\plus{} \\frac {1}{z} \\equal{} \\frac {1}{x \\plus{} y \\plus{} z}$\n[/quote]\r\nThis is equivalent to $ (x\\plus{}y)(y\\plus{}z)(z\\plus{}x)\\equal{}0$ . Done! :P", "Solution_4": "Yes. Thank N.T.Tuan very muck. It's easy." } { "Tag": [ "number theory open", "number theory" ], "Problem": "i read the following problem's solution and did not\r\nunderstand at some point allong the way:\r\n\r\nstatement:\r\nlet $p>0$ be prime and $p\\equiv 3 (mod 4)$\r\nproove that $p$ is prime also in $\\mathbb{Z}[i]$\r\n\r\nproof(that i partially understand):\r\nif p would not be prime in $\\mathbb{Z}[i]$ there would\r\nexist $\\alpha,\\beta \\in\\mathbb{Z}[i]$ so that\r\n$p^{2}=N(p)=N(\\alpha)N(\\beta)$ (N beeing the norm $N(a+bi)=a^{2}+b^{2}$)\r\nand through hypothesis( ?? ),we conclude that \r\n$N(\\alpha)=p$ but that would not be possible since the norm $N$ is\r\na sum of 2 squares wich is never congruent to 3 modulo 4.\r\n\r\n\r\n\r\nwhere ive posed the question marks is where i do not understand,\r\nwhat from hypothesis is used to state that $N(\\alpha)=p$,in my\r\noppinion $N(\\alpha)$ could also be $p^{2}$.\r\nwhere am i wrong ?", "Solution_1": "If $N(\\alpha)=p^{2}$ then $N(\\beta)$ is $1$ so $\\beta$ is a divisor of $1$ and the factorization $p=\\alpha\\beta$ is trivial..." } { "Tag": [ "group theory", "abstract algebra", "superior algebra", "superior algebra unsolved" ], "Problem": "Let $G$ be a finite group, and $p$ be a prime dividing $\\left|G\\right|$. Let $P$ be a $p$-Sylow subgroup of $G$, and let $H$ be a subgroup of $G$ such that $N_{G}\\left(P\\right)\\subseteq H\\subseteq G$. Show that $H=N_{G}\\left(H\\right)$.\n\nHere, for any subset $T$ of $G$, we denote by $N_{G}\\left(T\\right)$ the normalizer of $T$ in $G$, defined by $N_{G}\\left(T\\right)=\\left\\{g\\in G\\mid gTg^{-1}=T\\right\\}$.\n\nEven a combinatorics \"genius\" like me has solved it, so don't expect it to be particularly challenging, but it is one of our nicer homework problems.\n\n Darij", "Solution_1": "I remember mathmanman telling me this problem and having came with a nice solution. Here it is: take $g\\in N_{G}(H)$, then $gPg^{-1}$ is a $p$-Sylow of $H$ (it is already one of $G$!), as well as $P$. By Sylow theorems, they are conjugated IN $H$. Then $h^{-1}g\\in N_{G}(P)$ and thus in $H$, thus $g$ is also in $H$. Done. Even if a \"genius of combinatorics\" like me solved it quickly, I really found it interesting and nice (otherwise, I would have already forgot about it...)." } { "Tag": [ "search", "combinatorics unsolved", "combinatorics" ], "Problem": "g is a simple graph with n vertices none of the vertices have n-1 degrees\u3002and suppose any two of the vertices have and only have an adjacent vertex.prove \r\nthat every vertex has the same degree", "Solution_1": "If I understand the question correctly then the conclusion can in fact be much stronger: there is no such graph. My interpretation of the condition: $ G$ is a simple graph such that for every two distinct vertices $ u,v$ there is precisely one vertex which is adjacent to both $ u$ and $ v$ (this condition holds regardless of whether or not $ u$ and $ v$ are themselves adjacent. Since $ G$ is simple, $ z$ must be distinct from $ u$ and $ v$).\r\n\r\nThe only graphs which satisfy this condition are \"windmills\": graphs which are made up of triangles all fused at a single vertex. In other words, those are graphs where one vertex $ z$ is connected to all other vertices in the graph, which in turn are hooked up in pairs. \r\n\r\nIn particular, these graphs do have a vertex of degree $ n\\minus{}1$ where $ n$ is the number of vertices. If you exclude those, as this question seems to do, there's nothing left.\r\n\r\nI don't know any really \"elementary\" proof of the fact I quoted above. It's a nice example of a problem that can be effectively handled by algebraic methods. I'm willing to bet it was discussed here before, but if needed I can present a complete proof.", "Solution_2": "When you say \"handled by algebraic methods,\" do you mean by an eigenvalue computation? I can sort of see how the proof would go, but I'm curious about the details.", "Solution_3": "The friendship theorem: http://www.artofproblemsolving.com/Forum/viewtopic.php?search_id=132046071&t=48500\r\nhttp://www.artofproblemsolving.com/Forum/viewtopic.php?search_id=132046071&t=19197\r\nhttp://www.artofproblemsolving.com/Forum/viewtopic.php?t=55513" } { "Tag": [ "number theory", "Divisibility" ], "Problem": "Prove that for any set of distinct positive integers $n_1, n_2, \\dots, n_k$ there exists a set of integer coefficients $a_1, a_2, \\dots, a_k$ such that\n$\\sum_{i=1}^k a_i {n_i p\\choose p}$ is divisible by $p^{2k-1}$ for all large enough primes $p$.\n\nExamples:\n$2 {p\\choose p} - {2p\\choose p}$ is divisible by $p^3$ for all primes $p\\geq 3$.\n$12{p\\choose p} - 9{2p\\choose p} + 2{3p\\choose p}$ is divisible by $p^5$ for all primes $p\\geq 7$.\n$60{p\\choose p} - 54{2p\\choose p} + 20{3p\\choose p} - 3{4p\\choose p}$ is divisible by $p^7$ for all primes $p\\geq 11$.\n$840{p\\choose p} - 840{2p\\choose p} + 400{3p\\choose p} - 105{4p\\choose p} + 12{5p\\choose p}$ is divisible by $p^9$ for all primes $p\\geq 3$.", "Solution_1": "[url=https://www.artofproblemsolving.com/community/user/16261]Rust[/url] and I have proved this type of divisibilities in general in the preprint [url=http://arxiv.org/abs/1602.02632]On p-adic approximation of sums of binomial coefficients[/url]." } { "Tag": [ "trigonometry", "integration", "function", "calculus", "real analysis", "real analysis unsolved" ], "Problem": "Prove that the series $\\sum_{n\\geq1}\\frac{e^{i \\sqrt n}}{n}$ converges", "Solution_1": "Aplying the condensation test we get that $\\sum_{n\\ge 1}\\frac{e^{i\\sqrt n}}{n}$convrges iff $\\sum_{k\\ge 1}2^k \\frac{e^{i(\\sqrt 2)^k}}{2^k}=\\sum_{k\\ge 1}e^{i(\\sqrt 2)^k}$onverges. The last sum can be splited into $\\sum \\sin \\sqrt n$ and $\\sum \\cos \\sqrt n$ which diverges since $(\\sin{\\sqrt{n}})_n$ is dense in $[0,1]$ and , hence, do not tend to $0$. Am I wrong? :?", "Solution_2": "Are you sure that condensation test can be applied :?: I think all summands need to be positive", "Solution_3": "For the condensation test, the terms have to be positive and decreasing. It's a test for absolute convergence, and the convergence here is not absolute.\r\n\r\nNote that $\\int_1^\\infty \\frac{e^{i\\sqrt{x}}}{x}\\,dx$ converges (Integrate by parts to get an equal absolutely convergent integral). We can show that $\\left|\\frac{e^{i\\sqrt{n}}}{n}-\\int_n^{n+1}\\frac{e^{i\\sqrt{x}}}{x}\\,dx\\right| =O(n^{-\\frac32})$, so the series differs from a known convergent series by an absolutely convergent series, and the original series converges.\r\n\r\nThere are some details to fill in- I might get back to this later.", "Solution_4": "The [hide=\"full version\"]\n\nLemma 1: $\\int_1^\\infty \\frac{e^{i\\sqrt{x}}}{x}\\,dx$ converges.\n[hide=\"Proof\"]$\\int_1^M \\frac{e^{i\\sqrt{x}}}{x}\\,dx=\\int_1^M \\frac{e^{i\\sqrt{x}}}{2\\sqrt{x}}\\cdot\\frac2{\\sqrt{x}}\\,dx$\nIntegrating by parts (integrate the first term, differentiate the second),\n$\\int_1^M \\frac{e^{i\\sqrt{x}}}{x}\\,dx= \\left[-ie^{i\\sqrt{x}}\\cdot \\frac2{\\sqrt{x}}\\right]_{x=1}^{x=M} -\\int_1^M-ie^{i\\sqrt{x}}\\cdot-x^{-\\frac32}\\,dx$\nBoth terms converge as $M\\to\\infty$; the first converges because the function we are evaluating goes to zero, and the second converges by comparison to the absolutely convergent integral $\\int_1^M x^{-\\frac32}\\,dx$.[/hide]\n\nCorollary: The series $\\sum_{n=1}^\\infty \\int_n^{n+1} \\frac{e^{i\\sqrt{x}}}{x}\\,dx$ converges.\n\nLemma 2 (Error estimate for a Riemann sum): If $f$ is differentiable on an interval $I=[a,b]$ and $|f'(x)|\\le M$ on $I$, $\\left|\\int_a^b f(x)\\,dx-(b-a)f(a)\\right|\\le \\frac M2(b-a)^2$. For this problem, allow $f$ to be complex-valued.\n[hide=\"Proof\"]Let $g(x)=f(x)-f(a)$, so $\\int_a^b g(x)\\,dx=\\int_a^b f(x)\\,dx-(b-a)f(a)$. If $\\int_a^b g(x)\\,dx=e^{i\\alpha}K$ for some positive real number $K$, let $h(x)=e^{-i\\alpha}g(x)$, so $\\int_a^b h(x)\\,dx=K$ is a positive real number.\nDefine $H(x)=\\int_a^x \\text{Re}(h(t))\\,dt$. Taylor's theorem applied to $H$ says that $K=H(b)=H(a)+(b-a)(\\text{Re}(h(a)))+\\frac{(b-a)^2}{2}\\cdot \\text{Re}(h'(x))$\n$=\\frac{(b-a)^2}{2}\\cdot \\text{Re}(h'(x))$ for some $x\\in I$. Since $\\text{Re}(h'(x))\\le |h(x)|\\le M$, we have $K\\le \\frac M2(b-a)^2$ as desired.[/hide]\n\nBy this estimate,\n$\\left|\\int_n^{n+1}\\frac{e^{i\\sqrt{x}}}{x}\\,dx-\\frac{e^{i\\sqrt{n}}}{n} \\right|\\le \\max_{n\\le x\\le n+1}\\left|\\frac{\\frac{d}{dx}\\frac{e^{i\\sqrt{x}}}{x}}{2}\\right| \\le \\max_{n\\le x\\le n+1}\\left|\\frac{ie^{i\\sqrt{x}}}{4x^{\\frac32}} -\\frac{e^{i\\sqrt{x}}}{2x^2}\\right|$\n$\\left|\\int_n^{n+1}\\frac{e^{i\\sqrt{x}}}{x}\\,dx-\\frac{e^{i\\sqrt{n}}}{n} \\right|\\le \\frac1{4n^{\\frac32}}+\\frac1{2n^2}$\nDefine $e_n=\\int_n^{n+1}\\frac{e^{i\\sqrt{x}}}{x}\\,dx-\\frac{e^{i\\sqrt{n}}}{n}$. By comparison to the absolutely convergent series $\\sum_n \\frac1{4n^{\\frac32}}+\\frac1{2n^2}$, $\\sum_n e_n$ converges.\n\nLemma 3: If $\\sum_n a_n$ converges to $A$ and $\\sum_n b_n$ converges to $B$, $\\sum_n a_n+b_n$ converges to $A+B$.\n[hide=\" Easy Proof\"]\nLet $A_n=\\sum_{i=1}^n a_i$ and $B_n=\\sum_{i=1}^n b_i$. By hypothesis, $A_n\\to A$ and $B_n\\to B$. It is clear that $\\sum_{i=1}^n a_i+b_i=A_n+B_n$. By the standard arithmetic theorem on limits, this converges to $A+B$ and we are done.[/hide]\n\nLet $I_n=\\int_n^{n+1} \\frac{e^{i\\sqrt{x}}}{x}\\,dx$.\nPutting the pieces together,\n$\\sum_{n=1}^\\infty \\frac{e^{i\\sqrt{n}}}{n}=\\sum_{n=1}^\\infty I_n-e_n$. Since $\\sum_n I_n$ converges by the corollary to Lemma 1 and $\\sum_n -e_n$ converges by the remarks following Lemma 2, their sum $\\sum_n I_n-e_n$ converges by Lemma 3 and we are done.[/hide]" } { "Tag": [ "group theory", "abstract algebra", "ratio", "superior algebra", "superior algebra unsolved" ], "Problem": "The question:\r\n\r\nIf G is the additive group Q/Z, what are the elements of the subgroup G(2)? Of G(P) for any positive prime P?\r\n\r\nWhere G(n)={a e G| |a| = n^(k) for some k is greater than or equal to 0}...That is the set of all a in G, s.t. the order of a is some power of n. (But since it is the additive group, I suppose it would just a be a multiple of n)\r\n\r\nHow do I even begin with this? Aren't the elements of Q/Z sets? The collections of right cosets? and don't they have infinite order?....\r\n\r\nThanks..^^", "Solution_1": "$ \\mathbb{Q}/\\mathbb{Z}$ can be thought of as the set of rational numbers in $ [ 0, 1 )$ where the group operation is addition $ \\bmod 1$. It is true that the quotient is constructed abstractly as a collection of cosets, but the above is also perfectly valid.\r\n\r\nEvery element of this group does in fact have finite order (and in fact this group is the torsion subgroup of the [url=http://en.wikipedia.org/wiki/Circle_group]circle group[/url]). For example, $ \\frac {1}{2}$ has order $ 2$. (This example should help you figure out the question pretty well now.)", "Solution_2": "The operation is addition mod 1? Wouldn't that mean that 1/2 is the only order 2 element in G(2)? 1/2 + 1/2 = 1 ... since 1 is the only number to divide 1...", "Solution_3": "Did you mean ration number in (0,1]? or really [0,1) ...anyway...I think I understand...actually, any element n/2 when n is an integer will have order 2... But what is so special about order p, where p is a prime?", "Solution_4": "[quote=\"Frank1987\"]Did you mean ration number in (0,1]? or really [0,1)[/quote]\nIt doesn't matter; we just change the name of the identity, which is $ 1$ in the first range and $ 0$ in the second.\n\n[quote=\"Frank1987\"]any element n/2 when n is an integer will have order 2... [/quote]\nYes, but these elements are all identified with each other under the quotient. There are only two distinct such elements; the identity and $ \\frac {1}{2}$, and the identity has order $ 1$. Now: what are the elements of order $ 3$?\n\n[hide=\"Answer\"] $ \\frac{1}{3}$ and $ \\frac{2}{3}$. [/hide]\n[quote=\"Frank1987\"] But what is so special about order p, where p is a prime?[/quote]\r\nNothing here; elements of every order exist. But usually it is easier to understand the elements of prime order, and in some sense one can build up elements of other order from them." } { "Tag": [ "limit", "calculus", "integration", "calculus computations" ], "Problem": "compute:\r\n\r\n$ \\lim_{n \\to \\plus{} \\infty} \\sum_{k \\equal{} 1}^{n} coh(\\frac {1}{\\sqrt {n \\plus{} k}}) \\minus{} n$\r\n\r\n\r\nwith $ coh(x) \\equal{} \\frac {e^x \\plus{} e^{ \\minus{} x}}{2}$", "Solution_1": "This doesn't look like an infinite series.", "Solution_2": "It's corrected.", "Solution_3": "$ \\sum_{k \\equal{} 1}^n Cosh(\\frac {1}{\\sqrt {k \\plus{} n}}) \\minus{} n \\equal{} \\sum_{k \\equal{} 1}^n \\sum_{j \\equal{} 0}^{\\infty}\\frac {1}{(2j!)(k \\plus{} n)^j} \\minus{} n$\r\n\r\n$ \\equal{} \\sum_{k \\equal{} 1}^n \\frac {1}{2(k \\plus{} n)} \\plus{} \\frac {1}{24(k \\plus{} n)^2} \\plus{} ...$\r\n$ \\equal{} \\frac {1}{2n}\\sum_{k \\equal{} 1}^n \\frac {1}{\\frac {k}{n} \\plus{} 1} \\plus{} \\frac {1}{24n}\\sum_{k \\equal{} 1}^n \\frac {1}{n(\\frac {k}{n} \\plus{} 1)^2} \\plus{} ...$\r\n\r\nAs $ n \\rightarrow \\infty$, we see we have alternate representations of riemann integrals\r\n\r\nAll of the terms except the first will be dominated by the factors of n in the denominator, making them all goto zero. So all we are concerned with is the first term which is\r\n\r\n$ \\lim_{n\\to \\plus{}\\infty} \\frac {1}{2n}\\sum_{k \\equal{} 1}^n \\frac {1}{\\frac {k}{n} \\plus{} 1} \\equal{} \\frac {1}{2}\\int_0^1\\frac {1}{1 \\plus{} x}dx \\equal{} ln{\\sqrt {2}}$\r\n$ \\therefore \\lim_{n \\to \\plus{} \\infty} \\sum_{k \\equal{} 1}^{n} coh(\\frac {1}{\\sqrt {n \\plus{} k}}) \\minus{} n \\equal{} ln{\\sqrt {2}}$" } { "Tag": [ "inequalities", "number theory unsolved", "number theory" ], "Problem": "Let there $ S(N)$ be the sum of digits , for every $ N \\in \\mathbb{N}$\r\n\r\nExample : $ S(\\overline{a_1a_2 \\dots a_n}) \\equal{} a_1 \\plus{} a_2 \\plus{} \\dots \\plus{} a_n$ .\r\n\r\n\r\nProve that : $ \\boxed{\\frac {S(8N)}{S(N)} \\geq \\frac {1}{8}}$ .", "Solution_1": "obviosly, $ S(10^k N)\\equal{}S(N)$. So i get\r\n$ S(1000N)\\equal{}S(800N\\plus{}80N\\plus{}80N\\plus{}8N\\plus{}8N\\plus{}8N\\plus{}8N\\plus{}8N)$\r\nalso, $ S(a\\plus{}b) \\leq S(a)\\plus{}S(b)$.\r\nso i get $ S(N)\\leq 8S(8N)$ and hence the inequality holds.\r\n\r\nNaphthalin" } { "Tag": [ "inequalities", "inequalities solved" ], "Problem": "Let A,B,C be three angles of triangle ABC with A,B,C \\in [0, \\pi /2] \r\nProve that : \\sum 1/cos(A/2) - \\sum tg(A/2) \\geq \\sqrt 3", "Solution_1": "just the kinda problems i need to solve these days (with no time for maths :D :D )\r\nput X=90 - A/2 and similarly for the others.we see that X,Y,Z are also the angles of a triangle (with each less than 90)\r\ninequality equivalent to proving\r\n \\sum tan (X/2) \\geq \\sqrt 3\r\nisnt this known :D :D" } { "Tag": [ "geometry", "trapezoid", "parallelogram" ], "Problem": "Given trapezoid $ABCD$, $E$ and $F$ are points on $BC$ such that $DE\\parallel AB$ and $AF\\parallel DC$. Let $G$ be the intersection of $DE$ and $AF$. The area of parallelogram $ABED$ is $36$. If $AD=5$, $AG=3$ and $DG=4$, find the area of trapezoid $ABCD$.", "Solution_1": "Is $BC\\parallel{AD}$? :?", "Solution_2": "[quote=\"riddler\"]Is $BC\\parallel{AD}$? :?[/quote]\r\nThat was the only way I was able to draw the diagram.", "Solution_3": "now I realise. :blush:", "Solution_4": "[hide=\"I think...\"]Using the given information, we find that $\\triangle AGD$ is a $3-4-5$ right triangle. Because of congruent angles, $\\triangle EFG$ is similar to $\\triangle AGD$.\n\nThe height of $ABED$ is $7.2$. By drawing line segment $AE$, we can divide $ABED$ in half. So the area of $\\triangle AED$ is $18$, and we can find that $EG$ is $8$. Thus, the length of $EF$ is $10$.\n\nThe area of the trapezoid is $\\frac{1}{2}h(b_1+b_2)$. So it is $\\frac{1}{2}\\cdot7.2(5+10+5+5)=\\boxed{90}$.[/hide]", "Solution_5": "[quote=\"catcurio\"][quote=\"riddler\"]Is $BC\\parallel{AD}$? :?[/quote]\nThat was the only way I was able to draw the diagram.[/quote]Yeah, I forgot to mention that :blush: \n\n[quote=\"catcurio\"][hide=\"I think...\"]Using the given information, we find that $\\triangle AGD$ is a $3-4-5$ right triangle. Because of congruent angles, $\\triangle EFG$ is similar to $\\triangle AGD$.\n\nThe height of $ABED$ is $7.2$. By drawing line segment $AE$, we can divide $ABED$ in half. So the area of $\\triangle AED$ is $18$, and we can find that $EG$ is $8$. Thus, the length of $EF$ is $10$.\n\nThe area of the trapezoid is $\\frac{1}{2}h(b_1+b_2)$. So it is $\\frac{1}{2}\\cdot7.2(5+10+5+5)=\\boxed{90}$.[/hide][/quote]That's correct! ;)" } { "Tag": [ "number theory open", "number theory" ], "Problem": "Suppose that $ a$ is a positive integer. Is it always true that there is a prime $ p$ such that $ p^2\\mid a^{p\\minus{}1}\\minus{}1$?", "Solution_1": "Heuristic arguments suggest there this statement is likely to be true.\nBut it may be very hard to prove it rigorously.\n\nSee also\nhttp://oeis.org/A039951\nhttp://oeis.org/A096082" } { "Tag": [], "Problem": "where were u born?", "Solution_1": "CHINA!!!!!!!!!!!!!!!", "Solution_2": "i am glad with the added options!\r\nCHINA!", "Solution_3": "INDIA!!!! \r\n[hide=\"Indian Song\"]Welcome to India where the cows eat hey,\nand we drive auto-richshaws everyday,\nGoat Meats, Yummy Sweets, Wild Monkeys roaming\nThe Roosters Dont Crow till 5 in the morning[/hide]", "Solution_4": "hmm only one from west europe (belgium)", "Solution_5": "i'm from england!", "Solution_6": "US! Illinois.", "Solution_7": "CCCCCCCCCCCCCCHINA!!!!!!!!!", "Solution_8": "Hmm, I'm the first Russian! (AKA Soviet)", "Solution_9": "[quote=\"DarkKnight\"]US! Illinois.[/quote]\r\n\r\nWoot! Go Illinois!", "Solution_10": "Hi, Sponge. I am also Russian. Where you from? Myth, who posts on AoPS is russian and imo gold medalist.", "Solution_11": "from Seoul, Korea.", "Solution_12": "I posted this on the other thread, but....ARIZONA!!!!!! Parents born in Taiwan, though.", "Solution_13": "[quote=\"xxreddevilzxx\"]from Seoul, Korea.[/quote]\r\nWhee~ Seoul! Me, too..", "Solution_14": "cool. when did you come to america?", "Solution_15": "China! But I live in Illinois right now.", "Solution_16": "holdin it down in South Carolina, USA.", "Solution_17": "US of A...more specifically, good old INDIANA!!!", "Solution_18": "South Cackalackey", "Solution_19": "I was born in CHINA! :cool:", "Solution_20": "[quote=\"hello\"]Hi, Sponge. I am also Russian. Where you from? Myth, who posts on AoPS is russian and imo gold medalist.[/quote]\r\nI was born in Kiev in the Soviet days. Now, I live in Massachusetts. Meh, I'm Russian by ancestry and first language (first as in first learned, not better volcabulary), Ukranian by former/partial citizenship, and Soviet/Ukranian by birth. Now, I'm living in America. I'm reminded of a Chinese immigrant from Australia friend of mine.", "Solution_21": "turkey-istanbul\r\nthis is better than my post and poll.", "Solution_22": "summerville, SC", "Solution_23": "i'm from japan, moved to the us when i was 8", "Solution_24": "Wow, not many people in the \"rest of north of america\" I'm sure there's gotta be some others!", "Solution_25": "[quote=\"MathFiend\"]US of A...more specifically, good old INDIANA!!![/quote]\r\nDo you still live in Indiana and if you do what school do you go to?\r\nI was wondering if I might know you from one of these competitions." } { "Tag": [ "LaTeX" ], "Problem": "(X with the power of 1 )(X with the power of 2)(X with the power of 3).......................(X with the power of 2n) what does this question wants ? I dun understand what is the power of 2n. And X with the power of 1 multiply X with the power of 2 isnt it X with the power of 1+2 that is X with the power of 3?", "Solution_1": "could you rephrase that and use the \"^\" sign? \r\ni have no idea what u r talking about and it would be helpful if u replaced \"with the power of\" with \"^\" By the way, \"^\" basically translates into \"to the power of\"\r\n\r\n3^4 means\r\n3 to the power of 4\r\n3*3*3*3 \r\n81", "Solution_2": "The first one: $ (x^1)(x^2)(x^3)(x^4)...(x^{2n})$. This is equal to $ x^{1\\plus{}2\\plus{}3\\plus{}...\\plus{}2n}\\equal{}x^{n(2n\\plus{}1)}$.\r\nThe second one: $ x^1\\times x^2$, which does equal $ x^3$.\r\n\r\nIn the meantime, I'd suggest going to the LaTeX guide by [url=http://www.artofproblemsolving.com/Wiki/index.php/LaTeX]clicking here[/url]", "Solution_3": "What is latex? I dun understand the information stated there", "Solution_4": "[quote=\"shaun ong\"]What is latex? I dun understand the information stated there[/quote]\r\n\r\nLaTeX is a programming language that is used by mathematicians to express math symbols in an easier way. \r\n\r\nYou can say,\r\n\r\n(x+y^5-2x-y)/x+2z) and we'll have no idea what you're saying(or if we do, then we'll have to spend 5 painful minutes trying to unravel the mess). \r\n\r\nIn LaTeX, it can be expressed as \r\n\r\n$ \\frac{(x\\plus{}y^{5\\minus{}2x}\\minus{}y)}{x\\plus{}2z}$.\r\n\r\nClarity and easy to understand.", "Solution_5": "there is a LATEX program where you can learn it" } { "Tag": [ "inequalities", "algebra", "polynomial", "inequalities proposed" ], "Problem": "i)$a,b,c >0$.Prove that:\r\n $\\frac{ab(a+b)}{a^3+b^3+2c^3}+\\frac{bc(b+c)}{b^3+c^3+2a^3}+\\frac{ac(a+c)}{a^3+c^3+2b^3} \\leq \\frac{3}{2}$\r\nii)Find the best contanst $k$ such that the inequality hold:\r\n $\\sum \\frac{ab(a+b)}{a^3+b^3+kc^3} \\leq \\frac{6}{k+2}$ $(a,b,c >0)$", "Solution_1": "SOS again... Come on...", "Solution_2": "How to solve this one by SOS?I have some trouble.", "Solution_3": ":rotfl: ii) The maximum of $k\\,$ is the smaller positive root of the following polynomial:\r\n\r\n$23\\,k^6+374\\,k^5-3733\\,k^4+7456\\,k^3+1156\\,k^2-8192\\,k+3888,$\r\n\r\nthat is, $2.77948796...$", "Solution_4": "How to do the first one by SOS?" } { "Tag": [], "Problem": "\u725b\u987f\u5b9a\u7406\uff1aABCD\u662f\u5706\u5185\u63a5\u56db\u8fb9\u5f62\uff0c\u76f4\u7ebfAB\u4e0e\u76f4\u7ebfCD\u4ea4\u4e8eE\uff0c\u76f4\u7ebfBC\u4e0e\u76f4\u7ebfAD\u4ea4\u4e8eF\uff0cM,N,Q\u5206\u522b\u4e3aAC,BD,EF\u7684\u4e2d\u70b9\uff0c\u6c42\u8bc1\uff1aM,N,O\u5171\u7ebf.", "Solution_1": "See [url=http://www.mathlinks.ro/Forum/viewtopic.php?t=20264]http://www.mathlinks.ro/Forum/viewtopic.php?t=20264[/url].", "Solution_2": "\u5f88\u5947\u602a. \u6211\u7684\u4e00\u672c\u4e66\u4e0a\u8bf4: \u8fd9\u4e09\u70b9\u7684\u8fde\u7ebf\u53eb\u505a\"\u9ad8\u65af\u7ebf\", \u53ef\u4e3a\u4ec0\u4e48\u9898\u76ee\u53eb\u725b\u987f\u5b9a\u7406? \r\n\r\nBTW: \u5173\u4e8e\u5706\u7684\u6761\u4ef6\u53ef\u4ee5\u53bb\u6389. \u9898\u76ee\u7684\u7ed3\u8bba\u5bf9\u4e8e\u4efb\u610f\u51f8\u56db\u8fb9\u5f62\u90fd\u662f\u6210\u7acb\u7684.", "Solution_3": "\u5bf9\u554a,\u6211\u591a\u7ed9\u4e86\u4e00\u4e2a\u6761\u4ef6 :blush: :blush: :blush:", "Solution_4": "\u4f3c\u4e4e\u5927\u5bb6\u73fe\u5728\u90fd\u5f88\u5fd9. \u9019\u88e1\u53c8\u518d\u51b7\u6e05\u8d77\u4f86.", "Solution_5": "\u6d88\u70b9\u5427. \u628a\u7ebf\u6bb5\u6bd4\u6362\u6210\u9762\u79ef\u6bd4, \u518d\u7528\u4e00\u6b21\u5b9a\u6bd4\u5206\u70b9\u516c\u5f0f.", "Solution_6": "\u4e8b\u5b9e\u4e0a\u53ef\u4ee5\u7528Menelaus\u5b9a\u7406\u89e3\u51b3\u3002", "Solution_7": "[quote=\"dingdongdog\"]\u4e8b\u5b9e\u4e0a\u53ef\u4ee5\u7528Menelaus\u5b9a\u7406\u89e3\u51b3\u3002[/quote]\r\nHow?", "Solution_8": "\u8fd9\u9898\u53ef\u4ee5\u7ed3\u5408\u9762\u79ef\u6cd5\u89e3\u51b3\u3002\r\n\u6b32\u8bc1O\\M\\N\u5171\u7ebf\uff0c\u53ea\u9700\u8bc1S(FMN)=S(EMN)\r\n\u53d6AB\u4e2d\u70b9P,S(FMN)=S(FPM)+S(FPN)+S(PMN)=S(BPM)+S(APN)+S(PMN)=S(ANMB)\r\n=S(ADMB)/2=S(ABCD)/4\r\n\u540c\u7406S(EMN)=S(ABCD)/4\r\n\u4ece\u800cS(FMN)=S(EMN),\u547d\u9898\u5f97\u8bc1\u3002" } { "Tag": [ "algebra", "polynomial", "logarithms", "function", "rational function", "number theory unsolved", "number theory" ], "Problem": "prove that there aren't polynominal $P ,Q$ such that:$\\pi(n)=\\frac {P(x)}{Q(x)}$", "Solution_1": "Are you referring to polynomials $P, Q$ with integer coefficients? Or the coefficients can take on any real values?", "Solution_2": "It doesn't really matter. The Prime Number Theorem says that $\\pi(n)$ increases like $\\frac n{\\ln n}$, so it couldn't be represented as a rational function in $n$, since such functions behave like polynomials as $n\\to\\infty$." } { "Tag": [ "probability", "floor function", "probability and stats" ], "Problem": "If m balls are distributed to a boys and m girls. Find the probability that even no. of balls are distributed to boys :(", "Solution_1": "[hide=\"hint\"]The first ball is given to one of $ m \\plus{} a$ people, what's the probability that it will be given to one of the $ m$ girls? The second ball is given to one of the remaining $ m \\plus{} a \\minus{} 1$ people, what's the probability that it will be given to one of the remaining $ m \\minus{} 1$ girls? Keep going until there are no balls left. These $ m$ events are pairwise independent; how does that help?[/hide]", "Solution_2": "# ways to distribute balls:\r\n\r\n$ _{a\\plus{}m}C_m \\equal{} \\frac{(a\\plus{}m)!}{a!m!}$\r\n\r\n# ways to give boys 2k balls:\r\n\r\n$ _a C_{2k} \\cdot _m C_{m\\minus{}2k} \\equal{} \\frac {a!m!}{2k! (a\\minus{}2k)! 2k! (m\\minus{}2k)!}$\r\n\r\nprob of giving boys 2k balls:\r\n\r\n$ P(X\\equal{}2k) \\equal{} \\left(\\frac{a!m!}{k!}\\right)^2 \\frac{1}{(a\\minus{}2k)!(m\\minus{}2k)!(a\\plus{}m)!}$\r\n\r\n$ P(X even) \\equal{} \\sum_{k\\equal{}0}^{\\lfloor \\frac{a}{2} \\rfloor} P(X\\equal{}2k)$\r\n\r\ni dont see this cleaning up nicely though...", "Solution_3": "ans.\r\n\r\n1/2{(a+b)^m-(a+b)^n}/a+b)^m" } { "Tag": [ "inequalities", "limit", "inequalities unsolved" ], "Problem": "Let $ a,b,c$ are positive real numbers and $ n \\geq1$ prove that\r\n\r\n\r\n$ \\frac{a^{2}}{b}\\plus{}\\frac{b^{2}}{c}\\plus{}\\frac{c^{2}}{a}\\geq3\\frac{a^{n}\\plus{}b^{n}\\plus{}c^{n}}{a^{n\\minus{}1}\\plus{}b^{n\\minus{}1}\\plus{}c^{n\\minus{}1}}$", "Solution_1": "[quote=\"mestav\"]Let $ a,b,c$ are positive real numbers and $ n \\geq1$ prove that\n\n\n$ \\frac {a^{2}}{b} \\plus{} \\frac {b^{2}}{c} \\plus{} \\frac {c^{2}}{a}\\geq3\\frac {a^{n} \\plus{} b^{n} \\plus{} c^{n}}{a^{n \\minus{} 1} \\plus{} b^{n \\minus{} 1} \\plus{} c^{n \\minus{} 1}}$[/quote]\r\n\r\n(1), we can prove that\r\n\r\n\\[ \\frac {{a^{n \\plus{} 1} \\plus{} b^{n \\plus{} 1} \\plus{} c^{n \\plus{} 1} }}{{a^n \\plus{} b^n \\plus{} c^n }} \\ge \\frac {{a^n \\plus{} b^n \\plus{} c^n }}{{a^{n \\minus{} 1} \\plus{} b^{n \\minus{} 1} \\plus{} c^{n \\minus{} 1} }}\\]\r\nholds for any $ a,b,c > 0,n\\ge1$.\r\n\r\n(2), we can prove that\r\n\r\n\\[ \\mathop {\\lim }\\limits_{n \\to \\plus{} \\infty } \\frac {{a^n \\plus{} b^n \\plus{} c^n }}{{a^{n \\minus{} 1} \\plus{} b^{n \\minus{} 1} \\plus{} c^{n \\minus{} 1} }} \\equal{} \\max \\left\\{ {a,b,c} \\right\\}\\]\r\nholds for any $ a,b,c > 0$\r\n\r\n\r\nso, it's equivalent to prove that\r\n\\[ \\frac {a^{2}}{b} \\plus{} \\frac {b^{2}}{c} \\plus{} \\frac {c^{2}}{a}\\geq 3 \\max \\left\\{ {a,b,c} \\right\\}\\]\r\nbut this is not hold! :mad: \r\n\r\nso, the original inequality must have a counter example!\r\n\r\nI will try to find it...", "Solution_2": "OK, found, counter example:\r\n\r\n$ a \\equal{} 9, b \\equal{} 15, c \\equal{} 10, n \\equal{} 4$\r\n\r\n$ \\frac {a^{2}}{b} \\plus{} \\frac {b^{2}}{c} \\plus{} \\frac {c^{2}}{a} \\minus{} 3 \\cdot \\frac {a^4 \\plus{} b^4 \\plus{} c^4}{a^3 \\plus{} b^3 \\plus{} c^3} \\equal{} \\minus{} \\frac {55019}{114840} < 0$", "Solution_3": "by the way, when $ n \\equal{} 1,2,3$, inequality holds. I have proved that $ n \\equal{} 1,2$, who can prove that $ n \\equal{} 3$?\r\n\r\n$ \\frac {a^{2}}{b} \\plus{} \\frac {b^{2}}{c} \\plus{} \\frac {c^{2}}{a} \\minus{} (a \\plus{} b \\plus{} c) \\equal{} \\sum{\\frac {(a \\minus{} b)^2}{b}} \\ge 0$\r\n\r\n$ \\frac {a^{2}}{b} \\plus{} \\frac {b^{2}}{c} \\plus{} \\frac {c^{2}}{a} \\minus{} 3\\cdot \\frac {a^2 \\plus{} b^2 \\plus{} c^2}{a \\plus{} b \\plus{} c} \\equal{} \\sum{\\left(\\frac {1}{b} \\minus{} \\frac {1}{a \\plus{} b \\plus{} c}\\right)(a \\minus{} b)^2} \\ge 0$\r\n\r\nand ... $ n_{\\max}$ is ... ? :huh:" } { "Tag": [ "AMC", "AIME" ], "Problem": "This year I didn't take the AMC because basically nobody at my school could have qualified for AIME except possibly me with my special skill at guessing correctly without a clue. But, I want to know how I would have done so does anyone know of a link to a site with the questions? Thanks in advance!", "Solution_1": "No, but you could always buy the questions for $1 soon - [url]http://www.unl.edu/amc/d-publication/pubform.html[/url]" } { "Tag": [ "\\/closed" ], "Problem": "<= in the code is parsed to \r\n<\r\n=\r\neffectively disabling the code in the posts containing it. (>=, probably too, != no)\r\nCan we fix it?", "Solution_1": "I don't understand what's going on. Could you be more specific? What code? Asy code?", "Solution_2": "He's saying if we had something like\r\n\r\n[asy]real i; path p = (0,0)--(1,1); for(i=1; i<=10; ++i){ draw((p=shift((1,0))*p)); }[/asy]\r\n\r\nIt won't work because <= gets broken up on new lines. It seems to work for me though.\r\n\r\n\r\n[asy]real i; path p = (0,0)--(1,1); for(i=0; i<10; ++i){ draw((p=shift((1,0))*p)); }[/asy]", "Solution_3": "Oh, well that's just the display part (for the human eye :) ). It shouldn't break after < though, it's a good catch. I'll try to fix it.", "Solution_4": "Try to edit or quote Hamster1800 post without touching the code and resubmit. You'll get an error. The first time you post it works OK though and I accidentally discovered it when I tried to add a smile to a perfectly functioning post and got an error message. It took me half an hour to figure out what was going on :mad: :lol:", "Solution_5": "Ok, I've fixed that error, and also fixed the presentation for the for. Happy now? :P", "Solution_6": "[quote=\"Valentin Vornicu\"]Ok, I've fixed that error, and also fixed the presentation for the for.[/quote]\n:coolspeak: \n[quote] Happy now? :P[/quote]\r\nWay up, but I still have to multiply by 50 :heli:" } { "Tag": [ "geometry", "Euler", "invariant" ], "Problem": "Feuerbach's theorem:\r\nProve that in any triangle, the inscribed circle and the 3 exinscribed circles are tangent to Euler's circle.", "Solution_1": "i think i have a proof for the incircle touching euler's circle part,two of them to be precise.they are long ,i'll post em later.btw,i believe u do have the proofs ,mate :? ,and u just wanna know sum different solutions,dont ya??\r\ncheers", "Solution_2": "Well, the problem is that I remember having proved this, but I have no idea how.. :? So I have to say no, I don't currently have a proof. I could look in some books of course, but I hate reading proofs in books. :D", "Solution_3": "Okay .. let's see now... \r\n\r\nwe need two lemmas first:\r\n\r\n [b] Lemma 1 [/b]: Let A' be the middle of [BC] of triangle ABC. Using an inversion(grobber's favourite method :D ) T(A',k) (k is arbitrary) Euler circle of triangle ABC ( from which we take out point A') becomes a antiparalel with BC from A.\r\n\r\n[b] Proof of lemma 1 [/b]: It is well known that A' is on Euler circle. So using the inversion T(A',k) Euler circle( from which like I said we take out A') transforms itsself into a line d which is perpendicular on A'w, where w is the center of Euler circle. We know that w is the middle of HO (H- ortocenter and O - the center of the circumscribed circle). Let H' be the simetrical of H from A' . Then H' is diametrally opposite of A. \r\n\r\nIn the triangle HOH' the segment A'w is the middle line, so A'w || AH'. Because d is perpendicular on A'w is perpendicular on AH', which means that d is paralel with the tangent in point H' of the circumscriebed circle of triangle ABC. Let E be the point in with the tangent in H' intersects AB.\r\nThen m(AEH')=90-m(EAH')=m(BH'A)=n(ACB).\r\n\r\nSo we have that BC and EH' are antiparalel. Because d is paralel to EH' we get that d is antiparalel with BC.\r\n\r\n[b] Lemma 2 [/b]: Let C(I,r) be the incircle of triangle ABC and let C(I',r') be the exincribed circle corespunding to A. Let A' be the middle of BC and let A1 be the point in which the bisector of angle BAC intersects BC. If A\" is the projection of A on BC and F the projection of I on F and F' the projection of I' on BC then:\r\ni) BF=CF'\r\nii) A'F^2=A'A\"*A'A1\r\n\r\n[b] Proof of lemma 2 [/b]: For more clarity on your drawings lets consider that the triangle ABC has angle m(ABC)>90.\r\n\r\n[b] i) [/b]: If we denote F1=the projection of I on ABC, x=AF1, y=BF, z=CF, a=BC, b=CA, c=AB then:\r\nc=x+y\r\na=y+z\r\nb=z+x\r\nwe get that x+y+z=p which gives us that y=BF=p-b where p is the semiperimeter of ABC.\r\nusing the following notations: u=AF\", v=BF', alpha=CF' we get that\r\na=v+alpha\r\nb=u-alpha\r\nc=u-v\r\nwhich gives us that alpha=CF'=p-b so BF=CF'.\r\n\r\n[b] ii) [/b] from the bisector theorem we get that BA1/CA1=c/b from which we get that: BA1/(BA1+A1C)=c/(b+c) so BA1=ac/(b+c)\r\nso we get A'A1=a/2-ac/(b+c) or A'A1=a(b-c)/(2(b+c)) (*)\r\nfrom the right angled triangles AA\"B and AA\"C we get that :\r\nA\"A^2=AB^2-A\"B^2 and A\"A^2=AC^2-A\"C^2\r\nfrom the last two equalities we get that:\r\nb^2-c^2=(A\"C+A\"B)(A\"C-A\"B) or b^2-c^2=a(a+2A\"B) so we get that \r\nA\"B=(b^2-c^2-a^2)/(2a); because A'A\"=A'B+A\"b we get that A'A\"=(b^2-c^2)/(2a) (**)\r\nBecause BF=p-b we get that A'F=A'B-BF=a/2-(p-b) so A'F=(b-c)/2 (***)\r\nso now if we look at (*), (**) and (***) we get that:\r\nA'A1*A'A\"=a*(b-c)/(2(b+c))* (b^2-c^2)/(2a)=(b-c)^2/4=A'F^2\r\n\r\nNow we are done with the lemmas, back to the problem :D.\r\n\r\n[b] Proof of the problem [/b]We keep the denotes we made in lemma 2 and now we transfor that drawing using an inversion of pole A' and we use the power k=p ( the power of the point A' from the inscribed circle) so k=A'F^2.\r\n\r\nUsing this inversion the circle C(I,r) is transformed into itself. Because A'F=A'F' we also get that the circle C(I',r') is transformed into itself.\r\n\r\nThe equality A'A1*A'A\"=A'F^2 (lemma 2) shows that the point A\" ( which is on Euler's circle) is transformed into A1. Using lemma 1 we get that EUler circle (from which we take out A') is transformed in the straight line d which is antiparalel with BC which goes through A1. Because d pases through A1 and because it is antiparalel with BC we get that this is the 2nd exterior common tangent of circles C(I,r) and C(I',r'). d the inverse of Euler circle (from which we take out A') because it is tangent to C(I,r) and C(I',r') we get that also Euler circle will be tangent to C(I,r) and C(I',r') ( they are the invariants in the considered inversion)\r\n\r\nLet E (E' respectively) be the point in which d is tangent to circle C(I,r) (C(I',r') respectively). Then we have that Euler's circle is tangent to circle C(I,r) in the transformation of point E, and to the circle C(I',r') in the transformation of point E' with the considered inversion. We do the same to prove that Euler's circle is tangent to the exinscriebed circles coresponding to B and C.\r\n\r\nProof done! Hope you will enjoy it! :D", "Solution_4": "Holy Mother of Christ! That is enormous! :D But it works anyway. I'll try to remember what I came up with.", "Solution_5": "hmm.. grobber that was the exact thing that I said too :D\r\n\r\nguess that this is indeed(hopefully) the first and last solution I will give with inversions :D\r\n\r\nI really hate geometrical transformations! they are ingenious sometimes and also long sometimes :D", "Solution_6": "there is geometry book which tells a lot of Euclidean geometrical knowledge including Feuerbach's theorem.\r\n\r\n\"Modern Geometry\"\r\nby Roger A. Johnson\r\n(C) Houghton Mifflin 1929", "Solution_7": "well refer brindon's geometry series , though I don't have it", "Solution_8": "View: \n[i]Nathan Altshiller-Court[/i]: [b]COLLEGE GEOMETRY [/b](second Edition); p.105; Theorem 215.", "Solution_9": "Dear Mathlinkers, \nreading some old message, I found this one... \nIn 2003, I found an enterely synthetic proof that I put on my website in 2007. \nhttp://perso.orange.fr/jl.ayme (vol. 1 le th\u00e9or\u00e8me de Feuerbach) in French \nor : revistaoim directed by professor Francisco Bellot Rosado (in spanish). Revista Escolar de la Olimpiada \nIberoamericana de Matematica (Espagne) 26 (2006): http://www.oei.es/oim/revistaoim/index.html\nSincerely \nJean-Louis" } { "Tag": [ "college contests" ], "Problem": "I was wondering how many people here are going to participate in this year's competition... so here I am, the first one :). Anybody else?", "Solution_1": "Oh well, never mind :(. If there's anybody interested, here are the problems:\r\n\r\nhttp://www.osu.cz/fpr/kma/dokumenty/vjaimc/year2007.html\r\n\r\nCategory 1 wasn't too hard, five people managed to score full points. I'm not quite competent to judge the difficulty of category 2 problems though.", "Solution_2": "http://jarnik.osu.cz/index.php?page=home", "Solution_3": ":maybe: I think there will be none from Indonesia. (Sorrry if I do a junk, can the moderators make a forum for the junkers, about any news besides the topic that has been in the forum). But, Votech jarnik 2007 has ended. I hope I can join the competition in 2008, if I had a money :rotfl: :(", "Solution_4": "What was the turnout and # of teams for this competition?", "Solution_5": "[quote=\"jytang\"]What was the turnout and # of teams for this competition?[/quote]\r\n\r\nThis can be seen from the page with (non-zero) results:\r\nhttp://jarnik.osu.cz/index.php?page=year2007" } { "Tag": [ "inequalities" ], "Problem": "How do you graph the inequality x+y+|z| <= 3?", "Solution_1": "First, I would like to know, how do you draw a 3-d graph on 2-d paper?\r\n\r\nIf you have a way to do that, the best way would probably be to graph it for positive y, then take that graph, plus the graph resulting from it being reflected across the x-y plane, and you're done.", "Solution_2": "Do you have a graphing program on your computer?\r\nIf so you can plug it in, get the intercepts, and transfer it to paper.\r\nI'd like to know how to gragh 3D on 2D paper too. :D" } { "Tag": [ "algebra", "polynomial", "calculus", "integration", "search", "number theory proposed", "number theory" ], "Problem": "Prove or disprove that: if $P(x)\\in Z[x]$ and for each $n\\in N*$ there exist an integer $k\\in N*$ such that $n|P(k)$ then $P(x)$ has at least one integeral root. :) \r\n\r\n\r\nit's just my idea!", "Solution_1": "See http://www.mathlinks.ro/Forum/viewtopic.php?t=110231 .", "Solution_2": "I posted it some time ago, with additional question: find the minimal degree of a counterexample. Try to search for this topic.", "Solution_3": "$P(x)=(x^{2}-17)(x^{2}+1)(x^{2}-2)(x^{2}+2)$.", "Solution_4": "@Harazi: you are too slow :P :D \r\n\r\n@Rust: no, try $\\mod 16$.", "Solution_5": "I edit, before I forget even prime 2." } { "Tag": [ "algebra", "polynomial" ], "Problem": "1. Given the polynomial identity \r\n\r\n$ (x^6 \\plus{} 1) \\equal{} (x^2 \\plus{} 1)(x^2 \\plus{} ax \\plus{} 1)(x^2 \\plus{} bx \\plus{} 1)$\r\n\r\nWhat is the value of $ ab$?\r\n\r\nEDIT: I see what I did :oops:", "Solution_1": "[quote=\"Stokes93\"]1. Given the polynomial identity \n\n$ (x^6 \\plus{} 1) \\equal{} (x^2 \\plus{} 1)(x^2 \\plus{} ax \\plus{} 1)(x^2 \\plus{} bx \\plus{} 1)$\n\nWhat is the value of $ ab$?\n\n2. What is the number of distinct real solutions to the equation\n\n$ x^4 \\plus{} 6x^2 \\plus{} 9 \\equal{} 36x^2 \\minus{} 72x \\plus{} 36$ \n\nI know you can solve it by factoring... but I don't see what is wrong with my solution (below)\n\n[code]We have $x^4 - 30x^2 - 25 = 0$ [/code]\n\n let $ p \\equal{} x^2$ $ \\Rightarrow$ $ p^2 \\minus{} 30p \\minus{} 25 \\equal{} 0$ \n\n$ \\Rightarrow$ $ p \\equal{} 15 \\pm 5 \\sqrt {10}$ $ \\equal{}$ $ x^2$ \n\n$ \\Rightarrow$ $ x \\equal{}$ $ \\pm \\sqrt { 15 \\pm 5 \\sqrt {10}}$\n\nNow we can have $ \\plus{}$ or $ \\minus{}$ on the outside, but for a real solution the $ \\pm$ on the inside must be $ \\plus{}$ so we have two real distinct solutions.[/quote]\r\n\r\nHow does this come?", "Solution_2": "Oh! :blush: I totally left out the $ \\minus{}72x$ when I did it... \r\n\r\nThus substitution will not work well.... so never mind :oops:", "Solution_3": "[quote=\"Stokes93\"]\n\n2. What is the number of distinct real solutions to the equation\n\n$ x^4 \\plus{} 6x^2 \\plus{} 9 \\equal{} 36x^2 \\minus{} 72x \\plus{} 36$ \n\nI know you can solve it by factoring... but I don't see what is wrong with my solution (below)\n\nWe have $ x^4 \\minus{} 30x^2 \\minus{} 25 \\equal{} 0$ \n\n let $ p \\equal{} x^2$ $ \\Rightarrow$ $ p^2 \\minus{} 30p \\minus{} 25 \\equal{} 0$ \n\n$ \\Rightarrow$ $ p \\equal{} 15 \\pm 5 \\sqrt {10}$ $ \\equal{}$ $ x^2$ \n\n$ \\Rightarrow$ $ x \\equal{}$ $ \\pm \\sqrt { 15 \\pm 5 \\sqrt {10}}$\n\nNow we can have $ \\plus{}$ or $ \\minus{}$ on the outside, but for a real solution the $ \\pm$ on the inside must be $ \\plus{}$ so we have two real distinct solutions.[/quote]\r\n\r\nYou forgot about the 72x.\r\n\r\n\r\n\r\n[hide=\"#1\"]\nMultiplying out we have\n\n$ x^6\\plus{}1\\equal{}x^6\\plus{}(a\\plus{}b)x^5\\plus{}(ab\\plus{}3)x^4\\plus{}(2a\\plus{}2b)x^3\\plus{}(ab\\plus{}3)x^2\\plus{}(a\\plus{}b)x\\plus{}1$\n\nMatching we have\n\n$ a\\plus{}b\\equal{}0$ and $ ab\\plus{}3\\equal{}0$\n\nWe see $ a\\equal{}\\sqrt3$ and $ b\\equal{}\\minus{}\\sqrt3$ (or vice versa)[/hide]", "Solution_4": "[quote=\"Stokes93\"]1. Given the polynomial identity \n\n$ (x^6 \\plus{} 1) \\equal{} (x^2 \\plus{} 1)(x^2 \\plus{} ax \\plus{} 1)(x^2 \\plus{} bx \\plus{} 1)$\n\nWhat is the value of $ ab$?\n\n2. What is the number of distinct real solutions to the equation\n\n$ x^4 \\plus{} 6x^2 \\plus{} 9 \\equal{} 36x^2 \\minus{} 72x \\plus{} 36$ \n\nI know you can solve it by factoring... but I don't see what is wrong with my solution (below)\n\nWe have $ x^4 \\minus{} 30x^2 \\minus{} 25 \\equal{} 0$ \n\n let $ p \\equal{} x^2$ $ \\Rightarrow$ $ p^2 \\minus{} 30p \\minus{} 25 \\equal{} 0$ \n\n$ \\Rightarrow$ $ p \\equal{} 15 \\pm 5 \\sqrt {10}$ $ \\equal{}$ $ x^2$ \n\n$ \\Rightarrow$ $ x \\equal{}$ $ \\pm \\sqrt { 15 \\pm 5 \\sqrt {10}}$\n\nNow we can have $ \\plus{}$ or $ \\minus{}$ on the outside, but for a real solution the $ \\pm$ on the inside must be $ \\plus{}$ so we have two real distinct solutions.[/quote]\r\n\r\n[hide=\"#1\"]\n$ (x^2 \\plus{} 1)(x^2 \\plus{} ax \\plus{} 1)(x^2 \\plus{} bx \\plus{} 1)$\n$ \\equal{}x^2(x^2 \\plus{} ax \\plus{} 1)(x^2 \\plus{} bx \\plus{} 1)\\plus{}(x^2 \\plus{} ax \\plus{} 1)(x^2 \\plus{} bx \\plus{} 1)$\n$ \\equal{}x^2(x^4\\plus{}(a\\plus{}b)x^3\\plus{}(ab\\plus{}2)x^2\\plus{}(a\\plus{}b)x\\plus{}1)\\plus{}(x^4\\plus{}(a\\plus{}b)x^3\\plus{}(ab\\plus{}2)x^2\\plus{}(a\\plus{}b)x\\plus{}1)$\n$ \\equal{}x^6\\plus{}(a\\plus{}b)x^5\\plus{}(ab\\plus{}3)x^4$+2(a+b)x^3+(ab+3)x^2+(a+b)x+1$ Then we have$=x^6+(a+b)x^5+(ab+3)x^4$ \\plus{}2(a\\plus{}b)x^3\\plus{}(ab\\plus{}3)x^2\\plus{}(a\\plus{}b)x\\plus{}1\\equal{}x^6\\plus{}1$\n $ ab\\plus{}3\\equal{}0$\n $ ab\\equal{}\\minus{}3$\n\n[/hide]" } { "Tag": [ "geometry unsolved", "geometry" ], "Problem": "Well can anyone please provide me the link to the proof , or the proof itself, for the Van Aubels;s Theorem (For Triangles) . I have been trying to prove it for past several Days but have failed to devise any proof\r\n \r\n :maybe:", "Solution_1": "hello, see here\r\nhttp://planetmath.org/encyclopedia/ProofOfVanAubelsTheorem.html\r\nSonnhard.", "Solution_2": "Thank you very much !! Finally I got the proof !!! What a relief\r\n\\ :roll: :oops:" } { "Tag": [ "inequalities" ], "Problem": "$a,b\\ge1 , c\\ge0$and $a,b$are real numbers and $n$ is natural number then prove it:\r\n$(ab+c)^{n}-c\\le((b+c)^{n}-c)a^{n}$", "Solution_1": "who solved it???" } { "Tag": [ "geometry", "inradius", "inequalities", "inequalities proposed" ], "Problem": "let $ a$ , $ b$ , $ c$ be the lenghts of the sides of a triangle $ r$ inradius and $ R$ outradius , prove that :\r\n\r\n$ \\frac{(a\\plus{}b)(b\\plus{}c)(c\\plus{}a)}{4abc} \\le \\frac{R}{r}$", "Solution_1": "this is from Iran 98...\r\n\r\nfirst of all let $ p = \\frac {a + b + c}2$ now note that:\r\n\r\n\\begin{eqnarray*}(a + b)(b + c)(c + a) = a^2(b + c) + b^2(c + a) + c^2(a + b) + 2abc = 2p(a^2 + b^2 + c^2) - (a^3 + b^3 + c^3) + 2abc\\end{eqnarray*}\r\n\r\nnow note that:\r\n\r\n$ a^2 + b^2 + c^2 = 2(p^2 - r^2 - 4Rr)$\r\n\r\n$ a^3 + b^3 + c^3 = 2p(p^2 - 3r^2 - 6Rr)$\r\n\r\nso we have to show that:\r\n\r\n$ \\frac {r}{4R}\\leq\\frac {4Rpr}{4p(p^2 - r^2 - 4Rr) - 2p(p^2 - 3r^2 - 6Rr) + 8Rpr}$\r\n\r\n$ \\iff 8R^2 - 2Rr - r^2\\geq p^2$\r\n\r\nbut we have:\r\n\r\n$ p^2\\leq 4R^2 + 4Rr + 3r^2$\r\n\r\nso it's sufficient to show that:\r\n\r\n$ 8R^2 - 2Rr - r^2\\geq 4R^2 + 4Rr + 3r^2$\r\n\r\n$ \\iff 4R^2 - 6Rr - 4r^2\\geq 0$\r\n\r\n$ \\iff 2(R - 2r)(2R + r)\\geq 0$\r\n\r\n$ \\iff R\\geq 2r$\r\n\r\nwhich is true...", "Solution_2": "[quote=\"y-a-s-s-i-n-e\"]let $ a$ , $ b$ , $ c$ be the lenghts of the sides of a triangle $ r$ inradius and $ R$ outradius , prove that :\n\n$ \\frac {(a \\plus{} b)(b \\plus{} c)(c \\plus{} a)}{4abc} \\le \\frac {R}{r}$[/quote]\r\n$ \\frac{a\\plus{}b}{c}\\equal{}\\frac{sin(A)\\plus{}sin(B)}{sin(C)}\\equal{}\\frac{2sin{((A\\plus{}B)/2)cos((A\\minus{}B)/2)}}{2sin(C/2)cos(C/2)}\\equal{}$ $ \\frac{cos((A\\minus{}B)/2)}{sin(C/2)} \\le \\frac{1}{sin(C/2)}$\r\nso : $ \\frac {(a \\plus{} b)(b \\plus{} c)(c \\plus{} a)}{4abc} \\le \\frac{1}{4sin(A/2)sin(B/2)sin(C/2)}\\equal{} \\frac{R}{r}$; :)", "Solution_3": "thank's a lot\r\n nice solutions :)", "Solution_4": "I need some help regarding my solution approach.:help: \r\nWe know that A=area of triangle=abc/4R\r\nthus, R=abc/4A\r\nagain let S=(a+b+c)/2\r\nthus, r=A/S\r\nthus, R/r=abcS/4A^2=abc(a+b+c)/8[(a+b+c)(a+b-c)(b+c-a)(c+a-b)/16]\r\nor, R/r=2abc/(a+b-c)(b+c-a)(c+a-b)\r\nthus, the inequality becomes,\r\n (a+b)(b+c)(c+a)(a+b-c)(b+c-a)(c+a-b) <= 8a^2b^2c^2\r\nNow, [(a+b)(b+c)(c+a)(a+b-c)(b+c-a)(c+a-b)]^1/6 <= [a+b+b+c+c+a+a+b-c+b+c-a+c+a-b]/6=3[a+b+c]/6=S\r\nthus enough to show, S^3 <= 8^1/2*abc...............Hereafter I failed to continue.........:help:" } { "Tag": [ "number theory proposed", "number theory" ], "Problem": "Find out the first nonzero rightmost digit when $ 125!$ is represented in base $ 5$.", "Solution_1": "Let $ f(k)\\equal{}5k\\plus{}1)(5k\\plus{}2)(5k\\plus{}3)(5k\\plus{}4)$.\r\n\\[ 125!\\equal{}\\prod_{k\\equal{}0}^{24}f(k) *5^{25} \\prod_{k\\equal{}0}^4f(k)*5^{5}f(0).\\]\r\n$ 1*2*3*4 \\equal{} 4\\mod 5$, therefore $ f(k)\\equal{}4\\mod 5$ and $ 125!\\equal{}5^{25\\plus{}5\\plus{}1}*a, a\\equal{}4^{25\\plus{}5\\plus{}1} \\equal{} 4\\mod 5$." } { "Tag": [ "limit", "algebra", "polynomial" ], "Problem": "(a) Show that for every $n\\in\\mathbb{N}$ there is exactly one $x\\in\\mathbb{R}^+$ so that $x^n+x^{n+1}=1$. Call this $x_n$.\r\n(b) Find $\\lim\\limits_{n\\rightarrow+\\infty}x_n$.", "Solution_1": "Equation can be rewritten $1+ x = 1/x^n$\r\n\r\nLHS is strictly increasing from 1 to $+\\infty$, RHS is strictly decreasing from $\\infty$ to $0^+$ this proves existence and uniqueness of solution\r\n\r\n$x^n+x^{n+1}$ is strictly increasong and $1^n+1^{n+1} > 1$ so $1 > x_n > 0$ and $(1+x)^{1/n} \\rightarrow 1$ (typo corrected).\r\n\r\nEquation can be rewritten $x(1+ x)^{1/n} = 1$ proving that $x_n \\to 1$", "Solution_2": "i disagree since if you have n being odd, then the polynomial has an even lead coefficient, and since the roots pair up, there cannot be just 1 real root, there must be a multiple of 2 of real roots", "Solution_3": "[quote=\"Altheman\"]i disagree since if you have n being odd, then the polynomial has an even lead coefficient, and since the roots pair up, there cannot be just 1 real root, there must be a multiple of 2 of real roots[/quote]yes, I forgot a detail: x should be positive. The second solution will be negative in your case.\r\n\r\nT\u00b5t\u00b5 is right anyway. :)", "Solution_4": "[quote=\"t\u00b5t\u00b5\"] and $(1+x)^{1/n} \\rightarrow 0$.[/quote]\r\nbut $(1+x)^{1/n} \\rightarrow 1$ no?!", "Solution_5": "yes. he meant 1 I guess, since the rest of his solution uses 1. :)", "Solution_6": "(a)\n$x^n+x^{n+1}$ is strictly increasing and continuous on $\\mathbb R^{+}$.And $x^n+x^{n+1}$ goes from $0$ to $+\\infty$ on $\\mathbb R^{+}$.Then by [b]intermediate value theorem,[/b]\n$\\exists ! x_n>0$ s.t. $x_n^n+x_n^{n+1}=1\\blacksquare$ :coool:\n\n(b)\nObviously $0x_{n+1}^{n+1}+x_{n+1}^{n+2}$ which is absurd.Thus $x_{n+1}\\ge x_n$.Then $\\{x_n\\}$ is increasing and upper-bounded.Therefore $\\exists \\alpha$ s.t. $\\lim_{n\\to +\\infty} x_n=\\alpha$.$0<\\alpha \\le 1$.If $0<\\alpha <1,$then $\\lim_{n\\to +\\infty} \\ln x_n^n=\\lim_{n\\to +\\infty} n\\ln x_n=-\\infty$.Hence\n$\\lim_{n\\to +\\infty} x_n^n=0$.And $00$, the $\\mathrm{LHS}$ can have any value from the interval $\\langle 0;2\\rangle$ (with the additional assumption [b]me@home[/b]'s method is an easy way to prove that $2$ is the maximum).", "Solution_6": "${(1-t) x+(1-x) y+t (1-z)+(1-y) z\\geq 1}$\r\n\r\n${(1-t) x+(1-x) y+t (1-z)+(1-y) z\\geq 1}$\r\n${-x t-z t+t+x-x y+y-y z+z\\geq 1}$\r\n${(t+y-1) (x+z-1)\\leq 0}$\r\n\r\n${\\{x,y,z,t\\}<\\{0,0,0,0\\}}$\r\n\r\nSo it is true", "Solution_7": "[quote=\"Thales418\"]${(1-t) x+(1-x) y+t (1-z)+(1-y) z\\geq 1}$\n\n${(1-t) x+(1-x) y+t (1-z)+(1-y) z\\geq 1}$\n${-x t-z t+t+x-x y+y-y z+z\\geq 1}$\n${(t+y-1) (x+z-1)\\leq 0}$\n\n${\\{x,y,z,t\\}<\\{0,0,0,0\\}}$\n\nSo it is true[/quote]\r\nBut what if $x+z>1$ and $y+t>1$?", "Solution_8": "${x<0\\land y<0\\land z<0\\land t<0}$\r\n\r\n${x=-a\\land y=-b\\land z=-c\\land t=-d}$\r\n\r\n${a>0\\land b>0\\land c>0\\land d>0}$\r\n\r\n${(t+y-1) (x+z-1)\\leq 0}$\r\n\r\n${(-a-c-1) (-b-d-1)\\leq 0}$\r\n${-(a+c+1) (-1) (b+d+1)\\leq 0}$\r\n${(a+c+1) (b+d+1)\\leq 0}$\r\n\r\nSince ${a>0\\land b>0\\land c>0\\land d>0}$\r\n\r\nIt' s false\r\n\r\nMy error ... Sorry" } { "Tag": [ "geometry", "calculus", "integration", "trigonometry", "calculus computations" ], "Problem": "How do I find the area for the intersection of 2 polar curves without converting to Cartesian? \r\nFor example how do I get the integral for the area between $ r\\equal{} \\sin \\theta$ and $ r\\equal{}\\cos\\theta?$", "Solution_1": "The area of a \"slice\" of a polar curve $ r(\\theta)$ is given by $ \\frac{1}{2} \\int_{\\theta_1}^{\\theta_2} r^2 \\, d \\theta$. You need to find intersections $ \\theta_1, \\theta_2$ and then calculate the difference in the areas." } { "Tag": [ "AMC", "USA(J)MO", "USAMO", "probability", "videos", "AMC 10" ], "Problem": "Who's the youngest person ever to make the US IMO team? What was his/her name?\r\n\r\nAnd do you think that Evan O'Dorney dude will make it onto the team this year?", "Solution_1": "I probably could get 22 or higher but in a testing situation I'll probably end up 15-21.", "Solution_2": "whos evan o dorney?", "Solution_3": "He's a middle school math beast.\r\n\r\nHe also recently won the Scripps Spelling Bee, which earned him a lot of attention. A TV interview with some news woman earned him even more publicity (you can see it on youtube).", "Solution_4": "how beast imo?", "Solution_5": "There was this guy who made it to the IMO when he was 10. He lived in Australia at the time, I think.", "Solution_6": "[quote=\"1=2\"]There was this guy who made it to the IMO when he was 10. He lived in Australia at the time, I think.[/quote]\r\nYeah, I think you thought about Terence Tao :wink: Real genius :)", "Solution_7": "Yeah, Terence Tao won the Fields medal.", "Solution_8": "[quote=\"The Zuton Force\"]He's a middle school math beast.\n\n[b]He also recently won the Scripps Spelling Bee, which earned him a lot of attention. A TV interview with some news woman earned him even more publicity (you can see it on youtube).[/b][/quote]\r\n\r\nyou're kidding right?\r\n\r\ni remember all that publicity was very negative if you read all the comments...\r\npeople dont appreciate his talents.", "Solution_9": "Theres no such thing as bad publicity?", "Solution_10": "i watched an interview on youtube of Evan O. He seems like he has some issues is he autistic? some times academic achievement is not the most important thing. did he make IMO?", "Solution_11": "IMO, Evan O'Dorney won't make IMO. O.K. he made MOP last year. That was great. Making MOP in eighth grade is great. Yet, I think that others (Krishnau, Alex Anderson, etc) are more pro. Evan O'Dorney may have a chance next year.", "Solution_12": "[quote=\"blackbelt14253\"]Who's the youngest person ever to make the US IMO team? What was his/her name?\n\nAnd do you think that Evan O'Dorney dude will make it onto the team this year?[/quote]\r\n\r\nI'm not sure, but I'd think it would be either Reid Barton or Tiankai Liu? I'm guessing they were about 14 at the time. Actually, come to think of it, Ricky Liu and Po-Ru Loh are also possible.", "Solution_13": "Well it's impressive and cool that Evan is so successful, but I do agree that it is not good publicity. I feel like those interviews reinforce the negative stereotypes that people have about people who like math. Those interviews were the most awkward thing that I've ever seen. It portrays math students as lacking any social skills whatsoever.", "Solution_14": "USAMO top twelve last year who have not graduated:\r\n\r\n- Sergei Bernstein (11)\r\n- Eric Larson (11)\r\n- Haitao Mao (12)\r\n- Delong Meng (11)\r\n- Krishanu Shankar (12)\r\n- Jacob Steinhardt (12)\r\n- Alex Zhai (12)\r\n\r\nHonorable Mentions last year who have not graduated:\r\n\r\n- David Benjamin (11)\r\n- Yingyu (Dan) Gao (12)\r\n- Yan Li (12)\r\n- Gaku Liu (12)\r\n- Jeffrey Manning (12)\r\n- Palmer Mebane (12)\r\n- [b]Evan O'Dorney (9)[/b]\r\n- Alexander Remorov (12)\r\n- Max Rosett (12)\r\n- Qiaochu Yuan (that's me! :) ) (12)\r\n\r\nwhich puts Evan O'Dorney in some pretty respectable company. In any case, social skills (or lack thereof) notwithstanding, he's got three more years to make it onto the team and a pretty good head start. :)", "Solution_15": "[quote=\"PI-Dimension\"][quote=\"n^4+4\"]Haha is Jeremy Kahn approximately equal to Jeremy Hahn?[/quote]\n\n\nLol sure.\n\nI was like wow that looks familiar.[/quote]\r\n\r\nThis is on topic?\r\n\r\nBesides, if you watched the videos of Evan being interviewed, you would get it.", "Solution_16": "[quote=\"mathcrazed\"]Evan O'Dorney has Asperger's... [/quote]\r\n\r\nWhat is Asperger's?", "Solution_17": "aspergers is a type of autism", "Solution_18": "As a rule of thumb, you can't have autism and make MOP in eighth grade", "Solution_19": "A human with Asperger syndrome displays intense interest in restricted interests or activities. This may lead to that human being extremely knowledgeable or good at that activity/interest. However, because this is still a brain disorder, it is labeled as autism.\r\nI don't know whether Evan O'Dorney has Aspergers or not, but someone with autism [i]can[/i] make MOP. Actually, with some types of autism, you're more likely to.", "Solution_20": "Asperger's syndrome is a syndrome on the autism [i]spectrum.[/i] This is not quite the same thing as being autistic. In any case, the autism spectrum is quite wide and there are many gradations of autistic behavior.", "Solution_21": "[quote=\"#H34N1\"]As a rule of thumb, you can't have autism and make MOP in eighth grade[/quote]\r\n\r\nI would take offense to that if I was close to someone with autism. I don't know what your intention was with that comment, but one could interpret it as you completely berating autistic people. As far as I know, some autistic people have extraordinary abilities, in things such as memorizing pi and music. Who knows, maybe there is an autistic person out there, if Evan O'Dorney doesn't have Asperger's as some people are oddly suggesting, that excels at olympiad math. While I still may be grossly wrong, unless you really know your stuff about autism, you probably shouldn't make a generalization like that.", "Solution_22": "Evan shows signs of Asperger's... Although he does attempt to engage in social activities, he is unable to understand some types of irony, sarcasm, etc. He does have an intense interest in math, as uldivad9 implied, which is also a symptom of Asperger's.", "Solution_23": "I think we should all stop speculating, it's Evan's business, and by no means ours.\r\n\r\nAlso, see [url=http://www.artofproblemsolving.com/Forum/viewtopic.php?t=159137]this[/url] and [url=http://www.artofproblemsolving.com/Forum/viewtopic.php?t=162184]this[/url]", "Solution_24": "Yeah these threads seem to get really popular about him. Guys it shouldn't be our business to speculate his mental condition, and he probably gets enough beef if a lot of people perceive a problem anyway. At least not on an online forum :wink:. It isn't on topic anyway...\r\n\r\nAnyway I think this year has a lot of really good seniors and that USA will do really well at IMO, in my opinion. We might even win! :D", "Solution_25": "[quote=\"CatalystOfNostalgia\"][quote=\"#H34N1\"]As a rule of thumb, you can't have autism and make MOP in eighth grade[/quote]\n\nI would take offense to that if I was close to someone with autism. I don't know what your intention was with that comment, but one could interpret it as you completely berating autistic people. As far as I know, some autistic people have extraordinary abilities, in things such as memorizing pi and music. Who knows, maybe there is an autistic person out there, if Evan O'Dorney doesn't have Asperger's as some people are oddly suggesting, that excels at olympiad math. While I still may be grossly wrong, unless you really know your stuff about autism, you probably shouldn't make a generalization like that.[/quote]\r\n\r\nI'm not berating autistic people. I'm just saying that it doesn't make sense to claim that someone is autistic when they are pwnage at math skillz :|", "Solution_26": "h341\r\nfor the last time many people with autism are very good at math", "Solution_27": "[quote=\"#H34N1\"][quote=\"CatalystOfNostalgia\"][quote=\"#H34N1\"]As a rule of thumb, you can't have autism and make MOP in eighth grade[/quote]\n\nI would take offense to that if I was close to someone with autism. I don't know what your intention was with that comment, but one could interpret it as you completely berating autistic people. As far as I know, some autistic people have extraordinary abilities, in things such as memorizing pi and music. Who knows, maybe there is an autistic person out there, if Evan O'Dorney doesn't have Asperger's as some people are oddly suggesting, that excels at olympiad math. While I still may be grossly wrong, unless you really know your stuff about autism, you probably shouldn't make a generalization like that.[/quote]\n\nI'm not berating autistic people. I'm just saying that it doesn't make sense to claim that someone is autistic when they are pwnage at math skillz :|[/quote]\r\n\r\nPeople weren't claiming that he had Asperger's, or he was autistic, just because he was smart, they were claiming that because of his personality as seen on television. And I don't exactly see how what you are/were \"just saying\" can ever be interpreted from your comment. So long as we're clear...", "Solution_28": "Alright guys let's get back on topic? Evan O'Dorney may or may not have a mental condition, but I think that is a really private thing.\r\n\r\nAny subsequent posts off topic will be deleted. Feel free, of course, to discuss what you like over PM.", "Solution_29": "Agreed. I will be watching this topic as well." } { "Tag": [ "number theory" ], "Problem": "it is well known that there is no known closed-form of the formula for partitions.\r\nHas it been proven that there is no formula? If so, could someone prove it here if it isn't too complicated?", "Solution_1": "That's not entirely true.\r\nThere is the Rademacher Series, which is asymptotically correct.\r\nHere's the wikipedia link, but beware, the Rademacher series is pretty complex and involves Dedekind sums.\r\n[url=http://en.wikipedia.org/wiki/Partition_(number_theory)#Rademacher.27s__series]Here[/url]", "Solution_2": "[quote=\"DiscreetFourierTransform\"]That's not entirely true.\nThere is the Rademacher Series, which is asymptotically correct.\nHere's the wikipedia link, but beware, the Rademacher series is pretty complex and involves Dedekind sums.\n[url=http://en.wikipedia.org/wiki/Partition_(number_theory)#Rademacher.27s__series]Here[/url][/quote]\r\nI meant a closed form.", "Solution_3": "Well, I suppose that's as close to a closed form as we have. \r\nHowever, you're right that there is no exact closed form.\r\nThat link has all the information in it.\r\n\r\nEDIT: The link doesn't like the URL tags.\r\nJust copy+paste it into the browser." } { "Tag": [ "function", "calculus", "derivative", "real analysis", "real analysis solved" ], "Problem": "prove that for every n\\in N* the following ecuation\r\n\\sum {k=1,n}1/ \\sqrt (nx+k) = \\sqrt(n)\r\nhas a unique real root x=x_n and find lim{n->oo}x_n.", "Solution_1": "Let f(x) be the function f(x) = \\sum(k=1,..,n){1/\\sqrt (nx+k)} , defined for x > -1/n .\r\n\r\nIt's easy to see that lim(x-->+\\infty){f(x)} = 0 < \\sqrt(n) and lim(x-->-1/n){f(x)} = +\\infty > \\sqrt(n) , so there is at least a real root x(n) , -1/n < x(n) < 0\r\n\r\nManifestly the derivative f'(x) is ngative, so f(x) is decreasing , so the root x(n) is unique.\r\n\r\n-1/n < x(n) < 0 ===> lim(n-->\\infty){x(n)} = 0\r\n........", "Solution_2": "not very correct\r\nx_n is in (-1/n,oo)\r\nso its limit is not 0.\r\ncheck again your proof\r\nu can treat the more genrealy problem\r\n\r\n[i]f(x)=a \\sqrt n has a unique root x=x_n and find it's limit, where a>0[/i][/i]", "Solution_3": "Oh god, Diogene is really too bad, he mistook 0 for \\infty ! So I , his \" alter-ego\", decided to take his place . :D :D \r\nAnd now, come back to the problem ..\r\n\r\nS_n(x) = (1/n) \\sum(k=1,..,n){1/\\sqrt(x+k/n)} = A , A > 0 ; We know that there is an unique solution x(n), x(n) > -1/n .\r\n\r\nBriefly, we can relate S_n(x) to the Reamann sums for I(x) = \\int(0 < t <1){1/sqrt(1/(x+t)).dt} = 2*(\\sqrt (x+1) + \\sqrt(x)) = I(x) , for x>0.\r\n\r\nFinally the result is : \r\n0 < A =< 2 ===> I(x)=A has an unique solution ===> lim(n-->\\infty){x(n)} = (4-A2 )2/(16*A2)\r\n2 < A ===> I(x)=A has not solution ===> lim(n-->+\\infty ){x(n)} = 0\r\n\r\nIn general, if the (1/n)* \\sum (k=1,..,n){f(x,k/n)} = g(x) has a solution x(n) ,\r\n\r\nthen the study of the behavior of the x(n) sequence is relating to the equation \\int(0< t <1){f(x,t).dt} = g(x)\r\n...", "Solution_4": "very good!! :D:D" } { "Tag": [ "geometry" ], "Problem": "The farmer\u2019s pivot has an effective irrigation radius of \u00bc mile (640 acres in a square mile) and is positioned alongside a straight section of highway. He wants to apply a special fertilizer at the rate of 0.7 gallons per acre. How much does he need to get for the irrigation system?\r\n\r\nA) 45 gallons B) 88 gallons C) 32 gallons D) 50 gallons", "Solution_1": "His lot is a semi-circle with radius $ \\frac14$. The area is $ \\frac\\pi{32}$ square miles or $ 20\\pi$ acres.\\[ 20\\pi\\cdot0.7\\approx43.98\\]This is close to A." } { "Tag": [ "floor function", "induction", "combinatorics unsolved", "combinatorics" ], "Problem": "A word is a sequence of n letters of the alphabet {a, b, c, d}. A word is said to be complicated if it contains two consecutive groups of identic letters. The words caab, baba and cababdc, for example, are complicated words, while bacba and dcbdc are not. A word that is not complicated is a simple word. Prove that the numbers of simple words with n letters is greater than $2^n$, if n is a positive integer.", "Solution_1": "It is also Romanian TST 2003. And as I remember, this problem were discussed before on forum.", "Solution_2": "This problems is still unsolved. :(", "Solution_3": "Let us denote by $S(n)$ the set of simple words with $n$ letters and by $s_n$ the number of elements in $S(n)$. If we put a letter at the end of each of the simple words from $S(n)$ we obtain a set $T(n+1)$ of $t_{n+1}$ words of length $n+1$, their number being $t_{n+1}=4s_n$. Obviously $S(n+1) \\subset T(n+1)$, $S(n+1)\\neq T(n+1)$.\r\n\r\nLet $T_1(n+1)$ be the set of those words from $T(n+1)$ which have the last two letters the same, $T_k(n+1)$ the set of $T(n+1)$ which end in two consecutive identical groups of $k$ letters, for each $k\\in\\{1,2,\\ldots,m\\}$, where $m=\\displaystyle \\left\\lfloor \\frac { n+1 } 2 \\right\\rfloor$. Obviously\r\n\\[ f(n+1)\\geq t_{n+1}-|T_1(n+1)|-|T_2(n+1)|-\\cdots - |T_m(n+1)|\\]\r\nWe have $t_{n+1}=4f(n)$, $|T_1(n+1)|=f(n)$, and furthermore $|T_k(n+1)|\\leq f(n+1-k)$, because of the fact that the mapping of $S(n+1-k)$ into $T_k(n+1)$ given by adding to a simple word of $n+1-k$ letters its own last $k$ letters is obviously surjective.\r\n\r\nWe have $f(1)=4$, $f(2)=12>4$. By induction we want to prove $f(k+1)>2f(k)$. We have\r\n\\[ f(n+1)\\geq 4f(n)-f(n)-\\frac 12 f(n) - \\frac 14 f(n)- \\cdots > 2 f(n) \\] from which the conclusion follows.", "Solution_4": "But still the obvious next questions are :\r\nQ1: Is 2^n a tight bound ? \r\nQ2: Is the bound dependent on the alphabet size ? \r\n\r\n A precision : when I say obvious I mean the esayness to formulate such questions\r\n not that the answers are necessarily easy.\r\n\r\n REMARK : \r\n It seems that too often that thes olympiad/IMO problems concentrated on one fact. \r\n I believe it is nice to follow up , to give perspective to the result by showing how it\r\n hinges on the hypothesis, otherwise the solution as little meaning or constitutes merrely\r\n a useful technique. See and read Polya about problem solving in general and making \r\nVARIATION on problems.", "Solution_5": "Valentin Vornicu wrote:\r\n\r\n\\box{\r\nWe have $t_{n+1}= 4f(n)$, $|T_{1}(n+1)| = f(n)$, and furthermore $|T_{k}(n+1)| \\leq f(n+1-k)$, because of the fact that the mapping of $S(n+1-k)$ into $T_{k}(n+1)$ given by adding to a simple word of $n+1-k$ letters its own last $k$ letters is obviously surjective.}\r\n\r\nThe mapping of $S(n+1-k)$ into $T_{k}(n+1)$ defined latter is wrong defined. The mapping of $T_{k}(n+1)$ into $S(n+1-k)$ given by removing to a word of $T_{k}(n+1)$ its own last $k$ letters is obviously injective (and this mapping is well defined).", "Solution_6": "another solution I came across by [b]math154[/b] from [url=https://artofproblemsolving.com/community/c285h400665p2237686]here[/url]\n\n[hide=\"Solution\"]Let $f(n)$ denote the number of simple words with $n$ letters. We'll show by strong induction that $f(n+1)>2f(n)$ for all $n\\ge0$. Since $f(0)=1$ and $f(1)=4$, the base case is clear.\n\nNow consider a simple word $W$ with $n\\ge1$ letters, and prepend a letter $x\\in\\{a,b,c,d\\}$ to it. There exist strings $S,T$ (possibly empty) such that\n\\[x+W=x+S+x+S+T \\Longleftrightarrow W=S+x+S+T\\]iff $x+W$ is complicated (note that $x+S+T\\in W$ and is thus simple). Hence there is a surjection from simple words with $1\\le k=|x+S+T|\\le n$ letters to complicated words of the form $x+W$ (i.e. prepending the first $n+1-k$ letters of $x+S+T$ to get $x+W$), so by complementary counting,\n\\[f(n+1) \\ge 4f(n) - \\sum_{k=1}^{n}f(k) > 4f(n) - \\sum_{k=1}^{n}\\frac{f(n)}{2^{n-k}} > 2f(n),\\]as desired.[/hide]\n\nI hope it is not exactly the same as above" } { "Tag": [ "inequalities unsolved", "inequalities" ], "Problem": "Let $x,y,z\\geq 2$. Find the least $k$ such that \r\n$(x^3+y)(y^3+z)(z^3+x)\\leq kxyz$", "Solution_1": "For every $k$ we can add $y=z=2$ and $x$ can be a large number; so the left side will be greater than the right side.\r\nI think you mean this one : $x,y,z\\geq2$\r\n$(x^3+y)(y^3+z)(z^3+x)\\geq125xyz$", "Solution_2": "[quote=\"Ramtin Madani\"]\nI think you mean this one : $x,y,z\\geq2$\n$(x^3+y)(y^3+z)(z^3+x)\\geq125xyz$[/quote]\r\n\r\nRight, can you prove it?", "Solution_3": "$(x^3+y)(y^3+z)(z^3+x)\\geq(4x+y)(4y+z)(4z+x)\\geq 5\\sqrt[5]{x^4y}\\cdot 5\\sqrt[5]{y^4z}\\cdot 5\\sqrt[5]{z^4x}=125xyz$\r\nRamtin, sorry. :)" } { "Tag": [ "inequalities", "inequalities theorems" ], "Problem": "Hello \r\n\r\nPlz help me to find some free books about inequalities.\r\n\r\nThanks", "Solution_1": "hello, see this here\r\nhttp://www.eleves.ens.fr/home/kortchem/olympiades/Cours/Inegalites/tin2006.pdf\r\nSonnhard.", "Solution_2": "Thank you a lot for the book\r\nit's great\r\n\r\nare there any other books ? not about inequalities only\r\n\r\n^^\r\n\r\n10x" } { "Tag": [ "number theory solved", "number theory" ], "Problem": "solve in positive integers the following equation\r\n$x^2y^2=z^2(z^2-x^2-y^2)$", "Solution_1": "hmm.. discriminant of this equation in z^{2} is x^{4}+y^{4}+6x^{2}y^{2} which is never a perfect square .i proved this yesterday.and btw wot is the source of this problem or u made it up?\r\nnice one ,neway", "Solution_2": "This is a Bulgarian problem I think, i'm not sure :?", "Solution_3": "Bulgaria 1998 :D \r\nOn the site of the Bulgarian MO, another solution is given" } { "Tag": [ "function", "algebra proposed", "algebra" ], "Problem": "Find all functions $ f: [0,1] \\to [0,1]$ and \\[ f(2x - f(x)) = x\\] for all $x \\in [0,1]$.", "Solution_1": "[quote=\"MJ GEO\"]$ f$ form $ [0,1]$ to $ [0,1]$ and $ f(2x \\minus{} f(x)) \\equal{} x$[/quote]\r\n\r\nThis immediately implies $ f((n\\plus{}1)x\\minus{}nf(x))\\equal{}nx\\minus{}(n\\minus{}1)f(x)$ and so $ 1\\ge nx\\minus{}(n\\minus{}1)f(x)\\ge 0$ and so $ \\frac n{n\\minus{}1}x\\ge f(x)\\ge \\frac{nx\\minus{}1}{n\\minus{}1}$ and so $ \\boxed{f(x)\\equal{}x}$ which, indeed, is a solution.", "Solution_2": "i cant understand your solution.coud you write more about it? its mine\r\n$ g(x)\\equal{}2x\\minus{}f(x)$ so $ f(g(x))\\equal{}x$. $ g(g(x))\\equal{}2g(x)\\minus{}f(g(x))\\equal{}(2g(x)\\minus{}2x)\\plus{}x$ and similiary $ g^n(x)\\equal{}(ng(x)\\minus{}nx)\\plus{}x$ so $ g(x)\\equal{}x$ so $ f(x)\\equal{}x$" } { "Tag": [ "floor function", "logarithms", "number theory unsolved", "number theory" ], "Problem": "Please help me with this one:\r\n$A=3^{2004}+25$\r\n1. How many digits A have(in base $10$)? (easy, I found that $957$ digits)\r\n2. $S=$sum of all this digits. How much is S? \r\n3. How many do zeroes digits appear in A?", "Solution_1": "I doubt whether this can be done without using a computer.", "Solution_2": "Yes, Arnel. But using caculor is not a good way and it is meanless.\r\n I hope some could post a good solution for it! I know that it seems difficult but let try it ! :D", "Solution_3": "A \"good\" (elegant) solution simply cannot exist...", "Solution_4": "I know the answer to 1, however, I'm still thinking about 2 and 3. Here\r\ns 1's solution that can somehow be done without a calculator. I'm still thinking about doing it without a calculator...hopefully, it is possible.\r\n[hide]\n$\\\\\\lfloor\\log 3^2004+1\\rfloor\\\\=\\lfloor2004\\log 3+1\\rfloor\\\\=\\lfloor 956.15+1\\rfloor\\\\=957$\n[/hide]\r\n\r\nMasoud Zargar" } { "Tag": [], "Problem": "Do you know of any free (preferably free as in freedom) non-demo non-trial non-shareware music notation programs for Windows that can save/export as MIDI?\r\n\r\nI'm currently using Finale Notepad 2006, but it doesn't save/export as a MIDI file.", "Solution_1": "NoteWorthy Composer, I'm not sure you can save as a MIDI file, but I know it works. You could try.", "Solution_2": "The website says that it's a 30-day evaluation trial, and according to download.com and other sites, it's shareware. :(", "Solution_3": "-_- I've been on 30-day trial for a few years then...\r\n\r\nIs Finale Notepad 2006 good for just writing though?", "Solution_4": "Yes. Mr. Muer recommends it.\r\n\r\nDoes the demo have nag screens, etc.?", "Solution_5": "[quote=\"Eli\"]Yes. Mr. Muer recommends it.\n\nDoes the demo have nag screens, etc.?[/quote]\r\n\r\nnag screens? \r\n\r\nDo we have to use notepad for the assignment (and it's optional, correct?)?", "Solution_6": "What I mean is if there are major annoyances, etc. Are there any important features you can't use?\r\n\r\nMr. Muer wants us to use Notepad (probably because he doesn't know of anything else), but if it's a music file, he'll probably be fine with it.", "Solution_7": "[quote=\"Eli\"]What I mean is if there are major annoyances, etc. Are there any important features you can't use?\n\nMr. Muer wants us to use Notepad (probably because he doesn't know of anything else), but if it's a music file, he'll probably be fine with it.[/quote]\r\n\r\nNo, I believe it does not have \"nag screens.\" But don't take my word for it.", "Solution_8": "what is a MIDI?\r\n\r\nFor future purposes, if you don't need a WYSIWYG music editor then use musixtex package in MiKTeX. Ask on the TeX forum. I don't know how to use it myself :P", "Solution_9": "It's a type of music file (see http://en.wikipedia.org/wiki/Musical_Instrument_Digital_Interface), like mp3, wav, etc.\r\n\r\nI know. I like to have a WYSIWYG editor for music. I don't have to worry about the syntax with them.", "Solution_10": "Currently I use GenieSoft Score Writer v2.6. It costs money, but it's only about 30 dollars. It does all the MIDI import/export stuff with ease and multiple tracks. It has a WYSIWYG interface with easy-to-use mouse and keyboard controls. With the given price, I'd say it's one of the more economical software packages out there.\r\n\r\nHowever, changing the scores to PDF give some problems (yes, I use PDF pro v7).", "Solution_11": "sibelius :lol:" } { "Tag": [ "function", "algebra proposed", "algebra" ], "Problem": "$ S$ is a set of functions such that $ f: N \\rightarrow R$ and \r\n(1)$ f(1) \\equal{} 2$ \r\n(2) $ f(n \\plus{} 1) \\ge f (n) \\ge \\frac {n}{n \\plus{} 1} f(2n)$ .\r\n Determine the smallest $ M$ such that $ f(n) < M$ (for any $ f \\in S$ and for any $ n \\in N$)", "Solution_1": "Obviosly $ f(n)\\le f(2n)\\le (1\\equal{}\\frac 1n )f(n)$, therefore $ f(\\infty )\\le \\prod_{k\\equal{}0}^{\\infty }(1\\plus{}\\frac{1}{2^k})\\equal{}g(\\frac 12$, were $ g(x)\\equal{}\\prod_{k\\equal{}0}^{\\infty }(1\\plus{}x^k)$ (|x|<1).", "Solution_2": "I also managed to prove that $ f(\\infty) \\leq g(1/2)$ , but why $ g(1/2)$ would be the minimum value ?", "Solution_3": "$ g(\\frac {1}{2})$=? It's not finally answer.", "Solution_4": "[quote=\"Svejk\"]I also managed to prove that $ f(\\infty) \\leq g(1/2)$ , but why $ g(1/2)$ would be the minimum value ?[/quote]\r\nWe can define (maximal possible mean) $ f(n)\\equal{}\\prod_{i\\equal{}0}^{k}(1\\plus{}\\frac{1}{2^i})$, were $ k\\equal{}[log_2(n\\minus{}1)],n>1$.\r\n$ g(x)$ is special function considered by Euler." } { "Tag": [ "probability" ], "Problem": "Six people arrive to purchase $10 tickets for a movie. Three of them have only $10 dollar bills and 3 have only $20 dollar bills. The person selling the tickets initially has no change. What is the probability that the people line up in such a way that the person selling the tickets always has enough change?", "Solution_1": "[hide]THe probatilty is 1/4.\nObviously, the last person to enter the movie theater must have been a person with a 20 dollar bill. THerefore, all three people with 10 dollar bills and 2 people with 20 dollar bills must have entered before he did. 3/6*2/5*1/4*3/3*2/2=1/20. Of course, the people with 10 dollar bills didn't have to go first, but it makes no difference becuase the numerators and denominators are just switched around accordingly. Now, we just need to figure out how many ways this can be done. Obviously, the first person has a 10 dollar bill and the last has a 20 dollar bill. $4 \\choose 2$ is 6 so there's 6 ways to choose the remaining 4 people, except that 10,20,20 is not allowed. THerefore, there's 6-1=5 middle possibilities and 5*1/20=1/4.[/hide]" } { "Tag": [], "Problem": "Whats me to solve for this system\r\n\r\n( 2x-4y=32\r\n{3x+5y=4\r\n(", "Solution_1": "this system can be turned into 6x-12y=96 and 6x+10y=8, leaving us with y = -4 and x = 8.", "Solution_2": "Multiply $2x-4y=32$ by $3$ to get $6x-12y=96$. Multiply $3x+5y=4$ by $2$ to get $6x+10y=8$. Now, subtract the first equaiton from the second to get $22y=-88$. Divide both sides by $4$ to get $y=-4$. Now, plug in y back into the first equation to get $2x-4(-4)=32$ or $2x=16$. Divide both sides by $2$ to get $x=8$.\r\n\r\nProblems like these are more middle school level and not high school level and can be posted here: http://www.artofproblemsolving.com/Forum/index.php?f=299 instead.", "Solution_3": "[hide]I would just do substitution myself, because i'm wierd. Add 4y to both sides to get $\\displaystyle{2x=4y+32}$ Then divide by 2...\n$\\displaystyle{x=2y+16}$ Substitute that for x and solve for y... then sub y in for x in this equation and you get x=8 and y=-4...[/hide]", "Solution_4": "[quote]because i'm wierd[/quote]\r\n\r\nur also weird cuz u spelled weird wrong. :rotfl: \r\njust kidding.\r\nno offense." } { "Tag": [ "trigonometry", "algebra proposed", "algebra" ], "Problem": "$ \\sqrt[3]{x^3\\minus{}12x^2\\plus{}27x\\minus{}8}\\equal{}2x\\plus{}\\sqrt[3]{x^3\\minus{}3x}$", "Solution_1": "hello, this equation has no real solution.\r\nSonnhard.", "Solution_2": "i think there is some mistakes because this equation has more than one real root", "Solution_3": "[quote=\"hoangvn.fix\"]$ \\sqrt [3]{x^3 \\minus{} 12x^2 \\plus{} 27x \\minus{} 8} \\equal{} 2x \\plus{} \\sqrt [3]{x^3 \\minus{} 3x}$[/quote]\r\n\r\n$ x_1 \\equal{} 2\\cos \\frac {4\\pi}9$\r\n\r\n$ x_2 \\equal{} 2\\cos \\frac {10\\pi}9$\r\n\r\n$ x_3 \\equal{} 2\\cos \\frac {16\\pi}9$\r\n\r\n[hide=\"Why?\"]Let $ u\\equal{}\\sqrt[3]{x^3\\minus{}3x}$.\n\nThe equation is $ \\sqrt[3]{u^3\\minus{}12x^2\\plus{}30x\\minus{}8}\\equal{}2x\\plus{}u$\n\nRaising to power $ 3$ gives : $ u^3\\minus{}12x^2\\plus{}30x\\minus{}8\\equal{}8x^3\\plus{}12x^2u\\plus{}6xu^2\\plus{}u^3$\n\n$ \\iff$ $ u^3\\minus{}12x^2\\plus{}30x\\minus{}8\\equal{}8u^3\\plus{}24x\\plus{}12x^2u\\plus{}6xu^2\\plus{}u^3$\n\n$ \\iff$ $ 4(u^3\\plus{}1)\\plus{}6x^2(u\\plus{}1)\\plus{}3x(u^2\\minus{}1)\\equal{}0$\n\n$ \\iff$ $ (u\\plus{}1)(4u^2\\minus{}4u\\plus{}4\\plus{}6x^2\\plus{}3ux\\minus{}3x)\\equal{}0$\n\n$ \\iff$ $ (u\\plus{}1)\\frac{87(8u\\plus{}3x\\minus{}4)^2 \\plus{}(87x\\minus{}12)^2 \\plus{}4032}{87\\cdot 16}\\equal{}0$\n\n$ \\iff$ $ u\\equal{}\\minus{}1$ (since the other factor is always $ >0$\n\n$ \\iff$ $ x^3\\minus{}3x\\plus{}1\\equal{}0$.\n\nIt's easy to see that this equation has 3 real roots all in $ (\\minus{}2,\\plus{}2)$ and so let $ x\\equal{}2\\cos v$ and so $ 8\\cos^3v\\minus{}6\\cos v\\equal{}\\minus{}1$\n\n$ \\iff$ $ \\cos 3v\\equal{}\\minus{}\\frac 12 \\equal{}\\cos \\frac{4\\pi}3$\n\nHence the result[/hide]" } { "Tag": [ "MIT", "college", "Support" ], "Problem": "Can anyone tell me how much money cost one year at MIT and is there some scholarships?", "Solution_1": "[url]http://web.mit.edu/sfs/[/url]\r\nHope this helps. :)", "Solution_2": "Yeah,thanks.", "Solution_3": "MIT sure is an 'expensive' destination for an international student.", "Solution_4": "[quote=\"Gen8\"]MIT sure is an 'expensive' destination for an international student.[/quote]\r\nWell,there're scholarschips,right.", "Solution_5": "I am sure that If you get outstanding academic achivements,your government will support you with a large fund,quite enough for you to follow some higher degree in foreign countries.", "Solution_6": "I hope to have outstanding results,but I'm still at the beginning.What exams should I take to enter an American university?I know SAT 1 is obligatory, but how many SAT subjects are reqiured?My major is maths but what should the other subjects be? Are they optional or obligatory?\r\nWhat scores do I have to get to have to have chances for some grant or scholarships. I would appreciate any information.", "Solution_7": "[quote=\"barcelona\"]I hope to have outstanding results,but I'm still at the beginning.What exams should I take to enter an American university?I know SAT 1 is obligatory, but how many SAT subjects are reqiured?My major is maths but what should the other subjects be? Are they optional or obligatory?\nWhat scores do I have to get to have to have chances for some grant or scholarships. I would appreciate any information.[/quote]\r\n\r\nmost schools require 2-3 subject tests and some have a requirement that you take Math II as one of them. What score should you get? As high as you can if you're applying to MIT. But an 800 on everything wont guarantee a scholarship. Scholarships are almost never based on standardized tests." } { "Tag": [ "analytic geometry", "function" ], "Problem": "The point $ (a,b)$ is located in the third quadrant in the Cartesian plane. The coordinates are changed according to the functions given: $ f(a)\\equal{}a^2 \\plus{} 3$ and $ g(b)\\equal{}b\\minus{}4$. In which quadrant (1, 2, 3, or 4) is $ (f(a),g(b))$ located?", "Solution_1": "Third quadrant means (neg,neg). Say that it is (-1,-1). Using the functions, we would get (4,-5), which is in the fourth quadrant. Our answer is 4.", "Solution_2": "As Kiseki said, the third quadrant consists of the points that are (neg,neg).\r\n\r\nThe square of all negative numbers are positive numbers, and adding a positive number to a positive number always yields a positive number. \r\n\r\nAnd negative number subtracted by a positive numbers is a negative number.\r\n\r\nSo, we have the answer (pos,neg). This is represented by the $ \\boxed{4}$ the quadrant." } { "Tag": [ "trigonometry", "calculus", "integration", "calculus computations" ], "Problem": "u''(x)+c*u(x)=0 -differential equation.\r\n\r\nI was studying differential equations and when i reached the equations of grade 2 i encountered this equation.But it was saying''it is verified that'' and it just gave me the form of u(x).\r\nCould somebody please demonstrate the form of u(x)?", "Solution_1": "A solution of characteristic equation $\\lambda^{2}+c=0$ is $\\lambda=\\pm i\\sqrt{c}$.\r\nSo, a general solution to find is $y=C_{1}\\cos (\\sqrt{c}x)+C_{2}\\sin (\\sqrt{c}x)$.\r\n($C_{1}, C_{2}$ : Constant of Integration)", "Solution_2": "This is what i am trying to understand:why is y=C1cos(sqrt(c)*x)+.... ?...and i don't want to begin with the y and reach that y''+cy=0 .\r\nThere must be a way of reaching the general solution!", "Solution_3": "[quote=\"radac_mail\"]This is what i am trying to understand:why is y=C1cos(sqrt(c)*x)+.... ?...and i don't want to begin with the y and reach that y''+cy=0 .\nThere must be a way of reaching the general solution![/quote]\r\nIf it defines it as $z_{1}=\\cos \\sqrt{c}x+i \\sin \\sqrt{c}x, \\ z_{2}=\\cos \\sqrt{c}x-i \\sin \\sqrt{c}x$, \r\n$z_{1}'=-\\sqrt{c}\\sin \\sqrt{c}x+i \\sqrt{c}\\cos \\sqrt{c}x=i \\sqrt{c}(\\cos \\sqrt{c}x+i \\sin \\sqrt{c}x)=\\lambda_{1}z_{1}$\r\n$z_{1}''=\\lambda_{1}z_{1}'=\\lambda_{1}^{2}z_{1}$\r\nSo, $z_{1}''+c\\cdot z_{1}=(\\lambda_{1}^{2}+c)z_{1}=0$.\r\nLikewise, $z_{2}''+c\\cdot z_{2}=(\\lambda_{2}^{2}+c)z_{2}=0$\r\n$z_{1}, z_{2}$is a complex number value solution of $U''(x)+c\\cdot U(x)=0$.\r\nThese linear combination\r\n$U_{1}=\\frac{z_{1}+z_{2}}{2}=\\cos \\sqrt{c}x, \\ U_{2}=\\frac{z_{1}-z_{2}}{2i}=\\sin \\sqrt{c}x$\r\nis solutions of $U''(x)+c\\cdot U(x)=0$ again.\r\nTherefore, $U(x)=C_{1}\\cos (\\sqrt{c}x)+C_{2}\\sin (\\sqrt{c}x)$. \r\n($C_{1}, C_{2}$ : Constant of Integration)" } { "Tag": [], "Problem": "Se considera numerele reale pozitive $\\ a,b,c,x,y,z$, cu $\\ a>x,b>y,c>z$ si $\\ xyz=\\frac{1}{256^{2}}$.Aratati ca e adevarata inegalitatea $\\ (a+x^{2})(b+y^{2})(c+z^{2})\\geq\\frac{(a-x)(b-y)(c-z)}{(5x^{2}+a^{2}+9)^{4}(5y^{2}+b^{2}+9)^{4}(5z^{2}+c^{2}+9)^{4}}$", "Solution_1": "Nimeni nu se baga la inegalitatea asta? :(", "Solution_2": "ma io nu pricep o chestie...\r\n$a+x^{2}>a$.\r\nnumitorul membrului drept este enorm (mai mare decat $9^{12}$).\r\ndeci\r\nfacand cele mai directe aproximari, obtin\r\n$LHS>abc>(a-x)(b-y)(c-z)>9^{-12}(a-x)(b-y)(c-z)>RHS$.\r\nori am baut eu gaz, ori ceva nu`i bine p`acolo...", "Solution_3": "porneste de la faptul ca $\\frac{a+x^{2}}{a-x}=\\frac{x^{2}+x}{a-x}+1$...", "Solution_4": "io intrebam altceva\r\nce e gresit in rezolvarea mea de mai sus ?\r\ndaca rezolvarea mea e corecta, atunci inegalitatea e al dracului de slaba, si conditia cu xyz e in plus", "Solution_5": "Nu te-nfuria mai...cred ca e slaba.. :roll:", "Solution_6": "nu ma infurii\r\ndar am mai patit`o cu inegalitati sa dau o solutie \"corecta\" si sa ma prind dupaia ca de fapt nu e bine. \r\nd`asta cand vad semnu $\\geq$ ma gandesc ca trebuie sa fie un caz de egalitate, si daca mie imi da cu $>$ atunci ceva nu e bine..", "Solution_7": "Stii, unii propunatori mai pun si semnul asta $\\geq$ ca sa mai deruteze pe elevi..chiar daca normal ar trebui sa puna $\\ >$.Asta am vrut eu aici..pacalici. :)", "Solution_8": "Nu.O problema[sau parte din anuntul ei] nu trebuie sa deruteze elevul..Si semnul $\\geq$ nu are niciun sens dak egalitatea nu are loc..DEsi logic vorbind inseamna\"mai mare[b] sau[/b] egal..parerea mea", "Solution_9": "Mai, eu asa am mai vazut.Inegalitati menite sa mai deruteze concurentul..intr-un fel, asta e lipsa de fair-play.. :)" } { "Tag": [ "modular arithmetic", "number theory proposed", "number theory" ], "Problem": "if $\\frac{p^a-1}{p-1}=2^n$ that $p$ is a prime number and $n,a$ are positive integers.\r\nfind number of divisor of $n.a$", "Solution_1": "If a=2 then 2^n-1 is prime so the number of divisors of n is 2 since n must be prime. For a=2,3 its fairly easy to see there are no solutions. I don't know about the number of divisors in general.", "Solution_2": "1) Obviously $p>2$ else LHS would be odd\r\n2) We note that $a$ must be a power of $2$ in fact if it's not we would have let's say $d|a$ with $d$ an odd number $>1$. Then we will have $\\frac{p^d-1}{p-1} | \\frac{ p^a -1 } {p-1} = 2^n$ and so $p^{d-1} + p^{d-2} + ... + 1|2^n$. Thus $p^{d-1} + p^{d-2} + ... + 1$ must be a power of $2$ but it's a sum of an odd number of odd number and thus is odd and it's obviously $>1$.\r\n\r\n3) Thus $a=2^k$. Rewriting we get:\r\n\r\n$\\frac{p^{2^k} -1}{p-1} = (p^{2^{k-1}}+1)(p^{2^{k-2}} +1 )... (p+1) = 2^n$\r\n\r\nThus all the factors in LHS must be powers of $2$.\r\nBut $p^2+1 \\equiv 2 \\pmod{4}$ and so it can't be a power of $2$.\r\nThus $k=1$\r\n\r\n4) For $k=1$ and so $a=2$ we get $p= 2^n-1$ and so $n$ must be prime. So $an$ has always $4$ divisors" } { "Tag": [ "AMC", "USA(J)MO", "USAMO", "AIME", "AMC 12", "AMC 10", "AMC 10 A" ], "Problem": "Will I get 1.5 or 2.5 points for not answering a question? \r\n\r\nBilly", "Solution_1": "2.5?\r\n\r\nIs this a trick question?", "Solution_2": "I think it's changing to 1.5 in 2007. Forgot where I read that, though, so I may be wrong.", "Solution_3": "[quote=\"bookaholic\"]I think it's changing to 1.5 in 2007. Forgot where I read that, though, so I may be wrong.[/quote]\r\n\r\nThat's correct. It was in the USAMO qualifier book as well as on the AMC site (I think).", "Solution_4": "[quote=\"bookaholic\"]I think it's changing to 1.5 in 2007. Forgot where I read that, though, so I may be wrong.[/quote]\r\n\r\nif they change it to 1.5, are they changing the minimum score? :|", "Solution_5": "er, the minimum score is always going to be 0, pretty much?", "Solution_6": "I think he means minimum score to qualify, and no, as far as I know, they are not planning to. Why should they?", "Solution_7": "[quote=\"JesusFreak197\"]I think he means minimum score to qualify, and no, as far as I know, they are not planning to. Why should they?[/quote]\r\n\r\nErr.. maybe because most people will have a significantly lower score than before?", "Solution_8": "According to the 2005 Summary of AMC Results/Awards,\r\n\r\n[quote=\"AMC 12 Chair\"]Beginning in 2002 the number of points awarded for a blank response on the AMC12 was increased from 2 to 2.5. As a result, students could qualify for the AIME by solving as few as 11 problems. Analysis of student performances since then reveals that the average number of blank responses by AIME qualifiers has increased from about 8 to 11. This trend, suggesting that many bright students are simply igoring problems of even moderate difficulty, runs counter to our goals. For that reason, beginning in 2007, the reward for a blank response will decrease from 2.5 to 1.5 points. It should be emphasized that in making this change, we are not attempting to reduce the number of AIME qualifiers. Because we are simultaneously working to improve the overall success rate of the first ten problems, we expect that the number of students achieving the minimum qualifying score of 100 will remain roughly constant.[/quote]\r\n\r\nBTW I like this change, because 2 blank responses is almost 1 correct answer is... just not right.", "Solution_9": "Scoring on the 2006 AMC 10 A, 2006 AMC 10 B, 2006 AMC 12 A and the 2006 AMC 12 B will be 6 points for each correctly answered question, 2.5 points for each blank, or unanswered, question and 0 points for an incorrect answer.\r\n\r\nSee for instance item 4 on the facisimile of the 2006 AMC 10 A cover page which is on our website at:\r\nhttp://www.unl.edu/amc/d-publication/d1-pubarchive/2005-6pub/2006AMC1012/html/06tm-26-10Acover.html\r\n(or go to the AMC webstie, choose 2006 AMC 10A/ AMC 12 A Teacher's Manual from the What's New column, then using the table of contents, go to page 26).\r\n\r\nIn 2007, the scoring will change to 6 points for each correctly answered question, 1.5 points for each blank, or unanswered, question and 0 points for an incorrect answer. This scoring change was announced in the 2005 AMC 101/12 Summary of Results and Awards, will be announced again in the 2006 AMC 10/12 Summary of Results and Awards and will be noted on the advertising brochures for the 2007 contests. The scoring system will be noted on the cover of the 2007 AMC contests. The implications of the scoring change will be noted in the 2007 AMC 10/AMC 12 Teachers' Manual.\r\n\r\nQualification for the 2007 AIME from the 2007 AMC 10 will continue to be a total of 120 points, or in the top 1% on the AMC 10.\r\nQualification for the 2007 AIME from the 2007 AMC 12 will continue to be a total of 100 points or in the top 5% on the AMC 12.\r\n\r\nSteve Dunbar\r\nAMC Director", "Solution_10": "[quote=\"probability1.01\"][quote=\"JesusFreak197\"]I think he means minimum score to qualify, and no, as far as I know, they are not planning to. Why should they?[/quote]\n\nErr.. maybe because most people will have a significantly lower score than before?[/quote]\r\n\r\nNot all that much. I'm pretty sure that part of the point is to eliminate those people who qualify for AIME by answering maybe 13 questions on the AMC12 (in case of one wrong) and leaving the rest blank. That shouldn't allow you to qualify. Now, you'll have to answer at least 14 correctly, probably more, and 19 on the AMC10.", "Solution_11": "Umm... well even if the average borderline qualifying person leaves 8 questions blank, that's still 8 whole points, which disqualifies a lot of people, at least 1000 from a cursory glance at last year's statistics. Obviously I'm ignoring some factors here, but it still strikes me as quite a lot.", "Solution_12": "But it seems that the AMC people will make the first 10 problems easier.", "Solution_13": "It gives 2.5 for unanswered Q is becoz it doesn't want us to GUESS\r\nbut....now has been changed to 1.5\r\nwut's that mean?", "Solution_14": "But i figured that it would be a little pointless to change the minimum qualification score because then the qualification would be about the same difficulty, and there's a smaller penalty for guessing.", "Solution_15": "Maybe this will decrease the inflating USAMO qualifying indexes, and place some more emphasis on AMC over AIME. \r\n\r\n...on second thought, it probably will only have a small effect on that.", "Solution_16": "It shouldn't have much effect on the USAMO qualifying indices (although the USAMO expansion should). This change is targeted at the borderline AIME qualifiers, who generally do not have much effect on USAMO cutoffs. People on the borderline for USAMO probably aren't leaving very many questions blank, so they won't lose many points. I suppose it could still lower the cutoff by a few points, though.", "Solution_17": "AMC over AIME? Not in this lifetime... :P \r\n\r\nThe AIME is the coolest part, no doubt. Plus, if more people demonstrate that they can do better on the AMCs, I'm thinking they will have to make AIMEs harder. Uh-oh. There goes my hopes...", "Solution_18": "Doesn't matter much to me.\r\nIn the past, I've answered every question anyways. Well, that was on the AMC10. And, of course, I would make 2 or so minor errors or mistakes.", "Solution_19": "It would ruin the point of changing it from 2.5 to 1.5 if they also lowered the cutoff for AIME.", "Solution_20": "[quote=\"eryaman\"]Maybe this will decrease the inflating USAMO qualifying indexes, and place some more emphasis on AMC over AIME. \n\n...on second thought, it probably will only have a small effect on that.[/quote]\r\n\r\n\r\nWell they will also try to make the AMC's easier(first 10 problems easier, which hmmm does not really help too many people) so yeah your score will stay roughly the same.\r\n\r\nBut this does hurt a lot if you are leaving like 5-10 blanks (so yeah try not to leave too many blanks that's the intention of the AMC people)", "Solution_21": "Really good change ... about time. It was ridiculous that 21 right and 4 blank got you almost the same score as 23 right. Generally there isn't a single challenging problem until #21 at the absolute earliest.", "Solution_22": "Yeah, it was quite ridiculous to qualify for AIME without getting a good number right.", "Solution_23": "Yeah 20 right, 5 blank gets you a higher score than 22 right... that's not fair also. Basically I don't see how the current AMC scores mean anything, like a difference of 0.5 point is a much bigger difference than a difference of 6 points.\r\n\r\nWhy can't they just make AMC short-answer format(like AIME) instead of multiple choice?", "Solution_24": "[quote=\"beta\"]Yeah 20 right, 5 blank gets you a higher score than 22 right... that's not fair also. Basically I don't see how the current AMC scores mean anything, like a difference of 0.5 point is a much bigger difference than a difference of 6 points.\n\nWhy can't they just make AMC short-answer format(like AIME) instead of multiple choice?[/quote]\r\n\r\nIt would scare away newbies.", "Solution_25": "I've noticed that on the AMC 10 booklet, it does not actually say that the cutoff is 120 points, just top 1%. So is 120 a generally accepted cutoff score to represent the top 1%, or is it actually part of the competition policy?", "Solution_26": "[quote=\"log0\"]I've noticed that on the AMC 10 booklet, it does not actually say that the cutoff is 120 points, just top 1%. So is 120 a generally accepted cutoff score to represent the top 1%, or is it actually part of the competition policy?[/quote]\r\n\r\nYou might have an old booklet. I believe the policy changed in 2004 to include any 120+ AMC10 scores as qualifying.", "Solution_27": "[quote=\"paladin8\"][quote=\"log0\"]I've noticed that on the AMC 10 booklet, it does not actually say that the cutoff is 120 points, just top 1%. So is 120 a generally accepted cutoff score to represent the top 1%, or is it actually part of the competition policy?[/quote]\n\nYou might have an old booklet. I believe the policy changed in 2004 to include any 120+ AMC10 scores as qualifying.[/quote]\r\n\r\nI noticed that when reading the instructions last year. So basically the baseline is either 120 or the top 1%, whichever is lower.", "Solution_28": "[quote=\"hockeyadi23\"][quote=\"paladin8\"][quote=\"log0\"]I've noticed that on the AMC 10 booklet, it does not actually say that the cutoff is 120 points, just top 1%. So is 120 a generally accepted cutoff score to represent the top 1%, or is it actually part of the competition policy?[/quote]\n\nYou might have an old booklet. I believe the policy changed in 2004 to include any 120+ AMC10 scores as qualifying.[/quote]\n\nI noticed that when reading the instructions last year. So basically the baseline is either 120 or the top 1%, whichever is lower.[/quote]\r\n\r\nYeah. It's more like the AMC12 qualifying now, only 20 points higher and 1% instead of 5%.", "Solution_29": "A kid from our school actually did answer only 12 questions. Checked them over several times and slept for the remaining time. Poor kid bubbled one in wrong...\r\n\r\nI think this change is really necessary in order to avoid people from spending time strategizing over the test. In that same time, the person, if preparing, could practice more math, or if it's during the actual test, could be working on math. I personally would prefer an SAT-type system (say, base score 25, +5 for correct answer, -1 for wrong answer)" } { "Tag": [ "number theory", "relatively prime" ], "Problem": "1.) Prove that for any even $n>1$, $n^{2}-1$ divides $2^{n!}-1$. \r\n\r\n2.) Prove that for any $n>2$ the number $1989$ divides $n^{n^{n^{n}}}-n^{n^{n}}$.", "Solution_1": "What, how can 2) be true? 1989=9*13*17, am I missing something?", "Solution_2": "Hm these problems look familiar.\r\n\r\nOh wait I've seen these before.\r\n\r\nOh wait that's right I went to AMP :blush: \r\n\r\nYes the problem is right. Amazing isn't it :lol:", "Solution_3": "[hide=\"1\"] \\[n^{2}-1|2^{n!}-1 \\iff 2^{n!}\\equiv 1\\mod{(n^{2}-1)}\\] \\[\\iff 2^{n!}\\equiv 1\\mod{n-1}, 1\\mod{n+1}\\] because n-1 and n+1 are relatively prime, being both odd.\n\nBut 2 is also relatively prime to both $n-1$ and $n+1$, and therefore is a unit in both mods.\nThus, $2^{n!}\\equiv 1\\mod{n-1}, \\mod{n+1}\\iff \\phi(n-1)|n!, \\phi(n+1)|n!$\n\nBut $\\phi(k)0$)\r\nV\u00e0 $f(4x^{2}-(8a-1)x+5a^{2})=f(x^{2}-2(a+1)x-a^{2})$................\r\n....$\\to 3x^{2}-6ax+6a^{2}=0$\r\nx^2-2ax+2a^2=0\r\n........... a=0?????", "Solution_5": "[quote=\"VANCHANH123\"][quote=\"Beat\"]Let a be real parameter. Find all a for which the equality $x^{2}+(1-2a)x+2a^{2}+lg(4x^{2}-(8a-1)x+5a^{2})=lg(x^{2}-2(a+1)x-a^{2})$ has only one solution.[/quote]\n$f(x)=%Error. \"logx\" is a bad command.\n+3x$\n(V\u1eddi$x >0$)\nV\u00e0 $f(4x^{2}-(8a-1)x+5a^{2})=f(x^{2}-2(a+1)x-a^{2})$................\n....$\\to 3x^{2}-6ax+6a^{2}=0$\nx^2-2ax+2a^2=0\n........... a=0?????[/quote]\r\nnot ....\r\nno value a" } { "Tag": [ "inequalities", "complex numbers", "inequalities proposed" ], "Problem": "The triangle ABC has A=30 and AB=3/4 AC. Find the point P inside the triangle which minimises 5PA+4PB+3PC\r\n\r\n(16 th vietman mo , 1978) thanks!", "Solution_1": "Here's the best I could do up until now:\r\n\r\nWe take A to be the origin of the plane, B(3i), C(4w) and D(4) (the things in the brackets are complex numbers; w=cos60+i*sin60). We need to find a z s.t. the sum S=5|z|+4|z-3i|+3|z-4w| is minimised.\r\n\r\n5|z|=|4z-3iz|; 4|z-3i|=|12i-4z|; 3|z-4w|=|3iz-12iw|, so \r\nS=|4z-3iz|+|12i-4z|+|3iz-12iw|>=|4z-3iz+12i-4z+3iz-12iw|=|12i-12iw|=12, but I have no idea if this value is reached, or if it's reached for a z inside the triangle ABC. The condition is that the complex numbers z_1=4z-3iz, z_2=12i-4z and z_3=3iz-12iw are collinear and have the same direction: z_3/z_2 and z_1/z_2 are both positive reals. However, I'm a bit scared of the calculations. Can anyone check this? It could work.\r\n\r\nI was especially encouraged by the nice-looking result (12) :D It doesn't mean it's correct, though.." } { "Tag": [ "Euler", "number theory", "real analysis", "real analysis unsolved" ], "Problem": "Calculate: $ \\sum_{n\\equal{}1}^{\\infty}\\frac{(\\minus{}1)^{n\\plus{}1}}{2n\\plus{}1}\\cdot\\frac{\\zeta (2n)}{2^{2n}}.$", "Solution_1": "There's an obvious thing to try, which is to insert the definition of $ \\zeta$ and interchange the order of summation. Doing that, we discover that is it the exact same problem as [url=http://www.artofproblemsolving.com/Forum/viewtopic.php?t=218877]this.[/url]", "Solution_2": "$ \\sum_{n \\equal{} 1}^{\\infty}\\frac {( \\minus{} 1)^{n \\plus{} 1}}{2n \\plus{} 1}\\cdot\\frac {\\zeta (2n)}{2^{2n}} \\equal{} ?$\r\n\r\nThis is all I got so far.\r\n\r\n$ \\sum_{n \\equal{} 1}^{\\infty}\\frac {( \\minus{} 1)^{n \\plus{} 1}}{2n \\plus{} 1} \\equal{} 1 \\minus{} \\frac {\\pi}{4}$\r\n\r\n$ \\sum_{n \\equal{} 1}^{\\infty}\\frac {1}{2^{2n}} \\equal{} \\frac {1}{3}$\r\n\r\n$ \\zeta (2n) \\equal{} \\frac {2^{2n \\minus{} 2}\\pi^{2n}|B_{2n}|}{2n!}$ This is some sort of Euler identity from my Intro to Number Theory book. I just can't figure out how to incorporate it into the summation. Maybe someone will be able to telescope some part of it? ^_^ Or not use the derived values for the other parts and be able to cancel stuff. (Just trying to add input >_<) Also $ B$ is a Bernoulli number.\r\n\r\n\r\nEdited post with regards to Carcul.\r\nP.S -> My apologies :)", "Solution_3": "Exactly how did you found that $ \\sum_{n \\equal{} 1}^{\\infty}\\frac {1}{2^{2^{n}}} \\equal{} \\frac {1}{3}$?", "Solution_4": "It's not, of course -- it was a typo for $ \\sum_{n \\geq 1} \\frac{1}{2^{2n}}$." } { "Tag": [ "geometry", "circumcircle", "analytic geometry", "trigonometry", "inequalities", "calculus", "function" ], "Problem": "Let $ABC$ is a triangle with the circumcircle $(O)$ and $G$ is the centroid. $AG,BG, CG$ meet $(O)$ at $A', B', C'$ respectively. Prove that $S(A'B'C')\\geq\\ S(ABC)$ where S(ABC) means the area of the triangle ABC\r\n\r\nnice?", "Solution_1": "no one? I think it's very nice and not so hard", "Solution_2": "Notice that $3\\frac{S(GB'C')}{S(ABC)} = \\frac{S(G'B'C')}{S(GBC)} = \\frac{GB'.GC'}{GB.GC} = \\frac{(R^2-OG^2)^2}{GB^2.GC^2}$.\r\n\r\nAccording to Leibnitz's theorem, we have\r\n\r\n$3R^2-3OG^2 = GA^2+GB^2+GC^2$\r\n\r\nHence $3\\frac{S(A'B'C')}{S(ABC)} = \\sum3\\frac{S(GB'C')}{S(ABC)} = (GA^2+GB^2+GC^2)^2 \\sum \\frac{1}{GB^2.GC^2} \\geq 3$\r\n\r\nThus $S(A'B'C') \\geq S(ABC)$.", "Solution_3": "[quote=\"treegoner\"]Notice that $3\\frac{S(GB'C')}{S(ABC)} = \\frac{S(G'B'C')}{S(GBC)} = \\frac{GB'.GC'}{GB.GC} = \\frac{(R^2-OG^2)^2}{GB^2.GC^2}$.\n\nAccording to Leibnitz's theorem, we have\n\n$3R^2-3OG^2 = GA^2+GB^2+GC^2$\n\nHence $3\\frac{S(A'B'C')}{S(ABC)} = \\sum3\\frac{S(GB'C')}{S(ABC)} = (GA^2+GB^2+GC^2)^2 \\sum \\frac{1}{GB^2.GC^2} \\geq 3$\n\nThus $S(A'B'C') \\geq S(ABC)$.[/quote]\r\n\r\n Your solution is very good . I have to use Kamarta ineq in my solution after proving $(A',B',C')>>(A,B,C)$", "Solution_4": "treegoner, can you prove that $r_{ABC}\\leq\\ r_{A'B'C'}$ (r means the inradii)", "Solution_5": "That is a very nice question. I did think about it after doing this problem. But up till now, I can't say anything. Are you sure of this result?", "Solution_6": "[quote=\"treegoner\"]That is a very nice question. I did think about it after doing this problem. But up till now, I can't say anything. Are you sure of this result?[/quote]\r\n\r\n I have proved it for twice, but who know? maybe i have made mistakes for twice ???!!!", "Solution_7": "No one for the second one?", "Solution_8": "how it is demonstrated the Leibnitz's theorem ? :blush:", "Solution_9": "I 'm glad that finally I got some free time this morning to think again about this problem. Here is my solution to the problem $r_{ABC} \\leq r_{A'B'C'}$.\r\n\r\nBut first, I need to answer js131090's question. Leibnizt's theorem says that if $(x: y: z)$ is the barycentric coordinate of a point $P$, then for every point $M$ we have the equality $xMA^2 + yMB^2 + zMC^2 = (x+y+z)MP^2 + xPA^2 + yPB^2 + zPC^2$. In particular, if $P$ is the centroid $G$ and $M$ is the circumcenter $O$, Leibnitz's theroem yields that $3R^2 = 3OG^2 + GA^2+GB^2+GC^2$.\r\n\r\nLet's get back to the problem. Let $A_1B_1C_1$ be the pedal triangle of $G$ with respect to $ABC$. Then according to sine theorem and Euler's formula for area of pedal triangle, we obtain the sides and area of $A_1B_1C_1$ are respectively $\\frac{2}{3}m_a.sinA, \\frac{2}{3}m_b.sinB, \\frac{2}{3}m_c.sinC, \\frac{4S(a^2+b^2+c^2)}{9R^2}(=\\frac{1}{4}S(1-\\frac{OG^2}{R^2}))$\r\n\r\nWe have $\\frac{B'C'}{BC} = \\frac{GC'}{GB} = \\frac{GC.GC'}{GB.GC} = \\frac{a^2+b^2+c^2}{4m_bm_c}$. Hence $B'C' = \\frac{(a^2+b^2+c^2)a}{4m_bm_c}$. Since triangles $A_1B_1C_1$ and $A'B'C'$ are similar, we obtain $\\frac{r_{A'B'C'}}{r_{A_1B_1C_1}} = \\frac{B'C'}{B_1C_1} = \\frac{3(a^2+b^2+c^2)R}{4m_am_bm_c}$. On the other hand, we know that $r_{A_1B_1C_1} = \\frac{S_{A_1B_1C_1}}{p_{A_1B_1C_1}} = \\frac{S(a^2+b^2+c^2)}{12(\\sum m_a.sinA)R^2}$. Hence $r_{A'B'C'} = \\frac{S(a^2+b^2+c^2)^2}{8m_am_bm_c(\\sum am_a)}$.\r\n\r\nWe need to show that $\\frac{S(a^2+b^2+c^2)^2}{8m_am_bm_c(\\sum am_a)} \\geq \\frac{S}{p}$, which is equivalent to $(a^2+b^2+c^2)^2p \\geq 8(\\sum am_a)m_am_bm_c$. By applying Tchebychev's inequality and AM-GM inequality, we obtain $\\sum am_a \\leq \\frac{2}{3}p(m_a+m_b+m_c)$ and $m_am_bm_c(m_a+m_b+m_c) \\leq \\frac{1}{3}(m_a^2+m_b^2+m_c)^2 = \\frac{3(a^2+b^2+c^2)^2}{16}$. Therefore, the problem is completely solved.", "Solution_10": "[quote=\"treegoner\"]I 'm glad that finally I got some free time this morning to think again about this problem. Here is my solution to the problem $r_{ABC} \\leq r_{A'B'C'}$.\n\nBut first, I need to answer js131090's question. Leibnizt's theorem says that if $(x: y: z)$ is the barycentric coordinate of a point $P$, then for every point $M$ we have the equality $xMA^2 + yMB^2 + zMC^2 = (x+y+z)MP^2 + xPA^2 + yPB^2 + zPC^2$. In particular, if $P$ is the centroid $G$ and $M$ is the circumcenter $O$, Leibnitz's theroem yields that $3R^2 = 3OG^2 + GA^2+GB^2+GC^2$.\n\nLet's get back to the problem. Let $A_1B_1C_1$ be the pedal triangle of $G$ with respect to $ABC$. Then according to sine theorem and Euler's formula for area of pedal triangle, we obtain the sides and area of $A_1B_1C_1$ are respectively $\\frac{2}{3}m_a.sinA, \\frac{2}{3}m_b.sinB, \\frac{2}{3}m_c.sinC, \\frac{4S(a^2+b^2+c^2)}{9R^2}(=\\frac{1}{4}S(1-\\frac{OG^2}{R^2}))$\n\nWe have $\\frac{B'C'}{BC} = \\frac{GC'}{GB} = \\frac{GC.GC'}{GB.GC} = \\frac{a^2+b^2+c^2}{4m_bm_c}$. Hence $B'C' = \\frac{(a^2+b^2+c^2)a}{4m_bm_c}$. Since triangles $A_1B_1C_1$ and $A'B'C'$ are similar, we obtain $\\frac{r_{A'B'C'}}{r_{A_1B_1C_1}} = \\frac{B'C'}{B_1C_1} = \\frac{3(a^2+b^2+c^2)R}{4m_am_bm_c}$. On the other hand, we know that $r_{A_1B_1C_1} = \\frac{S_{A_1B_1C_1}}{p_{A_1B_1C_1}} = \\frac{S(a^2+b^2+c^2)}{12(\\sum m_a.sinA)R^2}$. Hence $r_{A'B'C'} = \\frac{S(a^2+b^2+c^2)^2}{8m_am_bm_c(\\sum am_a)}$.\n\nWe need to show that $\\frac{S(a^2+b^2+c^2)^2}{8m_am_bm_c(\\sum am_a)} \\geq \\frac{S}{p}$, which is equivalent to $(a^2+b^2+c^2)^2p \\geq 8(\\sum am_a)m_am_bm_c$. By applying Tchebychev's inequality and AM-GM inequality, we obtain $\\sum am_a \\leq \\frac{2}{3}p(m_a+m_b+m_c)$ and $m_am_bm_c(m_a+m_b+m_c) \\leq \\frac{1}{3}(m_a^2+m_b^2+m_c)^2 = \\frac{3(a^2+b^2+c^2)^2}{16}$. Therefore, the problem is completely solved.[/quote]\r\n\r\nThe geometric problem is always should be solve in geometric way but i totally stupid in geometry. Here is my approach tatally calculus, which may not please treegoner:\r\n\r\n[b] lemma [b] $(A',B',C')>>(A,B,C) (rad)$.\n[b] proof [/b]. WLOG we can assume that $A\\geq\\ B\\geq\\ C$ which means $GA\\leq\\ GB\\leq\\ GC$. then by the cosin theorem we can proof that $A,B,C\\geq\\ max(A',B',C')$ and $A,B,C\\leq\\ min(A',B',C')$ . Then our lemma is proven.\r\n\r\nThis is the how to apply our lemma:\r\nthe problem <=> $sin\\frac{A}{2}\\ sin\\frac{B}{2}\\ sin\\frac{C}{2}\\leq\\sin\\frac{A'}{2}\\ sin\\frac{B'}{2}\\ sin\\frac{C'}{2}$ <==> $\\sum\\ ln(sin\\frac{A}{2}\\ )\\leq\\sum\\ ln(sin\\frac{A'}{2}\\ )$. (2)\r\nlet consider the fucntion $f(x)=ln(sin(x))$ where $0\\leq\\ x\\leq\\pi\\ /2$. It is easy to see that $f''(x)<0$ Apply the KAMARATA ineq for the function $f(x)$ and 6 numbers $(A',B',C')>>(A,B,C)$ in (2) we have the result.\r\n\r\nhowever thank u treegoner." } { "Tag": [ "function", "topology", "real analysis", "real analysis unsolved" ], "Problem": "Suppose $ f,g : \\mathbb{R} \\to \\mathbb{C}$ are Borel. Prove that the function $ h(x,y)\\equal{}f(x\\minus{}y)g(y)$ is also Borel measurable.", "Solution_1": "Can you prove that $ \\tilde h(x,y)\\equal{}f(x)g(y)$ is Borel? Because $ h$ is $ \\tilde h$ composed with the homeomorphism $ (x,y)\\mapsto (x\\minus{}y,y)$." } { "Tag": [ "logarithms", "algebra proposed", "algebra" ], "Problem": "hallo, \r\nsolve the system:\r\n$2^{x}+3^{y}=5$ and $4^{x}+2^{y}=6$. :)", "Solution_1": "Only x=y=1.\r\nLet $z=2^{x}$. We have $00$ for $00,y>0,z>0$.\r\n\r\nI am not able to generate a proof using vector methods.But first of all I would want to know whether above statement is correct or not.", "Solution_1": "The statement is true. $ P(\\overrightarrow r)$be inside the triangle $ ABC$ except boundary. Use $ z\\equal{}1\\minus{}x\\minus{}y$ to prove." } { "Tag": [ "inequalities", "calculus", "algebra unsolved", "algebra" ], "Problem": "[u][i]Problem[/i][/u]\r\n\r\n [i] Find all real numbers $ x$ such that the following inequality holds for all non negative \n numbers $ a , b , c$\n\n $ [ a^{2} \\plus{} b^{2} \\plus{} (x\\minus{}1)c^{2} ] [ a^{2} \\plus{} c^{2} \\plus{} (x\\minus{}1)b^{2} ] [ b^{2} \\plus{} c^{2} \\plus{} (x\\minus{}1)a^{2} ]$\n\n $ \\leq (a^{2} \\plus{} xbc)(b^{2} \\plus{} xac)( c^{2} \\plus{} xab)$[/i]\r\n\r\n\r\n\r\n [i] My proposed problem[/i]\r\n\r\n[i] I hope it would be nice problem[/i]\r\n\r\n\r\n[i] Scorpius119 , Nayel, ....Let perform [/i]\r\n\r\n\r\n\r\n\r\n [i] Love An forever :love: \n Hu\u1ef3nh V\u00f5 Ph\u01b0\u01a1ng An[/i]", "Solution_1": "If we take $ c\\to 0$ we will get:\r\n$ (a^2 \\plus{} b^2)(a^2 \\plus{} (x \\minus{} 1)b^2)(b^2 \\plus{} (x \\minus{} 1)a^2)\\leq a^3 b^3 x$\r\nNow put here $ a \\equal{} b$ : \r\n$ 2a^6 x^2\\leq a^6 x$ or \r\n$ x\\left(x \\minus{} \\frac {1}{2}\\right)\\leq 0.$\r\nThis means that $ x \\in (0,\\frac {1}{2})$\r\nMaybe this helps... :)", "Solution_2": "for the post above x>0.\r\n$ c \\to 0 \\text{ and } b\\equal{}ka, a,k \\in R^\\plus{}$\r\nso we must x such that for all k we have:\r\n$ f(k)\\equal{}(k^2x^2)\\plus{}x(1\\minus{}k^4\\minus{}k^2\\minus{}k\\minus{}\\frac{k^3}{k^2\\plus{}1})\\plus{}(k^2\\plus{}k\\minus{}1\\minus{}k^3) \\le 0$\r\n$ \\implies f(k)\\equal{}(k^6\\minus{}2k^3\\minus{}k\\plus{}1)x\\plus{}k^2(k^2\\plus{}1)x^2\\minus{}(k^2\\plus{}1)(k\\plus{}1)(k\\minus{}1)^2 \\le 0$ but $ lim_{k \\to \\infty}\\equal{}\\plus{} \\infty$, contradiction.", "Solution_3": "Are you shure?\r\n$ \\frac {1}{k^2}x^2 \\plus{} x\\left(\\frac {1}{k^4} \\minus{} 1 \\minus{} \\frac {1}{k^2} \\minus{} \\frac {1}{k^3} \\minus{} \\frac {k}{k^2 \\plus{} 1}\\right) \\plus{} \\left(\\frac {1}{k^2} \\plus{} \\frac {1}{k^3} \\minus{} \\frac {1}{k^4} \\minus{} \\frac {1}{k}\\right)\\leq 0$\r\nIf we take $ k\\to \\infty$ we will have : $ \\minus{} x\\leq 0$. This is true if $ x\\geq 0$", "Solution_4": "[quote=\"Yuriy Solovyov\"]Are you shure?\n$ \\frac {1}{k^2}x^2 \\plus{} x\\left(\\frac {1}{k^4} \\minus{} 1 \\minus{} \\frac {1}{k^2} \\minus{} \\frac {1}{k^3} \\minus{}\\frac {1}{k(k^2 \\plus{} 1)} \\right) \\plus{} \\left(\\frac {1}{k^2} \\plus{} \\frac {1}{k^3} \\minus{} \\frac {1}{k^4} \\minus{} \\frac {1}{k}\\right)\\leq 0$\nIf we take $ k\\to \\infty$ we will have : $ \\minus{} x\\leq 0$. This is true if $ x\\geq 0$[/quote]\r\n\r\nhowever you're right, i'am wrong in calculos, in fact:\r\n$ p(x) \\equal{} x^2(k^2(k^2 \\plus{} 1)) \\plus{} x((k^2 \\minus{} 1)^2(k^2 \\plus{} 1) \\minus{} k^3) \\minus{} ((k^2 \\minus{} 1)^2(k^2 \\plus{} 1)) \\le 0$\r\nand we note $ x_1 \\le 0 < x_2 \\forall k$ and $ min(x_2) \\equal{} p(1) \\equal{} \\frac {1}{2}$\r\n\r\n(i've calcolated min{f (x) = ((x3-(((x2)-1)2)(x2+1))+sqrt((x3-(((x2)-1)2)(x2+1))2+4(x2)(x2+1)(x2+1)(x2-1)(x2-1)))/(2(x2)(x2+1))} with http://wims.unice.fr/wims/wims.cgi )\r\n\r\nin conclusion $ \\forall x \\in (0,\\frac {1}{2})$\r\n :roll:" } { "Tag": [ "real analysis", "advanced fields", "advanced fields unsolved" ], "Problem": "Prove that if $\\mathbb{R}^{k}\\subset \\bigcup_{1}^{\\infty}F_{n}$ where each $F_{n}$ is a closed subset of $\\mathbb{R}^{k}$, then at least one $F_{n}$ has a nonempty interior. \r\n\r\n(using only ideas developed up to chapter 2 in Rudin)", "Solution_1": "Don't double post, please. This post is nearly identical to [url]http://www.artofproblemsolving.com/Forum/viewtopic.php?t=146107[/url]." } { "Tag": [], "Problem": "The more you have of it, the less you see. What is it?\r\n\r\ni have made a riddles thing. like it feel free to post riddles and ANSWERS!!!", "Solution_1": "it has several answers\r\nCataract,myopia,hypermetropia,................ :rotfl: :rotfl: \r\n\r\nor\r\nKNOWLEDGE :rotfl:", "Solution_2": "hmmmm knowledge is a good answer, but it's actually darkness. good job though.", "Solution_3": "eye diseases r better option", "Solution_4": "riddles concerned with the eye onl;y?", "Solution_5": "next riddle i am posting.\r\nwat's in the middle of nowhere? common riddle.....easy", "Solution_6": "now[b]h[/b]ere\r\n\r\nanswer is [b]h[/b] :rotfl:", "Solution_7": "very smart :rotfl: :rotfl: :rotfl:", "Solution_8": "INFINITY :rotfl: :rotfl:", "Solution_9": "good job ur good\r\nwhat goes up an never comes down? this one shoud have more than one answer but it says it only has one.", "Solution_10": "a satellite launched with a speed greater than escape velocity :rotfl: :rotfl: :rotfl:", "Solution_11": "age :rotfl:", "Solution_12": "now hav a look at this one wat is such that it never degrades with respect to the human body?\r\n(ans=*****)", "Solution_13": "r u mad or wat.... :mad: :o \r\nu kno i don't mind using slang :P", "Solution_14": "hey its not a slang word?", "Solution_15": "AGE! good job!!!!!!", "Solution_16": "and can u prove my ans wrong?? :rotfl: :rotfl:", "Solution_17": "@rt and ritu,i am unable to understand you both", "Solution_18": "even i am not able to understand it................. :rotfl: :rotfl: :rotfl:", "Solution_19": ":rotfl: is telling a different story altogether\r\n\r\ncommon you two can tell it to me :P", "Solution_20": "tell this one\r\n\r\nin a boxing match, non of the men threw a punch ,yet there was a winner.How's so\r\n??????", "Solution_21": "what is the need of [b]throwing[/b] a punch :rotfl: :rotfl:", "Solution_22": "ok\r\nnobody hit a punch.....\r\n\r\nbut throwing a punch means the same neways :lol: :D", "Solution_23": "the other one gave up\r\n :rotfl: :rotfl: :rotfl:", "Solution_24": "without doing nething??????? :blush:", "Solution_25": "see thiss is a variation of a common ques (produced by myself) :P \r\n\r\nGeneral ans is \"it was a women's match so no men threw a punch\" :rotfl: \r\n\r\nBut i've twisted the ques and it says \"None [b]of[/b] the men\" threw a punch that means men were there in the match!!! :P \r\n\r\nNow don't say the referee of a woman's match didn't threw the punch :P" } { "Tag": [ "ARML", "probability" ], "Problem": "Here is the Weather Forecast for the Penn State site.\r\n[img]http://img98.imageshack.us/img98/7616/pennstateqb8.png[/img]\r\nSource: NOAA.gov, accessed Wed May 28, 9:30pm\r\n\r\nAssuming the Day lasts 12 hours and the Night lasts 12 hours, what is the probability it thunders for exactly 36 hours during ARML? (Showers != Thunder)", "Solution_1": "Well first of all, it can be any real number between 0 and 48...I don't know the techincal terms for this but wouldn't the probability be negligble? Also was your intention to put this in the ARML forum, because this isn't the ARML forum...", "Solution_2": "Possibly he was assuming that if it storms one night, it will storm the entire night.", "Solution_3": "Yeah it was, my bad. And yeah tjhance is right.", "Solution_4": "Are we supposed to solve that or is it just to inform us that it's likely going to rain?\r\n\r\nAnyways, answer below\r\n\r\n[hide]The probability for it raining 36 hours in a row is the probability that it rains for exactly three (in a row) of the five periods of day and night.\n\n20%*60%*60% + 60%*60%*50% + 60%*50%*40% = 37.2%[/hide]", "Solution_5": "That doesn't answer the question. The question asks to find the probability that it thunders exactly 36 hours, not necessarily consecutively, during ARML.", "Solution_6": "So you're going to have $ \\binom{5}{2}$ terms to add up. That's just ugly.\r\n\r\nEdit: Errr... never mind.", "Solution_7": "The forecast for Las Vegas: Friday high 91/low 66, mainly sunny. Saturday 94/69, sunshine. Sunday 94/70, abundant sunshine. What is the probability that it will ever rain in Vegas? ;)", "Solution_8": "As far as I can remember, there's been a thunderstorm at Penn State every single Friday night I've gone to ARML. (That's 7 years, I believe.) Not that this has anything to do with the subject at hand ....", "Solution_9": "Not gonna happen at Las Vegas. Forecast now is for sunny both Friday and Saturday, high of 89 on Friday (a little cooler than normal), low of 70 Friday night, high of 95 on Saturday.", "Solution_10": "Prediction for Penn State: It rains all Friday Night and then is relatively dry and sunny Saturday. \r\nThere was absolutely no logical reasoning behind this prediction :P but that's exactly what happened the last many years...", "Solution_11": "Chances have been upgraded to 70% / 70% / 60% :(" } { "Tag": [], "Problem": "Simplify ($ i^2 \\equal{} \\minus{} 1$):\r\n\\[ \\frac {5(3 \\plus{} i \\plus{} 2i^2)^2 \\minus{} (1 \\plus{} 2i \\plus{} 3i^2 \\plus{} 4i^3)^3 \\minus{} (i \\plus{} 2i^2)^4}{1 \\plus{} 3i \\minus{} 3i^2 \\minus{} i^3}\r\n\\]\r\n[hide]This question made me laugh when I saw it; I probably worked on it for half of the given time and still got it wrong. :( \n\nAnd just now, I typed it into my graphing calculator and it took me half a minute to get the right answer.[/hide]", "Solution_1": "\\[\\frac{5(3+i+2i^{2})^{2}-(1+2i+3i^{2}+4i^{3})^{3}-(i+2i^{2})^{4}}{1+3i-3i^{2}-i^{3}}\\]\n\\[\\frac{5(i+1)^2-(-2i-2)^3-(i-2)^4}{4i+4}\\]\n\\[\\frac{5(i+1)^{\\cancel2}}{4\\cancel{(i+1)}}-\\frac{(\\cancelto{-2}{-8})(i+1)^{\\cancelto23}}{4\\cancel{(i+1)}}-\\frac{(i-2)^4}{4+4i}\\]\n\\[\\left(\\frac{5i}{4}+\\frac{5}{4}\\right)-\\left[-2(-1+2i+1)\\right]-\\frac{1+8i+24+32i+16}{4+4i}\\]\n\\[1.25i+1.25-\\left(-4i\\right)-\\frac{40i+41}{4+4i}\\]\n\\[\\&5.25i+1.25-\\left(\\frac{40i+41}{4+4i}\\right)\\left(\\frac{4-4i}{4-4i}\\right)\\]\n\\[5.25i+1.25-\\frac{160i+160+164-164i}{16+16}\\]\n\\[5.25i+1.25-\\frac{-4i+324}{32}\\]\n\\[5.25i+1.25-\\left(\\frac{-i}{8}+\\frac{81}{8}\\right)\\]\n\\[5.25i+1.25+0.125i-10.125\\]\n\\[\\boxed{5.675i-8.875}\\]\n\nI might have made a mistake. If so, can someone point it out for me? Thank you." } { "Tag": [ "real analysis", "function", "real analysis unsolved" ], "Problem": "Let $\\mu$ be a measure that is defined on all Lebesgue measurable subsets of the real numbers. Suppose there is a function $u$ from $[0,\\infty)$ into $[0,\\infty)$ with 0 and 1 as fixed points, and the property that $\\mu((a,b)) = u(b-a)$ for $a < b$ in $\\mathbb{R}$. Is it true that $\\mu$ is the same as $\\lambda_{1}$?", "Solution_1": "it is a consequence of this: http://www.mathlinks.ro/Forum/viewtopic.php?highlight=measure&t=114694", "Solution_2": "by \"measure\" i hope you mean a positive measure, don't you?\r\n\r\nif so, the function $u$ has to be monotone, and it has no atoms (since $u(1)=1<+\\infty$). so it's continuous.\r\nbut $\\mu$ is additive, so $u(c-a) = u(c-b)+u(b-a)$ whenever $0\\le a\\le b\\le c$.\r\nnow define $\\tilde{u}: \\mathbb{R}\\to \\mathbb{R}$ as follows: $\\tilde{u}(x) = u(x)$ for $x\\ge 0$, and $\\tilde{u}(x) =-u(-x)$ for $x<0$.\r\nit's easy to check that this function is still additive (this time on the whole real line, and not just on positive reals), and since it's continuous, and $\\tilde{u}(1)=1$, $\\tilde{u}$ is the identity, hence the measure is lebesgue measure." } { "Tag": [ "function", "integration", "abstract algebra", "calculus", "complex analysis", "complex analysis unsolved" ], "Problem": "How can I show that if $ f \\in H^p$ then the harmonic function\r\n$ u_f(z) \\equal{} \\int_0^{2\\pi} P_z(t)|f(e^{it})|^p \\, \\frac {dt}{2\\pi}$\r\n($ P_z(t)$ is the Poisson kernel)\r\nis the least harmonic function that majorates $ |f|^p$.\r\n\r\nI can show that it does majorate (by writing $ f$ as a Blaschke product times a zero-free $ g \\in H^p$), but I have no idea about the least part.", "Solution_1": "Do you understand how to do it if $ f$ is smooth up to the boundary?", "Solution_2": "In this case I can write $ u_f$ as an integral over a circle of smaller radius (with an appropriate poisson kernel), right? So it would follow from the Poisson representation of a harmonic function that extends smoothly to the boundary of a circle.", "Solution_3": "Yes, that's the idea. Use smaller circles and then go to the limit pointwise.", "Solution_4": "Ok, thanks. :blush: I was convinced this problem was very hard, because it was stated without proof but also not as an exercise. Again the psychological effect of thinking something is easy/hard" } { "Tag": [], "Problem": "i do pop, mild rock, hip hop, maybe some rap if its good", "Solution_1": "Oldies through and through.", "Solution_2": "I voted for rock..\r\n\r\nYou missed tons of genres.. But even if there were 25 options, you would miss sth.. So I guess it`s ok..\r\n\r\nI like almost all genres.. But especially rock..\r\n\r\nMy favorite artists are: Pink Floyd, Radiohead, R.E.M., Air, Pearl Jam, A-Ha, Interpol, The Shins, Husker Du, Pixies, Suede, Vangelis, PJ Harvey, Belle & Sebastian, Bob Marley, Bad Religion, Erykah Badu...\r\n\r\nThese days I like to listen Goo Goo Dolls, Mike Oldfield, Charlatans UK, Grandaddy..", "Solution_3": "I listen to everything except Country.", "Solution_4": "I frequently find that some people miss the distinction between rock and rock'n'roll. The latter, I prefer.", "Solution_5": "I love classical music - especially from the romantic period for those who care - but I'll listen to everything except rap, country, and polka. And I'll even listen to some country.\r\n\r\nBob Dylan is a literary genius, though. I tell you, there's nothing like singing Bob Dylan at the top of your lungs with the car windows down and having people stare at you as if you were a leper...\r\n\r\n\"But I was so much older then; I'm younger than that now\" has to be the most profound line in all of music.", "Solution_6": "Marching band music!", "Solution_7": "im into christian rock and punk rock\r\n\r\nswitchfoot and reliant k are two of my favorite bands", "Solution_8": "[quote=\"devilsbrother190\"]im into christian rock and punk rock\n\nswitchfoot and reliant k are two of my favorite bands[/quote]\r\n\r\nAh, switchfoot. Good stuff.", "Solution_9": "Oldies, i.e. Rock 'n Roll. That means the Beatles, the Stones, the Bee Gees, Van Morrison, Kansas, Bob Seger, Jim Croce, Cat Stevens, Billy Joel, the Animals, BTO, the Beach Boys, Boston, CCR, Elvis, the Righteous Brothers, Queen, the Temptations, the Doobies, and so on.\r\n\r\nBut I like some modern rock (if that's the right genre) musicians - Counting Crows, DMB, Sting, and occasional songs by Matchbox 20, U2, etc.", "Solution_10": "[quote=\"mathfanatic\"]Oldies, i.e. Rock 'n Roll. That means the Beatles, the Stones, the Bee Gees, Van Morrison, Kansas, Bob Seger, Jim Croce, Cat Stevens, Billy Joel, the Animals, BTO, the Beach Boys, Boston, CCR, Elvis, the Righteous Brothers, Queen, the Temptations, the Doobies, and so on.\n[/quote]\r\n\r\nNice to meet a fellow Oldies fan.", "Solution_11": "Everything (esp. oldies) except rap.", "Solution_12": "EMINEM! EMINEM!....\r\n\r\nBesides him... Jimmy Eat World, Sum 41, Coheed and Cambria, Dashboard Confessional, Ben Folds Five, John Mayer, Funeral for a Friend, and some DMB...", "Solution_13": "I only listen to classical, especially Beethoven and Chopin. I can't stand other stuff.\r\nNo angry replies, please.", "Solution_14": "um...shouldn't u add a poll for \"none\"? i don't listen to music.", "Solution_15": "I really enjoy listening to the Beatles. Their songs are really good! My favorite: \"The Long and Winding Road\"\r\n\r\nBut I like all kinds of music, even Country! Actually, now that I think about it, I do not like rap. I believe that rap is distatesful and should not be considered music. Of course, that's my opinion.....", "Solution_16": "so...aren't you going to add a \"none\" poll?", "Solution_17": "[quote=\"Danbert\"]I only listen to classical, especially Beethoven and Chopin. I can't stand other stuff.\nNo angry replies, please.[/quote]\r\n\r\nROBINHOOD3000 ANGRY...only kidding, of course.\r\n\r\nWhat, no Lizst?", "Solution_18": "I like classical music and Weird Al music. I HATE rap and country. And I listen to any else. Yay classical is in second place. :)", "Solution_19": "[quote=\"A+MATH\"]I like classical music and Weird Al music.[/quote]\r\n\r\nThat's an interesting mix.\r\n\r\nAnd Robinhood3000, same to you. Although I think we've already discussed this in the Oldies thread.", "Solution_20": "True enough, but it can't be said enough.\r\n\r\nThat, and I think my Oldies thread has all but vanished.", "Solution_21": "there's still no \"none\" option? grr...", "Solution_22": "I despise rap.\r\nJ-pop, J-rock ballads, instrumental jazz, classical, and opera.\r\nNot a real big fan of American pop music." } { "Tag": [ "geometry", "circumcircle", "incenter", "ratio", "geometric transformation", "reflection", "parallelogram" ], "Problem": "Dear Mathlinkers,\r\nlet ABC be a triangle, 1 the circumcircle of ABC, I the incentre of ABC, M a point on BC, \r\nand 2, 3 the Th\u00e9bault's circles wrt to M.\r\nProve : the radical axis of 2 and 3 passes through the midpoint of IM.\r\nAny synthetic proof for my the two last problems?\r\nSincerely\r\nJean-Louis", "Solution_1": "In $ AM$ cuts the circumcircle $ (O)$ again at $ M'.$ Let $ P$ be tangency point of the A-mixtilinear incircle with $ (O).$ Let $ AI$ cut $ (O)$ again at $ X$ and let $ (X)$ be a circle with center $ X$ and radius $ XB \\equal{} XC \\equal{} XI.$ Let the Thebault circles $ (2), (3)$ touch $ BC$ at $ U, V$ and $ (O)$ at $ E, F.$ Inversion in $ (X)$ takes $ BC$ to $ (O)$ and $ U, V$ to $ E, F,$ hence $ \\overline{XU} \\cdot \\overline{XE} \\equal{} XI^2 \\equal{} \\overline{XV} \\cdot \\overline{XF},$ $ UVFE$ is cyclic and moreover, $ X$ is on the radical axis of $ (2), (3)$ (well known). According to [url]http://www.mathlinks.ro/viewtopic.php?t=233986[/url], $ UVPM'$ is also cyclic. Radical axis $ UV$ of the circles $ \\odot(UVFE), \\odot(UVPM'),$ radical axis $ EF$ of the circles $ \\odot(UVFE), (O)$ and radical axis $ M'P$ of the circles $ \\odot(UVPM'), (O)$ meet at their radical center $ Q \\in BC.$ $ E, F$ are external similarity centers of the circle pairs $ (O), (2)$ and $ (O), (3),$ hence $ Q \\in BC$ is the external similarity center of $ (2), (3).$ By Thebault theorem, $ IQ$ is the common center line of the incircle $ (I)$ and the circles $ (2), (3).$ \r\n\r\nParallel to $ AM$ through $ I$ cuts $ BC$ at $ S.$ According to the proof of [url]http://www.mathlinks.ro/viewtopic.php?t=233986[/url], $ XS \\perp IQ.$ Let perpendicular to $ IQ$ through $ I$ cut $ AM$ at $ K.$ $ \\triangle AIK \\sim \\triangle IXS$ are centrally similar, having parallel sides. Let the incircle touch $ BC$ at $ D,$ let $ A'$ be midpoint of $ \\overline{BC},$ and let parallel $ k \\parallel BC$ through $ K$ and perpendicular $ p \\perp BC$ through $ I$ intersect at $ T.$ $ T, A'$ are corresponding points of the centrally similar $ \\triangle AIK \\sim \\triangle IXS$ with similarity coefficient $ \\frac {\\overline{IT}}{\\overline{XA'}} \\equal{} \\frac {\\overline{AI}}{\\overline{IX}}.$ Let the angle bisector $ AI$ cut the side $ BC$ at $ X'$ and let $ (I_a)$ be the A-excircle of the $ \\triangle ABC$ centered at $ I_a.$ The double ratio $ \\frac {\\overline{AI}}{\\overline{AI_a}} \\equal{} \\minus{} \\frac {\\overline{X'I}}{\\overline{X'I_a}}$ is harmonic on account of $ A, X'$ being the external and internal similarity centers of $ (I), (I_a).$ In addition, $ X$ is the midpoint of $ \\overline{II_a},$ therefore $ \\frac {\\overline{AI}}{\\overline{IX}} \\equal{} \\minus{} \\frac {\\overline{IX'}}{\\overline{XX'}} \\equal{} \\minus{} \\frac {\\overline{ID}}{\\overline{XA'}}.$ As a result, $ \\overline{IT} \\equal{} \\minus{} \\overline{ID},$ so that the parallel line $ k \\equiv KT \\parallel BC$ is tangent to the incircle $ (I)$ at $ T.$ Since $ IK \\perp IQ$ by definition and $ I$ is on a midparallel $ l$ of $ k \\parallel BC,$ the line $ KQ$ is a reflection of both the incircle tangent $ BC$ in $ IQ$ and the incircle tangent $ KT$ in $ IK,$ i.e., a tangent of $ (I).$\r\n\r\nLet the midparallel $ l$ of $ k \\parallel BC$ cut $ AM$ at $ L.$ As $ IS \\parallel KL$ and $ \\overline{IS} \\equal{} \\overline{KL},$ $ KLSI$ is a parallelogram and $ SL \\parallel IK$ and $ SL \\perp IQ.$ As $ XS \\perp IQ$ as well and $ X$ lies on the radical axis of the circles $ (2), (3),$ $ XSL$ is their radical axis. But $ LMSI$ is a parallelogram, its diagonals $ SL, IM$ cutting each other in half.", "Solution_2": "Though it might be the same as Vladimir's, here is mine: [url]http://www.mathlinks.ro/Forum/viewtopic.php?search_id=777557517&t=214426[/url] (read posts #2, #3, #5)." } { "Tag": [ "calculus", "integration", "trigonometry", "function", "calculus computations" ], "Problem": "Supose you have the integral $ \\int_0^\\pi\\sin(\\theta)\\,d\\theta$\r\nEveryone knows that the result of this integral is 2, but.. what if we tried a variable change like:\r\n$ y \\equal{} \\sin(\\theta)$\r\nfor $ \\theta\\equal{}0$, $ y\\equal{}0$\r\nfor $ \\theta\\equal{}\\pi$, $ y\\equal{}0$\r\nso you get an integral going from 0 to 0 which means the result is zero... where's the flaw? :oops: :?: :ninja: :huh:", "Solution_1": "You have to go all the way through with the substitution- what about the $ dy$ and $ d\\theta$? We have $ dy \\equal{} \\cos\\theta\\,d\\theta$, or $ d\\theta \\equal{} \\pm\\frac1{\\sqrt {1 \\minus{} y^2}}\\,dy$. That $ \\pm$ is trouble, since it changes in the middle of the integral. We could make it into $ \\int_0^1 \\frac {y}{\\sqrt {1 \\minus{} y^2}}\\,dy \\plus{} \\int_1^0 \\frac { \\minus{} y}{\\sqrt {1 \\minus{} y^2}}\\,dy \\equal{} 2\\int_0^1 \\frac {y}{\\sqrt {1 \\minus{} y^2}}\\,dy$, and that form is clearly nonzero.\r\n[Edit- corrected transformed integrals]", "Solution_2": "Recall what the substitution rule actually states:\r\nGiven a continuously differentiable function $ g(y)$ and a continuous function $ f(x)$ it is true that\r\n\\[ \\int_a^b{f(g(y))g'(y)dy = \\int_{g(a)}^{g(b)}{f(x)dx}\r\n}\\]\r\nNow in your case $ f(x) = \\sin (x)$, but what is your $ g$? Well, you don't state it, but since you're formally making the substitution $ y = \\sin \\theta$, you're implicitly setting $ g(y) = \\arcsin y$. The problem with this is that $ \\arcsin y = \\pi$ has no solution (recall the range of $ \\arcsin$ is $ \\left[-\\frac{\\pi}{2},\\frac{\\pi}{2}\\right]$), so the substitution simply isn't valid. Though, as jmerry suggested, there are tweaks you can do to make it work. \r\n\r\nThe most important thing to get out of this is that the substitution rule is really a very precisely stated theorem -- and all theorems you encounter in mathematics are valid only if the stated conditions are met. If you ignore the conditions and proceed heuristically, you'll inevitably stumble onto 'contradictions' like this one.", "Solution_3": "We can always force a substitution through with an inverse- if we're substituting something one-to-one. If we're not, the d_ factor doesn't work uniquely, and we have exactly this sort of problem.", "Solution_4": "Great answers everyone! thanks a lot.\r\nThis is why we, physicists, need mathematicians... to kick us back into doing things the proper way." } { "Tag": [], "Problem": "HI, \r\nI am solving \"Problem solving strategies \" by ARthur Engel.I am able to solve all the chapters , save one i.e. Extremal principle.The approaches seem to me very ingenious and hard to guess.Do you know any of the sites or material where this principle is explained in detail ? \r\nthanks \r\nkishore", "Solution_1": "i think i know what you are talking about.\r\nin ACoPS it is referred to as the extreme principle.\r\nbasically what you do is is arrange what you are looking at it some sort of order. then you look at the smallest or largest element. sometimes this is combined w/ proof by contridiction to prove that some statement is true or false." } { "Tag": [ "MATHCOUNTS" ], "Problem": "i predict that alan from clarke will definitely make it to nats. David Fink also has a good chance, if he actually studies. Shen and Pejin also might make it. I predict that there is 1 random guy who nobody knows who gets to nats. for team, clarke 1st, diamond 2nd, acton ~7th :wink:", "Solution_1": "ugh, don't ask about art class today...\r\n\r\nyes, this is actually sorta relevant\r\n\r\nhint: shen (hao shen) is in my art class", "Solution_2": "[quote=\"illusiat\"]i predict that alan from clarke will definitely make it to nats. David Fink also has a good chance, if he actually studies. Shen and Pejin also might make it. I predict that there is 1 random guy who nobody knows who gets to nats. for team, clarke 1st, diamond 2nd, acton ~7th :wink:[/quote]\r\n\r\nNo. No predictions. They'll be wrong. They are useless. Especially a year beforehand. No. Bad.", "Solution_3": "Yeah, don't lower your already low self esteem.", "Solution_4": "He's in 8th grade, he will be in high school next year........I like how I am on the list, it makes me feel special. :lol: Art class? I am missing something?\r\n\r\nAnd you forgot about Jonathan Tidor from Clarke, that was sick this year, and all the incoming maybe beastly 5th graders.", "Solution_5": "Ok now you need to start lowering your ridiculously high self-esteem.", "Solution_6": "Can I predict 5 Clarke kids make State Countdown? Or is that hoping? I always get hoping and predicting mixed up :D", "Solution_7": "Yeah, what? Wasn't state like...9 days ago? wtf? But I'll be a hypocrite.\r\n\r\n2009 Results:\r\n\r\n1. Carl\r\n2. Carl\r\n3. Carl\r\n4. Alan Chou\r\n5. Samlmaml\r\n\r\nteam:\r\n\r\n1. Carl\r\n2. Clarke", "Solution_8": "Thanks cat nose. So I'm improving 2 places since last year? Well actually, since at least 2 of those carls would be DQed, I'd still make nats. But I'd probably have to go to mexico again like last year...(the reason i didn't go last year)", "Solution_9": "[quote]i predict that alan from clarke will definitely make it to nats. David Fink also has a good chance, if he actually studies. Shen and Pejin also might make it. I predict that there is 1 random guy who nobody knows who gets to nats. for team, clarke 1st, diamond 2nd, acton ~7th[/quote]\r\n\r\n\r\nwhy will acton get 7th next year???\r\nYou never know who will appear in RJ Grey next year......\r\n\r\n\r\nand\r\nmaybe I'll stay back in 8th grade ( I don't know why it is possible with the grades I have now... but...)\r\nand I might get first or second in state... yay!!!! :clap: :clap:\r\n\r\nwait.... is mathcounts based on age or grade (I think it's grade)", "Solution_10": "Yea grade.\r\nBut it would be pointless to waste one year of your life just to participate in a Math Competition.", "Solution_11": "[quote=\"shentang\"]He's in 8th grade, he will be in high school next year........I like how I am on the list, it makes me feel special. :lol: Art class? I am missing something?\n\nAnd you forgot about Jonathan Tidor from Clarke, [b]who[/b] was sick this year, and all the incoming maybe beastly 5th graders.[/quote]\r\n\r\noh yeah, I forgot, it was during lunch...\r\n\r\nHao, did you just call Jonathan \"that\"?", "Solution_12": "Since I think that's enough of my team predicting how they'll do next year when they still haveother things to focus on(AIME, being first in the nation in SIGMA, breaking the record for most points in IMLEM, a little thing called MC nats.....) I'm going to lock this one up. Wait until summer to start predicting your nonsense again :lol:" } { "Tag": [], "Problem": "what are the last three elements in the pattern A, 2, B, 0, C, 2, D, 0, E, 3, F, 3, G, 2, H, 4, I, J, 2, K, 4, L, 2, M, 2, N, 2, O, 0, P, 1, Q, 0, R, 2, S, 2, T, 3, U, 2, V, 2, W, 2, X, 4, Y, ?, ?, ?", "Solution_1": "Those are 3,Z,2", "Solution_2": "[quote=\"mr. math\"]what are the last three elements in the pattern A, 2, B, 0, C, 2, D, 0, E, 3, F, 3, G, 2, H, 4, I, J, 2, K, 4, L, 2, M, 2, N, 2, O, 0, P, 1, Q, 0, R, 2, S, 2, T, 3, U, 2, V, 2, W, 2, X, 4, Y, ?, ?, ?[/quote]is that supposed to be\r\n...H, 4, I, [b]4[/b], J, 2, K, 4...?", "Solution_3": "I don't get the pattern >.> ." } { "Tag": [ "quadratics", "algebra", "algebra unsolved" ], "Problem": "Let a,b,c,r,s be real numbers such that ar^2+br+c=0\r\n -as^2+bs+c=0\r\n Prove that there exist u is real numbers such that (u-r)(u-s)<=0 and (a/2)u^2+bu+c=0", "Solution_1": "It is quite simple. :) \r\n\r\n Note that $ar^2+br+c= 0$ and $-as^2+bs+c= 0$ implies that $a(r^2+s^2)= 0$, by substracting the two equations. Then, $a=0$ or $r^2+s^2=0$.\r\n If $a=0$, we have that $br+c=bs+c\\Rightarrow r=s$. Then, we have \r\n \r\n $(u-r)(u-s)\\leq0\\Leftrightarrow(u-r)^2\\leq0\\Leftrightarrow (u-r)^2=0\\Leftrightarrow u=r=s$.\r\nSo, if $a=0$, we have that $bu+c=0\\Rightarrow u=r=s=-c/b$.\r\n\r\nNow, if $r^2+s^2=0$, it happens that $r=s=0$. So, we have that \r\n$(u-r)(u-s)\\leq0\\Leftrightarrow(u-0)^2\\leq0\\Leftrightarrow u^2\\leq0\\Leftrightarrow u=r=s=0$.\r\nThen, if a is a real number different form zero, u=0 satisfies all the conditions of the problem.", "Solution_2": "[quote=\"carlaperez_20\"] Note that $ar^2+br+c= 0$ and $-as^2+bs+c= 0$ implies that $a(r^2+s^2)= 0$, by substracting the two equations.[/quote]\r\n\r\nIf I'm subtracting correctly, we get $a(r^2+s^2)+b(r-s)=0$", "Solution_3": "agree with farenhajt that carlaperez_20's solution is false.Subtract 2 equation we get a(r^2+r^2)+b(r-s)=0 not \r\na(r^2+s^2)=0\r\n carlaperez_20 can you check your solution???\r\nAnyone has the solution?? Please help me.", "Solution_4": "[quote=\"bui_haiha2000\"]...and (a/2)u^2+bu+c=0...[/quote]\r\nCheck it, please. I think the condition is false", "Solution_5": "It's easy to see that $r, s$ are the roots of the quadratic equation: $ax^2+bx+c=0$\r\nBy Vi\u00e8te's formula:\r\n $r+s=\\frac{-b}{a}$\r\n $r.s=\\frac{c}{a}$\r\nWe need to prove that thre exists a real number u satisfying:\r\n $(u-s)(u-r) \\le 0$\r\n $\\Longleftrightarrow u^2-(r+s)u+rs \\le 0$\r\n $\\Longleftrightarrow u^2+\\frac{b}{a}.u+\\frac{c}{a} \\le 0$\r\n $\\Longleftrightarrow au^2+bu+c \\le 0$\r\nThis is true because $\\Delta \\ge 0$ (see the function's graph).\r\nThen consider: \r\n $\\frac{a}{2}u^2+bu+c=0$\r\n $\\Longleftrightarrow \\frac{a}{2}u^2=au^2+bu+c$\r\n */if $a>0$\r\n $\\frac{a}{2}u^2=au^2+bu+c \\le 0$\r\n $\\Longleftrightarrow u^2 \\le 0$ \r\n $\\Longleftrightarrow u=0$\r\n */if $a<0$\r\n similarly like above...:$u=0$\r\n This means $r \\le 0 \\le s$, which is equivalent to $rs=\\frac{c}{a}<0$: unsure (not appear in the problems' condition)\r\n :?: :!: :?:" } { "Tag": [ "calculus", "integration", "trigonometry", "calculus computations" ], "Problem": "Calculate \r\n\\[\\int x\\cos (1+x^2) dx\\]", "Solution_1": "$\\int x\\cos(1+x^2)dx = \\frac{1}{2} \\int \\cos(1+x^2)d(1+x^2) = \\frac{1}{2}\\sin (1+x^2)+c$.", "Solution_2": "That's right! :)", "Solution_3": "[quote=\"kunny\"]Calculate\n\\[\\int x\\cos (1+x^{2}) dx \\]\n[/quote]\r\nSet $u=1+x^{2}\\Longleftrightarrow du=2xdx$, which yields:\r\n\r\n\\begin{eqnarray*}\\int{x\\cos (1+x^{2})dx}&=& \\frac{1}{2}\\int{\\cos udu}\\hfill \\\\ &=& \\frac{1}{2}\\sin (1+x^{2})+c \\hfill \\\\ \\end{eqnarray*}" } { "Tag": [ "inequalities", "inequalities proposed" ], "Problem": "For any three positive reals $a, b, c$, we have\r\n\r\n$\\displaystyle \\frac{a^3+abc}{\\sqrt{b^2+c^2}}+\\frac{b^3+abc}{\\sqrt{c^2+a^2}}+\\frac{c^3+abc}{\\sqrt{a^2+b^2}}\\ge\\frac{\\sqrt{2}}{3}(\\sqrt{ab}+\\sqrt{bc}+\\sqrt{ca})^2$", "Solution_1": "WLOG assume $a \\ge b \\ge c$. By Chebysev's inequality,\r\n\\[\\displaystyle\\sum \\frac{a^3+abc}{\\sqrt{b^2+c^2}} \\ge \\frac{1}{3}(\\displaystyle\\sum a^3+abc)(\\displaystyle\\sum \\frac{1}{\\sqrt{b^2+c^2}}).\\]\r\nBy Schur's inequality,\r\n\\[\\displaystyle\\sum a^3+abc \\ge \\displaystyle\\sum a(b^2+c^2),\\]\r\nso together with Cauchy Schwarz's inequality,\r\n\\[\\frac{1}{3}(\\displaystyle\\sum a^3+abc)(\\displaystyle\\sum \\frac{1}{\\sqrt{b^2+c^2}}) \\ge \\frac{1}{3}(\\displaystyle\\sum a(b^2+c^2))(\\displaystyle\\sum \\frac{1}{\\sqrt{b^2+c^2}}) \\ge \\frac{1}{3}(\\displaystyle\\sum \\sqrt{a\\sqrt{b^2+c^2}})^2.\\]\r\nSince $\\sqrt{\\sqrt{b^2+c^2}} \\ge \\sqrt{\\sqrt{2}}\\frac{\\sqrt{b}+\\sqrt{c}}{2}$, so\r\n\\[\\displaystyle\\sum \\frac{a^3+abc}{\\sqrt{b^2+c^2}} \\ge \\frac{1}{3}(\\displaystyle\\sum \\sqrt{a\\sqrt{b^2+c^2}})^2 \\ge \\frac{1}{3}(\\displaystyle\\sum \\sqrt{a}(\\sqrt{\\sqrt{2}}\\frac{\\sqrt{b}+\\sqrt{c}}{2}))^2 =\\frac{\\sqrt{2}}{3} (\\displaystyle\\sum \\sqrt{ab})^2.\\]", "Solution_2": "Here is my solution:\r\n $\\frac{a^3+abc}{\\sqrt{b^2+c^2}}-\\sqrt{2}bc=\\sqrt{2} \\left(\\sqrt{\\frac{a^2}{b^2+c^2}\\frac{1}{2}}(a^2+bc)-bc\\right)\\geq \r\n\\sqrt{2}\\left( (a^2+bc) \\frac{2}{\\frac{1}{\\frac{a^2}{b^2+c^2}}+2}-bc\\right)=\\sqrt{2}\\left( \\frac{2a^4-bc(b^2+c^2)}{2a^2+b^2+c^2}\\right)\\geq \\sqrt{2}\\frac{1}{2}\\left(2a^2-b^2-c^2\\right)$\r\n\r\n \r\nSo, $\\frac{a^3+abc}{\\sqrt{b^2+c^2}}+\\frac{b^3+abc}{\\sqrt{a^2+c^2}}+\\frac{c^3+abc}{\\sqrt{a^2+b^2}}\\geq \\sqrt{2}\\left(ab+ac+bc\\right)\\geq \\frac{\\sqrt{2}}{3}\\left( \\sqrt{ab}+\\sqrt{ac}+\\sqrt{bc}\\right)^2$ :)" } { "Tag": [ "puzzles" ], "Problem": "I was having trouble sleeping last night and I tossed and turned well into the night. Our local town hall has a clock which strikes on the hour and also strikes just once on the half hour. During one of my more awake moments I heard the clock strike once, but I could not tell what time it was. Half an hour later it struck once again, but I still could not tell what time it was. Finally, half an hour later it struck once again and I knew what the time was. What time was it?", "Solution_1": "was it half past one ???", "Solution_2": "good job!\r\nyou got it right. :)" } { "Tag": [ "inequalities proposed", "inequalities" ], "Problem": "$ a,b,c>0$ Prove that !\r\n$ (a \\plus{} b)(b \\plus{} c)(a \\plus{} c)(a \\plus{} b \\minus{} c)(b \\plus{} c \\minus{} a)(a \\plus{} c \\minus{} b) \\le 8(abc)^2$", "Solution_1": "[quote=\"bigbang195\"]$ a,b,c > 0$ Prove that !\n$ (a \\plus{} b)(b \\plus{} c)(a \\plus{} c)(a \\plus{} b \\minus{} c)(b \\plus{} c \\minus{} a)(a \\plus{} c \\minus{} b) \\le 8(abc)^2$[/quote]\r\n\r\n\r\n$ \\Longleftrightarrow \\sum(a^3(b \\plus{} c)(a \\minus{} b)( a\\minus{}c)) \\plus{} (a \\minus{} b)^2(b \\minus{} c)^2(c \\minus{} a)^2 \\plus{}$ $ 4abc\\sum(a(a \\minus{} b)(a\\minus{}c))\\geq 0$\r\n\r\n\r\n :lol:" } { "Tag": [], "Problem": "Find the sum of even positive divisors of 10,000.", "Solution_1": "$ (2\\plus{}2^{2}\\plus{}2^{3}\\plus{}2^{4})(5^{0}\\plus{}5\\plus{}5^{2}\\plus{}5^{3}\\plus{}5^{4})$\r\n\r\nps: is this a well known formula or something, it looks beautiful", "Solution_2": "how do you get that", "Solution_3": "Well, $ 10^{4}\\equal{} (2^{4})(5^{4})$\r\n\r\nWe can have 0, 1, 2, 3, or 4 factors of 5 in such a divisor, and 1, 2, 3, or 4 factors of 2 (we must have at least 1 because the number should be even).\r\n\r\nSo, the product $ \\left(\\sum_{i \\equal{} 1}^{4}2^{i}\\right)\\left(\\sum_{j \\equal{} 0}^{4}5^{j}\\right) \\equal{} 23430$ gives the sum of all possible numbers $ 2^{i}5^{j},\\; i\\in\\{ 1,2,3,4\\},\\; j\\in\\{0,1,2,3,4\\}$. This is exactly the sum that Mohamed wrote." } { "Tag": [ "linear algebra", "matrix", "geometry", "graph theory", "advanced fields", "advanced fields unsolved" ], "Problem": "Prove that bipartite graphs has no [url=http://mathworld.wolfram.com/IsospectralGraphs.html]isospectral[/url] graphs among connected graphs.", "Solution_1": "Let me add a few details: a graph on $n$ vertices $v_{1},\\dots,v_{n}$ can be represented by an $n\\times n$ matrix, in which the $(i,j)$-entry is the number of edges between $v_{i}$ and $v_{j}$. This [i]adjacency matrix[/i] depends on the enumeration of vertices, but its eigenvalues do not. Since the matrix is symmetric, it has $n$ eigenvalues (counting multiplicities), which form the spectrum of the graph. A natural (and very difficult) question is: which graphs are determined by the spectrum? \r\n\r\nHere we consider only [url=http://mathworld.wolfram.com/SimpleGraph.html]simple[/url] graphs (i.e. no multiple edges and no loops starting and ending at the same vertex). The adjacency matrix of such a graph has only $0$s and $1$s, with $0$s on the diagonal. For example, the complete graph on $3$ vertices (a triangle) has \r\n\\[A=\\begin{pmatrix}0& 1& 1\\\\ 1& 0 & 1 \\\\ 1& 1& 0 \\end{pmatrix}\\]\r\nLet $G$ be a [url=http://mathworld.wolfram.com/BipartiteGraph.html]bipartite graph[/url]. If we enumerate its edges so that $v_{1},\\dots,v_{m}$ are in the first part, and $v_{m+1},\\dots,v_{n}$ are in the second, then the adjacency matrix will look like this:\r\n\\[A_{G}=\\begin{pmatrix}0& C^{T}\\\\ C& 0 \\end{pmatrix}\\]\r\nwhere $C$ is a rectangular $m\\times(n-m)$ matrix. The claim is that there is no connected graph on $n$ vertices with the same spectrum as $G$. \r\n\r\nI don't have a solution, but can offer a simple observation: two $n\\times n$ matrices $A_{1}$ and $A_{2}$ have the same spectrum if and only if $A_{1}^{k}$ and $A_{2}^{k}$ have the same trace for all $k\\ge 1$ ($1\\le k\\le n$ is enough). One can try to interpret the trace of $A_{G}^{k}$ in terms of the geometry of $G$...", "Solution_2": "Thanks for your reply. Let me specialize the question for the moment. Prove that [i]complete[/i] bipartite graphs has no isospectral graphs among connected graphs.\r\n(PS I doubt the general case.)" } { "Tag": [ "function", "algebra", "rational function", "algebra unsolved" ], "Problem": "This is a generalization of \r\nhttp://www.artofproblemsolving.com/Forum/viewtopic.php?t=157768\r\nLet $ a,b,c$ be 3 reals s.t. $ a+b+c<0,a^{2}>bc,b^{2}>ac,c^{2}>ab$.\r\nWe seek 3 reals $ x,y,z$ s.t.: $ \\sqrt{yz}-x=a,\\sqrt{xz}-y=b,\\sqrt{xy}-z=c$.\r\n1) Find explicitly a solution which is a rational function of $ a,b,c$.\r\n2) Do there exist other solutions ?", "Solution_1": "Enough to consider the equation:\r\n$ x^2\\minus{}yz\\equal{}a,y^2\\minus{}xz\\equal{}b,z^2\\minus{}xy\\equal{}c$\r\nWe has :\r\n$ (x^2\\minus{}yz)^2\\minus{}(y^2\\minus{}xz)(z^2\\minus{}xy)\\equal{}a^2\\minus{}bc$\r\nso $ x(x^3\\plus{}y^3\\plus{}z^3\\minus{}3xyz)\\equal{}a^2\\minus{}bc$\r\nCase 1:$ x^3\\plus{}y^3\\plus{}z^3\\minus{}3xyz\\equal{}0$\r\nso $ x\\plus{}y\\plus{}z\\equal{}0$ or $ x\\equal{}y\\equal{}z$\r\n*)$ x\\plus{}y\\plus{}z\\equal{}0$ then\r\n$ a^2\\equal{}bc,c^2\\equal{}ba,a^2\\equal{}bc$\r\nSo we can find out $ (x,y,z)$\r\n*)$ x\\equal{}y\\equal{}z$ same method .\r\nCase 2:$ x^3\\plus{}y^3\\plus{}z^3\\minus{}3xyz$ different from 0.\r\nSo $ \\frac{x}{y}\\equal{}\\frac{a^2\\minus{}bc}{b^2\\minus{}ca}$\r\n $ \\frac{x}{z}\\equal{}\\frac{a^2\\minus{}bc}{c^2\\minus{}ab}$\r\nReplace to the first equation we can find out solution of this equations.", "Solution_2": "Hi TTsphn,\r\n1) you must also prove that, in my notations, necessarily $ x,y,z>0$.\r\nIn particular your case $ x^3\\plus{}y^3\\plus{}z^3\\minus{}3xyz\\equal{}0$ is impossible.\r\n2) In my notations the requested solution is :\r\n$ x\\equal{}\\dfrac{(a^2\\minus{}bc)^2}{\\minus{}d},y\\equal{}\\cdots$ where $ d\\equal{}a^3\\plus{}b^3\\plus{}c^3\\minus{}3abc\\equal{}(a\\plus{}b\\plus{}c)(a^2\\plus{}b^2\\plus{}c^2\\minus{}ab\\minus{}bc\\minus{}ca)$.\r\n3) You have proved also that there don't exist any other real solution.", "Solution_3": "I solve for any $ (a,b,c)\\in R$ not necessary your condition.\r\nSo can $ x^3\\plus{}y^3\\plus{}z^3\\minus{}3xyz\\equal{}0$" } { "Tag": [ "geometry", "perimeter", "inequalities", "trigonometry" ], "Problem": "A convex quadrilateral $ ABCD$ with area $ 2002$ contains a point $ P$ in its interior such that $ PA \\equal{} 24$, $ PB \\equal{} 32$, $ PC \\equal{} 28$, and $ PD \\equal{} 45$. FInd the perimeter of $ ABCD$.\r\n\r\n$ \\textbf{(A)}\\ 4\\sqrt {2002}\\qquad \\textbf{(B)}\\ 2\\sqrt {8465}\\qquad \\textbf{(C)}\\ 2\\left(48 \\plus{} \\sqrt {2002}\\right)$\r\n$ \\textbf{(D)}\\ 2\\sqrt {8633}\\qquad \\textbf{(E)}\\ 4\\left(36 \\plus{} \\sqrt {113}\\right)$", "Solution_1": "you have to get \"lucky\" with this question.\r\n\r\nas in noticing that 24*32+32*28+28*45+45*24=4004", "Solution_2": "[hide]Yea, I did it inelegantly too. I noticed that if I made the diagonals perpendicular, the area would be $ 2002$.\n\nThen, the perimeter is $ 40\\plus{}4\\sqrt{113}\\plus{}53\\plus{}51\\equal{}144\\plus{}4\\sqrt{113}\\equal{}4\\left(36\\plus{}\\sqrt{113}\\right) \\Rightarrow \\boxed{E}$.[/hide]", "Solution_3": "but seriously, honestly, if this is the \"best solution\", then i do not find this to be a very good problem for an AMC 12 #24 because its just pure luck that you come up with that. Also, the solution, once you find it, is pretty trivial, just 1 step using pythagorean theorem.", "Solution_4": "Then again, if you actually took the 2002 AMC 12B, I would assume that you would memorize that $ 2002 \\equal{} 11 \\times 13 \\times 14$, so the solution wouldn't be so surprising (as in luck-wise). \r\n\r\nA complete solution is on the wiki: [[2002 AMC 12B Problems/Problem 24]].", "Solution_5": "should I add how they found the inequality by using the area formula 1/2 absin C to clear things up?", "Solution_6": "could someone elaborate on this? (the inequality found in the wiki solution)", "Solution_7": "[color=darkred][b][u]PP[/u].[/b] A convex quadrilateral $ ABCD$ with area $ 2002$ contains a point $ P$ in its interior such that \n$ PA \\equal{} 24\\ ,\\ PB \\equal{} 32\\ ,\\ PC \\equal{} 28$ and $ PD \\equal{} 45$ . FInd the perimeter of $ ABCD$ .[/color]\n\n[b]Proof[/b] ([u][b]the future[/b][/u]). Denote the area $[ABCD]=S$ . Therefore, $2S=PA\\cdot PB\\cdot\\sin \\widehat {APB}+PB\\cdot PC\\cdot\\sin \\widehat {BPC}+$ \n\n$PC\\cdot PD\\cdot\\sin \\widehat {CPD}+PD\\cdot PA\\cdot\\sin \\widehat {DPA}\\implies$ $2S\\le PA\\cdot PB+PB\\cdot PC+PC\\cdot PD+PD\\cdot PA$ . \n\n[u]Observe that[/u] $2S=PA\\cdot PB+PB\\cdot PC+PC\\cdot PD+PD\\cdot PA$ . Thus, $\\sin \\widehat {APB}=\\sin \\widehat {BPC}=$\n\n$\\sin \\widehat {CPD}=\\sin \\widehat {DPA}=1$ , .e. $AC\\perp BD$ . In conclusion, the perimeter is $40+4\\sqrt {113}+53+51=$ $\\boxed{4\\left(36+\\sqrt {113}\\right)}$ .", "Solution_8": "[b]Solution:[/b] I always struggling on these $PA, PB, PC, PD$ problems but seriously what was this. So it is pretty clear that the diagonals multiply to a number greater than or equal to 4004. [hide=Explanation from AoPS Wiki] (Which is true for any convex quadrilateral: split the quadrilateral along $AC$ and then using the triangle area formula to evaluate $[ACB]$ and $[ACD]$)[/hide] Furthermore, notice that the diagonals have to be less than 52 and 77 respectively. However, 52 * 77 = 4004. This implies that the diagonals are straight lines, perpendicular, and that $PAC$ and $PBD$ are degenerate triangles. Use Pythagorean theorem and we are done. I guess the main point of this problem was to realize that there is no way other than bounding the diagonals and then ultimately solving the question." } { "Tag": [ "combinatorics unsolved", "combinatorics" ], "Problem": "What is the maximum number of non-overlapping $ 2\\times 1$ dominoes that can be placed on a $ 8\\times 9$ checkerboard if six of them are placed as shown? Each domino must be placed horizontally or vertically so as to cover two adjacent squares of the board.", "Solution_1": "We can place at most 34 dominoes.\r\n\r\nConsider two parts of the board: $ A$ consists of the squares above and to the left of the six dominoes and also the lower left corner, $ B$ consists of the squares below and to the right of the six dominoes and also the upper right corner.\r\n\r\nColor the squares in the usual checkerboard style, so that there are 14 black squares and 16 white squares in A, and there are 14 white squares and 16 black squares in B. Hence there are at most 14 dominoes in each part. Therefore there are at most $ 14\\plus{}14\\plus{}6\\equal{}34$. This is clearly achievable.", "Solution_2": "Let's prove that the answer is 34.\r\nWe color the $ 8x9$ checkerboard like a chessboard. Every domino is maked by a black square and a white square.\r\nThe square $ (a,b)$ is the intersection of the row $ a$ and column $ b$.\r\nLet divide the checkerboard into two parts as follow:\r\n\r\n\r\nWe can see that in the red part there are 17 black squares and 13 white squares.\r\nAlso we can see that in the blue part there are 13 black squares and 17 white squares.\r\nThe parts red and blue only have two intersections.\r\nThen when the maximun number of dominoes are placed, we have to place these two dominoes:\r\n1- (7,1)(8,1) and 2-(1,9)(2,9).\r\nAnd the answer is 28+6=34 because in the both parts the diference of the numbers of squares of diferents colors is at least 2.", "Solution_3": "Color the checkerboard in a chessboard fashion, and split it into sections, as shown to below to the left. Notice there are $14$ white squares in the orange section, and that it is not possible to place a domino that has a black square inside the orange region and white square outside the orange region. Since each domino covers a black and white square, there can be at most $14$ dominoes that cover squares in the orange region. Similarly, there are at most $14$ dominoes that cover squares in the blue region. There are $6$ dominos placed in the beginning, so we must have at most $14+14+6=34$ dominoes. A construction with $34$ dominoes is shown below to the left. $\\square$", "Solution_4": "We break the $8\\times 9$ grid into two sections: $A$ - including the rightmost column, and every square vertically below one of the placed dominoes and $B$ - including the leftmost column and every square which is vertically above one of the already placed dominoes. Then, we color the square in the usual chessboard style. Then, section $A$ has 14 black squares - so we can place almost 14 dominoes on it. Section $B$ has 14 white squares, so we can also place almost 14 dominoes on it. Thus, we cannot place more than $14+14+6=34$ dominoes on this board. Now, we shall give a construction which places 34 dominoes on such a $8\\times 9$ board.\n" } { "Tag": [ "function", "induction", "algebra proposed", "algebra" ], "Problem": "Find all functions $ f: \\mathbb{Z} - \\{0\\}\\to\\mathbb{Q}$ which satisfy\n\\[f\\left(\\frac {x + y}3\\right) = \\frac {f(x) + f(y)}2,\\]\nfor all $ x,y\\in\\mathbb{Z} - \\{0\\}$ such that $ 3\\mid x + y$.", "Solution_1": "[quote=\"BaBaK Ghalebi\"]you are right pco,the problem is not correctly stated,the original problem is the following one:\n\n[b]find all functions $ f: \\mathbb{Z} \\minus{} \\{0\\}\\to\\mathbb{Q}$ which fulfill the following condition:\n\n$ f\\left(\\frac {x \\plus{} y}3\\right) \\equal{} \\frac {f(x) \\plus{} f(y)}2$[/b]\n\n\n\nso we must have $ 3\\mid x \\plus{} y$ and also $ x,y\\in\\mathbb{Z} \\minus{} \\{0\\}$ for example $ x \\equal{} 1,y \\equal{} 2$[/quote]\r\n\r\nLet $ P(x,y)$ be the property $ f\\left(\\frac {x \\plus{} y}3\\right) \\equal{} \\frac {f(x) \\plus{} f(y)}2$\r\n\r\n$ P(x,2x)$ implies $ f(2x)\\equal{}f(x)$ $ \\forall x\\neq 0$\r\n$ P(3x,3x)$ implies $ f(2x)\\equal{}f(3x)$ $ \\forall x\\neq 0$ and so $ f(3x)\\equal{}f(2x)\\equal{}f(x)$ $ \\forall x\\neq 0$\r\n\r\nAssume then $ f(nx)\\equal{}f(x)$ $ \\forall x\\neq 0$ for some integer $ n\\neq 0$. Then :\r\n\r\n$ P(3nx,3x)$ implies $ f((n\\plus{}1)x)\\equal{}\\frac{f(3nx)\\plus{}f(3x)}{2}$ and, since we know $ f(3x)\\equal{}f(x)$, $ f((n\\plus{}1)x)\\equal{}\\frac{f(nx)\\plus{}f(x)}{2}$, and, since $ f(nx)\\equal{}f(x)$, $ f((n\\plus{}1)x)\\equal{}f(x)$\r\nSo $ f(nx)\\equal{}f(x)$ $ \\forall x\\neq 0$ and $ \\forall n>0$\r\n\r\nThen $ P(6x,\\minus{}3x)$ gives $ f(x)\\equal{}\\frac{f(6x)\\plus{}f(\\minus{}3x)}{2}$ $ \\equal{}\\frac{f(x)\\plus{}f(\\minus{}x)}{2}$ and so $ f(\\minus{}x)\\equal{}f(x)$ $ \\forall x$\r\n\r\nAnd so $ f(nx)\\equal{}f(x)$ $ \\forall x\\neq 0$, $ \\forall n\\in\\mathbb{Z}^*$\r\n\r\nAnd so the solutions are $ f(x)\\equal{}c$ for any $ c\\in\\mathbb{Q}$ (and it is easy to check that these necessary conditions are sufficient).\r\n\r\nBut I don't understand the interest of $ \\mathbb{Z}^*$ instead of $ \\mathbb{Z}$ or the interest of $ \\mathbb{Q}$ instead of $ \\mathbb{R}$", "Solution_2": "[quote=\"pco\"]But I don't understand the interest $ \\mathbb{Z}^{*}$ of instead of $ \\mathbb{Z}$ or the interest of $ \\mathbb{Q}$ instead of $ \\mathbb{R}$[/quote]\r\n\r\nthis problem was given in an iranian olympiad about 13 years ago,and the official solution uses induction to prove that $ f$ is constant,but as far as I can see,the official solution didn't use tha fact $ f: \\mathbb{Z}^{*}\\to\\mathbb{Q}$ either (instead of $ f: \\mathbb{Z}\\to\\mathbb{R}$)" } { "Tag": [ "algebra", "polynomial", "absolute value", "algebra solved" ], "Problem": "Let P be a polynomial in R[x] satisfied For all x in the interval [0,1] we have |P(x)|<=1.\r\nProve that:|P(-1/n)|<=2 n+1 -1.(n is degree of P)", "Solution_1": "Indeed, an easy, but beautiful problem. I hope Im not wrong with my computations, but here is what I think works: consider the numbers $ x_k=\\frac{k}{n} $ with $ k=0,1,,n $. Write the Lagrange interpolation formula for these points and take for x the value $ x=\\frac{-1}{n} $. Then, use the fact that $ |P(x_k)| $ is at most 1 and the triangle inequality. The absolute value of the terms that appear in the formula is $\\frac{(n+1)!}{(i+1)!(n-i)!} $ and the sum of these things is $ 2^{n+1}-1$. Is it correct?", "Solution_2": "Congratulation,your way is true! :D" } { "Tag": [ "inequalities", "algebra proposed", "algebra" ], "Problem": "Consider the inequality\n\\[(a_1+a_2+\\dots+a_n)^2\\ge 4(a_1a_2+a_2a_3+\\cdots+a_na_1).\\]\na) Find all $n\\ge 3$ such that the inequality is true for positive reals.\nb) Find all $n\\ge 3$ such that the inequality is true for reals.", "Solution_1": "(1)$n\\geq4$,(2)$n=4$.", "Solution_2": "Its surely not that obvious. Does anyone no a full solution?", "Solution_3": "Hint: Use Abel SUmmation Formula for the RHS.", "Solution_4": "\\[(a_1+a_2+a_3+a_4)^2= 4(a_1a_2+a_2a_3+a_3a_4+a_4a_1)+(a_1-a_2+a_3-a_4)^2\\]\\[\\ge 4(a_1a_2+a_2a_3+a_3a_4+a_4a_1).\\]", "Solution_5": "(a) For $n = 3$, if we let $a_1 = a_2 = a_3 = 1$, then we get $9 \\geq 12$, which is clearly false.\n\nWe claim that the inequality is true for $n \\geq 4$. We use induction on $n$.\n\nFor $n = 4$, the inequality can be rearranged to get $(a_1 - a_2 + a_3 - a_4)^2 \\geq 0$, which is true for all reals $a_1, a_2, a_3, a_4$. \nIf the inequality is true for $n$, then we will show it is also true for $n + 1$. Note that since the inequality is cyclic, we can assume that $a_1$ is the largest among all $a_i$'s. Then $(a_1 + \\ldots + a_{n-1} + (a_n + a_{n+1}))^2 \\geq 4(a_1a_2 + \\ldots + a_{n-2}a_{n-1} + a_{n-1}(a_n + a_{n+1}) + (a_n + a_{n+1})a_1) = 4(a_1a_2 + \\ldots + a_na_{n+1} + a_{n+1}a_1) + 4(a_{n-1}a_{n+1} + a_n(a_1 - a_{n+1})) \\geq 4(a_1a_2 + \\ldots + a_na_{n+1} + a_{n+1}a_1),$ which completes the induction.\n\n(b) We have already shown that the inequality is false for $n = 3$ and true for $n = 4$. For $n \\geq 5$, let $a_1 = a_2 = 1, a_3 = 0, a_4 = -2, a_5 = \\ldots = a_n = 0$. Then we get $0 \\geq 4$, which is clearly false. So the inequality is only true for $n = 4$.\n" } { "Tag": [ "geometry", "circumcircle", "geometric transformation", "homothety", "geometry proposed" ], "Problem": "[color=blue]\n$ABC$ is a triangle and $B',C'$ are two points on its circumcircle so that $\\angle{C'AB}=\\angle{B'AC}$.The line trough $C'$ parallel to $AC$ intersects the line trough $B'$ parallel to $AB$ at $A'$.Prove that the points $A,A'$ and $A''$ are collinear,where $A''$ is the point of intersection of the lines $BC'$ and $CB'$. [/color]", "Solution_1": "I think that the point $A'',$ must be defined as the intersection point of the segment lines $BB',$ $CC',$ instead of $BC',$ $CB'.$\r\n\r\nSo, we have that $\\angle C'CB = \\angle C'AB = \\angle B'AC = \\angle B'BC$ $\\Longrightarrow$ $B'C'\\parallel BC$ and now, we see that the sidelines of the triangles $\\bigtriangleup ABC,$ $\\bigtriangleup A'B'C',$ are parallel each other, one per one.\r\n\r\nHence, these triangles are perspective and so, we conclude that the segment lines $AA',$ $BB',$ $CC',$ are concurrent at one point. That is, the point $A''\\equiv BB'\\cap CC',$ lies on the segment line $AA'$ $($ also on the midperpendicular of the side segment $BC$ $)$ and the proof is completed.\r\n\r\nKostas Vittas.", "Solution_2": "Dear Vittas,actually there is a typo but it is not what you mentioned(it will be too easy that way :) ).\r\nThere should be $\\angle{C'BA}=\\angle{B'CA}$,insted of$\\angle{C'AB}=\\angle{B'AC}$ :roll: \r\n\r\nYou can also think of point $B',C'$ as two point so that $B'C'$ is perpendicular to the diametre trough the vertex $A$(probably this is a better way to define them because there again can be a confusion if both $B'$ and $C'$ lie on the midlle arc $BC$.)", "Solution_3": "Thank you dear Tiks for the clarification. Now is not so easy, but not difficult.\r\n\r\nWe denote as $B'',$ $C'',$ the intersection points of the circle $(O),$ from the segment lines $B'A'\\parallel AB,$ $C'A'\\parallel AC,$ respectively. Also we denote as $S,$ the intersection point of the segment lines $BB'',$ $CC''.$\r\n\r\nSo, we have that $\\angle B''CB = \\angle B'CA = \\angle C'BA = \\angle C''BC$ and hence, we have, as before, $B''C''\\parallel BC.$ \r\n\r\nBecause of the triangles $\\bigtriangleup ABC,$ $\\bigtriangleup A'B''C'',$ have their sidelines parallel each other, one per one, we conclude that these are perspective $($ also Homothetic $),$ with perpspector the point $S.$ So, the points $A,$ $A',$ $S,$ are collinear.\r\n\r\nFrom the non-convex inscribed hexagon $BC'C''CB'B'',$ by Pascal\u2019s theorem, we have that the points $A',$ $A'',$ $S,$ are collinear.\r\n\r\nHence the points $A,$ $A',$ $A'',$ are collinear and the proof is completed.\r\n\r\nKostas Vittas.", "Solution_4": "Congratulations with a very nice solution!!!!!!!!!!!!!!!!\r\n\r\nJust one more thing:from Dezarg's theorem we can see that this problem is equivalent to the fact that lines $BC,B'C'$ and $B_{1}C_{1}$ are concurent where $B_{1}\\in{{B'A'}\\cap{AC}}$ and $C_{1}\\in{{C'A'}\\cap{AB}}$.\r\n\r\nNow it is time to see what was mentioned in the description of this topic.\r\n\r\n[url]http://www.mathlinks.ro/Forum/viewtopic.php?t=55443[/url]\r\n\r\nTake the points $B'$ and $C'$ to be the same and you will get the problem from the link above.", "Solution_5": "Let $C''\\in BC'\\cap AC, B''\\in CB'\\cap AB, S\\in BC'\\cap A'B', T\\in CB'\\cap A'C'$.\r\n\r\nA little and easy angle-chasing shows that\r\n$C''AB\\sim C'A'S$\r\n$BAC\\sim SA'T$\r\n$CAB''\\sim TA'B'$\r\n$B''AC''\\sim B'A'C'$.\r\n\r\nThus, the quadrilaterals $C''BCB''$ and $C'STB'$ are also similar, having the corresponding sides parallel. Hence, there exist a homothety taking $C''BCB''$ to $C'STB'$ and thus $A$ to $A'$. But since $A''\\in C''C'\\cap B''B'$, $A''$ must be their center of similtude, hence $A'', A, A'$ are collinear.", "Solution_6": "I think the difficult part of this problem was to come up with the construcation of the additional points $B\"$ and $C\"$ :wink: .\r\n\r\nAnyway,thank you for your solution Yimin! :wink: \r\n\r\nAlso I am wondering,are you going to participate in the IMO 2007?", "Solution_7": "[quote=\"Tiks\"]Also I am wondering,are you going to participate in the IMO 2007?[/quote]\r\n\r\nProvided that I qualify...yes :)\r\nWhat about you?", "Solution_8": "[quote=\"Yimin Ge\"][quote=\"Tiks\"]Also I am wondering,are you going to participate in the IMO 2007?[/quote]\n\nProvided that I qualify...yes :)\nWhat about you?[/quote]\r\n\r\nMe too :wink: ,if I will qualify :) .\r\n\r\nSee you!!!!!!" } { "Tag": [ "geometry", "search", "number theory", "\\/closed" ], "Problem": "Hello,\r\n\r\nMay be what I am going to ask is somewhere and I just did not look appropertly for it, but, Is there a way to post a same topic in more than one forum at once. Let me try to explain myself better trying to make an example: Pretty often you can find problems (or topics in general) that fit in more than one of the subfora (just in case, plural of subforum) on MathLinks, for example, some problems that can be solved either with Geometry, or a totally different solution using only algebra, or with number theory or combinatorics, and some of us may want to place it in all the subfora that apply.\r\n\r\nFor example, this problem \r\n\r\nhttp://www.mathlinks.ro/Forum/viewtopic.php?t=26695\r\n\r\nI would like to place it both in Combinatorics and in Geometry, but if I do it placing it once on each fora, probably it turns messy, because it becomes to different discussions, that could fit nice in just one. \r\n\r\nIs there a way to do this? Was I clear enough with what I mean?\r\n\r\nBest regards,", "Solution_1": "In general, we prefer to discourage such things, because it takes up space in our database. Plus it make our forum too easy to spam. \r\n\r\nPlease don't post the same thing in more than one topic. Try to pick the most appropriate topic, and post it there.", "Solution_2": "As David pointed out we don't want that. Try not to double-post a problem, although this happens a lot (given the numbers of problems here, there are roughly more than 50% chances that a problem has been already posted somewhere) and use the search function. \r\n\r\nAs for problems that fit in two categories, placing them in one category is enough, the subforums for Geometry for example are not restricted to solutions only involving Geometry (etc.)" } { "Tag": [ "linear algebra", "matrix" ], "Problem": "Let \r\n$ A \\equal{} {\\begin{pmatrix} 1 & 1 & \\minus{}1 \\\\\r\n1 & 1 & 1 \\\\\r\n1 & 1 & 1 \\\\\r\n\\end{pmatrix}}$\r\n$ D \\equal{} {\\begin{pmatrix} 0 & 0 & 0 \\\\\r\n0 & 1 & 0 \\\\\r\n0 & 0 & 2 \\\\\r\n\\end{pmatrix}}$\r\n\r\nProve that $ \\exists P$ invertible matrix such that\r\n\r\n$ A \\equal{} P^{ \\minus{} 1}DP$", "Solution_1": "Hint: Why is it enough to show that both matrices have the same eigenvalues assuming that we are working, say, over the reals?", "Solution_2": "Observe the second and third columns of A: the sum of first elements is zero, while the sum of second, and third elements, is 2. Therefore, 2 is an eigenvalue. Also, the first two columns are equal, therefore 0 is another eigenvalue. Finally, the trace equals 3, therefore the other eigenvalue is 1. Since A has all eigenvalues distinct, it is diagonalizable, which means A is similar to the diagonal matrix of its eigenvalues, which is D. $ \\Diamond$" } { "Tag": [ "linear algebra", "matrix", "algebra", "polynomial", "invariant", "absolute value", "linear algebra unsolved" ], "Problem": "Let $A\\in M_n(Z)$ be an invertible matrix such that $|Tr(A^k)|\\leq n$ for all $k$. Prove that there exists a matrix $Q\\in GL_n(Z)$ such that $Q^{-1}AQ$ is an upper triangular matrix by blocks, with some 1 and -1 on the main diagonal and such that all the other blocks on the main diagonal have characteristic polynomials some cyclotomic polynomials.", "Solution_1": "It is sufficient to prove that the eigenvalues $\\alpha_i$ of $A$ are all roots of unity. We shall prove this under the weaker assumption that $\\vert Tr A^n\\vert < P(n)$ where $P$ is a polynomial.\r\n\r\nFirstly we prove that $\\vert\\alpha_i\\vert \\le 1$ for all $\\alpha_i$. We have $Tr A^n = \\sum_i \\alpha_i^n$. Consider the power series $F(z) = \\sum_{n=0}^\\infty (Tr A^n) z^n = \\sum_i \\frac{1}{1 -\\alpha_i z}$. Since the coefficient of $z^n$ in $F$ is bounded by a polynomial in $n$, we have $F$ convergent for any $z$ with $\\vert z \\vert < 1$ and so the radius of convergence of $F$ is 1. Hence $F$ is analytic in the open unit disc. We conclude that all the $\\vert \\alpha_i \\vert \\le 1$, otherwise there would be a pole of $F$ in the open disc.\r\n\r\nNow we show that if $f$ is a monic integer polynomial with roots non-zero algebraic integers all of modulus $\\le 1$ then the roots of $f$ are all roots of unity. This is well-known, but here's a proof I like. Consider a recurrent sequence $(x_n)_{n=1}^\\infty$ with driving (auxiliary) polynomial $f$ and initial terms $0,0,\\ldots,0,1$ (impulse response sequence). The general term is $x_n = \\sum_i A_i \\alpha_i^n$ for certain constants $A_i$. Since all $\\vert \\alpha_i \\vert \\le 1$, $x_n$ is bounded and so can take on only finitely many values. Hence the sequence must repeat from some point on and (since the constant term of $f$ is $\\pm 1$) the sequence is reversible, so the sequence is fully cyclic. Thus the sequence admits $X^M-1$ as a polynomial for for $M$, and so $f$ divides $x^M-1$. Hence the roots of $f$ are all roots of unity.", "Solution_2": "I am sorry, but may I ask you why from the fact that the eigenvalues are roots of unity it follows what I asked? I don't think it is so straightforward... or I am blind.", "Solution_3": "The characteristic polynomial of $A$ is monic with integer entries, so the eigenvalues are algebraic integers. $A$ is invertible, so they are all non-zero and indeed their product is $\\det A = \\pm 1$. My posting aimed to show that they all have absolute value at most 1, and that algebraic integers with this property which are roots of a monic polynomial with constant term $\\pm 1$ (another way of saying that all conjugates are also bounded by 1) must be roots of unity.", "Solution_4": "Oh, sorry, your post again I've just read. The quick answer is rational canonical form for the invariant subspaces.", "Solution_5": "Sorry, but is this theorem true in any ring? Here all matrices are supposed to have integer entries and yet the only version of the theorem you are speaking is working with fields.", "Solution_6": "Oh, I see your point: I'd read $Q$ for $Z$." } { "Tag": [ "group theory", "abstract algebra", "superior algebra", "superior algebra unsolved" ], "Problem": "Let $ G$ be a group, $ \\phi : G \\to G$ a group endomorphism. Is the center of $ G$ necessarily stable under $ \\phi$? Is there a finite counterexample?", "Solution_1": "What means stable here\u00bf\r\nIf $ \\phi$ is an automorphism, then the center is mapped to the center because it is a characteristic subgroup (it can be defined by all-quantified relations on $ G$).\r\n\r\nIf $ \\phi$ is just a general homomorphism, then this is no longer true. For example, take $ G\\equal{} S_3 \\times (\\mathbb Z / 3 \\mathbb Z)$. It's center is $ \\{ 0 \\} \\times (\\mathbb Z / 3 \\mathbb Z)$. Now take $ \\phi(a,b) \\equal{} ( (1,2,3)^b , 0)$. This is a homomorphism and maps the center not to itself." } { "Tag": [ "geometry", "circumcircle", "geometric transformation", "reflection", "symmetry", "homothety", "geometry proposed" ], "Problem": "Dear Mathlinkers,\r\nlet ABC be a triangle, A'B'C' the orthic triangle of ABC, A\"B\"C\" the Euler's triangle of ABC, X the meet point of B\"C' and C\"B', O the center of the circumcircle of ABC and N the center of the Euler's circle of ABC.\r\nProve : X, A and N are collinear. \r\nSincerely\r\nJean-Louis", "Solution_1": "Let $ K$ be the circumcenter of $ \\triangle AB'C'$ lying on $ AH,$ since rays $ AO,AH$ are isogonal WRT $\\angle BAC.$ Let $ K_0$ be the reflection of $ K$ about $ B'C'.$ Note that $ \\angle HC'B'' \\equal{} \\angle HB'C''$ $\\Longrightarrow$ Rays $C'X, C'K_0$ and $ B'X,B'K_0$ form equal and oppositely directed angles with respect to the pairs of rays $ C'B',C'A$ and $B'C',B'A$ $\\Longrightarrow$ $X,K_0$ are isogonal conjugates WRT $ \\triangle AB'C'$ $\\Longrightarrow$ Rays $ AK_0$ and $ AX$ are isogonal WRT $ \\angle BAC.$ Since $ B'C'$ is antiparallel to $ BC,$ the axial symmetry about the internal bisector of $ \\angle BAC$ takes $ B'C'$ into $ B''C''$ parallel to $ BC,$ $ K$ into $ K''$ on the ray $ AO$ and $ K_0$ into $ {K_0}'$ on the ray $AX.$ Then homothety with center $ A,$ that takes $ B''C''$ into $ BC,$ sends $ K''$ into $ O$ and carries ${K_0}'$ into the reflection $O_a$ of $ O$ about the sideline $ BC.$ Since $ AH \\equal{} OO_a$ $\\Longrightarrow$ $ AHO_aO$ is a parallelogram.", "Solution_2": "Dear Luis,\r\nwhy do you haven't thought at the Pascal's theorem?\r\nSincerely\r\nJean-Louis", "Solution_3": "Dear Jayme,\r\nYes Pascal's theorem is very useful if we let $ B_1, C_1$ be the midpoint of $ AC, AB$." } { "Tag": [ "inequalities", "inequalities solved" ], "Problem": "2. Proove that for every x,y,z>0 we have: \r\nsqrt((x+y)/(x+z))+sqrt((x+z)/(x+y)) <=(y+z)/sqrt(yz).\r\n\r\n\r\nthis is funny... just look at M ineq 11 posted by me! :D\r\n\r\ncheers!", "Solution_1": "We see :\r\n sqrt((x+y)/(x+z)) + sqrt((x+z)/(x+y)) \\leq (y+z)/sqrt(yz)\r\n <=> sqrt ((1+y/x)/(1+z/x) + sqrt((1+z/x)/(1+y/x)) \\leq (y/x+z/x)/ sqrt((y/x)*(z/x))\r\n We denote :\r\n a = y/x\r\n b = z/x\r\n The inequality becomes :\r\n sqrt((1+a)/(1+b)) + sqrt((1+b)/(1+a)) \\leq (a+b)/sqrt(ab)\r\n <=> 2ab + a 2 b + ab 2 \\leq a 2 + b 2 + a 3 +b 3 \r\n Which is obvious by Cauchy" } { "Tag": [ "number theory proposed", "number theory" ], "Problem": "Find all natural numbers $ k$ which can be represented as the sum of two relatively prime numbers not equal to 1.", "Solution_1": "$ k \\notin \\{ 1,2,3,4,6 \\}$ Otherwise $ k \\in \\{ 4m,4m\\plus{}1,4m\\plus{}2,4m\\plus{}3 \\}$\r\n\r\n$ 4m\\equal{}(2m\\minus{}1)\\plus{}(2m\\plus{}1)$\r\n\r\n$ 4m\\plus{}1\\equal{}2m\\plus{}(2m\\plus{}1)$\r\n\r\n$ 4m\\plus{}2\\equal{}(2m\\minus{}1)\\plus{}(2m\\plus{}3)$\r\n\r\n$ 4m\\plus{}3\\equal{}(2m\\plus{}1)\\plus{}(2m\\plus{}2)$", "Solution_2": "Or even shorter :P\r\n$ 2k \\equal{} (k\\minus{}1) \\plus{} (k\\plus{}1)$ and $ 2k\\plus{}1 \\equal{} (k) \\plus{} (k\\plus{}1)$", "Solution_3": "[quote=\"Mathias_DK\"]Or even shorter :P\n$ 2k \\equal{} (k \\minus{} 1) \\plus{} (k \\plus{} 1)$[/quote]\r\nk=6 :?:", "Solution_4": "[quote=\"Akashnil\"][quote=\"Mathias_DK\"]Or even shorter :P\n$ 2k \\equal{} (k \\minus{} 1) \\plus{} (k \\plus{} 1)$[/quote]\nk=6 :?:[/quote]\r\n^^ My fault, you have to have three cases.. But it is still shorter :P" } { "Tag": [ "geometry", "inequalities", "analytic geometry", "trigonometry", "angle bisector", "geometry unsolved" ], "Problem": "ABC is a right-angled triangle with the right angle at A. Let D be the foot of the perpendicular from A to BC, and let E and F be the intersections of the bisector of B with AD and AC respectively. Prove that DC>2EF.\r\n\r\nThanks", "Solution_1": "I believe a co-ordinate approach is very effective when solving problems involving right triangles and their altitudes, bisectors,etc\r\n\r\nConsider a co-ordinate system with origin at A. Let $ B\\equiv (u,0), C\\equiv (0,v)$ and $ \\ell (BC)\\equal{}a$\r\n\r\nSo, we have as follows\r\nEquation of internal angle bisector of B:$ vx\\plus{}(u\\plus{}a)y\\minus{}uv\\equal{}0$\r\nEquation of AD:$ ux\\minus{}vy\\equal{}0$\r\n\r\nSo, $ E\\equiv (\\frac {uv^2}{a(a\\plus{}u)}, \\frac {u^2v}{a(a\\plus{}u)})$\r\n\r\n$ F\\equiv (0,\\frac {uv}{u\\plus{}a})$\r\n\r\n$ \\ell (EF)\\equal{}\\frac {uv}{a(a\\plus{}u)} \\sqrt {2a^2\\minus{}2au}$\r\n\r\n$ \\ell (CD)\\equal{}\\frac {v^2}{a}$\r\n\r\nSo,\r\n$ DC>2EF\\Leftrightarrow \\frac {v^2}{a}>2\\frac {uv}{a(a\\plus{}u)} \\sqrt {2a^2\\minus{}2au}$\r\n$ \\Leftrightarrow v^2a^2\\plus{}u^2v^2\\plus{}2v^2au\\plus{}8au^3>8u^2a^2$\r\n\r\nThat inequality should be true (it has to be if the problem is correct).\r\nCan someone prove that inequality?? as i could'nt.", "Solution_2": "It very easy by Coordinate - Decarter:\r\nWe chose coordinate as picture.\r\nLet $ D(0;0), B(\\minus{}1; 0), A(0; a)$. Because $ AB^2\\plus{}AC^2\\equal{}BC^2$ $ \\implies$ $ C(a^2; 0)$.\r\nWe have EA/ED = BA/BD --> coordinate of E.\r\nWe have FA/EC = BA/BC --> coordinate of F.\r\nSo we have EF and DC by a. Very easy --> CD > 2EF. \r\nOK.", "Solution_3": "here's a simple proof .\r\nfirst observe that in the right triangle $ \\cos^{2} \\frac {B}{2} \\equal{} \\frac {(a \\plus{} c)^{2}}{4a^{2}}$\r\nalso $ BE \\equal{} \\frac {2cx \\cos \\frac {B}{2}}{c \\plus{} x}$ and $ BF \\equal{} \\frac {2ac \\cos \\frac {B}{2}}{a \\plus{} c}$\r\nwhere $ AD \\equal{} x$\r\nhence we are supposed to prove\r\n$ a \\minus{} x > 2(\\frac {2cx \\cos \\frac {B}{2}}{c \\plus{} x} \\plus{} \\frac {2ac \\cos \\frac {B}{2}}{a \\plus{} c})$\r\non simplification this gives\r\n$ \\frac {(a \\plus{} c)(c \\plus{} x)}{4c^{2}} > \\cos \\frac {B}{2}$\r\nuse $ \\frac {x}{c} \\equal{} \\cos B$\r\nhence we need to prove $ \\cos \\frac {B}{2} > \\frac {2c}{a \\plus{} c}$\r\nwhich after putting the value of $ \\cos \\frac {B}{2}$ simplifies to proving \r\n$ (a \\plus{} c)^{4} > 16a^{2}c^{2}$ which follows from AM-GM and since $ a \\neq c$ the inequality is strict", "Solution_4": "[quote=\"thanhnam2902\"]It very easy by Coordinate - Decarter:\nWe chose coordinate as picture.\nLet $ D(0;0), B( \\minus{} 1; 0), A(0; a)$. Because $ AB^2 \\plus{} AC^2 \\equal{} BC^2$ $ \\implies$ $ C(a^2; 0)$.\nWe have EA/ED = BA/BD --> coordinate of E.\nWe have FA/EC = BA/BC --> coordinate of F.\nSo we have EF and DC by a. Very easy --> CD > 2EF. \nOK.[/quote]\r\n\r\nCould you explain what you mean by implies coordinate of E, F.\r\n\r\nAlso could you explain what you mean EF and DC by a.\r\n\r\nAnd then explain how that implies CD>2Ef", "Solution_5": "[color=darkred] [size=117][b]Nice problem ![/b][/size] \n\n[/color] [quote=\"mathgeniuse^ln(x)\"][color=darkred]$ ABC$ is a right-angled triangle with the right angle at $ A$. \nLet $ D$ be the foot of the perpendicular from $ A$ to $ BC$ and let $ E$ and \n$ F$ be the intersections of the bisector of $ B$ with $ AD$ and $ AC$ respectively. \nProve that $ DC > 2\\cdot EF$ .[/color] [/quote]\r\n[b][u]Proof.[/u][/b] $ BE^2 \\equal{} BD\\cdot BA \\minus{} ED\\cdot EA$ $ \\implies$ $ \\underline {BE^2 < BD\\cdot BA} \\equal{} \\frac {c^3}{a}$ $ \\implies$ $ BE^2 < \\frac {c^3}{a}$ .\r\n\r\nProve easily that $ EF \\equal{} \\frac {a \\minus{} c}{c}\\cdot BE$ . Thus, $ \\boxed {EF^2 < \\frac {c(a \\minus{} c)^2}{a}}$ . But\r\n\r\n$ \\boxed {\\frac {c(a \\minus{} c)^2}{a} < \\left(\\frac 12\\cdot DC\\right)^2}$ $ \\Longleftrightarrow$ $ \\frac {4c(a \\minus{} c)^2}{a} < \\left(\\frac {b^2}{a}\\right)^2$ $ \\Longleftrightarrow$\r\n\r\n$ 4ac(a \\minus{} c)^2 < (a \\minus{} c)^2(a \\plus{} c)^2$ $ \\Longleftrightarrow$ $ \\underline {4ac < (a \\plus{} c)^2}$ , what is truly. \r\n\r\nTherefore, $ EF^2 < \\frac {c(a \\minus{} c)^2}{a} < \\left(\\frac 12\\cdot DC\\right)^2$ $ \\implies$ $ \\boxed {DC > 2\\cdot EF}$ .", "Solution_6": "A good candidate for PWW.\r\n\r\n To improve visibility draw bisector AX of angle DAC\r\n and midline MN of triangle DAC naturally parallel to DC.\r\n All 3 are concurrent. EF is always perpendicular to AX\r\n while MN is mostly not.\r\n \r\n\r\n T.Y.\r\n M.T.", "Solution_7": "[quote=\"armpist\"]A good candidate for PWW.\n\n To improve visibility draw bisector AX of angle DAC\n and midline MN of triangle DAC naturally parallel to DC.\n All 3 are concurrent. EF is always perpendicular to AX\n while MN is mostly not.\n \n\n T.Y.\n M.T.[/quote]\r\n\r\nWhat is PWW?\r\n\r\nIs it like some sort of geometry method.", "Solution_8": "proofs without words..." } { "Tag": [ "function", "search" ], "Problem": "I learned of a new nim-like game in economics, though I don't know what it's called.\r\no\r\noo\r\nooo\r\noooo\r\nooooo\r\n\r\nYou have stacks of pins (stacks of 1, 2, 3, 4, 5 pins in my class) and you can take away 1 or 2 pins at a time. The object of the game is to force your opponent into taking the last pin. The catch: if you take 2 pins, they must be adjacent and in the same stack. Of course, the pins to not collapse together.\r\n\r\nI represent any position as a set of positive integers. For instance, My class's position would be $\\{1,2,3,4,5\\}$. Now, overload the '+' operator to combine two sets (scalar multiplication follows from addition), let $[N]=1$ if you can force a win by moving next, and $[N]=0$ if you cannot force a win. \r\n\r\nHere are some rules I have gathered (maybe you want to verify them):\r\n1) $[N+\\{1,1\\}]=[N]$\r\n2) If $A$ is a position in which you can force a lose, and $[B]=0$, then $[A+B]=1$\r\n3) $\\{3\\}\\equiv\\{1,2\\}, \\{3,3\\}\\equiv\\{2,2\\}$\r\nwhere $A\\equiv B$ means $A$ and $B$ function the same and can replace eachother. I couldn't find any more useful equivalences.\r\n\r\nIn the $\\{1,2,3,4,5\\}\\equiv\\{1,2,1,2,4,5\\}\\equiv\\{2,2,4,5\\}$ position, I believe it is best to take one off the end from the stack with 5 pins. Some useful reduced losing positions to corner your opponent into are: $\\{4\\},\\{1,5\\},\\{2,2\\},\\{4,5\\},\\{1,4,4\\},\\{2,2,5\\},\\{1,2,2,4\\},\\{2,2,2,2\\},\\{2,2,4,4\\}$, and $\\{2,2,2,5\\}$\r\n\r\nBTW, does anyone know what a \"break-and-take game\" is?", "Solution_1": "[quote=\"juggle7\"]I learned of a new nim-like game in economics, though I don't know what it's called.[/quote]\n\nIt's probably called something like Misere k-Token Nim. The reason why I say maybe is because there are so many variations of nim out there, I can't imagine people agreeing on the same name for many variations. Misere refers to the style of play where the player that takes the last token/pin loses. k-Token Nim is the variation where each player may remove up to k-tokens each turn. In this case, k=2.\n\n\n[quote=\"juggle7\"]I represent any position as a set of positive integers. For instance, My class's position would be $\\{1,2,3,4,5\\}$. Now, overload the '+' operator to combine two sets (scalar multiplication follows from addition), let $[N]=1$ if you can force a win by moving next, and $[N]=0$ if you cannot force a win.[/quote]\n\nIt's great that you're coming up with your own notation; it shows that you've really been thinking about the games. In game theory, your $[N]=1$ is an N-position, which means that the next person that moves can win. $[N]=0$ is a P-position, which stands for previous player wins. Often, in game theory, we search for P-positions.\n\n\n[quote=\"juggle7\"]Here are some rules I have gathered (maybe you want to verify them):\n1) $[N+\\{1,1\\}]=[N]$\n2) If $A$ is a position in which you can force a lose, and $[B]=0$, then $[A+B]=1$\n3) $\\{3\\}\\equiv\\{1,2\\}, \\{3,3\\}\\equiv\\{2,2\\}$\nwhere $A\\equiv B$ means $A$ and $B$ function the same and can replace eachother. I couldn't find any more useful equivalences. [/quote]\n\nHere, you are referring to Bogus-Nim Heaps, and this is a big idea in game theory. You might want to look it up. It looks like you're on the right track so far!!\n\n\n[quote=\"juggle7\"]In the $\\{1,2,3,4,5\\}\\equiv\\{1,2,1,2,4,5\\}\\equiv\\{2,2,4,5\\}$ position, I believe it is best to take one off the end from the stack with 5 pins. Some useful reduced losing positions to corner your opponent into are: $\\{4\\},\\{1,5\\},\\{2,2\\},\\{4,5\\},\\{1,4,4\\},\\{2,2,5\\},\\{1,2,2,4\\},\\{2,2,2,2\\},\\{2,2,4,4\\}$, and $\\{2,2,2,5\\}$[\\quote]\n\nNice start. Actually, if you investigate it a bit more, nim has a VERY interesting solution for finding all P-positions. \n\n[hide] If you're still interested in exploring the game, try expressing the piles in base-2.[/hide]\n\n\n[quote=\"juggle7\"]BTW, does anyone know what a \"break-and-take game\" is?[/quote][/quote]\n\nYes, a take-and-break game is a game in which a player can remove tokens and/or (depending on the variation) \"break\" or divide the heaps into several smaller heaps. [/hide]" } { "Tag": [ "invariant", "search" ], "Problem": "Suppose that an ordinary chessboard is tiled with dominos, each of which covers two adjacent squares of the chessboard. Show that the number of horizontal dominos is even.", "Solution_1": "Generalizing to a $ 2k \\times 2k$ board. Let $ D$ be the number of horizontal dominoes in a tiling, and let $ a_1, a_2, \\ldots a_{2k}$ be such that $ a_i$ is the number of squares in column $ i$ covered by a horizontal domino.\r\n\r\nObserve the following facts:\r\n(1) $ a_1 \\plus{} a_2 \\plus{} a_3 \\plus{} \\ldots \\plus{} a_{2k} \\equal{} 2D$.\r\n(2) It is necessary (though not sufficient) for a full tiling to have $ 2k \\minus{} a_i$ even for all $ 1 \\leq i \\leq 2k$, since the remaining squares must be covered by vertical dominos. Equivalently, we require that all $ a_i$ are even.\r\n(3) Placing a horizontal domino between columns $ i$ and $ i\\plus{}1$ adds $ 1$ to both $ a_i$ and $ a_{i\\plus{}1}$. From this we can find the invariant $ a_1 \\plus{} a_3 \\plus{} \\ldots \\plus{} a_{2k\\minus{}1} \\equal{} a_2 \\plus{} a_4 \\plus{} \\ldots \\plus{} a_{2k}$. Using (1), we also determine that both expressions are equal to $ D$.\r\n\r\nSuppose $ D$ is odd. Then by (3), $ a_1 \\plus{} a_3 \\plus{} \\ldots \\plus{} a_{2k\\minus{}1} \\equal{} a_2 \\plus{} a_4 \\plus{} \\ldots \\plus{} a_{2k} \\equal{} D$. For those sums to be odd, the sequences $ a_1, a_3, \\ldots a_{2k\\minus{}1}$ and $ a_2, a_4, \\ldots a_{2k}$ each must contain at least one odd term. This contradicts (2). Therefore $ D$ is even.", "Solution_2": "Here's a wonderful link: http://www.cs.utexas.edu/users/EWD/indexEWDnums.html\r\n\r\nI discovered it by \"accident\" some time ago. Everybody should check it. Great stuff in there.\r\n\r\nThis particular problem is 1306a.", "Solution_3": "So, their problem is more general. For those who don't want to search the link, their problem is the number of horizontal dominoes with a white square on the left equals the number of horizontal dominoes with a white square on the right.\r\n\r\nThe approach of my solution completely fails to solve this. Time to rethink this.", "Solution_4": "A nice solution to the original problem is as follows:\r\n\r\nColor the columns of the chessboard alternately white and black. Then, a horizontal domino covers one of each color and a vertical domino covers two of one color. Finish with a simple parity argument.\r\n\r\nThe more general problem eludes this technique as well. (at least as far as I see)", "Solution_5": "[hide=\"the 1306a problem\"]\nConsider any vertical line between two columns and the set $ H$ of all horizontal dominoes crossing the line. Besides these dominoes, each domino is on one side of the line or the other. Say on the left side, there are $ a$ black squares and $ a$ white squares. Then if $ b$ is the number of dominoes in $ H$ covering a black square on the left side and $ w$ is the number of dominoes in $ H$ covering a white square on the left side, the number of uncovered white squares in the left section in $ a\\minus{}w$ and the number of uncovered black squares is $ a\\minus{}b$. Then $ a\\minus{}w\\equal{}a\\minus{}b\\implies b\\equal{}w$. We can apply this to all particular vertical lines, so the number of dominoes touching a black square on the left side is the same as the number of dominoes touching a white square on the left side.\n[/hide]", "Solution_6": "[b]Further Generalization:[/b] If you tile an $ nk\\times nk$ square using $ 1\\times n$ tiles, then the number of horizontal tiles is divisible by $ n$." } { "Tag": [ "probability", "number theory proposed", "number theory" ], "Problem": "There exists a partition of the set of positive integers in two classes such that from any three consecutive positive integers at least one is in each class and no class contains an infinite arithmetic progression.", "Solution_1": "Here's a nonconstructive argument. Color each odd integer red or blue randomly. Color each even integer differently from its predecessor. So we have randomly partitioned the integers into two color classes that definitely meet the \"three consecutive\" condition.\r\n\r\nBut there are only countable many arithmetic progressions, and each has probability 0 of being colored one color. So our partition has the required properties with probability 1. In particular, there is a good partition.\r\n\r\nIf you want a constructive argument, I guess we can use a diagonalization argument. Enumerate each of the arithmetic progressions, and kill each one as we go, maintaining the odd-even pattern.", "Solution_2": "Very nice construction RaviB. In fact the author indicated a concrete such partition. Put in the first class only those numbers $n$ for which $ n+[\\sqrt{n}]$ is even. But these solutions are much more natural and beautiful.", "Solution_3": "Thanks. I like that concrete partition too." } { "Tag": [ "inequalities", "geometry proposed", "geometry" ], "Problem": "Let $ P$ be an interior point of triangle $ ABC$, and let $ x,y,z$ denote the distance from $ P$ to $ BC,AC,$ and $ AB$ respectively. Where should $ P$ be located to maximize the product $ xyz$?", "Solution_1": "It is not so complicated... :blush: \r\n\r\nPut $ a\\equal{}BC,b\\equal{}CA,c\\equal{}AB$ then we have: $ ax\\plus{}by\\plus{}cz\\equal{}2S_{ABC}$.\r\n\r\nBy AM-GM inequality: $ 2S_{ABC}\\equal{}ax\\plus{}by\\plus{}ca\\geq 3\\sqrt[3]{abcxyz}\\Rightarrow xyz\\leq\\frac{8S_{ABC}^3}{27abc}$\r\n\r\nEquality holds if and only if $ ax\\equal{}by\\equal{}cz\\Leftrightarrow S_{PAB}\\equal{}S_{PBC}\\equal{}S_{PCA}\\Leftrightarrow P$ is the centroid of $ \\triangle ABC$" } { "Tag": [], "Problem": "When 15 is appended to a list of integers, the mean is increased by 2. When 1 is appended to the enlarged list, the mean of the enlarged list is decreased by 1. How many integers were in the original list?", "Solution_1": "[hide]let\nA: average \nX: number of integers in the original list\n\n(AX+15)/(X+1) = A+2\n(AX+15+1)/(X+1+1) = A+2-1\n\nCrossmultiply,\n\nAX+A+2X+2=AX+15\nAX+2A+X+2=AX+16\n\nSubtract second from first*2,\n3X+2=14\n\nX=4[/hide]", "Solution_2": "Yup. Welcome to the NYC forum!" } { "Tag": [ "calculus", "integration", "limit", "function", "real analysis", "calculus computations" ], "Problem": "Let $ f$ be defined on $ [1, \\infty]$ and suppose $ I_{n}= \\int_{1}^{n}f$ exists for all $ n \\in \\mathbb N$. Decide whether the statement is true or false with either a (short) proof or counterexample:\r\n\r\n1. If $ f$ monotonic decreasing and if $ \\lim_{n \\rightarrow \\infty}I_{n}$ exists, then $ \\int_{1}^{\\infty}f$ converges.\r\n\r\n2. If $ \\lim_{x \\rightarrow \\infty}f(x) = 0$ and $ \\lim_{n}I_{n}= I$, then $ \\int_{1}^{\\infty}f$ converges to $ I$.\r\n\r\n3. If $ I_{n}$ converges, then so does $ \\int_{1}^{\\infty}f$.\r\n\r\n4. Suppose that $ f'$ exists and is bounded on $ [1, \\infty]$, and that $ \\lim_{n}I_{n}= I$. Then $ \\int_{1}^{\\infty}f$ converges to $ I$.\r\n\r\n5. If $ \\int_{1}^{\\infty}f$ converges, then $ \\lim_{x \\rightarrow \\infty}f(x) = 0$.", "Solution_1": "#3 is false. Consider the following counterexample:\r\n\r\nLet $ f(x)=\\sin(2\\pi x).$ Then $ I_{n}=\\int_{1}^{n}\\sin(2\\pi x)\\,dx=0$ for all $ n\\in \\mathbb{N}.$ \r\n\r\nHowever, $ \\int_{0}^{\\infty}\\sin(2\\pi x)\\,dx$ does not converge.\r\n\r\nThis shows what the issue is for #1, #2, or #4, but the extra conditions in those problems rule out this exact example. I'll let others decide whether they are true.\r\n\r\n[hide=\"Hint.\"]Of the five statements, at least two are false.[/hide]", "Solution_2": "[quote=\"arp\"]\n5. If $ \\int_{1}^{\\infty}f$ converges, then $ \\lim_{x \\rightarrow \\infty}f(x) = 0$.[/quote]\r\nThis [b]can[/b] be made true with enough initial conditions on the hypothesis. \r\nBut consider a silly counterexample,\r\n\r\n$ f(x) = \\{ \\begin{array}{c}1 , \\ x\\in \\mathbb{N}\\\\ 0 , \\ x\\not \\in \\mathbb{N}\\end{array}$\r\n\r\nThis integral is clearly zero because one isolated point doest change the integral. But $ \\lim_{x\\to \\infty}f(x)$ doesnt even exist.\r\n\r\n--------\r\nFor #1 consider $ f(x) = 1$.", "Solution_3": "sylow_theory's example is not continuous. Very well, then, let us require continuity.\r\n\r\nDefine $ T(x)=\\max(1-|x|,0).$ This is the piecewise-linear continuous function with vertices at $ (-1,0),(0,1),$ and $ (1,0),$ taking on the value zero outside of $ [-1,1].$ (\"T\" is for \"tent\".)\r\n\r\nLet $ f(x)=\\sum_{n=1}^{\\infty}nT(n^{3}(x-n))$\r\n\r\nOnce $ n$ is large enough for the tents not to overlap, this a piecewise linear continuous function with vertices at $ (n-\\frac1{n^{3}},0),\\,(n,n)$ and $ (n+\\frac1{n^{3}},0).$\r\n\r\n$ \\int_{0}^{\\infty}f(x)\\,dx=\\sum_{n=1}^{\\infty}\\frac1{n^{2}}$ converges.\r\n\r\nBut not only does the function fail to tend to zero, it is unbounded as $ x\\to\\infty.$\r\n\r\nThe fact that this example is not smooth is unimportant; by replacing $ T$ by a $ C^{\\infty}$ \"bump\" function and repeating the idea, we could create a $ C^{\\infty}$ example with the same flaws.\r\n\r\n---\r\n\r\n[quote=\"sylow_theory\"]For #1 consider $ f(x) = 1.$[/quote]\r\nBut then the sequence $ I_{n}$ would diverge, so that's not a counterexample - at least if we read the problem as requiring $ \\lim I_{n}$ to exists as a finite number.", "Solution_4": "It seems like interest has been lost in this question, so I'll try to answer two more and see if someone wants to do the last.\r\n\r\nFor 1, sylow, kent is right that we are to assume $ \\lim I_{n}= I$ exists as a finite number. In fact, this makes the statement true. First, note that $ f \\geq 0$ everywhere because of monotocity, for if $ f(x_{0}) = l < 0$ for some $ x_{0}$, then $ f(x) \\leq l$ for all $ x \\geq x_{0}$ and so $ \\lim I_{n}$ could not exist. There is some $ N$ s.t. for $ n \\geq N$,\r\n $ |I_{n}-I| < \\epsilon$ for every $ \\epsilon > 0$. Let $ I(b) = \\int_{1}^{b}f$, $ b \\in \\mathbb R$. For any $ b \\geq N$, there is an $ n \\geq N$ such that $ n \\leq b < n+1$ ($ n = [b]$). Then $ I_{n}\\leq I(b) \\leq I_{n+1}$ since $ f$ is nonnegative everywhere, so we get $ I-\\epsilon < I_{n}\\leq I(b) \\leq I_{n+1}< I+\\epsilon$, which means $ \\int_{1}^{\\infty}f$ converges to $ I$.\r\n\r\nI'll cheat off Kent to answer 4: False. If $ f(x) = \\sin(2\\pi x)$, then $ f'$ is bounded and and $ \\lim I_{n}= 0$, but $ \\int_{1}^{\\infty}f$ doesn't even exist." } { "Tag": [ "combinatorics solved", "combinatorics", "graph theory" ], "Problem": "Here's a question I found in a paper download from this site a long time ago (I don't know if you can find the paper here now):\r\n\r\nProve that in a group of 2n people there are 2 people which have an even number of common acquaintances, where, as usual, if A knows B then B knows A.\r\n\r\nIt looks nice and easy, but I've been thinking abt it for quite some time and I don't even know if my soln is correct. Anyway, it inspired me to think of this:\r\n\r\n1. Is it true that in every graph there is a vertex for which the number of chains of length 2 starting from that vertex is even? I have to mention that I don't know the answer for this one but it seems to be 'yes'. Could ne1 help me with it? It's closely related to this:\r\n\r\n2. Is it true that in every graph there is a vertex linked to an even number of vertices of even degree? (the answer for 1 is yes iff the answer for 2 is yes, if you think abt it for a while). Any ideas?", "Solution_1": "The initial problem appeared in the Tounament of towns in 1995 (with n = 25, but it doesn't matter).\r\nSince the solution is a little boring to write here, you may find it (in french) at : \r\nhttp://boumbo.salle-s.org/xavier/maths/stmalo/ \r\n\r\nChoose the paper on graph theory. This is exercise #4.\r\n\r\n[Moderator edit: Or see http://www.mathlinks.ro/Forum/viewtopic.php?t=68109 ]\r\n\r\nPierre.", "Solution_2": "Thanx a lot, but, like I said, I think I have a soln for that one (it's also a bit boring so i didn't exactly check everything out but...). I think the other problems I wrote there are quite interesting so maybe some1 here has a cute or interesting or useful idea abt them?", "Solution_3": "Hmmm...did I miss something or the answer for your pb 1 is trivially 'no' as shown by the following graph :\r\nA-B-C-D\r\n\r\nPierre.", "Solution_4": "Thanks a lot! It's like taking a load off my chest. Even if the answer is no, at least I know that! Thx again!" } { "Tag": [ "induction", "number theory solved", "number theory" ], "Problem": "Prove that s_n = \\sum^{2n-1}_{k=1} 2^(k-1) * Cn(4n-2,2k) is a perfect square for all n = 1,2,3,...", "Solution_1": "the sum clearly equals 1/4((1+ \\sqrt 2)^(4n-2) + (1- \\sqrt 2)^(4n-2) -2)\r\ndefine an=2an-1 +an-2 with a1=6 and a2=198\r\nit remains to show that a4n-2 -2 is perfect square. \r\nwe show by induction that a4n-2 = a2n-1^2 +2 ,a4n=a2n^2 -2\r\na4n-1=1/2(a2n^2-a2n-1^2)-2 and that a4n+1=1/2(3a*2n^2+a2n-1^2)+6\r\nproof: lets assume that the hypothesises are true for all n<=4k.\r\na4k+1= 2a4k+a4k-1 = 2a2k^2-4 + 1/2(a2k^2-a2k-1^2)-2 \r\n=1/2(3a2k^2+a2k-1^2)+6\r\nsimilarly you can do it for a4k+2, a4k+3, a4k+4.", "Solution_2": "a type mistake .\r\nit should be a1=2, a2=6\r\nthe rest is the same." } { "Tag": [], "Problem": "I'm just wondering.", "Solution_1": "I like the ones that aren't to fake like things that you know will never happen. The type of movie that makes you afraid of the dark for like 2 weeks lol...", "Solution_2": "i don't like horror movies...", "Solution_3": "[quote=\"1=2\"]Re: Thriller/Horror movies\nWhat do you think is the best thriller/horror movie composed of?\nI'm just wondering.[/quote]\r\nWell, Have you watched any horror/thriller movies lately? (I mean, do you watch horror or thriller movies?) If you have watched some good horror movies, you wouldn't have asked a question like this.\r\n\r\n(Watch some horror movies, say like:The Exorcist, Evil Dead, The Ring etc; They're good horror movies)", "Solution_4": "By far the best horror movie is The exorcist 1. I mean , I couldn't sleep well for three entire weeks :P \r\n\r\nIn second place I would place H2O , but just the first one, because the following H2O suck :D \r\n\r\nIn third place I would place Scream , The ring , The profecy 1 and Poltergeist 1" } { "Tag": [ "geometry", "incenter", "circumcircle", "power of a point", "radical axis" ], "Problem": "[color=cyan]So, here they be. The rules are simple: you can draw a circle centered at a given point through a given point. You can draw a circle centered at a given point with a radius a given length. And you can draw a straight line through two given points. That's it. You can use previous problems to do later problems.\n\n1. Given 2 points, construct the midpoint of the segment joining them.\n\n2. Given a point and a line, construct the unique perpendicular to the line passing through the point.\n\n3. Construct the incenter of a given triangle.\n\n4. Construct the circumcenter of a given triangle.\n\n5. Construct the tangent to a given circle from a point not inside that circle. (Note that this has two cases.)\n\n6. Given a unit length, construct a length of :sqrt:n where n is any positive integer.\n\n7. Construct a regular hexagon of given side length.\n\n8. Construct a regular pentagon of given side length at least 2 different ways.\n\n9. Construct the radical axis of two circles. (The radical axis is the set of points whose powers with respect to the two circles are equal. That is, given 2 circles O1 and O2 with radii r1 and r2 respectively, the radical axis consists of all points P such that PO1:^2: - r1:^2: = PO2:^2: - r2:^2:.)\n\n10. Divide a segment into n equal pieces for any positive integer n.\n\n11. Construct the common tangents of 2 circles in all 3 cases for which tangents exist (0, 1, and 2 intersection points).\n\nLook in the Advanced Forum for three more difficult problems on construction, plus whatever anyone else has added.[/color]", "Solution_1": "Another one I like is: given a line segment, divide it into n equal parts for any positive integer n.", "Solution_2": "[color=cyan]Oops, as I was writing this I was going to put that one in near the bottom but then I forgot. So why not make that number 10?[/color]", "Solution_3": "Hm..that's some pretty interesting constructions. I'm taking drafting this year, and it'll probably pretty useful for it!\r\n\r\n-interesting_move" } { "Tag": [ "trigonometry", "geometry unsolved", "geometry" ], "Problem": "Given the quadrilateral $ABCD$ with $\\angle DAB=150^{o}$, $\\angle DAC+\\angle ABD=120^{o}$ and $\\angle DBC-\\angle ABD=60^{o}$. Evaluate $\\angle BDC$.", "Solution_1": "Answer 45 :D\r\nPutting $\\angle{BAC}= t$ then $\\angle{CAD}=150-t$, $\\angle{ABD}=t-30$, $\\angle{DAC}=t+60$, \\angle{ADB}=60-x, \\angle{ACB}=150-3t.\r\n\r\nIf $\\angle{BDC}=x$ then \r\n$\\frac{BC}{CD}=\\frac{BC}{AC}. \\frac{AC}{CD}$ and after few sine laws we get\r\n(1): $\\frac{sin x}{sin(60-t+x)}=\\frac{sin t. sin(t+60)}{sin (2t+30). sin (t+30)}$\r\n\r\nWe can check that if $x=45$ then (1) is true for every $t$. \r\n\r\nFrom here $\\frac{sin x}{sin(60-t+x)}=\\frac{sin 45}{sin(60-t+45)}$ and therefore $x= 45$." } { "Tag": [ "trigonometry", "limit", "real analysis", "real analysis unsolved" ], "Problem": "Find a real number $ r\\ne1$ for which $ \\lim_{n\\to \\infty}\\sin(r^n)$ converges to a number unequal to 0,\r\n(or show that such a number does not exist).", "Solution_1": "Do you have a solution?", "Solution_2": "no :(\r\n\r\n(The message was too small. So I made the message longer before submitting).", "Solution_3": "Maybe this is idea (only idea not the full solution, I really don't wanr to spend time to solving this problem :lol: )\r\nI will speak roughly by the approximation language).\r\nsuppose there exist limit, then we can write \r\n$ r^{n} \\equal{} 2\\pi k_{n} \\plus{} b \\plus{} \\eta_{n}(\\pi \\minus{} 2b) \\plus{} \\varepsilon_{n}$\r\nwhere $ k_{n} \\in \\mathbb{Z}$, $ \\eta_{n}$ takes value from the set $ \\{0,1\\}$ or $ \\eta_{n} \\in \\{0,1\\}$ and $ \\varepsilon_{n} \\to 0$ as $ n \\to \\infty$\r\nI also suppose that $ \\eta_{n} \\equal{} 0$ for all $ n$ (I believe that such steps as $ r > 1$, $ \\eta_{n} \\equal{} 0$ is true and one can explain it. :lol: )\r\nalso suppose that $ k_{0} \\equal{} 0$ then we have :\r\n$ r^{n \\plus{} 1} \\equal{} 2\\pi k_{n \\plus{} 1} \\plus{} b \\plus{} \\varepsilon_{n \\plus{} 1}$ thus we have :\r\n$ 2\\pi k_{n \\plus{} 1} \\plus{} b \\plus{} \\varepsilon_{n \\plus{} 1} \\equal{} r(2\\pi k_{n} \\plus{} b \\plus{} \\varepsilon)$ hence :\r\nfor sufficiently large $ n$ we have :\r\n$ k_{n \\plus{} 1} \\approx \\frac {b(r \\minus{} 1)}{2 \\pi} \\plus{} rk_{n}$ or \r\n$ k_{n \\plus{} 1} \\approx \\frac {b(r^{n \\plus{} 1} \\minus{} 1)}{2\\pi}$\r\non the other hand we have : $ r^{n \\plus{} 1} \\equal{} 2\\pi k_{n \\plus{} 1} \\plus{} b \\plus{} \\varepsilon_{n \\plus{} 1}$ hence, for the sufficiently large $ n$ we have \r\n$ 2\\pi k_{n \\plus{} 1}\\approx 2\\pi k_{n \\plus{} 1}b \\plus{} b^{2}$, but it's impossible , since such $ b$ does not exist.", "Solution_4": "No comment :what?:" } { "Tag": [ "algorithm" ], "Problem": "If I want to teach myself Computer Science to the point of being able to do well on an AP Computer Science [i]course[/i] that's online, what should I learn?\r\n\r\nIt's Java based from what I hear, so is learning Java sufficient?", "Solution_1": "Two great books:\r\n\r\nConcrete Mathemathics by Knuth,Graham,Patashnik - contains exercises + solutions to all of them\r\nIntroductio to Algorithms by Cormen, Leiserson, Rivest, Stein - a lot of useful stuff.\r\n\r\nAnother great resource:\r\n\r\nocw.mit.edu\r\n\r\nAnd judges( in case you'd like to practice):\r\n\r\ntrain.usaco.org - worth doing\r\nacm.uva.es\r\nspoj.pl\r\n...." } { "Tag": [ "pigeonhole principle", "geometry", "analytic geometry", "AMC", "USA(J)MO", "USAMO" ], "Problem": "1) Suppose a triangle can be placed inside a unit square such that the center of the square is not inside the triangle. Show that one side of the triangle has length less than 1.\r\n\r\n2) A region M in the plane has area > 1. Prove that there are two points (a, b) and (c, d) in M such that a - c and b - d are both integers.\r\n\r\n3) In a party, n girls and n boys are paired. One observes that in each pair, the difference in height is < 10 cm. Show taht the difference in height for the kth tallest boy and the kth tallest girl is also < 10 cm for k = 1, 2, ..., n.\r\n\r\n4) Prove that from any sequence of 2003 reals, one can choose a consecutive block whose sum differs from an integer by at most 1/1000.\r\n\r\n5) In a contest with 8 problems, 8 students took part. Each problem was solved by 5 students. (a) Prove that there are two students who between them solved all eight problems. (b) Is this true if 5 is replaced by 4 ?\r\n\r\n6) Prove that among any 6 integers, there will be a pair whose sum/difference is divisible by 9.", "Solution_1": "(6)[hide]more generally, if we are given n numbers, there exist 2 whose sum/diff. is divisible by 2n-3. We construct (n-1) boxes, labeled 0, 1&(2n-4), 2&(2n-5), 3&(2n-6), ..., (n-2)&(n-1). We put each of the n numbers in the box that contains its residue mod (2n-3). Now, by pigeonhole, as we have n numbers and (n-1) boxes, two must be in the same box. Either their sum or difference is 0, mod 2n-3. (here n=6)[/hide]", "Solution_2": "(4) [hide]let the sequence of reals be given by ak, 1:le:k:le:2003. define si as {:Sigma:(k=1 to i) ak}, where {x} represents the fractional part of x, or x-[x]. now, divide the real number line from 0 to 1 into 1000 parts. If one of the si is in the first or last block, from 0 to 1/1000, we're done. If not, there exist two of the si in another block. Taking the difference yields a consecutive block of that sequence, whose sum is within 1/1000 of an integer.[/hide]", "Solution_3": "(2) [hide]let us proceed by contradiction. superimpose (is that a real word?) this region on a cartesian plane tiled by horizontal and vertical points through the lattice line, subdividing the plane into 1 by 1 unit squares. now, if (x,y) is the ordered pair corresponding to a point in the plane, define its SPECIAL coordinates (creative, aren't i?) as ({x},{y}), where {a}=fractional part of a=a-[a]. then, assuming that there are no two points of m satisfying the given condition, there cannot be any two points in M having the same pair of SPECIAL coordinates. So take any 1 by 1 square in the plane, and note that M can be mapped into this square without any overlap by the preceding observation. however, this would mean the max area of M is 1, a contradiction.[/hide]", "Solution_4": "An extra question:\r\n\r\nThe bound 1/1000 for problem 4 can obviously be sharpened.\r\nWhat is the sharpest possible bound ?", "Solution_5": "I'm afraid I don't understand number 4 at all.\r\n\r\nMystic, I'm not entirely convinced of your number 2 -- I can design plenty of injective maps from regions of area greater than 1 to a 1x1 square -- you have to prove that just because two points aren't mapped to the same place means that the initial area is less than 1.", "Solution_6": "Okay, so lets do number 5.\n\n[hide]If any student solved all 8 problems, we're done. If any student solved 7 problems, some other student must have solved the 8th, so we take those 2 students and we're done. If any student solved 6 questions, consider the other 2 students. Each was solved by 5 people, thus they must both have been solved by at least 1 of the same people. Take that person and our six-question guy, and we're done.\n\n\n\nThus, the only scenario we are left with is that in which each student solved exactly 5 questions. Now then, returning to this much-neglected problem:\n\nTake any of the students, and WLOG he has solved questions 1, 2, 3, 4 and 5. We wish to show that some other student solved questions 6, 7, and 8. Now, between them, questions 6, 7, and 8 have been solved a total of 15 times by the 7 other students. By the pigeonhole principle, one of the students must have accounted for 3 of these solutions. Since he can't solved the same question 3 times (I would have had a perfect score on USAMO last year if I could have done that ), he must have solved each of 6, 7 and 8. Thus, we are all done.\n\n\n\nI highly suspect the same is not true for 4. I'll get on it now.[/hide]", "Solution_7": "Arne wrote:An extra question:\n\nThe bound 1/1000 for problem 4 can obviously be sharpened.\nWhat is the sharpest possible bound ?\n\n\n\n[hide]If the bound is of the form 1/k, with k an integer, we construct k boxes, and define si as before. now, we have that if any of the si are in the first or last box, we're automatically done. now, if not, we want at least 2 of the si in one of the remaining boxes so that we can take their difference. So i think k is 2004, and the lowest bound is 1/2004. however, i don't know why the bound should be of the form 1/k. sort of non-rigourously, we can see that if k is not an integer, we will have [k] boxes and then a mini-box in the middle, that will not sharpen the pigeonholing argument. but that's not too terribly rigorous.[/hide]\n\n\n\n\n\nugh, JBL, you're right on my #2. if i said [hide]construct the mapping from M to the unit square with vertices at (0,0), (1,1), (0,1), and (1,0), such that any point in M is mapped into its SPECIAL coordinates, and then note that this mapping must be injective by the above reasoning, and then note that this preserves area because basically splitting M based on the grid lines and then reassemble it in one square won't change area[/hide] would that fix it?", "Solution_8": "Yeah, I think that will do it. The way I would phrase it is to say that it preserves area over the open lattice squares, and since the other parts (all the grid lines) don't have any area, it does in fact preserve area. You might have to mess that around a little bit when you talk about the boundaries, but I don't think that's a really major point.", "Solution_9": "For 4), a hint : \r\n\r\nConsider the sequence 1/2004, 1/2004, ..., 1/2004 (2003 terms).\r\nThis sequence has k = 1/2004.", "Solution_10": "my bad", "Solution_11": "\"between them\" implies when the solutions from two students are put together, all 8 problems are solved.", "Solution_12": "In your particular example, students 1 and 8 (among several other pairs) between them solved all 8 questions." } { "Tag": [ "floor function" ], "Problem": "Solve \r\n\\[ \\left\\lfloor {\\frac{{x \\minus{} p}}\r\n{p}} \\right\\rfloor \\equal{} \\left\\lfloor {\\frac{{ \\minus{} x \\minus{} 1}}\r\n{p}} \\right\\rfloor \r\n\\]\r\n\r\nfor real $ x$ and nonzero integer $ p$", "Solution_1": "I think $ x$ belongs to $ [0 , \\frac {p\\minus{}1}{p}]$" } { "Tag": [ "FTW" ], "Problem": "A salesman buys a coat at $ \\$64$ less than $ 12.5 \\%$. He then sells the coat at a gain of $ 25\\%$ of his cost after allowing a $ 20\\%$ discount on the marked price. What is the marked price, in dollars, of the coat?", "Solution_1": "I don't understand that first sentence.", "Solution_2": "i know... the first sentence doesn't make sense... maybe it's saying he bought it at a 12.5% discount, and the final price was 64? 0.o sometimes the questions and solutions for FTW are a bit shaky, like for some reason, the answers are all like $ DFRAC(7)(11)$. 0.o it would be greatly appreciated if all of those bugs could be worked out, especially in the Alpha Test version. -.-", "Solution_3": "The first sentence actually means \"The salesman buys a coat at \\$64 [i]minus[/i] 12.5%\". \r\n\r\n12.5% = $ \\frac{1}{8}$, so $ 64 \\cdot \\frac{7}{8} \\equal{} \\$56$\r\nHe gains 25%, so $ \\$56 \\cdot 25\\% \\equal{} \\$14$ , $ \\$56 \\plus{} 14 \\equal{} 70$.\r\nTo find the marked price, we must reverse the discount, so we divide $ \\$70$ by $ \\frac{80}{100}$. The answer and marked price is $ \\$87.5$." } { "Tag": [ "geometry", "trigonometry" ], "Problem": "A surveyor is mapping a triangular plot of land. He measures two ofthe sides and the angle formed by these two sides and finds that thelengths are 400 yards and 200 yards and the included angle is 50\u00b0.What is the measure of the third side of the plot of land, to the nearest yard? What is the area of this plot of land, to the nearest square yard?", "Solution_1": "use the law of cosines...", "Solution_2": "Can you set up the equation using the law of cosines?\r\nI would be able to answer the question if I had the right equation." } { "Tag": [ "calculus", "inequalities unsolved", "inequalities" ], "Problem": "Find the maximum value of the expression: $ (x^3\\plus{}1)(y^3\\plus{}1)$ when $ x\\plus{}y\\equal{}1$. Without using calculus.\r\nI was trying cauchy-schwarz, but no success.", "Solution_1": "I solved, the answer is $ 2$.\r\nWhat do you mean by a solution without calculus? :?:\r\nEdit: don't read, I thought that $ x,y\\geq 0$.", "Solution_2": "[quote=\"danilorj\"]Find the maximum value of the expression: $ (x^3 \\plus{} 1)(y^3 \\plus{} 1)$ when $ x \\plus{} y \\equal{} 1$. Without using calculus.\n[/quote]\r\n$ (x^3 \\plus{} 1)(y^3 \\plus{} 1)\\leq4\\Leftrightarrow3 \\minus{} (x^3 \\plus{} y^3) \\minus{} x^3y^3\\geq0\\Leftrightarrow$\r\n$ \\Leftrightarrow3(x \\plus{} y)^6 \\minus{} (x \\plus{} y)^3(x^3 \\plus{} y^3) \\minus{} x^3y^3\\geq0\\Leftrightarrow$\r\n$ \\Leftrightarrow(x^2 \\plus{} 3xy \\plus{} y^2)^2(2x^2 \\plus{} 3xy \\plus{} 2y^2)\\geq0,$ which true.\r\nThe equality holds when $ x^2 \\plus{} 3xy \\plus{} y^2 \\equal{} 0$ id est, when $ x \\equal{} \\frac {1 \\plus{} \\sqrt5}{2},$ $ y \\equal{} \\frac {1 \\minus{} \\sqrt5}{2}$ or \r\n$ y \\equal{} \\frac {1 \\plus{} \\sqrt5}{2},$ $ x \\equal{} \\frac {1 \\minus{} \\sqrt5}{2}.$\r\nThus, the answer is $ 4.$", "Solution_3": "But how do you assure that 4 is the maximum?", "Solution_4": "[quote=\"danilorj\"]But how do you assure that 4 is the maximum?[/quote]\r\n\r\naquardy has proved that\r\n$ (x^3 \\plus{} 1)(y^3 \\plus{} 1) \\le 4$\r\nSo that the maximum is $ \\le 4$\r\n\r\nBut he also show example when the equality holds.\r\nHence 4 is the answer.", "Solution_5": "i have solution for this problem:\r\n Let $ xy\\equal{}t \\leq \\frac{1}{4}$\r\nand $ P\\equal{} 1\\plus{}x^3y^3\\plus{}(x\\plus{}y)^3\\minus{}3xy(x\\plus{}y)\\equal{}t^3\\minus{}3t\\plus{}2.$\r\nWe have:$ f'(t)\\equal{}3t^2\\minus{}3\\equal{}0 \\leftrightarrow t\\equal{}\\minus{}1 ,t\\equal{}1$\r\nThus,$ MaxP\\equal{}f(\\minus{}1)\\equal{}4$\r\nLast, we solve equation $ t^2\\minus{}t\\minus{}1\\equal{}0$, to find values$ x$ and $ y$", "Solution_6": "Let $x,y $ be reals such that $ x + 2y = 1$. Prove that\n$$ (x^3 + 1)(y^3 + 1)\\leq \\frac{11}{2}+3\\sqrt 2$$" } { "Tag": [ "AMC", "USA(J)MO", "USAMO", "MATHCOUNTS", "AMC 10", "\\/closed" ], "Problem": "In the middle School Tournaments, there have been some tourneys that involved USAMO and IMO problems, and I was wondering if there could be a forum such that they are separated? (ie. High School Tournaments)", "Solution_1": "Well, not sure exactly which forum you are referring to. But the mods of the respective forum should rather execute their moderator rights more effectively by moving all USAMO and IMO problems to the olympiad section, or other suitable fora possibly. If you see such cases you may also want to pm the mods in charge.", "Solution_2": "007math wants to create a USAMO/IMO Tournaments forum.\r\n\r\nBut how well would that work? A tournament of IMO problems? I think maybe IMO problems should just be presented individually.", "Solution_3": "gah...i mean an overall High School Tournaments Forum (like middle school :lol: )", "Solution_4": "[quote=\"007math\"]gah...[/quote]\r\n\r\noh... sorry if i frustrated you by overgeneralizing your suggestion. :)", "Solution_5": "I just like saying gah... :P :D", "Solution_6": "Subsection of MATHCOUNTS - Middle School Tournaments\r\nThere could be a section of High School for tournaments, as tournaments in MST are very boring with only MATHCOUNTS problems, and if someone tries harder problems, mods rush in and warn that those tournaments shouldn't be there.", "Solution_7": "Well we could start some tournaments in the high school forums, but how much would a separate tournaments forum be used?", "Solution_8": "We could create a whole new forum altogether, split into Middle School Tournaments and High School Tournaments.", "Solution_9": "Idea collapsed?", "Solution_10": "Reasons against:\r\n\r\nNewbies will promptly post all their AMC problems in this forum, since \"it's for High School Competitions, and the AMC is a High School Competition\", thus making it an interminable moderator headache for whoever is unlucky enough to mod that forum\r\n\r\nThere really aren't that many fullblown AoPS competitions in high school, whereas there are quite a few active mock Mathcounts competitions and they clutter up the main Mathcounts forum. Thus, a high school forum wouldn't be helpful.", "Solution_11": "[quote=\"solafidefarms\"] thus making it an interminable moderator headache for whoever is unlucky enough to mod that forum\n[/quote]\n\nI would mod it :D \nAnd also, every tournament with AMC problems, or similar could easily be locked, or set to doom.\n\n[quote=\"solafidefarms\"]\nwhereas there are quite a few active mock Mathcounts competitions and they clutter up the main Mathcounts forum. [/quote]\r\n\r\nMost users in MST are probably bored of easy MC problems (only newbies are the problem, but mostly the same people join every tournament). 007math pointed out that in his first post (MC problems are just too boring and easy to just post them in tournaments)", "Solution_12": "Well, I wouldn't have a problem modding it.\r\n\r\nA lot of people are past MC level, and would like a tournament with harder problems.", "Solution_13": "Heh, sometimes modding is more annoying than it looks. \r\n\r\nPerhaps instead tournaments with more difficult problems could just be posted in Middle School Tournaments.", "Solution_14": "Making harder problems that aren't computationally intensive or instantly solved by one application of something is fairly difficult. No offense to anyone, but the problems in many mock mathcount tests are quite computationally intensive. However, not all Mathcounts problems are boring. They might not be as difficult, but there are several with quite elegant solutions. \r\n\r\nI'm not sure about adding a high school tournaments forum...there are already some mock tests, although, to be fair, there aren't too many. I guess the main thing that I'm worrying about is that a lot of the tests that will come up are going to have half-baked problems. I agree with solafidefarms.\r\n\r\nBut hey, if you think that this is a good idea, go for it.", "Solution_15": "[quote=\"AIME15\"]We could create a whole new forum altogether, split into Middle School Tournaments and High School Tournaments.[/quote]\r\n\r\nThis is kinda a good idea.", "Solution_16": "I'm not sure that making a whole new forum for HST is the right way to go. I think that we should keep our Middle school tournament forum and the next time someone makes a mock test, they will just specify how hard the problems are going to be so that people who can't do USAMO problems don't sign up to some mock MATHCOUNTS test with USAMO level problems.", "Solution_17": "couldn't you just make a mock test in the high school forum and lable it \"mock test\" like everyone used to do in mathcounts forum before the new subforum?\r\n\r\nor is there a rule in the high school forum forbidding this?", "Solution_18": "xpmath, again I say that less tournaments are probably caused by lack of MC level problems.\r\nmathemagician1729, OK, but maybe renaming the forum MST to M&HST (ok, it's not good name)", "Solution_19": "[quote=\"vallon22\"]couldn't you just make a mock test in the high school forum and lable it \"mock test\" like everyone used to do in mathcounts forum before the new subforum?\n\nor is there a rule in the high school forum forbidding this?[/quote]\r\n\r\ni think this is a good idea. mock tests have helped me a tremendous amount for mathcounts, so im sure a mock amc 10 or 12 wouldnt hurt. or maybe, a subforum entitled High School Tournaments.", "Solution_20": "You could probably post Mock AMC's in the AMC forum.", "Solution_21": "My idea is just to move middle school tournements and put it into a forum called Tournements, which would also include High school tournements.", "Solution_22": "...which is EXACTLY what I said?" } { "Tag": [ "geometry", "circumcircle", "ratio", "trigonometry", "conics", "parallelogram", "geometry unsolved" ], "Problem": "There is a circle with its circumcenter as O and its radius is 1. There is a point A outside the circle. AM,AN is the tengents to the circle O with M N as the tangent points respectively. There is a point L in bad arc MN (not equal to M,N). A line passing point A parallel to line MN intesects with ML,NL at point P,Q respectively. if PQ=2, cos(angle POQ)=1/3, find the length of AM.\r\n\r\ni have no idea to use the given condition cos(angle POQ)=1/3. i dun know how to translate it.", "Solution_1": "Assuming that \"bad\" arc $ MN$ should be \"back\" arc $ MN$ (more distant arc from $ A.$ Let $ AL$ cut the circle $ (O)$ again at $ K$ on the front arc $ MN$ (closer to $ A$) and let $ X \\equiv MN \\cap AL$. Using directed line segments:\r\n\r\n$ \\overline{AP} \\cdot \\overline{AQ} \\equal{} \\frac{\\overline{AL}^2}{\\overline{XL}^2} \\cdot \\overline{XM} \\cdot \\overline{XN} \\equal{} \\frac{\\overline{AL}^2}{\\overline{XL}^2} \\cdot \\overline{XL} \\cdot \\overline{XK} \\equal{} \\frac{\\overline{AL}^2}{\\overline{XL}} \\cdot \\overline{XK} \\equal{}$\r\n\r\n$ \\equal{} \\frac{\\overline{AL}}{\\overline{AK}} \\cdot \\frac{\\overline{XK}}{\\overline{XL}} \\cdot (\\overline{AL} \\cdot \\overline{AK}) \\equal{} \\minus{}\\overline{AM}^2$\r\n\r\nbecause the cross ratio $ \\frac{\\overline{AL}}{\\overline{AK}} \\cdot \\frac{\\overline{XK}}{\\overline{XL}} \\equal{} \\minus{}1$ is harmonic (easily seen by projecting the circle $ (O)$ to a circle and the line $ PQ$ to infinity). Giving $ \\cos \\widehat{QOP}$ is obfuscation, we need $ \\tan \\widehat{QOP} \\equal{} ... \\equal{} 2 \\sqrt{2}.$\r\n\r\n$ \\tan \\widehat{QOP} \\equal{} \\frac{\\tan \\widehat{QOA} \\plus{} \\tan \\widehat{AOP}}{1 \\minus{} \\tan \\widehat{QOA} \\cdot \\tan \\widehat{AOP}} \\equal{} \\frac{\\frac{\\overline{QA}}{\\overline{OA}} \\plus{} \\frac{\\overline{AP}}{\\overline{OA}}}{1 \\minus{} \\frac{\\overline{QA}}{\\overline{OA}} \\cdot \\frac{\\overline{AP}}{\\overline{OA}}} \\equal{}$\r\n\r\n$ \\equal{} \\overline {OA} \\cdot \\frac{\\overline{QA} \\plus{} \\overline{AP}}{\\overline{OA}^2 \\minus{} \\overline{QA} \\cdot \\overline{AP}} \\equal{} \\frac{\\overline{OA} \\cdot \\overline{QP}}{\\overline{OA}^2 \\minus{} \\overline{AM}^2} \\equal{} \\frac{\\overline{OA} \\cdot \\overline{QP}}{\\overline{OM}^2} \\equal{} 2 \\sqrt{2}$\r\n\r\nSubstituting $ \\overline{OM}^2 \\equal{} 1,$ $ \\overline {QP} \\equal{} 2$ yields $ \\overline{OA} \\equal{} \\sqrt{2}$ and $ \\overline{AM}^2 \\equal{} \\overline{OA}^2 \\minus{} \\overline{OM}^2 \\equal{} 1.$ Done. The problem does not ask for this, but from $ \\overline{QA} \\cdot \\overline{AP} \\equal{} \\overline {AM}^2 \\equal{} 1$ and $ \\overline{QA} \\plus{} \\overline{AP} \\equal{} \\overline {QP} \\equal{} 2,$ we get $ \\overline {QA} \\equal{} \\overline {AP} \\equal{} 1,$ points $ P, Q$ are symmetrical with respect to the line $ OA$ and $ L \\in OA.$", "Solution_2": "[quote=\"yetti\"]\nbecause the cross ratio $ \\frac {\\overline{AL}}{\\overline{AK}} \\cdot \\frac {\\overline{XK}}{\\overline{XL}} \\equal{} \\minus{} 1$ is harmonic (easily seen by projecting the circle $ (O)$ to a circle and the line $ PQ$ to infinity).[/quote]\r\n\r\ni dun understand the reason for the harmonic division..(i am still a beginner of projective geom), can yo explain a little bit more?", "Solution_3": "Suppose you have 2 planes $ \\pi, \\pi',$ not parallel, and a point $ S$ not on either of them. Arbitrary line through $ S$ cuts $ \\pi, \\pi'$ at $ P, P'.$ $ P'$ is image of $ P$ in the central projection of $ \\pi$ to $ \\pi'$ from $ S.$ But lines through $ S$ parallel to $ \\pi'$ do not cut it, even though they cut $ \\pi.$ Collection of all lines parallel to $ \\pi'$ forms a plane $ \\sigma \\parallel \\pi'$ and the plane $ \\sigma$ cuts the original plane $ \\pi$ in a line $ l.$ The image $ l'$ of $ l \\in \\pi$ does not show in $ \\pi'.$ We say that the line $ l \\in \\pi$ was projected to the line at infinity $ l' \\in \\pi'.$ Each line through $ S$ parallel to $ \\pi'$ defines a direction in $ \\sigma$ or in $ \\pi'$ and collection of all these lines is a collection of all directions in $ \\sigma$ or in $ \\pi'.$ Therefore, point at infinity is the same thing as direction in a plane and line at infinity is collection of all directions in a plane. For example, 2 parallels have a common point at infinity - they have the same direction. Any 2 lines intersecting on the line $ l$ projected to infinity become parallel, because their common point is at infinity.\r\n\r\nGenerally, central projection takes a circle to a conic section and it preserves tangencies. I used a well known theorem (with long 3D proof): Given a circle and a line not intersecting this circle, a suitable central projection exists, which takes the given circle to another circle and the given line to infinity. In your problem, the circle $ (O)$ can be projected to another circle $ (O')$ and any line through $ A$ not intersecting the circle $ (O),$ for example the line $ PQ,$ to infinity. Since the image $ A'$ of $ A$ is at infinity, images $ A'M', A'N'$ of the circle tangents $ AM, AN$ and image $ A'K'L'$ of the circle secant $ AKL$ become parallel, $ M'N' \\perp K'L'$ becomes a diameter of the projected circle $ (O').$ The cross ratio of 4 points is preserved in a central projection,\r\n\r\n$ \\frac {\\overline{AL}}{\\overline{AK}} \\cdot \\frac {\\overline{XK}}{\\overline{XL}} \\equal{} \\frac {\\overline{A'L'}}{\\overline{A'K'}} \\cdot \\frac {\\overline{X'K'}}{\\overline{X'L'}} \\equal{} \\frac {\\overline{X'K'}}{\\overline{X'L'}} \\equal{} \\minus{} 1$\r\n\r\n$ \\overline{A'L'}, \\overline{A'K'}$ are infinite segments with the same direction, they cancel out, because their difference is insignificant compared to infinity. $ X'$ becomes the midpoint of $ K'L',$ $ \\overline{X'K'}, \\overline{X'L'}$ are oppositely directed segments of equal length, that's why $ \\minus{} 1.$\r\n__________________________________________\r\n\r\nYou could also use the following theorem: 2 diagonals of a complete quadrilateral cut the 3rd one harmonically. Let $ AB, BC, CD, DA$ be 4 lines forming a complete quadrilateral. $ X \\equiv AB \\cap CD,$ $ Y \\equiv BC \\cap DA,$ $ Z \\equiv AC \\cap BD.$ Lines $ AC, BD, XY$ are the 3 diagonals. How do you prove the theorem ? Project the line $ XY$ to infinity, the image $ A'B'C'D'$ of $ ABCD$ becomes a parallelogram, and use invariance of the cross ratio in a central projection. \r\n\r\nIn your problem, you also have to know that $ MN$ is a polar of the point $ A$ with respect to the circle $ (O).$ Let $ UV$ be another circle secant from $ A.$ Then $ B \\equiv KV \\cap LU,$ $ C \\equiv KU \\cap LV$ lie on the polar $ MN.$ Let $ KV, LU, KU, LV$ are the 4 lines forming a complete quadrilateral, then $ KL, UV, BC$ are the 3 diagonals. The 2 diagonals $ UV, BC \\equiv MN$ cut the 3rd diagonal $ KL$ harmonically at $ A, X,$ so that \r\n\r\n$ \\frac {\\overline{AL}}{\\overline{AK}} \\cdot \\frac {\\overline{XK}}{\\overline{XL}} \\equal{} \\minus{} 1.$\r\n\r\nThe 1st method (projecting the circle $ (O)$ to a circle and the line $ PAQ$ to infinity is much quicker and easier to imagine.", "Solution_4": "thank you very much,yetti~:)" } { "Tag": [], "Problem": "Okay firstly please be honest. And don't post srini does not sleep at all and rakesh sleeps only for 2.5 hours every year. Post proper stuff here.\r\n\r\nI used to be a typical late night sleeper. Sleeping after 2:30 and sometimes going on till 3:30. And then most of my morning was spoiled. I used to get up at 8:30 mull around for another hour. go back to sleep and then start productive work only at 1:00. However I wanted to shift my biological clock because this was not really helping me in comp exams and I was feeling sleepy and I started sleeping at 12:00 itself with disastrous results. I still feel sleepy in the morning. :D:D\r\n\r\nSo what time do you goto sleep? And how long do you sleep?", "Solution_1": "Used to be from 11:00 to 6:30 earlier abt a month before :D\r\nBut bcos of the influence of my friends n shre in particular, i went to late night and am not able to sleep till 12:00 am :( I try to but still, dont have the inclination!\r\nNowadays its been from 12:30 am to 8:30 am\r\n\r\nPS: Elastiboysai had written he slept from 3:50 to 9:30 in another post :)\r\nAnd ofc for the sake of saying it i shall say:\r\nsrini does not sleep at all and rakesh sleeps only for 2.5 hours every year :P", "Solution_2": "ha well, my sleeping patern is shockingly similar to shreyas's :D\r\nespecially in the last few months, i've been sleeping at around 2- 2.30 and wake up at 1030- 11. Effectively, i start studying properly only at around 1 :(\r\n\r\nand yesterday, i promised myself that from now on, i'd sleep at 12 itself to \"tune my biological clock\" :D \r\nbut it was a DISASTER. i just couldn't get proper sleep and for at least an hour or 2, i was just lying down and yawning. as a result i got up only at 930 today :(\r\n*yawns*", "Solution_3": "Baby, why did you post the same thing so many times? :D \r\n\r\nMy sleeping pattern is probably the most erratic but still.. I usually have dinner and go to sleep at around 9 and then get up at 11-11.30, then study/do something that people would tend to mistake for studying till 2.30, watch Dawson's Creek from 2.30 -3 and then go back to sleep and then get up in the morning at 7 - 7.30...\r\n\r\nGreen tea is a very potent stimulant :D", "Solution_4": "The mod can edit that :D Vetti mod is always here to the rescue :D\r\nThat was a consequence of the stupid server here which was swaying between various links all by itself without me touching anything :D\r\n\r\nAnd incase u dont believe that it wasnt my fault, i couldnt have posted all that as consecutive posts wont be allowed for less that 10 mins :P", "Solution_5": "hey to be frank i go to sleep at around 9 and wake up at 7 am :P and rohit dont tell i dont sleep and all for sleep is the most important aspect of my life da and one small correction rakesh sleeps for only 1:00:00:02 hrs and please dont exxagerate it to 2.5 hrs and all!!! :D", "Solution_6": "Dei srini, just bcos shre said u can write nething u feel is right, doesnt mean u can lie :D\r\n[quote=\"lankies\"]\nYou lie like a brute :P\n[/quote]", "Solution_7": "[quote=\"madness\"]Dei srini, just bcos shre said u can write nething u feel is right, doesnt mean u can lie :D\n[quote=\"lankies\"]\nYou lie like a brute :P\n[/quote][/quote]\r\nhey it shows u are offline but how did you post now???", "Solution_8": "dei its saying the truth da :D\r\nim offline :P :P", "Solution_9": "[quote=\"madness\"]dei its saying the truth da :D\nim offline [/quote]\r\nhey is there any facilities for you to change your active state!!! :?:", "Solution_10": "[quote=\"Albert Einstien\"][quote=\"madness\"]dei its saying the truth da \nim offline [/quote]\nhey is there any facilities for you to change your active state!!! [/quote]\r\n\r\nwell there is pathetic fallacy :P :D \r\nyou cant change the active state of a compd atleast :P", "Solution_11": "i don't any compromise with my sleep--i sleep at 11.30 and would have woken up at 7.30 if it weren't 4 school\r\ni've to wake at 6:15 but i do take a nice nap at school :D :D", "Solution_12": "Now that u all r talkin abt it,\r\ni used 2 sleep frm 11.30 to 6.30-7.00 normally :wink: \r\nfor the past 1.5 months my sleep pattern has drastically changed.\r\nSomthin like 3.30 ,4 to 9.40 10.\r\nFunnily I too tried to sleep early yest- 11.40, it worked bt i woke up only at 9. :rotfl:", "Solution_13": "I'm scared I'll sleep off during JEE paper I. :( My full momentum is usually gained only after 3. :(", "Solution_14": "go to sleep da now :)", "Solution_15": "I am scared abt paper 2 :oops: \r\nI really slept off during FT5 paper 2 (thats bcoz i had eaten 2 much n was ready 2 burst) :rotfl:", "Solution_16": "oh my god u all study till 3 and all ah ???how is it possible ??? u all kattans!!!!!!!!!!! :o :o :o", "Solution_17": "Look who is talking. Thats all I have to say", "Solution_18": "[hide=\"TO SRINI AND ROHIT:\"]\nI DON'T SLEEP AT ALL DA :P :P \n[/hide]\r\n\r\nAnyway the truth is I sleep around 12:00,12:30 and ofc get up only at 8:30 in the morning. It has been my routine for about 2 months now. :rotfl:", "Solution_19": "I generally sleep at around 12- 1230 and get up between 6:30 to 7:00 .\r\n\r\nvery few times i have tried 11 - 4:45 . \r\n\r\nBoth have equal productivity", "Solution_20": "Shifted to my old self. Sleeping only after 2. :rotfl: :rotfl:", "Solution_21": "u r damaging ur pineal gland\r\n\r\ni think soumya is practising to prevent jetlags :rotfl: :rotfl:" } { "Tag": [ "USAMTS", "LaTeX" ], "Problem": "I was looking through the USAMTS webiste, and I noticed it said that one must send both the pdf and the latex file if using LaTeX. I didn't do this when I submitted it, does that mean my work will get disqualified?", "Solution_1": "No, it won't be disqualified. If we don't have the TeX code, then we probably won't publish your solution. But your paper will still be graded." } { "Tag": [], "Problem": "Show that there are an infinity of natural numbers wich cannot be written as a^3+b^3 with a,b naturals but can be written in the same form with a,b rationals(positive).", "Solution_1": "However any natural number gives residue 0,1,8 mod9\r\nthe number 9k+3 can't be written as a^3+b^3 with a,b naturals.\r\nNow let a=9k^3-1,b=9k^3+1,then n=(a^3+b^3)/9k^3 is natural and gives residue 3 mod9,that we had to prove." } { "Tag": [ "probability" ], "Problem": "A fair, 6-sided die is thrown and a card is drawn from a 52-card, standard deck. Find the probability that a multiple of 3 appears on the die and a face card (Jack, Queen, King, or Ace) is drawn.", "Solution_1": "[hide]There are two multiples of 3 on a die, so the probability is $\\frac12.$\n\nThere are 12 face cards, so the probability is $\\frac{12}{52},$ or $\\frac{3}{13}.$\n\n$\\frac12\\cdot\\frac3{13}=\\boxed{\\frac3{26}}$[/hide]", "Solution_2": "[quote=\"i_like_pie\"][hide]There are two multiples of 3 on a die, so the probability is $\\frac12.$\n\nThere are 12 face cards, so the probability is $\\frac{12}{52},$ or $\\frac{3}{13}.$\n\n$\\frac12\\cdot\\frac3{13}=\\boxed{\\frac3{26}}$[/hide][/quote]\r\n\r\nwait... wouldn't the probability of the die thing be $\\frac13.$, and the face card one be $\\frac{16}{52},$ or $\\frac{4}{13}.$\r\nso then it would be $\\frac13\\cdot\\frac4{13}=\\boxed{\\frac4{39}}$\r\nam I wrong???...", "Solution_3": "xxxxx, It would not be 4/39 because you plus the two probabilities together, not times.", "Solution_4": "[quote=\"Necrowarrio0\"]xxxxx, It would not be 4/39 because you plus the two probabilities together, not times.[/quote]\r\nYou multiply, and xxxxx is correct. The correct answer is ${4\\over39}.$", "Solution_5": "Oh is it multiply? I thought it was plus..... :|\r\n\r\nThe above was edited.\r\n\r\nAH SORRY!!!! IT IS MULTIPLY.................... :(", "Solution_6": "In probablity \"and\" usually tells you to use multiplication\r\nwhile \"or\" usually tells you to use addition\r\n\r\nAren't face cards just jack, queen, and king? :P \r\n\r\nI guess it doesn't matter because the question tells you that it includes ace as a face card" } { "Tag": [ "abstract algebra", "superior algebra", "superior algebra solved" ], "Problem": "for $S= \\{m/n \\in Q| n Is Odd \\}$, under usual operations on real numbers. I must determine if this is a ring. Is it sufficient to show that $S$ satisfies the ring axioms? ie: $(m_1/n_1) + (m_2/n_2) = (m_2/n_2) + (m_1/n_1)$ so $S$ is abelian under addition, then show Closure, Associativity, and distributivity", "Solution_1": "[quote=\"zargon\"] I must determine if this is a ring. Is it sufficient to show that $S$ satisfies the ring axioms?[/quote]\r\n\r\nWell if an object satisfies all the properties in the definitition of a class, then it belongs to the class. Basic logic.", "Solution_2": "The least obvious thing to check is closure.\r\nTake $a/b,c/d\\in S$. Then $b,d$ are odd numbers. $a/b+c/d=(ad+bc)/bd$. This may not be a reduced form for $a/b+c/d$, but since $bd$ is odd, no matter by what you divide it, you still get an odd number, so $a/b+c/d\\in S$." } { "Tag": [ "modular arithmetic" ], "Problem": "In order that $998680883748524N5070273447265625$ equal $1995^{10}$, what is N?", "Solution_1": "[hide]\n$1995=3\\cdot 5\\cdot 7\\cdot 19$, so $1995^{10}$ is a multiple of $9$, which means the sum of all the digits must be divisible by $9$.\n\nAdding all the digits, we get $154\\equiv 1\\pmod{9}$, so $N=8$.\n(I hope I added them correctly... :roll: )[/hide]", "Solution_2": "[quote=\"frt\"][hide]\n$1995=3\\cdot 5\\cdot 7\\cdot 19$, so $1995^{10}$ is a multiple of $9$, which means the sum of all the digits must be divisible by $9$.\n\nAdding all the digits, we get $154\\equiv 1\\pmod{9}$, so $N=8$.\n(I hope I added them correctly... :roll: )[/hide][/quote]\r\n\r\nI was so intimidated by the number that I didn't think to use that simple method. \r\n\r\n(and you added correctly according to my super [2 dollar] calculator)", "Solution_3": "[quote=\"frt\"]\n$1995=3\\cdot 5\\cdot 7\\cdot 19$, so $1995^{10}$ is a multiple of $9$.[/quote]You mean it's a multiple of 3. ;)", "Solution_4": "[quote=\"4everwise\"][quote=\"frt\"]\n$1995=3\\cdot 5\\cdot 7\\cdot 19$, so $1995^{10}$ is a multiple of $9$.[/quote]You mean it's a multiple of 3. ;)[/quote]\r\n\r\nWell, since 1995 has one multiple of 3, then $1995^{10}$ has a factor of $3^{10}$ and $3^2=9$.", "Solution_5": "Right. I'm losing it today, lol. :blush: :rotfl:" } { "Tag": [], "Problem": "$ P$ and $ Q$ are each a distinct member of the set $ \\{6, \\minus{}\\frac 12, \\minus{}3, \\frac 13\\}$. What is the least possible value of the quotient $ P\\div Q$?", "Solution_1": "To minimize first we try to make $ P\\div Q$ negative. \r\n\r\nIf $ P$ is positive and $ Q$ is negative, then the minimum possible value of $ P\\div Q$ is $ \\frac {6}{ \\minus{} \\frac {1}{2}} \\equal{} \\minus{} 12$. \r\n\r\nIf $ P$ is negative and $ Q$ is positive, then the minimum is $ \\frac { \\minus{} 3}{\\frac {1}{3}} \\equal{} \\minus{} 9$. $ \\minus{} 12 < \\minus{} 9$, so $ \\boxed{ \\minus{} 12}$" } { "Tag": [ "inequalities", "function", "algebra solved", "algebra" ], "Problem": "a,b,c,d>0 Prove that:\r\n\r\na/(b+2c+3d)+ b/(c+2d+3a)+ c/(d+2a+3b)+ d/(a+2b+3c) \\geq 2/3", "Solution_1": "you can consider wlog that \r\n\r\na+b+c+d=1 (there is a post here that explains why this is a \"WLOG\" consideration)\r\nthen you use Jensen for the convex function f : (0,+oo)-->R, f(x)=1/x with wieghts a,b,c,d.\r\n\r\ncheers!", "Solution_2": "I think Cauchy also works. (All sums are cyclic.) \r\nLet L denote the left - hand side. \r\nPut A = a + b + c + d and B = ab + ac + ad + bc + bd + cd.\r\nThen we get L * \\suma(b + 2c + 3d) \\geq (a + b + c + d).\r\nNow \\suma(b + 2c + 3d) = 4B.\r\nSo we have L \\geq (a + b + c + d)/(4B).\r\nSo we want to prove that (a + b + c + d)/(4B) \\geq 2/3.\r\nThis is equivalent to 3A + 6B \\geq 8B or 3A \\geq 2B.\r\nNow we have\r\n1) A = a + b + c + d \\geq ab + bc + cd + da\r\n2) A = a + c + b + d \\geq ac + bc + bd + da\r\n3) A = b + a + c + d \\geq ab + ac + cd + bd\r\nand summing these gives indeed 3A \\geq 2B.\r\n\r\nPS. Nice solution Lagrangia !", "Solution_3": "This is from a shortlist isn't it? I think I've seen it before.", "Solution_4": "Yeah, I guess \"IS3\" means IMO 1993 Shortlist.", "Solution_5": "Here is my solution:\r\nwe have :\r\n\r\na/(b+2c+3d)+b/(c+2d+3a)+c/(d+2a+3b)+ d/(a+2b+3c)= a 2 /(ab+2ac+3ad) +b 2 /(bc+2bd+3ab)+ c 2 /(cd+2ac+3bc)+ d 2 /(da+2bd+3cd) \\geq (a+b+c+d) 2 /4(ab+bc+cd+da+ac+bd)\r\nSo now we have:\r\n(a+b+c+d) 2 /4(ab+bc+cd+da+ac+bd) \\geq 2/3\r\n3* \\sum a 2 + 6* \\sum ab \\geq 8* \\sum ab\r\n3* \\sum a 2 \\geq 2 *\\sum ab\r\n(a-b) 2 + (b-c) 2 + (c-d) 2 + (d-a) 2 + (a-c) 2 + (b-d) 2 \\geq 0" } { "Tag": [ "inequalities", "inequalities proposed" ], "Problem": "Let $ a_0, a_1, a_2,..., a_n $ be a real numbers such that:\r\n$ (i) 0=a_0 \\le a_1 \\le...\\le a_n$\r\n$(ii) a_i_+_1 - a_i \\le \\frac{1}{n}$ for $ i=0,1,...,n-1 $\r\nProve that:\r\n $ n(a_1^3+...+a_n^3) \\le (a_1+...+a_n)^2 $", "Solution_1": "http://www.mathlinks.ro/Forum/viewtopic.php?t=33412", "Solution_2": "Think's Siuhochung" } { "Tag": [ "algebra", "polynomial", "function", "superior algebra", "superior algebra unsolved" ], "Problem": "Let $ p$ be a prime. Here $ a_p$ is the number of irreducible polynomails of degree $ n$ in $ \\mathbb F_p[X]$.\r\n\r\n$ a_p(n) \\equal{} \\frac{1}{n} \\sum_{d|n}\\mu(d)p^{n/d}.$\r\nhere $ \\mu$ is the mobius function.\r\n\r\nHow can I show that $ a_p(n)$ is positive for all $ n$ and also when $ p\\geq 2$ is not a prime number.", "Solution_1": "It's a multiplicative function, and you can easily check that it's not zero when $ n$ is a prime power.\r\nSee [url=http://www.artofproblemsolving.com/Forum/viewtopic.php?&t=30473]here[/url].", "Solution_2": "but my question is different. I want to show that this is positive? how can I do that?", "Solution_3": "Well, you can just construct an irreducible polynomial in every field. \r\n[quote=\"peteryellow\"]Let $ p$ be a prime. How can I show that is positive for all and also when $ p$ is not a prime number.[/quote]\r\nDon't use same letters in different meanings. Often denotes $ \\mathbb{F}_p[X]$ the ring of polynomials over a field. In this case $ p$ is a power of prime. There are different rings with $ p$ elements if $ p$ is composite.", "Solution_4": "but can somebody give a hint of how to show it.", "Solution_5": "One way is to show the existence of the field $ \\mathbb{F}_{p^n}$ as the splitting field of $ x^{p^n}\\minus{}x$. Then since the multiplicative group of a field is cyclic, there must be some generator $ a$, so $ a$ is a primitive element, and thus the minimal polynomial of $ a$ over $ \\mathbb{F}_p$ must be an irreducible polynomial of degree $ n$.\r\n\r\nAlternatively, you can show directly that that formula gives a positive number. I clicked jmerry's link, but I'm slightly confused why you can do that, since $ f(k) \\equal{} p^k$ is not a multiplicative function.", "Solution_6": "[quote=\"peteryellow\"]but my question is different. I want to show that this is positive? how can I do that?[/quote]\r\nIf you already know that it counts something, then it cannot be negative; so only 0 must be excluded,", "Solution_7": "[quote=\"calc rulz\"]I clicked jmerry's link, but I'm slightly confused why you can do that, since is not a multiplicative function.[/quote]\r\nYou're right, and I made a mistake the first time around. Two quick fixes:\r\n(1): $ \\sum_{d|n}\\mu(d)q^{n/d}\\ge q^n\\minus{}q^{n\\minus{}1}\\minus{}q^{n\\minus{}2}\\minus{}\\cdots\\minus{}1\\equal{}q^n\\minus{}\\frac{q^n\\minus{}1}{q\\minus{}1}\\ge 1$\r\n(2): If $ D$ is the largest squarefree divisor, the sum is equal to $ \\mu(d)q^{n/D}$ mod $ q^{n/D\\plus{}1}$, and is not zero. Since it counts something, it's positive.", "Solution_8": "how do you get that:\r\n\r\n$ \\sum_{d|n}\\mu(d)q^{n/d}\\ge q^n\\minus{}q^{n\\minus{}1}\\minus{}q^{n\\minus{}2}\\minus{}\\cdots\\minus{}1$", "Solution_9": "$ \\mu(k)\\ge \\minus{}1$ for all $ k$, and subtracting the terms that don't match to divisors makes it smaller than not subtracting them. In fact, we never have equality, and I don't need that last $ \\minus{}1$ term. It comes closest for $ n\\equal{}2$, with a value of $ q^2\\minus{}q$ in that case.", "Solution_10": "can this result be related in any way to the number of irreducible polynomials with coefficients in the set $ {0,1,2,...,9}$ ?", "Solution_11": "No, but it can be related to something close. $ a_p(n)$, for arbitrary $ p$, also counts the number of Lyndon words on $ p$ letters of length $ n$, which when $ p$ is prime can be put in bijection with irreducible polynomials; see the definition and proof [url=http://qchu.wordpress.com/2009/11/03/the-cyclotomic-identity-and-lyndon-words/]here[/url].\r\n\r\nTo prove that $ a_p(n)$ is always positive when $ p \\ge 2$, it suffices to show that Lyndon words of every length exist. But this is clear; if $ a < b$ are two letters in the alphabet, then $ aaa...b$ is always a Lyndon word. (This is actually not enough to prove that irreducible polynomials of every degree exist over $ \\mathbb{F}_p$; the bijection between the two requires the existence of finite extensions of every degree.)", "Solution_12": "[quote=\"t0rajir0u\"]No, but it can be related to something close. $ a_p(n)$, for arbitrary $ p$, also counts the number of Lyndon words on $ p$ letters of length $ n$, which when $ p$ is prime can be put in bijection with irreducible polynomials; see the definition and proof [url=http://qchu.wordpress.com/2009/11/03/the-cyclotomic-identity-and-lyndon-words/]here[/url].\n\nTo prove that $ a_p(n)$ is always positive when $ p \\ge 2$, it suffices to show that Lyndon words of every length exist. But this is clear; if $ a < b$ are two letters in the alphabet, then $ aaa...b$ is always a Lyndon word. (This is actually not enough to prove that irreducible polynomials of every degree exist over $ \\mathbb{F}_p$; the bijection between the two requires the existence of finite extensions of every degree.)[/quote]\r\n\r\nyes but can I compute the number of irreducibles with coeffs in 0..9 using $ a_p(n)$ ?", "Solution_13": "No; at best, you can take $ p$ to be a prime power. The problem is that these are the only values of $ p$ for which there exists a finite field of cardinality $ p$. There are two other interpretations of \"irreducible polynomial with coefficients in $ \\{ 0, ... p\\minus{}1 \\}$\" when $ p$ is not a prime power: irreducible in $ \\mathbb{Z}[x]$ or irreducible in $ \\mathbb{Z}/p\\mathbb{Z}$, and neither of them are counted by this expression." } { "Tag": [ "Gamebot" ], "Problem": "Ellen baked $ 2$ dozen cupcakes of which half contained chocolate, two-thirds contained raisins, one-fourth contained chocolate chips, and one-sixth contained nuts. What is the largest possible number of cupcakes that had none of these ingredients?", "Solution_1": "Since $ \\frac23$ have raisins, $ \\frac23\\times{24}\\equal{}16$ have raisins. Each of the other ingredients can be mixed in with any of these cookies, so the maximum is $ 24\\minus{}16\\equal{}\\boxed{8}$.", "Solution_2": "This is one of those \"trick\" problems that love to show up on math contests. Before you plunge right in and start drawing a four-circle Venn diagram, you need to stop, take a step back, take a deep breath, and then re-read the problem. \n\nI approached this problem in the same way as gaussintraining.\n\nIn the end, we have 16 cupcakes with raisins and maybe or maybe not something else, and therefore 8 cupcakes with none of the ingredients. We don't have anywhere near enough information to fill in a four-circle Venn diagram. For one, it's entirely possible that all 16 cupcakes have raisins paired with at least one other ingredient, and it's also possible that 4 of those cupcakes are raisin only. Don't even try to wander into that quagmire; you'll never be seen again! ;).\n\nTim", "Solution_3": "Tim is right, bust I really think that this shouldn't be a level 19 problem...it even gives 100+ points.", "Solution_4": "[hide]I had a quick way, the largest venn diagram is 16, to make outside portiion the largest, we can put all other venn diagrams inside the largest one, like 4,6,12 all goes inside the 16, that way, the outside will be the largest which has none of these 4 ingredients at all, so 24-16=[b]8![/b][/hide]", "Solution_5": "Was it really necessary to revive a four-year old thread with a solution?", "Solution_6": "[quote=no_name]Was it really necessary to revive a four-year old thread with a solution?[/quote]\n\nYes, it is okay to revive alcumus threads if you are actually posting something relevant to the thread.", "Solution_7": "And it's not 8! It's 8", "Solution_8": "I think he meant that the answer was 8 with an exclamation mark behind it because he was excited or something.", "Solution_9": " :( could someone explain this. I don't get it to well. ", "Solution_10": "[hide=Solution]Because we are trying to find the maximum number of plain cupcakes Ellen could have made, we can assume that she will try and mix fillings whenever possible. Because the largest fraction of cupcakes ($2/3$) contain raisins, if she tries to mix fillings, all of the cupcakes that contain chocolate will contain raisins, too. This also applies to all of the other fillings, so we can ignore those and just focus on the raisins. $2/3$ of the cupcakes contain raisins, so $1/3$ don't. $12 * 1/3= \\boxed{4}$ [/hide]", "Solution_11": "Let me know if that solution is too wordy.", "Solution_12": "[quote=elevate]Tim is right, bust I really think that this shouldn't be a level 19 problem...it even gives 100+ points.[/quote]\n\nit is a level 23 problem", "Solution_13": "[quote=HWGelber]Let me know if that solution is too wordy.[/quote]\n\n(You're probably not going to see this, but..)\n\nYour solution is not wordy, but you said $1$ dozen cupcakes instead of $2$. The answer should be $8$. (But you were on the right track :) )\n\n", "Solution_14": "[quote name=\"jcl-12\" url=\"/community/p15744495\"]\n[quote=elevate]Tim is right, bust I really think that this shouldn't be a level 19 problem...it even gives 100+ points.[/quote]\n\nit is a level 23 problem\n[/quote]\n\nhi, there's no point of responding when they probably haven't been on the site for 8-9 years\n\nalso alcumus was probably different then with different levels and stuff", "Solution_15": "Should I quit math if I got this one wrong after spending an hour on it? Seems like I'm not cut out for this.", "Solution_16": "[quote=arguewithplato]Should I quit math if I got this one wrong after spending an hour on it? Seems like I'm not cut out for this.[/quote]\nYou can't avoid maths. It will crop up wherever you go; when you go to the shops or become a scientists, maths will crop up [b]everywhere[/b] you go. It's in your genetics (literally). \n\ntldr: no.", "Solution_17": "Hi, \nI believe there\u2019s another way to do it, but it doesn\u2019t have anything to do with counting and probability.\nI might be wrong.\n\nLet the total cupcakes be the area of a square with side a, which is a^2\nSo the ones with other ingredients in it be squares or rectangles.\n(You could draw a square to start then cut that square out to get pieces that represent others, like the chocolate is half of that square, to get the nuts first cut the square a half then cut that half into three pieces. But note that all of this is to get the visual pieces of each category not taking pieces out of the square a^2 piece)\nThen laying all the pieces out \n(a x a) - all the cupcakes,\n(a x a/2) - chocolate,\n(a x (2/3)a) - raisins,\n(a/2 x a/2) - chocolate chips, \n(a/3 x a/2) - nuts, \n\nThe question ask what is the largest possible number of cupcakes that have none of these ingredients is the same as how to crunch all the pieces together so that when fit it in the big piece, the a^2 square, it left the largest area. To get that we crunch all the smaller pieces into that biggest piece in all those smaller piece (a x (2/3)a).\nSo the largest possible space left is a^2 - (2/3)a^2 = (1/3)a^2 \nAnd a^2 = 24 so the space is 24/3 = 8\n" } { "Tag": [ "inequalities", "inequalities proposed" ], "Problem": "Let $x,y,z$ be positive reals such that $4xyz = x+y+z+1$. Prove that\r\n\r\n\\[(x+1)(y+1)(z+1) \\geq 8.\\]", "Solution_1": "Note that $x+y+z\\geq 3(xyz)^{1/3}$. Let define $a:=(xyz)^{1/3}$, then by assumption we have\r\n\\[\r\n(a-1)(4a^2+4a+1)=4a^3-3a-1=4xyz-3(xyz)^{1/3}-1\\geq 0\r\n\\]\r\nIt follows that $a^n\\geq a\\geq 1$ for every $n$. Now \r\n\\[\r\n(x+1)(y+1)(z+1)=xyz+(xy+xz+yz)+4xyz\\geq 5xyz+3(xyz)^{2/3}=5a^3+3a^2\\geq 8.\r\n\\]", "Solution_2": "Cool. My (contrived) solution uses the substitution $a=\\frac{1}{2x+1}$, etc so our condition (after some algebra) rewrites as $a+b+c=1$. Our inequality becomes\r\n\\[\\left(1+\\frac{1}{a}\\right)\\left(1+\\frac{1}{b}\\right)\\left(1+\\frac{1}{c}\\right) \\geq 64.\\]\r\nHowever, this follows easily by homogenizing and using AM-GM.", "Solution_3": "[quote=\"ThAzN1\"]Cool. My (contrived) solution uses the substitution $a=\\frac{1}{2x+1}$, etc so our condition (after some algebra) rewrites as $a+b+c=1$. Our inequality becomes\n\\[\\left(1+\\frac{1}{a}\\right)\\left(1+\\frac{1}{b}\\right)\\left(1+\\frac{1}{c}\\right) \\geq 64.\\]\nHowever, this follows easily by homogenizing and using AM-GM.[/quote]\r\nhuygens and amgm works as well.", "Solution_4": "[quote=\"ThAzN1\"]Cool. My (contrived) solution uses the substitution $a=\\frac{1}{2x+1}$, etc so our condition (after some algebra) rewrites as $a+b+c=1$. Our inequality becomes\n\\[\\left(1+\\frac{1}{a}\\right)\\left(1+\\frac{1}{b}\\right)\\left(1+\\frac{1}{c}\\right) \\geq 64.\\]\nHowever, this follows easily by homogenizing and using AM-GM.[/quote]\r\n\r\nHi ThazN1, this problem seem closely related to yours, check if your method also working for this problem\r\n\r\nhttp://www.mathlinks.ro/Forum/topic-41928.html\r\n\r\nLet me know it it is working. Thanks" } { "Tag": [], "Problem": "It is given that $x$ and $y$ are positive integers and $3x^{2}+x=4y^{4}+y$. Show that: $x-y$, $3x+3y+1$ and $4x+4y+1$ are squares of integers.", "Solution_1": "3[(x^2)-(y^2)]+(x+y)=y^2\r\n(x-y)(3(x-y)+6y+1)=y^2\r\nif d=(x-y,3(x-y)+6y+1) if d>1, an p is prime number.\r\np devides d and p devides (x-y) and p devides y^2 so p devides y.\r\nso p devides 3(x-y)+6y and we know p devides d so p devides 3(x-y)+6y+1 so p=1 but it is impossible so d=1 .\r\nwe know [3(x-y)+6y+1)].[(x-y)]=a^2 so (x-y)=m^2.\r\nother questions is same .", "Solution_2": "[quote]3[(x^2)-(y^2)]+(x+y)=y^2 \n[/quote]\r\nIt is $y^{4}$, but not $y^{2}$" } { "Tag": [ "Princeton", "college", "search", "articles", "real analysis", "Duke", "geometric series" ], "Problem": "UP Math prof proves Princeton man wrong...\r\n\r\nhttp://www.manilatimes.net/national/2005/may/05/yehey/top_stories/20050505top4.html\r\n\r\nI'm not too sure what to believe... I know the website looks bogus, but if you search for it online, there are some other pages with the same story.\r\n\r\nWhat do you guys think?", "Solution_1": "*i* coulda told you that!\r\n\r\n-peter ruse\r\n-kamaldeep gandhi\r\n-ricky sharma\r\n-rathanak chuon", "Solution_2": "Dunno...that letter by Wiles sure sounds fake...", "Solution_3": "Listen ... false axioms? Gimme a break. This guy sounds just like Doron Shamdi (perhaps that is misspelled, but you can find his posts in Other Problem Solving Topics yourself) who has decided that mathematics is false. I mean, fine, you don't like the axioms, but that doesn't mean that your own axioms are [i]truer[/i], just that they are an alternative choice. This is a fairly classic example of a crank and you shouldn't worry about it.", "Solution_4": "If it would comfort you that modern mathematics will remain intact despite his antics, I could go through one of his articles (such as the editorial in the Manila Times found [url=http://www.manilatimes.net/national/2004/jun/18/yehey/opinion/20040618opi7.html]here[/url]) and explain why the various things he says are either nonsense or at least don't demonstrate the kind of thing he's trying to show. Or you could just take my word for it :)", "Solution_5": "lol. Everyone go to the link that JOel sent you. There are about 10000000 ways to prove 1 = 0.9999999...\r\n\r\nyou could use the standard let x = 0.999999, then 10x = 9.9999999, etc. \r\n\r\nor you could use a1=0.9, a2=0.09, etc. and use sum of an infinite geometric series and more\r\n\r\nWow, thank you Joel. Thou hast saved us all.", "Solution_6": "[quote=\"MithsApprentice\"]Dunno...that letter by Wiles sure sounds fake...[/quote]\r\n\r\nYes it did....it sounded very \"unprofessional\"", "Solution_7": "eww... this guy needs some common sense!", "Solution_8": "a) that guy is a quack (he's a fairly regular poster on sci.math)\r\n\r\nb) that newspaper should be completely ashamed of itself, but certain ethnic groups tend to take excessive pride in the 'accomplishments' of members of said ethnic group. A publication with higher standards, however, would have laughed that article away.", "Solution_9": "haha he says that average of two numbers must lie right in between them and then states that when you average 0.99999.... and 1 you get 0.99999..... which is a contradiction since it doesn't lie in between\r\n\r\nyet he just proved exactly what he tried to disprove: that 0.99.... =1 since two same numbers averaged together will give you that number itself", "Solution_10": "[quote]The most advanced computer today can only deal with numbers having no more that 11 digits.[/quote]\r\n\r\nSomehow, I doubt this...\r\n\r\nBut anyway, yeah, his \"proof\" of the nonsense of, well, mathematics is crazy. This guy needs to go to an asylum... for math. Why can't he just accept the beauty and logic of it all like the rest of us? :D", "Solution_11": "Actually, blah^3, he was the [i]editor[/i] of the math and science portion of that paper, if I read correctly. Which explains in part why he managed to get so much space from them for his silliness.", "Solution_12": "Well, I guess every paper has its employees that they would like swept under the carpet (although it seems that they don't even know the magnitude of the absurdities there)\r\n\r\nThe thing that I find really ironic is that our Edgar Escultura got his Ph.D the same year, and from the same class, as Walter Rudin, the famed author of those difficult-to-the-point-of-unreadable analysis texts that every undergrad runs into sooner or later.", "Solution_13": "[quote=\"blahblahblah\"]The thing that I find really ironic is that our Edgar Escultura got his Ph.D the same year, and from the same class, as Walter Rudin, the famed author of those difficult-to-the-point-of-unreadable analysis texts that every undergrad runs into sooner or later.[/quote]\r\n\r\nI'm not sure what you mean, but Walter Rudin got his PhD from Duke in 1949. He was a professor at UW-Madison in the 70s, but I don't think he was Escultura's advisor. Escultura's advisor appears to have been Laurence Young - who was a very famous and well respected mathematician. I can imagine he would roll over in his grave if he could see what his former student is doing today.", "Solution_14": "Yeah, somehow the list of Ph.D candidates and their advisors blurred together in my head :| . I know this because I did a google search for escultura's name and Rudin's name jumped out at me on the same webpage, and I must have just mentally associated them as getting their Ph.D's at the same time.", "Solution_15": "I have to admit that the complete article seems to be fake. Remember that sometimes this articles are written for jurnalists that seem not to understand what happened, even when it seems not to be the case. I remember that once I read an article about a girl, 22 in Sweden that had proven the Riemann Hypothesis, and at the end it was that she had earned a price for some research in something related (as many prices that are given to young mathematicians) and the jurnalist had listened about the Clay institute price the same day and mixed the information. It was not too long ago, three years or less. Any way, I would not trust too much such article!", "Solution_16": "these quacks are quack quacks. (yes indeed!) how? they don't know the first thing about good quacking or *where* to start quacking. there's nothing wrong with BS'ing if you do it skillfully, in which case it's not really so much BS anymore, but stuff actually worthy of attention. i mean, there's Godel, who allows us to BS on certain levels. but these superficial ones! it's gotta be more polyphonic than that, oh hotdammit!\r\n\r\n-petee outta the blue", "Solution_17": "The damage has already been done:\r\n\r\n[url=http://houseonahill.net/index.php/blog/wiles-escultura-and-pierre-de-fermats-equation/]houseonahill.net/index.php/blog/wiles-escultura-and-pierre-de-fermats-equation/[/url]\r\n\r\n[quote]No wonder I couldn\u2019t understand algebra.[/quote]\r\n\r\nEdit: The first time I read Wiles' letter, I didn't get that he was being sarcastic. Woops. I wouldn't have guessed him for it. That was harsh: \"Also I\u2019d like to have the address of the guy who let you get a PhD 30 years ago. I\u2019d like to discuss few things with him. . . \"", "Solution_18": "I doubt wiles wrote the letter.", "Solution_19": "So then I was right not to have guessed him for it.", "Solution_20": "i highly doubt that the letter is from Wiles, it is totaly unprofessional..., it just doesnt make sense\r\non the other hand, i would be very happy to see one of the counter examples, that they have proposed... ;)", "Solution_21": "Well...if Wiles's proof is true, and as ameteurs, we certainly have no right to assert that it's faulty, then there are no real counterexamples.", "Solution_22": "My assumption is that it's phony" } { "Tag": [ "number theory unsolved", "number theory" ], "Problem": "find all [b]x,y[/b] in integer number:\r\n [b]xy[/b] divisible by [b]x+y[/b].", "Solution_1": "$x=abc$, $y=ab(b-c)$. Right?", "Solution_2": "How do you think he can answer you if it is an unsolved problem for him? :D :D :D \r\n\r\nPlease post complete solution, otherwise this one will remain here for years before going into the solved section.\r\n\r\nPierre.", "Solution_3": "Let $d=(x,y)$, $x=du$, $y=dv$, then $u+v\\mid duv$, but $(u,u+v)=(v,u+v)=1$, so $u+v\\mid d$, i.e. $d=ab$, $u+v=b$. Denote $c=u$.\r\nWe see $x=abc$, $y=ab(b-c)$.\r\n\r\nThat's all." } { "Tag": [ "AMC", "AIME", "AwesomeMath", "summer program" ], "Problem": "Is a person who scored a 132 on AMC 10 and 7 on AIME (me) this year good enough to go to this camp? I just want a rough idea...", "Solution_1": "i think grade might matter..\r\nanyway you shouldnt have a problem getting in\r\nthey accept way worse people\r\n(except if youre a senior or something)\r\nlast year i only got slightly higher and i got in without testing", "Solution_2": "So AwesomeMath isn't like ridiculously high level? IF I do get in, I'm not sure if I can learn with the rest of the people there.", "Solution_3": "There are classes of varying difficulties. You can always find things more difficult if you ask. I think that the easier classes should be manageable for students who did well at mathcounts." } { "Tag": [ "linear algebra", "matrix", "combinatorics unsolved", "combinatorics" ], "Problem": "There is a n*n matrix filled with 0 and 1.We can do the following operation:\r\nIf there is more than m (m<=n) 1 in a row (or a column), then we can change all the zeros in this row (or column) into 1. \r\nProve that:\r\nIf there is at most (m^2-1) 1s in the matrix at the very beginning then we can never change the matrix into all 1s.\r\n(Here m^2-1 is the best number)", "Solution_1": "Just a first observation: $ m^2\\minus{}1$ is not the\"best number\" (also, I think you mean if there are at least $ m$ ones in a row or column we can change it into all ones). For a simple example, take $ n\\equal{}10$ and $ m\\equal{}2$. Clearly $ m^2\\minus{}1 \\equal{} 3$ ones are not enough, while $ m^2 \\equal{} 4$ are enough if we position them in a $ 2 \\times 2$ subsquare. But if we take the diagonal of the square to be all ones ($ 10$ in all), we can still not make even one move. We may need to rephrase this as $ m^2\\minus{}1$ is the \"best\" in the sense that, for more, [b]there exists[/b] a configuration that allows the square to be filled with ones, but that is trivial, since a $ m \\times m$ subsquare of ones does the job.", "Solution_2": "Dear mavropnevma\r\nWhat you say is right.Here the best means that there exists a condition such that it can be turned into all 1s\r\nFor all number less than m^2-1,one can never turn it into all 1s.\r\nDid you find the solution to this?\r\nI think I got it. :D", "Solution_3": "This is one of those delightful cases when one needs to prove a stronger statement in order the get the required result as a particular case, due to the easier, unrestricted manipulation allowed by the general case. This general context therefore will be \r\n\r\n[color=blue]Given integers $ h \\geq c \\geq 1$, $ w \\geq r \\geq 1$, a rectangular $ h \\times w$ array, containing $ 0 \\leq N \\leq rc$ entries equal to $ 1$, and the rule that anytime a row contains at least $ r$ ones it may be filled with ones, while anytime a column contains at least $ c$ ones it may be filed with ones, then the only case that allows the full array to end up with all entries equal to $ 1$ is $ N \\equal{} rc$ (for some special configurations).[/color]\r\n\r\nDenote by $ x$ the number of those rows containing each at least $ r$ ones, and by $ R$ the set of these entries, while we denote by $ y$ the number of those columns containing each at least $ c$ ones, and by $ C$ the set of these entries. Then $ N \\geq | R \\cup C | \\equal{} | R | \\plus{} | C | \\minus{} | R \\cap C | \\geq xr \\plus{} yc \\minus{} xy$. It is easily seen that the problem now reduces to filling with ones a $ (h' \\equal{} h\\minus{}x) \\times (w' \\equal{} w\\minus{}y)$ array, containing $ 0 \\leq N' \\equal{} N \\minus{} | R \\cup C | \\leq rc \\minus{} (xr \\plus{} yc \\minus{} xy) \\equal{} (r\\minus{}y)(c\\minus{}x) \\equal{} r'c'$ entries equal to $ 1$. When $ x \\equal{} y \\equal{} 0$ there is status-quo, therefore no move can be made, and the array remains unfilled, while otherwise the problem reduces in steps to the base case of $ N\\equal{}0$, which is trivial. \r\n\r\nThe particular case is for $ h\\equal{}w\\equal{}n$ and $ r\\equal{}c\\equal{}m$; when $ 0\\leq N \\leq m^2\\minus{}1$ the array cannot therefore be filled up, while when $ N\\equal{} m^2$ the array may be possibly filled up, for example when the entries equal to $ 1$ make up a $ m \\times m$ subarray (but in other cases also, although not for all configurations).", "Solution_4": "Dear mavropnevma\r\nThat's exactly the way how I did that.\r\nIt's really interesting,isn't it?" } { "Tag": [ "calculus", "integration", "trigonometry", "function", "real analysis", "real analysis unsolved" ], "Problem": "I know that there are several proofs for $\\sum_{n\\in\\mathbb{N}}n^{-2}=\\pi^{2}/6$. I was trying to prove something, but I'm not sure if I've made an error here:\r\n\r\nLet $f\\left(x\\right)=x^{2}-x+1/6$ for $0\\leq x\\leq 1$ and require that $f\\left(x\\right)=f\\left(x-n\\right)$ for $n\\leq x\\leq n+1$ where $n\\in\\mathbb{Z}$. Part integration shows us that\r\n\r\n\\[\\int_{0}^{1}f\\left(x\\right)e^{-2\\pi inx}\\,dx=\\begin{cases}1/2\\pi^{2}n^{2}& n\\neq 0 \\\\ 0 & n=0\\end{cases}.\\]\r\nThe next step is to show that\r\n\r\n\\[f\\left(x\\right)=\\frac{1}{\\pi^{2}}\\sum_{n\\in\\mathbb{N}}\\frac{\\cos\\left(2\\pi nx\\right)}{n^{2}},\\]\r\nwhich clearly implies that $\\sum_{n\\in\\mathbb{N}}n^{-2}=\\pi^{2}/6$. I know that since $f$ is a continuous function, we can say that\r\n\r\n\\[f\\left(x\\right)=\\sum_{n\\in\\mathbb{Z}}a_{n}e^{-2\\pi inx},\\]\r\nwhere $a_{n}=1/2\\pi^{2}n^{2}$. However, what is the next step?", "Solution_1": "[quote=\"amcavoy\"]I know that there are several proofs for $\\sum_{n\\in\\mathbb{N}}n^{-2}=\\pi^{2}/6$. I was trying to prove something, but I'm not sure if I've made an error here:\n[/quote]\r\n\r\nWhat about this?\r\n\r\nDefine $f(x)=x^{2}$ on $[-\\pi , \\pi]$.\r\nDefine $f(x+2\\pi n)=x^{2}$ for all $n\\in \\mathbb{Z}$.\r\n\r\nThis function satisfies the \"Dirichlet conditions\".\r\n\r\nExpand it as a Half-Range Cosine Series.\r\n\r\nPlay around with it until you bring it to the desired form.", "Solution_2": "Oh yeah:\r\n\\[f\\left(x\\right)=\\sum_{n\\in\\mathbb{Z}}a_{n}e^{2\\pi inx}=\\frac{1}{\\pi^{2}}\\sum_{n\\in\\mathbb{Z}}\\frac{e^{2\\pi inx}}{2n^{2}}=\\frac{1}{\\pi^{2}}\\sum_{n\\in\\mathbb{N}}\\frac{\\cos\\left(2\\pi nx\\right)}{n^{2}}, \\]\r\nso $f\\left(0\\right)=1/6=\\left(1/\\pi^{2}\\right)\\sum_{n\\in\\mathbb{Z}}n^{-2}$." } { "Tag": [ "function", "real analysis", "real analysis unsolved" ], "Problem": "If $ f(x) \\in C(\\minus{}\\infty,\\infty)$\r\n\r\ncan we prove that there exists a interval $ (a,b) \\subset C(\\minus{}\\infty,\\infty)$ ,such that\r\n\r\n$ f(x)$ is monotony as $ x \\in (a,b)$\r\n\r\n :idea: :roll:", "Solution_1": "If $ \\ f(x)$ happens to be an open map, we can say that the function is monotonic.", "Solution_2": "No, we can not, because this is not true (just take continuos function which is nowhere differentiable) :wink:" } { "Tag": [ "inequalities unsolved", "inequalities" ], "Problem": "Let $ a,b,c\\geq 0$ and $ ab\\plus{}bc\\plus{}ca\\equal{}1$.prove that $ \\sum \\frac{ab\\plus{}1}{a\\plus{}b} \\geq 3$", "Solution_1": "[quote=\"euler_vn\"]Let $ a,b,c\\geq 0$ and $ ab \\plus{} bc \\plus{} ca \\equal{} 1$.prove that $ \\sum \\frac {ab \\plus{} 1}{a \\plus{} b} \\geq 3$[/quote]\r\n\r\nVery nice :P \r\n$ < \\equal{} > p^2 \\plus{} pr \\plus{} 2 \\plus{} 3r \\ge 3p$\r\nWith:$ p \\equal{} a \\plus{} b \\plus{} c;q \\equal{} ab \\plus{} bc \\plus{} ac \\equal{} 1;r \\equal{} abc$\r\n$ \\plus{} ) p \\ge 2$\r\n$ \\equal{} > p^2 \\plus{} pr \\plus{} 2 \\plus{} 3r \\equal{} (p \\minus{} 1)(p \\minus{} 2) \\plus{} pr \\plus{} 3r \\plus{} 3p \\ge 3p$ \r\n$ \\plus{} ) p \\le 2$\r\n$ \\equal{} > r \\ge \\frac {p(4 \\minus{} p^2)}{9}$ by shur :)\r\n$ \\equal{} > p^2 \\plus{} pr \\plus{} 2 \\plus{} 3r \\ge p^2 \\plus{} \\frac {p^2(4 \\minus{} p^2)}{9} \\plus{} 2 \\plus{} \\frac {3p(4 \\minus{} p^2)}{9} \\ge 3p$\r\nWe have prove that:\r\n$ p^2 \\plus{} \\frac {p^2(4 \\minus{} p^2)}{9} \\plus{} 2 \\plus{} \\frac {3p(4 \\minus{} p^2)}{9} \\ge 3p$\r\n$ < \\equal{} > (p \\minus{} 2)(3p \\minus{} p^3 \\minus{} 5p^2 \\minus{} 9) \\ge 0$-It's true by $ p \\le 2$\r\n :P", "Solution_2": "[quote=\"caubetoanhoc94\"][quote=\"euler_vn\"]Let $ a,b,c\\geq 0$ and $ ab \\plus{} bc \\plus{} ca \\equal{} 1$.prove that $ \\sum \\frac {ab \\plus{} 1}{a \\plus{} b} \\geq 3$[/quote]\n\nVery nice :P \n$ < \\equal{} > p^2 \\plus{} pr \\plus{} 2 \\plus{} 3r \\ge 3p$\nWith:$ p \\equal{} a \\plus{} b \\plus{} c;q \\equal{} ab \\plus{} bc \\plus{} ac \\equal{} 1;r \\equal{} abc$\n$ \\plus{} ) p \\ge 2$\n$ \\equal{} > p^2 \\plus{} pr \\plus{} 2 \\plus{} 3r \\equal{} (p \\minus{} 1)(p \\minus{} 2) \\plus{} pr \\plus{} 3r \\plus{} 3p \\ge 3p$ \n$ \\plus{} ) p \\le 2$\n$ \\equal{} > r \\ge \\frac {p(4 \\minus{} p^2)}{9}$\n$ \\equal{} > p^2 \\plus{} pr \\plus{} 2 \\plus{} 3r \\ge p^2 \\plus{} \\frac {p^2(4 \\minus{} p^2)}{9} \\plus{} 2 \\plus{} \\frac {3p(4 \\minus{} p^2)}{9} \\ge 3p$\nWe have prove that:\n$ p^2 \\plus{} \\frac {p^2(4 \\minus{} p^2)}{9} \\plus{} 2 \\plus{} \\frac {3p(4 \\minus{} p^2)}{9} \\ge 3p$\n$ < \\equal{} > (p \\minus{} 2)(3p \\minus{} p^3 \\minus{} 5p^2 \\minus{} 9) \\ge 0$-It's true by $ p \\le 2$\n :P[/quote]\r\nwhy $ p\\leq 2$ :?:", "Solution_3": "[quote=\"peine\"][quote=\"caubetoanhoc94\"][quote=\"euler_vn\"]Let $ a,b,c\\geq 0$ and $ ab \\plus{} bc \\plus{} ca \\equal{} 1$.prove that $ \\sum \\frac {ab \\plus{} 1}{a \\plus{} b} \\geq 3$[/quote]\n\nVery nice :P \n$ < \\equal{} > p^2 \\plus{} pr \\plus{} 2 \\plus{} 3r \\ge 3p$\nWith:$ p \\equal{} a \\plus{} b \\plus{} c;q \\equal{} ab \\plus{} bc \\plus{} ac \\equal{} 1;r \\equal{} abc$\n$ \\plus{} ) p \\ge 2$\n$ \\equal{} > p^2 \\plus{} pr \\plus{} 2 \\plus{} 3r \\equal{} (p \\minus{} 1)(p \\minus{} 2) \\plus{} pr \\plus{} 3r \\plus{} 3p \\ge 3p$ \n$ \\plus{} ) p \\le 2$\n$ \\equal{} > r \\ge \\frac {p(4 \\minus{} p^2)}{9}$\n$ \\equal{} > p^2 \\plus{} pr \\plus{} 2 \\plus{} 3r \\ge p^2 \\plus{} \\frac {p^2(4 \\minus{} p^2)}{9} \\plus{} 2 \\plus{} \\frac {3p(4 \\minus{} p^2)}{9} \\ge 3p$\nWe have prove that:\n$ p^2 \\plus{} \\frac {p^2(4 \\minus{} p^2)}{9} \\plus{} 2 \\plus{} \\frac {3p(4 \\minus{} p^2)}{9} \\ge 3p$\n$ < \\equal{} > (p \\minus{} 2)(3p \\minus{} p^3 \\minus{} 5p^2 \\minus{} 9) \\ge 0$-It's true by $ p \\le 2$\nwhy $ p\\leq 2$ :?: \n :P[/quote][/quote]\r\n\r\nI Consider two cases $ p \\le 2;p \\ge 2$ :lol:", "Solution_4": "[quote=\"caubetoanhoc94\"][/quote][quote=\"peine\"][quote=\"caubetoanhoc94\"][quote=\"euler_vn\"]Let $ a,b,c\\geq 0$ and $ ab + bc + ca = 1$.prove that $ \\sum \\frac {ab + 1}{a + b} \\geq 3$[/quote]\n\nVery nice :P \n$ < = > p^2 + pr + 2 + 3r \\ge 3p$\nWith:$ p = a + b + c;q = ab + bc + ac = 1;r = abc$\n$ + ) p \\ge 2$\n$ = > p^2 + pr + 2 + 3r = (p - 1)(p - 2) + pr + 3r + 3p \\ge 3p$ \n$ + ) p \\le 2$\n$ = > r \\ge \\frac {p(4 - p^2)}{9}$\n$ = > p^2 + pr + 2 + 3r \\ge p^2 + \\frac {p^2(4 - p^2)}{9} + 2 + \\frac {3p(4 - p^2)}{9} \\ge 3p$\nWe have prove that:\n$ p^2 + \\frac {p^2(4 - p^2)}{9} + 2 + \\frac {3p(4 - p^2)}{9} \\ge 3p$\n$ < = > (p - 2)(3p - p^3 - 5p^2 - 9) \\ge 0$-It's true by $ p \\le 2$\nwhy $ p\\leq 2$ :?: \n :P[/quote][/quote][quote=\"caubetoanhoc94\"]\n\nI Consider two cases $ p \\le 2;p \\ge 2$ :lol:[/quote]\r\nokay, I didn't see that you considered two case, now I understand, thank's.", "Solution_5": "[quote=\"peine\"][/quote][quote=\"caubetoanhoc94\"][/quote][quote=\"peine\"][quote=\"caubetoanhoc94\"][quote=\"euler_vn\"]Let $ a,b,c\\geq 0$ and $ ab + bc + ca = 1$.prove that $ \\sum \\frac {ab + 1}{a + b} \\geq 3$[/quote]\n\nVery nice :P \n$ < = > p^2 + pr + 2 + 3r \\ge 3p$\nWith:$ p = a + b + c;q = ab + bc + ac = 1;r = abc$\n$ + ) p \\ge 2$\n$ = > p^2 + pr + 2 + 3r = (p - 1)(p - 2) + pr + 3r + 3p \\ge 3p$ \n$ + ) p \\le 2$\n$ = > r \\ge \\frac {p(4 - p^2)}{9}$\n$ = > p^2 + pr + 2 + 3r \\ge p^2 + \\frac {p^2(4 - p^2)}{9} + 2 + \\frac {3p(4 - p^2)}{9} \\ge 3p$\nWe have prove that:\n$ p^2 + \\frac {p^2(4 - p^2)}{9} + 2 + \\frac {3p(4 - p^2)}{9} \\ge 3p$\n$ < = > (p - 2)(3p - p^3 - 5p^2 - 9) \\ge 0$-It's true by $ p \\le 2$\nwhy $ p\\leq 2$ :?: \n :P[/quote][/quote][quote=\"caubetoanhoc94\"]\n\nI Consider two cases $ p \\le 2;p \\ge 2$ :lol:[/quote][quote=\"peine\"]\nokay, I didn't see that you considered two case, now I understand, thank's.[/quote]\r\n\r\nOkay.\r\nP/s:H\u1ecfi m\u00e3i m\u1edbi bik x\u00e9t 2 TH ti\u1ebfng anh l\u00e0 th\u1ebf :(" } { "Tag": [ "modular arithmetic", "number theory proposed", "number theory" ], "Problem": "Let $p$ be an odd prime. Assume $\\sigma$ is a permutation of $\\{1,2,\\ldots,p\\}$. Show that we can find $i\\ne j\\in\\{1,2,\\ldots,p\\}$ s.t. $i\\sigma(i)\\equiv j\\sigma(j)\\pmod p$.\r\n\r\nP.S. People will probably find this a joke :)", "Solution_1": "Oh, really :lol: :lol: :lol: \r\nSuppose the contrary. It is clear $\\sigma(p)=p$. Then $\\sigma(1),2\\sigma(2),...,(p-1)\\sigma(p-1)$ is a permutation of $1,2,...,p-1$. Multyplying all number $i\\sigma(i)$ we obtain $(p-1)!^2\\equiv (p-1)!\\pmod{p}$. Contradiction with Wilson'n theorem.", "Solution_2": "First assume $\\sigma (p) =0$, because otherwise its trivial.\r\nNow assume that all $ i\\cdot \\sigma (i), i=1,...,p-1$ are different.\r\nThen you get $ \\prod i\\cdot \\sigma (i) \\equiv -1 \\not\\equiv 1 \\equiv \\prod i \\cdot \\prod \\sigma (i) \\mod p$\r\n\r\nnice joke :lol:" } { "Tag": [ "blogs", "MATHCOUNTS", "function", "algebra", "polynomial", "AMC", "AIME" ], "Problem": "I am interested in collecting a small but fun list of topics for write about in my blog. In particular, I would like to hear about topics that middle school math teamers typically say \"wow!\" about upon first discover. What would students here suggest?", "Solution_1": "Maybe this is a little advanced, but a lot of people in my class were really suprised when they found out that some infinities are bigger than others and the teacher proved it.\r\n\r\n[hide=\"Also Interesting\"]There was a problem in my class: \\[x^{x^{x^{\\cdots}}}=2\\]\n\nWho's answer is: \\[\\sqrt{2}\\]\n\nThat people found interesting.\n\nHowever, they found this:\n\nWhat is x if \\[x^{x^{x^{\\cdots}}}=y\\]\n\neven more interesting because it's answer is \\[^y\\sqrt{y}\\] because then \\[\\sqrt{2}^{\\sqrt{2}^{\\cdots}}=2=4\\] which obviously isn't true because \\[2\\ne 4\\]\n\nWhat happened after that if that most people claimed that math as we know it was false, and as such, we should not learn math in school. :D \n\nAnd then the teacher explained how infinity can't be treated like a normal number, so we hadn't \"disproved\" math.[/hide]\r\n\r\nWhat I'm getting at is that proofs of things that aren't true are always interesting.", "Solution_2": "pi formulae are always awesome", "Solution_3": "I like RC-7th 's Idea....\r\n\r\nInfinity can be tough, but I think it would be fun to learn it...", "Solution_4": "While a perfectly good subject to blog about, I think that kind of self-similarity and infinity discussion is more advanced that what I'm asking about.\r\n\r\nPicture that MATHCOUNTS student who can score around half the points at the Chapter competition. That's the audience I'm interested in.", "Solution_5": "Well, according to Treething, you have to solve infinity problems in MATHCOUNTS...\r\n\r\nI looked at some MATHCOUNTS practice problems and they definitley had infinity problems..\r\n\r\nlike I expected..\r\n\r\nand in most schools, you don't get to learn about infinities..", "Solution_6": "I would love to learn about infinities, i think it's hard to get around the fact that it's really going on FOREVER... most middle schools don't teach much about it because it's kinda hard except for like MATHCOUNT people... so I think it would be great. But I don't think you should start off with problems like the ones RC-7th mentioned, they are a little to hard to start off with", "Solution_7": "I think people would love stuff like how pi goes on forever, how to prove anything to the 0th power is 1, and how 0.999999 = 1. We've always like to ask these questions in math class, but the teachers usually don't know. I was amazed when I found out how to prove x^0 = 1... when I told my classmates, they thought that was really cool", "Solution_8": "[quote=\"LuCy4EvA\"]I think people would love stuff like how pi goes on forever, how to prove anything to the 0th power is 1, and how 0.999999 = 1. We've always like to ask these questions in math class, but the teachers usually don't know. I was amazed when I found out how to prove x^0 = 1... when I told my classmates, they thought that was really cool[/quote]\r\n\r\n$0^0\\ne 1$\r\n\r\nThe problem I posted was the only one the teacher had given to us that related to infinity.\r\n\r\nGame theory is pretty fun. There was a question from mathcounts that went like this : Two people are playing a game with 40 counters. On each turn, a person may take 1,2,3,4, or 5 counters. The person who takes the last counter wins. How many counters should the person who goes first take to ensure that they win?", "Solution_9": "is 0^0 undefined?", "Solution_10": "okay first of all\r\n\r\n0^0\r\n\r\nmeans nothing to the nothing power\r\n\r\ndo you understand the concept X^0 = 1 ?\r\n\r\nbut it only happens when X is like numbers not including 0\r\n\r\n0 means nothing..\r\n\r\nthere won't be nothing to power over...\r\n\r\nbut when you set a group like this, {0}\r\n\r\nyou always have to count 0 as one of the elements...\r\n\r\ngod!! this is hard to tell..", "Solution_11": "wait, I get ur explanation, but is it 1 or undefined?", "Solution_12": "$0^0$ is undefined.\r\n\r\nFor most numbers you can product chains of exponents using multiplication and go backwards in these chains using division:\r\n\\[ \\frac{2^3}{2} = \\frac{8}{2} = 4 = 2^2. \\]\r\n\\[ \\frac{2^2}{2} = \\frac{4}{2} = 2 = 2^1. \\]\r\n\\[ \\frac{2^1}{2} = \\frac{2}{2} = 1 = 2^0. \\]\r\n\\[ \\frac{2^0}{2} = \\frac{1}{2} = \\frac{1}{2} = 2^{-1}. \\]\r\n\\[ \\frac{2^{-1}}{2} = \\frac{\\frac{1}{2}}{2} = \\frac{1}{4} = 2^{-2}. \\]\r\n\r\nDivision by 0 is undefined, so this reversal of the multiplication chain is not possible.", "Solution_13": "I remember one of the things that first wowed me was combinatorics, especially counting arguments and (simple) generating functions. Like seeing combinatoric identities proved in three different ways: 1) just using normal algebra, 2) block-walking on Pascal's triangle, and 3) counting arguments. Finding out that those \"How many ways can you make 83 cents using pennies, nickels, and dimes\" questions could actually be solved through a real method (!) was also neat. Not sure if that stuff is too advanced, though.", "Solution_14": "[quote=\"kidnappedturtle\"]I remember one of the things that first wowed me was combinatorics, especially counting arguments and (simple) generating functions. Like seeing combinatoric identities proved in three different ways: 1) just using normal algebra, 2) block-walking on Pascal's triangle, and 3) counting arguments. Finding out that those \"How many ways can you make 83 cents using pennies, nickels, and dimes\" questions could actually be solved through a real method (!) was also neat. Not sure if that stuff is too advanced, though.[/quote]\r\n\r\nThis are topics I would call \"Getting Started\" topics. A little beyond the basics. Generating functions I would call Intermediate -- even at the simple level -- because they involve using polynomials together with counting arguments. Students need to know two subjects fairly well and be able to go between then easily -- the kinds of skills that are required for AIME problems. Those are certainly topics I might blog about however. Thanks for the suggestions.", "Solution_15": "Things that blew my hair back:\r\n\r\n1.) Proof by contradiction.\r\n2.) Euclid's proof for infinitely many primes.\r\n3.) Fundamental theorem of arithmetic.( Every integer factors as a product of primes)\r\n4.) Parity arguments.(The checkerboard tiling one comes to mind)", "Solution_16": "Things that blow my hair back:\r\n1) hair dryer\r\n2) riding in convertible\r\n3) roller coaster\r\n\r\nSeriously (and on topic)\r\nI went to a middle school that didn't go very far into much of anything so I was ecstatic when we learned graphing. While the class was struggling with lines, I graphed my first *gasp* parabola! It amazes me that 4 years ago, I couldn't find slope. :what?: \r\n\r\nBut since we're talking about MathCounts, I really liked counting and geometry problems. I had a teacher who showed us how to derive the area formula for a circle, essentially doing some integral calculus for 7th graders. :wacko: He wasn't using anything we didn't understand, though. He just introduced the idea of making the pieces of the circle small enough and sticking them back together so that it would look enough like a rectangle to use base x height.\r\nSo I guess I'm really talking about limits and approximations, and proofs to some extent.", "Solution_17": "Proofs of irrationality are some of my favroites.", "Solution_18": "I did a bunch of parity argument proofs with my middle school students, and the ones who I imagine could get about 1/4 or more of the points on a MATHCOUNTS Chapter competition really seemed to enjoy them. My favorite is one about whether or not it is possible to create a magic square with the first n2 primes.\r\n\r\nThere are also things in the MATHCOUNTS handbook that could use expanding upon. Kids are often given formulas from there without understanding why they work, like the formula for the sum of all the factors of a number.\r\n\r\nMy favorite math proof of all time was the proof that there is a countable infinity of rational numbers (Cantor's diagonal argument). I was much older than middle school when I first encountered it, but I think that it could be presented in a way that the MATHCOUNTS kids could understand.\r\n\r\nWhen I was in middle school myself, I was really hung up on 00. I would have loved to see a good explanation of why it is neither 0 nor 1, both of which seemed like perfectly reasonable values. Similarly, 0!=1 intrigues my middle schoolers, and telling them that it just is that way because it makes all the formulas work out properly isn't very satisfactory to them :)", "Solution_19": "By the way, I should mention that many mathematicians, including myself, define $0^0$ to be 1. My arguments, and counterarguments by Simon and others, appeared in a topic on a [url=http://www.artofproblemsolving.com/Forum/viewtopic.php?t=11999]Limit Problem[/url].\r\n\r\nBasically, some mathematicians say it is undefined, and some set it to 1. It's a matter of what is the most convenient definition.", "Solution_20": "but 0 means none... :(", "Solution_21": "Many kids are exposed to Magic Squares in elementary school. But Middle School is when you can begin talking about nested patterns and recursion and algorithms. Attached is a 12x12 Magic Square as an example. Ask the students to find smaller magic squares hidden in this big magic square. Either show the kids the pattern I used to make this square or challenge them to find the pattern for themselves. In either case, ask [b]why[/b] does it work? Invite them to create their own pattern that will create a 9x9, 12x12, or 15x15 square. There are [b]many[/b] different patterns / algorithms to make these larger squares.\r\n$\\begin{tabular}{|c|c|c|c|c|c|c|c|c|c|c|c|}\r\n\\hline 113 & 1 & 81 & 127 & 15 & 95 & 126 & 14 & 94 & 116 & 4 & 84 \\\\\r\n\\hline 33 & 65 & 97 & 47 & 79 & 111 & 46 & 78 & 110 & 36 & 68 & 100 \\\\\r\n\\hline 49 & 129 & 17 & 63 & 143 & 31 & 62 & 142 & 30 & 52 & 132 & 20 \\\\\r\n\\hline 124 & 12 & 92 & 118 & 6 & 86 & 119 & 7 & 87 & 121 & 9 & 89 \\\\\r\n\\hline 44 & 76 & 108 & 38 & 70 & 102 & 39 & 71 & 103 & 41 & 73 & 105 \\\\ \r\n\\hline 60 & 140 & 28 & 54 & 134 & 22 & 55 & 135 & 23 & 57 & 137 & 25 \\\\\r\n\\hline 120 & 8 & 88 & 122 & 10 & 90 & 123 & 11 & 91 & 117 & 5 & 85 \\\\\r\n\\hline 40 & 72 & 104 & 42 & 74 & 106 & 43 & 75 & 107 & 37 & 69 & 101 \\\\\r\n\\hline 56 & 136 & 24 & 58 & 138 & 26 & 59 & 139 & 27 & 53 & 133 & 21 \\\\\r\n\\hline 125 & 13 & 93 & 115 & 3 & 83 & 114 & 2 & 82 & 128 & 16 & 96 \\\\\r\n\\hline 45 & 77 & 109 & 35 & 67 & 99 & 34 & 66 & 98 & 48 & 80 & 112 \\\\\r\n\\hline 61 & 141 & 29 & 51 & 131 & 19 & 50 & 130 & 18 & 64 & 144 & 32 \\\\\r\n\\hline\r\n\\end{tabular}$", "Solution_22": "what's the magic square?\r\n\r\nI don't really get it....\r\n\r\nIs it like a matrix or something? :?", "Solution_23": "All the rows, columns and diagonals add up to the same number.", "Solution_24": "to see the different solutions of the same problem was always aweful for me.particularly different solution of the same problem using different field of maths.intutive and easy solution using elemantry language r also helpful.", "Solution_25": "[quote=\"A+MATH\"]Proofs of irrationality are some of my favroites.[/quote]\r\n\r\nWell, whether or not that is middle-school level depends. The proof that $\\sqrt{2}$ is irrational is very clever and uses very simple math. Many middle schoolers could understand that.\r\n\r\nA proof that $\\pi$ is irrational would go into one of the advanced/college boards...", "Solution_26": "Things that surprised and intrigued me in my middle school years:\r\n\r\n1) Figuring out how to calculate the number of shortest paths from one corner to another on a grid (already mentioned)\r\n\r\n2) Finding that the sum of factors of a number like 120 is (1+5)(1+3)(1+2+4+8)\r\n\r\n3) Finding the proof that 1^2 + 2^2 + 3^2 + ... + n^2 = n(n+1)(2n+1)/6\r\n\r\n4) Proof of the Pythagorean theorem\r\n\r\n5) A non-calculus proof of the volume of sphere formula (given the area of circle formula)\r\n\r\n6) Discovering the concept of instantaneous slope\r\n\r\nIn general, I usually found it most fascinating when I read about or discovered a neat way to calculate something that would normally be tedious or impossible.\r\n\r\n\r\nI also think it would be great if you could, instead of just showing a proof or idea, walk viewers through to make them discover it as much by themselves as possible. Of course you're probably already doing that.", "Solution_27": "I think that you should host online 24 games. You know that game where you make 24 with four numbers 1-10. That would be lots of fun.", "Solution_28": "[quote=\"jimli\"]I think that you should host online 24 games. You know that game where you make 24 with four numbers 1-10. That would be lots of fun.[/quote]\r\n\r\nHosting games is somewhere down our list of priorities.", "Solution_29": "My FAVORITE is the decimal expansion of $\\frac{1}{n}$, where n is a prime number. Some of them are pure awesome. For example: \r\n\\[\\frac{1}{7}=0.142857. . . . . . . .\\]\r\n\\[\\frac{2}{7}=0.285714. . . . . . . .\\]\r\n\\[\\frac{3}{7}=0.428571. . . . . . . .\\]\r\n\\[. . . . . . . . . . \\]\r\nOther primes such as 13 and 17 have similar characteristics. :)", "Solution_30": "lots and lots of factoring things\r\n\r\n$(x+y)^2 = x^2 + 2xy + y^$\r\n\r\n$(x+y)(x-y) = x^2 - y^2$\r\n\r\nthose sorts of things plus the fact that the coefficients of $(x+y)^n$ is the $n^{{th}}$ row of pascals triangle. I thought that that was really cool. (note that the top row, 1, is row #0.", "Solution_31": "Pascal's triangle and Fibonacci numbers always caught my attention. There are some very basic but cool things you might be able to do with the golden ratio. \r\n\r\nClever factorization is always cool.\r\n\r\nBasic number theory or rearranging of numbers to make multiplication easier (that was pretty cool, you already did that). \r\n\r\nDifferent properties of numbers, perfect numbers, or the dividing by 11 rule, etc. \r\n\r\nPerhaps go over some cool things with palindromes. \r\n\r\nI still don't really get magic squares much :P", "Solution_32": "[quote=\"LuCy4EvA\"]is 0^0 undefined?[/quote]\r\n\r\n0^0=0/0 -->undefined.", "Solution_33": "The most fun thing to do for me was play with a graphing calculator for the first time, after some instructions." } { "Tag": [ "geometry", "algebra", "polynomial", "inequalities", "trigonometry", "vector", "analytic geometry" ], "Problem": "Math Zoom Year-round courses in Southern California for Fall 2008 will start in September. The following are class schedules. For further information, visit [url]http://www.mathzoom.org/mathzoom_yearround.html[/url].\r\n\r\n[b]- Math Challenge II[/b]\r\nIn Math Challenge II, students learn and practice in areas such as algebra and geometry on the high school level, as well as advanced number theory and combinatorics. Topics include polynomials, inequalities, special algebraic techniques, trigonometry, triangles and polygons, collinearity and concurrency, vectors and coordinates, numbers and divisibility, modular arithmetic, residue classes, advanced counting strategies, binomial coefficients, pigeonhole principle, sequence and series, and various other topics and problem solving techniques involved in math contests such as the American Mathematics Competition (AMC) 10, 12 and ARML, and also the beginning AIME level.\r\n[u]MC II schedules:[/u]\r\n [b]Arcadia[/b]: Saturdays starting 9/13, 9am-11am, 41 W Santa Clara St, Arcadia, CA 91007\r\n [b]Irvine[/b]: Fridays starting 9/5, 7:30pm-9:30pm, 4255 E Campus Drive, Suite 290, Irvine, CA 92612\r\n\r\n[b]- Math Challenge III[/b]\r\nThis course is for experienced students who can solve at least a few problems in the AIME contest, and up to the USAMO level. In this course, the students learn more in-depth math concepts and problem-solving strategies that involves more rigorous mathematical writing and a broad range of topics. The topics includes everything covered in MC II with more depth, and additional topics such as quadratic residue, diophantine equations, recurrence, proving inequalities, mathematical induction, functional equations, generating functions, inversion, projective geometry, and rigorous proof-writing practices. Through effective guidance and direction by our experienced teachers, students develop strong problem solving skills that make them perform well in AIME and USAMO.\r\n[u]MC III schedule:[/u]\r\n[b]Arcadia[/b]: Saturdays starting 9/20, 2pm-4pm, 411 E Huntington Dr, Suite 309, Arcadia, CA 91006", "Solution_1": "[b]- Math Challenge I[/b] (grades 6-8)\r\nStudents learn the Math skills at a deeper level with topics in beginning algebra, fundamental geometry, basic number theory concepts and counting strategies, as well as logic and probabilities. The students not only learn practical skills of challenging problem solving that are supplemental to their school curricula, but also develop skills in creative thinking, logical reasoning, oral and written presentation, and team work. This course helps 6th to 8th graders (some advanced 5th graders may attend upon approval) to participate in the American Mathematics Competition (AMC) 8, MathCounts and Math Olympiads for Elementary and Middle Schools MOEMS. Through effective guidance and direction by our experienced teachers, students develop strong problem solving skills that make them perform well in the MathCounts, AMC 8, and MOEMS Contests.\r\n[u]MC I schedules:[/u]\r\n[b]Arcadia:[/b] Saturdays starting 9/13, 11:15am-1:15pm, 41 W Santa Clara St, Arcadia, CA 91007\r\n[b]Diamond Bar:[/b] Saturdays starting 9/13, 6:30pm-8:30pm, 315 S. Diamond Bar Blvd., #C, Diamond Bar, CA 91765\r\n\r\n[b]- Young Math Olympians[/b] (grades 3-5)\r\nThis course is focused on nurturing the interests of young kids from 3rd to 5th grades in challenging Math problem solving. With the effective guidance of our experienced teachers, kids explore the fun of solving interesting and meaningful Math related problems, and develop their logical and critical thinking skills. Some basic concepts covered in the Math Olympiads for Elementary School (MOEMS) are introduced.\r\n[u]YMO schedules:[/u]\r\n[b]Arcadia:[/b] Saturdays starting 9/13, 9:30am-10:30am, 41 W Santa Clara St, Arcadia, CA 91007" } { "Tag": [], "Problem": "Em c\u00f3 1 b\u00e0i to\u00e1n sau : \r\ncho 2 s\u1ed1 N v\u00e0 k, \u0111i\u1ec1u ki\u1ec7n N > k v\u00e0 N kh\u00f4ng chia h\u1ebft cho k.\r\n\r\nt\u1ee9c l\u00e0 N chia k d\u01b0 Q ( hay N = Q mod k )\r\n\r\nL\u00fac \u0111\u1ea7u N s\u1ed1 \u0111ang \u1edf tr\u1ea1ng th\u00e1i A, m\u1ed7i l\u1ea7n ta \u0111\u1ed5i \u0111\u01b0\u1ee3c k s\u1ed1 sang tr\u1ea1ng th\u00e1i B. ng\u01b0\u1ee3c l\u1ea1i ta c\u0169ng th\u1ec3 \u0111\u1ed5i t\u1eeb B v\u1ec1 A, nh\u01b0ng m\u1ed7i l\u1ea7n b\u1eaft bu\u1ed9c ph\u1ea3i \u0111\u1ed5i k ng\u01b0\u1eddi. T\u1ee9c l\u00e0 n\u1ebfu \u0111\u1ed5i i ng\u01b0\u1eddi b\u00ean B v\u1ec1 A th\u00ec t\u01b0\u01a1ng \u0111\u01b0\u01a1ng v\u1edbi \u0111\u1ed5i k - i b\u00ean A v\u1ec1 B. hix ! Em n\u00f3i lung tung qu\u00e1 nh\u1ec9 ! \r\nCMR : n\u1ebfu k ch\u1eb5n v\u00e0 Q l\u1ebb th\u00ec kh\u00f4ng bao h \u0111\u1ed5i h\u1ebft N s\u1ed1 sang tr\u1ea1ng th\u00e1i B \u0111\u01b0\u1ee3c !\r\nEm th\u1eed r\u1ea5t nhi\u1ec1u test v\u00e0 c\u1ea3m th\u1ea5y n\u00f3 \u0111\u00fang, nh\u01b0ng ch\u01b0a t\u00ecm ra 1 c\u00e1ch CM n\u00e0o t\u1ed5ng qu\u00e1t. C\u00e1c anh gi\u00fap em v\u1edbi nh\u00e9 ! \r\n\r\nV\u00ed d\u1ee5 : N\u1ebfu A = 333 v\u00e0 k = 50 th\u00ec kh\u00f4ng c\u00f3 c\u00e1ch.\r\n N\u1ebfu A = 15 v\u00e0 k = 10 th\u00ec kh\u00f4ng c\u00f3 c\u00e1ch...\r\n\r\nN\u1ebfu A = 1000 v\u00e0 k = 35 th\u00ec c\u00f3 c\u00e1ch : \u0111\u1ea7u ti\u00ean ta \u0111\u1ed5i 28 l\u1ea7n \u0111\u01b0\u1ee3c 980 s\u1ed1 sang tr\u1ea1ng th\u00e1i B. C\u00f2n d\u01b0 l\u1ea1i 20 s\u1ed1. L\u1ea7n k\u1ebf ti\u1ebfp ta l\u1ea5y 10 s\u1ed1 b\u00ean B v\u00e0 25 s\u1ed1 b\u00ean A \u0111\u1ed5i l\u1ea1i. Ta \u0111\u01b0\u1ee3c 25 s\u1ed1 + 10 s\u1ed1 v\u1ec1 tr\u1ea1ng th\u00e1i A. L\u1ea7n cu\u1ed1i \u0111\u1ed5i l\u1ea1i 35 s\u1ed1 v\u1ec1 B xong. \r\nN\u1ebfu em ghi c\u00f3 ch\u1ed7 n\u00e0o kh\u00f4ng r\u00f5 r\u00e0ng th\u00ec m\u1ea5y anh c\u1ee9 n\u00f3i \u1ea1 ! Em s\u1ebd s\u1eefa l\u1ea1i. C\u00e1m \u01a1n m\u00e1y anh r\u1ea5t nhi\u1ec1u !", "Solution_1": "C\u00e1c b\u00e0i to\u00e1n trong box Vi\u1ec7t Nam s\u1ebd \u0111\u01b0\u1ee3c post trong box n\u00e0y http://www.mathlinks.ro/Forum/index.php?f=284 . M\u00e0 theo anh kh\u00f4ng n\u00ean th\u1ea3o lu\u1eadn v\u1ec1 To\u00e1n trong box Vi\u1ec7t Nam, c\u00e1i n\u00e0y sang ddth t\u1ed1t h\u01a1n, em n\u00ean post b\u1eb1ng ti\u1ebfng Anh ra b\u00ean ngo\u00e0i.", "Solution_2": "Da em hi\u1ec3u r\u1ed3i ! Do l\u1ea7n \u0111\u1ea7u em ch\u01b0a bi\u1ebft n\u00ean post nh\u1ea7m ch\u1ed7 ! Srry anh nh\u00e9 :oops:" } { "Tag": [ "function", "number theory unsolved", "number theory" ], "Problem": "What is the number of partitions of a positive integer in $ k$ integers, such that there aren't two equal numbers in the partition. \r\n\r\nExample. n=7, k=3\r\nI need only 1+2+4.", "Solution_1": "If we ignore the condition(to be not equal each other), it would be a well-known open problem. However, your question may have a solution. Try to use \"generating functions\".", "Solution_2": "I forgot how to manipulate the generating functions regarding the partitions :blush:. I think it's very difficult to find a closed form for the partitions I need.", "Solution_3": "Your question is equivalent to the following question \"What is the coefficient of [img]http://www.artofproblemsolving.com/Forum/latexrender/pictures/a/2/9/a297bb91c9703af1975402dded3ab9b7e6431dde.gif[/img] in [img]http://www.artofproblemsolving.com/Forum/latexrender/pictures/6/f/c/6fc57c3597157b964300706a9f5848b93f22e8c8.gif[/img] ?\"", "Solution_4": "Moreover, you can generalize the problem allowing to use i [img]http://www.artofproblemsolving.com/Forum/latexrender/pictures/1/b/a/1ba9b59bdee92f38c1698c784b67ba70f803331d.gif[/img] times. Then the question becomes \"What is the coefficient of [img]http://www.artofproblemsolving.com/Forum/latexrender/pictures/5/3/8/5381feaab83ed3656546a8cdfac424159d680155.gif[/img] in [img]http://www.artofproblemsolving.com/Forum/latexrender/pictures/9/8/d/98d7dd9bab313baf7c0edde3b6e61c4bbbd35ff1.gif[/img] ?\"", "Solution_5": "I've found on the internet a closed form. It's $ p_{3}(6j\\plus{}k)\\equal{}3j^{2}\\minus{}(3\\minus{}k)j$, for $ k\\in [1,5]$ and $ p_{3}(6j\\plus{}6)\\equal{}3j^{2}\\plus{}3j\\plus{}1$. Nice", "Solution_6": "[quote=\"matehan\"]Your question is equivalent to the following question \"What is the coefficient of $ x^{n}$ in $ (1\\plus{}x)(1\\plus{}x^{2})\\ldots(1\\plus{}x^{n})$ ?\"[/quote]\r\n\r\nThis is wrong.He asked for the number of partitions in exactly $ k$ integer.\r\n\r\nHis question is equivalent,for example,to find the coefficient of $ x^{n}y^{k}$ in \r\n\r\n$ (xy\\plus{}1)(x^{2}y\\plus{}1)\\ldots (x^{n}y\\plus{}1)$", "Solution_7": "Yes, you are right. I didn't see that condition. :blush:" } { "Tag": [ "inequalities proposed", "inequalities" ], "Problem": "Let $a,b,c,d$ be non-negative numbers sucht that $a^2+b^2+c^2+d^2=1$.\r\nProve that\r\n$a^3+b^3+c^3+d^3+abc+bcd+cda+dab \\geq 1$.\r\n\r\n[b]Remark1[/b]. I am interested in a solution without squaring and expanding.\r\n\r\n[b]Remark2[/b]. If $x_1^2+x_2^2+...+x_n^2=1$ and $p=\\frac 6{(n-2)(\\sqrt{n}+1)}$, then\r\n$\\sum x_1^3+p\\sum x_1x_2x_3 \\leq 1$.", "Solution_1": "i think we can use AC method just like in [url]http://www.artofproblemsolving.com/Forum/viewtopic.php?t=90766[/url] what nhat did i.e let \r\n\r\n$f(a,b,c,d)=a^3+b^3+c^3+d^3+abc+abd+acd+bcd$ then\r\n\r\n$f((a+b)/2,(a+b)/2,c,d)-f(a,b,c,d)=\\frac{1}{4}(a-b)^2(c+d-3(a+b))$\r\n$f(0,a+b,c,d)-f(a,b,c,d)=ab(3(a+b)-(c+d))$\r\n\r\nSo $f(a,b,c,d)\\leq Max\\{f((a+b)/2,(a+b)/2,c,d),f(0,a+b,c,d)\\}$\r\n\r\nSo by AC method ,\r\n\r\n$f(a,b,c,d)\\leq Max\\{f(0,0,0,1),f(0,0,1/\\sqrt{2},1/\\sqrt{2}),f(0,1/\\sqrt{3},1/\\sqrt{3},1/\\sqrt{3}),f(1/2,1/2,1/2,1/2)\\}=1$\r\n\r\nequality case when $a=b=c=0,d=1$(and its permutation) or $a=b=c=d=1/2$\r\n\r\nMaybe Vasc want another solution , i guess ?", "Solution_2": "It is wrong, because AC-Theorem is valid if $a+b+c+d=constant$.\r\nWe must have $a^2+b^2+c^2+d^2=(a+b)^2/4+(a+b)^2/4+b^2+c^2$ and $a^2+b^2+c^2+d^2=0^2+(a+b)^2+b^2+c^2$ .\r\nSee:\r\nhttp://www.mathlinks.ro/Forum/viewtopic.php?p=532271#p532271", "Solution_3": "Has anyone got a solution?" } { "Tag": [ "inequalities", "inequalities proposed" ], "Problem": "$a>0,b>0,c>0.$ Prove that:\r\n$\\frac{a^2}{b}+\\frac{b^2}{c}+\\frac{c^2}{a}\\geq\\sqrt[3]{9(a^3+b^3+c^3)}.$", "Solution_1": "See http://www.mathlinks.ro/Forum/viewtopic.php?t=38369 and http://www.mathlinks.ro/Forum/viewtopic.php?t=43403 for an even stronger inequality which is still unsolved.\r\n\r\n Darij", "Solution_2": "Thank you, Darij.", "Solution_3": "I hope, this is unknown:\r\n$a>0,b>0,c>0.$ Prove that $\\frac{a^3}{b^2}+\\frac{b^3}{c^2}+\\frac{c^3}{a^2}\\geq\\sqrt[4]{27(a^4+b^4+c^4)}.$", "Solution_4": "The next is proved.\r\nIf $a>0,b>0,c>0,a^{10}+b^{10}+c^{10}=3,$ then $\\frac{a^3}{b^2}+\\frac{b^3}{c^2}+\\frac{c^3}{a^2}\\geq3.$", "Solution_5": "[quote=\"arqady\"]I hope, this is unknown:\n$a>0,b>0,c>0.$ Prove that $\\frac{a^3}{b^2}+\\frac{b^3}{c^2}+\\frac{c^3}{a^2}\\geq\\sqrt[4]{27(a^4+b^4+c^4)}.$[/quote]\r\nI've found a proof of that one. :)\r\n edit: :( No, sorry. It seems I have not." } { "Tag": [], "Problem": "is there any trick to do this problem:\r\n\r\nwhat is the sum of the digits in (111111111) squared?", "Solution_1": "All numbers of this form (edit:less than or equal to 9 digits) squared gives us a pattern of 123... up to the number of digits and then down to one, e.g. $1111^{2}=1234321$. Thus. $111111111=12345678987654321$. The sum of the digits is $\\frac{9(10)}{2}+\\frac{8(9)}{2}=81$.\r\n\r\nMasoud Zargar", "Solution_2": "Smart method. What if there are more than 9 1's?", "Solution_3": "Then you would just have to carry over. You would have:\r\n\r\n1 2 3 4 5 6 7 8 9 10 9 8 7 6 5 4 3 2 1\r\nwhich turns into\r\n1 2 3 4 5 6 7 8 10 0 9 8 7 6 5 4 3 2 1\r\nwhich turns into\r\n1 2 3 4 5 6 7 9 0 0 9 8 7 6 5 4 3 2 1\r\n\r\nso $1111111111^{2}=1234567900987654321$.", "Solution_4": "Thank You Robin\r\n\r\nThat is a good thing to remember.\r\n\r\nI like that trick.\r\n\r\n111^2=12321" } { "Tag": [ "function", "algebra unsolved", "algebra" ], "Problem": "Let $ f: [0;1] \\rightarrow \\mathbb{R}$ be a function such that:\r\n$ (i)$ $ f(1) \\equal{} 1$,\r\n$ (ii)$ $ f(x) \\ge 0$ for all $ x \\in [0,1]$,\r\n$ (iii)$ if $ x,y$ and $ x \\plus{} y$ all lie in $ [0,1]$, then $ f(x \\plus{} y) \\ge f(x) \\plus{} f(y)$.\r\nProve that $ f(x) \\le 2x$ for all $ x \\in [0;1]$.", "Solution_1": "[quote=\"N.N.Trung\"]Let $ f: [0;1] \\rightarrow \\mathbb{R}$ be a function such that:\n$ (i)$ $ f(1) \\equal{} 1$,\n$ (ii)$ $ f(x) \\ge 0$ for all $ x \\in [0,1]$,\n$ (iii)$ if $ x,y$ and $ x \\plus{} y$ all lie in $ [0,1]$, then $ f(x \\plus{} y) \\ge f(x) \\plus{} f(y)$.\nProve that $ f(x) \\le 2x$ for all $ x \\in [0;1]$.[/quote]\r\n\r\nLet $ a\\in[0,1]$ such that $ f(a)>2a$\r\nIf $ a\\leq\\frac 12$, and using $ f(x \\plus{} y) \\ge f(x) \\plus{} f(y)$, we get $ f(2a)\\geq 2f(a) > 4a\\equal{}2(2a)$. Using this process as much as necessary, we get $ b\\in(\\frac 12,1]$ such that $ f(b)>2b>1$\r\nThen $ f(1)\\equal{}f(b\\plus{}(1\\minus{}b))\\ge f(b)\\plus{}f(1\\minus{}b)>1\\plus{}0$ (remember $ f(x) \\ge 0$ for all $ x \\in [0,1]$) which is a contradiction with $ f(1)\\equal{}1$\r\n\r\nSo no such $ a$ exists and $ f(x)\\leq 2x$ $ \\forall x\\in[0,1]$" } { "Tag": [ "absolute value", "algebra solved", "algebra" ], "Problem": "The absolute value of every number in the sequence $\\{a_n\\}$ is smaller than 2005, and \\[a_{n+6}=a_{n+4}+a_{n+2}-a_n.\\] holds for all positive integers n. Prove that $\\{a_n\\}$ is periodic.\r\n\r\nIncredibly, this was probably the most difficult problem of our independent study problems in the 1st TST (excluding the final exam).", "Solution_1": "so just define two sequences $b, c$ the even and odd terms so that $b_{n+3} = b_{n+2} + b_{n+1} - b_n$, similar one for $c$. Then characteristic equation is $r^3 - r^2 - r + 1 = 0 \\Rightarrow (r-1)(r^2-1) = 0 \\Rightarrow r = -1, 1, 1$ so $b_n = \\alpha + n\\beta + \\gamma (-1)^n$; if $\\beta \\ne 0$, then for $n$ arbitrarily large, the modulus will be unbounded, which cannot happen, so $\\beta = 0$ and $b_n = \\alpha + \\gamma (-1)^n$ which has period 2; a similar analysis takes place for the $c_n$ so $a_n$ has period $4$." } { "Tag": [], "Problem": "Problem 2 (Group II). The glass of a fluorescent lamp is composed mainly by silica (SiO2), alumina (Al2O3), calcium oxide (CaO), magnesium oxide (MgO), and sodium oxide (Na2O).\r\n\r\n2.1 Regarding those compounds, select from the following statements the correct one:\r\n\r\nA - Alumina is the responsible for the formation of the net structure of glass.\r\nB - The ion Na+ is one of the responsibles for the breaking of Si-O bonds in the silica structure.\r\nC - Alumina is added in order to reduce the melting point of glass, facilitating its production.\r\nD - The less the quantity of CaO and Na2O, the smaller will be melting point of glass.\r\n\r\n2.2 Consider that the general formula of the glass used in those lamps production is $ SiO_{2}(Na_{2}O)_{m}(CaO)_{n}$. Supposing now that, in the above formula, m = 0.08 and n = 0.06, compute the minimum mass of sodium sulphate (Na2SO4) necessary for the production of 1.0 kg of that glass.\r\n\r\n$ Na_{2}SO_{4}(s)+heat \\longrightarrow Na_{2}O(s)+SO_{3}(g)$.", "Solution_1": "Let $ M$ be the molar mass of glass..$ r$ is molar mass of $ Na_{2}O$ .$ k$ molar mass of $ Na_{2}SO_{4}$\r\nin 1 mole of glass i.e in $ \\frac{M}{1000}$ Kg we have 0.08 moles $ Na_{2}O$ hence in 1kg we have \r\nhave $ \\frac{80m}{M}$ g of $ Na_{2}O$\r\nin $ k$ g of $ Na_{2}SO_{4}$ we have $ m$ g of $ Na_{2}O$\r\nso minimum mass of $ Na_{2}SO_{4}$ required is $ \\frac{80k}{M}$", "Solution_2": "Your answer is correct. How about question 2.1?" } { "Tag": [ "email", "articles", "USAMTS", "function", "Valentin was here" ], "Problem": "Well, people were talking about me when I first joined, so let's see who will be the next person to hit 20000th user :D", "Solution_1": "Haven't we reached 30000 already?", "Solution_2": "How do we know that you were no 10000? Your profile number is 10153 (roll over username and see the status bar)", "Solution_3": "well here it says he was 9999th...\r\nhttp://www.artofproblemsolving.com/Forum/memberlist.php?mode=joindate&order=ASC&start=9950\r\n\r\nstrange.", "Solution_4": "I'm sorry 10000th user, your name is really cool but as they say, you are just 9999\r\n\r\nthe 10000th user is conway, he joined the same day, and made one post immediately to disappear again\r\n\r\nCan anyone understand mathI, he joined just one day after Creation by Valentin, waited until december [b]2004[/b] to disappear forever then??", "Solution_5": "I somehow think that the $9999$ comes from banning/deleting one user or somewhat like that...", "Solution_6": "Banning? I thought that that was extremely rare on mathlinks?\r\n\r\nHowever one suffices, and that it is sad to take that pride away, :o :( sorry 10000th user\r\n\r\n\r\nI just discovered I'm a pretty early bird, I am user 359 according to that list, I joined in july 2003. :roll:", "Solution_7": "How do you know that math1 has disappeared forever? He may turn up again now. Maybe he makes a post every one and a half year!\r\n\r\nAnd why does it show 78 when I roll over your name?", "Solution_8": "[quote=\"bubka\"]How do you know that math1 has disappeared forever? He may turn up again now. Maybe he makes a post every one and a half year!\n\nAnd why does it show 78 when I roll over your name?[/quote]\r\n\r\n :huh: :?: No idea, I am number 359 according to the member list\r\n\r\nyou show 17356, while you are in fact 17201", "Solution_9": "Possibly one counts the number of registrations, the other one the number of existing users.\r\nOr somewhat like that.\r\n\r\nPS: yes, bans are very rare, but one out of $9999$ in about one year isn't much.", "Solution_10": "Hmm, I heard about a member who got agressive about Iranian-Israeli relations, ultimately ending in a ban, but Valentin accepted his pleas to come back to matlinks. I don't if I got that story completely, but CAN he do that? When you are 'killed', is your existence wiped off of this forum? (your private messages, number of posts, your list of posts)\r\n\r\nAnd now one other thing , if microsoft decides to prey on me, and I lose my hotmail account... are my mathlink days over then. Or will I be able to come back with another email account, but the same username??", "Solution_11": "Well, there is a difference between ban and deletion, the first one makes the forum just unaccessible for you, the second one deletes everything like e.g. PMs (but posts get just \"Guests\" or somewhat like that, not sure). At my last post, I meant more or less deletion instead of ban.\r\nAnd as much as I know, there where two users banned in these Iranian-Israeli relations, only one of them got back (but they both really only got banned I think).\r\nBut I know of at least one user that was really deleted.\r\n\r\nFor the second, you can change your mail-address mathlinks uses as long as you have the MathLinks-password. When you loose both, you get into troubles... (but there should be a way, too)", "Solution_12": "interesting :\r\n\r\nsome questions : \r\n\r\n1) can valentin see our passwords? And our email accounts (that are hidden if you refuse to display them)?\r\n\r\n2) do banned/deleted people get warnings before they are thrown off?\r\n\r\n3) can moderators ban/delete as well or is that decision only up to valentin himself?\r\n\r\n4)can mathlinks do anything against double account people?\r\nI really hate that, people who get into arguments and nobody agrees with them... except one other guy that doesn't say anything else , or banned people who refuse to let the site be (you should the IMO article on wikipedia, it's pathetic on the talk page)", "Solution_13": "Just to clear a bit of the mystery here :P Math1 is a test account that I occasionally use to check stuff up :D", "Solution_14": "[quote=\"fredbel6\"]interesting :\n\nsome questions : \n\n1) can valentin see our passwords? And our email accounts (that are hidden if you refuse to display them)?\n\n2) do banned/deleted people get warnings before they are thrown off?\n\n3) can moderators ban/delete as well or is that decision only up to valentin himself?\n\n4)can mathlinks do anything against double account people?\nI really hate that, people who get into arguments and nobody agrees with them... except one other guy that doesn't say anything else , or banned people who refuse to let the site be (you should the IMO article on wikipedia, it's pathetic on the talk page)[/quote]\r\n\r\nI think he can see your email but not your password, and while the other questions are really for moderators to answer, I would say it depends on the circumstances.", "Solution_15": "/bump......", "Solution_16": "oh my a 14 year old thread\n\nlook at how far aops has come; it now has like 500k useres i think", "Solution_17": "[quote=Potato2017]oh my a 14 year old thread\n\nlook at how far aops has come; it now has like 500k useres i think[/quote]\n\nI think 600k now", "Solution_18": "woah...\n2006", "Solution_19": "[quote=10000th User]Well, people were talking about me when I first joined, so let's see who will be the next person to hit 20000th user :D[/quote]\n\nRias Gremory is the OG of AoPSers! ", "Solution_20": "Currently the newest user is s128190 which is user #687863 (I found him)\n\nhttps://artofproblemsolving.com/community/user/687863", "Solution_21": "[quote=Valentin Vornicu]Haha, I know the secret identity of the user #20000 (with the username #20000) that is and it's not user #10000 :)\n\nAbout admins: no you need not 42 points in the IMO. There is no condition that can be fulfilled that guarantees us needing you in the team or anything like that. There are though conditions that, if not fulfills, automatically disqualify any attempt: being very fluent in English is one of them for example.[/quote]\n\n\nWho-", "Solution_22": "bruh how do u have a space in ur username", "Solution_23": "that's the 2nd user with there being no first", "Solution_24": "prob admin or dev.", "Solution_25": "Special users can do that\nspaces I mean", "Solution_26": "Who will be the 1,000,000th user? I hope they're active. [tip=nothing to see here dotted]they will probably be more orzed than dotted and cents[/tip]", "Solution_27": "[quote=StarshipTigerTheNext]Who will be the 1,000,000th user? I hope they're active. [tip=nothing to see here dotted]they will probably be more orzed than dotted and cents[/tip][/quote]\n\nI see what you did there :-D ", "Solution_28": "How do you even check what user you are", "Solution_29": "Almost all of you in the top posts died anyway." } { "Tag": [ "geometry", "circumcircle", "geometry solved" ], "Problem": "$ABCDEF$ is a cyclic haxegon with $AB=CD=EF$. $G, H, I$ are the midpoints of $BC, DE, AF$. \r\n\r\nIs $\\triangle{GHI}$ an equalateral one? Prove your answer.", "Solution_1": "[quote=\"shobber\"]$ABCDEF$ is a cyclic haxegon with $AB=CD=EF$. $G, H, I$ are the midpoints of $BC, DE, AF$. \n\nIs $\\triangle{GHI}$ an equalateral one?[/quote]\r\n\r\nThis holds only when AB = CD = EF is the radius of the circumcircle of ABCDEF. This fact is known under the name \"asymmetric propeller theorem\". Check out http://www.mathlinks.ro/Forum/viewtopic.php?t=16908 and http://www.mathlinks.ro/Forum/viewtopic.php?t=45874 .\r\n\r\n Darij", "Solution_2": "Thanks darij. :)" } { "Tag": [], "Problem": "If the nth term of a product is $ {(4(2k\\minus{}1)^4)}\\plus{}1$ as numerator and $ {(4(2k)^4)}\\plus{}1$ as denominator find the product up to 6 terms.\r\nThis is a star question", "Solution_1": "i guess using sophie germain identity we will get a telescopic cancellation but with me that got too computational", "Solution_2": "[hide]By the Sophie Germain Identity, we have $ 4x^4\\plus{}1\\equal{}(2x^2\\minus{}2x\\plus{}1)(2x^2\\plus{}2x\\plus{}1)$. Also note that $ 2x^2\\minus{}2x\\plus{}1\\equal{}2(x\\minus{}1)^2\\plus{}2(x\\minus{}1)\\plus{}1$. Hence:\n\\[ \\frac{4\\cdot 1^4\\plus{}1}{4\\cdot 2^4\\plus{}1}\\cdot\\frac{4\\cdot 3^4\\plus{}1}{4\\cdot 4^4\\plus{}1}\\cdots\\frac{4\\cdot 11^4\\plus{}1}{4\\cdot 12^4\\plus{}1}\\equal{}\\frac{2\\cdot 1^2\\minus{}2\\cdot 1\\plus{}1}{2\\cdot 12^2\\plus{}2\\cdot 12\\plus{}1}\\equal{}\\frac 1{313}.\\][/hide]", "Solution_3": "Your solution is a little bit vague", "Solution_4": "I did not feel like typing all the computation out. Let me do this again:\r\n\\begin{align*}\\prod_{k=1}^6 \\frac{4(2k-1)^4+1}{4(2k)^4+1}&= \\frac {4\\cdot 1^4 + 1}{4\\cdot 2^4 + 1}\\cdot\\frac {4\\cdot 3^4 + 1}{4\\cdot 4^4 + 1}\\cdots\\frac {4\\cdot 11^4 + 1}{4\\cdot 12^4 + 1} \\\\\r\n&= \\frac{(2\\cdot 1^2-2\\cdot 1+1)(2\\cdot 1^2+2\\cdot 1+1)}{(2\\cdot 2^2-2\\cdot 2+1)(2\\cdot 2^2+2\\cdot 2+1)}\\cdot \\frac{(2\\cdot 1^3-2\\cdot 3+1)(2\\cdot 1^3+2\\cdot 3+1)}{(2\\cdot 4^2-2\\cdot 4+1)(2\\cdot 4^2+2\\cdot 4+1)}\\\\\r\n&\\cdots\\frac{(2\\cdot 11^2-2\\cdot 1+1)(2\\cdot 11^2+2\\cdot 11+1)}{(2\\cdot 12^2-2\\cdot 12+1)(2\\cdot 12^2+2\\cdot 12+1)}.\\end{align*}\r\nNote that since $ 2n^2-2n+1=2(n-1)^2+2(n-1)+1$ for any $ n$, this product telescopes to leave:\r\n\\begin{align*}\r\n&= \\frac {2\\cdot 1^2 - 2\\cdot 1 + 1}{2\\cdot 12^2 + 2\\cdot 12 + 1} \\\\\r\n&= \\frac 1{313}.\\end{align*}", "Solution_5": "i was plainly lazy to do that calculation" } { "Tag": [ "geometry", "geometric transformation", "rotation", "geometry proposed" ], "Problem": "The plane is covered with network of regular congruent disjoint hexagons. Prove that there cannot exist a square which has its four vertices in the vertices of the hexagons.\r\n\r\n[i]Gabriel Nagy[/i]", "Solution_1": "We can even assume the lattice is triangular: for each hexagon, add a point in its center and connect it to the six vertices of the hexagon. The conclusion holds for this larger lattice as well:\r\n\r\nAssume WLOG that we can find points $ A,B $ belonging to this lattice s.t. if $ O $ is the origin of the plane, $ OB $ is obtained from $ OA $ through a rotation of angle $ \\frac\\pi 2 $ around $ O $. This is equivalent to finding integers $ a,b,c,d $ s.t. $ \\frac{a+b\\varepsilon}{c+d\\varepsilon}=i $, where $ \\varepsilon $ is a solution to $ x^2+x+1=0 $ ($ a+b\\varepsilon $ represents the point $ A $, while $ c+d\\varepsilon $ - the point $ B $). In turn, this implies that we can find rationals $ u,v $ with $ u+v\\varepsilon=i $. This is claerly absurd, since eliminating $ u $ between $ u+v\\varepsilon=i,u+v\\bar\\varepsilon=-i $ gives an irrational value for $ v $." } { "Tag": [ "inequalities", "trigonometry" ], "Problem": "Prove that $ (\\sin x \\plus{}\\cos x)^4 \\leq 4$", "Solution_1": "[quote=\"enndb0x\"]Prove that $ (\\sin x \\plus{} \\cos x)^4 \\leq 4$[/quote]\r\n\r\n$ \\Leftrightarrow (\\sin \\ x \\plus{} \\cos \\ x)^2\\le 2\\Leftrightarrow \\sin^2\\ x \\plus{} \\cos^2 \\ x \\plus{} 2\\sin\\ x\\cos \\ x\\le 2\\Leftrightarrow 2\\sin \\ x\\cos\\ x\\le$ $ 1\\Leftrightarrow \\sin \\ 2x\\le 1$\r\n\r\nbetter for intermediate forum .", "Solution_2": "[quote=\"enndb0x\"]Prove that $ (\\sin x \\plus{} \\cos x)^4 \\leq 4$[/quote]\r\n\r\n$ \\sin x \\plus{} \\cos x \\equal{} \\sqrt{2} \\ \\sin (x\\plus{} \\frac{\\pi}{4})$\r\n\r\n$ ( \\sin x \\plus{} \\cos x)^4 \\equal{} 4 \\ \\sin ^4 (x \\plus{} \\frac{\\pi}{4}) \\leq 4$" } { "Tag": [ "quadratics", "algebra", "polynomial", "LaTeX" ], "Problem": "Just doing some homework and I came across an equation that I can't seem to factorise. The surd has confused me :oops: .\r\n\r\nThe equation:\r\n[b]4x^2 - 12\u221a2x + 18[/b]\r\n\r\nThis is what I have done for the next step but I'm not sure if it's right:\r\n2(2x^2 - 6\u221a2x +9)\r\n\r\nCan anyone please help?", "Solution_1": "[hide=\"hint\"]Let $y=x\\sqrt{2}$. The polynomial looks nicer in terms of $y$.[/hide]", "Solution_2": "So it would be\r\n\r\n2(2x^2 - 6yx +9)?", "Solution_3": "You would need to convert all of the $x$'s to $y$'s in order to make the substitution useful.\r\n\r\nThis is $2(y^2-6y+9)$, which can be factored further, of course.\r\n\r\nAfter factoring completely, rewrite in terms of $x$.", "Solution_4": "so it's \r\n\r\n2(y^2 -6y+9)\r\n[b]2(y-3)^2[/b]\r\n\r\nbut when you expand it again it doesn't work out back to the original equation? because the surd is in the first section too.. sorry i must seem really dumb.\r\n\r\n2(x\u221a3-3)^2\r\n2(x\u221a3-3)(x\u221a3-3)\r\n2(2x^2\u221a3 - 6\u221a3 +9)\r\n4x^2\u221a3- 12\u221a3 +18", "Solution_5": "2(y^2 - 6y + 9)\r\n2(y - 3)^2\r\nexpanding out again..\r\n2(sqrt[2]x - 3)^2\r\n2( (sqrt[2]^2*x^2 - 2*3*sqrt[2]*x + 3^2)\r\n2(2x^2 - 6*sqrt[2]x + 9)\r\nand you're back to the same thing =]", "Solution_6": "Thanks heaps. I get it now.", "Solution_7": "What's a surd? :?", "Solution_8": "square roots that cant be simplified into an integer, i think", "Solution_9": "[quote=\"laurenluvspink\"]Just doing some homework and I came across an equation that I can't seem to factorise. The surd has confused me :oops: .\n\nThe equation:\n[b]4x^2 - 12\u221a2x + 18[/b]\n\nThis is what I have done for the next step but I'm not sure if it's right:\n2(2x^2 - 6\u221a2x +9)\n\nCan anyone please help?[/quote]\r\n\r\n4X^2-12\u221a2X+18=0.\r\n\r\nRoot (Only one): (3/2)\u221a2.\r\n\r\nThen, 4X^2-12\u221a2X+18=4(X-3/2\u221a2)(X-3/2\u221a2).", "Solution_10": "How do you type those square-root thingies without $\\LaTeX$?", "Solution_11": "It's a special character. You can go into a text formatter (MS Word) make one, copy/paste it, or copy/paste his into the text field in which you reply with.", "Solution_12": "[quote=\"JavaMan\"]square roots that cant be simplified into an integer, i think[/quote]\r\n\r\nSpecifically, square roots that are irrational--can't be simplified to rational numbers" } { "Tag": [], "Problem": "I understand long division but synthetic i dont like...so can someone solve this division problem using synthetic division....thanks\r\n\r\n\r\n$x^3-4x^2-19x=9/x+3$\r\n\r\nthanks", "Solution_1": "-3[_1__-4___-19___0___\r\n ______ -3 ___21 __-6\r\n____1 _ -7____2 __ -6\r\n\r\nX^2 - 7X + 2 Remainder -6\r\nSorry for the sloppyness I'm not sure how to use latex. Do you see what I did?\r\n\r\n:starwars:", "Solution_2": "[hide]\nYou use 3 for the divider, but make it -3.\nYou take the coefficents of $x^3-4x^2-19$\nThen, you bring down the one, multiple -3 by the number and put it below the next number and subtract:\n\n\n-3\n 1 -4 -19\n -3 3 \n_____________________\n 1 -1 -22\nSo: $x^2-x-\\frac{22}{x+3}$\nUmmm... I think that's how you do it.\nThe -3 is supposed to be under the -4, and the 3 under the -19.[/hide]" } { "Tag": [ "geometry", "angle bisector", "area of a triangle" ], "Problem": "Prove that it is impossible to have a triangle in which the trisectors of an angle also trisect the opposite side.", "Solution_1": "[hide=\"Hint\"]Begin by showing that if an angle-bisector in a triangle is also the median to the opposite side, it must be perpendicular to that opposite side, and the triangle is isosceles.[/hide]", "Solution_2": "The area of a triangle is $ \\frac {1}{2}absin\\theta$.\r\n\r\nBy trisecting the side, you now have 3 triangles with equal area. Label the sides going from the vertex $ p,q,r,s$ from left to right.\r\n\r\n$ \\frac{1}{2}pq*sin\\theta \\equal{} \\frac{1}{2}qr*sin\\theta \\equal{} \\frac{1}{2}rs*sin\\theta$\r\n\r\nOr $ pq \\equal{} qr \\equal{} rs$ so From the first equality, $ p \\equal{} r$ from the second $ q \\equal{} s$.\r\n\r\nNote that they can't all be equal, since there can be at most 2 segments from a point to a line with equal length.\r\n\r\nNow, it is a simple argument. Ask yourself if $ p > q$ or $ p < q$ can you have the segments in that order? Hint, consider the foot of the altitude from the vertex, and the distances from the bases of the segments to the foot of the altitude." } { "Tag": [ "LaTeX" ], "Problem": "The sum of the first n terms of the series 1^2+2.(2^2)+3^2+2.(4^2)+5^2+2.(6^2)...... is {n(n+1)^2}/2, when n is even.What will be the sum when n is odd?", "Solution_1": "[quote=\"papu24\"]The sum of the first n terms of the series 1^2+2.(2^2)+3^2+2.(4^2)+5^2+2.(6^2)...... is {n(n+1)^2}/2, when n is even.What will be the sum when n is odd?[/quote]\r\n\r\nDo you mean:\r\n\r\n$ 1^2 \\plus{} 2(2^2) \\plus{} 3^2 \\plus{} 2(4^2) \\plus{} 5^2 \\plus{} 2(6^2)\\equal{}\\frac{n(n \\plus{} 1)^2}{2}$\r\n\r\n? Please use $ LaTeX$ in the future.", "Solution_2": "[hide]\n$ 1^2 \\plus{} 2(2^2) \\plus{} 3^2 \\plus{} 2(4^2)...\\plus{} 2(n^2) \\equal{} \\frac {n(n \\plus{} 1)^2}{2}$\n\nWe are attempting to find the value of N+1. We currently have the value of N. Set $ m \\equal{} n \\minus{} 1$, so that we can work with the odd series without being confused\n\n$ 1^2 \\plus{} 2(2^2) \\plus{} 3^2 \\plus{} 2(4^2)...\\plus{} (m^2) \\equal{} \\frac {(m\\minus{}1)(m)^2}{2}\\plus{}m^2$\n\n$ \\equal{} \\frac {(m\\minus{}1)m^2\\plus{}2m^2}{2}$\n\nFactor out m^2\n\n$ \\equal{} \\frac {(m\\minus{}1\\plus{}2)m^2}{2}$\n\n$ \\equal{} \\frac {(m\\plus{}1)m^2}{2}$[/hide]", "Solution_3": "Uh, in your second line, don't you mean set $ m\\equal{}n\\plus{}1$?" } { "Tag": [ "geometry", "circumcircle", "angle bisector", "congruent triangles", "geometry solved" ], "Problem": "Let ABC be a triangle. M is a point in it's plane. The bisectrix of angles BMC, AMC,AMB intersect BC,AC,AB in A1,B1,C1.\r\nShow that:\r\na) AA1 \\cap BB1 \\cap CC1=P\r\nb) PA/PA1*PB/PB1*PC/PC1=8 if and only if M is the circumcenter of ABC", "Solution_1": "Do you mean that M lies inside ABC ? If so, the first part is quite obvious.\r\n\r\nWe have t(a) = CA1/BA1 = CM/BM by the angle bisector theorem. Similarly, t(b) = AB1/CB1 = AM/CM and t(c) = BC1/AC1 = BM/AM. Now t(a)t(b)t(c) = (CM/BM)(AM/CM)(BM/AM) = 1 so AA1, BB1 and CC1 are concurrent at P by Ceva's theorem.\r\n\r\nNow we have to show that PA*PB*PC = 8*PA1*PB1*PC1 if and only if M is the circumcenter. Now PA/PA1 = AB/A1B, PB/PB1 = BC/B1C, PC/PC1 = AC/A1C. So we should prove that AB*BC*CA = 8*A1B*B1C*C1A if and only if M is the circumcenter. The if-part is obvious. If M is indeed the circumcenter, then triangles MAB, MBC, MCA are isosceles so A1, B1, C1 are the midpoints of BC, CA, AB. In that case AB = 2*C1A, BC = 2*A1B, CA = 2*B1C so indeed AB*BC*CA = 8*A1B*B1C*C1A. Now the \"only if\" -part remains.", "Solution_2": "Here's the proof of the \"only-if\" part. So we want to prove that if 8*A1B*B1C*C1A = AB*BC*CA, then M is the circumcenter.\r\nNow note that A1B*B1C*C1A = A1C*C1B*B1A (by Ceva).\r\nSo A1B*B1C*C1A = \\sqrt(A1B*A1C)*\\sqrt(B1C*B1A)*\\sqrt(C1A*C1B).\r\nThen \\sqrt(A1B*A1C) \\leq 1/2(A1B + A1C) = BC/2 with equality only if A1B = A1C, i.e. if A1 is the midpoint of BC.\r\nSimilarly, \\sqrt(B1C*B1A) \\leq AC/2 and \\sqrt(C1A*C1B) \\leq AB/2.\r\nMultiplying, we have\r\n8\\sqrt(A1B*A1C)*\\sqrt(B1C*B1A)*\\sqrt(C1A*C1B) \\leq AB*BC*CA \r\nwith equality only if A1, B1, C1 are the midpoints of BC, CA, AB.\r\nSo if 8\\sqrt(A1B*A1C)*\\sqrt(B1C*B1A)*\\sqrt(C1A*C1B) = AB*BC*CA then A1, B1, C1 are the midpoints of BC, CA, AB. This means M is the circumcenter (this is obvious, it can be proven by congruent triangles for example.)", "Solution_3": "It is very easy to see, that PA/PA_1=AB_1/B_1C+AC_1/C_1B= MA/MC+MA/MB.\r\n\r\n(MA/MC+MA/MB)(MC/MA+MC/MB)(MB/MC+MB/MA)\\geq 8 and equality holds iff MA=MB=MC" } { "Tag": [ "inequalities open", "inequalities" ], "Problem": "If $ r\\geq 0$ and $ \\infty >f(r),g(r)\\geq 0$ then\r\n\\[ \\left(\\frac {w_1f^r(r) \\plus{} w_2g^r(r)}{w_1 \\plus{} w_2}\\right)^{\\frac {1}{r}}\\geq \\left(f^{w_1}(r)g^{w_2}(r)\\right)^{\\frac {1}{w_1 \\plus{} w_2}}\r\n\\]\r\nhere $ w_1,w_2 > 0$", "Solution_1": "...and $ f(r),g(r)$ is increasing functions. No one interested?", "Solution_2": "[quote=\"Sunjee\"]If $ r\\geq 0$ and $ \\infty > f(r),g(r)\\geq 0$ then\n\\[ \\left(\\frac {w_1f^r(r) \\plus{} w_2g^r(r)}{w_1 \\plus{} w_2}\\right)^{\\frac {1}{r}}\\geq \\left(f^{w_1}(r)g^{w_2}(r)\\right)^{\\frac {1}{w_1 \\plus{} w_2}}\n\\]\nhere $ w_1,w_2 > 0$[/quote]Is this what you mean really ? If so, simplily put $ f(r)\\equal{}: a$, $ g(r)\\equal{}: b$, $ \\frac {w_1}{w_1\\plus{}w_2}\\equal{}: \\lambda$, $ \\frac {w_2}{w_1\\plus{}w_2}\\equal{}: \\mu$, and your ineqality becomes\r\n\r\n$ (\\lambda a^r\\plus{}\\mu b^r)^{\\frac1r}\\ge a^\\lambda b^\\mu$. Weigthed power mean. :lol: :rotfl:", "Solution_3": "[quote=\"Amunaho\"][quote=\"Sunjee\"]If $ r\\geq 0$ and $ \\infty > f(r),g(r)\\geq 0$ then\n\\[ \\left(\\frac {w_1f^r(r) \\plus{} w_2g^r(r)}{w_1 \\plus{} w_2}\\right)^{\\frac {1}{r}}\\geq \\left(f^{w_1}(r)g^{w_2}(r)\\right)^{\\frac {1}{w_1 \\plus{} w_2}}\n\\]\nhere $ w_1,w_2 > 0$[/quote]Is this what you mean really ? If so, simplily put $ f(r) \\equal{} : a$, $ g(r) \\equal{} : b$, $ \\frac {w_1}{w_1 \\plus{} w_2} \\equal{} : \\lambda$, $ \\frac {w_2}{w_1 \\plus{} w_2} \\equal{} : \\mu$, and your ineqality becomes\n\n$ (\\lambda a^r \\plus{} \\mu b^r)^{\\frac1r}\\ge a^\\lambda b^\\mu$. Weigthed power mean. :lol: :rotfl:[/quote]\r\n\r\n $ f(r)\\equal{}a,g(r)\\equal{}b$ this is not true" } { "Tag": [ "analytic geometry", "complex numbers", "complex analysis", "complex analysis unsolved" ], "Problem": "What are the Cauchy-Riemann Equations in polar coordinates FOR the modulus and the argument?", "Solution_1": "The Cauchy-Riemann equations state that:\r\n\r\n$ \\frac{\\partial u}{\\partial x}\\equal{}\\frac{\\partial v}{\\partial y}$ and $ \\frac{\\partial v}{\\partial x}\\equal{}\\minus{}\\frac{\\partial u}{\\partial y}$\r\n\r\nThat's if $ z\\equiv x\\plus{}iy$\r\n\r\nIf $ z\\equiv re^{i\\theta}$ we get \r\n\r\n$ \\frac{\\partial u}{\\partial r}\\equal{}\\frac{1}{r}\\frac{\\partial v}{\\partial \\theta}$ and $ \\frac{1}{r}\\frac{\\partial u}{\\partial \\theta}\\equal{}\\minus{}\\frac{\\partial v}{\\partial r}$\r\n\r\nwhich is what you were asking.", "Solution_2": "Thank you for your answer, let me tell you why Im confused.\r\n\r\nWe know that if $ f(z) \\equal{} u(x,y) \\plus{} i v(x,y)$ and $ f$ is analytic then the real and the imaginary part are connected by the\r\nCauchy-Riemann equations: \r\n\r\n$ \\frac {\\partial v}{\\partial x} \\equal{} \\minus{} \\frac {\\partial u}{\\partial y},$ and $ \\frac {\\partial u}{\\partial x} \\equal{} \\frac {\\partial v}{\\partial y}.$\r\n\r\nIf $ f(z) \\equal{} u(r,\\theta) \\plus{} i v(r,\\theta)$ then we have the Cauchy-Riemann equations in polar coordinates:\r\n\r\n$ \\frac {\\partial u}{\\partial r} \\equal{} \\frac {1}{r}\\frac {\\partial v}{\\partial \\theta},$ and $ \\frac {1}{r}\\frac {\\partial u}{\\partial \\theta} \\equal{} \\minus{} \\frac {\\partial v}{\\partial r}.$\r\n\r\nNow if we put $ f(z) \\equal{} R(x,y)e^{i \\phi(x,y)}$ then the modulus $ R$ and the argument $ \\phi$ are connected by the relations\r\n\r\n$ \\frac {\\partial R}{\\partial x} \\equal{} R \\frac {\\partial \\phi}{\\partial y}$ and $ \\frac {\\partial R}{\\partial y} \\equal{} \\minus{} R \\frac {\\partial \\phi}{\\partial x}$.\r\n\r\nSo my questions is: The latter are the Cauchy-Riemann equations FOR the modulus and the argument? If so, then I guess I can deduce them in polar coordinates, in the same way we can do it for the real and the imaginary part.\r\n\r\nI mean if $ f(z) \\equal{} R(r,\\theta)e^{i \\phi(r,\\theta)}$ what are the equations relating the modulus and the argument.\r\n\r\nThank you" } { "Tag": [ "inequalities proposed", "inequalities" ], "Problem": "Let $ a,b,c$ be positive real numbers such that $ abc\\equal{}a\\plus{}b\\plus{}c\\plus{}2$.Prove that:\r\n$ \\frac{ab}{(a\\plus{}c)(b\\plus{}c)}\\plus{}\\frac{bc}{(b\\plus{}a)(c\\plus{}a)}\\plus{}\\frac{ca}{(c\\plus{}b)(a\\plus{}b)}\\le \\frac{3(abc\\minus{}4)}{8\\plus{}abc}$", "Solution_1": "[quote=\"vo thanh van\"]Let $ a,b,c$ be positive real numbers such that $ abc \\equal{} a \\plus{} b \\plus{} c \\plus{} 2$.Prove that:\n$ \\frac {ab}{(a \\plus{} c)(b \\plus{} c)} \\plus{} \\frac {bc}{(b \\plus{} a)(c \\plus{} a)} \\plus{} \\frac {ca}{(c \\plus{} b)(a \\plus{} b)}\\le \\frac {3(abc \\minus{} 4)}{8 \\plus{} abc}$[/quote]\r\nIT's easy because $ (abc)^2\\geq \\frac{9}{8}(a\\plus{}b\\plus{}c)(ab\\plus{}bc\\plus{}ca) \\geq 64$! :P" } { "Tag": [ "induction" ], "Problem": "Prove that none of the digits 2,4,7,9 can be the last digit of a number equal to the sum $1+2+3+ . . . +n$ where $n$ is an arbitrary natural number.", "Solution_1": "$\\sum_{i=1}^{n}{n}=\\frac{n(n+1)}{2}$ , after this just check remainders on 10.", "Solution_2": "[hide]It is easily verifiable for n = 1, 2, 3, 4....., 10\nSo we will prove it by induction for n+10\n\nAssume true for n=k, ie (n+1)(n)/2 not equal no 2,4,7,9 (mod 10)\nSubbing in k+10 and simplifing we get (n+1)(n)/2 + 55 + 10n\ntherefore by putting in k +10 we either add 5 or 0 (mod 10) so we end up that once again, (k+11)(k+10)/2 does not end in 2,4,7,9... and by the princable or induction, is true for all n. (This is a quick, not reall detailed proof, but not to hard to verify yourself...)[/hide]", "Solution_3": "Curses!I hate when someone puts in their message while your typing yours! :)", "Solution_4": "[hide=\"Even simpler is\"]\nmod 5 rather than 10 :) But of course, in the few seconds it takes you to think of that, you could probably have finished the mod 10 way..\n[/hide]", "Solution_5": "[quote=\"seamusoboyle\"]Curses!I hate when someone puts in their message while your typing yours! :)[/quote]\r\nI guess that \"someone\" don't know when you are typing your message. :)", "Solution_6": "Tis what i ment! :)" } { "Tag": [ "floor function", "ceiling function" ], "Problem": "How many positive multiples of 13 are three-digit integers?", "Solution_1": "There are 900 positive 3-digit integers, from 100~999. So, our answer is 900/13, or 69.23..., and you ignore the decimal, so 69.\r\nP.S. do not make the mistake of rounding...for example, when 900/7, it is NOT 129.", "Solution_2": "Hmm, actually first you do $ 99 \\div 13 \\approx 69$ and then $ 100 \\div 13 \\equal{} \\approx 8$. $ 69 \\minus{} 8 \\plus{} 1 \\equal{} \\boxed{62}$.", "Solution_3": "[quote=\"isabella2296\"]Hmm, actually first you do $ 99 \\div 13 \\approx 69$ and then $ 100 \\div 13 \\equal{} \\approx 8$. $ 69 \\minus{} 8 \\plus{} 1 \\equal{} \\boxed{62}$.[/quote]\nI think isabella2296 meant $\\lfloor\\frac{999}{13}\\rfloor=76$ and $\\lceil\\frac{100}{13}\\rceil=8$. Therefore, the number of 3-digit multiples of $13$ is $76-8+1=68+1=\\boxed{69}$." } { "Tag": [], "Problem": "Warum gelten reelle Formeln f\u00fcr $ sinx$, $ cosx$, $ sinhx$, $ coshx$, $ e^x$ automatisch auch im Komplexen? Wie ist die Situation fuer Funktionen, die nicht ueberall definiert sind?($ arctan x$, $ log x$(!) unw.)\r\n\r\n\r\n\r\nFitim", "Solution_1": "Diese Funktionen, wie auch alle daraus resultierenden, sind lokal holomorph/analytisch (d.h. zu jedem Punkt im Definitionsbereich gibt es eine offene Umgebung, in dem die Funktion komplex differenzierbar ist/in dem die Funktion durch eine Potenzreihe gegeben ist; beide Begriffe stimmen faktisch \u00fcberein). Nun gibt es den Eindeutigkeitssatz f\u00fcr holomorphe Funktionen:\r\n$ f,g: U \\to \\mathbb C$ seien holomorph auf einem (offenen, zusammenh\u00e4ngenden) Gebiet $ U$. Enthalte weiter $ M \\subset U$ einen seiner H\u00e4ufungspunkte. Dann gilt: Stimmen $ f,g$ auf $ M$ \u00fcberein, so auf ganz $ U$.\r\n\r\nNimmt man also einen zusammenh\u00e4ngenden Definitionsbereich in dem genannten Funktionen holomorph sind, so gilt eine \"Formel\" bereits dann, wenn sie z.B. auf einem Intervall einer positiven L\u00e4nge gilt.", "Solution_2": "Danke ZetaX! :wink:" } { "Tag": [ "inequalities", "function", "inequalities theorems" ], "Problem": "[b]Triangles relation for inequalities:[/b]\r\n\r\nHi: I just wanted to know from you, some of the relations between triangle things, specially, I would like to find expressions in terms of $R, r ,s (ABC)$, for the symetric functions of the sides $a,b,c$. If someone knows something, I will be very pleased if he could pot it here...", "Solution_1": "[quote=\"Pascual2005\"][b]Triangles relation for inequalities:[/b]\n\nHi: I just wanted to know from you, some of the relations between triangle things, specially, I would like to find expressions in terms of $R, r ,s (ABC)$, for the symetric functions of the sides $a,b,c$. If someone knows something, I will be very pleased if he could pot it here...[/quote]\r\n\r\nIt's well known that the following three identities hold in every triangle.\r\n$a+b+c=2s$;\r\n$ab+bc+ca=s^2+4Rr+r^2$;\r\n$abc=4Rrs$.\r\nWith them, we can ratiocinate all other symetric functions of the sides $a,b,c$.\r\nFor example,\r\n$a^3+b^3+c^3=(a+b+c)[(a+b+c)^2-3(ab+bc+ca)]+3abc=2s(s^2-6Rr-3r^2)$." } { "Tag": [ "number theory unsolved", "number theory" ], "Problem": "I post this not to receive a solution, because this seems to be rather impossible, but because the results is magnificient:\r\n There is a constant $C$ such that there are no numbers of the form $ y^n$ with $y>1$ and $n>C$ in the Fibonacci sequence.", "Solution_1": "Harazi: I have an idea that i cant finish...\r\n\r\nSuppose no such constant exist, first we can prove we can find a composite index $m$, such that $F_m=y^k$, now i think we can extend this lemma [url]http://www.mathlinks.ro/Forum/viewtopic.php?t=27757[/url] to the field $a+b\\sqrt{5}$. using it we can prove that $m=ty$, and that $F_t=y^{k-1}$ but then $F_m/F_t=y$ wich fails for big enough $k$ I dont know, it is just some crazy idea.... ;) and i have not been trainig lately ( i am on VACATION!!)" } { "Tag": [ "trigonometry", "ratio" ], "Problem": "Can two triangles have two equal sides and three equal angles, and still be noncongruent?\r\n\r\nIf yes, then how? ;)", "Solution_1": "well we know that they are similar, but we could have it so the corresponding sides are not equal", "Solution_2": "No, of course they have to be congruent (unless I misunderstood what you meant :? ). If any 2 angles and 1 side of the triangles are equal, they are congruent, and here more than 1 side and more than 2 angles are equal.", "Solution_3": "They must be congruent by SAS or ASA.", "Solution_4": "[quote=\"Altheman\"]well we know that they are similar, but we could have it so the corresponding sides are not equal[/quote]\r\n\r\nI don't think that's true, because consider that you have a $3,4,5$ triangle, and using trig we see that the angles are $36.9, 53.1, 90$ degrees, let's say we have another triangle with the same angles and sides $4$ and $3$ but let's say $4$ is the hypotenuse, but $3$ can't be another side because it goes against the ratio of the sides and law of sines." } { "Tag": [ "geometry", "3D geometry" ], "Problem": "What is the maximum number of regions that 9 planes can divide a box?", "Solution_1": "I'm guessing 81 (which is 9 squared). I think there may be more, but I can't find that solution. :blush:\r\n\r\nnever mind (edit), what the heck am I thinking?", "Solution_2": "[hide]\n1 plane-2\n2-4\n3-8\n4-14?\n\npattern\n\n9planes-58\n[/hide]", "Solution_3": "I think with four cuts, you can have 16 pieces, just cut into the shape of an 8-sided asterisk. \r\n\r\nSo now I think it's 2^9, which is 512.", "Solution_4": "I got fifteen regions for four cuts now...\r\n\r\n1-2\r\n2-4\r\n3-8\r\n4-15?\r\n\r\ndifferences-2,4,7...\r\n11,16,22,29,37\r\n\r\nterms-\r\n5-26\r\n6-42\r\n7-64\r\n8-93\r\n\r\n9-130\r\n\r\n\r\nEDIT:\r\nI just noticed,\r\n\r\npoints cutting a line (points, section)\r\n1-2\r\n2-3\r\n3-4\r\n4-5...difference is 1\r\n\r\nlines cutting a plane (lines, regions)\r\n\r\n1-2\r\n2-4\r\n3-7\r\n4-11...differences are the ammount of sections from \"points cutting a line\"\r\n\r\nplanes cutting a space (planes, regions)\r\n\r\n1-2\r\n2-4\r\n3-8\r\n4-15\r\n5-26...differences are the ammount of regions from \"lines cutting a plane\"", "Solution_5": "[quote=\"pianoforte\"]I got fifteen regions for four cuts now...\n\n[/quote]\r\n\r\nStrive for excellence: try to find one more :D", "Solution_6": "[hide]\nI think each plane divides the current space into 2 parts, so 9 planes would divide this into 2^9=512 parts??!!\n[/hide]", "Solution_7": "[quote=\"236factorial\"][quote=\"pianoforte\"]I got fifteen regions for four cuts now...\n\n[/quote=\"pianoforte\"]\n\nStrive for excellence: try to find one more :D[/quote]\n\nI can't find one more, and my answer fits the pattern. :? \n\n[quote][hide]EDIT:\nI just noticed,\n\npoints cutting a line (points, section)\n1-2\n2-3\n3-4\n4-5...difference is 1\n\nlines cutting a plane (lines, regions)\n\n1-2\n2-4\n3-7\n4-11...differences are the ammount of sections from \"points cutting a line\"\n\nplanes cutting a space (planes, regions)\n\n1-2\n2-4\n3-8\n4-15\n5-26...differences are the ammount of regions from \"lines cutting a plane\"[/hide][/quote][/quote]\r\n\r\nEDIT: I don't see how the forth plane could split all of the existing 8 planes, at least one has to be left out :?", "Solution_8": "sorry piano forte, I was thinking of something else. How did you get 8 for three cuts?", "Solution_9": "One plane going side to side and up and down, \r\nanother going side to side and forward and back, \r\nthe third going up and down, and forward and back, :P\r\n\r\nlike stacking eight cubes into a bigger cube, four on top, four on bottom.", "Solution_10": "______ \r\n|_|_|_|_|\r\n|_|_|_|_|\r\n|_|_|_|_|\r\n|_|_|_|_| = 16*4=64\r\n\r\n64 regions" } { "Tag": [], "Problem": "What makes a good math teacher/tutor/professor?\r\n\r\nI believe that a good math teacher is someone who can break the material down to the evel of students.\r\n\r\nYour view?\r\n\r\nI want several replies.", "Solution_1": "I believe a good math teacher is fair and appreciates everyone's skill in math, whether good or bad. \r\n\r\nA good math teacher should not give a lot of homework lol", "Solution_2": "a good math teacher is one who only cares about kids who do math competitions and doesnt teach the other people.", "Solution_3": "one that lets you go fast if you are getting it but understands when you don't", "Solution_4": "[quote=\"funcia\"]one that lets you go fast if you are getting it but understands when you don't[/quote]\r\n\r\none who believes that mistakes do happen and should be less strict about grades and more eabout learning the material itself", "Solution_5": "Anyone can take something and make it more complicated. It takes true insight and understanding to take something that's complicated and make it simple. It takes a truly good teacher to do this with mathematics.\r\n\r\nBuilding on that, mathematics teachers (before college, anyway) are notorious for making math seem like a collection of formulas. This goes with taking something simple and making it more complicated. A good mathematics teacher can show the relationships between concepts and unify ideas.\r\n\r\nA good math teacher is willing to deviate from the curriculum a bit and explore different concepts. Most math teachers will not teach things like Ptolmey's, finite differences, etc.\r\n\r\nA good math teacher does not teach to the test. Most math teachers teach to their own tests, and a lot of AP teachers teach to the AP test. I find this very annoying." } { "Tag": [ "function", "symmetry", "conics", "parabola", "calculus" ], "Problem": "Function $y=kx^2-4x-1$ is decreasing on $[3,6]$. Find the range of $k$.", "Solution_1": "[hide=\"Solution\"]We have $y'=2kx-4$ and it must be negative on $\\lbrack 3,6\\rbrack$. The derivation is a linear function, so its maximal value is in the left boundary for $k<0$ or in the right boundary for $k>0$:\n\n\\begin{eqnarray*}1^\\circ\\quad k<0 \\land y'(3)\\leq 0\\implies k<0 \\land 6k-4\\leq 0\\implies k<0\\land k\\leq{2\\over 3}\\implies k<0\\end{eqnarray*}\n\n\\begin{eqnarray*}2^\\circ\\quad k>0\\land y'(6)\\leq 0\\implies k>0\\land 12k-4\\leq 0\\implies k>0\\land k\\leq{1\\over 3}\\implies k\\in(0,{1\\over 3}\\rbrack\\end{eqnarray*}\n\n$3^\\circ\\quad k=0\\implies y=-4x-1$, so the function is decreasing on the whole $\\mathbb{R}$.\n\nHence, the answer is $k\\in(-\\infty,{1\\over 3}\\rbrack$[/hide]", "Solution_2": "[hide=\"solution\"]\n\nFor negative $k$ , we have $y=k(x-\\frac{2}{k})^2-1-\\frac{4}{k}$ . So the symmetry line of the parabola is negative value and hence over $[3,6]$ , $y$ definitely decreases .\n\nFor positive $k$ , since the symmetry line is at $x=\\frac{2}{k}$ . So in order for $y$ to decrease over $[3,6]$ , the symmetry line must be at least at $6$ which means $\\frac{2}{k}\\geq 6$ or $k\\leq \\frac{1}{3}$\n\nHence combine both cases to get the range for $k$ to be $(-\\infty,\\frac{1}{3}]$\n[/hide]", "Solution_3": "No calculus, Farenhajt! :)" } { "Tag": [ "geometry" ], "Problem": "what proportion of the area of a circle lies inside an inscribed regular hexagon?", "Solution_1": "[hide=\"My sorution\"]\nDivide the hexagon into 3 equliteral triangles with side $s$. All of these have area $\\frac{\\sqrt{3}}{4}s^{2}$. So the area of the hexagon is\n\\[\\frac{\\sqrt{3}}{4}s^{2}\\cdot 6=\\frac{3\\sqrt{3}}{2}s^{2}\\]\nTherefore, the proportion of the area of the circle that lies inside the hexagon is\n\\[\\frac{\\frac{3\\sqrt{3}}{2}s^{2}}{\\pi s^{2}}=\\frac{3\\sqrt{3}}{2\\pi}\\]\n[/hide]" } { "Tag": [ "ratio", "geometry", "perimeter" ], "Problem": "The ratio of the perimeters of two similar pentagons is 3 : 4. If the length of the shortest side of the smaller pentagon is 48, what is the length of the shortest side of the larger pentagon?\r\n\r\nThe choices are:\r\n\r\n(A) 24\r\n\r\n(B) 36\r\n\r\n(C) 64\r\n\r\n(D) 72", "Solution_1": "This should be in Classroom Math...\r\n\r\n[hide]$ \\frac{48}{3}=\\frac{x}{4}$\n\n$ x=64$[/hide]", "Solution_2": "[url]http://www.artofproblemsolving.com/Forum/viewtopic.php?t=153452[/url]\r\n\r\nPlease.", "Solution_3": "He has the right to post questions, but they have to be in the right forum.", "Solution_4": "I am sharkman not interval. Maybe this interval person was a real pain but I am tired of being compared to someone who probably lives on the other side of the globe.\r\n\r\nI am sharkman!! Why sharkman???\r\n\r\nI like sharks, the fish.\r\n\r\nGet it?\r\n\r\nThank you i_like_pie for your defense but I have no idea what these people are talking about.", "Solution_5": "He used similarity. You know, if we say:\r\n\r\n$ \\frac{1}{x}= \\frac{2}{3}$\r\n\r\nthen $ x = \\frac{3}{2}$.\r\n\r\nSimilarity is a crucial part of geometry. Good luck learning it." } { "Tag": [], "Problem": "What is the sum of the numbers less than 200 that have exactly 9 divisors?", "Solution_1": "[hide]36,100,196 so 332[/hide]", "Solution_2": "[quote=\"usaha\"][hide]36,100,196 so 332[/hide][/quote]\r\nHow did you get those numbers?", "Solution_3": "[quote=\"usaha\"][hide]36,100,196 so 332[/hide][/quote]\n\n\n[hide]A number can only have an odd number of factors if it is a perfect square. Now to find the amount of factors a number has you add 1 to each exponent and multiply the sums. So this can equal 9 only when the number equals one exponent which has to be eight or, two exponents which are 2 and 2. [/hide]" } { "Tag": [ "abstract algebra", "group theory", "superior algebra", "superior algebra unsolved" ], "Problem": "[color=violet][size=134]An abelian group $ M$ is called indecomposable if it can't be writen by a direct sum of two non-zero subgroup .\n\nIs any indecomposable abelian group $ M$ torsion-free(no finite order elements)? except the trival situation:$ Z[1/p]/Z,Z_{p^{n}}$ \n\nin other word ,if an indecomposable abelian group $ M$ is not torsion-free, it must be $ Z[1/p]/Z$, or $ Z_{p^{n}}$??[/size][/color]\r\n\r\n\r\n\r\n[size=150]i don't know whether this problem has been solved ?\n\nbut in the condition that $ M$ has only finite maximal subgroups we have got it !!\n\nthanks very much for who gives advices ![/size] ", "Solution_1": "This is theorem 10, page 22 of Kaplansky's textbook on Infinite Abelian Groups: an indecomposable abelian group is either Z[1/p]/Z, Z/p^nZ, or torsion-free.", "Solution_2": "thanks \r\nbut i don't have read this book.\r\nbut in another book there is this result.\r\n\r\nINFINITE ABELIAN GROUPS\r\nLd.szlo Fuchs\r\nTtclane University\r\nNew Orleans, Louisiana\r\nV O L U M E I\r\n\r\n\r\n\"Theorem 3.1. A group C is cocyclic if and only if CgZ(pk) with\r\nk = 1, 2, - - .o r 00.\r\n\r\nCorollary 27.3 (Kulikov [ l ] ) . If a group contains elements of fitiite order,\r\nthen it contains a cocyclic direct summand.\r\nCorollary 27.4 (Kulikov [l]). A directly indecomposable group is either torsion-free or cocyclic.\"\r\n\r\nso the guess is right....but it isnot proved by me.... :(" } { "Tag": [ "inequalities", "inequalities unsolved" ], "Problem": ":pilot: :censored:: Let a,b,c be posive numbers such that $ a\\plus{}b>c, b\\plus{}c>a, c\\plus{}a>b$ and $ a^2\\plus{}b^2\\plus{}c^2\\equal{}3$. Prove that $ \\sum \\sqrt{2\\minus{}a^2}\\geq a\\plus{}b\\plus{}c$", "Solution_1": "[quote=\"thegod277\"]:pilot: :censored:: Let a,b,c be posive numbers such that $ a \\plus{} b > c, b \\plus{} c > a, c \\plus{} a > b$ and $ a^2 \\plus{} b^2 \\plus{} c^2 \\equal{} 3$. Prove that $ \\sum \\sqrt {2 \\minus{} a^2}\\geq a \\plus{} b \\plus{} c$[/quote]\r\nI think, it's wrong. Try $ a\\equal{}1.4$ and $ b\\equal{}c\\equal{}\\sqrt{0.52}.$ :wink:", "Solution_2": "sorry. i think you right. thank you! :(" } { "Tag": [ "algebra", "polynomial" ], "Problem": "could someone help me factor these:\r\n -x^3+8x^2-8x-32\r\nand -x^3+3x+2", "Solution_1": "[quote=\"mikejones\"]could someone help me factor these:\n -x^3+8x^2-8x-32\nand -x^3+3x+2[/quote]\r\n\r\nWrong difficulty level, I think. ;) \r\n\r\nBut as for your problems, think Rational roots. ;)", "Solution_2": "1 . (x-4)[(4-x)(2+x)+2x]\r\n\r\n2 . (x+1)(x+1)(2-x)", "Solution_3": "$x^3-8x^2+8x+32\\ \\ |\\ \\cdot (-1)$\r\n$x^3+0x^2-3x-2$\r\n$============$\r\n$8x^2-11x-34\\ \\ \\ \\ \\ \\ \\ |\\ \\cdot (-x)$\r\n$x^3+0x^2-3x-2\\ \\ \\ |\\ \\ \\cdot \\ 8$\r\n$============$\r\n$11x^2+10x-16\\ \\ \\ \\ \\ \\ |\\ \\ \\ \\cdot \\ 8$\r\n$8x^2-11x-34\\ \\ \\ \\ \\ \\ \\ \\ |\\ \\ \\cdot (-11)$\r\n$============$\r\nA.S.O. These polynominals hasn't common factors. I think you make a mistake in the coefficients !" } { "Tag": [ "calculus", "integration", "analytic geometry", "trigonometry", "Gauss", "calculus computations" ], "Problem": "Calculate the integral\r\n\r\n$\\int \\int_V \\int \\frac{1}{x^2 + y^2 + z^2} \\text dx \\text dy \\text dz$\r\n\r\nover the volume $V$ defined by $x^2 + y^2 + (z - 1)^2 \\le 1$.", "Solution_1": "I am in a hurry right now, but let's see. I suppose that the best way to do it is make a change of coordinates to spherical coordinates, and so, If I am not mistaken (As I said, right now I don't have time to review my calculations), the integral becomes,\r\n\r\n\\[\\int_{0}^{2\\pi}\\int_{-\\frac\\pi2}^{\\frac\\pi2}\\int_{0}^{2\\cos\\phi}\\frac{1}{\\rho^2}\\;d\\rho\\;d\\phi\\;d\\theta\\]\r\n\r\nWhat is easier to compute. Sorry, gotta go. Bye,", "Solution_2": "In hurry you forgot to multiply by the Jacobian $\\rho^2\\sin\\varphi$. ;) If you do that the integral becomes really easy...", "Solution_3": "[quote=\"fedja\"]In hurry you forgot to multiply by the Jacobian $\\rho^2\\sin\\varphi$. ;) If you do that the integral becomes really easy...[/quote]\r\n\r\nIt was not just the missing jacobian $r^2 \\sin \\theta$, but also the integration limits. If you go to spherical coordinates $r, \\phi, \\theta$\r\n\r\n$x = r \\cos \\phi \\sin \\theta$\r\n\r\n$y = r \\sin \\phi \\sin \\theta$\r\n\r\n$z = r \\cos \\theta$\r\n\r\nthe integration limits are not constant for either of $r, \\phi$ or $\\theta$. If you go to spherical coordinates $r, \\phi, \\theta$\r\n\r\n$x = r \\cos \\phi \\sin \\theta$\r\n\r\n$y = r \\sin \\phi \\sin \\theta$\r\n\r\n$z = r \\cos \\theta + 1$\r\n\r\nthen the integration limits are constant and the integral becomes\r\n\r\n$\\int \\int_V \\int \\frac{1}{x^2 + y^2 + z^2} \\text dx \\text dy \\text dz = \\int_0^{2 \\pi} \\int_0^{\\pi} \\int_0^1 \\frac{r^2 \\sin \\theta}{r^2 + 2r \\cos \\theta + 1} \\text dr \\text d\\theta \\text d\\phi = 2 \\pi \\int_0^{\\pi} \\int_0^1 \\frac{r^2 \\sin \\theta}{r^2 + 2r \\cos \\theta + 1} \\text dr \\text d\\theta$\r\n\r\nThis can be integrated in a rather laborious way by expanding the integrand into a power series $\\sum_{n = 0}^{\\infty} a_n(\\cos \\theta) r^n$, integrating term by term and then figuring out how to sum the results. But there is another solution, much simpler than going to spherical coordinates.", "Solution_4": "[quote=\"yetti\"] If you go to spherical coordinates $r, \\phi, \\theta$\n\n$x = r \\cos \\phi \\sin \\theta$\n\n$y = r \\sin \\phi \\sin \\theta$\n\n$z = r \\cos \\theta$\n\nthe integration limits are not constant for either of $r, \\phi$ or $\\theta$. \n\n[/quote]\r\nI think djimenez used \r\n$x = \\rho \\sin \\phi \\cos \\theta$\r\n\r\n$y = \\rho \\sin \\phi \\sin \\theta$\r\n\r\n$z = \\rho \\cos \\phi$\r\n\r\nThen his limits of integration are correct ;)", "Solution_5": "There are too many different angle conventions for spherical coordinates here.\r\n\r\ndjimenez is using the convention (standard in calculus books) that $\\theta$ is longitude, $\\phi$ is colatitude, and the radius is $\\rho$, but his limits are slightly wrong as well as the Jacobian: it should be $\\int_0^{2\\pi}\\int_0^{\\frac\\pi2}\\int_0^{2\\cos\\phi}\\sin\\phi\\, d\\rho\\,d\\phi\\, d\\theta$\r\n(The entire ball is in the \"north\" half-space $0\\le\\phi\\le\\frac\\pi2$. The southern half-space would be $\\frac\\pi2\\le\\phi\\le\\pi$, and I don't know where $\\frac{-\\pi}2$ came from.)\r\nThis is a simple integral- easier than yetti's method.\r\n\r\nyetti is using the convention (standard in physics books) that $\\theta$ is colatitude, $\\phi$ is longitude, and the radius is $r$.\r\n\r\nSpherical coordinates centered at the origin make the integrand nice. Since the limits aren't too bad, it works better than spherical coordinates centered at the ball of integration.", "Solution_6": "[quote=\"jmerry\"] it should be $\\int_0^{2\\pi}\\int_0^{\\frac\\pi2}\\int_0^{2\\cos\\phi}\\sin^2\\phi\\, d\\rho\\,d\\phi\\, d\\theta$\n[/quote]\r\nOops :blush: jmerry is right as to the limits. As to the power of $\\sin\\phi$, I disagree:\r\nthe volume element is $d\\rho\\cdot \\rho \\,d\\phi \\cdot \\rho\\sin\\phi \\,d\\theta$, so the integral becomes $\\int_0^{2\\pi}\\int_0^{\\frac\\pi2}\\int_0^{2\\cos\\phi}\\sin\\phi\\, d\\rho\\,d\\phi\\, d\\theta$. Seems like we all are not at our best today :P", "Solution_7": "Oops... I was confused about a physical interpretation when I edited that in. $\\sin^2\\phi$ comes from the $z$-component of an inverse-square force.", "Solution_8": "[quote=\"fedja\"]\nThen his limits of integration are correct ;)[/quote]\r\n\r\nNot entirely. It should be\r\n\r\n$\\int_0^{2 \\pi} \\int_0^{\\frac \\pi 2} \\int_0^{2 \\cos \\phi} \\frac{r^2 \\sin \\phi}{r^2} \\text dr \\text d\\phi \\text d\\theta$\r\n\r\nWhat I had in mind was to substitute $\\frac{1}{r^2} = \\text{div}\\left( \\frac{\\vec r}{r^2} \\right)$ and then use Gauss' theorem\r\n\r\n$\\int \\int_V \\int \\frac{1}{r^2} \\text dV = \\int \\int_V \\int \\text{div}\\left( \\frac{\\vec r}{r^2} \\right) \\text dV = \\int_S \\int \\frac{\\vec r}{r^2} \\cdot \\text d\\vec S$\r\n\r\nSubstituting $\\vec \\rho = \\vec r - \\vec k$, the integration surface becomes a unit ball $\\Sigma$ and\r\n\r\n$r^2 = |\\vec \\rho + \\vec k|^2 = \\rho^2 + 2\\rho \\cos \\theta + 1 = 2(1 + \\cos \\theta)$\r\n\r\n$\\vec \\rho \\cdot \\text d\\vec \\Sigma = \\text d\\Sigma$ [color=white].[/color] and [color=white].[/color] $\\vec k \\cdot \\text d\\vec \\Sigma = \\cos \\theta\\ \\text d\\Sigma$\r\n\r\n$\\int_{\\Sigma} \\int \\frac{(\\vec \\rho + \\vec k)}{|\\vec \\rho + \\vec k|^2} \\cdot \\text d\\vec \\Sigma = \\int_{\\Sigma} \\int \\frac{1 + \\cos \\theta}{2(1 + \\cos \\theta)} \\text d\\Sigma = \\frac 1 2 \\int_{\\Sigma} \\int \\text d\\Sigma = 2 \\pi$", "Solution_9": "[quote=\"fedja\"]In hurry you forgot to multiply by the Jacobian $\\rho^2\\sin\\varphi$. ;) If you do that the integral becomes really easy...[/quote]\r\n\r\nUpsss :blush: :blush: , as I told, I was in a hurry, and well, you already saw the resoult of that :oops: .", "Solution_10": "[quote=\"djimenez\"]..., as I told, I was in a hurry, and well, you already saw the result of that :oops: .[/quote]\r\n\r\nIt was very nice and quick solution. At first, I could not get it, because of the incorrect limits $-\\frac \\pi 2, \\frac \\pi 2$. This gave me the impression that the missing jacobian must be $\\rho^2 \\cos \\phi$ and than it did not add up.\r\n\r\nyetti" } { "Tag": [ "geometry", "algebra", "polynomial", "quadratics", "absolute value" ], "Problem": "Let y = ax^2 + bx + c be a second degree polynomial with real roots, where a < 0. Find the area enclosed by the x axis and the quadratic in terms of a,b,c . \r\n\r\nAnd don't give me Archimedes' 2/3bh ! I want it in terms of a,b,c.", "Solution_1": "why must you know this??\r\n\r\nx1 = (-b - :sqrt: (b^2-4ac))/2a\r\nx2 = (-b + :sqrt: (b^2-4ac))/2a\r\n\r\n\r\ny = ax^2 + bx + c\r\n\r\nax^3 / 3 + bx^2 / 2 + cx from x1 to x2\r\nthat's assuing x2 is to the right of x1, if u get a negative answer... just absolute value it.\r\n\r\narea ( a/3*(x2)^3 +b/x*(x2)^2 +c(x2) ) -\r\n( a/3*(x1)^3 +b/x*(x1)^2 +c(x1) )\r\n\r\n\r\ni think that's right, i dunno, its a ridiculous formula, not too bad when you use actual numbers though.", "Solution_2": "You must go further! hehehe don't worry, keep working on it. It actually works out to a pretty nice formula", "Solution_3": "it's just |integral_(x1)^(x2)(f(x))|=|[2ax+b]_x1^x2|=|2ax1+b-2ax2-b|=|2ax1-2ax2|\r\nx1=lower root of f(x)\r\nx2=upper root of f(x)\r\n|2ax1-2ax2|=|2a((-b- :sqrt: (b :^2: -4ac))/2a-(-b+ :sqrt: (b :^2: -4ac))/2a)|\r\n=|2a(-2 :sqrt: (b :^2: -4ac)/2a)|=|-2 :sqrt: (b :^2: -4ac)|=2 :sqrt: (b :^2: -4ac)" } { "Tag": [ "geometry" ], "Problem": "The area of a square is 64 square centimeters. What is the number of square centimeters in the area of a new square made by decreasing each side of the original square by $ 75\\%$?", "Solution_1": "Each of the sides of the square has length $ \\sqrt{64}\\equal{}8$. So, $ 8(1\\minus{}.75)\\equal{}2$. Hence, the new area is $ 2^2\\equal{}\\boxed{4}$." } { "Tag": [], "Problem": "Interested in what IPO is about? Here is the [url=http://www.philosopiad.org/]site[/url]. Enjoy :)", "Solution_1": "I actually didn't know that such an olympiad exists until now !! :( And anyway, it might be sometime before India begins to participate..." } { "Tag": [ "percent" ], "Problem": "If the word REPETITION is repeatedly written until there are 1000 letters, what percent of the letters are E's?", "Solution_1": "Well, there are exactly $ 10$ letters, so you can divide the 1000 letters by 10 and by 10 again. Thus, there are only $ \\boxed{20}$ percent E's.", "Solution_2": "Note that $ \\dfrac{\\frac{1000}{10}}{10}\\equal{}10$ :wink:", "Solution_3": "No, ernie is correct. There are 2 e's. We write the word REPETITION 100 times. We have 20 e's. Thus the percent that we want is $ \\boxed{\\%20}$.\r\n\r\nEdit: Oh...", "Solution_4": "AIME15 wasn't stating an answer, just stating an observation.", "Solution_5": "Sorry if I'm being ignorant but I don't quite understand what AIME15's observation was about. What does it have to do with the problem?", "Solution_6": "I just looked at the problem and saw it was 2 e's out of 10. no matter what multiple of 10, it will always be 20%." } { "Tag": [ "inequalities", "inequalities open" ], "Problem": "hey friends!\r\n\r\ngoing thru many resources i hav come across the symmetric and cyclic summation ....symmetric mostly...\r\n\r\nthe expression of a symmetric summation is generally done in a single step as if it were on their tips...\r\n\r\ni find it tedious and irritating to find these summations by the beginners' method..\r\n\r\nso if any of u r aware of any ways tricks or shortcuts to find these expressions please post them here..\r\n\r\nsfsreporter", "Solution_1": "does no one want 2 help or is it that there not any known methods 2 do this?", "Solution_2": "http://www.mathlinks.ro/Forum/viewtopic.php?t=103773", "Solution_3": "i think u got me wrong i have problems expanding an expression in the symmetric\\cyclic forms\r\n\r\nfor example---\r\n\r\nhttp://www.mathlinks.ro/Forum/viewtopic.php?t=32470&start=20\r\n\r\npost #22\r\n\r\nhow did this man got the expression in a single step...... fine he must have done some computations on paper beforehand ...but this seems 2 be genius.\r\n\r\neven in assignments like Kiran Kedlaya and Mildrof they do this saying \"after some [b]simple computations[/b] we get.....\"\r\n\r\ni keep trying these \"simple computations\" and it seems that i am a fool!! who finds this difficult.\r\n\r\nso i wanted 2 know if there is any method to do all this by \"SIMPLE COMPUTATIONS\"???", "Solution_4": "By \"simple computations\" they mean something that, say, a computer could do for you. It isn't hard to keep applying the distributive law over and over and over and over and over (as long as you don't make a mistake in the middle).\r\n\r\nIt doesn't mean that they found a really fast way to do it by magic. It means that they actually sat down and expanded and expanded and expanded, and didn't feel like writing all of the expanding down so they skipped right to the end of all the expanding in their explanation for us.\r\n\r\nAn analogy: somebody asks you what 534/716 is to eight decimal places. You tell them, \"after some simple calculations, I got 0.74581005.\" They say, \"ohmigod how did you figure that out, I kept guessing 0.7457 or maybe 0.74583??? How did you do it in one step??\" The truth is you spent three hours with a pencil and paper doing long division...", "Solution_5": "thx for that kindda consolidation.\r\n\r\ni feel better now..\r\n\r\nbut then if i get Iran 1996... i mean a similar problem in an olympiad ...so do i need to expand it all sitting dere in da examination hall?? \r\n\r\nis there no other possible way?", "Solution_6": "sorry cant edit but i meant consolation...not consolidation", "Solution_7": "[quote=\"sfsreporter\"]thx for that kindda consolidation.\n\ni feel better now..\n\nbut then if i get Iran 1996... i mean a similar problem in an olympiad ...so do i need to expand it all sitting dere in da examination hall?? \n\nis there no other possible way?[/quote]\r\nFirst of all: Skip the abbreviations. It is very difficult to read what you have written, when you use such bad english.\r\nIn general there is a lot of other ways than expanding.\r\nLet's take an example:\r\n$ \\sum_{cyc} \\frac {x \\plus{} z }{2y} \\ge \\sum_{cyc} \\frac {2y}{x \\plus{} z}$ for positive $ x,y,z$.. Now you could multilply with $ xyz(x \\plus{} y)(y \\plus{} z)(z \\plus{} x)$ expand it and then use muirhead and schur or something like that.\r\nA easy way to solve this is to notice that $ x \\plus{} z \\ge 2\\sqrt {xy}$ and thus $ \\frac {y}{\\sqrt {xz}} \\ge \\frac {2y}{x \\plus{} z}$.\r\nBut the inequality $ \\sum_{cyc} \\frac {x \\plus{} z}{2y} \\ge \\sum_{cyc} \\frac {y}{\\sqrt {xz}}$ is very easy to prove. It comes from muirhead since $ (1,0, \\minus{} 1) \\succ (1, \\minus{} \\frac {1}{2}, \\minus{} \\frac {1}{2})$. Also $ \\frac {y}{x} \\plus{} \\frac {y}{z} \\ge \\frac {y}{\\sqrt {xz}}$, and so on with all three variables. Addition yields the desired.\r\nSometimes you can also make a minor change in the inequality like i did, and then expand. It comes from experience:)", "Solution_8": "[quote]Skip the abbreviations. It is very difficult to read what you have written, when you use such bad english. [/quote]\r\n\r\nsorry but i was in a hurry.. i think it was not that much of sms language..it was quite moderate but its fine...i'll prefer the other way now..\r\n\r\n\r\ni'm afraid but u hav made a typo in the problem ...cud you plz edit it so that i can have a better look at the thing.\r\n\r\nthanks!", "Solution_9": "[quote=\"sfsreporter\"][quote]Skip the abbreviations. It is very difficult to read what you have written, when you use such bad english. [/quote]\n\nsorry but i was in a hurry.. i think it was not that much of sms language..it was quite moderate but its fine...i'll prefer the other way now..\n\n\ni'm afraid but u hav made a typo in the problem ...cud you plz edit it so that i can have a better look at the thing.\n\nthanks![/quote]\r\nI sound more arrogant than I actually am, when I'm writing. I just get confused when I see \"R\" instead of \"are\" :)\r\n\r\nAnyway. The example is fixed." } { "Tag": [], "Problem": "I am sorry for asking so much but:\r\nHow many positive two-digit integers are there in which each of the two digits is prime?\r\nI got 4 but the answer says 16...\r\n\r\nakhil0422", "Solution_1": "There are $ 4$ prime digits: $ 2$, $ 3$, $ 5$, and $ 7$. Thus, there are $ 4$ choices for the first prime digit, and $ 4$ choices for the second prime digit. Multiplying yields $ 4 \\times 4 \\equal{} \\boxed{16}$." } { "Tag": [ "MATHCOUNTS", "AMC", "AMC 8", "email", "articles", "search" ], "Problem": "Sigh...you people who left early to have time to trick-or-treat or whatever missed out on sooo much. Oh well. I hope you have the info necessary to do the homework. If not I can explain stuff here I guess. If you stayed, thanks for being patient. Either way, discuss the problem sets here. Last week's answers will be up eventually.\r\n\r\nOh yeah--AMC-8 NEXT TUESDAY! Sign up! Yay.", "Solution_1": "hey, I don't think you're doing a good job of making the mc kids come here.\r\nthis forum is like, pretty inactive. No one(from jls) posts here cept susan, linda and that other slt person that I dont know. \r\nand do everyone who sign up for AMC8 get to take it? that's nice....", "Solution_2": "susan, linda, slt don't post here anymore. this forum is dead. now i can't post since there aren't any new posts. also can i take the amc 8 for fun?", "Solution_3": "Huh, I think I'll start demanding that everyone post at least one solution to a problem beginning the week after next. I don't know if anyone will listen to me but whatever. David, we've only got 30 tests. I strongly suggest you not waste one that could have been used by a JLSer. Though I suppose you could go in case less than 30 people show up.", "Solution_4": "Wait a sec...the AMC-8 is NEXT week?! I thought it was today! Darn it. That means I shouldn't have gone last week, when I did, and should have gone today, when I didn't. I'm so easily confused. Darn it. I hope Ms. Beddoes didn't wait too long for me. Better email her about this.", "Solution_5": "Uh... As amusing as that was...no more spam.\r\n\r\nIt's a good thing that the AMC wasn't today: now I can recruit that former 6th grader that totally pwned math class to math club. :)", "Solution_6": "[quote=\"ccy\"]It's a good thing that the AMC wasn't today: now I can recruit that former 6th grader that totally pwned math class to math club. :)[/quote]\r\n\r\nWho's that? Does (s)he go to Terman?", "Solution_7": "according to ccy, its jeffrey wang's brother", "Solution_8": "[quote=\"moogra\"]according to ccy, its jeffrey wang's brother[/quote]\r\nand according to you, you're a retard", "Solution_9": "[quote=\"junggi\"][quote=\"moogra\"]according to ccy, its jeffrey wang's brother[/quote]\nand according to you, you're a retard[/quote]\r\n\r\nno thats you. ur the only one here trying to screw around. im answering questions.", "Solution_10": "[quote=\"moogra\"][quote=\"junggi\"][quote=\"moogra\"]according to ccy, its jeffrey wang's brother[/quote]\nand according to you, you're a retard[/quote]\n\nno thats you. ur the only one here trying to screw around. im answering questions.[/quote]\r\nno you weren't.\r\nyou were giving answers that made no sense.\r\naccording to you, ppl throw helmets at each other when theyre a tiny bit pissed. of course that's complete nonsense. even if that was true, you still haven't justified your action, because from what I saw from your face before you threw the fing shiit, you weren't just pissed. You were EXTREMELY angry. Just because i threw a piece of paper. wtf is that? (Remember that you tried to punch my head off before you threw that helmet.)\r\n\r\njust to remind you what happened, bc, given your intelligence level, I doubt that you remember.\r\n\r\nI tried to get you to come to mandelbrot because I thought you would have liked to(because you spend a lot of time on mathcounts and all that. and of course I was wrong to think that you would still have interest left in math. oh and btw, you coming to this forum is a complete waste of your time, because what do you do? YOU POST SOLUTIONS FOR MATHCOUNTS AND MOEMS PROBLEMS. I'd be fine with it if you were trying to teach them something, but no, you post either wrong solutions, or completely inelegant solutions. I wouldn't be saying this if posting solutions there actually helped you. But it doesn't. You only do the trivial problems. You'd be better off playing Maple story or some other games for retards.). Then you keep insulting me, and how I'm superlame because I like math and that bs.\r\nSo I get pissed, like everyone would when theyre insulted. So I throw a piece of paper. Then you suddenly turn into this beast thingy with an extremely angry face on, and you try to punch my head off. Then you stop for some reason, so I try to pick up the paper. Then when I had my head down, you, like an fing coward, pick up the helmet, and threw it at my head. WTF", "Solution_11": "You do have a point. and i never said math was lame, and i don't post wrong solutions. Also good job on the essay. It was interesting from your point of view. \r\n-edit-\r\n(even if I do post wrong solutions (might be 1 somewhere), i don't try to and that shows I actually tried to do the problems)", "Solution_12": "This is weird.", "Solution_13": "I agree.", "Solution_14": "Yes this is weird. :D This scares everyone away", "Solution_15": "er...hi...am I interrupting anything??????\r\n\r\nOK. SO bookaholic. I'm gonna drop out of Analysis soon. Another thing. Tell me. Just HOW do you avoid idoticly retarded mistakes, like the one I did on the practice Chapter Round and the five retardedly idiotic mistakes I made on the Analysis test?\r\n\r\nBy the way, junggi, nice er...essay...you have there....\r\n\r\nSomething else I forgot........", "Solution_16": "[quote=\"moogra\"]Yes this is weird. :D This scares everyone away[/quote]\nok f u, retard.\nand seriously, stop coming here, and play maple story or something\n\n[quote]By the way, junggi, nice er...essay...you have there.... [/quote]\r\nlol yea did you actually read that? of course not lol", "Solution_17": "[quote]Quote:\nBy the way, junggi, nice er...essay...you have there....\n\nlol yea did you actually read that? of course not lol\n[/quote]\r\n\r\nWRONG junggi, I actually read it. Shows how bored I got. 0.0\r\n\r\nBOOKAHOLIC WHENEVER YOU'RE ON I NEED SOME AMC 8, 10. and 12 PRACTICE PROBLEMS!!! (Er...I don't exactly know where to find them so.... :?: :| )", "Solution_18": "[quote=\"Bluestalk\"][quote]Quote:\nBy the way, junggi, nice er...essay...you have there....\n\nlol yea did you actually read that? of course not lol\n[/quote]\n\nWRONG junggi, I actually read it. Shows how bored I got. 0.0\n\nBOOKAHOLIC WHENEVER YOU'RE ON I NEED SOME AMC 8, 10. and 12 PRACTICE PROBLEMS!!! (Er...I don't exactly know where to find them so.... :?: :| )[/quote]\r\n................................................................................\r\nTRY THE AOPS FORUMS? :?: :?:\r\n(THAT MEANS MATHCOUNTS,AND HSB FORUMS)", "Solution_19": "Drop out of Analysis?! Why?! You're an eighth grader, your grades this year won't be recorded. Even if you de-skip a grade, shouldn't you at least wait until it actually matters? Or is it taking too much time or something?\r\n\r\nAs for stupid mistakes...I'm not sure. I think there's an article about eliminating stupid mistakes in the Resources section. You should try the ideas in there. I didn't find it very helpful but that's probably just me. My carelessness is too ingrained in my nature. Um. If the problem is longish and complicated, I would suggest running through the solution in your head or even writing it out (very abbreviated of course) if you have time, and stopping at each step to make sure it's logical. Keep your scratch work very organized if possible. Make sure you're reading the problems right, I think misreading is the most preventable of stupid mistakes. What else...if you're making arithmetic mistakes or something, that should theoretically get better with practice. I don't know if it actually does.\r\n\r\nOh. Practice problems? They're all over AoPS. http://www.artofproblemsolving.com/Forum/resources.php?c=182 is the official page for all the former AMC series contest problems, but the unofficial problems in the forums are almost as useful. If you search in the AMC forum, people have been writing a lot of mock AMCs and AIMEs. I think there have been mock MathCounts and AMC-8 contests in the MathCounts forum. You can also buy some from http://www.unl.edu/amc . They're not really all that expensive.", "Solution_20": "[quote=\"Bluestalk\"]\nI'm gonna drop out of Analysis soon. \n[/quote]\n :o whoa i didnt see that before...\nas for stupid mistakes, this might help\n[url]http://www.artofproblemsolving.com/Resources/AoPS_R_A_Mistakes.php[/url]\n[quote=\"moogra\"]and i never said math was lame, and i don't post wrong solutions. Also good job on the essay. It was interesting from your point of view.\n[/quote]\ndude, stop lying. \nI clearly remember you saying \"YOURE TRYING TO MAKE TO COME TO A [i]MATH [/i]CONTEST! HOW LAME CAN YOU GET?\" and similar stuff\n[quote=\"moogra\"]\n(even if I do post wrong solutions (might be 1 somewhere), i don't try to and that shows I actually tried to do the problems)\n[/quote]\r\ndude, I saw you posting SUPER inelegant(means retarded soln in case you didnt know) solns to trivial problems. Also, PLEASE dont post solns after 1028084107 people already posted the exact same sonl you have", "Solution_21": "[quote=\"bookaholic\"]Drop out of Analysis?! Why?! You're an eighth grader, your grades this year won't be recorded. Even if you de-skip a grade, shouldn't you at least wait until it actually matters? Or is it taking too much time or something?\n\nAs for stupid mistakes...I'm not sure. I think there's an article about eliminating stupid mistakes in the Resources section. You should try the ideas in there. I didn't find it very helpful but that's probably just me. My carelessness is too ingrained in my nature. Um. If the problem is longish and complicated, I would suggest running through the solution in your head or even writing it out (very abbreviated of course) if you have time, and stopping at each step to make sure it's logical. Keep your scratch work very organized if possible. Make sure you're reading the problems right, I think misreading is the most preventable of stupid mistakes. What else...if you're making arithmetic mistakes or something, that should theoretically get better with practice. I don't know if it actually does.\n\nOh. Practice problems? They're all over AoPS. http://www.artofproblemsolving.com/Forum/resources.php?c=182 is the official page for all the former AMC series contest problems, but the unofficial problems in the forums are almost as useful. If you search in the AMC forum, people have been writing a lot of mock AMCs and AIMEs. I think there have been mock MathCounts and AMC-8 contests in the MathCounts forum. You can also buy some from http://www.unl.edu/amc . They're not really all that expensive.[/quote]\r\n\r\nUh bookaholic... Next time you see bluestalk, ask her about the retarded (for the lack of a better word) way our teacher grades tests and quizzes. I got points knocked off because I didn't round correctly :( and for some other trivial mistakes I can't remember.\r\n[hide=\"testimony by junggi\"][20:05:37] me: do you agree that beck grades retardedly?\n[20:05:43] junggi: of course\n[20:05:49] junggi: she takes off 3 pts for one mistake\n[20:06:00] me: thanks im quoting you\n[20:06:05] junggi: ok[/hide]\r\n\r\n@bluestalk: Here's a nice resource for AMC practice problems: http://www.unl.edu/amc/mathclub/04,1-problems.html (click the IAP links). What will you do if you drop analysis? School service? :maybe:", "Solution_22": "Gee, people, FYI, I'm stupidder than you think. Stupidder. Stupider. Which one...?\r\n\r\nOK, thanks for all those links....I'm only allowed like 10 minutes on the computer at home, so that helps. =P", "Solution_23": "It's probably stupider. But at least you probably have never written \"2n^2-2n+1=2n^2-1\" on a contest problem sheet...\r\n\r\n(See, the thing is, I distributed 2n(n-1)+1 as 2n^2-2+1, so I wrote 2n(n-1)+1=2n^2-2+1=2n^2-1 on my paper. Later I realized I'd left out an n and scribbled it into the expression in the middle but FORGOT that that made the expression on the right invalid and just used it anyway. Argh.)", "Solution_24": "Er...OK.....I do that periodically...\r\n\r\nBookaholic, I pmed you about some Mock AMC questions I didn't get.....\r\n\r\nIf you have the time, could you post solutions? Thanks!!\r\n\r\nOH YEAH. One of the orchestra flute players is the president of PALY Math Club. Adrian Sanborn or somethinglikethat...anyway he says you're good at math. =P Don't deny it....\r\n\r\nDeny it in your head if you must then...." } { "Tag": [ "combinatorics solved", "combinatorics" ], "Problem": "We have a regular polygon with 12 sides. How many edges can we draw between 2 of the vertex in such a way that there are no triangles???", "Solution_1": "Do you mean triangles whose vertices are vertices of the 12-gon? Or whose vertices are allowed to be intersection points of some diagonals?\r\n\r\nPierre.", "Solution_2": "I mean triangles whose vertices are vertices of the 12-gon.", "Solution_3": "So, the maximum is 12 2 /4 = 36 as a direct consequence of Turan's theorem on graph without triangle.\r\n\r\nPierre." } { "Tag": [ "modular arithmetic", "number theory unsolved", "number theory" ], "Problem": "Let $ n$ be a natural number, $ n \\geq 2$.Prove that if $ \\frac{a^n\\minus{}1}{a\\minus{}1}$ is a prime power for some positive integer $ a$, then $ n$ is a prime number.", "Solution_1": "Assume that n is a composite number. We can write it as n=x.y x and y are positive integers greater than 1. (Not neccesarily distinct.) W.l.o.g. we may assume that x is greater than or equal to y. (a^(x.y) - 1)/(a-1) = p^k for some prime p and positive integer k. We can rewrite this equation as (a^x - 1)((a^x)^(y-1) + (a^x)^(y-2) + ... + 1)/(a-1). For the sake of the proof we will use the following notation. a^x = r. (a^(x-1) + a^(x-2) + ... + 1)(r^(y-1) + r^(y-2) + ... + 1) =p^k. r=1 (mod a^(x-1) + ... + 1). (Since r=(a-1).(a^(x-1)+ ... 1) +1.) So the first is smaller than the second. a^(x-1) + a^(x-2) + ... + 1 divides r^(y-1) + ... +1. r=1 (mod a^(x-1) + ... +1) So r^(y-1) + ... + 1= y (mod a^(x-1) + ... + 1) So a^(x-1) + ... + 1 should divide y. But at first, we assumed that x is greater than or equal to y. a^(x-1) + ... + 1 is greater than x. (It can be shown by using Bernoulli's Inequality.) So our assumption was wrong hence n must be a prime. :P", "Solution_2": "$ n = kl$ $ \\implies$ $ (k,l > 1)$\r\n$ \\frac {a^{n} - 1}{a - 1} = p^{s}$ $ \\implies$\r\n$ (a^{k})^{l} - 1 = p^{s}.(a - 1)$ $ \\implies$ \r\n$ (a^{k} - 1)((a^{k})^{l - 1} + ... + a^{k} + 1) = p^{s}.(a - 1)$\r\n$ p|a^{k} - 1,p|(a^{k})^{l - 1} + ... + a^{k} + 1$ $ \\implies$ $ p|l$\r\nsimilarlay; $ (a^{l})^{k} - 1 = (a - 1).p^{s}$ $ \\implies$ $ a^{l}\\equiv 1\\pmod{p}$ \r\n$ \\implies$ $ p|k$ $ k = p.k_{1},l = p.l_{1}$ $ \\implies$\r\n$ k_{1}.l_{1} = q$ $ \\implies$ $ p|q$ $ \\implies$\r\n$ n = p^{\\alpha}.x$ $ (p\\nmid x)$ $ \\implies$\r\n$ (a^{p^{\\alpha})^{x} - 1 = (a - 1)p^{s}}$ $ \\implies$\r\n$ p|x$ contradiction! $ \\implies$ if $ n$ is composite number we get contradiction.", "Solution_3": "it can be easily proved by the Lemma $ v_p(a^n\\minus{}1)\\equal{}v_p(a\\minus{}1)\\plus{}v_p(n)$ :) :D" } { "Tag": [ "probability" ], "Problem": "A coin is flipped, a 6 sided die numbered 1-6 is rolled, and a 10 sided die numbered 0-9 is rolled.\r\nWhat is the probability that the coin is heads and the sum of the numbers on the faces is 8?\r\n[hide]\nprobability coin is heads = 1/2\nthere are 7 ways to have a sum of eight: (1,7) (2,6) (6,2) (3,5) (5,3) (4,4)\nthe chance of roling each of these is 1/6 x 1/10 = 1/60\nthe chance of rolling an eight is 6/60, multiply that by 1/2 to get 6/120\nso the answer is [b]1/20[/b][/hide]", "Solution_1": "Six possibilites on the first die, ten on the second. Sixty in all. Only six combinations sum to eight, so we have $ p_2\\equal{}\\frac{1}{10}$. Of course, $ p_1\\equal{}\\frac{1}{2}$. Looks like the probability is $ \\frac{1}{20}$.", "Solution_2": "Thanks for the problem.", "Solution_3": "I wonder why people post solutions on the first post...\r\n\r\nWouldn't that just allow people to cheat off of it? :maybe: \r\n\r\nHmmm... :|" } { "Tag": [ "MATHCOUNTS" ], "Problem": "Perhaps more realistic. Which individual do you think will win at nationals?\r\nArgh, spelled Neil Gurram wrong, can a mod correct that, please?\r\n\r\n[size=75][color=green][LadyKn1ght: Fixed the spelling][/color][/size]", "Solution_1": "Someone else from CA, TX, MI, or IN <- That's condescending considering Washington tied MI in cutoff and OK tied TX for cutoff.\r\n\r\nI voted Nathan btw but I think these people should also be included in your poll\r\n\r\nSachin Shinde (Indiana) \r\nAnand Oza (Maryland) \r\nMatthew Vengalil (Michigan) \r\nDaniel Li (Virginia)", "Solution_2": "BIff actually does have a good chance of making it. Have you seen some of his recent scores on nats rounds.", "Solution_3": "Obviously he can't include all prospective winners. However, I think the major favorites are Andrew Ardito, Neal Wu, and Nathan Benjamin, for their performance last year.", "Solution_4": "Argh, forgot Karan Batra, too. Hmm.", "Solution_5": "[quote=\"Ignite168\"]Someone else from CA, TX, MI, or IN <- That's condescending considering Washington tied MI in cutoff and OK tied TX for cutoff.\n\nI voted Nathan btw but I think these people should also be included in your poll\n\nSachin Shinde (Indiana) \nAnand Oza (Maryland) \nMatthew Vengalil (Michigan) \nDaniel Li (Virginia)[/quote]\r\n\r\nYou were just taking the first places from those states right?\r\n\r\nMatthew Vengalil wrote 7th in Michigan, and used countdown to gain first place, so there probably isn't a really high chance he'll make countdown, unless that was a choke.", "Solution_6": "[quote=\"kyyuanmathcount\"][quote=\"Ignite168\"]Someone else from CA, TX, MI, or IN <- That's condescending considering Washington tied MI in cutoff and OK tied TX for cutoff.\n\nI voted Nathan btw but I think these people should also be included in your poll\n\nSachin Shinde (Indiana) \nAnand Oza (Maryland) \nMatthew Vengalil (Michigan) \nDaniel Li (Virginia)[/quote]\n\nYou were just taking the first places from those states right?\n\nMatthew Vengalil wrote 7th in Michigan, and used countdown to gain first place, so there probably isn't a really high chance he'll make countdown, unless that was a choke.[/quote]\r\n\r\nNo, Sachin Shinde got 4th, Anand Oza got 3rd or 4th. Daniel Li keeps getting 30 on practice sprints. If I was just taking 1sts, I'd put first place in Texas up there or even 2nd as Kevin Chen was 3rd.", "Solution_7": "Wow, thanks for putting me on your poll.", "Solution_8": "Where is all your self-esteem, Klebian? I see that you did not put yourself up there. (this doesn't include someone else from IN, TX...)", "Solution_9": "[quote=\"Ignite168\"] \nMatthew Vengalil (Michigan) \nDaniel Li (Virginia)[/quote]\r\n^^^ know them in real life.\r\n\r\nThey are both very good compeditors. If Matthew V. makes it to CD, he will surely go far. I don't know Daniel Li's ability in CD, but I am sure he is proficient at it.", "Solution_10": "Yeah, this is true.\r\nI am also pretty good at countdown, but I just choked at state countdown. Usually, I do best for some reason when I am 3rd Place or 4th Place.\r\n\r\nIntrepid Math, did you qualify for nats?", "Solution_11": "I feel so happy to be included in the poll! haha....I only have 2 votes though :( \r\n\r\nIt's going to be an interesting year...", "Solution_12": "[quote=\"samath\"]I feel so happy to be included in the poll! haha....I only have 2 votes though :( \n\nIt's going to be an interesting year...[/quote]\r\n\r\nNow you have 3 :P I just voted for you.", "Solution_13": "Thanks. Yay now I'm in 4th place...not counting other.", "Solution_14": "I'm both None of the Above and Person from MI, TX,indiana so I have 8 votes.", "Solution_15": "Actually, since you are from TX, IN, MI, etc, you can't be none of the above, since that category is \"above\".", "Solution_16": "SOMEONE ACTUALLY VOTED FOR ME!!!\r\n\r\nOh wait that was me.\r\n\r\nI think that we are overrating Neal and Nathan. I know Neal has godly speed, but nathan really needs to sharpen up if he wants to win. I figure i have a decent chance as long as i stop misreading. So, my money's on Neal. Sorry, gurram, never even heard of you. I'd like to meet you though.\r\n\r\n[size=75][color=orange][Around here we don't worship Aphrodite and Zeus and Hermes. And we consider references to their *cough* practices rather uncalled-for. Would you say OM** to a girl you liked in front of your pastor and principal? -Billy][/color][/size]", "Solution_17": "Actually as far as Daniel Li in VA, I've seen him the past 2 years at state. I don't deny his written round abilities, but I personally beat him in the countdown round at state this year. (And I'm not half as good as some of you guys!)", "Solution_18": "[quote=\"aznness\"]SOMEONE ACTUALLY VOTED FOR ME!!!\n\nOh wait that was me.\n\nI think that we are overrating Neal and Nathan. I know Neal has godly speed, but nathan really needs to sharpen up if he wants to win. I figure i have a decent chance as long as i stop misreading. So, my money's on Neal. Sorry, gurram, never even heard of you. I'd like to meet you though.\n\n[size=75][color=orange][Around here we don't worship Aphrodite and Zeus and Hermes. And we consider references to their *cough* practices rather uncalled-for. Would you say OM** to a girl you liked in front of your pastor and principal? -Billy][/color][/size][/quote]\r\n\r\nOverrating? In 7th grade, one of them got #2 written and the other was National Champion. There is no question that they are the big favorites heading into the competition.", "Solution_19": "I voted Gurram because Michigan people are KO0o0OLer than everybody else.\r\n\r\nP.S. How many of those people up there are AoPSers?", "Solution_20": "Everyone but Neal Wu. :D\r\nNathan Benjamin (I_Would_Say)\r\nSam Keller (samath)\r\nAndrew Ardito (hexahedron)\r\nKevin Chen (bionuniquineist (sp?))\r\nNeil Gurram (mathgeniuse^ln(x))\r\nKevin Yang (aznness)", "Solution_21": "Go Nathan!! Woo! :D", "Solution_22": "Hey just wanted to let everyone know they shouldn't base 8th grade performance very much on 7th grade performance..... speaking from personal experience :oops: I got 31st as a 7th grader (5th best for 7th grade), and then got 48th as an 8th grader. It was a bad day... and harder problems than I was expecting.\r\n\r\nThis could also serve as a warning to those contending for countdown round (don't get less than 3 hours of sleep the night before the competition, even if it means using sleeping pills)\r\n\r\nBut don't drink coffee to wake yourself up either. When I was in 7th grade my teammate did that and crashed mid-Target round and only managed to solve 1 Team round.", "Solution_23": "[quote=\"jrshoch\"]Hey just wanted to let everyone know they shouldn't base 8th grade performance very much on 7th grade performance..... speaking from personal experience :oops: I got 31st as a 7th grader (5th best for 7th grade), and then got 48th as an 8th grader. It was a bad day... and harder problems than I was expecting.\n\nThis could also serve as a warning to those contending for countdown round (don't get less than 3 hours of sleep the night before the competition, even if it means using sleeping pills)\n\nBut don't drink coffee to wake yourself up either. When I was in 7th grade my teammate did that and crashed mid-Target round and only managed to solve 1 Team round.[/quote]\r\n\r\nAbsolutely. Always sleep :blush: before the competition. And if you make Nats in 7th grade, you need to know that not many people improve in 8th, judging from Mathcounts and Word Power. \r\n\r\nAll of these examples are from another competition - NOT MC! : From the first year of going to nats to the second (in another competition), one person went from 4th to 7th, 12th to 13th, 7th to 13th, and 11th to not making nats at all. How many people improved enough to get in the top 10 the second year who had returned? Nobody. (Of course, 3rd year people, having gotten disappointed the second year, generally do much much better; in that competition, two different people have made it to nats one year, didn't make the finals or nats the second, and won 3rd the third year.) Remember, this ISN'T from MC results.\r\n\r\nSo don't be overconfident. You are not the best in the world.", "Solution_24": "I would also point out this goes for state competition as well. Last year, a 7th grader placed 5th at our State Competition and was there as an individual. Top 4 were all 8th graders. (including jrshoch who won) This year, that 7th grader's team was there as well and he placed 6th while 7th graders who placed 8th and 11th went to 3rd and 1st respectively and the two other people who made the team were seventh graders.", "Solution_25": "[quote=\"solafidefarms\"][quote=\"jrshoch\"]Hey just wanted to let everyone know they shouldn't base 8th grade performance very much on 7th grade performance..... speaking from personal experience :oops: I got 31st as a 7th grader (5th best for 7th grade), and then got 48th as an 8th grader. It was a bad day... and harder problems than I was expecting.\n\nThis could also serve as a warning to those contending for countdown round (don't get less than 3 hours of sleep the night before the competition, even if it means using sleeping pills)\n\nBut don't drink coffee to wake yourself up either. When I was in 7th grade my teammate did that and crashed mid-Target round and only managed to solve 1 Team round.[/quote]\n\nAbsolutely. Always sleep :blush: before the competition. And if you make Nats in 7th grade, you need to know that not many people improve in 8th, judging from Mathcounts and Word Power. More examples of falling: From the first year of going to nats to the second (in another competition), one person went from 4th to 7th, 12th to 13th, 7th to 13th, and 11th to not making nats at all. How many people improved enough to get in the top 10 the second year who had returned? Nobody. (Of course, 3rd year people, having gotten disappointed the second year, generally do much much better; in that competition, two different people have made it to nats one year, didn't make the finals or nats the second, and won 3rd the third year.)\n\nSo don't be overconfident. You are not the best in the world.[/quote]\r\nI agree, however you are ignoring people who do well after returning. Looking ONLY at 04 and 05 I see that both david and sergei were in lower parts of countdown. Pardha got 30th in 04. Mark Zhang got 40th in 04.\r\nAlmost half of last years countdown made the top 57 in 04. Half of the people making it into countdown are returning. You only named 4 drops in the past many years.", "Solution_26": "[quote=\"pgpatel\"][quote=\"solafidefarms\"][quote=\"jrshoch\"]Hey just wanted to let everyone know they shouldn't base 8th grade performance very much on 7th grade performance..... speaking from personal experience :oops: I got 31st as a 7th grader (5th best for 7th grade), and then got 48th as an 8th grader. It was a bad day... and harder problems than I was expecting.\n\nThis could also serve as a warning to those contending for countdown round (don't get less than 3 hours of sleep the night before the competition, even if it means using sleeping pills)\n\nBut don't drink coffee to wake yourself up either. When I was in 7th grade my teammate did that and crashed mid-Target round and only managed to solve 1 Team round.[/quote]\n\nAbsolutely. Always sleep :blush: before the competition. And if you make Nats in 7th grade, you need to know that not many people improve in 8th, judging from Mathcounts and Word Power. More examples of falling: From the first year of going to nats to the second (in another competition), one person went from 4th to 7th, 12th to 13th, 7th to 13th, and 11th to not making nats at all. How many people improved enough to get in the top 10 the second year who had returned? Nobody. (Of course, 3rd year people, having gotten disappointed the second year, generally do much much better; in that competition, two different people have made it to nats one year, didn't make the finals or nats the second, and won 3rd the third year.)\n\nSo don't be overconfident. You are not the best in the world.[/quote]\nI agree, however you are ignoring people who do well after returning. Looking ONLY at 04 and 05 I see that both david and sergei were in lower parts of countdown. Pardha got 30th in 04. Mark Zhang got 40th in 04.\nAlmost half of last years countdown made the top 57 in 04. Half of the people making it into countdown are returning. You only named 4 drops in the past many years.[/quote]\r\n\r\nOoops, I didn't make it clear I wasn't talking about MC at all in those examples.", "Solution_27": "[quote=\"solafidefarms\"][/quote][quote=\"pgpatel\"][quote=\"solafidefarms\"][quote=\"jrshoch\"]Hey just wanted to let everyone know they shouldn't base 8th grade performance very much on 7th grade performance..... speaking from personal experience :oops: I got 31st as a 7th grader (5th best for 7th grade), and then got 48th as an 8th grader. It was a bad day... and harder problems than I was expecting.\n\nThis could also serve as a warning to those contending for countdown round (don't get less than 3 hours of sleep the night before the competition, even if it means using sleeping pills)\n\nBut don't drink coffee to wake yourself up either. When I was in 7th grade my teammate did that and crashed mid-Target round and only managed to solve 1 Team round.[/quote]\n\nAbsolutely. Always sleep :blush: before the competition. And if you make Nats in 7th grade, you need to know that not many people improve in 8th, judging from Mathcounts and Word Power. More examples of falling: From the first year of going to nats to the second (in another competition), one person went from 4th to 7th, 12th to 13th, 7th to 13th, and 11th to not making nats at all. How many people improved enough to get in the top 10 the second year who had returned? Nobody. (Of course, 3rd year people, having gotten disappointed the second year, generally do much much better; in that competition, two different people have made it to nats one year, didn't make the finals or nats the second, and won 3rd the third year.)\n\nSo don't be overconfident. You are not the best in the world.[/quote]\nI agree, however you are ignoring people who do well after returning. Looking ONLY at 04 and 05 I see that both david and sergei were in lower parts of countdown. Pardha got 30th in 04. Mark Zhang got 40th in 04.\nAlmost half of last years countdown made the top 57 in 04. Half of the people making it into countdown are returning. You only named 4 drops in the past many years.[/quote][quote=\"solafidefarms\"]\n\nOoops, I didn't make it clear I wasn't talking about MC at all in those examples.[/quote] :idea:", "Solution_28": "[quote=\"jrshoch\"]\nBut don't drink coffee to wake yourself up either. When I was in 7th grade my teammate did that and crashed mid-Target round and only managed to solve 1 Team round.[/quote]\r\n\r\nI like coffee. :D Yeah, my friend and I both went to the nationals last year as 7th graders, but this year, my friend got 7th place I think at states. And I dropped from 2nd to 3rd at states. :oops: Oh well.", "Solution_29": "Thanks for not including my name in your list of other potential winners :( Just because MathCounts was mean and didn't post my 44th placing last year on their returning competitors list doesn't mean i didn't get 44th :( :( :(. (Btw, Daniel Li got 47th. I beat him on tiebreaker :D).\r\n\r\nAnyway, both his written and CD skills are good, but I've beaten him at countdown more times than he's beaten me (if I remember correctly), so as long as my written round is good enough to get into CD i'll have a good shot, but i mainly wanna get top 4 for masters round and top 12 for being on TV ;)\r\n\r\nA few other points:\r\n1) Daniel's only getting 30s on practice sprints cuz he's done em all recently and remembers them. There's no way of knowing how he'll do on a new thing considering he's doing better at these nats competitions than the 2006 state one.\r\n2) Everyone from VA is getting 2x on all of their practice sprints with 14/16 on targets. \r\n3) Written isn't everything.\r\n\r\nI voted for someone else meaning someone from VA.", "Solution_30": "[quote=\"lotrgreengrapes7926\"]I like coffee. :D Yeah, my friend and I both went to the nationals last year as 7th graders, but this year, my friend got 7th place I think at states. And I dropped from 2nd to 3rd at states. :oops: Oh well.[/quote]\r\n\r\nSame in Washington. I won again, but he dropped from 3rd to 7th and so he's not on my team. It's because he's not an AOPSer, I bet :D" } { "Tag": [ "AMC", "USA(J)MO", "USAMO", "trigonometry", "inequalities", "inequalities proposed" ], "Problem": "Let $a,b,c$ be non-negative numbers such that \r\n $a^2 + b^2 + c^2 + abc = 4$\r\nFind the least and the greatest values of $a+b+c$", "Solution_1": "I think it is a USAMO question.\r\n\r\nSubsitute $a = 2\\cos{\\alpha}$, $b = 2\\cos{\\beta}$ and $c = 2\\cos{\\gamma}$ where $\\alpha, \\beta, \\gamma$ are the angles of an acute triangle. It is then very easy using the Hardy-Littlewood-Polya majorization inequality. I haven't checked it properly but it seems the values are 2 and 3, am I right?", "Solution_2": "I doubt if the min is 2.\r\nI don't think equality case can be attained.", "Solution_3": "Yes!!\r\n$2 \\le a+b+c \\le 3$\r\nsory, Arne! I'd like to know Hardy-Littlewood-Polya majorization inequality :)", "Solution_4": "[quote=\"minhkhoa\"]Yes!!\n$2 \\le a+b+c \\le 3$\nsory, Arne! I'd like to know Hardy-Littlewood-Polya majorization inequality :)[/quote]\r\n\r\nminhkhoa, here's majorization inequality, http://www.artofproblemsolving.com/Forum/viewtopic.php?t=14975\r\nmay i know when a+b+c=2?", "Solution_5": "Dear Suihochung!\r\n(a,b,c)=(2,0,0) :)", "Solution_6": "[quote=\"minhkhoa\"]Dear Suihochung!\n(a,b,c)=(2,0,0) :)[/quote]\r\n\r\nah! stupid me... :blush:", "Solution_7": "Sorry, this is not a USAMO question.\r\n\r\nThe USAMO question I had in mind is similar:\r\n\r\nIf $a, b, c$ are positive reals such that $a^2 + b^2 + c^2 + abc = 4$, prove that $0 \\leq ab + bc + ca - abc \\leq 2$.", "Solution_8": "[quote=\"Arne\"]\nSubsitute $a = 2\\cos{\\alpha}$, $b = 2\\cos{\\beta}$ and $c = 2\\cos{\\gamma}$ where $\\alpha, \\beta, \\gamma$ are the angles of an acute triangle.[/quote]\r\n\r\nI'm not exactly a stranger to trigonometric identities, but I didn't know this one. \r\n\r\nAre you saying that for a triangle, $\\sum cos^2(\\alpha) + 2 \\Pi cos(\\alpha) = 1$?\r\n\r\n(so that triangles parametrize the $a,b,c$ satisfying the identity)", "Solution_9": "[quote=\"fleeting_guest\"]I'm not exactly a stranger to trigonometric identities, but I didn't know this one. \n\nAre you saying that for a triangle, $\\sum cos^2(\\alpha) + 2 \\Pi cos(\\alpha) = 1$?[/quote]\n\nYes, exactly. We have:\n\n[b]Theorem 1.[/b] Given a triangle with angles A, B, C, then, denoting, x = cos A, y = cos B, z = cos C, we have $x^2+y^2+z^2+2xyz=1$.\n\nThere exists a kind of converse of this theorem for positive numbers and acute-angled triangles:\n\n[b]Theorem 2.[/b] If x, y, z are three positive reals such that $x^2+y^2+z^2+2xyz=1$, then there exists an acute-angled triangle whose angles A, B, C satisfy x = cos A, y = cos B, z = cos C.\n\nI have proven this theorem in http://www.mathlinks.ro/Forum/viewtopic.php?t=23888 post #8.\n\nNow, using Theorem 2, we can prove the original inequality:\n\n[quote=\"minhkhoa\"]Let $a,b,c$ be non-negative numbers such that \n $a^2 + b^2 + c^2 + abc = 4$\nFind the least and the greatest values of $a+b+c$[/quote]\r\n\r\nIn fact, we will show that $2\\leq a+b+c\\leq 3$. Equality in $2\\leq a+b+c$ is obtained for a = 2, b = 0, c = 0, and equality in $a+b+c\\leq 3$ is obtained for a = 1, b = 1, c = 1; thus, the problem of finding the least and the greatest values of a + b + c will be solved.\r\n\r\nIn order to prove $2\\leq a+b+c\\leq 3$, we consider two cases:\r\n\r\n[b]Case 1.[/b] All three numbers a, b, c are positive.\r\n[b]Case 2.[/b] At least one of the numbers a, b, c is 0.\r\n\r\n(In fact, we know that the three numbers a, b, c are nonnegative, so that each of them is either positive or 0.)\r\n\r\nIn Case 1, define the numbers $x=\\frac{a}{2}$, $y=\\frac{b}{2}$, $z=\\frac{c}{2}$. Then, a = 2x, b = 2y, c = 2z, and thus, the equation $a^2 + b^2 + c^2 + abc = 4$ rewrites as $\\left(2x\\right)^2+\\left(2y\\right)^2+\\left(2z\\right)^2+2x\\cdot 2y\\cdot 2z=4$, what simplifies to $x^2+y^2+z^2+2xyz=1$. Furthermore, since the three numbers a, b, c are positive, the numbers $x=\\frac{a}{2}$, $y=\\frac{b}{2}$, $z=\\frac{c}{2}$ are positive, too. Thus, after Theorem 2, there exists an acute-angled triangle whose angles A, B, C satisfy x = cos A, y = cos B, z = cos C. Now, a [url=http://www.mathlinks.ro/Forum/viewtopic.php?t=21188]really well-known inequality[/url] yields that $1<\\cos A+\\cos B+\\cos C\\leq\\frac32$; since $\\cos A=x=\\frac{a}{2}$, $\\cos B=y=\\frac{b}{2}$ and $\\cos C=z=\\frac{c}{2}$, this rewrites as $1<\\frac{a}{2}+\\frac{b}{2}+\\frac{c}{2}\\leq\\frac32$. Multiplication with 2 yields $2 1, then $x^{2}+y^{2}+z^{2}+xyz > 1+1+1+1 = 4$. Contradiction. So $z \\leq 1$. Hence $xy+yz+zx \\geq xy \\geq xyz$.\n\nPut x = u + v, y = u - v, so that $u, v \\geq 0$. Then the equation given becomes $u^{2}\\left(2+z\\right)+\\left(2-z\\right)v^{2}+z^{2}= 4$. So we we keep z fixed and reduce v to nil, then we must increase u. But $xy+yz+zx-xyz = \\left(u^{2}-v^{2}\\right)\\left(1-z\\right)+2zu$, so decreasing v and increasing u has the effect of increasing xy + yz + zx - xyz. Hence xy + yz + zx - xyz takes its maximum value when x = y. But if x = y, then the equation gives $x = y = \\sqrt{2-z}$. So to establish that $xy+yz+zx-xyz \\leq 2$ it is sufficient to show that $2-z+2z\\sqrt{2-z}\\leq 2+z\\left(2-z\\right)$. Evidently we have equality if z = 0. If z is non-zero, then the relation is equivalent to $2\\sqrt{2-z}\\leq 3-z$ or $\\left(z-1\\right)^{2}\\geq 0$. Hence the relation is true and we have equality only for z = 0 or 1.[/quote]\r\n\r\n darij" } { "Tag": [ "inequalities" ], "Problem": "find all values of $ x/x \\plus{} y \\plus{} y/y \\plus{} z \\plus{} z/x \\plus{} y$,x,y,z>0", "Solution_1": "Do you mean $ \\frac{x}{x\\plus{}y}\\plus{}\\frac{y}{y\\plus{}z}\\plus{}\\frac{z}{x\\plus{}y}$?", "Solution_2": "yes. exactly that", "Solution_3": "\\[ \\left(\\frac{x}{x\\plus{}y}\\plus{}\\frac{y}{y\\plus{}z}\\plus{}\\frac{z}{z\\plus{}x}\\right)\\in (1,2)\\]", "Solution_4": "please post your solution :!:", "Solution_5": "You should do the same in the Inequalities Marathon . \r\n\r\n\\[ \\sum_{cyc}\\frac{x}{x\\plus{}y}\\ge \\sum_{cyc}\\frac{x}{x\\plus{}y\\plus{}z}\\equal{}1\\]\r\n\r\nand \\[ \\sum_{cyc}\\frac{x}{x\\plus{}y}\\plus{}\\sum_{cyc}\\frac{y}{x\\plus{}y}\\equal{}3\\] and use the fact that \\[ \\sum_{cyc}\\frac{y}{x\\plus{}y}\\ge 1\\]" } { "Tag": [ "calculus", "trigonometry", "parameterization", "quadratics", "algebra" ], "Problem": "A driver in a car is in the desert, 10 km from the nearest point, $ N$, on a long, straight road. The driver can cover 30 km/h in the desert, and 50 km/h on the road. What is the shortest time for the driver to reach a point on the road that is:\r\n\r\n(a) 20 km from $ N$?\r\n(b) 30 km from $ N$?", "Solution_1": "I would imagine the road is perpendicular to the path from the driver to $ N$?", "Solution_2": "Yes, since the $ N$ is the nearest point on the road, 10 km is the perpendicular distance from the driver to the road.", "Solution_3": "[hide=\"Ugly Solution\"]While on the desert, the path will contain only lines, because the shortest distance between any two points is a line, and once we hit the road, it is the most efficient to continue to the finish line by the Triangle Inequality. Now assume that the path while on the desert is not just one line, but a composition of them. Start from the last two lines; we can make this shorter with only one line by the Triangle Inequality. We can continue like this to show that the shortest desert path to some given point on the road is always a line.\n\nSo let the point that the desert path ends be $ x$ away from $ N$, and thus $ 20 \\minus{} x$ from the finish point. The time taken would then be $ \\frac {\\sqrt {x^2 \\plus{} 100}}{30} \\plus{} \\frac {20 \\minus{} x}{50} \\equal{} \\frac25 \\plus{} \\frac1{150}(5\\sqrt {x^2 \\plus{} 100} \\minus{} 3x)$. Now we want to find the maximum $ k$ such that $ 5\\sqrt {x^2 \\plus{} 100} \\minus{} 3x\\ge k$ for all $ 0\\le x\\le20$. So moving $ 3x$ to the RHS and simplifying yields $ (16x \\minus{} 3k)^2 \\plus{} 25(1600 \\minus{} k^2)\\ge0$. Keeping in mind that $ k$ is positive, if $ k \\equal{} 40$, $ 0\\le x \\equal{} \\frac {120}6 \\equal{} \\frac {15}2\\le20$ is where the minimum occurs, if $ k < 40$, the minimum is not achievable for any $ x$ since $ 1600 \\minus{} k^2 > 0$ so certainly not $ 0\\le x\\le20$, and if $ k > 40$, then for $ 0\\le x \\equal{} \\frac {15}2\\le20$, $ 5\\sqrt {x^2 \\plus{} 100} \\minus{} 3x < k$, which is a contradiction. Thus the minimum time occurs when $ x \\equal{} \\frac {15}2$, which makes the time $ \\frac23$ of an hour or $ 40$ minutes.\n\nSimilarly, for the second part, we want to minimize $ \\frac35 \\plus{} \\frac1{150}(5\\sqrt {x^2 \\plus{} 100} \\minus{} 3x)$. But this obviously also occurs when $ x \\equal{} \\frac {15}2$, giving a time of $ \\frac {13}{15}$ of an hour or $ 52$ minutes.[/hide]\r\n\r\nAhh there should be a nicer solution...", "Solution_4": "[quote=\"math154\"] and if $ k > 40$, then for $ 0\\le x \\equal{} \\frac {15}2\\le20$, $ 5\\sqrt {x^2 \\plus{} 100} \\minus{} 3x < k$, which is a contradiction. [/quote] How was this contradiction proved (without circular reasoning of $ 5\\sqrt{x^{2}\\plus{}100}\\minus{}3x < k$ for $ k>40$ since the maximum $ k$ is $ k\\equal{}40$)?", "Solution_5": "OK, so if $ k > 40$, then by definition, $ 5\\sqrt {x^2 \\plus{} 100} \\minus{} 3x\\ge k > 40$ for all $ 0\\le x\\le20$. But $ x \\equal{} \\frac {15}2$ (which is between $ 0$ and $ 20$) yields $ 5\\sqrt {x^2 \\plus{} 100} \\minus{} 3x \\equal{} 40$, which is less than $ k$, contradicting the role of $ k$.", "Solution_6": "OK I see... for some strange reasons I misunderstood the definition of $ k$... :huh:", "Solution_7": "Now we want to find the maximum $ k$ such that $ 5\\sqrt {x^2 \\plus{} 100} \\minus{} 3x\\ge k$ for all $ 0\\le x\\le20$.\r\n\r\nThis is the first sentence of the solution that I don't understand. Could you please explain it differently? Among other things, I'm not sure why the sentence calls for a maximum when we're trying to minimize the driving time.\r\n\r\nThanks.", "Solution_8": "[quote=\"math154\"]Now we want to find the maximum $ k$ such that $ 5\\sqrt {x^2 \\plus{} 100} \\minus{} 3x\\ge k$ for all $ 0\\le x\\le20$.[/quote]\r\n\r\nThis is the first sentence of the solution that I don't understand. Could you please explain it differently? Among other things, I'm not sure why the sentence calls for a maximum when we're trying to minimize the driving time.\r\n\r\nThanks.", "Solution_9": "Of course. What if we picked, say $ k\\equal{}\\minus{}100$? Clearly all $ 5\\sqrt{x^2\\plus{}100}\\minus{}3x\\ge k$, but it's not [i]achievable[/i], which is what we need for $ k$ to really be a minimum. So we need to find $ k$ such that $ 5\\sqrt{x^2\\plus{}100}\\minus{}3x\\ge k$ for all $ 0\\le x\\le20$, and so that for some $ x$ in the range $ 0\\le x\\le20$, $ 5\\sqrt{x^2\\plus{}100}\\minus{}3x\\equal{}k$. This corresponds to finding the maximum possible $ k$ with these conditions. For example, here $ k\\equal{}40$ is the maximum possible lower bound, achievable when $ 0\\le x\\equal{}7.5\\le20$, because if $ k<40$, then there wouldn't exist any $ x$ in the range achieving the minimum, and if $ k>40$, then when $ x\\equal{}7.5$, we would have $ 5\\sqrt{x^2\\plus{}100}\\minus{}3x\\equal{}40R , $f(x)= \\frac{x^{2}+mx+n }{x^{2}+1}$ , with $m$, $n$ real. find m,n so as $f ( \\mathbb{R})=(1,3]$", "Solution_1": "You have $f(x)>1$ for all $x$. What condition does this impose on $m$?", "Solution_2": "0 :blush:" } { "Tag": [ "vector", "function", "complex analysis", "complex analysis unsolved" ], "Problem": "Could someone help me with this problem:\r\n\r\nf is a conformal mapping from the unit disk onto a convex set.\r\nShow that the image of any disk centered at the origin is convex.", "Solution_1": "[url=http://www.mathlinks.ro/viewtopic.php?t=280607]this[/url] is one of the most beautiful problems in complex analysis I have ever seen.", "Solution_2": "The image of the circle |z|=r is convex iff u(z)= Re(1+zf\"(z)/f'(z)) is non-negative on this circle.\r\n(This is an easy computation: parametrize the circle by r.exp(it) and then write the condition\r\nthat the argument of the tangent vector to the image of the circle is increasing as a function\r\nof t.)\r\nNow u is a harmonic function (because f'(z) is never 0), so if the image of some circle |z|=r\r\nis convex then the image of any smaller circle |z|=r' 0$\r\n\r\nProve that the product $P = {x_1}^{a_1}..{x_n}^{a_n}$ attain max value when $x_i = a_i$.\r\n\r\nNote: $a$ is fixed, $x$ can be changed.", "Solution_1": "just a thought, could you use strong induction on $n$? i am not sure that it would be rigirous through, assume for n and 1, multiply them for n+1", "Solution_2": "That's what I tried, but it looks ugly since I had to used the polynomial expansion. I wonder if there's another way that look better.", "Solution_3": "I tried Jensen's, but it only allows you to conclude $x_n, a_n$ similarly sorted, but they aren't necessarily equal...", "Solution_4": "[quote=\"t0rajir0u\"]I tried Jensen's, but it only allows you to conclude $x_n, a_n$ similarly sorted, but they aren't necessarily equal...[/quote]\r\n\r\nHmm, can you show me a case when they are not equal?", "Solution_5": "Note that $\\ln{P} = \\ln{(x_1^{a_1} \\cdots x_n^{a_n})} = \\sum a_i\\ln{x_i}$.\r\n\r\nTake the concave function $f(x) = \\ln{x}$. Let $\\sum x_i = \\sum a_i = A$ for convenience.\r\n\r\nBy Jensen's with weights $\\frac{a_i}{A}$, we have\r\n\r\n$\\frac{\\ln{P}}{A} = \\sum \\frac{a_i}{A}\\ln{x_i} \\le \\ln{\\left(\\frac{1}{A}\\sum a_ix_i\\right)}$.\r\n\r\nSo it is only necessary to maximize $\\sum a_ix_i$. But by Cauchy,\r\n\r\n$\\sum a_ix_i \\le \\sqrt{(a_1^2+\\cdots+a_n^2)(x_1^2+\\cdots+x_n^2)}$\r\n\r\nwhich has equality at $a_i = x_i$ (because $a_i$'s are constant).\r\n\r\nEDIT: On second thought, since $\\sum x_i^2$ isn't constant, that Cauchy step might not work...\r\n\r\nEDIT2: Maybe this approach will work better... Assume it has a maximum at $x_i = a_i$. Then show that by changing any two terms, we have $(a_j-\\delta)^{a_j}(a_k+\\delta)^{a_k} \\le a_j^{a_j}a_k^{a_k}$. I think we can show this by analyzing the derivative of the function $f(\\delta) = (a_j-\\delta)^{a_j}(a_k+\\delta)^{a_k}$ and showing that it's always negative...\r\n\r\n$f'(\\delta) = -\\ln{a_j} \\cdot (a_j-\\delta)^{a_j} - \\ln{a_k} \\cdot (a_k+\\delta)^{a_k}$ if I did that correctly.", "Solution_6": "Thank you, the problem is finally solved. Taking the derivative is a good approach :)", "Solution_7": "I already considered the 'changing two terms' possibility, but I don't think it works because the maximum might be attainable by changing more than two terms simultaneously... I'm also very suspicious of the Cauchy solution because I don't think you can show that the Jensen's and Cauchy parts have the same maximum case. :(\r\n\r\nLaputa: For example $x_1 = 1, x_2 = 3, x_3 = 6$ and $a_1 = 1, a_2 = 2, a_3 = 7$ have the same sum and are similarly sorted, but $x_i \\neq a_i$.\r\n\r\nEdit: [b]WAIT.[/b] The problem as stated is incomplete! In the above example, $\\sum x_i = \\sum a_i = 10$ and we are supposed to show that the maximum comes for $x_i = a_i$, however, $1^1 \\times 3^2 \\times 6^7 > 1^1 \\times 3^3 \\times 6^6$ ! There has to be some extra condition. (In fact, the maximum comes when $x_1 = x_2 = 1, x_3 = 8$ and $a_1 = a_2 = 1, a_3 = 8$, I think. But even so, now the wording of the problem confuses me.)", "Solution_8": "I don't think you're understanding the problem quite correctly... The given $a_i$ are FIXED. As in numbers. Then we want to show to maximize $P$ we have to set all the $x_i = a_i$.\r\n\r\nFor instance, if $a_1 = 1, a_2 = 2, a_3 = 7$ as in your example, then we want to show that $x_1 = 1, x_2 = 2, x_3 = 7$. I think it's true.\r\n\r\nI'm pretty sure the Cauchy/Jensen solution doesn't work; I'm still working out the specifics of the derivative solution...\r\n\r\nMaybe if we take $f(\\delta) = \\prod (a_i-m_i\\delta)^{a_i}$ for constants $m_i$ such that $\\sum m_i = 0$. That should cover all possibilities. The derivative of that is a beast, but I think it's always negative (of course given the condition that $a_i > m_i\\delta$).", "Solution_9": "Well, if you're going to use derivatives, it's easiest to use partial derivatives having in mind the given conditions. Then you directly get $x_i=a_i.$", "Solution_10": "Couldn't we use the rearrangemente inequality between base and exponent, saying that P is maximum when $x_i>x_j\\rightarrow a_i>a_j$?", "Solution_11": "There's actually an easy proof of this. You're given a sum, and you're trying to maximize a product. Does anything come to mind?", "Solution_12": "Apply the weighted AM-GM inequality to this?", "Solution_13": "I have a solution like this, waiting for any comments:\r\n\r\n[b]1. [/b] Case ${a^a}{b^b}$\r\n\r\nConsider the the function $f(x) = {a*ln(a-x) + b*ln(b+x)}$ with $0 \\leq x \\leq a$\r\n\r\nwe want to maximize the product so $a*ln(a-x) + b*ln(b+x)$ should be maximized as well.\r\n\r\n$f'(x) = \\frac{b}{b+x} - \\frac{a}{a-x}$\r\n\r\nSince $\\frac{b}{b+x} \\leq 1$ & $\\frac{a}{a-x}\\geq 1, f'(x) \\leq 0$, so $x = 0$ to attain the maximum value.\r\n\r\n[b]2.[/b] Consider the case of of ${a^a}{b^b}{c^c}$:\r\n\r\nlet $f(x_1, x_2) = {a^a}{b^b}{c^c} - {(a-x_1-x_2)^a}{(b+x_1)^b}{(c+x_2)^c}$\r\n\r\nSo $f(x_1, x_2) = [{a^a}{b^b}{c^c} - {(a-x_1)^a}{(b+x_1)^b}{c^c}] + [{(a-x_1)^a}{(b+x_1)^b}{c^c} - {(a-x_1-x_2)^a}{(b+x_1)^b}{(c+x_2)^c}]$\r\n\r\nFrom 1. above, we know that the derivative of the first square bracket is $\\leq 0$, so consider the second bracket:\r\n\r\nLet $a\\\" = a-x_1$ then\r\n\r\n$(2nd Bracket) = {(b+x_1)^b}[{(a\\\")^a}{c^c} - {(a\\\"-x_2)^a}{(c+x_2)^c}]$\r\n\r\nAgain, consider the function $g(x_2) = a*ln(a\\\"-x_2) + c*ln(c+x_2)$\r\n\r\n$g'(x) = \\frac{c}{c+x} - \\frac{a}{a\\\"-x}$ with $0 \\leq x \\leq a\\\"$\r\n\r\nSince $a = a\\\" + x \\geq a\\\"-x$, $\\frac{a}{a\\\"-x} \\geq 1$ while $\\frac{c}{c+x} \\leq 1$ so $g'(x) \\leq 0$.\r\n\r\n--> The derivative of $f(x_1, x_2) \\leq 0$\r\n\r\nSo we can conclude that if we decrease from one variable and increase in other variables then the product will be less than before. So this problem is proven.", "Solution_14": "Note: Please put any comment on my proff on this page:\r\n\r\n[url]http://www.artofproblemsolving.com/Forum/viewtopic.php?p=419269#p419269[/url]", "Solution_15": "[quote=\"zeus_three\"]Apply the weighted AM-GM inequality to this?[/quote]\r\n\r\nYes." } { "Tag": [ "algebra", "difference of squares", "special factorizations", "AMC" ], "Problem": "How many right triangles have integer leg lengths $ a$ and $ b$ and a hypotenuse of length $ b\\plus{}1$, where $ b<100$?\r\n\r\n$ \\textbf{(A)}\\ 6 \\qquad\r\n\\textbf{(B)}\\ 7 \\qquad\r\n\\textbf{(C)}\\ 8 \\qquad\r\n\\textbf{(D)}\\ 9 \\qquad\r\n\\textbf{(E)}\\ 10$", "Solution_1": "[hide]the difference of squares:\nbetween b^2 and (b+1)^2\n(a+b)(a-b)\na-b is always 1\na+b=2b+1\n\na^2+b^2=(b+1)^2\n\nso basically... a^2=2b+1\nyou just have to find all the b's possible where 2b+1=a square number... 2b+1 = odd number so all the odd squares from 3-199 work.\n\n3^2,5^2,7^2,9^2,11^2,13^2\n\nthat's 6\nAns:A[/hide]", "Solution_2": "[hide=\"Solution\"]\n(Inspired by one of the Proofs Without Words things on the AoPS sidebar)\n\nSay $ b$ is the side length of a square, and the hypotenuse, $ b \\plus{} 1$, is the side length of another square.\n\nSince $ (b \\plus{} 1)^2 \\minus{} b^2 \\equal{} a^2$,\n\n$ \\frac{a^2 \\minus{} 1}{2} \\equal{} b$.\n\nNow all we have to do is determine for which positive values of $ a$ that $ 0 < b < 100$\n\nNote that $ a$ needs to be an odd number for $ b$ to be an integer\n\n$ 3 \\leq a \\leq 13$\n\n$ \\frac{13 \\minus{} 3}{2} \\plus{} 1 \\equal{} \\boxed{6}$, so $ A$.\n[/hide]", "Solution_3": "[hide=\"Lucky Way\"]\nIf you know some of your pythagorean triplets, then you know:\n3, 4, 5\n5, 12, 13\n7, 24, 25\nif you watch, the numbers seem increasing by a significant amount so I assumed that there would be no more than 6, which is the smallest answer choice.\nthe next ones would somewhere around 50-55 for b and then 70-75, and then 90-95\n(as a guess)\ntoo bad I didn't realize this simple and lucky way DURING the test :wallbash: \n[/hide]", "Solution_4": "[quote=\"madshock\"][hide=\"Lucky Way\"]\nIf you know some of your pythagorean triplets, then you know:\n3, 4, 5\n5, 12, 13\n7, 24, 25\nif you watch, the numbers seem increasing by a significant amount so I assumed that there would be no more than 6, which is the smallest answer choice.\nthe next ones would somewhere around 50-55 for b and then 70-75, and then 90-95\n(as a guess)\ntoo bad I didn't realize this simple and lucky way DURING the test :wallbash: \n[/hide][/quote]\r\n\r\nWow, great solution!", "Solution_5": "[hide]$ a^2\\plus{}b^2 \\equal{} b^2\\plus{}2b\\plus{}1 \\implies a^2 \\equal{} 1\\plus{}2b$, where $ 0 0)$ prove that $ x^2 \\plus{} y^2 \\plus{} z^2\\ge\\sqrt {2}(xy \\plus{} yz)$\r\n2.$ (c\\ge b\\ge a\\ge 0)$ prove that $ (a \\plus{} 3b)(b \\plus{} 4c)(c \\plus{} 2a)\\ge 60abc$", "Solution_1": "to 1. It's easy! \r\n$ L\\equal{}(x^2\\plus{}\\frac{y^2}{2})\\plus{}(\\frac{y^2}{2}\\plus{}z^2)\\ge \\sqrt{2}(xy\\plus{}yz)\\equal{}R$ (by AM-GM)\r\nIt's good!\r\n\r\n\r\nbut 2.?", "Solution_2": "[quote=\"sxgfly\"]to 1. It's easy! \n$ L \\equal{} (x^2 \\plus{} \\frac {y^2}{2}) \\plus{} (\\frac {y^2}{2} \\plus{} z^2)\\ge \\sqrt {2}(xy \\plus{} yz) \\equal{} R$ (by AM-GM)\nIt's good!\n\n\nbut 2.?[/quote]\r\n\r\n\r\n\r\nit is done by AM-GM and the fact that $ c\\ge b\\ge a\\ge 0$, $ LHS \\ge 60 a^{\\frac{11}{12}}b^{\\frac{19}{20}}c^{\\frac{17}{15}} \\ge 60 abc$", "Solution_3": "http://www.mathlinks.ro/Forum/viewtopic.php?search_id=1289497002&t=104592\r\n\r\nhttp://www.mathlinks.ro/Forum/viewtopic.php?search_id=1289497002&t=208321" } { "Tag": [ "geometry", "3D geometry", "tetrahedron", "circumcircle", "geometry unsolved" ], "Problem": "Let $ ABCD$ is a tetrahedron and let $ AB\\equal{}a, CD\\equal{}b, AC\\equal{}AD\\equal{}BC\\equal{}BD\\equal{}1$. Find radius of the circumsphere of $ ABCD$.", "Solution_1": "There are \"ready made\" expressions for that question but, once the tetrahedron is not \"very irregular' since all the faces are isosceles, we'll try to make it from basics. \r\n\r\nThe center of the circumsphere, $ O$ , will be the intersection of the perpendiculars $ JO$ and $ KO$ to the faces $ BCD$ and $ ACD$ , with $ J$ and $ K$ as their circumcenters, in the medians $ BM$ and $ AM$ of length $ h \\equal{} \\sqrt {1 \\minus{} c^2}$ where $ c \\equal{} {b\\over2}$.\r\nThe distance from $ J$ and $ K$ to $ M$ , is $ y \\equal{} {{h^2 \\minus{} c^2}\\over{2h}}$.\r\nProlonging $ MO$ it'l meet $ BA$ , perpendicularly , at it's center $ N$ , so that the rectangular triangles, $ JMO$ and $ MNB$ are similar, and the dinstance between $ M$ and $ O$ is $ x \\equal{} {{hy}\\over{\\sqrt {h^2 \\minus{} d^2}}} \\equal{} {{h^2 \\minus{} c^2}\\over{2\\sqrt {h^2 \\minus{} d^2}}}$, here $ d \\equal{} {a\\over2}$.\r\n\r\nNow the distance from $ O$ to any vertex will be the circumradius; for instance $ R \\equal{} \\sqrt {c^2 \\plus{} x^2} \\equal{} {{4(h^2 \\minus{} d^2)c^2 \\plus{} {(h^2 \\minus{} c^2)}^2}\\over{4(h^2 \\minus{} d^2)}}$." } { "Tag": [ "puzzles" ], "Problem": "Below are a number of statements: \r\n1.Precisely one of these statements is untrue. \r\n2. Precisely two of these statements are untrue. \r\n3. Precisely three of these statements are untrue. \r\n4. Precisely four of these statements are untrue. \r\n5 .Precisely five of these statements are untrue. \r\n6. Precisely six of these statements are untrue. \r\n7. Precisely seven of these statements are untrue. \r\n8. Precisely eight of these statements are untrue. \r\n9. Precisely nine of these statements are untrue. \r\n10. Precisely ten of these statements are untrue. \r\n \r\n\r\n\r\n Which of these statements is true?", "Solution_1": "since untrue=false,[hide]only the ninth one is true[/hide]", "Solution_2": "[hide=\"Explanation\"]\n\n Firstly, its impossible for there to be 0 untrue statements, it would mean everyone is telling the truth, a contradiction. Now, we know that the number of untrue statements varies from:\n\n1 - 10\n\n We also know one and only one number of untrue statements is correct. Thus, only one statement is true, and the other nine are false. Which means the 9th statement is true. [/hide]", "Solution_3": "But [u][i][b]noooooooooooooooooooooooo[/b][/i][/u]. The statement that \"below are a number of statements\" is true. Therefore there are eight untrue statements.", "Solution_4": "[quote=\"mathnerd314\"]But [u][i][b]noooooooooooooooooooooooo[/b][/i][/u]. The statement that \"below are a number of statements\" is true. Therefore there are eight untrue statements.[/quote]\r\n\r\nNice try, but there's still 9 untrue statements ;)", "Solution_5": "Oh. So... uh... what's the correct answer?!", "Solution_6": "[quote=\"mathnerd314\"]So... uh... what's the correct answer?![/quote]\r\n\r\n#9." } { "Tag": [ "arithmetic sequence" ], "Problem": "What is the value of $ 100\\minus{}81\\plus{}64\\minus{}49\\plus{}36\\minus{}25\\plus{}16\\minus{}9\\plus{}4\\minus{}1$?", "Solution_1": "hello, we have $ \\sum_{i\\equal{}1}^{10}(\\minus{}1)^i\\cdot i^2\\equal{}55$.\r\nSonnhard.", "Solution_2": "Which doesn't really explain the solution.\r\n\r\nConsider the terms in pairs.\r\n\r\n19+15+11+7+3, this is an arithmetic sequence with 5 terms and a middle term of 11. Therefore the sum is 5*11=55.", "Solution_3": "hello, this will explain my solution it comes from the sum $ \\sum_{i\\equal{}1}^n(\\minus{}1)^i\\cdot i^2\\equal{}\\frac{1}{2}(\\minus{}1)^nn(n\\plus{}1)$.Sonnhard." } { "Tag": [ "geometry", "3D geometry" ], "Problem": "This is from ASHME 1954 (also problem #80 chapt. 4 in aops book 1) \r\n\r\nIf x varies as the cube of y, and y varies as the fifth root of z, then x varies as the nth power of z. what is n?\r\n\r\ndoes varies mean that x is inversly proportional to y? what does varies mean. \r\n\r\nThanks", "Solution_1": "hmmm... I think it means x is proportional to y^3, etc.\r\n[hide=\"so here is my solution\"]$x=ky^3; y=nz^{1/5} \\implies x=k\\cdot n^3 \\cdot z^{3/5} = mz^{3/5}$ So x varies as the 3/5 power of z, I belive[/hide]", "Solution_2": "o i get it! thanks" } { "Tag": [ "group theory", "abstract algebra", "superior algebra", "superior algebra solved" ], "Problem": "Prove that for any group $ G, \\, \\, |G/Z(G)| \\neq 91.$ \r\n\r\n[hide=\"The hint in the book says ...\"] that assuming $ |G/Z(G)| = 91$, prove that $ G/Z(G)$ is cyclic.[/hide]", "Solution_1": "if $ H$ is a group of order $ 7*13$, sylow theorems yield normal subgroups of order $ 7$ and $ 13$. they have to intersect trivially, thus their product is $ H$. then $ H \\cong \\mathbb{Z}/7 \\times \\mathbb{Z}/13 \\cong \\mathbb{Z}/7*13$, i.e. $ H$ is cyclic.\r\n\r\nif $ G/Z(G)$ is cyclic, then $ G$ is abelian (easy exercise), i.e. $ G/Z(G)$ is trivial.", "Solution_2": "How does one get $ H \\cong \\mathbb{Z}/7 \\times \\mathbb{Z}/13 \\cong \\mathbb{Z}/91$?\r\nThanks.", "Solution_3": "well think over it, then the only non-obviousl fact is that the elements of the two normal subgroups commute with each other. this comes from a general fact:\r\n\r\nlet $ N,M$ be normal subgroups of a group $ G$ with trivial intersection. then $ mn=nm$ for all $ m \\in M, n \\in N$ (and thus $ NM \\cong N \\times M$).\r\n\r\nthis follows from $ mnm^{-1}n^{-1}\\in M \\cap N$." } { "Tag": [ "linear algebra" ], "Problem": "Given $ \\pi_{v} \\in L(V \\oplus W, V)$, $ S \\in L(U,V)$, and $ T \\in L(U,W)$, show that there is a natural linear transformation $ R \\equal{} S \\times T \\in L(U, V \\oplus W)$ such that $ \\pi_{v} \\circ R \\equal{} S$. Also, $ \\pi_{v}(x,y) \\equal{} x$, where $ x$ and $ y$ are vectors.\r\n\r\nI proceeded with this problem by toying around with $ S$ and $ T$. Since we know that $ S: U \\longrightarrow V$ and $ T: U\\longrightarrow W$ then i just wrote \r\n\r\n$ S(u) \\equal{} ( v_{1},v_{2})$ and $ T(u) \\equal{} (w_{1}, w_{2}).$ where $ u \\in U$ and $ (v_{1}, v_{2}) \\equal{} v \\in V$ and similarly with $ w\\in W$\r\n\r\nSince i m trying to construct $ R: U \\longrightarrow V \\oplus W$ I added $ S(u)$ and $ T(u)$ to get \r\n\r\n$ R \\equal{} S(u) \\plus{} T(u) \\equal{} (v_{1} \\plus{} w_{1}, v_{2} \\plus{} w_{2})$.\r\n\r\nthen to show $ \\pi_{v} \\circ R \\equal{} S$, I simply applied $ \\pi_{v}$ on $ R$.Thus,\r\n\r\n$ \\pi_{v}(R) \\equal{} \\pi_{v}(v_{1} \\plus{} w_{1}, v_{2} \\plus{} w_{2}) \\equal{} v_{1} \\plus{} w_{1} \\in S(u)$???\r\n\r\nI m new to linear transformations, so can anyone tell me how to actually do this correctly? better yet, if it's all wrong, please tell me the best way to start. Thanks.", "Solution_1": "it's all wrong (remember the definitions), you have to define R(u)=(S(u),T(u))." } { "Tag": [ "Asymptote", "algebra", "function", "domain", "vector", "LaTeX" ], "Problem": "(Maybe this belongs in a lower level of mathematics but still found it in a high school textbook)\r\n\r\nWhen we are required to draw graphs (e.g. using excel), do we have to show all the\r\n-asymptotes\r\n-breaking points in the graph if it is not continuous at at least one point?\r\n\r\nIf so how much should we show of each? If the graph is starting to curve down at one side and up the other side at a certain value, can we just show the start and say that it is an asymptote?\r\n\r\nAlso if the graph stops at a point, (i.e. Domain of the function being drawn is x<10) Do we have to draw the graph upto x=10 or upto x=10.1 to show that the graph in discontinuous at x=10?", "Solution_1": "Usually it depends on the requirement. What are the graphs for? A class assignment? Personal clarification? \r\n\r\nIf you wish to leave out the asymptote, you may, but you'll have to note somewhere that there is an asymptote and whatever value of $ x$ or $ y$. Generally, you should always display holes as a simple open circle in your graph.", "Solution_2": "but can we actually show those using excel and nothing else?", "Solution_3": "I'm not sure; I'm terrible at Excel other than graphing simple data graphs for science labs. \r\n\r\nYou can however, learn [url=http://www.artofproblemsolving.com/Wiki/index.php/Asymptote_(Vector_Graphics_Language)]Asymptote[/url], which is a programming language similar to LaTeX, in which you can easily express asymptotes and such on graphs." } { "Tag": [ "inequalities" ], "Problem": "Let $ a,b,c$ be positive real numbers. Prove that:\r\n$ \\sum_{cyclic}\\frac{a^3}{a^3\\plus{}b^3\\plus{}abc} \\ge 1$\r\n\r\n\r\n\r\n\r\n\r\n\r\n\r\n\r\n\r\n\r\n\r\n-Nguyen Van Thach", "Solution_1": "I can't find any better way than expanding :( \r\nExpanding, we get\r\n$ a^6b^3 \\plus{} b^6c^3 \\plus{} c^6a^3 \\ge a^5b^2c^2 \\plus{} a^2b^5c^2 \\plus{} a^2b^2c^5$\r\nLet $ a^2b \\equal{} x$, $ c^2a \\equal{} y$, $ b^2c \\equal{} z$, then this is equivalent to\r\n$ x^3 \\plus{} y^3 \\plus{} z^3 \\ge x^2y \\plus{} y^2z \\plus{} z^2x$\r\nBut $ x^3 \\plus{} x^3 \\plus{} y^3 \\ge 3x^2y$, so adding this cyclically gives the desired result.", "Solution_2": "Normalize to $ abc\\equal{}1,a^3\\equal{}x,b^3\\equal{}y,c^3\\equal{}z,xyz\\equal{}1$\r\n\r\n$ \\sum \\frac{x}{x\\plus{}y\\plus{}1}\\ge 1$\r\n\r\nThis is better for expanding, get to similar as dgreen, and finish it off.", "Solution_3": "[quote=\"dgreenb801\"]I can't find any better way than expanding ....[/quote]\r\nthis what i did substitute $ x\\equal{}\\frac{a}{b}$ and similarly define $ x,y,z$ the inequality becomes direct cauchy also i think it can be solved by writing the one in the RHS as $ \\frac{a^3\\plus{}b^3\\plus{}c^3}{a^3\\plus{}b^3\\plus{}c^3}$ and taking it to the LHS and using sos.", "Solution_4": "[hide=\" solution\"]\n\nthe $ LHS$ is homogeneous so let $ a^3b^3c^3 \\equal{} 1$\n\nlet $ a^3 \\equal{} \\frac {x}{y}, b^3 \\equal{} \\frac {y}{z}, c^3 \\equal{} \\frac {z}{x}$\n\ntherefore\n$ \\sum_{cyclic}\\frac {a^{3}}{a^{3} \\plus{} b^{3} \\plus{} abc} \\equal{} \\sum_{cyclic}\\frac {xz}{xz \\plus{} y^2 \\plus{} yz} \\equal{} \\sum_{cyclic}\\frac {(xz)^2}{(xz)^2 \\plus{} xy^2z \\plus{} xyz^2}$\n\nNo by cauchy\n\n$ (xy \\plus{} yz \\plus{} zx)^2 \\left(\\sum_{cyclic}\\frac {(xz)^2}{(xz)^2 \\plus{} xy^2z \\plus{} xyz^2} \\right) \\ge (xy \\plus{} yz \\plus{} zx)^2$\n\nand we're done\n\n[/hide]" } { "Tag": [ "group theory", "abstract algebra", "superior algebra", "superior algebra unsolved" ], "Problem": "Let $H$ be a subgroup of $G$ with $h$ elements. Suppose $\\exists a\\in G,\\ \\forall x\\in H\\ (xa)^{3}=1$. Let $P=\\{x_{1}ax_{2}a\\dots x_{n}a|x_{i}\\in H, n\\in \\mathbb N\\}$. Find an upper bound for $|P|$.\r\nPS This is a simple problem. It might be helpful if you consider the case $G$ being abelian first.", "Solution_1": "Ok, here is the final [hide=\"answer\"]$3h^{2}$[/hide].Now, any idea?" } { "Tag": [ "geometry", "circumcircle", "parallelogram" ], "Problem": "[color=red]Three equal circles with a comon point H, are intersecting two by two in the points A,B,C.Prove that the circumcircle of the triangle ABC is equal with the three circles mentioned.(This problem is known as the Titeica's theorem[/color])\r\n[i]Proof 1[/i] $ BH$ is the segment of intersection $ C(O_3,R)$ and $ C(O_1,R)$. The triangle $ O_3BH$ is isoscel, and alsow $ O_1BH$ is isoscel,and results that $ O_3BO_1H$ romb$ \\Longrightarrow$ and therefore $ O_3B_1\\equal{}O_1B_1$ where $ B_1$ is the midle of the segment $ BH$.In the same way $ O_2HCO_1$ romb $ \\Longrightarrow$ $ O_2C_1\\equal{}O_1C_1$ where $ C_1$ is the midle of the segment $ CH$,and $ O_2A_1\\equal{}O_3A_1$. $ C(O_1)\\equal{}C(O_2)$ and $ C(O_1) \\cap C(O_2)\\equal{}CH$ $ \\Longrightarrow$ $ AB$ is the comon tangent to this circles $ \\Longrightarrow$ $ O_1B \\bot AB$ , $ O_2A \\bot AB$ \\Longrightarrow AO_2O_1B$ is a paralelogram with$$=$ $ where $ X,Y\\in{M_{n,1}(C)}$ , $ A^*$ is the adjunct matrix(the transpose of the conjugate), and$ $ is the scalar product of $ 2$ column vectors.\r\nIn the both case for the demonstration $ Y=X$", "Solution_2": "2) should say that if $ A\\equal{}\\minus{}A^T,$ then $ \\lambda\\equal{}i\\mathbb{R}$ - in other words, all of the eigenvalues are pure imaginary.\r\n\r\nSaying $ \\mathbb{C}\\setminus\\mathbb{R}$ is simultaneously too weak - \"pure imaginary\" is (for the most part) a much stronger condition than \"not real\" - while at the same time being incorrect, since an antisymmetric matrix could have $ 0$ as an eigenvalue, and $ 0$ is real.", "Solution_3": "Another approach would be the following : \r\n\r\n[hide=\"Notations\"]\nLet $ z \\equal{} a \\plus{} ib \\in \\mathbb{C}$ , $ a,b \\in \\mathbb{R}$ .I denote $ \\overline{z} \\equal{} a \\minus{} ib$.\nIf $ X \\in \\mathcal{M}_n(\\mathbb{C})$ , $ X \\equal{} (x_{i,j})_{1 \\leq i,j \\leq n }$ .By $ \\overline{X}$, I understand $ \\overline{X} \\equal{} ( \\overline{x}_{i,j})_{1 \\leq i,j \\leq n}$\n[/hide]\r\n\r\n1.\r\n\r\nLet $ \\lambda$ be an eigenvalue of $ A$ and $ X \\in \\mathcal{M}_{n,1}(\\mathbb{C})$ be the corresponding eigenvector. \r\nFrom $ AX \\equal{} \\lambda X$ we get $ \\overline{A}\\cdot \\overline{X} \\equal{} \\overline{\\lambda}\\cdot \\overline{X}$ $ \\Rightarrow$ \r\n$ \\overline{X}^T \\cdot \\overline{A}^T \\equal{} \\overline{\\lambda}\\cdot \\overline{X}^T$ \r\nBut because $ A\\in \\mathcal{M}_n(\\mathbb{R})$ we have that $ \\overline{A} \\equal{} A$ ,therefore $ \\overline{A}^T \\equal{} A^T \\equal{} A$ .(the last equality is from our hypothesis i.e. $ A^T \\equal{} A$)\r\nSo $ \\overline{X}^T \\cdot A \\equal{} \\overline{\\lambda}\\cdot \\overline{X}^T$ $ \\Rightarrow$ \r\n$ \\overline{X}^T \\cdot AX \\equal{} \\overline{\\lambda}\\cdot \\overline{X}^TX$ $ \\Leftrightarrow$ \r\n$ \\overline{X}^T \\cdot \\lambda X \\equal{} \\overline{\\lambda}\\cdot \\overline{X}^TX$ and from here we conclude that $ \\lambda \\equal{} \\overline{\\lambda}$ $ \\Rightarrow$ $ \\lambda \\in \\mathbb{R}$\r\n\r\nWe can prove the second part analogously." } { "Tag": [ "trigonometry", "complex numbers" ], "Problem": "Prove that cos 36 - cos 72 = 0.5, and find the numerical values of cos 36 and cos 72. I couldn't solve this problem without looking at the answers. :wink:", "Solution_1": "[quote=vishalarul]Prove that cos 36 - cos 72 = 0.5[/quote]\r\n\r\n\r\nMaybe use complex numbers.\r\n :)", "Solution_2": "[hide]\nConsider the 5th roots of unity. They are of the form $\\cos{\\frac{2k\\pi}{5}}+i\\sin{\\frac{2k\\pi}{5}}$, and we're looking for $\\cos{\\frac{2\\pi}{5}}$.\n\nWe find them by solving $x^{5}-1=0$. Find, divide by $x-1$ to get $x^{4}+x^{3}+x^{2}+x+1=0$. Then, divide by $x^{2}$ to get $x^{2}+x+1+\\frac{1}{x}+\\frac{1}{x^{2}}=0$. Let $w: =x+\\frac{1}{x}$ This becomes $w^{2}+w-1=0$, so $w=\\frac{-1\\pm\\sqrt{5}}{2}=-\\phi,\\frac{1}{\\phi}$.\n\nIf we want x, we know that $x^{2}-wx+1=0$ or $x=\\frac{w \\pm \\sqrt{w^{2}-4}}{2}$. The real part, which is $\\cos{\\frac{2k\\pi}{5}}$ (depending on the root), is $\\frac{w}{2}$. If we want $\\cos{72}$, we use the positive value for w, so $\\cos{72}=\\frac{\\sqrt{5}-1}{4}=\\frac{1}{2\\phi}$.\n\nWe know that $\\cos{36}$ is a positive number such that $\\cos^{2}{36}-1=\\cos{72}=\\frac{1}{2\\phi}$.\n\nWe see that $\\cos{72}+\\frac{1}{2}= \\frac{1}{2\\phi}+\\frac{1}{2}=\\frac{1}{2}(\\frac{1}{\\phi}+1)=\\frac{\\phi}{2}$. \nThen, $2(\\cos{72}+\\frac{1}{2})^{2}-1=2\\frac{\\phi}{2}^{2}-1=\\frac{\\phi^{2}}{2}-1=\\frac{\\phi+1}{2}-1=\\frac{\\phi-1}{2}=\\cos{72}$.\nSo $\\cos{36}-\\cos{72}=\\frac{1}{2}$.\n[/hide]", "Solution_3": "[hide=\"Nice solution for cos36-cos72:\"]\n$\\cos(36)=2\\cos^{2}(18)-1=1-2\\sin^{2}(18)=1-2\\cos^{2}(72)$\n$\\cos(72)=2\\cos^{2}(36)-1$\n$\\cos(36)+\\cos(72)=2(\\cos^{2}(36)-\\cos^{2}(72))$\n$\\frac12=\\cos(36)-\\cos(72)$[/hide]", "Solution_4": "Nice solutions. The first solution was an ingenious example of using the roots of unity, while the other was similar to the solution in the answer key. Thanks. :lol:" } { "Tag": [ "search", "number theory theorems", "number theory" ], "Problem": "Could you tell me if $ \\left(a,b,c\\right)=\\left(m^2 + n^2 - p^2 - q^2,2\\left(mp + nq\\right),2\\left(mp - nq\\right)\\right)$ and any cyclyc permutation and $ d=m^2 + n^2 + p^2 + q^2$ for integer $ m,n,p,q$ are all integer solutions of $ a^2 + b^2+ c^2 = d^2$?", "Solution_1": "Yes. See for example [url=http://en.wikipedia.org/wiki/Pythagorean_quadruple]the Wikipedia article[/url] (interestingly, the Mathworld article does not contain the complete parameterization). I gave a sketch of one proof [url=http://www.artofproblemsolving.com/Forum/viewtopic.php?search_id=729475814&t=176802]here[/url] and scorpius119 gave another which is quite nice." } { "Tag": [ "percent", "geometry", "3D geometry" ], "Problem": "What percent of the volume of a $ 10 \\times 10 \\times 10$ box can be filled with $ 4 \\times 4 \\times 4$ wooden cubes? Express your answer as a decimal to the nearest tenth.", "Solution_1": "Does this have to be as many as possible? Or is this just $ \\frac{4^3}{10^3}$? If the latter is correct, then the answer is $ \\boxed{25.6}$.", "Solution_2": "We can fill it up to 8x8x8=512/1000=51.2%", "Solution_3": "Why 8x8x8? I think ernie's correct.", "Solution_4": "Maybe because it says cube[b]s[/b].", "Solution_5": "Think about it. With 4x4x4, you can fill it to axbxc , but a b and c must be multiples of 4.", "Solution_6": "Mew's right. You can definitely fit 8x8x8 into a 10x10x10x :P", "Solution_7": "[quote=\"AIME15\"][size=200][b]Mew's right.[/size][/b] You can definitely fit 8x8x8 into a 10x10x10x :P[/quote]\r\n\r\nNow that's not something you see every day....", "Solution_8": ":spam: mew....does post count really matter that much? And yes, you don't see that every day, especially from MOI\r\n\r\n\r\nErnie, the reason why your wrong is it says What percent of the volume of a 10x10x10 box CAN BE FILLED, so it implies it's looking for the MAXXXXX (too much caffeine!)", "Solution_9": "I don't suppose you guys read the stickies too often, do you? I suggest you do so.\r\n\r\nLocked." } { "Tag": [ "inequalities", "inequalities proposed" ], "Problem": "If $a,b,c$ are nonnegative numbers such that $a\\le b\\le c$ and $ab+bc+ca=3$, then\r\n\r\n$\\frac 1{2a+b}+\\frac 1{2b+c}+\\frac 1{2c+a}\\ge 1$.", "Solution_1": "Thank you, VASC,\r\n\r\nBy my result post in your topic\r\n\r\nhttp://www.mathlinks.ro/Forum/viewtopic.php?t=144140\r\n\r\nI refer that it suffices to consider the inequality in case $\\min(a,b,c)=0$. Moreover, I already proved that for all $k\\ge 1$ and $a\\ge b\\ge c$ then\r\n\r\n\\[\\frac{1}{ka+b}+\\frac{1}{kb+c}+\\frac{1}{kc+a}\\ge \\frac{3}{k+1}.\\]\r\n\r\nHowever, I will try to solve your problem without my result. Thank you again,", "Solution_2": "[quote=\"hungkhtn\"] I already proved that for all $k\\ge 1$ and $a\\ge b\\ge c$ then\n\\[\\frac{1}{ka+b}+\\frac{1}{kb+c}+\\frac{1}{kc+a}\\ge \\frac{3}{k+1}. \\]\nHowever, I will try to solve your problem without my result. Thank you again,[/quote]\r\n\r\nThe inequality is not true for $k=1$. See Iran inequality.", "Solution_3": "I am sorry. I misunderstand at the first time. I did prove that for all $k\\ge 1$ and $a\\le b\\le c$ then\r\n\\[\\frac{1}{ka+b}+\\frac{1}{kb+c}+\\frac{1}{kc+a}\\ge \\min\\left(\\frac{3}{k+1},\\frac{1}{k\\sqrt{3}}+\\frac{1}{\\sqrt{3}}+\\frac{1}{(k+1)\\sqrt{3}}\\right). \\]\r\nThe equality must holds for $a=b=c=1$ or $a=0,b=c=\\sqrt{3}$. It is my implication.\r\n\r\nNotice that for $k=2$, we have\r\n\r\n\\[\\frac{1}{\\sqrt{3}}+\\frac{1}{2\\sqrt{3}}+\\frac{1}{3\\sqrt{3}}=1.05>1.\\]\r\n\r\nSo your problem is really sharp, VASC.", "Solution_4": "There is a nice solution to the original inequality. Who find it?", "Solution_5": "[quote=\"Vasc\"]If $a,b,c$ are nonnegative numbers such that $a\\le b\\le c$ and $ab+bc+ca=3$, then\n\n$\\frac 1{2a+b}+\\frac 1{2b+c}+\\frac 1{2c+a}\\ge 1$.[/quote]\r\nUsing the known $(x+y+z)^{2}\\ge 3(xy+yz+zx)$, it suffices to prove that\r\n\\[\\sum\\frac{1}{(2a+b)(2b+c)}\\ge \\frac{1}{ab+bc+ca}\\]\r\nOr\r\n\\[\\frac{3(a+b+c)}{(2a+b)(2b+c)(2c+a)}\\ge \\frac{1}{ab+bc+ca}\\]\r\nOr\r\n\\[ab^{2}+bc^{2}+ca^{2}\\ge a^{2}b+b^{2}c+c^{2}a \\]\r\nIt is trivial since $a \\le b \\le c$.\r\nWe are done. :)", "Solution_6": "Nice (only nice and not very nice, because this is also my solution :lol: ).", "Solution_7": "Much simpler than I though! Thanks for VASC and Can, :lol:", "Solution_8": "Toanhocmuonmau, you are really very clever. :)" } { "Tag": [], "Problem": "1) Outline a synthesis of $ C_6H_5\\minus{}C \\equiv C\\minus{}CH_2OH$.\r\n\r\n2) What product will be obtained when the alcohol of 1) is treated first with $ Br_2/CH_2Cl_2$, and then with $ Ph_3P, Br_2$?", "Solution_1": "Probably start with C6H6:\r\n[hide=\"click to reveal the hidden content\"]1.Gattermann Hydroformylation --> Benzaldehyde\n2.Treat with ylide PPh3=CH2 --> Styrene\n3.Treat with Br2/CCl4-->dibromo product\n4.Treat with alc.NaNH2 to get phenyl ethyne\n5.Treat it again with NaNH2 to get the carbanion\n6.Treat it with formaldehyde(Nef synthesis)-->Required product[/hide]", "Solution_2": "My method:\r\n1. Get benzaldehyde from benzene\r\n2. Perkins rn to get cinnamic acid\r\n3. $ \\alpha$ halogenation (?doubtful)\r\n4. Alc. $ \\ce{KNH2}$\r\n5. Use LAH to reduce the COOH to $ \\ce{CH2OH}$\r\n :)", "Solution_3": "U can't alpha chlorinate cinnamic acid :) :)", "Solution_4": "Chemrock's synthesis is correct, but:\r\n\r\n[quote=\"Chemrock\"]alc.NaNH2[/quote]\n\n?\n\n[quote=\"Chemrock\"]Treat it with formaldehyde(Nef synthesis)[/quote]\r\n\r\n?", "Solution_5": "strong base to do 2 dehalogenation to get alkyne.\r\n\r\nNef synthesis is treating metal alknyl with carbonyl compound. It is a nucleophilic addition reaction.", "Solution_6": "I know that. But what does it mean \"alc.NaNH2\"?\r\n\r\nAnd how about a shorter synthesis.", "Solution_7": "What is alc.KOH then?\r\nIt is just that in alcoholic medium it acts as a good base and hence eliminate them.\r\nProbably from benzaldehyde:\r\n[hide=\"click to reveal the hidden content\"]\n1.Do claisen smidt reaction with CH3-CHO-->cinnamaldehyde\n2.Then brominate and then dehalogenate\n3.reduce it with LAH[/hide]", "Solution_8": "[quote=\"Chemrock\"]What is alc.KOH then?[/quote]\r\n\r\nNo. Now you are further away from the correct reasoning.", "Solution_9": "[quote=\"Carcul\"]I know that. But what does it mean \"alc.NaNH2\"?\n\nAnd how about a shorter synthesis.[/quote]\r\n\r\nDo you want him to say that its a strong base :maybe:", "Solution_10": "[quote=\"chemrock\"]U can't alpha chlorinate cinnamic acid [/quote]\r\n\r\nhey y cant u alpha chlorinate it??\r\nu can do HVZ rite??", "Solution_11": "no because it will prefer the benzylic carbon rather than the carbon alpha to teh acid.", "Solution_12": "Excuse me the mechanism of HVZ doesn't permit that. :maybe: \r\n[quote=\"valeriummaximum\"]no because it will prefer the benzylic carbon rather than the carbon alpha to teh acid.[/quote]\r\nthat also is a sp2-s bond and is stable mr.valerium :huh:", "Solution_13": "then can u giove the reason why u cant a-chlorinate cinnamic acid...", "Solution_14": "I have given it just above urs.\r\nHere's a better explanation with the mechanism below:\r\nThe enol content of cinnamyl chloride is very low so that's why the reaction gives very poor yields", "Solution_15": "where have u given the reason?>??", "Solution_16": "I also do not see the reason...\r\n\r\nWhat you have there is a mechanism for the HVZ reaction.", "Solution_17": "So does that mean you can alpha chlorinate? :maybe: :maybe:", "Solution_18": "[quote=\"chemrock\"]\nThe enol content of cinnamyl chloride is very low so that's why the reaction gives very poor yields[/quote]\r\nHere!! :| :huh:", "Solution_19": "[quote=\"Valeriummaximum\"]then can u giove the reason why u cant a-chlorinate cinnamic acid...[/quote]\r\n\r\nThe reason is simple: the alpha carbon of cynnamic acid is a sp2 carbon, not a sp3.", "Solution_20": "I also gave that before here see:\r\n[quote=\"chemrock\"]It is a sp2-s bond and is stable mr.valerium[/quote]" } { "Tag": [ "geometry proposed", "geometry" ], "Problem": "Let O be the center of a circle on the plane.\r\nWe define a fix point M on a circle. We define a fix chord AB.\r\nLet CD be a variable chord (not AB) such that 2, n^k > k^n$ if and only if $k > n$. So $n^k - k^n$ would be smallest if $k = n + 1$, and $n^{(n+1)} - (n+1)^n$ increases with $n$. \r\n\r\n$(0,k)$ and $(n,0)$ work for all positive $n$ and $k$. $(0,0)$ doesn't work. These are trivial.\r\n\r\nIf $n<0$, the LHS is a fraction, and a fraction mod an integer is a fraction (if you even consider it to exist), so that doesn't work.\r\n\r\nIf $k<0$, you've got fractions again, so if you want to worry about fractional mods, it seems like there ought to be a solution. Namely, $(2,-3)$. For positive $n$, that should be the only solution, by the same argument as above. \r\n\r\nFor both $n$ and $k$ less than 0, we've got $(-2,-1)$. I believe that's the only one, but I'm tired and I don't want to think about how to prove that." } { "Tag": [], "Problem": "If we count by 3's starting with 1, the following sequence is obtained: 1, 4, 7, 10,...\r\nWhat is the 100th number in the sequence?", "Solution_1": "[hide=\"Answer\"]$300$[/hide]", "Solution_2": "i think that's wrong because the sequence starts with 1.", "Solution_3": "Forgot to subtract 2 :( \r\n[hide=\"Corrected answer\"]$298$[/hide]", "Solution_4": "[hide]1 + 3(100 - 1) = 1 + 297 or 298.[/hide]", "Solution_5": "99*3+1=297+1=298", "Solution_6": "298. We see that the 100th number is obviously 3x100 which is 300. Subtracting 2 by the first and last digits according to a formula which I forget what it is.", "Solution_7": "pplz u just revived so many old topic.\r\nplz stop as nick42 has already mentioned" } { "Tag": [ "inequalities", "analytic geometry", "trigonometry", "Pythagorean Theorem", "geometry", "triangle inequality" ], "Problem": "Given: isosceles triangle $ PAB$, with $ |AP|\\equal{}|PB|$ and $ C$ on side $ PB$, strictly between $ P$ and $ B$.\r\n\r\nProve: $ |CB|<|AC|.$", "Solution_1": "On the coordinate plane, let $ A \\equal{} ( \\minus{} m,0), B \\equal{} (m,0), P \\equal{} (0,n), C \\equal{} (x,y)$. Drop a perpendicular from $ C$ onto $ AB$ and call the intersection point at $ (x,0)$ $ D$. Since $ C$ is on side $ PB$, strictly between $ P$ and $ B, 0 < x < m$. Therefore, $ AD > DB$ and from the Pythagorean theorem, it follows that $ AC > BC$.", "Solution_2": "[quote=\"gooeymeister\"]On the coordinate plane, let $ A \\equal{} ( \\minus{} m,0), B \\equal{} (m,0), P \\equal{} (0,n), C \\equal{} (x,y)$. Drop a perpendicular from $ P$ onto $ AB$ and call the intersection point $ D$. Since $ C$ is on side $ PB$, strictly between $ P$ and $ B, 0 < x < m$. Therefore, $ AD > DB$ and from the Pythagorean theorem, it follows that $ AC > BC$.[/quote]\r\n\r\nI'm not following this -- could you clarify? Specifically:\r\n\r\nFirst, since the triangle is isosceles with apex $ P$, $ D$ is the midpoint of $ A$ and $ B$, and is thus the origin.\r\n\r\nSecond, for the same reason, $ AD \\equal{} DB$, not $ AD > DB$.\r\n\r\nThird, in your last sentence, if the proof is still viable despite the first two comments, to what triangle are you applying Pythagoras's theorem?\r\n\r\n(Note that my intuition is that there must be a way to solve this through repeated application of the triangle inequality, but I'm not seeing it.)", "Solution_3": "[quote=\"Peterhi\"]Given: isosceles triangle $ PAB$, with $ |AP| \\equal{} |PB|$ and $ C$ on side $ PB$, strictly between $ P$ and $ B$.\n\nProve: $ |CB| < |AC|.$[/quote]\r\n\r\nIt is clear that $ \\angle CAB < \\angle PAB$ , because $ AC$ is an internal ray to the angle $ \\angle PAB$ but $ \\angle PAB \\equal{} \\angle PBA \\longrightarrow \\angle PBA > \\angle CAB \\longrightarrow CA > CB$ as desired :)", "Solution_4": "[quote]First, since the triangle is isosceles with apex P, D is the midpoint of A and B, and is thus the origin. [/quote]\r\n\r\nSorry, I miswrote $ C$ as $ P$ and have now corrected it.", "Solution_5": "[quote=\"gooeymeister\"][quote]First, since the triangle is isosceles with apex P, D is the midpoint of A and B, and is thus the origin. [/quote]\n\nSorry, I miswrote $ C$ as $ P$ and have now corrected it.[/quote]\r\n\r\nGooey, could you explain your use of the Pythagoras's theorem in the last point of your original post?\r\n\r\nThanks.", "Solution_6": "Since $ AD \\bot CD$, $ AD^2\\plus{}CD^2\\equal{}AC^2$. Similarly, $ BD^2\\plus{}CD^2\\equal{}BC^2$. Therefore, $ AD>BD \\Rightarrow AC^2>BC^2 \\Leftrightarrow AC>BC$.", "Solution_7": "[quote=\"gooeymeister\"]Since $ AD \\bot CD$, $ AD^2 \\plus{} CD^2 \\equal{} AC^2$. Similarly, $ BD^2 \\plus{} CD^2 \\equal{} BC^2$. Therefore, $ AD > BD \\Rightarrow AC^2 > BC^2 \\Leftrightarrow AC > BC$.[/quote]\r\n\r\nVery nice, thanks.", "Solution_8": "Trigonometric Solution:\r\n\r\n [geogebra]dc33a2b03ce2ab5b74b9c06b2776cc4dc4452079[/geogebra]\r\n\r\nBy the Sine Law, it follows that $ \\frac{CB}{AC}\\equal{}\\frac{\\sin{\\angle{CAB}}}{\\sin{\\angle{CBA}}}$. Since $ \\angle{CAB} < \\angle{CBA} < 90$, and because $ \\angle{CAB}$ increases so does $ \\sin{\\angle{CAB}}$ (by the graph of sine) it also follows that $ \\sin{\\angle{CAB}} < \\sin{\\angle{CBA}}$ therefore $ \\frac{\\sin{\\angle{CAB}}}{\\sin{\\angle{CBA}}}\\equal{}\\frac{CB}{AC}<1$ and $ CB is normal, and since z centralizes itself and N, is abelian normal. Since N is maximal, =N, and N is its own centralizer. Hence the map from G into Aut(N) has kernel N, and G is an extension of the finite abelian group N by a finite abelian subgroup of the finite group Aut(N)." } { "Tag": [], "Problem": "Fie $f: R\\to R$, $f\\neq 0$ o functie continua, periodica, F o primitiva a sa si g un polinom de grad cel putin 2. Sa se arate ca $Fog$ nu este periodica.\r\n-----\r\nAm reusit sa rezum problema la cazul F periodica. Altfel zis, cum pot demonstra ca daca compunem o periodica cu o polinomiala de grad cel putin 2, nu mai obtinem o periodica?", "Solution_1": "La asta am dat o demonstratie pe forumu de calculus. Era un post a lu perfect-radio. Vezi si tu daca e buna solutia. Poate gasesti una mai simpla :)" } { "Tag": [], "Problem": "The product of the base seven numbers $ 24_7$ and $ 30_7$ is expressed in base seven. What is the base seven sum of the digits of this product?", "Solution_1": "Converting to base 10, $ 18\\cdot{21}\\equal{}378$ which is $ 1050$ in base 7, so $ 6$", "Solution_2": "Or, you can say\r\n[code]\n 1\n 24\n x30\n-----\n1050\n[/code]" } { "Tag": [ "probability", "quadratics", "MATHCOUNTS", "AMC", "USA(J)MO", "USAMO", "algebra" ], "Problem": "This round isnt going to be hard, for this is the first of the series of competition and I wanted to open it up to even the least skilled people (no offence :))\r\n\r\nOK, Question 1:\r\nCorrect Answer: 1 points\r\nEach different ways of arriving at the answer: 1/4 points\r\n\r\nWhat is the probability of rolling 2 dices and getting (2^(1/3)+1)(4^(1/3)-2^(1/3)+1) as the sum of the two numbers rolled?\r\n\r\nQuestion 2:\r\nCorrect Answer: 2 points\r\nEach different ways of arriving at the answer: 1/4 points\r\n\r\nFind the equation of a line going through the origin and (2, (5^(1/3)+25^(1/3)-(2*40^(1/3))/(5^(1/3)-1)) in point-intercept form\r\n\r\nQuestion 3:\r\nCorrect Answer: 3 points\r\nEach different ways of arriving at the answer: 1/2 points\r\n\r\nFind all quadratic equation which goes through (a,b), (b,c), and (c,b) where as a=(5^(1/3)+25^(1/3)-(2*40^(1/3))/(5^(1/3)-1), b=8^(1/3)+64^(1/3), and c=((((8/27)^(-1/3))^(3/2))/(3/2)^(-1/2))^1/2\r\n\r\nEnjoy! and remember, use spoilers!", "Solution_1": "Finally I get an answer! \n\nAs for the question, [hide]your correct, but can you explain the (2^1/3+1)(2^2/3-2^1/3+1)=3 part a bit more?[/hide]\n\n\n\nYou know, I never realized how hard it is to select a phrase or an equation from a Spoiler text", "Solution_2": "I should give you partial credit because you double-posted, but oh well. Oh, and by the way, [hide]you're correct![/hide]", "Solution_3": "i feel confused and betrayed...", "Solution_4": "Yup, that's definitely one of the key points to this problem.", "Solution_5": "why does he only get partial credit? he was busy", "Solution_6": "note that I was not competing in the first place...", "Solution_7": "[quote=\"Syntax Error\"]why does he only get partial credit? he was busy[/quote]\r\n :?: Why does busy people gets to double-post? I mean, those two things aren't connected...(or so I think? :) )\r\n\r\nand MysticTerminator, sorry.", "Solution_8": "im not participating either, just postin mass amounts of problems", "Solution_9": "Okey dokey. I'd still love it if you refrain from posting anymore questions until I have sufficient participants for the competition. I mean, your problems can be solved anytime but the competition has to end by September 30th, you know. :) \r\n\r\nAm I being too self-centered? :P", "Solution_10": "Waiit..then who IS going to participate?", "Solution_11": "i dunno, but ill keep on posting problems cause it draws MORE attention. :twisted:", "Solution_12": "Personally, I think there should be different divisions for member of the month...\r\n\r\nOne for the MathCounts participants\r\n\r\nOne for the AMC people\r\n\r\nOne for the USAMO making people\r\n\r\nand One for Simon and MysticTerminator :wink:", "Solution_13": "um...you realize simon would easily destroy me in such a competition...but yeah, besides that i think that's a good idea", "Solution_14": "I should learn how to write problems, so maybe I should write such contests for others rather than solving other problems.", "Solution_15": "[quote]I should learn how to write problems, so maybe I should write such contests for others rather than solving other problems.[/quote]\r\n\r\nSimon, are you majoring in math in college? Anyways, you still have the Putnam.", "Solution_16": "I wouldn't consider not majoring in math for a second.", "Solution_17": "Oh, come on, Syntax!\r\nWe're just dividing those precious attention amongst ourselves.\r\n\r\nCan you do this, as a favor?", "Solution_18": "[quote=\"TheSouthpaw\"]Personally, I think there should be different divisions for member of the month...\n\nOne for the MathCounts participants\n\nOne for the AMC people\n\nOne for the USAMO making people\n\nand One for Simon and MysticTerminator :wink:[/quote]\r\nThe last one's no good *booing and geering from the audience* but as for the rest, I'll post my \"plans\" in October. Till then, you'll just have to speculate :)", "Solution_19": "[quote=\"MysticTerminator\"]i feel confused and betrayed...[/quote]\r\n\r\nAgain, I publically apologize.\r\nI mean, what else can I do? harakiri? Chinese water torture? The ripply thing!? :)", "Solution_20": "[hide]1. (2^(1/3)+1)(4^(1/3)-2^(1/3)+1)=8^(1/3)-4^(1/3)+2^(1/3)+4^(1/3)-2^(1/3)+1=3\n\nTotal possiblities of 2 dice = 6*6 = 36\n\nCombinations that yield 3 = 1,2 and 2,1 = 2 combinations\n\n2/36 = 1/18, the answer\n\n\n\n2. The equation of the line will be y = m*x + b.\n\nThe line passes through 0,0 so that when x = 0, and y = 0, b must equal 0. So, 2*m = (5^(1/3) +25^(1/3) - (2*(40^(1/3))))/(5^(1/3)-1)\n\n\n\nSimplify\n\n2*m = (25^(1/3)-(3*5^(1/3)))/(5^(1/3)-1)\n\n\n\nso, y = 2*(25^(1/3)-(3*5^(1/3)))/(5^(1/3)-1)*x, or\n\ny = 2*(5^(1/3)*(5^(1/3)-3))/(5^(1/3)-1)\n\n\n\n\n\n3. You can get three equations, with a, b, and c currently not simplified.\n\n\n\nd*a^2+e*a+f = b\n\n\n\nd*b^2+e*b+f = c\n\n\n\nd*c^2+e*c+f = b\n\n\n\nSolving for f in the first equation, pluging it into the second and solving for e, pluging both of them into the third and solving for d gives that:\n\nd = 1/(a-b)\n\ne = -(a+c)/(a-b)\n\nf = (a*(b+c)-b^2)/(a-b)\n\n\n\nWhen simplified, b comes out as 6, and c as 3/2.\n\na simplifies to (5^(1/3)*(5^(1/3)-3))/(5^(1/3)-1)\n\n\n\nLeaving a as a (otherwise the final equation is very long), I get:\n\n\n\ny = (2*x^2-(2*a+3)*x+3*(5*a-24))/(2*(a-6))[/hide]", "Solution_21": "OK,. be patient with me here bookworm, because I'm making some tough decisions to make here...I'll be done in a minute. \r\n\r\nValo, (OK in Spanish, fsi (for somebody's info :)) here is the official score: 4 1/2 points \r\nExplanation: The first one is correct. 1 and a quarter points. \r\nThe second one is correct, but it could be improved upon, so half credit for the answer. However, you're on the right track, so I'll give you the quarter points. Total of 1 and a quarter points. \r\nThe third one is the same as the second. Good answer, not the best. 2 points. \r\nMaking up the total of 4 and a half points.\r\n\r\n\r\n'Tis truly sad that there are less people actually participating than the people who said \"Count me in!\" :(" } { "Tag": [ "analytic geometry", "graphing lines", "slope", "AMC", "AIME", "quadratics", "calculus" ], "Problem": "=) Hello. This is a question from the 2005 Fermat contest, which is written in Ontario, Canada. It was the last, most difficult question, and nobody in the school solved it (although some guessed). Apparently it took the entire math department and a few grade 12's about half an hour working together to solve it (the contest itself was only an hour long). A full solution written for someone at a Grade 11 level (so that I can understand this interesting problem :)) would be greatly, greatly appreciated! An interesting problem for all, I hope ^_^\r\n\r\nA 'triline' is a line with the property that three times its slope is equal to the sum of its $x$-intercept and its $y$-intercept. For how many integers $q$ with 1<= q <= 10,000 is there at least one integer $p$ so that there is exactly one triline through $(p,q)$?\r\n\r\nA) 60\r\nB) 57\r\nC) 58\r\nD) 61\r\nE) 59", "Solution_1": "btw, Fermat is written in more than just Ontario. Students from all over Canada write it.", "Solution_2": "I'll sketch a solution, the problem isn't that hard (if I'm reading it correctly).\r\n\r\nConsider the line through $p,q$ with slope $m$. By the triline equation, we get:\r\n\r\n$3m=q-pm+p-q/m\\Rightarrow$\r\n$(p+3)m^2-(p+q)m+q=0\\Rightarrow$ two solutions unless $\\triangle=0$, therefore\r\n$(p+q)^2-4pq-12q=0\\Rightarrow$\r\n$(p-q)^2=12q$\r\n\r\nThis is equivalent to the number of integers between $1,346$ divisible by $6$; the answer is thus $57$.", "Solution_3": "Hey... My Solution was actually correct.. That's surprising..\r\n\r\nNow I should just hope that I didn't do anything silly.. :D", "Solution_4": "By the way, I am in grade 11, and blahblahblah's solution is exactly the same as mine..", "Solution_5": "[quote=\"blahblahblah\"]I'll sketch a solution, the problem isn't that hard (if I'm reading it correctly).\n\nConsider the line through $p,q$ with slope $m$. By the triline equation, we get:\n\n$3m=q-pm+p-q/m\\Rightarrow$\n$(p+3)m^2-(p+q)m+q=0\\Rightarrow$ two solutions unless $\\triangle=0$, therefore\n$(p+q)^2-4pq-12q=0\\Rightarrow$\n$(p-q)^2=12q$\n\nThis is equivalent to the number of integers between $1,346$ divisible by $6$; the answer is thus $57$.[/quote]\r\n\r\nI'm not sure I understand what you did here -- could you expand on your solution a bit more for a relative beginner? :lol: \r\n\r\nI especially don't understand your last part... \"the number of integers between $1,346$ divisible by $6$\"... did you mean the number of integers between $1,346$ *and* $1,000$? If not, then there are more than 57 integers divisible by six between 1-1,346. If so, where'd that come from? \r\n\r\nSorry for all these questions :blush: I'd just like to understand what you did here....", "Solution_6": "In general, I never use commas when writing out numbers, so that should be read as the number of integers between $1$ AND $346$.\r\n\r\nYou should be able to follow the first part pretty easily, for the last part, notice that $12\\vert p^2$ if and only if $6\\vert p$.", "Solution_7": "I'd say this is a pretty difficult problem for a high school competition. I wouldn't expect to see it before the end of the AIME, and maybe not even there.", "Solution_8": "good AIME problems are much harder. This is a very straightforward problem. I'll expand slightly on blah^3's soln (which is in fact the way I worked it, and really seems to be the most logical way to work it)\r\n\r\nSo we are given this fairly random condition for trilines. Let's try to quantify this. First, we know any line can be written in the form $y = mx + b$, where m is the slope and b is the y-intercept. It's also pretty easy to find the x-intercept in this general case - simply plug 0 in for y and solve for x to get $-\\frac{b}{m}$. So, the condition for \"trilinity\" is that thrice the slope is the sum of the intercepts, i.e. $3m = b + -\\frac{b}{m}$. Cleaning this up a little, we have $b = \\frac{3m^2}{m-1}$, so the general form of a triline is $y = mx + \\frac{3m^2}{m-1}$. Now, we are interested in the trilines that pass through some point $(p,q)$. In specific, we want there to be a single solution for $m$ (the slope). So, plugging $(p,q)$ in for $(x,y)$, we have $q = mp + \\frac{3m^2}{m-1}$. We want to solve for $m$ (as we have a condition on $m$), so doing so, we find $(p+3)m^2 - (p+q)m + q = 0$. So, as this is a quadratic in $m$, if we want this to have a single solution, the discriminant must be $0$. So, we have $(p+q)^2 - 4(p+3)q = 0$. We want to solve this for $p$ (as we have a condition for $p$ in terms of a specific $q$). Doing so, we find $(p-q)^2 = 12q$. As $q$ is an integer, there will be an integral solution for $p$ if and only if $12q$ is a perfect square. We can make this a bit easier on ourselves by noting $12q$ will be a perfect square if and only if $q$ is $3$ times a perfect square. From here, all we have to note is that $3 = 3 \\cdot 1^2$ is the smallest integer of this form in the given range, and $9747 = 3 \\cdot 57^2$ is the largest integer of this form in the range, for a total of $57$ eligible values of $q$.", "Solution_9": "MysticTerminator, you have made my day. I fully understand that solution, and now I see where blahblahblah is coming from :P Thank you so much for your patience with that solution!\r\nI love this place :lol:", "Solution_10": "How do you know that this:\r\n\r\n[quote=\"MysticTerminator\"]From here, all we have to note is that $3 = 3 \\cdot 1^2$ is the smallest integer of this form in the given range, and $9747 = 3 \\cdot 57^2$ is the largest integer of this form in the range, for a total of $57$ eligible values of $q$.[/quote]\n\nis the same as this:\n\n[quote=\"blahblahblah\"]This is equivalent to the number of integers between $1,346$ divisible by $6$; the answer is thus $57$.[/quote]", "Solution_11": "Okay, so $346$ is the largest integer whose square is less than $120000$\r\n\r\nWe want integers whose square is divisible by $12$. $12=2^3\\cdot 3$, and the extra factor of $2$ doesn't help. So we're just looking for integers divisible by $6$.", "Solution_12": "By the way, dr. rice mentioned that there's something funny about this problem.\r\n(not to worry, the answer is 57) anybody sees anything interesting or extremely funny?" } { "Tag": [ "vector", "linear algebra", "linear algebra unsolved" ], "Problem": "Let $V$ be a 10-dimensional real vector space and $U_1,U_2$ two linear subspaces such that $U_1 \\subseteq U_2, \\dim U_1 =3, \\dim U_2=6$. Let $\\varepsilon$ be the set of all linear maps $T: V\\rightarrow V$ which have $T(U_1)\\subseteq U_1, T(U_2)\\subseteq U_2$. Calculate the dimension of $\\varepsilon$. (again, all as real vector spaces)", "Solution_1": "Trivial number 1 as always: pick a basis of $U_1$, extend it to a basis of $U_2$, extend that to a basis of $V$. Then we have fixed $3(10-3)+(6-3)(10-6)=33$ elements, and the rest can be chosen completely free. So $\\dim\\varepsilon = 100-33=67$." } { "Tag": [ "email", "AMC", "AIME", "USA(J)MO", "USAMO", "LaTeX", "geometry" ], "Problem": "I emailed a school in Kobe, Japan (I'm moving there by early February) asking if someone could proctor the AMCs for me to take. Here is the reply:\r\n\r\n\r\nDear Soo-Hyun Yoo,\r\n\r\nI am the high school counselor at Canadian Academy on Rokko Island in\r\nKobe, Japan. Our headmaster forwarded your request for a test proctor\r\nto me. Before I can look for a proctor I will need more information.\r\n How much does the test cost to take and how will we get it? Are\r\nstudents such as yourself allowed to take the test independent of\r\nattending our school? I haven't had time yet to look at the websites.\r\n Please make sure that you have reviewed the conditions necessary for\r\nyou to take the test as an individual and then let me know exactly\r\nwhat you will need from us.\r\n\r\n[hide=\"Original email\"]Dear Mr. Wesson,\n \nI am a high school freshman (9th grade in high school) currently living in the United States of America. I will be moving to Kobe, Japan by early February.\n \nI have regularly taken the American Mathematics Competitions (AMC) since 6th grade. This is a huge contest in which over 250,000 students participate annually. I am seeking a place to take this year's AMCs and subsequent tests in Kobe, and a proctor to administer the test.\n \n \nTest format:\nAMC 10/12 - taken by over 250,000 students - high scorers qualify to take the AIME\nAIME - score calculated with AMC score - qualification step for USAMO\nUSAMO - taken by only about 500 U.S. residents, qualification step for International Math Olympiad team\n \n \nCould you please see if one of the teachers at your school can offer his/her time to proctor the AMC? A quiet classroom after school is sufficient. Even a corner in the school office where I can be monitored (to ensure the authenticity of my answers) is a fine place to take the test. I would greatly appreciate it.\n \nMore information on the AMC, offered by the MAA, can be found on their website: http://www.unl.edu/amc/\n \nInformation for the AMC 10/12, as well as the order forms, can be found in their information packet: http://www.unl.edu/amc/d-publication/d1-pubarchive/2007-8pub/1012TM/2008AMC1012A-TM.pdf\n \nThank you so much for taking the time to read this. Please reply as soon as possible.\n \nSincerely,\nSoo-Hyun Yoo\n[/hide]\r\n\r\nI'm aware that there is a \\$40 registration/shipping fee per school with each bundle of 10 exams sold at \\$14 or \\$16 for the AMC 10 and 12, respectively.\r\n\r\nThe overnight shipping cost is \\$60.\r\n\r\nThe completed order form needs to be faxed to the AMC office by the deadline.\r\n\r\n[b]However,[/b] I am not sure if I am allowed to take the test officially, registered as a student at a school that I don't attend.\r\n\r\nAlso, how should I pay the fees? Is mailing cash okay?\r\n\r\nCould someone confirm the conditions necessary and any additional information I should provide for the school?", "Solution_1": "If you aren't sure, just take the AMC B, since by that time you'll be in Japan. I think you can register online, but I'm not sure if you can pay by credit card or not. If you want to ask the MAA if they will allow your condition, you should try to contact them directly.\r\n\r\nBy the way, to use Latex, surround your code with dollar symbols. For example, instead of \"x^2+z \\$10\" you should type\r\n[code]$x^2+z$ $\\$10$[/code]\r\nwhich will give you $ x^2\\plus{}z$ $ \\$10$.", "Solution_2": "It wasn't if if or not I could take the A test that I wasn't sure about - it's if or not I can take the tests officially registered as a school I don't attend. I'll be homeschooled - but registered as a student at a public school.\r\n\r\nI've already contacted the MAA weeks ago, but I haven't gotten a reply yet. Should I email them again?\r\n\r\nThanks though.\r\n\r\nEDIT: I've just emailed to amcinfo@maa.org my original post above.", "Solution_3": "The AMC staff member who formerly answered email at amcinfo@maa.org retired effective December 22, then the AMC office was closed for the holidays. Then the new person who answers amcinfo@maa.org was at the Joint Math Meetings in San Diego almost immediately after returning. That probably accounts for the delay in answering. As of late last week, information requests were caught up again. \r\n\r\nPlease email amcinfo@maa.org and state your question simply and clearly. From the posts above I do not understand your request. \r\n\r\nFurther pursuit of the question here on the Forum will probably not help, your request appears to be too specfiic to your individual situation to make general advice possible.\r\n\r\nSteve Dunbar\r\nMAA Director, American Mathematics Competitions", "Solution_4": "My school only offers the AMC A tests. Would it be possible to take the B test at another school in the area (a school that I am not enrolled in)?", "Solution_5": "You can take the B tests at a local university or higher education site:\r\n\r\n[url]http://www.unl.edu/amc/b-registration/b1-archive/2007-2008/CU2008/2008-CU-list.shtml[/url]\r\n\r\nprovided that you are lucky enough to be near one." } { "Tag": [ "abstract algebra", "group theory", "Galois Theory", "superior algebra", "superior algebra unsolved" ], "Problem": "I thought this was a nice problem. \r\n\r\n[b]Problem. [/b] A [i]minimal non-abelian group[/i] $G$ is a group such that $G$ is not abelian, but every proper subgroup of $G$ is abelian. Likewise, a minimal non-nilpotent group is a group that is not nilpotent, but every proper subgroup is nilpotent.\r\n\r\n(a) Prove that a minimal non-abelian group cannot be simple;\r\n(b) Prove that a minimal non-nilpotent group cannot be simple;\r\n(c)* Can a minimal non-solvable group be simple?\r\n\r\n[I have solved (a) and (b), but not (c). ]\r\n\r\n--Vesselin", "Solution_1": ":D Just in case ... (c) is a joke ;) .", "Solution_2": "First (c):\r\n\r\n$A_{5}$ is minimal non-solvable and it is simple. I believe this is what you meant when you said that it was a joke :).\r\n\r\n\r\n(a)\r\n\r\nIn [url=http://www.artofproblemsolving.com/Forum/viewtopic.php?highlight=abelian&t=54910&sid=5bc9072c1c1cc41498c0e44e1a303d6d]this[/url] other thread there is a proof (which can be simplified) of the stronger fact that if a group has a maximal subgroup which is abelian then it is solvable. Here's a proof of the statement (stronger than the problem (a) but weaker than the problem in that other topic):\r\n\r\nIf a finite group (non-abelian) $G$ has a maximal subgroup $A$ which is abelian, then $G$ cannot be simple.\r\n\r\nAssume $G$ is simple. Let $g$ be an element outside $A$. If we were to have an element $1\\ne x\\in A\\cap gAg^{-1}$, then the centralizer of $x$ in $G$ would be a subgroup of $G$ containing the maximal subgroup $A$ strictly, so $x$ would be central in $G$, which contradicts the simplicity of $G$. This means that $gAg^{-1}\\cap A=\\{1\\}$ whenever $g\\in G\\setminus A$. I claim that this implies $(|A|,[G: A])=1\\ (*)$. \r\n\r\nMake $A$ act by multiplication to the left on the set of $[G: A]$ left cosets of $A$. Let $a\\in A$ be an element of prime order $p\\ |\\ |A|$. $a$ fixes $A$ (regarded as the coset of $1$), and if $g\\not\\in A$, then, because $g^{-1}ag\\not\\in A$, we have $agA\\ne gA$ (i.e. $a$ does not fix $gA$). This means that the cycle structure of $a$ as a permutation of the left cosets of $A$ consists of one cycle of length $1$, and several cycles of length $p$. This means that $p|[G: A]-1$, which proves $(*)$. \r\n\r\nNow let $P$ be a Sylow subgroup of $A$. By $(*),\\ P$ is Sylow subgroup of $G$. It\u2019s not normal in $G$ because we assumed $G$ to be simple, so the normalizer of $P$ is $A$. Clearly, it\u2019s also its centralizer, so $P$ is a Sylow subgroup of $G$ contained in the center of its normalizer. By Burnside\u2019s Normal Complement theorem (discussed on the forum several times), $P$ has a normal complement in $G$. In particular, $G$ is not simple, and we have our contradiction.\r\n\r\n\r\n(b)\r\n\r\nI don\u2019t really have a proof to this, except for one (which sort of feels like cheating :)) using the following theorem of Hall (which I\u2019ve also cited in [url=http://www.artofproblemsolving.com/Forum/viewtopic.php?t=47434&sid=807169e25f4b3b935f52c90249cadc15]this[/url] other thread started by vess):\r\n\r\nLet $G$ be a finite group, and let $P$ be a Sylow $p$-subgroup of $G$ with the property that for every subgroup $Q$ of $P$, every element of $G$ of order prime to $p$ which normalizes $Q$ centralizes $Q$. Then $G$ has a normal complement of $P$. \r\n\r\nUsing this, the problem becomes easy. Let $P$ be a Sylow $p$-subgroup of our minimal non-nilpotent group $G$, assumed simple, and let $Q\\le P$. $Q$ is not normal, so its normalizer is contained in some nilpotent subgroup $N$ of $G$. $N$ is the direct product of its Sylow subgroups, so every element in $G$ with order prime to $p$ which normalizes $Q$ certainly centralizes $Q$. The hypotheses of the theorem above are satisfied, so $G$ has a normal subgroup, contradicting the assumption that it\u2019s simple." } { "Tag": [ "trigonometry", "algebra", "polynomial", "complex numbers", "algebra unsolved" ], "Problem": ":arrow: [color=red]Let k be a positive integer. Prove that sqrt(k+1)-sqrt(k) is not the real part of the complex number z with z^n=1 for some positive integer n.[/color]", "Solution_1": "If $\\cos{\\alpha}=\\sqrt{k+1}-\\sqrt{k}$, here $z=\\cos{\\alpha}+i\\sin{\\alpha}$ and $z^{n}=1$. Then $\\cos{n\\alpha}=1$ therefore $\\sqrt{k+1}-\\sqrt{k}$ is a root of $g(x)-1$, here $g(\\cos{\\alpha})=\\cos{n\\alpha}$. But minimal polynomial of $\\sqrt{k+1}-\\sqrt{k}$ is $f(x)=x^{4}-(4k+2)x^{2}+1$, so $f(x)|[g(x)-1]$ contradiction because product of moduls of all roots of $f(x)$ is equal to $1$.", "Solution_2": "And now try this problem : \r\nhttp://www.mathlinks.ro/Forum/viewtopic.php?t=113831" } { "Tag": [ "geometry unsolved", "geometry" ], "Problem": "[img]http://www.mathlinks.ro/Forum/viewtopic.php?mode=attach&id=14613[/img]\r\n\r\n :(", "Solution_1": "You posted this problem at [url]http://www.mathlinks.ro/viewtopic.php?t=205642[/url]", "Solution_2": "[img]http://www.mathlinks.ro/Forum/viewtopic.php?mode=attach&id=14623[/img]\r\n\r\n :thumbup:" } { "Tag": [ "algebra unsolved", "algebra" ], "Problem": "[b](1) Solve for x,y If \n\nx^2+xy = a^2\nx^2+xy = b^2\nx^2+y^2 = c^2[/b]", "Solution_1": "are you sure about the second equation :?:", "Solution_2": "i think the second equation must be y^2 + xy = b^2 \r\nthen it reduces to the most trivial thing ..\r\njust add (i) and (ii) .. get 2xy first add and then subtract and get x+y , x-y ... solve them :)[/img]" } { "Tag": [ "search", "number theory unsolved", "number theory" ], "Problem": "Show that for any $ a > 0$ there exist infinitely many $ n\\in N$ such that $ n^2 \\plus{} 1$ divides $ [an]!$. \r\n(where $ [x]$ is integer part of x.)", "Solution_1": "Posted before. \r\nI will solve it again :\r\nConsider the Pell's equation :\r\n$ x^2\\plus{}1\\equal{}(d^2\\plus{}1)y^2$ where $ d>0$\r\nIt has smallest positive solution : \r\n$ (x,y)\\equal{}(d,1)$\r\nSo it has infinite solution : \r\n$ y_1\\equal{}$\r\n$ y_{n\\plus{}2}\\equal{}2(d^2\\plus{}1)y_{n\\plus{}1}\\minus{}y_n$\r\nThis is a sequence of root of this equation. \r\nSo $ \\gcd(y_{2n\\plus{}1},d^2\\plus{}1)\\equal{}1$\r\nConsider the sequence $ \\{y_{2n\\plus{}1}\\}$\r\nChose d,n large enough we have $ ax_{2n\\plus{}1}>2y_{2n\\plus{}1},\\forall n\\geq n_0$\r\nSo $ [ax_{2n\\plus{}1}]\\geq y_{2n\\plus{}1}$ and $ d^2\\plus{}1<[ax_n\\plus{}1]$ for all $ n\\geq n_0$\r\nSo $ y_{2n\\plus{}1}^2|[ax_{2n\\plus{}1}]!$ and $ d^2\\plus{}1|[ax_n]!$\r\nBut from $ \\gcd(y_{2n\\plus{}1},d^2\\plus{}1)\\equal{}1$ so \r\n$ x_{2n\\plus{}1}^2\\plus{}1|[ax_{2n\\plus{}1}]!$\r\nProof is complete . \r\nNote : please use search to find the general problem I have post . \r\nExist infinite $ x$ such that \r\n$ ax^2\\plus{}b|[cx]!$ :wink:" } { "Tag": [ "function", "algebra proposed", "algebra" ], "Problem": "Find all functions $ f: \\mathbb{R} \\rightarrow \\mathbb{R}$ such that \r\n$ f(x\\plus{}y)\\plus{}f(x)f(y)\\equal{}f(xy)\\plus{}f(x)\\plus{}f(y)$ for all $ x,y \\in \\mathbb{R}.$", "Solution_1": "$ P(x,0): (f(x) \\minus{} 2)f(0) \\equal{} 0$\r\nIf $ f(0)\\ne0$, we get the first solution is $ f(x) \\equal{} 2$.\r\nNow $ f(0) \\equal{} 0$.\r\n$ P(2,2): f(2)^2 \\equal{} 2f(2)$. So $ f(2) \\equal{} 0$ or $ f(2) \\equal{} 2$.\r\n$ P(1,1): f(1)^2 \\minus{} 3f(1) \\plus{} f(2) \\equal{} 0$. We can solve $ f(1)$ for all possible $ f(2)$ and there are four cases.\r\n\r\nCase 1) $ f(2) \\equal{} 2,f(1) \\equal{} 2$\r\n$ P(x,1): f(x \\plus{} 1) \\equal{} 2$, No solution.\r\n\r\nCase 2) $ f(2) \\equal{} 0,f(1) \\equal{} 3$\r\n$ P(x,1): f(x \\plus{} 1) \\equal{} 3 \\minus{} f(x)\\Rightarrow f(x \\plus{} 2) \\equal{} f(x)$\r\n$ P(x,2): f(2x) \\equal{} f(x \\plus{} 2) \\minus{} f(x) \\equal{} 0$. No solution\r\n\r\nCase 3) $ f(2) \\equal{} 0,f(1) \\equal{} 0$\r\n$ P(x,1): f(x \\plus{} 1) \\equal{} 2f(x)\\Rightarrow f(x \\plus{} n) \\equal{} 2^nf(x)$, where $ n$ can be any integer.\r\nWe also see $ f(n) \\equal{} 0$\r\n$ P(x,n): f(nx) \\equal{} (2^n \\minus{} 1)f(x)$\r\n$ P(nx,2x)$ implies $ f(x^2) \\equal{} \\frac3{2^n \\minus{} 1}(f(x)^2 \\plus{} f(x))$ for any integer $ n$.\r\nWe can let $ n$ be arbitrarily large and we get the second solution $ f(x) \\equal{} 0$.\r\n\r\nCase 4) $ f(2) \\equal{} 2,f(1) \\equal{} 1$\r\n$ P(x,1): f(x \\plus{} 1) \\equal{} f(x) \\plus{} 1\\Rightarrow f(x \\plus{} n) \\equal{} f(x) \\plus{} n$ and $ f(n) \\equal{} n$.\r\n$ P(x,n): f(nx) \\equal{} nf(x)\\Rightarrow f(rx) \\equal{} rf(x),\\forall r\\in \\mathbb Q$ and $ f(r) \\equal{} r$ \r\n$ P(x,r): f(x \\plus{} r) \\equal{} f(x) \\plus{} r$\r\n$ P(x,x): f(x^2) \\equal{} f(x)^2\\ge0$\r\nFor any $ x$, we find rational sequences $ r_n\\uparrow x,R_n\\downarrow x$.\r\n$ f(x) \\equal{} f(x \\minus{} r_n) \\plus{} r_n\\ge r_n$ and $ f(x) \\equal{} R_n \\minus{} f(R_n \\minus{} x)\\le R_n$.\r\nSo $ f(x) \\equal{} x$.\r\n\r\nConclusion: $ f(x) \\equal{} x$, $ f(x) \\equal{} 0$ and $ f(x) \\equal{} 2$." } { "Tag": [ "inequalities", "inequalities proposed" ], "Problem": "Suppose that $ f$ $ : (0,\\infty) \\to (0,\\infty) $ is continuous differentiable and strict monotonic on $ (0,\\infty) ,$ such that $ \\displaystyle f^{-1}(x)=\\displaystyle f^{\\prime}(x) \\; ,\\; \\forall x\\in (0,\\infty) .$\r\n[b] If $ \\varphi:=\\displaystyle \\frac{1+\\sqrt{5}}{2} , $ find $ \\displaystyle f(\\varphi)\\; .$ [/b]\r\n[b]Remark : [/b] My solution need some inequalities.", "Solution_1": "I just love this problem. I saw it for the first time at oral ENS and after more than one month I finally posted a solution on the analysis forum. Fedja gave a more elegant solution. It is a very very difficult problem, I think." } { "Tag": [ "vector", "analytic geometry", "Functional Analysis", "advanced fields", "advanced fields unsolved" ], "Problem": "Let $ a,b$ be unit vectors in real Banach space such that $ \\parallel{}2a \\plus{} b\\parallel{} \\equal{} \\parallel{}a \\minus{} 2b\\parallel{} \\equal{} 3$. prove that there exists continuous linear functionals $ f,g$ with the norm $ 1$ such that $ f(a) \\equal{} f(b) \\equal{} 1$, $ g(a) \\equal{} \\minus{} g(b) \\equal{} 1$.", "Solution_1": "Are the functionals required to have unit norm too? Otherwise the problem is easy: just check that $ a$ and $ b$ cannot be collinear.", "Solution_2": "[quote=\"mlok\"]Are the functionals required to have unit norm too? Otherwise the problem is easy: just check that $ a$ and $ b$ cannot be collinear.[/quote]\r\n\r\nYes,yes, sorry. they are both required to have norm $ 1$.", "Solution_3": "Actually, such a and b cannot exist. Mark the points (1,0), (0, 1), (2,1), (1,-2) on a coordinate plane, then mark the points symmetric to them about (0,0). There is no convex region that contains all these points on the boundary.", "Solution_4": "Once again, sorry, i've edited the problem conditions: there must be $ \\parallel{}2a \\plus{} b\\parallel{} \\equal{} \\parallel{}a \\minus{} 2b\\parallel{} \\equal{} 3$.", "Solution_5": "Mark the points (1,0), (0, 1), (2/3,1/3), (1/3,-2/3) on a coordinate plane, then mark the points symmetric to them about (0,0). The only convex region that contains all these points on the boundary is the square with vertices $ (\\pm 1,0)$, $ (0,\\pm 1)$. So the norm on the span of $ a,b$ is the $ \\ell_1$ norm, and the required functionals can be given explicitly (and extended to the entire space by Hahn-Banach)." } { "Tag": [ "inequalities", "calculus", "function", "inequalities unsolved" ], "Problem": "Find the largest value for real $k$ such that $a^2+b^2+c^2+3k\\geq (k+1)(a+b+c)$ for all $a,b,c\\in \\mathbb{R^{+}}$ satisfy $abc=1$.\r\n(Clearly $k\\geq 1$)", "Solution_1": "And clearly $k<3$, because if we set $c$ close to $1$, and $a=b$, we get the inequality become false for $k=3$.\r\nCan anyone help me?", "Solution_2": "very nice problem.\r\nI guess the largest value of $k$ is $\\frac 8 3$ ;) \r\nbut have no proof :(", "Solution_3": "Nice problem ! :) \r\n We use mixing variables and calculus to solved this problem : \r\n The ineq is equivalent to : $\\frac{a^2+b^2+c^2-a-b-c}{a+b+c-3} \\geq k$ \r\n Let $f(a,b,c)= \\frac{a^2+b^2+c^2-a-b-c}{a+b+c-3}$ \r\n [b]Step 1:[/b] prove $f(a,b,c) \\geq f(a,\\sqrt{bc},\\sqrt{bc})$ , where $a= min (a,b,c)$ \r\n Indeed , \r\n$f(a,b,c) - f(a,\\sqrt{bc},\\sqrt{bc}) =(\\sqrt{b}-\\sqrt{c})^2[3-3(\\sqrt{b}+\\sqrt{c})^2+a(\\sqrt{b}+\\sqrt{c})^2-a^2+2\\sqrt{bc}(b+c+\\sqrt{bc})] \\geq (a+2\\sqrt{bc})(\\sqrt{b}+\\sqrt{c})^2-3(\\sqrt{b}+\\sqrt{c})^2+3-a-2\\sqrt{bc} =(a+2\\sqrt{bc}-3)((\\sqrt{b}+\\sqrt{c})^2-1) \\geq 0$ \r\n \r\n [b]Step 2:[/b] we find the minimum value of $f(a,t,t)$ , where $at^2=1$ and $t\\geq 1$ \r\n We have : \r\n $f(a,t,t)=g(t)=\\frac{2t^6-2t^5-t^2+1}{2t^5-3t^4+t^2}$\r\n $g'(t)=\\frac{2t(t+1)(t-1)^4(t^2+t+1)(2t^2-2t-1)}{(2t^5-3t^4+t^2)^2}$ \r\n $g'(t)=0 \\iff t=\\frac{1+\\sqrt{3}}{2}$\r\n So $g(t) \\geq g(\\frac{1+\\sqrt{3}}{2}) = 2,803847578$ \r\n So the maximum value of $k$ is $g(\\frac{1+\\sqrt{3}}{2}) = 2,803847578$", "Solution_4": "Great solution, thanks so much!", "Solution_5": "Hmm but the solution using calculus bothers me. Is there a solution without using calculus?\r\nBecause the function simplifies to $g(t) = (t+\\frac{1}{2}) + \\frac{\\frac{3}{4}}{t+\\frac{1}{2}} + \\frac{1}{t^2}$\r\nAnd also, if $x = \\frac{\\sqrt{3}+1}{2}$ then $x^2 = x+\\frac{1}{2}$\r\nSo maybe there is a no-calculus way to find the minimum of $g(t)$?\r\n\r\nBtw, I calculated that $g(\\frac{\\sqrt{3}+1}{2}) = 8 - 3\\sqrt{3}$" } { "Tag": [ "logarithms", "calculus", "calculus computations" ], "Problem": "Compute $ \\sum_{n\\equal{}1}^{\\infty}\\frac{(\\minus{}1)^{n\\plus{}1}n^2}{n^3\\plus{}1}$", "Solution_1": "Let $ \\xi = e^{i\\pi/3}$. Then\r\n\r\n\\begin{eqnarray*}\r\n\\sum_{n=1}^{\\infty} \\frac{(-1)^{n-1}n^2}{n^3+1}\r\n& = & \\frac{1}{3} \\sum_{n=1}^{\\infty} (-1)^{n-1} \\left[ \\frac{1}{n+1} + \\frac{1}{n-\\xi} + \\frac{1}{n+\\xi^2} \\right] \\\\\r\n& = & \\frac{1}{3} (1 - \\log 2) + \\frac{1}{3} \\sum_{n=-\\infty}^{\\infty} \\frac{(-1)^n}{\\xi - n}\\\\\r\n& = & \\frac{1}{3} [ 1 - \\log 2 + \\pi \\, \\text{sech} ( \\sqrt{3} \\pi / 2 ) ]\r\n\\end{eqnarray*}" } { "Tag": [ "AMC", "USA(J)MO", "USAMO" ], "Problem": "Find all real solutions (x,y,z) to the following system of equations.\r\n\r\nx + y = :sqrt: (4z - 1)\r\ny + z = :sqrt:(4x - 1)\r\nx + z = :sqrt:(4y - 1)\r\n\r\nI know it's symmetric but I haven't gotten anywhere past squaring everything and adding to get 2(x :^2: + y:^2: + z:^2:) + 2(xy + yz + xz) = 4(x + y + z) - 3. From there though I'm stuck. Any help'd be appreciated! :-)", "Solution_1": "First thing I thought of was subtract the first two equations to get rid of y. However that didn't quite work so instead I [hide] let x+y = a and y+z = b. Then to get rid of square roots I took a^2 and b^2. So b^2 - a^2 = 4(x-z). But what does b-a equal?\n\n[/hide]", "Solution_2": "WarpedKlown1335 wrote:Find all real solutions (x,y,z) to the following system of equations.\n\nx + y = sqrt(4z - 1)\ny + z = sqrt(4x - 1)\nx + z = sqrt(4y - 1)\n\nI know it's symmetric but I haven't gotten anywhere past squaring everything and adding to get 2(x^2 + y^2 + z^2) + 2(xy + yz + xz) = 4(x + y + z) - 3. From there though I'm stuck. Any help'd be appreciated! \n\n[hide] square the first two => x:^2:+y:^2:+2xy=4z-1, y:^2:+z:^2:+2yx=4x-1, subtract => x:^2:-z:^2:=4(z-x)=> either x=z or x+z=4. We can do a similar analysis on other pairs of equations, so now we have either x=y=z OR x+y=4, x+z=4, y+z=4 => x=y=z=2 OR x=y, x+z=y+z=4 (w/ permutations among the variables in this case). The last case (the interesting one) gives us 16=4x-1, and 4x:^2:=4z-1 => x=y=17/4, z=293/16 (and permutations). The second case doesn't work. Period. The first case gives us 4x:^2:=4x-1=> x=y=z=1/2. So, I get all solutions as (17/4, 17/4, 293/16), (17/4, 293/16, 17/4), (293/16, 17/4, 17/4), (1/2, 1/2, 1/2). Correct?[/hide]", "Solution_3": "Ummm...(y+z) :^2: isn't y:^2: + 2xy + z:^2:...sorry :( . And the only solution I got when I solved it on my own time (didn't use your method) was (.5,.5,.5) I didn't try out your answers yet though.", "Solution_4": "What about this ?\r\n\r\nIf x < z then x + y < y + z so :sqrt:(4z - 1) < :sqrt:(4x - 1) so z < x.\r\nContradiction. So x, y, z must be equal.\r\nNow we have 2x = :sqrt:(4x - 1) iff (2x - 1) = 0.\r\nSo the only solution is (x, y, z) = (1/2, 1/2, 1/2).", "Solution_5": "[color=cyan]Mystic, you should have checked the \"interesting\" solutions.[/color]", "Solution_6": "[quote=\"Arne\"]What about this ?\n\nIf x < z then x + y < y + z so :sqrt:(4z - 1) < :sqrt:(4x - 1) so z < x.\nContradiction. So x, y, z must be equal.\nNow we have 2x = :sqrt:(4x - 1) iff (2x - 1) = 0.\nSo the only solution is (x, y, z) = (1/2, 1/2, 1/2).[/quote]\r\nThat was ridiculously elegant. Wow.", "Solution_7": "[quote=\"Arne\"]What about this ?\n\nIf x < z then x + y < y + z so :sqrt:(4z - 1) < :sqrt:(4x - 1) so z < x.\nContradiction. So x, y, z must be equal.\nNow we have 2x = :sqrt:(4x - 1) iff (2x - 1) = 0.\nSo the only solution is (x, y, z) = (1/2, 1/2, 1/2).[/quote]\r\n\r\nHmm, is this a trick that is used a lot? That is, when given a bunch of equations that, when added, are symmetrical, trying to let one variable be < than another and then finding out that all of the variables are equal? Because I seem to recall a few other solutions that were previously posted around with forum using that strategy...I think...", "Solution_8": "Try USAMO #2 2002.", "Solution_9": "Hey Arne, that is incredibly beautiful. I got (1/2, 1/2, 1/2) also methodically, but it was a bit longer than what you did. Eventually I figured out that x=y=z so then I solved the system by substitution to get (1/2, 1/2, 1/2). :)", "Solution_10": "Well this trick can be very helpful when given a symmetric system.\r\n\r\nDo you remember the system \r\nx = 3y - 2\r\ny = 3z - 2\r\nz = 3x - 2 ?\r\n\r\nThis (selfmade) problem was solved using the same method.\r\n\r\nAnother idea when solving symmetric systems is using inequalities.\r\nAs an example, British Olympiad 1998 :\r\n\r\nDetermine all positive real solutions to\r\nxy + yz + zx = 2\r\nxyz = x + y + z + 2." } { "Tag": [], "Problem": "How many triangles are in the figure to the right?\n\n[asy]draw((-10,0)--(0,13)--(13,0)--cycle);\ndraw((0,0)--(0,13));\ndraw((7,0)--(0,13));\ndraw((7,0)--(11,2));[/asy]", "Solution_1": "Here is the diagram again:\r\n\r\n\r\n[asy]draw((-10,0)--(0,13)--(13,0)--cycle);\ndraw((0,0)--(0,13));\ndraw((7,0)--(0,13));\ndraw((7,0)--(11,2));[/asy]\r\n\r\n\r\nIt is easier to at first disregard the smallest line of the figure (appears to be cutting the rightmost triangle into two parts). Once we can count the number of triangles without that little line, we can add back in the number of extra triangles resulting from the line.\r\n\r\nThe number of triangles in the diagram w/o the little line is actually C(4,2). You may notice that the number of triangles will be the number of ways we can randomly choose 2 vertices of the total 4 on the bottom of the diagram since 1 vertex of the triangle is at the top. (Triangles have 3 vertices). Since the order in which we choose the vertices does not matter, we use Combination, not Permutations (P). \r\n\r\nC(4,2) = [u]4![/u]\r\n 2!2!\r\n\r\nAfter calculating that out, you will eventually arrive at the answer 6.\r\n\r\nBut wait, 6 is not the answer! Now we must count the number of extra triangles that little line creates!\r\n\r\nThe little line creates 2 more triangles by cutting the rightmost big triangle into 2 parts: a small part and a bigger part.\r\n\r\nThus, the answer is 6+2= [b][i][u]8[/u][/i][/b][/u][/i]", "Solution_2": "Oops. C(4,2)=4!/(2!)(2!), just in case anyone gets confused. I did it wrong.\r\n\r\n4!=24, 2!=2, 24/(2)(2)=6. There we go.\r\n\r\nAnswer is still 8." } { "Tag": [ "videos" ], "Problem": "[youtube]giAy8wEvoVsv[/youtube]\r\nHAHA\r\nThis video is awesome", "Solution_1": "i saw this video so long ago\r\nbut is still funny\r\ni love the asian mother part and the rapping part\r\n*yo yo yo\r\nmy name's macbeth\r\nmy wife wants me to kill the king (etc.)*\r\n :rotfl: :rotfl: :rotfl: \r\nis hilarious..." } { "Tag": [ "inequalities", "function", "inequalities proposed" ], "Problem": "Imagine a function f on the naturals. \r\nSuppose n = akak-1...a2a1 in base 2.\r\nLet f(n) be the sum of all indices i for which ai = 1. For example, \r\n21 = (10101)2 => f(21) = 5 + 3 + 1 = 9.\r\n29 = (11101)2 => f(29) = 5 + 4 + 3 + 1 = 13. \r\n13 = (1101)2 => f(13) = 4 + 3 = 1 = 8.\r\nNow prove that for any positive integer n:\r\nf(3n) \\leq 2f(n) + \\sqrt(f(n))\r\nand determine when equality occurs.", "Solution_1": "Hey, no solution? :?" } { "Tag": [ "inequalities", "calculus", "derivative", "function", "AMC", "USA(J)MO", "USAMO" ], "Problem": "Let $n$ be a positive integer, and $a_1,...,a_n, b_1,..., b_n$ be $2n$ positive real numbers such that \r\n$a_1 + ... + a_n = b_1 + ... + b_n = 1$.\r\n\r\nFind the minimal value of \r\n$ \\frac {a_1^2} {a_1 + b_1} + \\frac {a_2^2} {a_2 + b_2} + ...+ \\frac {a_n^2} {a_n + b_n}$.", "Solution_1": "this has surely been posted on the forum before, because I have seen it :P it's trivial indeed using cauchy.", "Solution_2": "Lets denote with E that expression!\r\n\r\nIt's easy to see that we have: $ E \\geq \\frac{ (\\sum a_{1})^{2}}{\\sum a_{1}+ \\sum b_{1}} =\\frac{1}{2}$\r\nSo the minimum is $\\frac{1}{2}$.\r\n\r\nright?", "Solution_3": "We can easily generalize this:\r\n\r\nLet n be a positive integer and $a_{1},a_{2}, \\ldots,a_{n},b_{1},b_{2},\\ldots, \\b_{n}$ be 2n positive real numbers so that $ \\sum a_{1}= \\sum b_{1}=k$ with k>0 and q $\\in N$ ($ q\\geq2$).\r\n\r\nFind the minimal value of:\r\n\r\n$ \\sum \\frac{a_{1}^{q}}{(a_{1}+b_{1})^{q-1}}$.\r\n\r\nIn the same way we get that: $ \\sum \\frac{a_{1}^{q}}{(a_{1}+b_{1})^{q-1}} \\geq \\frac{( \\sum a_{1})^{q}}{( \\sum a_{1}+ \\sum b_{1})^{q-1}}= \\frac{k^{q}}{(2k)^{q-1}}$. for k=1 and q=2 we have this problem.\r\n\r\ncheers! :D :D", "Solution_4": "This was problem 4...But a TST problem with an one-line solution, as noticed by Valentin, it is maybe really too easy... :( \r\n\r\nPierre.", "Solution_5": "a1 a1b1 a1+b1 \r\n______=a1- ______>=a1- ________ \r\na1+b1 a1+b1 4 \r\n\r\n\r\n\r\nwhen u sum u obtain 1/2 \r\nso easy man\r\nsorry guys", "Solution_6": "[quote=\"pbornsztein\"]Let $n$ be a positive integer, and $a_1,...,a_n, b_1,..., b_n$ be $2n$ positive real numbers such that \n$a_1 + ... + a_n = b_1 + ... + b_n = 1$.\n\nFind the minimal value of \n$ \\frac {a_1^2} {a_1 + b_1} + \\frac {a_2^2} {a_2 + b_2} + ...+ \\frac {a_n^2} {a_n + b_n}$.[/quote]\r\n\r\nIt's really easy. But let's try another solution.\r\n\r\nLet $ \\displaystyle X=\\frac {a_1^2} {a_1 + b_1} + \\frac {a_2^2} {a_2 + b_2} + \\cdots + \\frac {a_n^2} {a_n + b_n}$ and define it's dual $ \\displaystyle Y=\\frac {b_1^2} {a_1 + b_1} + \\frac {b_2^2} {a_2 + b_2} + \\cdots + \\frac {b_n^2} {a_n + b_n}$. Then\r\n\r\n$\\displaystyle X-Y = \\sum_{i=1}^n \\frac {a_i^2-b_i^2} {a_i + b_i} = \\sum_{i=1}^n (a_i-b_i) = 0$. On the other hand\r\n\r\n$\\displaystyle X+Y = \\sum_{i=1}^n \\frac {a_i^2+b_i^2} {a_i + b_i} \\geq \\sum_{i=1}^n\\frac{1}{2}(a_i+b_i) = 1$.\r\n\r\nTherefore $X=Y \\geq \\frac{1}{2}$.", "Solution_7": "Yes, it has been solved like that by some of the contestants. :) \r\nMost of the solvers did with Cauchy.\r\nOne uses partial derivatives for a 3 pages solution :D \r\nOther uses convexity for a function with more than one variable to get a wrong solution. :maybe: \r\nAnd many didn't solve it at all... :( \r\n\r\nPierre.", "Solution_8": "[quote=\"pbornsztein\"]Yes, it has been solved like that by some of the contestants. :) \nMost of the solvers did with Cauchy.\nOne uses partial derivatives for a 3 pages solution :D \nOther uses convexity for a function with more than one variable to get a wrong solution. :maybe: \nAnd many didn't solve it at all... :( \n\nPierre.[/quote]\r\n\r\nLOL :D :D \r\n\r\nBy the way, how many are allowed to take TST? In US, only USAMO winners (max. 12) can take TST.\r\n\r\nLook at problem 11 of http://www.bath.ac.uk/~masgcs/ukimo2004/team_sheet2.pdf\r\n\r\nand at the top of the solutions http://www.bath.ac.uk/~masgcs/ukimo2004/ts2.pdf\r\n\r\nThe same problem was being used for preparation last year by the top nation at IMO 2003\r\n\r\nAnd it was being reused by UK and France for IMO 2004 preparation and TST!", "Solution_9": "Another solution:\r\n\r\nIt's easy to verify that $\\displaystyle \\frac{x^2}{x+y} \\geq \\frac{3}{4}x - \\frac{1}{4}y$ for $x,y>0$.\r\n\r\nSo $\\displaystyle \\sum_{i=1}^n \\frac {a_i^2} {a_i + b_i} \\geq \\frac{3}{4} \\sum_{i=1}^n a_i - \\frac{1}{4} \\sum_{i=1}^n b_i = \\frac{1}{2}$.\r\n\r\nPierre, did anybody use this method? :-)", "Solution_10": "There were around 25 contestants (I do not remember exactly) :\r\nThe main part comes from our correspondance program. We add some from the 'Concours gnral' (which is now a big problem with a lot of questions for a lot of pages, and absolutely not an olympiad type competition), and some students from the lyce Louis-le-Grand, which is one of the best lycees in France (and because the test was organised there).\r\n\r\nFrom what I've said, there is no surprise that each of the 6 members of the french Imo-team were coming from our correspondance program (the others had never seen an olympiad problem).\r\n\r\nPierre.", "Solution_11": "[quote=\"math_sipo\"]a1 a1b1 a1+b1 \n______=a1- ______>=a1- ________ \na1+b1 a1+b1 4 \n\n\n\nwhen u sum u obtain 1/2 \nso easy man\nsorry guys[/quote] you can use latex to write up your stuff. Check out the latex tutorial in the latex help. :D", "Solution_12": "1/2 .easy :lol:", "Solution_13": "By Titu's lemma we have that:\n$$\\sum_{k=1}^{n} \\dfrac{a_k^2}{a_k+b_k} \\ge \\dfrac{\\left(\\sum_{k=1}^{n} a_k \\right)^2}{\\sum_{k=1}^{n} a_k+\\sum_{k=1}^{n} b_k}=\\dfrac{1}{2}$$", "Solution_14": "This problem must be a joke. Using Cauchy-Schwarz we are done!\nI wonder why this problem was problem 4....." } { "Tag": [ "inequalities", "trigonometry" ], "Problem": "The last one for now :D :\r\n\r\nSuppose a, b, c > 0 such that ab + bc + ca = 1. Prove that\r\n\r\n(1 - a)/(1 + a) + (1 - b)/(1 + b) + (1 - c)/(1 + c) <= 3/2.\r\n\r\nHave fun ! :D", "Solution_1": "A quick try, before grobber, Maverick or an other guy from the belgian-romanian very high-skilled gang comes back :mrgreen: :mrgreen: :wink: \r\n\r\n\r\nAs said in a previous message, in a triangle\r\ntgA+tgb+tgC = tgA tgb tgC. \r\nif x = 1/tgA, y = 1/tgB, z=1/tgC,\r\nxy + yz + zx = 1\r\n\r\nAfter a little trigonometry what we have to prove is\r\n\r\n3 - 2(cosA+cosB+cosC) <= 3/2\r\n\r\nor (cosA+cosB+cosC) >= 3/4\r\nor (sinA+sinB+sinC) <= 9/4\r\n\r\n\r\nAlas, sin is [b]not[/b] convex :evil: \r\n\r\nC = :pi: - (A+B)\r\nsinA+sinB+sinC = sinA+sinB+sin(A+B)\r\n\r\nNow I used sin(a+b), sin=1-cos, cos(a+b) to get\r\n\r\nsinA+sinB+sinC = 2 + 2cosAcosBcosC\r\n\r\nAs shown before, cosA+cosB+cosC<=3/2\r\nSo by AM-GM, cosAcosBcosC <= (cosA+cosB+cosC)/3) :^3: <= 1/8 \r\n\r\nPhew !", "Solution_2": "A somewhat shorter way to prove that cos A + cos B + cos C >= 3/4 :\r\n\r\nIn every triangle we have\r\ncos A + cos B + cos C\r\n= 1 - 2 cos A cos B cos C\r\n>= 1 - 2((cos A + cos B + cos C)/3)\r\n>= 1 - 2((3/2)/3) \r\n= 3/4\r\n\r\nwhere cos A + cos B + cos C <= 3/2 follows from Jensen's inequality. \r\n(The fact that\r\ncos A + cos B + cos C + 2 cos A cos B cos C = 1\r\nin every triangle is quite well - known)\r\n\r\nNice one huh ? :D \r\n\r\nTrigonometric substitution can be VERY useful !", "Solution_3": "[quote=\"Arne\"]cos A + cos B + cos C + 2 cos A cos B cos C = 1 in every triangle is quite well - known)\n[/quote]\r\n\r\nQuite well known ?\r\nI didn't know that just 4 hours ago :!:", "Solution_4": "OK then ... You know, there are lots of useful identities like this.\r\nIt's no bad idea to study them for a while ...", "Solution_5": ":Sigma: (1+a^2)/(1+a^2)- :Sigma: 2a^2/(1+a^2)<= 3/2\r\n3-3/2<= :Sigma: 2a^2/(1+a^2)\r\n3/2<=2 :Sigma:a^2/(a^2+1)\r\n3/4<=.... but\r\n :Sigma: a^2/(1+a^2)>=(a+b+c)^2/(3+a^2+b^2+c^2)>=3/4\r\n4(a^2+b^2+c^2)+8(ab+bc+ca)>=9+3(a^2+b^2+c^2)\r\na^2+b^2+c^2>=ab+bc+ca=1\r\n\r\n\r\nHope my soln is good! :lol:", "Solution_6": "That's also correct ... a non - trig solution ! Nice work !" } { "Tag": [ "number theory", "algebra unsolved", "algebra" ], "Problem": "Let $x,y,z,w >0$ positive numbers ;)\r\n\r\nSuch that: \r\n$x^2 + y^2 + z^2 + w^2= 3(x + y + z + w).$\r\n\r\n Fin all solution:$(x,y,z,w)$.\r\n\r\n-----------------\r\nFabiola Bravo :rotfl: \r\nLima -PERU", "Solution_1": "Ani body ??\r\n\r\nAny idea to solve this question??\r\n\r\n\r\n\r\nFabiola Bravo :pilot:\r\nLIma- PERU", "Solution_2": "Looks like number theory but indeed it's more algebra.\r\n\r\n\r\nWlog $x \\leq y \\leq z \\leq w$\r\n$3*x - x^2 \\leq 2$ $\\forall x \\in 1,2,....$\r\nSo $w^2 \\leq 3w+6$ or $w \\leq 4$\r\n\r\nI find 4 solutions with $x \\leq y \\leq z \\leq w$" } { "Tag": [ "Diophantine equation", "number theory proposed", "number theory" ], "Problem": "$ n$ is a positive integer.Prove that $ (2^n \\minus{} 1)(3^n \\minus{} 1)$ isn't a square.", "Solution_1": "Hey,no one is interested in my questions?It's a nice problem! :blush:", "Solution_2": "lemma 1: If $ v_2(3^n\\minus{}1)$ is even then $ n\\equal{}2^k$\r\nLemma 2: $ 3$ divides $ 2^{2^n}\\minus{}1$ but $ 9$ does not.\r\nTwo lemmas above solve your problem!", "Solution_3": "[quote=\"Allnames\"]lemma 1: If $ v_2(3^n \\minus{} 1)$ is even then $ n \\equal{} 2^k$\nLemma 2: $ 3$ divides $ 2^{2^n} \\minus{} 1$ but $ 9$ does not.\nTwo lemmas above solve your problem![/quote]\r\nNo ,the first lemma is wrong . I have a solution for this problem but it 's quite complex . It is very clear that $ n$ must be even ,if not then $ 2\\parallel{}3^n \\minus{} 1$ , so we can write in the form $ n \\equal{} 2k$ . Suppose the contradiction this product is a perfect square ,then exist d,x,y such that :\r\n\\[ (3^k)^2 \\minus{} 1 \\equal{} dx^2 ; (2^k)^2 \\minus{} 1 \\equal{} dy^2\\]\r\nIf d is a perfect square , it is not hard to show that $ k \\equal{} 0$ ,which gives contradiction to the fact that $ n > 0$ . So d is not a perfect square and two equations show that $ 3^k$ and $ 2^k$ are solutions of Pell's equation :\r\n\\[ X^2 \\minus{} dY^2 \\equal{} 1\\]\r\nSuppose the smallest solution of this Pell equation is $ (a,b)$ then all roots of this equation are :\r\n\\[ X_{0} \\equal{} 1,X_1 \\equal{} a ; X_{n \\plus{} 2} \\equal{} 2aX_{n \\plus{} 1} \\minus{} X_n (*)\\]\r\nOur purpose is showing that this sequence can't both contain $ 2^k$ and $ 3^k$ . Suppose exist $ i,j$ such that $ X_i \\equal{} 2^k ,X_j \\equal{} 3^k$ . \r\n[u]Claim 1 [/u]: a must be an even number . \r\nProof : If not then by the relation (*) , all terms of this sequence are odd , which means that it doesn't contain a power of 2 . \r\n[u]Claim 2 : [/u] $ X_{2k \\plus{} 1}\\equiv 1 (\\mod 2 )$and $ X_{2k}\\equiv 0 (\\mod 2)$ for all positive integer $ k$ . \r\nThis fact is so obvious . Notice that we have $ X_i \\equal{} 2^k,X_j \\equal{} 3^k$ , by claim 2 we have $ i$ is even and $ j$ is odd . \r\n[u]Claim 3 [/u]: $ X_{2n \\plus{} 1}\\equiv 0 (\\mod a ),\\forall n\\geq 0$ and $ x_{2k}\\equiv 1, \\minus{} 1 (\\mod a )$ .\r\nProof :We only need to notice that $ X_{n \\plus{} 2}\\equiv \\minus{} X_n (\\mod a)$ and $ X_1\\equiv 0 (\\mod a)$ . \r\nBy claim 2 and claim 3 we have $ x_i\\equiv 0 (\\mod a )$ or $ a|2^k$ ,which means that exist $ t\\leq k$ such that $ a \\equal{} 2^t$ . \r\n[u]Claim 4 :[/u] 3 is a divisor of $ a$ . If not we consider two cases : \r\ni) $ a\\equiv 1 (\\mod 2 )$ then the sequence $ x_n(\\mod 3)$ is {1,1,1,..} ,and it doesn't contain any power of 3 . \r\nii) $ a\\equiv 2 (\\mod 3)$ then the sequence $ x_n(\\mod 3)$ is {1,2,1,2...} , and it also doesn't contain any power of 3 . \r\nSo we must have $ 3|a$ ,which gives contradiction to the fact that $ a \\equal{} 2^k$ .", "Solution_4": "TTsphn has the right idea(very nice one, I must say) and almost the right proof (I think. Of course, the mistake is probably mine).\r\n\r\nBy claim 2 and claim 3 we have that $ a \\equal{} 3^{t}$ and not $ a \\equal{} 2^{t}$ (as $ a | X_{2n\\plus{}1}$) which shortens the proof a little, as that immediately contradicts claim 1.", "Solution_5": "[hide=\"Edit, this doesnt work\"] Nvm, I realized that my solution doesnt work. :oops: [/hide]", "Solution_6": "[quote=\"shenyixin\"]$ n$ is a positive integer.Prove that $ (2^n \\minus{} 1)(3^n \\minus{} 1)$ isn't a square.[/quote]\r\nthis problem is China TST 2002.\r\nYou can read offical solution(I have :D ).", "Solution_7": "To math 10\r\nCan you post your official soluton of it?I think it will be nice material.\r\nThanks.", "Solution_8": "[quote=\"gb2124\"]To math 10\nCan you post your official soluton of it?I think it will be nice material.\nThanks.[/quote]\r\nYes,I will posst them in tomorrow.Becausse Now I verry busy :) \r\nPlease wait.\r\n\r\nPs.Offical solution be a nice proof :D", "Solution_9": "[quote=\"math10\"]this problem is China TST 2002.\nYou can read offical solution(I have :D ).[/quote]\r\nI knew this problem from my teacher,with a solution different from TTsphn's.(TTsphn's is also a nice one :) )\r\nMaybe it's just the official solution.", "Solution_10": "[quote=\"Allnames\"]lemma 1: If $ v_2(3^n \\minus{} 1)$ is even then $ n \\equal{} 2^k$[/quote]\r\nActually\uff0clemma 1 should be:$ v_2(3^n \\minus{} 1)\\equal{}v_2(n)\\plus{}2$", "Solution_11": "While working on this problem I made a MAPLE-assisted observation: if p is a 'large' prime divisor of m^n - 1, it is VERY often the case that p - 1 is divisible by n.\r\n\r\nIs this a well known theorem (with the addition of extra conditions -- like the obvious p > n -- as appropriate)?", "Solution_12": "[quote=\"shenyixin\"][quote=\"Allnames\"]lemma 1: If $ v_2(3^n \\minus{} 1)$ is even then $ n \\equal{} 2^k$[/quote]\nActually\uff0clemma 1 should be:$ v_2(3^n \\minus{} 1) \\equal{} v_2(n) \\plus{} 2$[/quote]\r\n\r\nHello shenyixin can you prove this lemma :) \r\n\r\nthank you !! :lol:", "Solution_13": "This is a direct consequence of the [url=http://reflections.awesomemath.org/2007_3/Lifting_the_exponent.pdf]lifting the exponent lemma[/url]. We have $ v_2(\\frac{3^2 \\minus{} 1}{2}) \\equal{} 2$, so by the lemma $ v_2(3^n \\minus{} 1) \\equal{} 2 \\plus{} v_2(n)$.", "Solution_14": "[quote=\"Zhero\"]This is a direct consequence of the [url=http://reflections.awesomemath.org/2007_3/Lifting_the_exponent.pdf]lifting the exponent lemma[/url]. We have $ v_2(\\frac {3^2 \\minus{} 1}{2}) \\equal{} 2$, so by the lemma $ v_2(3^n \\minus{} 1) \\equal{} 2 \\plus{} v_2(n)$.[/quote]\r\n\r\nThank you \"Zhero\" :lol:", "Solution_15": "See also http://www.mathlinks.ro/viewtopic.php?p=1389586#1389586", "Solution_16": "Regarding #12 (not central to this discussion anyway) I got some [url=http://www.groupsrv.com/science/about443292.html]help[/url] at sci.math that might be of some interest.", "Solution_17": "EDIT: This doesn't work", "Solution_18": "[quote]The number $ (a^n \\minus{} 1)(3^n \\minus{} 1)$ is not a perfect square $ \\forall a \\geq 2,n\\geq1$. [/quote]\r\n\r\nWell, you also need $ a \\neq 3$, for sure! In fact for your use of the Pythagorean triplets to work it is good to have $ a$ even ... but for sure it's a short and sweet proof for $ a \\equal{} 2$ (the original problem). However, at second thought, I think that the co-primality of the elements of that Pythagorean triple also needs to be argued." } { "Tag": [ "algebra", "polynomial", "quadratics", "number theory", "relatively prime" ], "Problem": "There is a polynomial $f(x)$ of degree $n$. The remainder is $1$ when $(x-1)$ divides it. When $(x-3)$ divides $f(x)$, the remainder is $3$. When $(x-5)$ divides $f(x)$, the remainder is $21$. \r\n\r\nWhat would be the remainder when $f(x)$ is divided by $(x-1) \\cdot (x-3) \\cdot (x-5)$ ?", "Solution_1": "[hide=\"Method 1\"] The remainder is some quadratic polynomial, and you can plug in $x = 1, 3, 5$ to solve for its coefficients. [/hide] [hide=\"Method 2\"] The Chinese Remainder Theorem applies to polynomials. ;) [/hide]", "Solution_2": "[quote=\"t0rajir0u\"][hide=\"Method 1\"] The remainder is some quadratic polynomial, and you can plug in $x = 1, 3, 5$ to solve for its coefficients. [/hide] [hide=\"Method 2\"] The Chinese Remainder Theorem applies to polynomials. ;) [/hide][/quote]\n\n[hide]Do you mean to say that the problem cannot be solved definitively? I tried with CRM, but could not complete it since $n$ is variable and unknown. :( [/hide]", "Solution_3": "well...when you divide it by a first degree polynomial, the remainder is constant. what does that say about division by a third degree polynomial?\r\n\r\n\r\nedit: tor already mentoined that its second degree", "Solution_4": "[quote=\"xxxyyyy\"]I tried with CRM, but could not complete it since $n$ is variable and unknown. :([/quote]\r\n\r\n$n$ is irrelevant. (In fact, you can assume WLOG that $n = 2$, which makes your life [b]very[/b] easy ;) )\r\n\r\nThe Chinese Remainder Theorem is a theorem about two things:\r\n\r\n- A modulus (rather, a product of relatively prime moduli)\r\n- A remainder (rather, a set of remainders)\r\n\r\nBoth are definite in this problem.", "Solution_5": "[hide]\nUm, if I remember and if I have time, I'll type my solution in latex. Anyway, I just did what tor hinted at. I got $2x^{2}-7x+4$.[/hide]" } { "Tag": [ "LaTeX", "calculus", "calculus computations" ], "Problem": "If f(x)= m (subscript 1)x+b(subscript 1) and g(x)= m(subscript 2)x+b(subscript 2), prove (f(g(x)))'= f '(x) times g '(x).\r\n\r\n\r\n\r\nI found out that (fog)'= f ' g(x) times g ' (x) and that f '(x)= m (subscript 1) and g '(x)= m (subscript 2) but not everything works out right.\r\n\r\nPlease help me. Thanks!", "Solution_1": "[quote]If $ f(x) \\equal{} m_1 x \\plus{} b_1$ and $ g(x) \\equal{} m_2 x \\plus{} b_2$, then prove that $ \\left(f(g(x))\\right)' \\equal{} f'(x) \\cdot g'(x)$.[/quote]\r\n$ f'(x) \\equal{} m_1$, $ g'(x) \\equal{} m_2$, thus $ f'(x) \\cdot g'(x) \\equal{} m_1m_2$.\r\n\r\n$ \\frac{d}{dx}\\left[f(g(x))\\right] \\equal{} f'(g(x))g'(x) \\equal{} m_1 m_2$.\r\n\r\nNote that $ f'(g(x)) \\equal{} m_1$ as $ f'(x) \\equal{} m_1$.\r\n\r\nWhat's not working out?", "Solution_2": "wouldn't f'(g(x))= m(subscript 2)?\r\n\r\nbecause it would be f'(m (subscrpt 2)x+b(subscript 2)) which then equals m (subscript 2)", "Solution_3": "If you don't want to use $ \\text{\\LaTeX}$, commonly accepted shorthand for \"a subscript b\" is a_b. Wrapping this in dollar signs gives you $ a_b$. Just for future reference.\r\n\r\nAnyway, $ f(x)=m_1x+b_1$, so $ f'(x)=m_1$. This is a constant function. It is a horizontal line. Regardless of what value of $ x$ you put in there, you're going to get $ m_1$ as the output. Therefore, $ f'(g(x))$ is simply $ m_1$.", "Solution_4": "$ \\frac{d}{dx}\\left[f(g(x))\\right] \\neq f'(g(x))$" } { "Tag": [ "number theory", "greatest common divisor", "least common multiple", "prime factorization" ], "Problem": "it can be proven that where (a,b) means the gcd of a and b, and [a,b] means the lcm of a and b\r\n\r\n$(a,b)[a,b]=ab$\r\n\r\n$(a,b,c)[ab,bc,ca]=abc$\r\n\r\n$(ab,bc,ca)[a,b,c]=abc$\r\n\r\ncan you find more complex identites like these, or ones which generalize to arbitratily many variables?", "Solution_1": "[hide=\"Solution\"] Let $e_{p}(n)$ denote the greatest $k$ such that $p^{k}| n$. \n\n[b]Lemma:[/b] $n = m \\Leftrightarrow e_{p}(n) = e_{p}(m) \\forall p \\in \\mathbb{P}$\n\n[b]Lemma:[/b] $e_{p}(nm) = e_{p}(n)+e_{p}(m)$\n\n[b]Lemma:[/b] $gcd(a, b, c, ...) = \\prod_{p \\in \\mathbb{P}}p^{min(e_{p}(a), e_{p}(b), e_{p}(c), ...)}$\n\n[b]Lemma:[/b] $lcm(a, b, c, ...) = \\prod_{p \\in \\mathbb{P}}p^{max(e_{p}(a), e_{p}(b), e_{p}(c), ...)}$\n\nHence the following gcd identities reduce to the simple identities\n\n$min(x, y)+max(x, y) = x+y$\n$min(x, y, z)+max(x+y, y+z, z+x) = x+y+z$\n$max(x, y, z)+min(x+y, y+z, z+x) = x+y+z$\n\nWhere $x = e_{p}(a), y = e_{p}(b), z = e_{p}(c)$ for some unspecified prime $p$. It is easy to see where these identities come from; simply assume WLOG $x \\ge y \\ge z$.\n\nSimilarly, arbitrary generalizations can be had in the following way:\n\nGiven $x_{1}\\ge x_{2}\\ge ... \\ge x_{n}$ with sum $S$ we have the identities\n\n$min(x_{i})+max(S-x_{i}) = S$\n$min(x_{i}+x_{j})+max(S-x_{i}-x_{j}) = S$\n...\n\nThis chain of identities is obvious because by our WLOG assumption we have\n\n$min(x_{i}) = x_{1}, max(S-x_{i}) = S-x_{1}$\n$min(x_{i}+x_{j}) = x_{1}+x_{2}, max(S-x_{i}-x_{j}) = S-x_{1}-x_{2}$\n...\n\nAnd so forth. [/hide]", "Solution_2": "i don't understand all that fancy notation :blush: \r\n\r\ncan you write out the identities with the (a,b) and [a,b] notation i used?", "Solution_3": "They come out to what you expect. For example, the four-variable identities are given by\r\n\r\n$(a, b, c, d)[abc, bcd, cda, dab] = abcd$\r\n$(ab, ac, ad, bc, bd, cd)[ab, ac, ad, bc, bd, cd] = abcd$\r\n$(abc, bcd, cda, dab)[a, b, c, d] = abcd$\r\n\r\nEdit: Let me explain myself a little more.\r\n\r\n[hide=\"Slower this time\"] Let me explain the two-variable case using $e_{p}(n)$.\n\n$e_{p}(n)$ is just a convenient notation for the exponents in the prime factorization of $n$. That is, we have\n\n$n = 2^{e_{2}(n)}3^{e_{3}(n)}5^{e_{5}(n)}7^{e_{7}(n)}...$\n\nGiven two numbers $a, b$ with some values of $e_{p}(a), e_{p}(b)$, $gcd(a, b)$ is the unique positive integer that satisfies\n\n$e_{p}( gcd(a, b) ) = min(e_{p}(a), e_{p}(b))$\n\nFor each prime $p$ (because $x | y$ if and only if $e_{p}(x) \\le e_{p}(y)$ for each prime; hence we require that $e_{p}( gcd(a, b)) \\le e_{p}(a), e_{p}(b)$ for each prime). \n\nOn the other hand, and for the same reason, $lcm(a, b)$ is the unique positive integer that satisfies\n\n$e_{p}( lcm(a, b) ) = max(e_{p}(a), e_{p}(b))$\n\nWhen multiplying two numbers we simply add the exponents in their prime factorizations; in other words,\n\n$e_{p}(xy) = e_{p}(x)+e_{p}(y)$\n\nHence, taking the identity\n\n$(a, b) [a, b] = ab$\n\nAnd comparing the exponents of the prime factorizations of both sides (which must be equal) we get the identity\n\n$min(e_{p}(a), e_{p}(b))+max(e_{p}(a), e_{p}(b)) = e_{p}(a)+e_{p}(b)$\n\nWhich is easy to show for the reasons I gave in my first post, and easy to generalize. \n\nThe sequence of min-max identities corresponds to a sequence of $gcd, lcm$ identities. [/hide]" } { "Tag": [ "AMC", "AIME", "AMC 10", "AMC 12 A", "AMC 10 A" ], "Problem": "At least 5% of students participating in the 2007 AMC 12 A scored 97.5 or more, so the AIME qualifying score on the 2007 AMC 12 A is 97.5.\r\n\r\nAt least 1% of students participating in the 2007 AMC 10 A scored 117.0 or more, so the AIME qualifying score on the 2007 AMC 10 A is 117.0.\r\n\r\nWe still have much to do in the AMC office with the scoring and tabulating. AMC 10 A and AMC 12 A school reports will go out to contest managers approximately Friday, Feb 23, 2007 assuming all continues to go well with no problems.\r\n\r\nPlease note: This is an extremely busy time at the AMC office. As a result, I will not answer any inquiries, specific or general at any time. Any inquiry will be immediately deleted. Your contest manager will receive your official score report when we are able to send them.\r\n\r\nSteve Dunbar\r\nAMC Director", "Solution_1": "A couple more things regarding this:\r\n\r\n- a thread for discussion of this announcement (though I'm not sure what there is to \"discuss\") is [url=http://www.artofproblemsolving.com/Forum/viewtopic.php?t=133662]here[/url]. \r\n\r\n- When Mr. Dunbar says \"Any inquiry will be immediately deleted\", he means any inquiry that you send to the AMC offices. You can still post questions here, and though I doubt Mr. Dunbar will have time to respond to them, an AoPS user or admin might know the answer. But [b]please[/b], do at least a minimal amount of research before asking a question; for most questions that I see here, the answer is readily obtainable by visiting the [url=http://www.unl.edu/amc]AMC website[/url]. \r\n\r\n- If you doubt how busy the AMC is, check out [url=http://www.unl.edu/amc/e-exams/e6-amc12/e6-1-12archive/2007-12a/07-1-1012-dir,pic.shtml]these photos[/url].", "Solution_2": "By the way, thanks very much to the AMC Director for posting this information!" } { "Tag": [ "vector", "function" ], "Problem": "Find all $f(x)$ that map the reals to the reals such that $f(2x)=2f(x)-2x^{2}$.", "Solution_1": "Substitute $f(x)=g(x)-x^{2}$ to get $g(2x)=2g(x)$. Hence $g(x)$ is linear, giving $f(x)=ax-x^{2}, a\\in\\mathbb{R}$", "Solution_2": "[quote=\"Farenhajt\"]Substitute $f(x)=g(x)-x^{2}$ to get $g(2x)=2g(x)$. Hence $g(x)$ is linear, giving $f(x)=ax-x^{2}, a\\in\\mathbb{R}$[/quote]\r\n\r\nThat is not necessarily true. Only if $g$ is continous at 0. Because the solution to the Cauchy's equation $f(x+y) = f(x)+f(y)$ has non-trivial (non-linear solutoions). If one accepts the axiom of choice. He can contruct something called the \"Hamel Basis\". It is the basis for the vector space of the reals over the rationals. So it does not need to be linear.", "Solution_3": "Let $h(x)$ and $j(x)$ be two functions that map $[0,1)$ to the reals.\r\n\r\nThen, a generic solution is given by $g(x)=\\left \\{ \\begin{array}{ll}0&{ \\text{if}\\ \\ x = 0}\\\\ h( \\{ log_{2}x \\})x&{ \\text{if}\\ \\ x>0}\\\\ j( \\{ log_{2}(-x) \\})x&{ \\text{if}\\ \\ x < 0}\\end{array}\\right.$\r\n\r\n$g(x)$ is equivalent to a linear function iff. $j(x)=h(x)=k$ for some real number $k$. However, the linear case is the only case in which $g$ (and therefore $f$) is continuous at $0$." } { "Tag": [ "geometry", "geometry unsolved" ], "Problem": "The incircle of an isosceles triangle $ABC$ with $AB = AC$ is centered at $O$ and touches $BC, CA, AB$ at $K, L, M$ , respectively. Lines $KM$ and $OL$ meet at $N$ , and line $BN$ meets $CA$ at $Q$. Let $P$ be the projection of $A$ onto $BQ$. Suppose that $BP = AP+2PQ$. Find all possible values of $\\frac{AB}{BC}.$", "Solution_1": "http://www.mathlinks.ro/Forum/viewtopic.php?t=5830" } { "Tag": [ "\\/closed" ], "Problem": "We have recently made some changes on our servers, and while we tried to make sure we haven't missed anything, given the size of our site, we might have. So if you see any missing images (that were here before) or files, please let us know.\r\n\r\nAlso, the wiki is back online.", "Solution_1": "It looks like transcripts of recent Math Jams are missing.\r\n\r\n(Separate point: On the transcripts page, it would be nice if the Jam Date listed the year in addition to the month and day.)" } { "Tag": [ "number theory", "relatively prime", "Diophantine equation", "number theory proposed" ], "Problem": "For every positive integer $ n$, $ b(n)$ denote the number of positive divisors of $ n$ and $ p(n)$ denote the sum of all positive divisors of $ n$. For example, $ b(14)\\equal{}4$ and $ p(14)\\equal{}24$. Let $ k$ be a positive integer greater than $ 1$.\r\n\r\n(a) Prove that there are infinitely many positive integers $ n$ which satisfy $ b(n)\\equal{}k^2\\minus{}k\\plus{}1$.\r\n\r\n(b) Prove that there are finitely many positive integers $ n$ which satisfy $ p(n)\\equal{}k^2\\minus{}k\\plus{}1$.", "Solution_1": "(a) it is enough to take $ n\\equal{}p^{k^2\\minus{}k}$,where $ p$ is a prime number.\r\n(b) Obviously if $ n>k^2\\minus{}k\\plus{}1$,then $ p(n)>n>k^2\\minus{}k\\plus{}1$,am i miss something?", "Solution_2": "[quote=\"Erken\"](b) Obviously if $ n > k^2 \\minus{} k \\plus{} 1$,then $ p(n) > n > k^2 \\minus{} k \\plus{} 1$,am i miss something?[/quote]Why is $ n>k^2\\minus{}k\\plus{}1$? Did you notice that I wrote [b]finitely many[/b], not infinitely many? :wink:", "Solution_3": "[quote=\"Johan Gunardi\"][quote=\"Erken\"](b) Obviously if $ n > k^2 \\minus{} k \\plus{} 1$,then $ p(n) > n > k^2 \\minus{} k \\plus{} 1$,am i miss something?[/quote]Why is $ n > k^2 \\minus{} k \\plus{} 1$? Did you notice that I wrote [b]finitely many[/b], not infinitely many? :wink:[/quote]\r\nI take enough large $ n$,in our case it is $ n>k^2\\minus{}k\\plus{}1$,so that our equality can not be true,so there is no $ n>k^2\\minus{}k\\plus{}1$,such that $ p(n)\\equal{}k^2\\minus{}k\\plus{}1$,that's why there are finitely many $ n$.", "Solution_4": "[quote=\"Erken\"][quote=\"Johan Gunardi\"][quote=\"Erken\"](b) Obviously if $ n > k^2 \\minus{} k \\plus{} 1$,then $ p(n) > n > k^2 \\minus{} k \\plus{} 1$,am i miss something?[/quote]Why is $ n > k^2 \\minus{} k \\plus{} 1$? Did you notice that I wrote [b]finitely many[/b], not infinitely many? :wink:[/quote]\nI take enough large $ n$,in our case it is $ n > k^2 \\minus{} k \\plus{} 1$,so that our equality can not be true,so there is no $ n > k^2 \\minus{} k \\plus{} 1$,such that $ p(n) \\equal{} k^2 \\minus{} k \\plus{} 1$,that's why there are finitely many $ n$.[/quote]Okay. Then why did you write 'am i miss something?'?", "Solution_5": "dont we have to show the existance of atleast one such n ?", "Solution_6": "[quote=\"gagan_goku\"]dont we have to show the existance of atleast one such n ?[/quote]Actually I think so. But the official solution does not mention it. Erken's solution is essentially the same as the official solution.", "Solution_7": "but if there are no such n , shouldnt we say there is no such n\r\ndont know , you can also count 0 in finite numbers. anyways , someone please tell me how to increase my brain activity.\r\ni feel really good when im able to solve some nice problem. but im too dumb :(", "Solution_8": "$ 0$ is a finite number. \r\n\r\nThere are solutions for some $ k$: $ p(p^2) \\equal{} p^2 \\plus{} p \\plus{} 1 \\equal{} (p \\plus{} 1)^2 \\minus{} (p \\plus{} 1) \\plus{} 1$ for $ p$ a prime. There aren't solutions for every $ k$; I believe there are no solutions for $ k \\equal{} 5$, for example. (Additionally, $ p(k^2 \\minus{} k \\plus{} 1) \\equal{} k^2 \\minus{} k \\plus{} 1$ whenever $ k^2 \\minus{} k \\plus{} 1$ is prime, but it's an open question whether infinitely many such primes exist.)\r\n\r\nEdit: Two other sporadic solutions. $ p(36) \\equal{} 91 \\equal{} 10^2 \\minus{} 10 \\plus{} 1$ and $ p(16) \\equal{} 31 \\equal{} 6^2 \\minus{} 6 \\plus{} 1$. If two consecutive values $ k, k \\plus{} 1$ have (relatively prime) solutions, then so does $ k^2 \\plus{} 1$.\r\n\r\nEdit 2: When $ n$ is a power of two, we are trying to solve the Diophantine equation $ 2^a \\minus{} 1 \\equal{} k^2 \\minus{} k \\plus{} 1 \\implies 2^a \\equal{} k^2 \\minus{} k \\plus{} 2 \\implies 2^{a \\plus{} 2} \\equal{} 4k^2 \\minus{} 4k \\plus{} 8 \\equal{} (2k \\minus{} 1)^2 \\plus{} 7$. The solutions here are given by the solutions to [url=http://mathworld.wolfram.com/RamanujansSquareEquation.html]Ramanujan's Square Equation[/url], which here gives the rather large solution $ k \\equal{} 91, n \\equal{} 2^{a \\minus{} 1} \\equal{} 2^{13}$ (in addition to what we found above). This is a difficult equation to solve, so it seems that even the case that $ n$ is a power of a prime is not easy." } { "Tag": [ "inequalities", "inequalities unsolved" ], "Problem": "Let $ x,y,z>0$, show\r\n$ (\\frac{x}{2x\\plus{}y})^3\\plus{}(\\frac{y}{2y\\plus{}z})^3\\plus{}(\\frac{z}{2z\\plus{}x})^3\\geq \\frac{1}{9}$.", "Solution_1": "Bump...\r\n\r\nAlthough I'm not the original poster, I would very much like to know the solution. :)", "Solution_2": "The problem is equivalent to show $ (\\frac {1}{2 \\plus{} a})^3 \\plus{} (\\frac {1}{2 \\plus{} b})^3 \\plus{} (\\frac {1}{2 \\plus{} b})^3\\ge 1/9$, where $ a,b,c\\in R_ \\plus{}$ and $ abc \\equal{} 1$. \r\nIt remains to show $ 2(\\frac {1}{2 \\plus{} a})^3 \\plus{} (\\frac {1}{2 \\plus{}1/a^2} )^3 \\ge 1/9$ for $ a > 0$.", "Solution_3": "[quote=\"miwalin\"]The problem is equivalent to show $ (\\frac {1}{2 \\plus{} a})^3 \\plus{} (\\frac {1}{2 \\plus{} b})^3 \\plus{} (\\frac {1}{2 \\plus{} b})^3\\ge 1/9$, where $ a,b,c\\in R_ \\plus{}$ and $ abc \\equal{} 1$. \nIt remains to show $ 2(\\frac {1}{2 \\plus{} a})^3 \\plus{} (\\frac {1}{2 \\plus{} 1/a^2} )^3 \\ge 1/9$ for $ a > 0$.[/quote]\r\nIt should be \r\n$ (\\frac {1}{2 \\plus{} a})^3 \\plus{} (\\frac {1}{2 \\plus{} b})^3 \\plus{} (\\frac {1}{2 \\plus{} c})^3\\ge 1/9$, where $ a,b,c\\in R_ \\plus{}$ and $ abc \\equal{} 1$. \r\nIt remains to show $ 2(\\frac {1}{2 \\plus{} a})^3 \\plus{} (\\frac {1}{2 \\plus{} 1/a^2} )^3 \\ge 1/9$ for $ a > 0$. \r\nSorry for carelessness.", "Solution_4": "The inequality can be proved in the following way:\r\n$ \\large\\begin{array}{rcl}\r\n\\frac{2}{(2\\plus{}a)^3}\\plus{}\\frac{1}{\\left (2\\plus{}\\frac{1}{a^2}\\right )^3}&\\ge &\\frac{1}{9}\\qquad\\Leftrightarrow\\\\\r\n& & \\\\\r\n\\frac{a^9\\plus{}6a^8\\plus{}12a^7\\plus{}24a^6\\plus{}24a^4\\plus{}12a^2\\plus{}2}{(2\\plus{}a)^3(2a^2\\plus{}1)^3}&\\ge &\\frac{1}{9}\\qquad\\Leftrightarrow\\\\\r\n& & \\\\\r\n9\\left (a^9\\plus{}6a^8\\plus{}12a^7\\plus{}24a^6\\plus{}24a^4\\plus{}12a^2\\plus{}2\\right )\\minus{}(2\\plus{}a)^3(2a^2\\plus{}1)^3&\\ge &0\\qquad\\Leftrightarrow\\\\\r\n& & \\\\\r\na^9\\plus{}6a^8\\plus{}80a^6\\minus{}150a^5\\plus{}84a^4\\minus{}73a^3\\plus{}54a^2\\minus{}12a\\plus{}10&\\ge &0\\qquad\\Leftrightarrow\\\\\r\n& & \\\\\r\n(a\\minus{}1)^2\\cdot (a^7\\plus{}8a^6\\plus{}15a^5\\plus{}102a^4\\plus{}39a^3\\plus{}60a^2\\plus{}8a\\plus{}10)&\\ge &0.\r\n\\end{array}$\r\nBut this is true for all $ a>0$." } { "Tag": [ "inequalities" ], "Problem": "Find all positive real solutions to x^3 + y^3 + z^3 = x + y + z, x^2 + y^2 + z^2 = xyz.", "Solution_1": "Such systems of equations having more variables than equations are usually solved using inequalities, especially when you are given the information that x, y, z must be positive.\r\n\r\nIn fact, from your two equations\r\n\r\n$x^{3}+y^{3}+z^{3}=x+y+z$\r\n\r\nand\r\n\r\n$x^{2}+y^{2}+z^{2}=xyz$\r\n\r\nit follows that\r\n\r\n$\\left( x^{3}+y^{3}+z^{3}\\right) \\left( x^{2}+y^{2}+z^{2}\\right) ^{2}=\\left( x+y+z\\right) \\left( xyz\\right) ^{2}$.\r\n\r\nBut for any three positive numbers x, y, z we have\r\n\r\n$\\left( x^{3}+y^{3}+z^{3}\\right) \\left( x^{2}+y^{2}+z^{2}\\right) ^{2}>\\left( x+y+z\\right) \\left( xyz\\right) ^{2}$,\r\n\r\nsince\r\n\r\n$\\left( x^{3}+y^{3}+z^{3}\\right) \\left( x^{2}+y^{2}+z^{2}\\right) ^{2}$\r\n$=\\left( x^{3}+y^{3}+z^{3}\\right) \\left( x^{4}+y^{4}+z^{4}+2y^{2}z^{2}+2z^{2}x^{2}+2x^{2}y^{2}\\right) $\r\n$>\\left( x^{3}+y^{3}+z^{3}\\right) \\left( y^{2}z^{2}+z^{2}x^{2}+x^{2}y^{2}\\right) $\r\n$=x^{3}\\left( y^{2}z^{2}+z^{2}x^{2}+x^{2}y^{2}\\right) +y^{3}\\left( y^{2}z^{2}+z^{2}x^{2}+x^{2}y^{2}\\right) +z^{3}\\left( y^{2}z^{2}+z^{2}x^{2}+x^{2}y^{2}\\right) $\r\n$>x^{3}y^{2}z^{2}+y^{3}z^{2}x^{2}+z^{3}x^{2}y^{2}=\\left( x+y+z\\right) \\left( xyz\\right) ^{2}$.\r\n\r\nTherefore, we get contradiction, and your system does not have any positive solutions.\r\n\r\n Darij" } { "Tag": [ "Stanford", "college", "function", "geometry", "algebra", "floor function" ], "Problem": "Did anyone attend this contest today?", "Solution_1": "yea i did, took the general, didn't stick around for the awards and solutions though, so i don't know what i got", "Solution_2": "yeah, i put n^2 instead of m^2 too...grrrr... and on the floor function one... i did the exact opposite function and got x to equal 3 instead of 2. how uncool. i was surprised i actually got 10th though....\r\n\r\ni bombed geometry too... haha.. but we won't go into that..." } { "Tag": [ "probability", "calculus", "calculus computations" ], "Problem": "Q1 Find probability of getting 3 consecutive sixes in 10 throws of a die.\r\n\r\nQ2 A die is thrown again and again until we obtain 2 consecutive sixes are obtained .find the probabiltiy of getting 2 consecutive sixes", "Solution_1": "Neither of these are calculus problems, and the second is nonsensical. For the first, a standard attack with recursions or equivalently state diagrams works nicely to compute the probability of the complement. See [url=http://www.artofproblemsolving.com/Forum/viewtopic.php?t=190619]here[/url] for one example of the method; searching for the words \"state diagram*\" (e.g., in the High School Forums) should find more." } { "Tag": [ "algebra", "polynomial", "superior algebra", "superior algebra open" ], "Problem": "This is a shameless reposting from MathOverflow but the question is extremely beautiful and I really want to know what mathlinkers can make of it. I have already spent 3 days thinking of it myself but with very little progress :wallbash:.\r\n\r\nDo there exist an infinite field $ k$ and a polynomial $ p\\in k[x]$ such that the image of $ k$ under the mapping $ x\\mapsto p(x)$ is not all $ k$ but its complement in $ k$ is finite?\r\n\r\nI can only show that if $ \\text{char\\,} k\\ne 2,3$, then we must have $ \\text{deg\\,}p\\ge 4$ and handle a few very special polynomials of degree $ 4$ and $ 5$. :(", "Solution_1": "I agree it's a great question! Just for the record the MO question is [url=http://mathoverflow.net/questions/6820/can-a-non-surjective-polynomial-map-from-an-infinite-field-to-itself-miss-only-fi]here[/url] and the comments there are certainly helpful, even if they don't give a complete answer." } { "Tag": [ "counting", "derangement", "complementary counting" ], "Problem": "hi again,\r\n\r\nLast time i posted a qstn on a derangement problem. BUt how about a variation like this?. I have two vatying answers from two different methods. Can anyone confirm either of them? \r\n\r\nLetters A,B, C D E F are arranged in order such taht A is not in the 1st positon, B is not in the 2nd position, and F is not in the 6th position. How many possible ways are there of arranging the letters?", "Solution_1": "[hide=\"Method of attack\"]For each of C, D and E, consider whether it is in its \"natural place.\" This gives you eight (or four with an obvious simplification) cases (representative example: C and E are in their natural places, but D is not), and each is an instance of the derangement problem for some number of letters.[/hide]\nA more general version of this problem is the following:\n\nSuppose we have $ n$ letters with $ k$ special letters. How many permutations are there of these $ n$ letters such that none of the $ k$ special letters occurs in its natural place?\n[hide=\"Answer to the general version of this problem\"]$ \\sum_{j \\equal{} 0}^{n \\minus{} k} \\binom{n \\minus{} k}{j} D_{k \\plus{} j}$. Here $ j$ counts the number of non-special letters that don't go to their natural place. $ \\binom{n \\minus{} k}{j}$ chooses every possible subset of them, while $ D_{k \\plus{} j}$ arranges them and the $ k$ special letters so that none goes to the natural place. (The other $ n \\minus{} k \\minus{} j$ letters have only one choice each, their natural places.)[/hide]\r\nDoes this answer simplify?", "Solution_2": "The way I'm reading the problem (which is most likely wrong, considering this is in the pre-olympiad forum :P) is that we could fix A,B,F and check the number of permutations that way ($ 3!$) and subtract that from the total number of permutations. So our answer would be $ 6!\\minus{}3!$. What is it that I'm missing that leads JBL to his solution?", "Solution_3": "[quote=\"max_tm\"]we could fix A,B,F[/quote]\r\n\r\nNope. The complement of the case that none of $ A, B, F$ are in their natural position is that [b]at least one of[/b] $ A, B, F$ is in its natural position, not all of them.", "Solution_4": "Sorry torajirou but could you elaborate on that a bit? It's not coming as immediately intuitive to me.", "Solution_5": "With no conditions, there are $ 6!$ permutations. We are considering the subset of these permutations that satisfies [b]all[/b] of the following three conditions:\r\n\r\n- $ A$ is not in the first position.\r\n- $ B$ is not in the second position.\r\n- $ F$ is not in the sixth position.\r\n\r\nThe number of permutations fixing $ A, B, F$ in the first, second, and sixth positions is the number of permutations that satisfies [b]none[/b] of the above three conditions. However, your logic completely ignores the permutations that satisfy [b]some, but not all[/b] of the above three conditions.", "Solution_6": "Ah, I see. Could we patch up my logic? As in counting:\r\n\r\n- When A is fixed\r\n- When B is fixed\r\n- When C is fixed\r\n- When A,B is fixed\r\n- When B,C is fixed\r\n- When A,C is fixed\r\n- When all three of A,B,C are fixed\r\n\r\n.... Although I would never be proud of a solution like that :P.", "Solution_7": "A slight modification of that idea (to avoid overcounting) works, but each of those individual cases is just as hard as the original problem (I think, except for the last). Look at JBL's idea.", "Solution_8": "max_tm's idea should work for a perfectly respectable solution by inclusion-exclusion. If we interpret \"A is fixed\" as \"A is fixed and we permute the others however we like,\" then each individual case is simple and inclusion-exclusion takes care of the overcounting. It might even result in a simpler expression for the final sum -- give it a try!", "Solution_9": "yehh guys. i think i have it. I seem to be getting 426 with 3 different approaches. \r\n\r\nAs you guys mentioned, one method is complementary counting. It is not as complicated as it seems. Working through each of the cases, we want to subtract all subsets with at least one of A B or F in their correct position (ie, only A, B, F is in their correct position, only, AB, AF, and BF are in their correct positions, and when all A E F are in their correct positions). \r\n\r\n1. So now, for the case where all ABF are in correct position is simple; if we treat positions 1,2 and 6 as one group and then the rest as another, we get that there is only one way to permute ABF in positions 1,2 ,6 such that they are all in the correct position, and then there are 3! ways of permuting the others in the other 3 positions). \r\n\r\n2. Now, when only one of A E or F are in their correct positions, there are 3 ways of choosing which to be in their correct positions only. Now for the other two who are not to be in their fixed position (ie say i chose A to be in its fixed position, so B and F can either be in the group positon 1,2 6 and not in their position in various permutations)\r\ni) when all 2 others that are not to be in their fixerd positions are not to be in the 1,2 6 position group, the number of ways is 3*2*3!. (since two of the other 3 letters C D or E must be placed in the remaining 2 unoccupied positions in 1,2 6)\r\nii) when only 1 other that are not to be in their fixed positions are not to be in 1,2 6 position group, the number of ways is 2*3*3! (this time, the 2 meaning there are two choices of which of the 2 unfixed numbers are placed in the 126 positonal group)\r\nii) when all the other 2 unfixed letters are to be placed in the 1,2,6 group --> 3! (since there is only one way of derranging the 2 unfixed letters such that they dont appear in their forbidden positions)\r\n\r\nNow for this case, (from i), ii), iii)), you get 3(2*3*3! + 2*3*3! +3!)\r\n\r\n3. Now, when all 2 other unfixed letters are to be in their correct positions, then there would 3(3*3!) ways\r\n\r\nHence the total number of arrangements are 6! - 3! - 3(3*3!) - 3(2*3*3! + 2*3*3! +3!) = 426. \r\n\r\nI understand this is a rather longwinded and elaborate method, which is not very desirable. But a better method is applying the Principle of Inclusion and Exclusion. and u simply get: 6! - (3C1)(5!) + (3C2)(4!) - (3C3) (3!) = 426.\r\n\r\nA THIRD METHOD, can follow along the lines of the first method, but not do complementary counting, but rather look at the 4 cases, again treating positions 1,2 and 6 as one group and the other positions as another (ie. when all A B F are in the 126 group, when only 2 of them are in the 126 group, when only 1 of them are in the 126 group, and when all of them are out of the 126 group). THis method givse:\r\n\r\n2*3! + (3!)(3!) + (3C1)(2)(3*2)(3!) + (3C2)(3)(3)(3!) = 426\r\n\r\nI would assume from 3 different methods, the answer should be 426. Funnily enough, from my first attempt, i achieved answers of 372 from my third method, and 480 for my 1st method. And then i did my second, and got 426, and realised how my previous 2 methods, the first was down by 54 and the other was up by 54. I thought this could not be a coincidence, and so i thought since one method (the first) was subtracting cases, and the last was adding cases (as methods 1 and 3 are mirrors of each other), I realised i must have made the same mistake for both! ....and eventually i found it LOL!).", "Solution_10": "Just for your satisfaction (if you need it), your answer is definitely right :)\r\n\r\n[hide][code]p = Permutations[{1, 2, 3, 4, 5, 6}];\nTotal[Table[Which[(p[[n]][[1]] - 1)*(p[[n]][[2]] - 2)*(p[[n]][[6]] - 6) == 0, 0,True, 1], {n, 1, Length[p]}]][/code][/hide]" } { "Tag": [ "function", "integration", "calculus", "calculus computations" ], "Problem": "[color=darkred]$ 1\\blacktriangleright a\\in\\mathcal R$ and a continue monoton function $ f: [0,1]\\rightarrow\\mathcal R$ $ \\implies$ $ \\int_{0}^{1}\\left|f(x)\\minus{}a\\right|\\ dx\\ge\\int_{0}^{1}\\left|f(x)\\minus{}f\\left(\\frac{1}{2}\\right)\\right|\\ dx$ ([b]level ****, i.e. difficult ![/b]).\n$ 2\\blacktriangleright$ A continue function $ f: [a,b]\\rightarrow\\mathcal R$ $ \\implies$ exists $ c\\in (a,b)$ so that $ \\int_{a}^{c}f(x)\\ dx\\plus{}\\int_{b}^{c}f(x)\\ dx\\equal{}(a\\plus{}b\\minus{}2c)\\cdot f(c)$ ([b]level **, i.e. easy ![/b]).[/color]", "Solution_1": "$ 1.$ First we assume $ f$ is an increasing function (if $ f$ satisfies $ \\rightarrow\\minus{}f$ satisfies the conditions with $ \\minus{}a$). Then it's easy to see that it's enough to prove the statement for $ a\\in[f(0),f(1)]$. Then let $ f(b) \\equal{} a$, and suppose $ b\\ge\\frac{1}{2}$ (for $ b <\\frac{1}{2}$ it's the same). Then we have:\r\n$ \\int_{0}^{\\frac{1}{2}}f(b)\\minus{}f(x)dx\\plus{}\\int_{\\frac{1}{2}}^{b}f(b)\\minus{}f(x)dx\\plus{}\\int_{b}^{1}f(x)\\minus{}f(b)dx\\ge$\r\n$ \\int_{0}^{\\frac{1}{2}}f(\\frac{1}{2})\\minus{}f(x)dx\\plus{}\\int_{\\frac{1}{2}}^{b}f(x)\\minus{}f(\\frac{1}{2})dx\\plus{}\\int_{b}^{1}f(x)\\minus{}f(\\frac{1}{2})dx$\r\nThis is equivalent with $ (b\\minus{}\\frac{1}{2})f(b)\\ge\\int_{\\frac{1}{2}}^{b}f(x)dx$, but $ \\int_{\\frac{1}{2}}^{b}f(x)dx \\equal{} (b\\minus{}\\frac{1}{2})f(c), c\\in(\\frac{1}{2},b)$, so we get\r\n$ f(b)\\ge f(c)$ which is obviously true($ b\\ge c$).", "Solution_2": "$ 2.$ We apply Rolle to the following function:\r\n$ G(t) \\equal{} (a\\plus{}b\\minus{}2t)(\\int_{a}^{t}f(x)dx\\plus{}\\int_{b}^{t}f(x)dx)$\r\n\r\nThe first problem can easily be generalized:\r\n$ f: [a,b]\\rightarrow\\mathbb{R}$,$ a < b$ continuous and monotonic function,$ c\\in\\mathbb{R}$ then:\r\n$ \\int_{a}^{b}\\left|f(x)\\minus{}c\\right|\\ dx\\ge\\int_{a}^{b}\\left|f(x)\\minus{}f\\left(\\frac{a\\plus{}b}{2}\\right)\\right|\\ dx$\r\n\r\nThis can easily be reduced to the original problem with the substitution $ t \\equal{}\\frac{x\\minus{}a}{b\\minus{}a}$. \r\nIt's a well-known problem, but thank you for reminding me of it. Va multumesc! :)" } { "Tag": [ "algebra", "polynomial", "algebra proposed" ], "Problem": "Show in the neatest possible way that \\[\\frac{c(a - b)^3 + a(b - c)^3 + b(c - a)^3}{c^2(b - a) + a^2(c - b) + b^2(a - c)} = a + b + c.\\]", "Solution_1": "Let $f(a,b,c)=c(a-b)^3+a(b-c)^3+b(c-a)^3$, $g(a,b,c)=c^2(b-a)+a^2(c-b)+b^2(a-c)$,we have $f(b,b,c)=0,f(c,b,c)=0,f(-(b+c),b,c)=0$, so we obtain $f(a,b,c)=(a-b)(b-c)(c-a)(a+b+c)$ and then \r\n\r\n$g(b,b,c)=0,g(a,b,a)=0$, so we obtain $g(a,b,c)=(a-b)(b-c)(c-a)$,yielding $\\frac{f(a,b,c)}{g(a,b,c)}=a+b+c$.Q.E.D.", "Solution_2": "Another way to factor the numerator: consider the polynomial $P$ whose roots are $a-b,b-c,c-a$. Since they add up to zero, $P$ has the form\r\n\\[P(x)=x^3+mx+n,\\]\r\nwhere $n=-(a-b)(b-a)(c-a)$.\r\nBecause $cP(a-b)+aP(b-c)+bP(c-a)=0$, we obtain\r\n\\[\\sum {c(a-b)^3}+m\\sum {c(a-b)}-(a+b+c)(a-b)(b-c)(c-a)=0.\\]\r\nObvously, $\\sum {c(a-b)}=0$, hence the result." } { "Tag": [ "geometry", "circumcircle", "geometric transformation", "homothety", "search" ], "Problem": "Prove that $ E$ is midpoint of major arc $ \\stackrel{\\frown}{AB}$.", "Solution_1": "What is $ E$?\r\n\r\nAh, that $ E$ looked like an $ F$ to me", "Solution_2": "Click on figure above :idea:", "Solution_3": "[hide=\"Hint\"]\nDraw perpendicular bisector of AB, let it hit the circle at F, and prove that E and F coincide.[/hide]", "Solution_4": "[quote=\"Ubemaya\"][hide=\"Hint\"]\nDraw perpendicular bisector of AB, let it hit the circle at F, and prove that E and F coincide.[/hide][/quote]\r\n\r\nSomebody can show how to do that? No other approach?\r\n\r\nMaybe the figure helps.", "Solution_5": "If $ l$ is the line that is tangent to the circumcircle of triangle $ ABE$ at $ E$, then $ l//AC$ since a homothety center $ D$ maps one circle (and tangent line) to another. Then let $ A'$, $ B'$ be on $ l$ so $ A'XB'$ is a line, and $ AB$, $ A'B'$ are similarly oriented.\r\n\r\nThen $ \\text{arc}\\, AE\\equal{}\\angle ABE\\equal{}\\angle BEB'\\equal{}\\text{arc}\\, BE$.", "Solution_6": "Thank you for your answer. \r\nI would like to know if there are another solution. The book where i got this problem don't say anything about homothetic center.", "Solution_7": "See http://www.mathlinks.ro/viewtopic.php?search_id=1711576097&t=155800", "Solution_8": "[hide=\"Alternate Solution\"]\nCall the center of the circle containing chord $ AB$ to be $ O$ and that containing point $ C$ to be $ Q$. So, we call $ \\angle AOB \\minus{} 2\\theta$. Thus, we have that $ \\angle OAB \\equal{} \\angle OBA \\equal{} 90 \\minus{} \\theta\\rightarrow \\angle OBC \\equal{} 90 \\plus{} \\theta$. Now, we call $ \\angle QCD \\equal{} \\phi \\equal{} \\angle QDC$, so $ \\angle BCD \\equal{} 90 \\minus{} \\phi$ and $ \\angle CDO \\equal{} 180 \\minus{} \\phi$. Thus, $ \\angle BOD \\equal{} 360 \\minus{} (\\angle OBC \\plus{} \\angle BCD \\plus{} \\angle CDO) \\equal{} 2\\phi \\minus{} \\theta$. Also, $ \\angle CDQ \\equal{} \\angle ODE \\equal{} \\phi \\equal{} \\angle OED$, so $ \\angle DOE \\equal{} 180 \\minus{} 2\\phi$. Thus, $ \\angle AOE \\equal{} 360 \\minus{} (\\angle AOB \\plus{} \\angle BOD \\plus{} \\angle OED) \\equal{} 180 \\minus{} \\theta$. Also, $ \\angle BOE \\equal{} \\angle BOD \\plus{} \\angle DOE \\equal{} 180 \\minus{} \\theta \\equal{} \\angle AOE$, so arcs $ AE$ and $ BE$ have the same angle, and thus $ E$ is the midpoint of major arc $ AB$. [/hide]\r\nEDIT: Oh, sorry, that should be $ \\angle QCD \\equal{} \\phi \\equal{} \\angle QDC$, edited.", "Solution_9": "[quote=\"The QuattoMaster 6000\"]Now, we call $ \\angle QCD \\equal{} \\phi \\equal{} \\angle ODC$ [/quote]\r\n\r\nMaybe $ \\angle QCD \\equal{} \\phi \\Longrightarrow \\angle ODC\\equal{}90^{\\circ}\\plus{}\\phi$?" } { "Tag": [ "calculus", "calculus computations" ], "Problem": "are the phase curves of the following differential equation closed?\r\n\r\n$ x''\\plus{}100x\\equal{}0$", "Solution_1": "Yes. The general solution is $ x(t) \\equal{} \\dots$, and the parametric equation $ t\\mapsto (x(t),x'(t))$ describes an ellipse.", "Solution_2": "There are more challenging questions of whether phase curves are closed. For instance, this well-known classic:\r\n\r\n$ x'\\equal{}ax\\minus{}bxy$\r\n$ y'\\equal{}\\minus{}cy\\plus{}dxy$\r\n\r\nwith $ a,b,c,d>0.$ And the only phase curves we care about are in the first quadrant, $ x>0,y>0.$", "Solution_3": "Topic-changing replies to this have been moved into the Round Table forum." } { "Tag": [ "linear algebra", "matrix" ], "Problem": "1)Let T:V-->V be any linear map and V* be its dual.Let W be a k-dimensional subspace of V.(V is finite dimensional).Then prove that there exits a Linearly independent subset {f1,f2,...f(n-k)} subset of V* such that W=inetsection of Kernel(fi), where i=1,2,...,n-k.\r\n\r\n\r\n2)Let T* denote the adjoint of T. If T is surjective then show that T* is injective", "Solution_1": "1) $ dim(V)\\equal{}n$. Let $ \\{w_1,\\cdots,w_k,v_1,\\cdots,v_{n\\minus{}k}\\}$ be a basis of $ V$ s.t. $ \\{w_1,\\cdots,w_k\\}$ is a basis of $ W$. We define $ f_i$ as follows: if $ x\\equal{}\\sum\\lambda_rw_r\\plus{}\\sum\\mu_jv_j$ then $ f_i(x)\\equal{}\\mu_iv_i$.\r\n2) \"Let $ V^*$ be its dual\" is a nonsense. You must define a basis of $ V$: $ \\{e_1,\\cdots,e_n\\}$.\r\nIf $ T$ is onto then $ T$ is one to one. Then the correct question is: if $ T$ is invertible then $ T^*$ is invertible.\r\nAssume $ T^*(f)\\equal{}0$. Then for all $ i$, $ T^*(f)e_i\\equal{}0\\equal{}f(T(e_i))$. Thus $ f(Im(T))\\equal{}0$ or $ Im(T)\\subset{k}er(f)$. Conclusion: $ f\\equal{}0$ and $ T^*$ is invertible.", "Solution_2": "I wrote to quickly. There are 2 corrections:\r\n1) Replace \"let $ V^*$ be its dual \" is a nonsense with \"Let T* denote the adjoint of T\" is a nonsense. Indeed there do not exist any canonical isomorphisms between $ V$ and $ V^*$.\r\n2) Replace $ f_i(x)\\equal{}\\mu_iv_i$ with $ f_i(x)\\equal{}\\mu_i$.", "Solution_3": "Third correction !\r\nThe adjoint, or the transpose, of $ T$ is well-defined. -I thought about the transpose of a matrix (Too much champagne during the festivals !)- \r\nIf $ T: V\\rightarrow{W}$ then $ T^*: W^*\\rightarrow{V^*}$ is defined by $ T^*(f)\\equal{}foT$.\r\nAssume that $ T$ is onto and $ T^*(f)\\equal{}0$ that is $ foT\\equal{}0$. \r\nThen $ f(W)\\equal{}0$ and $ f\\equal{}0$ and $ T^*$ is one to one." } { "Tag": [ "logarithms", "pigeonhole principle", "continued fraction", "absolute value" ], "Problem": "Prove or disprove that any positive real number $ n$ may be represented to an arbitrary degree of precision (so, any degree of precision as high as you desire - pi could be represented to 10 digits, 100, 1000...) as $ 2^j*3^k$, where j and k are integers.", "Solution_1": "Maybe I'm understanding this wrong, but $ 5\\neq2^k3^j$ because the only prime factor is 5.", "Solution_2": "$ k,j$ are not necessarily positive integers, and by \"degree\" sponge means \"degree of precision\", i.e. that for any $ \\epsilon>0$, you can choose $ j,k$ so that $ |2^j3^k\\minus{}n|<\\epsilon$.", "Solution_3": "Thanks for the clarification, Temperal; I seem to be remarkably bad at writing down mathematical thoughts translated to English. (fixed the first post)", "Solution_4": "You can rewrite the relation as \r\n\r\n$ a\\log_2 2\\plus{}b\\log_2 3\\equal{}\\log\\plus{}2 n$ where $ a,b\\in\\mathbb{Z}$.\r\n\r\nConsider all $ m^2\\minus{}1$ pairs $ (a,b)$ with $ a,b\\in \\{1,2,...,m\\}$\r\n\r\nNow $ 1 := dist(x, \\mathbb{Z}) $ for $ x \\in \\mathbb{R} $. Then define\r\n$ f(x):= max(1 - 4, 0) $. Then $ f $ is of course a continuous function\r\n$ \\mathbb{R} \\rightarrow \\mathbb{R} $, $ f(x) = 1 $ for $ x \\in \\mathbb{Z} $, and\r\n$ f(x) > 0 \\Leftrightarrow dist(x, \\mathbb{Z}) < \\frac{1}{4} $. Now define\r\n$ f_{n}(x) := nf(2^{n}x) $ for $ n \\in \\mathbb{N} $.\r\nThen of course for every $ x \\in \\Gamma $, we have for $ n \\in \\mathbb{N} $, that\r\n$ 2^{n}x \\in \\mathbb{Z} $, so for $ n $ large enough, we have $ f_{n}(x) = n $, so\r\n$ lim_{n\\rightarrow \\infty} f_{n}(x) = \\infty $. Now suppose that we have a number\r\n$ x \\in \\mathbb{R} $, and $ lim_{n\\rightarrow \\infty} f_{n}(x) = \\infty $.\r\nThen in particular there exists $ n_{0} $, such that for every $ n \\geqslant n_{0} $,\r\nwe have $ f_{n}(x) > 0 $, so $ f(2^{n}x) > 0 $, so for $ n \\geqslant n_{0} $ we have that\r\n$ dist(2^{n}x, \\mathbb{Z}) < \\frac{1}{4} $. Now it is easy to see that if we have\r\na real number $ y \\in \\mathbb{R} $, such that $ dist(y, \\mathbb{Z}) < \\frac{1}{4} $,\r\nthen $ dist(2y, \\mathbb{Z}) = 2dist(y, \\mathbb{Z}) $. Therefore for $ n \\geqslant n_{0} $\r\nwe have that $ dist(2^{n+1}x, \\mathbb{Z}) = 2 dist(2^{n}x, \\mathbb{Z}) $, but on the\r\nother hand, all $ dist(2^{n}x, \\mathbb{Z}) $ are bounded by $ \\frac{1}{2} $, so the only\r\nconclusion is that $ dist(2^{n_{0}}x, \\mathbb{Z}) = 0 $, so $ 2^{n_{0}}x \\in \\mathbb{Z} $,\r\nso $ x \\in \\Gamma $. Therefore we see that $ lim_{n\\rightarrow \\infty} f_{n}(x) = \\infty $\r\nif and only if $ x \\in \\Gamma $.", "Solution_3": ":( :( :( :( \r\nAll my work is vain.\r\n\r\nThe complete answer for this question is in http://www.mathlinks.ro/Forum/viewtopic.php?t=22177", "Solution_4": "Ops... I solved [i]slighly[/i] another problem in that topic.", "Solution_5": "I feel Leva's proof is very illuminating to say the least....", "Solution_6": "What do you mean by that? :?", "Solution_7": "I meant the Myth's reply ...", "Solution_8": "Myth, what another problem did you solved?", "Solution_9": "I have erased everything I had written, because it was stupid :(.", "Solution_10": "[quote=\"eugene\"]Does there exist the sequence of continious real functions $f_{n}(x)$ s.t. $f_{n}(x)\\to+\\infty$ ($n\\to\\infty$) at rational points $x$ only.[/quote]\r\n\r\nFor an enumeration of rationals $q_i$, let $f_n(x)$ be the n'th partial sum of $a_1|x - q_1| + a_2|(x-q_1)(x-q_2)| + a_3|(x-q_1)(x-q_2)(x-q_3)| + ...$. \r\nIt is possible to choose $a_i$ and the enumeration to guarantee that the sum is infinite at irrational points.", "Solution_11": "[quote=\"Leva1980\"]Myth, what another problem did you solved?[/quote]\r\nFollow the link I wrote above.\r\nI proved that if $f_n(x)$ is unbounded for all rationals, then there is irrational $s$ s.t. $f_n(s)$ is unbounded too. :?", "Solution_12": "[quote=\"fleeting_guest\"][quote=\"eugene\"]Does there exist the sequence of continious real functions $f_{n}(x)$ s.t. $f_{n}(x)\\to+\\infty$ ($n\\to\\infty$) at rational points $x$ only.[/quote]\n\nFor an enumeration of rationals $q_i$, let $f_n(x)$ be the n'th partial sum of $a_1|x - q_1| + a_2|(x-q_1)(x-q_2)| + a_3|(x-q_1)(x-q_2)(x-q_3)| + ...$. \nIt is possible to choose $a_i$ and the enumeration to guarantee that the sum is infinite at irrational points.[/quote]\r\nWhy did you said that?", "Solution_13": "[quote=\"eugene\"]Does there exist the sequence of continious real functions $f_{n}(x)$ s.t. $f_{n}(x)\\to+\\infty$ ($n\\to\\infty$) at rational points $x$ only.[/quote]\r\n\r\nI think the following works as well: \r\n\r\nIf $(q_n)_{n\\ge 1}$ is an enumeration of the rationals, we take $f_n$ to be $n$ in $q_1,q_2,\\ldots,q_n$, and $0$ outside $A_n=\\bigcup_{k=1}^n(q_k-\\varepsilon_n,q_k+\\varepsilon_n)\\ (*)$. If $\\varepsilon_n>0$ is small enough s.t. the intervals in $(*)$ are disjoint for every $n$, and $(\\varepsilon_n)_n$ decreases (to $0$) fast enough s.t. $A_n\\cap A_{n+1}=A_{n+1}\\setminus(q_{n+1}-\\varepsilon_{n+1},q_{n+1}+\\varepsilon_{n+1}),\\ \\forall n$, then every irrational will lie outside infinitely many $A_n$'s: \r\n\r\nLet $t\\in\\mathbb R$ and $n_0$ is large enough s.t. $t\\in A_n,\\ \\forall n\\ge n_0$. Since $t\\in A_n$, it lies in some $(q_k-\\varepsilon_n,q_k+\\varepsilon_n)$. $t\\in A_{n+1}$ as well, so, from the conditions imposed on the $A_n$'s, we must have $t\\in(q_k-\\varepsilon_{n+1},q_k+\\varepsilon_{n+1})$ (the same $k$ as before). We repeat this for $n\\to\\infty$, and we get $t=q_k$ because $\\varepsilon_n\\to 0$, so $t$ is rational.\r\nIs there anything wrong with this? :?" } { "Tag": [ "logarithms", "number theory unsolved", "number theory" ], "Problem": "Which number is bigger: $3{}^{3{}^{3^{...}^3}}$ (2005 3's) or $2^{2^{{{{...}{}^2}{}^6}}}$ (2004 2's and one 6).\r\n\r\nThanks for help", "Solution_1": "Nice problem!\r\nSuppose $3^m<2^{\\frac{5}{6}n}$, and $n\\geq 6$. Then $3^{3^m}<2^{\\frac{5}{6}2^n}$ :!:.\r\nIndeed, \\[3^{3^m}<2^{\\frac{5}{6}2^n}\\iff 3^m<\\frac{5}{6}\\cdot {2^n}\\log_32,\\] but $2^{\\frac{5}{6}n}<\\frac{5}{6}\\cdot{2^n}\\log_32$ for $n\\geq 6$ !!!\r\nWe initially have $3^3=27<32=2^{\\frac{5}{6}\\cdot 6}$. Using :!: we obtain that $3^{3^{...^3}}<2^{\\frac{5}{6}2^{...2^6}}$. Hence the result." } { "Tag": [], "Problem": "$\\frac{x+x^{2}+x^{3}....+x^{50}}{x^{-1}+x^{-2}+...+x^{-50}}$", "Solution_1": "Mods, can this be put to Intermediate or High School Basics?", "Solution_2": "[quote=\"ashrafmod\"]$\\frac{x+x^{2}+x^{3}....+x^{50}}{x^{-1}+x^{-2}+...+x^{-50}}$[/quote]\r\n$x^{51}$ :P :P" } { "Tag": [ "greatest common divisor" ], "Problem": "For those of you who don't know, Bezout's Lemma states that for any two integers $ a$ and $ b$, there exist integers $ m$ and $ n$ such that $ am \\plus{} bn \\equal{} (a, b)$, where $ (a, b)$ denotes the greatest common factor of $ a$ and $ b$. \r\n\r\nHow can we show that there exist infinitely many ordered pairs $ (m, n)$ satisfying $ am \\plus{} bn \\equal{} (a, b)$?", "Solution_1": "$ am \\plus{} bn \\equal{} a(m \\plus{} bk) \\plus{} b(n \\minus{} ak)$.", "Solution_2": "Thanks. Part two: \r\n\r\nLet's say $ (m, n)$ is one solution to the equation. Then do all other solutions $ m'$ and $ n'$ necessarily take the form $ m \\plus{} bk$ and $ n \\minus{} ak$ for some integers $ k$? If so, prove it. If not, give a counterxample.", "Solution_3": "Thats incorrect, but it is true if you write $ m\\plus{}\\frac{b}{(a,b)}\\cdot k$, etc\r\n\r\nWe prove this by saying that $ (m,n)$ is a solution so $ am\\plus{}bn\\equal{}(a,b)$.\r\n\r\nThen suppose that $ (x\\plus{}m,y\\plus{}n)$ is another solution...\r\n\r\n$ a(x\\plus{}m)\\plus{}b(y\\plus{}n)\\equal{}(a,b)$\r\n$ ax\\plus{}by\\plus{}(am\\plus{}bn\\minus{}(a,b))\\equal{}0$\r\n$ ax\\equal{}\\minus{}by$. \r\n\r\nThen it is easy to see that $ \\frac{b}{(a,b)}$ must divide $ x$, and such..." } { "Tag": [ "geometry unsolved", "geometry" ], "Problem": "[img]http://www.mathlinks.ro/Forum/viewtopic.php?mode=attach&id=14780[/img]\r\n\r\n :(", "Solution_1": "Let's solved with your figure.\r\nLet M,N be on AC such that AM=MN=NC.\r\nS(AHDB)=S(ABM)=S(ABC)/3, so S(BDH)=S(BMH), thus BH is parallel to DM. H have just been constructed.\r\nThe line through N and parallel to BH meets BC at I. S(IHN)=S(IBN), so S(IHC)=S(BNC)=S(ABC)/3=S(CEKH), which follows that S(IHE)=S(KHE), hence IK is parallel to EH.", "Solution_2": "[img]http://www.mathlinks.ro/Forum/viewtopic.php?mode=attach&id=14791[/img]\r\n\r\n\r\n[b]\"The line through N and parallel to BH meets BC at I\" not correct\n\n\"The line through N and parallel to AB meets BC at I\" correct[/b]\r\n\r\n\r\n :thumbup:" } { "Tag": [], "Problem": "I have trouble understanding this... (excerpted from The Art and Craft of Problem Solving by Paul Zeitz, pg. 224 - 225)\r\n\r\n[quote](d)If $ g|a$ and $ g|b$, then $ g|ax \\plus{} by$, where x and y can be any integers. WE call a quantity like ax + by a linear combination of a and b. [...] 7.1.5 An important consequence of the division algorithm, plus 7.1.3 (d), is that [i]The greatest common divisor of a and b is the smallest positive linear combination of a and b.[/i] [...] Let's prove 7.1.5; it is rather dry but a great showcase for the use of the extreme principle pls arguement by contradiction. Define u to be the smallest positive linear combination of a and b and let g := (a,b). We know from 7.1.3 (d) that g divides any linear combination of a and b, and so certainly g divides u. This means that $ g \\le u$. We would like to show that in fact, $ g\\equal{}u$. We will do this by showing that /u is a common divisor of a and b. Since u is greater than or equal to g, and g is the greatest common divisor of a and b, this would force g and u to be equal. \nSuppose, on the contrary, that u is not a common divisor of a and b. Without loss of generality, suppose that u does not divide a. Then yb the division algorithm, there exists a quotient $ k /ge 1$ and the positive remainder $ r < u$ such that \\[ a \\equal{} ku \\plus{} r\\]\nBut then $ r \\equal{} a \\minus{} ku$ is positive and less than u. [b]This is a contradiction, because r is also a linear combination of a and b, yet u was defined to be the smallest positive linear combination![/b] Consequently, u divides a, and likewise u divides b. So u is a common divisor; thus $ u \\equal{} g$. [/quote]\r\n\r\nCan anyone please explain the bolded part - namely, \"This is a contradiction, because r is also a linear combination of a and b, yet u was defined to be the smallest positive linear combination!\" \r\n\r\nI mean, can anyone explain in detail how r is a linear combination of a and b???????\r\n\r\nI thank you for your time and patience.", "Solution_1": "just for purposes of clarification: I understand everything up to the bolded point. Thanks.", "Solution_2": "$ u$ is a linear combination of $ a,b$; hence so is $ a \\minus{} ku$.", "Solution_3": "sorry, but apparently i can't think striaght today for some reason :( \r\n\r\nCan you explain in detail? (some algebra would be nice)", "Solution_4": "$ u$ is a linear combination of $ a,b$; hence we may write $ u \\equal{} ax \\plus{} by$, where $ x,y$ are integers. Then $ r \\equal{} a \\minus{} ku \\equal{} a \\minus{} k(ax \\plus{} by) \\equal{} (1 \\minus{} kx)a \\plus{} (\\minus{}ky)b$, where $ 1\\minus{}kx, \\minus{}ky$ are obviously integers." } { "Tag": [ "geometry", "3D geometry", "LaTeX", "Olimpiada de matematicas" ], "Problem": "[color=green]disculpen la molestia, si alguien me pudiera proporcionar los enunciados de la olimpiada de mayo 2005 y 2006 se los agradeceria mucho, he pasado miles de horas buscando en internet y he encontrado hasta la 2004, y los estoy necesitando ahorita. gracias de antemano x su ayuda![/color] :)", "Solution_1": "[color=blue]Ya me pidieron tambi\u00e9n algunos chicos, y en ning\u00fan lado se consiguen, as\u00ed como los enunciados de la XVI Olimpiada Matem\u00e1tica del Cono Sur.\n\u00bfQui\u00e9n podr\u00eda explicar el motivo?[/color]", "Solution_2": "aqui tem tanto da cone sul, como da olimpiada de maio, mas est\u00e1 em portugu\u00eas! :blush: \r\n\r\nhttp://www.obm.org.br/frameset-provas.htm", "Solution_3": "Aqui tengo el examen del 2006", "Solution_4": "muchas gracias a todos x su ayuda :lol: se los agradezco mucho, la verdad q estos examenes no son muy faciles de encontrar :)", "Solution_5": "Buenas tardes,\r\n\r\naki tengo las fuentes de latex q generaron este archivo ..\r\n\r\nclaro si alguien las desea para editar otros exams de este formato ..\r\n\r\nsaludos,", "Solution_6": "[color=blue]Aviso que mis \"disc\u00edpulos\" realizar\u00e1n el examen de la mencionada olimpiada este s\u00e1bado 11 de mayo.\nEs bien sabido que est\u00e1 prohibido divulgar hasta el mes siguiente.[/color]", "Solution_7": "Buenas noches,\r\n\r\neso es cierto!!\r\n\r\nNo publicar los exams de Mayo no al menos dentro de 1 mes\r\n\r\nGracias!!", "Solution_8": "Para ser m\u00e1s exactos no se puede divulgar las pruebas hasta el 25 de Mayo.\r\n\r\n$Tipe$", "Solution_9": "[quote=\"tipe\"]Para ser m\u00e1s exactos no se puede divulgar las pruebas hasta el 25 de Mayo.\n\n$Tipe$[/quote]\r\n\r\nPerfecto Jorge,\r\n\r\nGracias por la aclaracion...\r\n\r\n\r\nSaludos,", "Solution_10": "[quote=\"tipe\"]Para ser m\u00e1s exactos no se puede divulgar las pruebas hasta el 25 de Mayo.\n\n$Tipe$[/quote]\r\n\r\nYa es 26 de mayo :P , \u00bfalguien tiene los enunciados? Gracias y saludos!", "Solution_11": "yo los tengo :) solo q para tener un poco mas de orden mejor los voy a poner en topicos separados! en un momento los subo" } { "Tag": [ "Pythagorean Theorem", "geometry" ], "Problem": "I was doing team round problems, I could do all but 7 and 8, could some 1 help?\r\n\r\nhttp://www.mathcounts.org/webarticles/articlefiles/510-07_Sc_Team.pdf\r\n\r\n\r\nEdit: I found out 7", "Solution_1": "Next time, please post the problems\r\n\r\n[hide=\"hint for number 8\"]\nUse Pythagorean Theorem (or memorize some triples) to find AG, and then AG=AC because they are both radii and then figure it out from there\n[/hide]", "Solution_2": "Ok so um heres some hints.\r\nFor number 7, use the fact that the radius of the circle is the same as a line from the center of the hexagon to one of it's vertexes (Plural form.... :| ), which means that it is also the length of one of the sides of the hexagon.\r\nFor number eight, just use the Pythagorean Theorem to find out the radius of the circle, and subtract 7 from it." } { "Tag": [ "college", "geometry", "Stanford", "Berkeley Math Circle" ], "Problem": "Hi, \r\nI'm at UC Berkeley right now for the summer (until august 14) for the summer college program.\r\nI'm a high school student, and I'm hoping that I can get involved in some math clubs or activities nearby.\r\nIf there's any program or meeting held during the summer near the UC berkeley campus, and if it's not too late, can anyone provide me with information regarding nearby math circles etc.?\r\n(My goal for this is to prepare for upcoming olympiads next year.)\r\n\r\nThanks\r\n\r\nDavid Lee", "Solution_1": "I'm also going to be around UC Berkeley for the summer. There's Berkeley Math Circle on campus, but that won't start until September. \r\n\r\nThe only math circle in the area that has summer sessions is the Stanford Math Circle http://www.stanfordmathcircle.org/Main_Schedule.html It's from 7-9 on Tuesday nights - It's fairly far from Berkeley - if you want to get there by public transportation you'll have to take BART then transfer to Caltrain. Then its a walk to campus.", "Solution_2": "Sounds fun, Looks fun, Seems fun!\r\nBut, yea.. I don't think I'll be able to make it.. \r\nStill, I'll try my best to get there if I can, if I have time and no writing homework :D" } { "Tag": [ "geometry solved", "geometry" ], "Problem": "I want some easy inversion problem,if anybody have plz send. :D", "Solution_1": "Have you proved that The inverse of a line that doesn't pass by the inversion center is a circunferencia thought the center?", "Solution_2": "[quote=\"hucht\"]Have you proved that The inverse of a line that doesn't pass by the inversion center is a circunferencia thought the center?[/quote]\r\nof course. :D", "Solution_3": "A problem of mine is a example of using inversion to solve problems easily.\r\n\r\nSee\r\n\r\nhttp://www.mathlinks.ro/Forum/viewtopic.php?t=55206", "Solution_4": "http://www.mathlinks.ro/Forum/viewtopic.php?t=14958\r\nhttp://www.mathlinks.ro/Forum/viewtopic.php?t=56658\r\n\r\n Darij" } { "Tag": [ "vector", "function", "geometry", "parallelogram", "search", "linear algebra" ], "Problem": "A norm on a vector space U is a function from U to nonnegative real such that ||u||>=0, ||au|| = |a|||u|| for all a in F and all u in U, and ||u+v|| is less than or equals to ||u||+||v|| for all u,v in U. Prove that a norm satisfying the parallelogram equality comes from an inner product(that there exists an inner product < , > that ||u||=$ < u,u > ^\\frac {1}{2}$ for all u in U)", "Solution_1": "See [url=http://www.artofproblemsolving.com/Forum/viewtopic.php?search_id=1961927883&t=224293]here[/url].", "Solution_2": "[quote=\"t0rajir0u\"]See [url=http://www.artofproblemsolving.com/Forum/viewtopic.php?search_id=1961927883&t=224293]here[/url].[/quote]\r\n\r\nI got confused, the problem want to show that the function is the norm of an inner product space, but the first step of the proof just say $ {\\parallel{}x\\plus{}y\\parallel{}}^2\\equal{}$, which I think only holds if the function is the norm of an inner product space. Is this a logical error?", "Solution_3": "You're reading the .pdf file downloaded from that, right? What you're quoting is from a \"discovery\" phase of the project: suppose there were a valid inner product. How would it be related to the norms?\r\n\r\nThe proof then starts about halfway down the page, with the statement (at least for the real case) that \"Our goal is to show that $ \\langle x,y\\rangle\\equal{}\\frac{\\|x\\plus{}y\\|^2\\minus{}\\|x\\minus{}y\\|^2}{4}$ does indeed define an inner product.\"", "Solution_4": "Oh,by bad\r\nI'm so careless!", "Solution_5": "I'm sorry, but I'm not able to prove the continuity of for a.(last step for proving real case) Can anyone help?", "Solution_6": "At this point we've already proved the linearity for all real and homogeneity for all rational. So we can use these results. \r\n\r\nThe last step is Use Parallelogram identity to prove that , for any x,z, the function a - is a continuous function of a. (a is real)\r\n\r\nHelp me please :maybe:" } { "Tag": [ "algebra unsolved", "algebra" ], "Problem": "Let n be a natural number grater or equal to 2. Determine the numbers a_k naturals and different from 0, k={1,2,...,n}, distinct two by two, knowing that\r\n$ a_1\\plus{}a_2\\plus{}...\\plus{}a_n\\equal{}\\dfrac{n^2\\plus{}n\\plus{}2}{2}$", "Solution_1": "[quote=\"St. Elena\"]Let n be a natural number grater or equal to 2. Determine the numbers a_k naturals and different from 0, k={1,2,...,n}, distinct two by two, knowing that\n$ a_1 \\plus{} a_2 \\plus{} ... \\plus{} a_n \\equal{} \\dfrac{n^2 \\plus{} n \\plus{} 2}{2}$[/quote]\r\n\r\n$ a_i\\equal{}i$ $ \\forall i\\in[1,n\\minus{}1]$\r\n$ a_n\\equal{}n\\plus{}1$" } { "Tag": [ "Princeton", "college" ], "Problem": "I bought Barron's SAT II Math IIC book and like people said, it was much harder than some of the practice tests I took. But how much harder is it? Will questions like: what is the 3rd term of (8x-y)^(1/3) be in the test?\r\n\r\nThanks for any answer.", "Solution_1": "Ok let's put it this way. Under practice-test (meaning extremely distracted) conditions, not bothering to double-check, etc, I got a best score of 45 out of 50 on the Barron's practice tests (I took all 6). \r\n\r\nI went into the test thinking I would get a 750 at best.\r\n\r\nI got an 800 and thought it was extremely easy. \r\n\r\nBarron's is notorious for making their prep books harder than the real test to \"over-prepare\" you because they assume that there's no way that you can know all the material in a review book anyways.", "Solution_2": "I find the practice tests in the SparkNotes Math 1C and 2C books to be pretty close in level to the real thing.", "Solution_3": "Out of curiosity, how do you have a third term for $\\sqrt[3]{8x-y}$?", "Solution_4": "haha, i think he probably meant to the third power, not third root.", "Solution_5": "Or it could have been referring to the infinite [url=http://mathworld.wolfram.com/BinomialSeries.html]binomial series[/url], which is indeed too advanced for SAT II math.", "Solution_6": "It has some errors, but they aren't too much of a burden. However, it doesn't parallel the actual test at all - nowhere on the test you'll have to provide multiple answers for a trigonometric equation, for example. It's actually nice for improving one's knowledge of Pre-Calc concepts.", "Solution_7": "Hmmm I always thought those books were routine, but I guess I did learn Algebra from Barron's \"Algebra the Easy Way.\" I have to say from all that I've seen/heard, Barron has some good books.", "Solution_8": "[quote=\"Ravi B\"]Or it could have been referring to the infinite [url=http://mathworld.wolfram.com/BinomialSeries.html]binomial series[/url], which is indeed too advanced for SAT II math.[/quote]Wow I never knew that.", "Solution_9": "[quote=\"eryaman\"][quote=\"Ravi B\"]Or it could have been referring to the infinite [url=http://mathworld.wolfram.com/BinomialSeries.html]binomial series[/url], which is indeed too advanced for SAT II math.[/quote]Wow I never knew that.[/quote]\r\n\r\nDiscovered by Newton. :lol:", "Solution_10": "Don't stress too much over the IIC. It's not that hard.", "Solution_11": "im taking it this saturday, june 4th. i've been using the princeton review though.", "Solution_12": "best of luck to everybody including my brother (he's taking bio) :)", "Solution_13": "[quote=\"bubala\"]If you don't make careless errors--read the question--and go at the right pace, then you will get an 800 easily.[/quote]\r\n\r\nI'm sure that there're a lot of people who do those things, but don't get an 800. What's easy to you may not be easy to everyone else. You have to respect people's individuality.", "Solution_14": "well, I think that most, if not all people, on THIS FORUM in particular, are capable of getting an 800 or close to that. \r\n\r\nI wouldn't say something like that to all my friends in school, for example.\r\n\r\nIt depends on the environment.\r\n\r\nHowever, I apologize for such comments because the above statement may not indeed be true.", "Solution_15": "I took the IIC on May 7th and scored an 800....it was extremely easy, and I'm not surprised that 800 is only 90th percentile.\r\n\r\nIf you are in even the second highest level of 11th grade math offered at your school, this test should be a breeze, no book required", "Solution_16": "What is the lowest raw score that guarantees a 800?", "Solution_17": "[quote=\"ffdbzathf\"]im taking it this saturday, june 4th. i've been using the princeton review though.[/quote]\r\n\r\ni used the princeton review book too. i just toke it 1hr and 15mins ago :D", "Solution_18": "[quote=\"anhnguyen\"]What is the lowest raw score that guarantees a 800?[/quote]\r\n\r\nyou can miss like 5 or 6 and still get a 800 on IIC", "Solution_19": "Each administration of the test is scaled differently, so the number of questions that equals an 800 will vary from administration to administration.", "Solution_20": "So it's been concluded that a question like the oneproposed in the topic will not be present in the SAT II Math IIC?\r\n\r\nIf not, how would you work it out? I saw the link, but something doesn't sink in unless I see it being done.", "Solution_21": "No it will not be in the SAT II. I asked the question and I just took the test this morning. It was much easier than Barron's. Don't worry about those types of questions. They are too advance to be in the test.", "Solution_22": "Awesome" } { "Tag": [ "algebra", "polynomial", "AMC" ], "Problem": "AMC 1976 #30.\r\n\r\nHow many distinct ordered triples (x,y,z) satisfy the equations\r\nx+2y+4z=12\r\nxy + 4yz + 2xz = 22\r\nxyz = 6\r\n\r\nShow your work.", "Solution_1": "[hide] x+2y+4z=12, 2xy+8yz+4xz=44, and x(2y)(4z)=48, so polynomial with roots x, 2y, and 4z is x^3 -12x^2 +44x +48= 0[/hide]\n\n\n\nmore later gotta run.", "Solution_2": "Since x+2y+4z=12, xy+4yz=12y-2y\u22272\r\nSince xy+4yz+2xz=22, xy+4yz=22-2xz\r\nSo 12y-2y\u22272=22-2xz\r\n\t6y-y\u22272=11-xz\r\n\ty\u22272-6y=9=xz-2\r\n\t(y-3) \u22272=xz-2\tsince xyz=6\r\nso\t(y-3) \u22272=6/y-2\r\n\ty\u22273-6y\u22272+9y=6-2y\r\n\ty\u22273-6y\u22272+11y-6=0\r\n\ty\u22273-y-(6y\u22272-12y+6)=0\r\n\t(y-1)(y-2)(y-3)=0\r\nso y can be 1, 2, or 3\r\nsubstitute the values of y into the original equation and get the answer" } { "Tag": [], "Problem": "[url=http://www.imo-official.org/year_reg_team.aspx?code=ROU]Participantii[/url]", "Solution_1": "Subiectul de anul trecut:", "Solution_2": "Rezultatele finale de la baraje:", "Solution_3": "[url=http://www.imo-official.org/year_info.aspx?year=2009]Rezultatele oficiale[/url]" } { "Tag": [ "inequalities", "symmetry", "geometric series" ], "Problem": "[color=blue][b] Problem.[/b]\n\n Let $a, b, c\\in (0,1)$. Prove that the following inequality holds true:\n\n \\[ \\frac{a}{1-a}+\\frac{b}{1-b}+\\frac{c}{1-c}\\geq\\frac{3\\sqrt [3] {abc}}{1-\\sqrt [3] {abc}}. \\]\n\n [/color]", "Solution_1": "$\\\\\\frac{a}{1-a}+\\frac{b}{1-b}+\\frac{c}{1-c}\\\\ =\\sum_{k=1}^{\\infty}(a^k+b^k+c^k)\\geq 3\\sum_{k=1}^{\\infty}\\sqrt[3]{abc}^k$\r\nIt is suffiient enough, due to symmetry, to prove that $a^k+b^k+c^k\\geq 3\\sqrt[3]{abc}^k$, which is true by AM-GM.\r\nQ.E.D.\r\n\r\nMasoud Zargar", "Solution_2": "[quote]$\\frac{a}{1-a}+\\frac{b}{1-b}+\\frac{c}{1-c} =\\sum_{k=1}^{\\infty}(a^k+b^k+c^k)$[/quote]\r\nCan you please tell me why this is true?\r\n\r\nI think I also have a proof for the inequality:\r\n\r\nBy Chebyshev inequality, we obtain: \\[ \\displaystyle\\frac{a}{1-a}+\\frac{b}{1-b}+\\frac{c}{1-c} \\geq \\frac{a+b+c}{3}.\\left(\\frac{1}{1-a}+\\frac{1}{1-b}+\\frac{1}{1-c}\\right) \\]\r\n\r\nBy AM-HM and AM-GM, we know that \\[ \\frac{1}{1-a}+\\frac{1}{1-b}+\\frac{1}{1-c} \\ge \\frac{9}{3-(a+b+c)} \\ge \\frac{3}{1-\\sqrt[3]{abc}} \\]\r\nUsing this, we get \\[ \\displaystyle\\frac{a+b+c}{3}.\\left(\\frac{1}{1-a}+\\frac{1}{1-b}+\\frac{1}{1-c}\\right) \\ge \\frac{a+b+c}{1-\\sqrt[3]{abc}} \\ge \\frac{3\\sqrt[3]{abc}}{1-\\sqrt[3]{abc}} \\]\r\n\r\n$\\mathbb{QED}$", "Solution_3": "[quote=\"Jan\"][quote]$\\frac{a}{1-a}+\\frac{b}{1-b}+\\frac{c}{1-c} =\\sum_{k=1}^{\\infty}(a^k+b^k+c^k)$[/quote]\nCan you please tell me why this is true?\n[/quote]\r\nHe used $\\frac{1}{1-x}=1+x+x^2+x^3+\\cdots =\\sum_{k=0}^\\infty x^k$ for any real number $\\in (0,1)$. Hmm... strange solution. :)", "Solution_4": "Well, since $a,b,c\\in (0,1)$ and we have $\\frac{a}{1-a}$ and such terms, I thought of using geometric series; $\\sum_{k=1}^{\\infty} x^k=\\frac{x}{1-x}$ :)", "Solution_5": "[hide]Divide out the numerators: $\\frac{a}{1-a}=\\frac{1}{1-a}-1$ (similar for the others) and the right side is $\\frac{3}{1-\\sqrt[3]{abc}}-3$.\n\nSo this inequality is equivalent to $\\frac{1}{1-a}+\\frac{1}{1-b}+\\frac{1}{1-c}\\geq \\frac{3}{1-\\sqrt[3]{abc}}$.\n\nBy AM-HM, $\\frac{\\frac{1}{1-a}+\\frac{1}{1-b}+\\frac{1}{1-c}}{3}\\geq \\frac{3}{(1-a)+(1-b)+(1-c)}$.\n\nThis becomes $\\frac{1}{1-a}+\\frac{1}{1-b}+\\frac{1}{1-c}\\geq \\frac{9}{3-(a+b+c)}=\\frac{3}{1-\\frac{a+b+c}{3}}$.\n\nNow, from AM-GM, $\\frac{a+b+c}{3}\\geq \\sqrt[3]{abc}\\Rightarrow 1-\\frac{a+b+c}{3}\\leq 1-\\sqrt[3]{abc}\\Rightarrow \\frac{3}{1-\\frac{a+b+c}{3}}\\geq \\frac{3}{1-\\sqrt[3]{abc}}$\n\n(where the last inequality is true because $a,\\ b,\\ c\\in (0,1)\\Rightarrow \\frac{a+b+c}{3},\\ \\sqrt[3]{abc}<1$.\n\nSince $\\frac{1}{1-a}+\\frac{1}{1-b}+\\frac{1}{1-c}\\geq \\frac{3}{1-\\frac{a+b+c}{3}}\\geq \\frac{3}{1-\\sqrt[3]{abc}}$, the inequality is true.[/hide]" } { "Tag": [ "Gauss" ], "Problem": "Esmeralda has a bottle with 9 liters of a mixture that has 50% alcohol and 50% water. She wants to put water in the bottle so that only 30% of the mixture is alcohol. How many gallons of water will put it?\r\n\r\nPortuguese question (if necessary):\r\nEsmeralda tem uma garrafa com 9 litros de uma mistura que tem 50% de \u00e1lcool e 50% de \u00e1gua. Ela quer colocar \u00e1gua na garrafa de tal forma que apenas 30% da mistura seja de \u00e1lcool. Quantos litros de \u00e1gua ela ir\u00e1 colocar?\r\n\r\n\r\n\r\nHow to do ? :|", "Solution_1": "We see there are 9 liters of the mixture and half of it is water, so there's $ 4.5$ liters of water.\r\n\r\nLet $ x$ be the amount of water in liters to produce a 30% alcohol (70% water) solution.\r\n\r\n$ \\frac{4.5\\plus{}x}{9\\plus{}x}\\equal{}\\frac{7}{10}$\r\n\r\nCross multiplying givexs\r\n\r\n$ 45\\plus{}10x\\equal{}63\\plus{}7x$\r\n$ 3x\\equal{}18$\r\n$ x\\equal{}6$\r\n\r\nSo $ 6$ liters, or $ \\frac{6}{3.78541178} \\approx \\boxed{1.585}$ gallons need to be added.", "Solution_2": "Bom dia luiseduardo.\r\n\r\nSuppose she puts in $ x$ litres of water. Then she has $ 4.5$ litres of alcohol and $ 4.5 \\plus{} x$ litres of water. \r\n\r\n$ \\frac{4.5}{9\\plus{}x} \\equal{} \\frac{3}{10}$\r\n\r\n$ 27 \\plus{} 3x \\equal{} 45$\r\n\r\n$ x \\equal{} 6$", "Solution_3": "Hello Andrew and Gauss, Thank you very much ;)" } { "Tag": [ "inequalities", "AMC", "USA(J)MO", "USAMO" ], "Problem": "this is similar to an USAMO problem i did\r\n\r\nProve that:\r\n\r\n$a^ab^bc^c \\ge a^bb^cc^a$\r\n\r\nfor $a,b,c >0$", "Solution_1": "it seems kinda like rearrangement. \r\nif u know the prove of rearrangment, here is the idea:\r\n\r\n$a\\ge b$\r\n$a^{a-b}\\ge b^{a-b}$\r\n$a^a\\cdot b^b\\ge a^b\\cdot b^a$\r\n\r\n;) im 2 lazy to write up the whole prove but u only have to look at 2 cases because WLOG $a\\ge b\\ge c$ or $a\\le b\\le c$ so u can do the thing above first with $a$ and $b$ and then with $b$ and $c$", "Solution_2": "heh nice job\r\n\r\nthis is pretty similar to the USAMO problem:\r\n\r\n$a^ab^bc^c \\ge (abc)^{(a+b+c)/3}$\r\n\r\nfor $a,b,c>0$\r\n\r\ni think thats the problem at least", "Solution_3": "If you take the log of both sides, it is direct rearrangement. Actually if you apply your inequality cyclically you get the USAMO inequality.", "Solution_4": "Here is a nice proof of the USAMO inequality\r\nIt comes from John Scholes -his solution to Question 2 Canadian MO 1995.\r\n\r\nAssume a>= b >= c.\r\n\r\nThen a/c, a/b and b/c are all at least 1 and \r\n\r\n(a-c)/3 , (a-b)/3 and (b-c)/3 are all positive. \r\n\r\nTherefore (a/c) ^([a-c]/3) , (a/b)^([a-b]/3) and ( b/c)^([b-c]/3) are all at least 1.\r\nHence their product is at least 1, which leads to the required inequality.", "Solution_5": "[quote=\"gotztahbeazn\"]heh nice job\n\nthis is pretty similar to the USAMO problem:\n\n$a^ab^bc^c \\ge (abc)^{(a+b+c)/3}$\n\nfor $a,b,c>0$\n\ni think thats the problem at least[/quote]\r\nI got the generialization from this USAMO inequality. :P \r\nhttp://www.mathlinks.ro/Forum/viewtopic.php?t=46247" } { "Tag": [ "superior algebra", "superior algebra unsolved" ], "Problem": "Prove that a left annihilitor of a right ideal of a ring $ R$ with unity is generated by an idempotent", "Solution_1": "I have no idea what you could mean,\r\nbut take $ R\\equal{}\\mathbb{Z}/4\\mathbb{Z}$ and ideal $ I\\equal{}2\\mathbb{Z}/4\\mathbb{Z}$." } { "Tag": [ "puzzles", "WindClan for the win" ], "Problem": "1. A man had a book that was worth $\\$40,000$. He threw it in the furnace, reducing it to a pile of soot. Why did he do this?\r\n[hide=\"hints\"] \nA) He is not mentally disordered or retarded. He did this for a very good reason.\nB) He does care about money.\n[/hide]\r\n2) The police arive at a house. There is a fresh corpse on the floor. Other than that, there is only a tape recorder in the room. When the police played it, they heard this: \"I can't go on. I have nothing to live for\" After this, they hear a gunshot. How did they know the person had been [i]murdered?[/i]", "Solution_1": "My educated guess for the 2nd situation:\r\n[hide]Those quoted words are the words of the killer. He recorded them (his own words) and the gunshot's sound on purpose. He turned off the tape recorder right after that so that sounds following that would not be heard on the playing of the tape.[/hide]", "Solution_2": "not quite. Youre on the right track though.", "Solution_3": "[hide=\"My Thoughts on the situation\"][hide=\"1)\"]He did not care about money, so he just used the book to burn and keep himself warm.[/hide][hide=\"2)\"]The question reads [quote=\"SoxFan34\"]How did they know the person had been killed?[/quote]Uhhh...maybe because he's lying there dead? :rotfl: [/hide][/hide]", "Solution_4": "[quote=\"kstan013\"][hide=\"My Thoughts on the situation\"][hide=\"2)\"]The question reads [quote=\"SoxFan34\"]How did they know the person had been killed?[/quote]Uhhh...maybe because he's lying there dead? :rotfl: [/hide][/hide][/quote]\r\n\r\nnice thought on #2 :rotfl: ...but the person can be dead even if it was suicide. :ninja:", "Solution_5": "[hide=\"2\"]\nIf it had been a suicide, the tape would never have stopped. The fact that it was not only stopped but also [b]rewound[/b] indicates that someone else was involved.\n[/hide]", "Solution_6": "Contributing to the first one:\r\n\r\n[hide] Perhaps there were only two copies of the book in the entire world, and he had both...so by destroying one, he made the other significantly more valuable?[/hide]\nOr\n[hide]In the form of soot, it turns out to be more valuable than in its original form[/hide]", "Solution_7": "[hide=\"1\"]the book was worth and insured for 40,000 dollars but he payed a lot less for it[/hide]", "Solution_8": "I made a topic exactly like number 2 with the tape recorder[/hide]", "Solution_9": "[hide] \n\nK81o7's first solution is right; the man destroys it because he has two, and by only having one, the value goes way up.\nKstan, mathmeister22, youre both wrong, but that's partly my fault... :rotfl: Mathnerd is right: If he'd commited suicide, then the tape recorder wouldve kep going. It was Rewound. [/hide]", "Solution_10": "16 year bump" } { "Tag": [ "AMC", "AMC 8" ], "Problem": "I took the AMC 8 on tues. and I was wondering when we get our \"Offical\" scores. Does anyone know?", "Solution_1": "My teacher says that our official scores don't come out until like Late January/Early February.", "Solution_2": "Oh they come out normally around Dec. 10th.", "Solution_3": "The AMC office will begin mailing official scores and reports in early to mid-December, precise start of mailing depending on the scoring and processing. Most schools in the US should have results before the end-of-year holidays.\r\n\r\nPlease also read Section VI, Report of Results, page 5 in the 2009 AMC 8 Teachers' Manual,\r\nhttp://www.unl.edu/amc/d-publication/d1-pubarchive/2009-10pub/AMC8/2009-amc8TM.pdf\r\n\r\nSteve Dunbar\r\nDirector, MAA American Mathematics Competitions", "Solution_4": "Since the teachers are allowed to open the solutions on November 25, and they can keep the actual test booklet, aren't we able to know unofficial results as early as next week? I remember circling every answer on the test booklet, so that should be enough, right?", "Solution_5": "yes\r\nwhen I took the AMC8, my math coordinator checked the answers on our test booklets with the official answers, so we had the unofficial results within a week", "Solution_6": "is that even allowed?", "Solution_7": "[quote=\"AlphaBetaTheta\"]is that even allowed?[/quote]\r\n\r\nYes, if you only discuss within the school. My teacher did that too.", "Solution_8": "[quote]is that even allowed?[/quote]\r\n\r\nTeachers' Manual, Section VI, Item 4. page 5. Pay attention to items 4.a. and 4.b. too.\r\n\r\nSteve Dunbar\r\nDirector, MAA American Mathematics Competitions", "Solution_9": "ya my school also disussed", "Solution_10": "aren't the teachers unable to discuss the answers until next week.\r\ni mean she said that she couldn't tell us the answers yet because other people take it at different times.", "Solution_11": "i'm not certain, but don't all competitors take the test at the same time?\r\nthat's how we did it at my school, anyway\r\n\r\nthere is an AMC8B that some people may take, but it should be different enough from AMC8A that discussion of the answers should not affect the score of those taking AMC8B that much", "Solution_12": "I'm pretty sure they are the same test, but there is an alternate date for taking it or something like that, in case of conflicts.", "Solution_13": "really?\r\nwell, idk\r\nmy coordinator never ordered the second one =)\r\n\r\ni would think there would be some difference tho\r\notherwise, couldn't they just give one test with a range of dates to administer them?", "Solution_14": "There is only one AMC 8.", "Solution_15": "Go to the [url=http://www.unl.edu/amc/d-publication/d1-pubarchive/2009-10pub/AMC8/2009-10-AMC8-bro.pdf]AMC8 brochure[/url], the second page, at the top it says \"Administration\". That explains it a little.\r\n\r\nHow it works is there is a single test that is mailed to all the proctors. They are supposed to open them for the first time and give the tests to the students on Tuesday, November 17. If a school has a conflict or problem with that date, they can request special permission to administer the test on another date. Solutions (which are mailed to proctors with the tests) should not be opened until November 25, when everybody should have taken the test. Also, students may not discuss the test with anyone but the people who took the test at the same place and time (you signed something at the bottom of the answer sheet pledging that). On November 24th, the proctors mail the answer sheets back to the AMC headquarters, where the official grading is done. Since the actual question pamphlets don't have to be mailed, if the person who took the test circled all the answers he/she had in the test pamphlet, the proctor can compare the solutions with the circled answers and give an unofficial grade. The only time when this is different than the official grade is if you bubbled incorrectly or your proctor graded incorrectly." } { "Tag": [ "quadratics", "algebra", "quadratic formula", "special factorizations" ], "Problem": "I need help with the steps turning the standard form into the quadratic formula can someone please help me with the proof and the steps involved. Thanks", "Solution_1": "One way to prove the quadratic formula by completing the square - since the general composition of a quadratic equation is of the form $ ax^2 + bx + c$, we can divide both steps of the equation by $ a.$\r\n\r\nSo far we have:\r\n\r\n$ ax^2 + bx + c = 0$\r\n(Note that $ a\\neq0.$)\r\n$ x^2 + \\dfrac{bx}{a} + \\dfrac{c}{a} = 0$\r\n$ x^2 + \\dfrac{bx}{a} = - \\dfrac{c}{a}$\r\nCompleting the square, we add $ \\dfrac{b^2}{4a^2}$ to both sides of the equation.\r\nWe now have the following:\r\n$ x^2 + \\dfrac{bx}{a} + \\dfrac{b^2}{4a^2} = - \\dfrac{c}{a} + \\dfrac{b^2}{4a^2}$.\r\n$ (x + \\dfrac{b}{2a})^2 = \\dfrac{ - 4ac}{4a^2} + \\dfrac{b^2}{4a^2}$.\r\n$ (x + \\dfrac{b}{2a})^2 = \\dfrac{b^2 - 4ac}{4a^2}$\r\nTherefore, \r\n$ x + \\dfrac{b}{2a} = \\pm \\sqrt {\\dfrac{b^2 - 4ac}{4a^2}}$\r\n$ x + \\dfrac{b}{2a} = \\pm \\dfrac{\\sqrt {b^2 - 4ac}}{2a}$\r\n$ x = - \\dfrac{b}{2a}\\pm \\dfrac{\\sqrt {b^2 - 4ac}}{2a}$\r\nAdding the two like terms we get $ \\dfrac{ - b\\pm \\sqrt {b^2 - 4ac}}{2a}$.\r\n\r\nDid you understand my steps? If you need any help, just ask.", "Solution_2": "Of course though, $ a\\neq0.$ Otherwise the expression wouldn't be a quadratic and you'd be dividing by 0. The rest is okay.", "Solution_3": "Oh right, forgot about that. :)", "Solution_4": "You should have also explained why the step $ \\sqrt {4a^2} \\equal{} 4a$ is valid. Remember that in general $ \\sqrt {x^2} \\equal{} |x|$. Using a bit of casework ($ a < 0$ and $ a > 0$),you'll notice that regardless of the sign of $ a$, the final expression will ultimately remain the same.", "Solution_5": "Which step are you talking about? $ \\sqrt{4a^2} \\neq 4a$.", "Solution_6": "He meant $ \\sqrt{4a^2}\\equal{}2a$. Also, remember that since were working with variables, $ a$ could be negative." } { "Tag": [ "inequalities", "algebra", "polynomial", "calculus", "2-variable inequality" ], "Problem": "$a,b$ are non negative reals not both equal to $0$. Prove that\n\\[\\frac{a^4+b^4}{(a+b)^4} + \\frac{\\sqrt{ab}}{a+b} \\geq \\frac58\\]", "Solution_1": "Without loss of generality we may assume that $a+b=1$. We must prove that $a^4+b^4+\\sqrt{ab} \\geq \\frac58$.\nOn the boundary $a=0$ the inequality is trivial. Than by the Lagrange principle there exists $\\lambda$ such that the following equalities hold:\n\\[4a^3+\\frac12\\sqrt{\\frac{b}a}=\\lambda, \\ \\ \\ \\ 4b^3+\\frac12\\sqrt{\\frac{a}b}=\\lambda\\]\n\nthen $8(a^3-b^3)=\\frac{a-b}{\\sqrt{ab}}$. So $a-b=0$ (then $a=b=\\frac12$ and inequality become to equality), or $8(1-ab)\\sqrt{ab}=1$. It is easy to see that \n$\\sqrt{ab} \\in \\left(0;\\frac12\\right)$, but there is no root of the polynomial $t^3-t+\\frac18$ in that interval.", "Solution_2": "Your proof is good, Fedor.\r\nI am thinking about a proof without calculus but I don't know if it exists.", "Solution_3": "We can prove a stronger result: \n\\[\\frac{a^4 + b^4}{(a+b)^4} + \\frac{3ab}{(a+b)^2} \\geq \\frac78 \\ \\ (1)\\]\n(by an easy computation it is equivalent to $(a-b)^4 \\geq 0$.\n\nOur desired inequality now follows from $(1)$ because \n\\[ \\frac12 \\geq \\frac{\\sqrt{ab}}{a+b} \\geq \\frac{2ab}{(a+b)^2}\\]", "Solution_4": "Congratulation Namdung, your proof is very nice and very neat.\r\nA beautiful reasoning!!" } { "Tag": [ "geometry", "3D geometry", "MATHCOUNTS" ], "Problem": "Figure A is a single wooden cube that has edge length 3 meters. Square holes with side length 1 meter centered in each face are cut through to the opposite face. The edges of the holes are paralell to the edges of the cube. Figure B is formed by connecting 27 of these cubes. What is the number of square meters in the total surface area of Figure B?\r\n\r\nI don't have the original diagram, but if anyone would like to post it, feel free to do so. :lol: \r\nI've never understood this question for some reason, though the rest of the questions from that year are EASY.", "Solution_1": "this problem isn't particularly difficult...im pretty sure aops vol. 1 has a diagram. was this a target?\r\nanyways, basically just imagine a cube with a hole in each center square on each face..the way i do it is i find the surface area of the outside faces first (which in this case is $ (9\\minus{}1)\\cdot 6$ then imagine what the inside of the cube looks like...that should help. :)", "Solution_2": "But wouldn't that only apply for Figure A? Figure B's 27 smaller squares would have some of the faces covered by other squares, right?", "Solution_3": "Im assuming that fig B is a $ 3 \\times 3 \\times 3$ of Fig A.\r\nThus you can see the interior surface from the holes is always counted.\r\nThere are 54 faces of Fig A on the cube.\r\n\r\ninterior surface area $ 8*27\\equal{}216$\r\nExterior $ 54*8\\equal{}432$ \r\nSurface Area$ \\equal{}216\\plus{}432\\equal{}648$", "Solution_4": "firgure B has 9*6=54 faces of figure A on it. each face of figure A has a surface area of 9+4=13. then do 54*13=702. \r\n\r\n[b]frankel-[/b]where did you get you interior surface area?", "Solution_5": "[quote=\"abacadaea\"]firgure B has 9*6=54 faces of figure A on it. each face of figure A has a surface area of 9+4=13. then do 54*13=702. \n\n[b]frankel-[/b]where did you get you interior surface area?[/quote]\r\n\r\nFigure A has holes in it.", "Solution_6": "abacadaea, that's what I got, but the answer's [hide]1080[/hide] :huh:", "Solution_7": "Ahh I miscounted.\r\nThe exterior surface is the same.\r\nThe interior surface of Fig A is $ 27*24\\equal{}648$\r\nTotal Surface Area$ \\equal{}648 \\plus{}54*8\\equal{}1080$", "Solution_8": "[quote=\"Fraenkel\"][quote=\"abacadaea\"]firgure B has 9*6=54 faces of figure A on it. each face of figure A has a surface area of 9+4=13. then do 54*13=702. \n\n[b]frankel-[/b]where did you get you interior surface area?[/quote]\n\nFigure A has holes in it.[/quote]\r\nyes i realize that. the surface area of each hole is 4 more thean the side would be without a square. then there would be 5*54+8*54\r\n\r\nEDIT: you beat me to the post i probably misinterpreted the diagram", "Solution_9": "I think this was #6 on a handout (I vaguely remember looking over some kids on my team's paper...and they were a bit confused since they got it wrong)\r\n\r\nso just do\r\n\r\n[hide]\n6(72) for outer part + 27(24) for inner part\n\n432+648=1080\n\nso the 72 is just each side with 9 'blocks' of 8 squares in each so 72 (and 6 of them so 1080)\n\nand the 24 is just the surface area for each 'small' cube (then you just multiply by 27 because all the holes are connected, so it doesn't decrease the area amount)\n\nthere's probably an easier way :oops: [hide]\n\nhope that helps![/hide][/hide]", "Solution_10": "I don't understand. How did you get 24? I keep getting 30 for the surface area of the small cube. I think that there is a 1x1x1 cube in the center of figure A, so it's faces should be included in the surface area, right? :blush:", "Solution_11": "[quote=\"Math Geek\"]I don't understand. How did you get 24? I keep getting 30 for the surface area of the small cube. I think that there is a 1x1x1 cube in the center of figure A, so it's faces should be included in the surface area, right? :blush:[/quote]\r\n\r\nok so the surface area for the inner part of a 3x3x3 cube is 24 for the following reason (imagine this...). alright, so you have 6 'tunnels' each from a different side of the cube, and in these tunnels you have open space surrounded by 4 1x1 'walls', thus the surface area of each of these is 4...multiply that by 6 and you have 24. the reason why you don't have any surface area in the middle is because it's empty space surrounded by those 'tunnels' (aka no 'walls' -> no surface area)...so 24 is it...hope that helps \r\n(and you multiply by 27 because none of the inner surface area is blocked off by other 3x3x3 cubes)\r\n\r\ni hope my visual (without being visual) explanation was ok\r\n\r\nmaybe an experienced mathcounts person can help on the image if you still don't understand", "Solution_12": "im lost could someone post a diagram?", "Solution_13": "It's a bit hard to post any more than this...\r\n\r\n(i'm attempting to attach it)", "Solution_14": "oh whoops. i thought there was a little indent on each side of a figure A", "Solution_15": "so, umm, the answer is 1080? that's what i got.....i get dizzy when i see this problem. :D", "Solution_16": "If you read some of the other entries, you would know that the answer is 1080. Please don't revive :wink:" } { "Tag": [], "Problem": "how do we factor this? \r\n\r\nx^2 - 7x + 12\r\n\r\nand\r\n\r\nx^2 - 7x - 60\r\n\r\n\r\nThank you.", "Solution_1": "[quote] Knowledge is power. [/quote]\r\n\r\nYou are absolutely right, thus you'd better do your homework by yourself. :P \r\n\r\nPierre.", "Solution_2": "Factors of 12 that add up to -7, (-3 and -4) (x-3)(x-4)\r\nFactors of -60 that add up to -7, (-12 and 5) (x-12)(x+5)", "Solution_3": "i would do this by: \r\n\r\n1. What multiplie to 12\r\n and adds to -7\r\n well -4 and -3 do.\r\nso it would be (x-4)(x-3)\r\n\r\n2.what multiplies to 60\r\n and adds to -7\r\nwell, -12 and 5 do.\r\nso it would be (x-12)(x+7)", "Solution_4": "Just like me :lol:", "Solution_5": "[quote=\"nat mc\"]Just like me :lol:[/quote]\r\nyea, i'm just trying to get better at typing.", "Solution_6": "hey good job on trying! :lol:" } { "Tag": [ "function", "induction" ], "Problem": "Let the function $f: \\mathbb{R}\\to\\mathbb{R}$ satisfy:\r\n$f(1)=2007$ and $f(1)+f(2)+\\cdots+f(n)=n^2f(n), \\forall n \\in \\mathbb{N}$.\r\nFind $f(2006)$.", "Solution_1": "It is similar to British Mathematical Olympiad, 1996, question 2.\r\n[hide]\n$f(1)+f(2)=2^2f(1)$ implies $f(2)=\\frac{1}{(2^2-1)}\\cdot f(1)$.\nNext,\n$f(3)=\\frac{1}{3^2-1}\\cdot (f(1)+f(2))=\\frac{1}{3^2-1}\\cdot [ f(1) + \\frac{1}{2^2-1}] \\cdot f(1) \\\\ = \\frac{1}{2^2-1}\\cdot \\frac{2^2}{3^2-1} \\cdot f(1)$\n$f(4) = ........$\nGood luck!\n\n[/hide]", "Solution_2": "$f(1)+f(2)+\\cdots+f(n)=n^2f(n)$\r\n$f(1)+f(2)+\\cdots+f(n)+f(n+1)=(n+1)^2f(n+1)$\r\nBy subtracting first equation from the second we obtain:\r\n$f(n+1)=(n+1)^2f(n+1)-n^2f(n)$\r\n$f(n+1)=\\frac{n}{n+2}f(n)$\r\nBy induction we have:\r\n$\\boxed{f(n)=\\frac{4014}{n(n+1)}}$", "Solution_3": "omg...its the OMK2006 question!", "Solution_4": "whats OMK?", "Solution_5": "Olympiad Matematik Kebangsaan. Means Malaysia National Mathematic Olympiad" } { "Tag": [ "geometry", "3D geometry", "sphere", "LaTeX" ], "Problem": "we drop small balls (can considered to be points) on a concave half sphere (that is the half sphere is open upwards) of radius $ R$. We drop the balls from height $ h$ with respect to the point it hits in the half sphere. find $ h$ so that all the balls will bounce after one single fully elastic collision to the very bottom point of the sphere.", "Solution_1": "Probably i misunderstood the problem, anyway, let's try..\r\nThe balls hit the half sphere in the point P and rebounce elastically in the direction of the radius (OP). The velocity is [b]v[/b] with an angle of [b]90 - t[/b] relative to the horizontal axis (look at the picture!). Now i consider the parabolic motion of a single ball and i use the parametric equation.\r\n\r\n[i]y = y(0) + tg(90 - t) * x - (g*x^2)/(2*(v*cos(90-t)^2)[/i]\r\n\r\nThe ball has a horizontal distance from the bottom point H equal to [b]R*sin(t)[/b], while it stands at the height [b]R*(1 - cos(t))[/b]. Setting y=0 and x=[b]R*sin(t)[/b], we calculate the velocity v\r\n\r\n0 = R(1-cos(t)) + R sin(t)*cos(t)/sin(t) - (g*(Rsin(t))^2)/(2*(v*sin(t))^2)\r\n\r\n1 - cos (t) + cos(t) - g*R/(2*(v^2)) = 0\r\n\r\nv^2 = g*R/2 (1)\r\n\r\nThe ball is dropped from the height h with respect to point P, with an inizial velocity=0, so\r\nv^2 = 2*g*h\r\n\r\nFrom the (1) we have: h= R/4\r\n\r\nSorry, but i haven't latex yet...let me know if it's right, even if i think as i understood the problem is quite easy..\r\nenjoy :D", "Solution_2": "That's not quite right.\r\n\r\nAfter elastic collision the angle with the normal (t) is the same. So if that is correct what you say, then t=90-t which is clearly wrong... :wink:", "Solution_3": "so the balls don't fall in the normal direction...well, i considered the balls' direction after they hit the sphere as the direction of reflected rays...\r\ni'm not sure of what u say, the direction should be normal to the surface, so radial!\r\nCan you post a picture? If the direction is the same before and after they hit the sphere, we only need to substitue t to 90-t...\r\nsee u", "Solution_4": "ok here is a rough diagram :) \r\n\r\nso after the collision the angle of the velocity with the radius is $ \\phi$ hence the angle of the velocity with the horizontal is $ 90\\minus{}2\\phi$.", "Solution_5": "ok!! Now it's clear..the ball rebounces with the same angle relative to the radial direction, because the collision is elastically (i didn't understood how toe express this fact)\r\nSubstituing 90 - 2t in my wrong solution, we have:\r\n\r\nh= R/(8*cos(t)*(2cos(t)-1))\r\nor something like that...\r\nlet's see if u can post harder problems :lol: :P", "Solution_6": "yes thats correct :) \r\n\r\nWell I can post harder problems, check my previous ones. some of them are really tough. Anyway I will post another hard one if you wish :)" } { "Tag": [ "algebra", "polynomial", "geometry", "rhombus", "national olympiad" ], "Problem": "\\[\\text{Day 1}\\]\r\n1. Polynomial $P(x)$ has integer coefficients. Prove, that if polynomials $P(x)$ and $P(P(P(x)))$ have common real root, they also have a common integer root.\r\n\r\n2. $ABCDE$ is a convex pentagon and:\r\n\\[BC=CD, \\;\\;\\; DE=EA, \\;\\;\\; \\angle BCD=\\angle DEA=90^{\\circ}\\]\r\nProve, that it is possible to build a triangle from segments $AC$, $CE$, $EB$. Find the value of its angles if $\\angle ACE=\\alpha$ and $\\angle BEC=\\beta$.\r\n\r\n3. We are given equilateral triangle with side $n$ which is divided into $n^{2}$ congruent equilateral triangles with side $1$. Let's call these small triangles plates. Each plate is either black or white. We perform the following operation:\r\nwe choose a plate which has common sides with at least two plates of diiferent colour from chosen plate and then we change the colour of our plate. For every $n\\geq 2$ decide whether exists a configuration of colours that enables to perform infinite sequence of operations.\r\n\\[\\text{Day 2}\\]\r\n4. $a$, $b$, $c$, $d$ are positive integers and\r\n\\[ad=b^{2}+bc+c^{2}\\]\r\nProve that\r\n\\[a^{2}+b^{2}+c^{2}+d^{2}\\]\r\nis a composed number.\r\n\r\n5. A convex quadrilateral $ABCD$ with $AB\\neq CD$ is cyclic. Quadrilaterals $AKDL$ and $CMBN$ are rhombs with side $a$. Prove that quadrilateral $KLMN$ is cyclic.\r\n\r\n6. $a$, $b$, $c$, $d$ are positive real numbers satisfying the following condition:\r\n\\[\\frac{1}{a}+\\frac{1}{b}+\\frac{1}{c}+\\frac{1}{d}=4 \\]\r\nProve that:\r\n\\[\\sqrt[3]{\\frac{a^{3}+b^{3}}{2}}+\\sqrt[3]{\\frac{b^{3}+c^{3}}{2}}+\\sqrt[3]{\\frac{c^{3}+d^{3}}{2}}+\\sqrt[3]{\\frac{d^{3}+a^{3}}{2}}\\leq 2(a+b+c+d)-4. \\]\r\nThanks robpal and barasawala :wink:", "Solution_1": "1. http://www.mathlinks.ro/Forum/viewtopic.php?t=136487\r\n2. http://www.mathlinks.ro/Forum/viewtopic.php?t=136488\r\n3. http://www.mathlinks.ro/Forum/viewtopic.php?t=136490\r\n4. http://www.mathlinks.ro/Forum/viewtopic.php?t=136491\r\n5. http://www.mathlinks.ro/Forum/viewtopic.php?t=136489\r\n6. http://www.mathlinks.ro/Forum/viewtopic.php?t=135374", "Solution_2": "This is version pdf" } { "Tag": [], "Problem": "What's your favorite fruit?\r\n\r\nSee Fav. fruit 1 and Fav. fruit 3 if your favorite fruit is not here. The \"other\" option is in Fav. fruit 3. Please only vote in one of the polls.\r\n\r\nFav. fruit 1 is the main thread for discussion.\r\n\r\nSorry if your favorite fruit is bunched up in a category and you don't like the other fruits in that category.", "Solution_1": "*bump* for those who might not know about this and number 3..." } { "Tag": [ "calculus", "derivative", "function", "calculus computations" ], "Problem": "Hi, i have two problem that i cant resolve \"please help me\" :P\r\n\r\n1.- Change in temperature along a circunference\r\n\r\nA particle is moving in the direction of a clockwise around the circle of 1m radius centered at the origin. Rate of 2 m/s.\r\n\r\nHow fast the temperature experimented by the particle change in degrees Celsius per meter, in the point\r\n\r\n[url=http://imageshack.us][img]http://img17.imageshack.us/img17/7596/pointuo6.jpg[/img][/url]\r\n\r\n2.- I have to prove the wave equation:\r\n\r\n [url=http://img145.imageshack.us/my.php?image=waveecuationqm0.jpg][img]http://img145.imageshack.us/img145/2910/waveecuationqm0.jpg[/img][/url]\r\n\r\n\r\nThanks !", "Solution_1": "The first problem doesn't make any sense as written. I suspect that you just forgot half of the conditions.\r\n\r\nAs to the second, you cannot \"prove the wave equation\". I suspect that you were asked to prove that the given function is a solution to it. Then all you need to do is to differentiate a few times using the chain rule.", "Solution_2": "Thanks Fedja for the reply.\r\n\r\nThe first problem was taken from a book, it must be rigth :S \r\n\r\nThe only thing i could do is find the ecuation of the particle movement but i don know how to find a conection with the temperature ecuation.\r\n\r\nThe particle ecuation -> r(t) = sen (2t) i + cos (2t) j \r\n\r\nThis ecuation move the particle in the direction of a clockwise around a circle of 1m radius centered at the origin with rate of 2 m/s.\r\n\r\n\r\nI have to find: \r\n\r\ndt = (Directional Derivative in the point) . df\r\n\r\nI think the direction of the derivative is the direction of the particle velocity in the point, but i dont know how to find 'df'\r\n\r\n\r\nAnd in the second problem i have to 'prove that the given function is a solution', :P but when i tried to prove that i find some complications, if someone can teach me the steps to resolve that :oops:" } { "Tag": [], "Problem": "What about if the question is like this:\r\n\r\n$ 1. 3x_{1}+5x_{2}+x_{3}+x_{4}= 10$\r\n$ 2. 3x_{1}+5x_{2}+x_{3}+x_{4}= 100$", "Solution_1": "What is your question? Would you like to solve this system in integers?\r\nIf I was not failed to understand the question it is extremely easy.\r\n\r\nSubtraction gives $ x_{1}=90$ and the system becames $ 5x_{2}+x_{3}+x_{4}=-107$. \r\nTake any $ x_{2}$ and $ x_{3}$ and calculate $ x_{4}$ from this.\r\nAnswer:\r\n$ x_{1}=90$\r\n$ x_{2}=u$\r\n$ x_{3}=v$\r\n$ x_{4}=-107-5u-v$\r\nfor arbitrary integers $ u$ and $ v$.", "Solution_2": "[quote=\"digger\"]What is your question? Would you like to solve this system in integers?\nIf I was not failed to understand the question it is extremely easy.\n\nSubtraction gives $ x_{1}=90$ and the system becames $ 5x_{2}+x_{3}+x_{4}=-107$. \nTake any $ x_{2}$ and $ x_{3}$ and calculate $ x_{4}$ from this.\nAnswer:\n$ x_{1}=90$\n$ x_{2}=u$\n$ x_{3}=v$\n$ x_{4}=-107-5u-v$\nfor arbitrary integers $ u$ and $ v$.[/quote]\r\n\r\nIts finding the number of non-negative solutions for each equation. the two are independent from each other. I just tried number 2 if its easy to solve given a higher sum. Can't seem to find an easy way to do it." } { "Tag": [ "integration", "calculus", "calculus computations" ], "Problem": "How can we compute $\\underset{k=0}{\\overset{+\\infty }{\\sum }}\\frac{(-1)^{k}x^{2k+1}}{(2k+1)k!}$ for a certain real $x$ using fourier series ? (if possible of course :lol: )", "Solution_1": "This is $\\int_0^xe^{-t^2}\\,dt.$\r\n\r\nWe're not going to find a closed form formula; we know that $e^{-t^2}$ has no elementary antiderivative.\r\n\r\nThe limit as $x\\to\\infty$ is well known. If we want numerical approximations, for $x$ not too large, the power series itself converges satisfactorily rapidly. There's a useful asymptotic expression we could use for large $x.$\r\n\r\nI don't see a natural or straigtforward link to Fourier series.", "Solution_2": "Ok, I just wanted to know if there is a way to link it with fourier series, because my objectif was to compute that well known integral through it. But as it seems, it's not possible :D \r\nThanks" } { "Tag": [], "Problem": "CuSO4(aq) + Fe(s) --> FeSO4(aq) + Cu(s)\r\nFeSO4(aq) + Cu(s) --> CuSO4(aq) + Fe(s)\r\n\r\n[img]http://img106.imageshack.us/img106/6140/chem3fp.png[/img]\r\n\r\nWhy does the iron plate becomes brown?\r\nI have no precise idea, but I think it might be related to the loss of electrons of the iron plate. Could someone help, please?", "Solution_1": "It is because the iron is going into solution and the copper is coming out of solution forming the brown you see on the iron plate.", "Solution_2": "CuSO4(aq) + Fe(s) --> FeSO4(aq) + [b]Cu(s)[/b]\r\n\r\nthats why..." } { "Tag": [ "algebra", "function", "domain" ], "Problem": "For what values of $x$ is this defined?\r\n\r\n$\\frac{23}{(x-1)(x+2)(x-3)(x+4)}$\r\n\r\nWrite in interval notation if you can.", "Solution_1": "[quote=\"Drunken_Math\"]For what values of $x$ is this defined?\n\n$\\frac{23}{(x-1)(x+2)(x-3)(x+4)}$\n\nWrite in interval notation if you can.[/quote]\r\n\r\n[hide]The only values not in the domain are -4,-2,1,3\n\nSo the values in this domain are $(-\\infty,-4)U(-4,-2)U(-2,1)U(1,3)U(3, \\infty)$.[/hide]", "Solution_2": "[quote=\"mathgeniuse^ln(x)\"][quote=\"Drunken_Math\"]For what values of $x$ is this defined?\n\n$\\frac{23}{(x-1)(x+2)(x-3)(x+4)}$\n\nWrite in interval notation if you can.[/quote]\n\n[hide]The only values not in the domain are -4,-2,1,3\n\nSo the values in this domain are $(-\\infty,-4)U(-4,-2)U(-2,1)U(1,3)U(3, \\infty)$.[/hide][/quote]\r\n\r\nGood job! :D" } { "Tag": [ "trigonometry", "inequalities", "function", "calculus", "derivative", "geometry", "inequalities unsolved" ], "Problem": "prove that for x in (0,pi/2) we have sin(2x)<2/(3x-x^3)", "Solution_1": "can anybody tell me the proof of this inequality? and similar ones, that's sinx > a function of x. I have no idea how to prove this kind of inequality. Or are there some inequalities that are well-known to prove this kind of inequalities? Thx", "Solution_2": "would differentation help? or one could try a geometrical approach to compare areas or side lengths.\r\n\r\nlike the proof of tan x > x > sin x and the proof of (tan x + sin x)/2 > x (i think the second's correct its an old hungarian problem)", "Solution_3": "x^3+2>=3x and x=1>=sin(2x)\r\nCases of equality are different so equality never holds." } { "Tag": [], "Problem": "Suppose you could uniformly heat a liquid in a container under the normal gravitational pull of the Earth. Assume that liquid exerts a pressure due to its own weight . Will the liquid start to boil on the bottom, at the top, or uniformly throughout? \r\n\r\nMy theory is that the liquid at the bottom must bear the extra pressure of not only atmospheric but also the water above it, so it boils at a higher temperature (b/c boiling point happens when vapor presure equals atmospheric pressure. So is my logic right? Or can someone explain this in a more clear way?", "Solution_1": "[quote=\"Bythecliff\"]Assume that liquid exerts a pressure due to its own weight[/quote]\n\nA liquid always exerts a pressure on its container. Are you taking into account in your argument the currents of convection?\n\n[quote=\"Bythecliff\"]boiling point happens when vapor presure equals atmospheric pressure[/quote]\r\n\r\nIt would be more correct to say \"... when vapour pressure equals the external pressure\", since, for example, when cooking in a \"pressure apparatus\" (I apologize, I don't know how to say this in English) the boiling doesn't happen when the vapour pressure equals atmospheric pressure since the external pressure is quite greater than atmospheric pressure - because the container is closed.", "Solution_2": "The English for that is \"pressure cooker.\"\r\n\r\nBythecliff's thought experiment might be difficult to do in practice, but if you could, the fluid [i]wouldn't[/i] convect. The ambient pressure would be lowest at the top, and boiling would happen first at the top. Furthermore, it wouldn't bubble. It wouldn't even look like it was boiling - but vapor would rise from it and the fluid level would drop.\r\n\r\nOur experience of boiling, with pots of water on the stove, involves situations in which all, or almost all, of the heat is added to the lower surface of the container. So the water boils from the bottom, bubbling and convecting furiously. Even a coffee cup stuck in a microwave oven isn't that close to the heat being added uniformly - the rate of heat transfer should be greater towards the outside of the cup and lower in the interior.", "Solution_3": "[quote=\"Kent Merryfield\"]The English for that is \"pressure cooker.\"[/quote]\n\nOk, thanks.\n\n[quote=\"Kent Merryfield\"]the fluid wouldn't convect. The ambient pressure would be lowest at the top[/quote]\r\n\r\nYes, I agree with that. With uniform heating, convenction don't make sense. The external pressure is lowest at the top, that's true, since in the interior of the liquid we have to take into account the hydrostatic pressure. However, note that the hydrostatic pressure contibution is neglegible: for example, for a column of water with 50 cm tall the pressure at the bottom would be about 1.05 atm, just 0.05 atm higher than atmospheric pressure. So, I think it would be also reasonable to consider that the liquid would start to boil is a very close uniform manner throughout the liquid." } { "Tag": [ "irrational number" ], "Problem": "take two irrationals q1 and q2, with q1 does not equal q2. is it true that there is always a rational r1 such that q1$ \\frac{x_2\\minus{}x_1}{\\pi} $ 0<(x_2\\minus{}x_1)(\\pi\\minus{}1)$ which is true.", "Solution_2": "Both of these statements are true because,\r\n\r\n[i]The set of rational numbers is dense in the reals.[/i]\r\n\r\nHere's a proof for the first statement,\r\n\r\n[hide=\"Proof\"]\n(i) Let $ q_1,q_2\\in\\mathbb{R}: q_10$ and $ \\frac{1}{q_2\\minus{}q_1}>0$.\n\nLemma: Let $ x\\in\\mathbb{R}$. Then there exists $ n\\in\\mathbb{N}$ such that $ n>x$.\n\n(iii) By the lemma, there exists $ n\\in\\mathbb{N}$ such that $ n>\\frac{1}{q_2\\minus{}q_1}>0$.\n\n(iv) Therefore, $ 0<\\frac{1}{n}q_1$.\n\n(Note: This is true because, by the lemma, there is an $ m_1\\in\\mathbb{N}: m_1>nq_1$. Dividing through by $ n$ yields $ \\frac{m_1}{n}>q_1$.)\n\n(vi) With the same logic as in (v), we know there exists $ m_2\\in\\mathbb{N}$ such that $ \\minus{}\\frac{m_2}{n}q_1$ is non-empty, and bounded below. \n\n(viii) Thus is has a smallest element $ m$. Therefore, $ \\frac{m\\minus{}1}{n}q_1$ and $ \\frac{m}{n} q_1$.\n[/quote]\r\n\r\nJust very minor tecnhicalities, the proof overall is great :D", "Solution_4": "Thanks for the insight. I kind of confused myself too :) . Trying to make a proof readable is almost as hard as the proof itself sometimes." } { "Tag": [ "algebra", "binomial theorem", "complex numbers", "combinatorics unsolved", "combinatorics" ], "Problem": "Show that if n is a positive multiple of 6,\r\n\r\n1) nCr(n, 1) - 3 nCr(n,3) + 3^2 nCr(n,5) - .... = 0\r\n\r\n2) nCr(n, 1) - 1/3 nCr(n,3) + 1/3^2 nCr(n,5) - .... = 0\r\n\r\nCombintorial argument or whatever cleverness is welcome.", "Solution_1": "Such statements can be proved using complex numbers. By binomial theorem it's quite obvious, taht the first one is equivalent to showing that the imaginary part of $(1+i\\sqrt 3)^n$ is equal to 0, which is true, because $(1+i\\sqrt 3)^6=2^6$, so if 6 divides n, the imginary part is 0. The second one is equivalent of showing that imaginary part of $\\left(1+i\\frac{\\sqrt 3} 3\\right)^n$ is equal to 0, \r\n$\\left(1+i\\frac{\\sqrt 3} 3\\right)^6=\\frac{2^6} {3^3}$, so the imaginary part is 0 again." } { "Tag": [ "Harvard", "college", "calculus", "Princeton", "percent", "Stanford", "MIT" ], "Problem": "I am interested in majoring in computer science/mathematics.\r\n\r\nI have reasearching the top schools and I know that Harvard has a good math department. However, I was wondering if anyone could give me any information regarding the quality of Harvard's CS department.\r\n\r\nAlso, I am currently enrolled in a Community college and will earn an AS degree and high school diploma concurrently. Does anyone know weather or not I can transfer my credits? I have read the site, but the information seems a little vague.\r\n\r\nAny information would be greatly appreciated.", "Solution_1": "You cannot transfer community college credits to most of the highly selective elite colleges, such as Harvard. Are you planning to enter Harvard as a first-year undergraduate, or as a transfer student?", "Solution_2": "Tokenadult is 100% correct. I've talked to two Harvard representatives and they both told me the same thing. (:()\r\n\r\nA good idea might be to take the corresponding AP test to get the credit. That's what I did for Calculus BC and I had to do about 2 hrs of self-study and got a 5, and I plan to take a few sciences to get some credit also. You might need to pay some money out of your pocket depending on your school but I'm sure that it'll cost you less than a semester or two of college.\r\n\r\nGood luck!\r\n\r\nQuestion:\r\n\r\nIt would probably be at my disadvantage to apply to Harvard as a transfer from a community college rather than a highschool applicant, correct?", "Solution_3": "Odds of admission to most of the elite colleges are much lower for transfer applicants than for freshman applicants. Princeton claims to not accept transfer applications at all. So the smart way to take college-level courses before applying to an elite college is to take them as part of your \"high school\" program, which is something done by about 5 percent of the high school students in the United States. \r\n\r\n[url=http://www.ed.gov/news/pressreleases/2005/04/04062005a.html]High School Students Using Dual-Enrollment Programs[/url]", "Solution_4": "[quote=\"tokenadult\"]Odds of admission to most of the elite colleges are much lower for transfer applicants than for freshman applicants. [/quote]\r\n\r\nThat statistic is not a reliable indicator. Transfer students are subject to different application procedures and the standards may be higher or (sometimes dramatically) lower than for the regular population.", "Solution_5": "What are successful examples of transfer applications known to readers of this thread that fit the pattern described by the original poster?", "Solution_6": "[quote=\"tokenadult\"]What are successful examples of transfer applications known to readers of this thread that fit the pattern described by the original poster?[/quote]\r\n\r\nThe original poster was talking about Harvard CS. I know several successful transfer applicants to Harvard from community college (or state institutions comparable to community college), including at least one in CS. Whether or not they received one-to-one transfer credits, the transfer generally did not increase the number of semesters it took them to graduate. Transfer of credits is judged on arrival by professors from the relevant departments.", "Solution_7": "Thank you for your information regarding transferability of credits. Now how about some information regarding the quality of Harvard's computer science department", "Solution_8": "I should say, having reviewed your first message, that the issue really is that if you do your college credits as part of a high school program, that will help you be more likely to be admitted, but you won't get any transfer credits at all. For sure. As long as you are getting a high school diploma as part of the program you are doing, you are a high school student in Harvard's eyes, irrespective of the associates degree you simultaneously earn. Advanced standing at Harvard is based on test scores, not on previous courses taken. \r\n\r\n[url=http://www.fas.harvard.edu/~fdo/publications/advancedstanding0607/]Advanced Standing at Harvard College[/url] \r\n\r\nBut you will always be placed in appropriate courses, based on your background, and Harvard takes care to make sure that it's courses are challenging even for very well prepared students. \r\n\r\nAs for whether you should apply to Harvard if you are very interested in computer science, sure, why not? The traditional list of \"top five\" computer science undergraduate departments in the United States includes Stanford, MIT, Carnegie Mellon, and one or two other non-Harvard colleges that I'm sure someone will fill in here, but Harvard's department is not bad.", "Solution_9": "[quote=\"tokenadult\"] you will always be placed in appropriate courses, based on your background [/quote]\r\n\r\nThe top US universities, including Harvard, tend to have plain vanilla placement exams to calibrate matriculants' level within the basic calculus, French, etc course sequences. Apart from that and some perfunctory supervision by academic advisors, students can enroll in a nearly unrestricted set of classes. Attempts to take a third-year PhD class by somebody just out of high school might require some permission, but the general philosophy, particularly at Harvard, was to \"give them enough rope to hang themselves\". There has been a trend toward increased supervision in the past decade, but also increased awareness that some students enter with a specialized background in science and should be given relatively free rein. \r\n\r\nHarvard CS in the past had some notable faculty such as Rabin and Valiant, but otherwise was not a strong department. More recently they invested a lot of money to upgrade the department, and the quality is closer to top-level now.", "Solution_10": "Harvard faculty is strong across the board, including CS, however, the CS dept is clearly not as strong as the biggies like MIT. Perhaps the biggest difference at Harvard is that the CS dept is much much smaller than at the elite CS schools. \r\n\r\nFrom an undergrad point of view, the biggest differences would be: (a) far fewer courses to choose from (although cross-enrollement at MIT is possible, of course), and (b) smaller, more intimate atmosphere. You'll probably see the same people in your classes and know everyone who is concentrating in CS. The students in the Harvard CS dept are probably as good as those anywhere. (Again, because of numbers, you probably don't hear much about them.)\r\n\r\nAlso important is how sure you are that you want to do CS.", "Solution_11": "[quote=\"yenlee\"]Also important is how sure you are that you want to do CS.[/quote] \r\n\r\nYes, that is very important. Harvard offers many other majors (\"concentrations,\" in Harvard terminology) and most of those are academically strong and well regarded. Some other schools with notable CS departments have few other strong departments, so Harvard has value as a place where you can change your mind and still end up with a good major program." } { "Tag": [ "inequalities", "inequalities proposed" ], "Problem": "Find all positive integers $ n$ such that the following inequality holds for positive reals $ a_{1},a_{2},\\cdots,a_{n}$:\r\n\r\n$ \\left(\\sum^{n}_{i=1}a^{2}_{i}\\right)\\left(\\sum^{n}_{i=1}a_{i}\\right)-\\left(\\sum^{n}_{i=1}a^{3}_{i}\\right)\\geq6\\;a_{1}a_{2}\\cdots a_{n}$", "Solution_1": "This problem purposed in Polish Mathematical Contests And: n=3.", "Solution_2": "[quote=\"navid\"]This problem purposed in Polish Mathematical Contests And: n=3.[/quote]\r\n\r\nwell nice Navid, would you please send you solution too! :wink:", "Solution_3": "If $ n>3$, then since the degree of right side is larger than 3 and both right side and left side is positive, for big positive real, the inequality flips. In the similar way, if $ n<3$, contradiction. So, the remain case is $ n=3$, and the inequality changes to $ \\sum_{sym}a_{1}^{2}a_{2}\\geq 6a_{1}a_{2}a_{3}$, which is true by muirhead." } { "Tag": [ "function", "algebra proposed", "algebra" ], "Problem": "Find a closed-form formula for the following sequence: $ a_n\\equal{}\\frac{2}{n}(a_1\\plus{}\\cdots\\plus{}a_{n\\minus{}1})\\plus{}kn$ (in terms of $ k$ and $ a_1$).", "Solution_1": "Obviosly $ na_n \\minus{}(n\\plus{}1)a_{n\\minus{}1}\\equal{}k(2n\\minus{}1),n>1$.\r\nLet $ f(x)\\equal{}\\sum_n a_nx^n$. Then we get $ f'(x)(1\\minus{}x)\\minus{}2f(x)\\equal{}a_1\\plus{}\\frac{1\\plus{}x}{(1\\minus{}x)^2}.$\r\nIt give $ f(x)\\equal{}\\frac{a_1(x\\minus{}x^2/2)\\minus{}kx\\minus{}2ln|1\\minus{}x|}{(1\\minus{}x)^2}$. It give closed form for $ a_n\\equal{}\\frac{f^{(n)}(0)}{n!}$.", "Solution_2": "That's not really a closed-form answer; otherwise any generating function would work as a closed-form answer\r\nfor a sequence. (Although the g.f. is often the best you can do...)", "Solution_3": "I think Rust's point is that this particular generating function has a Taylor series which is relatively straightforward to compute.", "Solution_4": "[quote=\"bambaman\"]Find a closed-form formula for the following sequence: $ a_n \\equal{} \\frac {2}{n}(a_1 \\plus{} \\cdots \\plus{} a_{n \\minus{} 1}) \\plus{} kn$ (in terms of $ k$ and $ a_1$).[/quote]\r\n\r\nI dont think there is a closed form for this sequence :\r\n\r\n$ \\forall n > 1$ : $ na_n \\equal{} 2\\sum_{i \\equal{} 1}^{n \\minus{} 1}a_i \\plus{} kn^2$\r\n$ \\forall n > 2$ : $ (n \\minus{} 1)a_{n \\minus{} 1} \\equal{} 2\\sum_{i \\equal{} 1}^{n \\minus{} 2}a_i \\plus{} k(n \\minus{} 1)$ and so $ (n \\plus{} 1)a_{n \\minus{} 1} \\equal{} 2\\sum_{i \\equal{} 1}^{n \\minus{} 1}a_i \\plus{} k(n \\minus{} 1)^2$\r\n\r\nSo, $ \\forall n > 2$ : $ na_n \\minus{} (n \\plus{} 1)a_{n \\minus{} 1} \\equal{} k(2n \\minus{} 1)$\r\n\r\nSo, $ \\forall n > 2$ : $ \\frac {a_n}{n \\plus{} 1} \\minus{} \\frac {a_{n \\minus{} 1}}{n} \\equal{} k\\frac {2n \\minus{} 1}{n(n \\plus{} 1)}$ $ \\equal{} k(\\frac {3}{n \\plus{} 1} \\minus{} \\frac 1n)$\r\n\r\nAdding all these equalities (for index greater than 2), we get :\r\n\r\n$ \\forall n\\geq 3$ : $ \\frac {a_n}{n \\plus{} 1} \\minus{} \\frac {a_{2}}{3}$ $ \\equal{} k(\\sum_{i \\equal{} 3}^n\\frac {3}{i \\plus{} 1} \\minus{} \\sum_{i \\equal{} 3}^n\\frac 1i)$\r\n\r\n$ \\forall n\\geq 3$ : $ \\frac {a_n}{n \\plus{} 1} \\equal{} \\frac {a_{2}}{3} \\plus{} k(\\sum_{i \\equal{} 3}^n\\frac {3}{i \\plus{} 1} \\minus{} \\sum_{i \\equal{} 3}^n\\frac 1i)$\r\n\r\nAnd so, since $ a_2 \\equal{} a_1 \\plus{} 2k$ and simplifying the sums :\r\n\r\n$ \\forall n\\geq 3$ : $ \\boxed{a_n \\equal{} (a_1 \\minus{} 10k)\\frac {n \\plus{} 1}{3} \\plus{} 3k \\plus{} 2k(n \\plus{} 1)\\sum_{i \\equal{} 1}^n\\frac 1i}$\r\n\r\nAnd finding a closed form for this expression would mean finding a closed form for $ \\sum_{i \\equal{} 1}^n\\frac 1i$ ...", "Solution_5": "Hello Bambaman !\r\n\r\nSince you posted in the \"proposed and own problems\", you have the solution to this problem.\r\n\r\nCould you kindly give us this solution, or the hint to find the closed form since it seems most of us think there is no such closed form.\r\nOr at least tell us where we are wrong.\r\n\r\nThanks a lot.", "Solution_6": "Actually, I considered $ H_n$ as a legitimate element in a closed-form formula. My solution was along the lines of Rust's solution (he has some calculation mistakes, though). I was delighted to see your elegant solution!", "Solution_7": "OKi :)\r\nThanks for your answer" } { "Tag": [ "trigonometry" ], "Problem": "A jet fighter takes off at an angle of 27.0 degrees w/ the horizontal, accelerating at 2.62 m/s*s. The plane weighs 79,300 N. I need to find the thrust T of the engine on the plane and the lift force L exerted by the air perpendicular to the wings. \r\n\r\nShould i switch the axis around, and then the Force of the thrust = (2.62cos27-gsin27)m and L equal to (2.62sin27+gcos27)m?", "Solution_1": "I'm not sure how you came to those numbers, could you explain? According to my calculations, it should be\r\n\r\n[hide]\n$T=ma+mg\\sin\\phi$, $L=mg\\cos\\phi$[/hide]" } { "Tag": [ "modular arithmetic", "quadratics", "Vieta", "number theory", "relatively prime", "algebra" ], "Problem": "Find all pairs of positive integers $ a,b$ such that \\begin{align*} b^2 + b+ 1 & \\equiv 0 \\pmod a \\\\ a^2+a+1 &\\equiv 0 \\pmod b . \\end{align*}", "Solution_1": "First I will show that $ a\\ne b\\plus{}1$. If instead $ a\\equal{}b\\plus{}1$, then we would have\r\n\r\n$ b\\plus{}1\\equal{}a|b^2\\plus{}b\\plus{}1$\r\n$ b\\plus{}1|b(b\\plus{}1)\\equal{}b^2\\plus{}b$\r\n$ b\\plus{}1|(b^2\\plus{}b\\plus{}1)\\minus{}(b^2\\plus{}b)\\equal{}1$\r\n\r\nwhich is impossible since $ b\\ge 1$ and hence $ b\\plus{}1\\ge 2$.\r\n\r\nNow, I will show the problem is equivalent to $ a^2 \\plus{} a \\plus{} b^2 \\plus{} b \\plus{} 1\\equiv 0\\mod ab$.\r\n\r\nWe know that $ a$ and $ b$ must be relatively prime. Otherwise, $ \\gcd(a,b)|a^2 \\plus{} a$ so $ \\gcd(a,b)\\not| a^2 \\plus{} a \\plus{} 1$ if $ \\gcd(a,b)>1$. Now $ a^2 \\plus{} a \\plus{} b^2 \\plus{} b \\plus{} 1\\equiv 0\\mod a$ and $ a^2 \\plus{} a \\plus{} b^2 \\plus{} b \\plus{} 1\\equiv 0\\mod b$, and combining these, since $ a$ and $ b$ are relatively prime, gives $ a^2 \\plus{} b^2 \\plus{} a \\plus{} b \\plus{} 1\\equiv 0\\mod ab$.\r\n\r\nThis goes the other way to. If $ a^2 \\plus{} b^2 \\plus{} a \\plus{} b \\plus{} 1\\equiv 0\\mod ab$ then $ a^2\\plus{}a\\plus{}b^2\\plus{}b\\plus{}1\\equiv 0\\mod a\\implies b^2\\plus{}b\\plus{}1\\equiv 0\\mod a$ and $ a^2\\plus{}a\\plus{}b^2\\plus{}b\\plus{}1\\equiv 0\\mod b\\implies a^2 \\plus{} a \\plus{} 1\\equiv 0\\mod a$.\r\n\r\nSo now we just need to solve $ a^2\\plus{}a\\plus{}b^2\\plus{}b\\plus{}1\\equiv0\\mod ab$. Let\r\n\r\n$ a^2 \\plus{} a \\plus{} b^2 \\plus{} b \\plus{} 1 \\equal{} kab$\r\n\r\nfor some integer $ k$. Also, assume that $ a>b$ and that $ a,b>1$.\r\n\r\nThen\r\n\r\n$ a^2 \\minus{} (kb\\minus{}1)a \\plus{} b^2 \\plus{} b \\plus{} 1 \\equal{} 0$\r\n\r\nThis is a quadratic equation, so if $ a$ is a solution, then by Vieta's, $ a'\\equal{}kb\\minus{}1\\minus{}a\\equal{}\\frac{b^2\\plus{}b\\plus{}1}{a}$ is also a solution. Since $ b^2\\plus{}b\\plus{}1$ and $ a$ are postive, $ a'$ must be positive. Now (by assumption that $ a>b$),\r\n\r\n$ a'\\equal{}\\frac{b^2\\plus{}b\\plus{}1}{a}<\\frac{b^2\\plus{}b\\plus{}1}{b}\\equal{}b\\plus{}1\\plus{}\\frac{1}{b}$\r\n$ a'\\le b\\plus{}1$.\r\n\r\n(because $ b>1$ by assumption, we know that $ 0<\\frac{1}{b}<1$, but $ a'$ is an integer). Also, $ a'\\ne b\\plus{}1$ by above, so $ a'\\le b$. So $ (a',b)$ is another solution if we already had $ (a,b)$. Since $ a>b$ and $ a'\\le b$, our $ (a',b)$ solution is smaller. So, we know that we can always produce a smaller solution as long as $ a,b>1$ and $ a\\ne b$. Now let's look at the cases where either $ a\\equal{}b$ or one of $ a,b$ is $ 1$.\r\n\r\n$ a\\equal{}b$. We know that $ a$ and $ b$ are relatively prime so for $ a$ to equal $ b$ we must have $ a\\equal{}b\\equal{}1$.\r\n\r\n$ a\\equal{}1$ or $ b\\equal{}1$. If $ a\\equal{}1$ then $ b$ divides $ 1^2\\plus{}1\\plus{}1\\equal{}3$, so the only pairs with $ 1$ in them are $ (1,3)$ and $ (1,1)$.\r\n\r\nSo we have $ (1,1)$ and $ (1,3)$. Note than $ (1,3)$ can be reduced, as $ 5ab\\equal{}1^2\\plus{}1\\plus{}3^2\\plus{}3\\plus{}1$, so $ k\\equal{}5$ and $ a'\\equal{}kb\\minus{}1\\minus{}a\\equal{}5(1)\\minus{}1\\minus{}3\\equal{}1$, so $ (1,3)$ is reduced to $ (1,1)$. Thus, all pairs besides $ (1,1)$ can be reduced in this manner.\r\n\r\nSince this reduction process does not change $ k$, and $ k\\equal{}5$ for $ (1,1)$, we have $ k\\equal{}5$ for all solutions. So since all solutions can be reduced to $ (1,1)$ by $ (a,b)\\to (5b\\minus{}a\\minus{}1,b), (a>b)$ any solution can be obtained by the reverse procedure $ (a,b)\\to (5b\\minus{}a\\minus{}1,b), (a\\le b)$. Therefore if we define the sequence $ a_0\\equal{}1,a_1\\equal{}1$, and $ a_{n\\plus{}2}\\equal{}5a_{n\\plus{}1}\\minus{}a_{n}\\minus{}1$, all solutions $ (a,b)$ are consecutive terms of this sequence." } { "Tag": [ "ratio", "puzzles" ], "Problem": "Can you generate Golden Ratio $\\phi$ using $4$ number of $4's$ .\r\n\r\nUse any mathematical operator you can, but use exactly $4$ number of $4's$.\r\n\r\ncan you?", "Solution_1": "[hide]\\[ 4^{-1/2} + \\sqrt{(4/4 + 4^-1)} \\][/hide]", "Solution_2": "[quote=\"agolsme\"][hide]\\[ 4^{-1/2} + \\sqrt{(4/4 + 4^-1)} \\][/hide][/quote]\n\n[hide]your solution uses $1$ and $2$. Remember only $4$ number of $4$'s but any number of operators are allowed.\n\n$\\sqrt{4}$ can also be written as $4^{1/2}$, so this is OK.... but others are not valid.\n\n?[/hide]", "Solution_3": "[hide]$\\frac{\\sqrt{4}+\\sqrt{4!-4}}{4}$[/hide]" } { "Tag": [ "function", "real analysis", "real analysis unsolved" ], "Problem": "Hello,\r\nI want this problem again suggest.Maybe we found a good solution. The problem is:\r\n\r\nFind all $ f: \\mathbb{R} \\to \\mathbb{R}$ continuous functin with $ F(f(x))\\plus{}f(F(x))\\equal{}x^2 \\forall x \\in \\mathbb{R}$, where $ F(x)\\equal{}f'(x), F(0)\\equal{}0$", "Solution_1": "Perhaps you meant $ f(x) \\equal{} F'(x)$, $ F(0) \\equal{} 0$?", "Solution_2": "Yes! Sorry :blush:", "Solution_3": "So the problem is\r\nFind all $ f: \\mathbb{R}\\to \\mathbb{R}$ continuous finction such that $ F(f(x))\\plus{}f(F(x))\\equal{}x^2$ $ (\\forall) x\\in \\mathbb{R}$, where $ F'(x)\\equal{}f(x)$, and $ F(0)\\equal{}0$" } { "Tag": [], "Problem": "$ \\binom{3n}{0}\\plus{}\\binom{3n}{3}\\plus{}\\binom{3n}{6}\\plus{}...\\plus{}\\binom{3n}{3n}\\equal{}?$", "Solution_1": "It's been posted many times. See the discussion [url=http://www.artofproblemsolving.com/Forum/weblog_entry.php?t=220351]here[/url] and [url=http://www.artofproblemsolving.com/Forum/weblog_entry.php?t=212490]here[/url]." } { "Tag": [ "induction" ], "Problem": "Show that every power of (\u221a2 - 1) can be written in the form \u221a(k+1) - \u221ak.", "Solution_1": "Induction? \r\n\r\n Let $\\large{(\\sqrt{2}-1)^n\\,=\\,\\sqrt{k+1}-\\sqrt{k}\\,\\Longrightarrow{\\,(\\sqrt{2}-1)^{n+1}\\,=\\,(\\sqrt{2}-1)(%Error. \"sqty\" is a bad command.\n{k+1}-\\sqrt{k})\\,=\\,\\sqrt{2(k+1)}+\\sqrt{2k}-\\sqrt{(k+1)}-\\sqrt{k}\\,=\\,X}}$\r\n\r\n See that $\\large{\\sqrt{a}+-\\sqrt{b}\\,=\\,\\sqrt{a+b+-2\\sqrt{ab}}}$ , then $\\large{X\\,=\\,\\sqrt{2(k+1)+k-2\\sqrt{2k(k+1)}}\\,-\\,\\sqrt{2k+(k+1)-2\\sqrt{2k(k+1)}\\,}}$\r\n\r\n $=\\,\\sqrt{[3k-2\\sqrt{2k(k+1)}]+1}\\,-\\,\\sqrt{(3k-2\\sqrt{2k(k+1)}}\\,=\\,\\sqrt{k_1+1}-\\sqrt{k_1}$\r\n\r\n Proved", "Solution_2": "[hide]\nLet $s_n$ be the $n$th power of $\\sqrt{2}-1$. We will proceed by induction. For the base case, $\\sqrt{2}-1 = \\sqrt{2}-\\sqrt{1}$ so that works. Suppose\n\n$s_n = a+b\\sqrt{2} = \\sqrt{k+1}-\\sqrt{k}$.\n\nThen $|a^2-2b^2| = 1$. So\n\n$s_{n+1} = (\\sqrt{2}-1)(a+b\\sqrt{2}) = (2b-a)+\\sqrt{2}(a-b)$.\n\nBut since $|(2b-a)^2-2(a-b)^2| = |2b^2-a^2| = |a^2-2b^2| = 1$, we know\n\n$s_{n+1} = (2b-a)+\\sqrt{2}(a-b)$ can be written as $\\sqrt{k+1}-\\sqrt{k}$ for some $k$ as well, completing the induction.[/hide]\r\n\r\nEDIT: Boo, someone beat me to it again." } { "Tag": [ "function", "calculus", "integration", "complex analysis", "complex analysis unsolved" ], "Problem": "Suppose that a function $ f$ is analytic throughout the finite plane except for a finite number of singular points $ z_1, z_2, \\ldots, z_n$. Show that\r\n\r\n$ \\text{Res}_{z\\equal{}z_1}f(z)\\plus{}\\text{Res}_{z\\equal{}z_2}f(z)\\plus{} \\cdots \\plus{} \\text{Res}_{z\\equal{}z_n}f(z)\\plus{}\\text{Res}_{z\\equal{}\\infty}f(z)\\equal{}0$.\r\n\r\nI don't see how to show this right now. Any hints on where to go would be nice. Thank you.", "Solution_1": "It's practically the definition of the residue at $ \\infty$. Consider the integral of $ f$ around a very large circle, which encloses all singularities.", "Solution_2": "This is not true. A counterexample is 1/z.\r\n\r\nPym.", "Solution_3": "Sorry, I did not notice that the residue at infinity is also included.\r\nThe reply of jmerry is of course correct.\r\n\r\nPym." } { "Tag": [ "rate problems" ], "Problem": "let v(t) be the amount of water in a tank at time t. If the average rate of change of V between times t = 3 and t = 6 is 3, and the average rate between t = 6 and t = 9 is 4, what is the average rate of change of V between t =3 and t = 9?", "Solution_1": "Average rate = :Sigma: change / :Sigma: time", "Solution_2": "[quote=\"blahblahblah\"]Average rate = :Sigma: change / :Sigma: time[/quote]\r\n\r\nAre you allowed to do that, considering we are working with averages?", "Solution_3": "[quote=\"mikewd\"]Are you allowed to do that, considering we are working with averages?[/quote]\r\n\r\nYou know the average, though, and how many numbers you averaged, so you can multiply to get the sums and add the sums together, then divide by the time." } { "Tag": [ "vector", "geometry unsolved", "geometry" ], "Problem": "The vertex $P$ of a rectangular block with edges $a,b,c$ is selected. Consider the plane passing through the vertices adjacent to $P$. Denote the distance of the plane from $P$ is $m$. Prove that \\[\\frac{1}{m^{2}}=\\frac{1}{a^{2}}+\\frac{1}{b^{2}}+\\frac{1}{c^{2}}\\]", "Solution_1": "Put origin at P and x, y, z-axes along the edges a = PA, b = PB, c = PC. Equation of the plane (ABC) is $\\frac{x}{a}+\\frac{y}{b}+\\frac{z}{c}=1.$ Vector product of any 2 vectors parallel to (ABC) is perpendicular to (ABC).\r\n\r\n$\\vec{AB}\\times \\vec{AC}= (-a, b, 0) \\times (-a, 0, c) = (bc, ca, ab) = abc \\left(\\frac{1}{a},\\frac{1}{b},\\frac{1}{c}\\right)$\r\n\r\n$\\vec{PN}= k \\left(\\frac{1}{a},\\frac{1}{b},\\frac{1}{c}\\right) \\perp (ABC)$ for any $k \\neq 0$. If $N \\in (ABC),$ then $k \\left(\\frac{1}{a^{2}}+\\frac{1}{b^{2}}+\\frac{1}{c^{2}}\\right) =1,$ yielding k, and $m^{2}= \\vec{PN}\\cdot \\vec{PN}= k^{2}\\left(\\frac{1}{a^{2}}+\\frac{1}{b^{2}}+\\frac{1}{c^{2}}\\right) = k,$ $\\frac{1}{m^{2}}= \\frac{1}{k}= \\frac{1}{a^{2}}+\\frac{1}{b^{2}}+\\frac{1}{c^{2}}.$", "Solution_2": "Thank for your help." } { "Tag": [ "inequalities proposed", "inequalities" ], "Problem": "$n$ distinct points are on a plane.\r\n\r\nProve that the number of pairs of points \r\n\r\nwhich have a unit distant is less than $2n\\sqrt{n}$.", "Solution_1": "Using cauchy will help you.", "Solution_2": "can the number of pairs of points which have a unit distance can be more than $2n-3$ (for $n\\geq 3$)?" } { "Tag": [], "Problem": "Question attached.\r\n\r\nI know that it involves simple stoichiometry, but this is the first time I am applying my knowledge of stoichiometry on redox reactions (and the sort), and it is overly complicated for me to comprehend.\r\n\r\nPlease provide an outline of what to do, and why each step is done, so I may solve similar problems in the future.\r\n\r\nThanks", "Solution_1": "The problem gives you an equation as balanced as you need it, and asks you to do the stochiometry calculations. The fact that there is oxidation-reduction does not matter once you have a balanced equation. They are not asking for grams and making you use molecular weights. There is no change of units, just moles, liters and mililiters. It is an easy problem if you can interpret the meaning.\r\n\r\nThe problem is in the wording. I'll try to translate.\r\nIt seems like they used the same 25.00 mL sampling through the whole procedure.\r\nThe first titration was to determine the moles of $ Fe^{+2}$ ion.\r\nThe second titration was to determine the total moles of $ Fe$ (found all as $ Fe^{+2}$ after the zinc treatment).\r\nYou are only concerned with the titrations. How al the iron was reduced to $ Fe^{+2}$ does not matter. What matters is that it was all reduced to $ Fe^{+2}$ before the second titration.\r\nIn the titration each mole of permanganate oxidizes 5 moles of $ Fe^{+2}$ ion. (Each milimole of permanganate oxidizes 5 milimoles of $ Fe^{+2}$ ion).\r\nYour permanganate titrant contains 0.0200 moles/L (0.0200 mmoles/mL).\r\nHow many milimoles of permanganate titrant were used in the first titration?\r\n(0.0200 mmoles/mL) X (23.0 mL) = 0.46 mmoles\r\nHow many milimoles of $ Fe^{+2}$ ion were oxidized in the first titration?\r\n(5 mmoles $ Fe^{+2}$/1 mmol permanganate) X (0.46 mmoles permanganate) = 2.3 mmoles $ Fe^{+2}$\r\nWhat was the $ Fe^{+2}$ ion concentration if those milimoles were in a 25.0 mL volume?\r\n(2.3 mmoles ) / (25.0 mL) = 0.092 mmol/mL = 0.092 M = 92 mM\r\nThe second titration would be calculated the same way to give total $ Fe$.You can find the ${ Fe^[+3}$ concentration by difference.\r\n\r\nThere's no magic and no need to memorize formulas or multistep procedures. You only need to understand the problem. It's just a \"word problem.\" It helped to have been an analytical chemist back in the early seventies when all we could do was titrations, though." } { "Tag": [ "inequalities", "inequalities proposed" ], "Problem": "Let $ a,b,c>0$ and $ a^2\\plus{}b^2\\plus{}c^2\\equal{}1$ prove that\r\n\\[ a\\plus{}b\\plus{}c\\plus{}\\frac{1}{abc}\\ge \\frac{4\\sqrt{3}}{9}(a\\plus{}b\\plus{}c)(\\frac{1}{a}\\plus{}\\frac{1}{b}\\plus{}\\frac{1}{c})\\]", "Solution_1": "[quote=\"M.A\"]Let $ a,b,c > 0$ and $ a^2 \\plus{} b^2 \\plus{} c^2 \\equal{} 1$ prove that\n\\[ a \\plus{} b \\plus{} c \\plus{} \\frac {1}{abc}\\ge \\frac {4\\sqrt {3}}{9}(a \\plus{} b \\plus{} c)(\\frac {1}{a} \\plus{} \\frac {1}{b} \\plus{} \\frac {1}{c})\n\\]\n[/quote]\r\nLet $ a\\plus{}b\\plus{}c\\equal{}3u,$ $ ab\\plus{}ac\\plus{}bc\\equal{}3v^2$ and $ abc\\equal{}w^3.$ Then your inequality is equivalent to\r\n$ uw^3\\plus{}3(3u^2\\minus{}2v^2)^2\\geq4uv^2\\sqrt{3u^2\\minus{}2v^2}.$\r\nBut by Schur we obtain: $ 2uw^3\\geq15u^2v^2\\minus{}9u^4\\minus{}4v^4.$\r\nThus, it remains to prove that $ \\frac{1}{2}(15u^2v^2\\minus{}9u^4\\minus{}4v^4)\\plus{}3(3u^2\\minus{}2v^2)^2\\geq4uv^2\\sqrt{3u^2\\minus{}2v^2}.$\r\nLet $ u^2\\equal{}tv^2.$ Then $ t\\geq1.$\r\nHence, $ \\frac{1}{2}(15u^2v^2\\minus{}9u^4\\minus{}4v^4)\\plus{}3(3u^2\\minus{}2v^2)^2\\geq4uv^2\\sqrt{3u^2\\minus{}2v^2}\\Leftrightarrow$\r\n$ \\Leftrightarrow45t^2\\minus{}57t\\plus{}20\\geq8\\sqrt{t(3t\\minus{}2)}\\Leftrightarrow(t\\minus{}1)^2(2025t^2\\minus{}1080t\\plus{}672)\\plus{}272(t\\minus{}1)\\geq0.$\r\nDone!", "Solution_2": "[quote=\"M.A\"]Let $ a,b,c > 0$ and $ a^2 \\plus{} b^2 \\plus{} c^2 \\equal{} 1$ prove that\n\\[ a \\plus{} b \\plus{} c \\plus{} \\frac {1}{abc}\\ge \\frac {4\\sqrt {3}}{9}(a \\plus{} b \\plus{} c)(\\frac {1}{a} \\plus{} \\frac {1}{b} \\plus{} \\frac {1}{c})\n\\]\n[/quote]\r\n\r\nI kill problem by mixing variable \r\n$ a \\plus{} b \\plus{} c \\plus{} \\frac {1}{abc}\\ge \\frac {4\\sqrt {3}}{9}(a \\plus{} b \\plus{} c)(\\frac {1}{a} \\plus{} \\frac {1}{b} \\plus{} \\frac {1}{c})$\r\n\r\n$ \\leftrightarrow a \\plus{} b \\plus{} c \\plus{} \\frac {1}{abc} \\minus{} \\frac {4\\sqrt {3}}{9}(\\frac {b \\plus{} c}{a} \\plus{} \\frac {a \\plus{} c}{b} \\plus{} \\frac {a \\plus{} b}{c})\\ge \\frac {4\\sqrt {3}}{3}$\r\n\r\nsetting $ f(a,b,c) \\equal{} a \\plus{} b \\plus{} c \\plus{} \\frac {1}{abc} \\minus{} \\frac {4\\sqrt {3}}{9}(\\frac {b \\plus{} c}{a} \\plus{} \\frac {a \\plus{} c}{b} \\plus{} \\frac {a \\plus{} b}{c})$\r\n\r\n$ \\rightarrow f(a,b,c) \\minus{} f(a,\\sqrt {\\frac {b^2 \\plus{} c^2}{2}},\\sqrt {\\frac {b^2 \\plus{} c^2}{2}})\\ge (b \\minus{} c)^2[\\frac {1}{abc(b^2 \\plus{} c^2)} \\plus{} \\frac {4\\sqrt {3}}{9a(b \\plus{} c \\plus{} \\sqrt {2(b^2 \\plus{} c^2)})} \\minus{} \\frac {1}{b \\plus{} c \\plus{} \\sqrt {2(b^2 \\plus{} c^2)}} \\minus{} \\frac {4\\sqrt {6(b^2 \\plus{} c^2)} \\plus{} 4\\sqrt {3}a}{9bc\\sqrt {2(b^2 \\plus{} c^2)}}]$\r\nwe needs prove that:\r\n\r\n$ \\frac {1}{abc(b^2 \\plus{} c^2)} \\plus{} \\frac {4\\sqrt {3}}{9a(b \\plus{} c \\plus{} \\sqrt {2(b^2 \\plus{} c^2)})} \\minus{} \\frac {1}{b \\plus{} c \\plus{} \\sqrt {2(b^2 \\plus{} c^2)}} \\minus{} \\frac {4\\sqrt {6(b^2 \\plus{} c^2)} \\plus{} 4\\sqrt {3}a}{9bc\\sqrt {2(b^2 \\plus{} c^2)}}]\\ge 0$\r\n\r\n$ A \\equal{} \\frac {4\\sqrt {3}}{9a(b \\plus{} c \\plus{} \\sqrt {2(b^2 \\plus{} c^2)})} \\minus{} \\frac {1}{b \\plus{} c \\plus{} \\sqrt {2(b^2 \\plus{} c^2)}} \\equal{} \\frac {4\\sqrt {3} \\minus{} 9a}{9a(b \\plus{} c \\plus{} \\sqrt {2(b^2 \\plus{} c^2)})}$\r\nassume that $ a \\equal{} min{a,b,c}$ $ \\rightarrow a\\le \\frac {1}{\\sqrt {3}}$ $ \\rightarrow 9a\\le 3\\sqrt {3} < 4\\sqrt {3}$ $ \\rightarrow A\\ge 0$\r\nthen, we needs proof\r\n\r\n$ B \\equal{} \\frac {1}{abc(b^2 \\plus{} c^2)} \\minus{} \\frac {4\\sqrt {6(b^2 \\plus{} c^2)} \\plus{} 4\\sqrt {3}a}{9bc\\sqrt {2(b^2 \\plus{} c^2)}}\\ge 0$\r\n\r\n$ \\leftrightarrow 4\\sqrt {6}a(b^2 \\plus{} c^2) \\plus{} 4\\sqrt {3}a^2\\sqrt {b^2 \\plus{} c^2} \\le 9\\sqrt {2}$\r\n\r\n$ 4\\sqrt {6}a(b^2 \\plus{} c^2) \\equal{} 4\\sqrt {6}a(1 \\minus{} a^2)\\ge 4\\sqrt {2}$ and \r\n\r\n$ 4\\sqrt {3}a^2\\sqrt {b^2 \\plus{} c^2} \\equal{} 4\\sqrt {3}a^2\\sqrt {1 \\minus{} a^2}\\le 4\\sqrt {3}a\\frac {1}{2}\\le 2$ (am-gm)\r\n\r\n$ \\rightarrow 4\\sqrt {6}a(b^2 \\plus{} c^2) \\plus{} 4\\sqrt {3}a^2\\sqrt {b^2 \\plus{} c^2} \\le 2 \\plus{} 4\\sqrt {2} < 9\\sqrt {2}$\r\n$ \\rightarrow B\\ge 0$\r\n\r\n$ \\rightarrow f(a,b,c)\\ge f(a,\\sqrt {\\frac {b^2 \\plus{} c^2}{2}},\\sqrt {\\frac {b^2 \\plus{} c^2}{2}})$\r\n\r\nwe needs prove that $ f(a,b,c)\\ge \\frac {4\\sqrt {3}}{3}$ with $ a \\equal{} \\sqrt {1 \\minus{} 2t^2}$ and $ b \\equal{} c \\equal{} t (t \\equal{} \\sqrt {\\frac {b^2 \\plus{} c^2}{2}})$\r\nDone!", "Solution_3": "there is a really simple solution to this one.\r\nhint:\r\nit also holds $ a\\plus{}b\\plus{}c\\plus{}\\frac{1}{abc}\\geq \\frac43 (\\frac1a\\plus{}\\frac1b\\plus{}\\frac1c)$.", "Solution_4": "[quote=\"Albanian Eagle\"]there is a really simple solution to this one.\nhint:\nit also holds $ a \\plus{} b \\plus{} c \\plus{} \\frac {1}{abc}\\geq \\frac43 (\\frac1a \\plus{} \\frac1b \\plus{} \\frac1c)$.[/quote]\r\nYou are right Albanian Eagle this is an easy problem :lol: \r\n\\[ a \\plus{} b \\plus{} c \\plus{} \\frac {1}{abc}\\geq \\frac43 (\\frac1a \\plus{} \\frac1b \\plus{} \\frac1c)\\Leftrightarrow\\]\r\n\\[ (a\\minus{}b)^4\\plus{}(b\\minus{}c)^4\\plus{}(c\\minus{}a)^4\\plus{}(a^4\\plus{}b^4\\plus{}c^4\\minus{}a^2bc\\plus{}b^2ca\\plus{}c^2ab)\\ge 0\\]\r\nso clear." } { "Tag": [ "probability", "expected value", "probability and stats" ], "Problem": "Hi all again, here with another problem that I get:\r\nbuscar\r\n\r\nA medical office patients arrive at a constant rate of $ 1/10$ \t\r\npatients per minute, following a Poisson process. If the doctor does not begin to patients unless they meet in the waiting room has three such. What is the expected time for the doctor see the first patient? What is the probability that the doctor did not see \t\r\nany patient during the first hour?\r\n\r\n\t\r\nI've done this, point b): $ Pr[X<\\equal{}2] \\equal{} Pr(X\\equal{}0) \\plus{} Pr(X\\equal{} 1) \\plus{} Pr(X\\equal{}2)$ calculated with the Poisson distribution.\r\n\r\nThe a) does not come out yet, I appreciate any help.\r\n\r\n \t\r\nGreetings and many thanks :)", "Solution_1": "Just answer a simple question: \r\n\r\nFor Poisson process with intensity $ \\lambda$, what is the distribution of the times between arrivals?", "Solution_2": "hello, I think I did well like this:\r\n\r\nThe question what is the expected time for the doctor to see the first patient?\r\nMay arise as follows: what is the expected time until they reach three patients?,\r\n\r\n\t\r\nSince the arrival of patients is a Poisson process of rate \u03bb = 1 / 10 patients per minute the expected time till the third patient $ E(S_3) \\equal{} n/\\lambda \\equal{} 3/1/10 \\equal{} 30min$. $ S_n$ follows a Gamma distribution.", "Solution_3": "You're right, but you put too much mathematics into this problem. \r\n\r\nYes, $ S_3$ is Gamma distributed. \r\nBut to solve this problem, one needs not to know what is Gamma distribution. Times between arrivals are exponentially distributed, expected value of sum = sum of expected values = 3*1/$ \\lambda$ =30.", "Solution_4": "Ok. Thank you for your help.\r\n\r\nYou know anything about this?: \r\nhttp://www.artofproblemsolving.com/Forum/viewtopic.php?t=298329\r\n\r\n\t\r\nthank you very much again and greetings", "Solution_5": "[quote=\"extranger\"]\nYou know anything about this?: \nhttp://www.artofproblemsolving.com/Forum/viewtopic.php?t=298329\n[/quote]\r\nI'm not sure I know. I will be able to read your text only if you make the formulae readable.", "Solution_6": "Ok. I appreciate if suddenly I can help, especially with code R." } { "Tag": [], "Problem": "Anyone here ever play the game Rummy-Q, I think there is a lot of strategy involved there.", "Solution_1": "Is Rummy-Q the card game where you have like 13 cards and you have to try get them in order or something?", "Solution_2": "Something like that, but they are not cards, yeah u need like a combination of at least 3 things depending on color or order.", "Solution_3": "[quote=\"hello\"]Anyone here ever play the game Rummy-Q, I think there is a lot of strategy involved there.[/quote]\r\nAre you talking about Rummikub or is it something else?\r\n\r\nEDIT: I guess we're talking about the same thing :)", "Solution_4": "ive played Rummy-Q, or something very like it. Your description sounds a lot like what i played." } { "Tag": [ "geometry", "geometry proposed" ], "Problem": "Prove that in any convex polygon with $4n+2$ sides ($n\\geq 1$) there exist two consecutive sides which form a triangle of area at most $\\frac 1{6n}$ of the area of the polygon.", "Solution_1": "Hmm... I can't prove it for $n=1$ :blush: (Kuba said me that it is the hardest thing in this problem :P )\r\nAll over cases can be derived from $n=1$.\r\nLet's show it.\r\nSuppose the contrary, i.e. there are no triangle of area $\\leq 1/6n$.\r\nConsider $4n+2$-gon with $n\\geq 2$ and cut a 6-gon from it by mean of a diagonal ($AB$ on figure). Let's prove that the triangle of area $<1/6n$ can't adjoin the diagonal $AB$. Otherwise if $ABC$ is a such triangle then draw a line throw $A$ parallel to $BC$. We will necessary have $D$ and $E$ lies in the other halfplane than $BC$. So $4n+2$ is not convex.Therefore we can divide $4n+2$-gon into $n$ 6-gons by mean of diagonals and no triangle which adjoins these diagonals will have area $<1/6n$.", "Solution_2": "[quote=\"Myth\"] Hmm... I can't prove it for n=1 (Kuba said me that it is the hardest thing in this problem :P ) [/quote] In my humble opinion, it is not harder than the part done by Myth and it uses pretty much the same idea. Here is a little [hide=\"hint\"] Assume for a moment that the diagonals AD, BE and CF of a hexagon ABCDEF intersect at one point O. Then one of the six triangles into which they divide the hexagon has the area at most $1/6$ of the area of the hexagon. If it is, say, the triangle AOB, what can you say about the areas of ACB and AFB? [/hide] Nice problem, by the way! :)", "Solution_3": "Nice idea, Myth! I can prove the n=1 case:\r\n\r\nIn hexagon ABCDEF, $P=AD\\cap CF$, $Q=CF\\cap EB$, $R=EB\\cap AD$. The six triangles APB, BPC, CQD, DQE, ERF, FRA cover less than the whole hexagon (if the three diagonals are not concurrent) or the whole triangle (if they are). In either case, one of those six triangles will have area $\\le \\frac{1}{6}(\\text{area})$, so say it is APB. Then one of AFB and ACB has area at most the area of APB, so we're done.", "Solution_4": "Sorry, I don't see it. \r\n\r\nIndeed, the case described by Fedor is clear. But common case... I am missing something.", "Solution_5": "I really don't understand what Myth did, especially the part:\r\n[quote=\"Myth\"]We will necessary have $D$ and $E$ lies in the other halfplane than $BC$.[/quote]\r\nAnd also, how exactly is the case $n>1$ derived from $n=1$?", "Solution_6": "If $E$ lies in the same halfplane, then $S_{EBC}\\leq S_{ABC}\\leq \\frac{1}{6n}$, so we find a small requared triangle $EBC$. Contradiction.\r\n\r\nThe general cases can be done in the following way: cut the polygon into $n$ hexagons by mean of diagonals. One of this hexagon have area $\\leq 1/n$ and we can cut a tringle of area $\\leq 1/6$ from it, therefore this triangle is at most $\\frac{1}{6n}$ of the area of the initial polygon. And we prooved that this triangle can't be \"inner\" triangle, so it is a required triangle.", "Solution_7": "okay. now i understand the general case.\r\n\r\n[quote=\"Myth\"]If $E$ lies in the same halfplane, then $S_{EBC}\\leq S_{ABC}\\leq \\frac{1}{6n}$, so we find a small requared triangle $EBC$. Contradiction.\n[/quote]\r\n\r\ni still don't understand the first part.\r\nWhy $S_{EBC}\\leq S_{ABC}\\leq \\frac{1}{6n}$ yields a contradiction?", "Solution_8": "Because we supposed the contrary, i.e. there is no such triangle.", "Solution_9": "Case n=1 is also [url=http://www.mathlinks.ro/Forum/viewtopic.php?t=27331]here[/url]" } { "Tag": [ "pigeonhole principle", "number theory unsolved", "number theory" ], "Problem": "A positive integer is [i]dapper[/i] if at least one of its multiples begins with $ 2008$. For example, $ 7$ is dapper because $ 200858$ is a multiple of $ 7$ and begins with $ 2008$. Observe that $ 200858 \\equal{} 28694\\times 7$.\r\n\r\nProve that every positive integer is dapper.", "Solution_1": "Let m be a positive integer. Let $ n \\equal{} [log_{10}{m}] \\plus{} 1$. Consider $ 2008 \\cdot 10^n \\plus{} a, 0 \\le a \\le m \\minus{} 1 ( < 10^n)$. These are m numbers that begin with 2008 and have a different residue when divided by m, thus one of them is a multiple of m. We conclude that all positive integers are dappers.", "Solution_2": "Consider the m following numbers:\r\n\r\n2008, 20082008, 200820082008, ..., 20082008...20082008\r\n\r\nThe i-th term have i 2008's.\r\nIf one of them is multiple of m, ok.\r\nSuppose that there aren't multiples of m.\r\nBy the Pigeonhole Principle, there are 2 numbers with the same remainder at the divison by m.So, take the difference beetween this 2 numbers.It's obviously that this new number is multiple of m and begins with 2008. (q.e.d.)\r\n\r\n\r\nCya people,\r\nSecco", "Solution_3": "let n < $ 10^{m}$\r\n\r\nlet us consider 2008 X $ 10^{m \\plus{} 1}$. let it give r as remainder when divided by n.\r\nthen 2008 X $ 10^{m \\plus{} 1}$ +n-r is certainly a multiple of n and hence proved" } { "Tag": [ "trigonometry", "limit", "geometric series", "complex numbers", "geometric sequence" ], "Problem": "$\\displaystyle\\sum_{n=1}^{\\infty} 3^{n-1}\\cdot\\sin^3 \\left(\\frac{\\pi}{3^{n+1}}\\right)$\r\n\r\nI tried doing it by changing the sin expression into the Im part of $e^{\\frac{\\pi}{3^{n+1}}}$, then putting it into the inf geometric series formula and then taking the imaginary part of that at the end.\r\n\r\nHowever, I got $\\frac{\\sqrt{3}}{14}$ which is approximately .12, while Mathematica approximates this sum to 0.045293. Anybody care to look at it?", "Solution_1": "Use $\\sin 3\\theta=3\\sin \\theta-4\\sin ^ 3 \\theta\\Longleftrightarrow \\sin ^ 3 \\theta=\\frac{1}{4}(3\\sin \\theta-\\sin 3\\theta),$ then telescoping method and $\\lim_{\\theta \\longrightarrow 0} \\frac{\\sin \\theta}{\\theta}=1.$", "Solution_2": "i am pretty sure that this is not a geometric sequence\r\n\r\n$\\Im \\left(\\sum_{n=1}^{\\infty} 3^{n-1}*e^{\\frac{\\pi i}{3^n}}\\right)$\r\n\r\nnotice that it would if it were $e^{n}$, instead of $e^{3^{n}}$, it would be a geometric sequence", "Solution_3": "The answer is $\\frac{1}{4}\\left(\\frac{\\pi}{3}-\\frac{\\sqrt{3}}{2}\\right).$", "Solution_4": "can you show exactly what you did. i am not able to completely follow.", "Solution_5": "I'm able to get everything except that $\\frac{\\pi}{3}$ term to come up.", "Solution_6": "$\\sum_{k=1}^n 3^{k-1}\\sin ^ 3 \\frac{\\pi}{3^{k+1}}$\r\n\r\n${=\\frac{1}{4}\\sum_{k=1}^n 3^{k-1} (3\\sin \\frac{\\pi}{3^{k+1}}-\\sin \\frac{\\pi}{3^k}})$\r\n\r\n${=\\frac{1}{4}\\sum_{k=1}^n (3^k\\sin \\frac{\\pi}{3^{k+1}}-3^{k-1}\\sin \\frac{\\pi}{3^k}})$\r\n\r\n$=\\frac{1}{4}(3^n\\sin \\frac{\\pi}{3^{n+1}}-\\sin \\frac{\\pi}{3})$\r\n\r\n\r\n$=\\frac{1}{4}(\\frac{\\pi}{3} \\frac{\\sin \\frac{\\pi}{3^{n+1}}}{\\frac{\\pi}{3^{n+1}}}-\\frac{\\sqrt{3}}{2})$\r\n\r\n$\\longrightarrow \\frac{1}{4}(\\frac{\\pi}{3}-\\frac{\\sqrt{3}}{2})\\ (n\\rightarrow \\infty)$", "Solution_7": "Ah, I see that now. Very inginuitive.", "Solution_8": "very nice! :D" } { "Tag": [], "Problem": "This is an easy, in fact, very easy problem...\r\n\r\nI spent 3 hours this morning to find a better formula....\r\n\r\n1 coconut... 1st column\r\n\r\n3 coconuts 2nd column\r\n\r\n6 coconuts 3rd colum\r\n\r\n10 coconuts 4th colum\r\n\r\n\r\n\r\nnow find the number of coconuts in 100th colum....\r\n\r\nI just posted it because there weren't enough posts in this forum...", "Solution_1": "[hide=\"solution\"]\nformula is:\nn(n+1)/2\n\n100(100+1)/2\n100(101)/2\n5050\n(triangle numbers formula)\n[/hide]", "Solution_2": "[hide=\"to Treething\"]yup, you got it right, but I hid it so people could still try to solve the problem...[/hide]\r\n\r\n\r\n :lol: ;) :blush: :P", "Solution_3": "[quote=\"shinwoo\"]This is an easy, in fact, very easy problem...\n\nI spent 3 hours this morning to find a better formula....\n\n1 coconut... 1st column\n\n3 coconuts 2nd column\n\n6 coconuts 3rd colum\n\n10 coconuts 4th colum\n\n\n\nnow find the number of coconuts in 100th colum....\n\nI just posted it because there weren't enough posts in this forum...[/quote]\r\n\r\n[hide]x(x+1)/2\n100*101/2=5050[/hide]", "Solution_4": "[hide]\nn(n+1)/2\n=100*101/2\n=5050[/hide]", "Solution_5": "[hide]5050[/hide]\r\n\r\n[color=darkred]<_<\n>_>\n\nNext time don't check the \"Disable BBCode in this post\" box.[/color]", "Solution_6": "yuppy do guys!!!\r\n\r\ngood work!! :D", "Solution_7": "[hide] 5050[/hide]" } { "Tag": [ "function", "calculus", "derivative", "trigonometry", "calculus computations" ], "Problem": "I'm getting a wildly different answer from the back of the book on this one, and looking at the solutions manual, it (the book) seems to be wrong. Can someone verify? I hope this is the appropriate forum.\r\n\r\nThe function $T(x)=\\frac{200}{1+x^2}$ represents the temperature in degrees Celsius percieved by a person standing $x$ meters from the fire. If the person moves away from the fire at 2 m/s, how fast is the temperature changing when the person is 5 meters away?\r\n\r\nThanks in advance :)", "Solution_1": "So we're talking temperature in relation to distance from fire. Then we're looking at a \"change in temperature\" when there's a \"change in distance\" at a certain distance...what does this shout to you? \"DERIVATIVE!!!!! DO ME!!!!!\" So we do it:\r\n\r\n$dT=\\frac{-400x}{(1+x^{2})^{2}}dx$\r\n\r\nAt $5$m away and moving at $2$m/s, we substitute $x=5$ amd $dx=2$ and we get $dT=-2.96$\r\n\r\n\r\nMy calculation might be wrong but I suck at arithmetic.", "Solution_2": "$\\frac{dT}{dx} = \\frac{-400x}{(1+x^2)^2} \\rightarrow \\frac{dT}{dt} = \\frac{dT}{dx}\\frac{dx}{dt} = \\frac{-400x}{(1+x^2)^2} \\frac{dx}{dt}$\r\n\r\nFrom the problem statement, $\\frac{dx}{dt} = 2 m/s$ and $x=5m$. This should yield the same answer. Note also that this formula is dimensionally incorrect.", "Solution_3": "[quote=\"FMako\"]Note also that this formula is dimensionally incorrect.[/quote]\r\n\r\nThen ... doesn't this mean the formula is incorrect...? Or am I completely misunderstanding what you meant there? :blush:", "Solution_4": "FMako is talking about units of measurement whether stated or implied.\r\n\r\nThe biggest problem is that denominator, $1+x^2.$ We have that $x^2$ is the square of a distance; we measure it in $\\text{m}^2.$ What does that make $1?$ We can only add or subtract two quantities if they have the same units.\r\n\r\nIf we assume that the given $1$ is actually meant to be $1\\text{m}^2$ and the $200$ in the numerator is actually $200\\text{ deg}\\cdot\\text{m}^2$, then we can make the whole thing make sense.\r\n\r\nThat's not very satisfying. I like to use implied units of measurement - \"dimensional analysis\" - as a continuing check for the accuracy of computations, perhaps especially the intermediate steps. It's hard to get that kind of help from a formula like this.\r\n\r\nI'd be happier with $T=\\frac{k}{a^2+x^2}.$\r\n\r\nThen I read it as $a$ being a distance and $k$ being a constant of proportionality - and constants of proportionality carry whatever units they need to carry.\r\n\r\nOne little principle from this dimensional analysis outlook: suppose you see a formula like $e^{-kt}.$ Then if $t$ is a time and is measureed in units of measurement appropriate for a time, then $k$ must be the reciprocal of a time. Why? Because the argument of a \"transcendental function\" like the sine* or the exponential must be a pure unitless number.\r\n\r\n--\r\n\r\n* Footnote: an angle given in radians [i]is[/i] a \"pure unitless number.\"" } { "Tag": [ "trigonometry", "calculus", "ceiling function", "algebra", "polynomial", "induction", "complex analysis" ], "Problem": "Let $\\theta=\\frac{\\pi}{2n+1}\\/$ where $n$ is a positive integer.\r\nProve that \r\n1. $\\prod_{r=1}^{n} \\sin r\\theta = 2^{-n}\\sqrt{2n+1}\\/$\r\n2. $\\prod_{r=1}^{n} \\cos r\\theta = 2^{-n}\\/$\r\n\r\n(I don't know if it is suitable to post this question here :? :? :? )", "Solution_1": "i am sure that here is a elementary way and a complex analysis way to solve it. \r\nbut for calculus , \r\ni tried to break sin into a series and add them up , but got screwed up , \r\n\r\ncan anyone help ?", "Solution_2": "[quote=\"lisai\"]i am sure that here is a elementary way and a complex analysis way to solve it. \nbut for calculus \n[/quote]\r\nAny way is good as long as it yields a solution. So feel free to post the proof you know regardless of whether you consider it \"calculus\" or not. :)", "Solution_3": "[quote=\"pipi\"]Let $\\theta=\\frac{\\pi}{2n+1}\\/$ where $n$ is a positive integer. Prove that \n1. $\\prod_{r=1}^{n} \\sin r\\theta = 2^{-n}\\sqrt{2n+1}\\/$\n2. $\\prod_{r=1}^{n} \\cos r\\theta = 2^{-n}\\/$ [/quote]\r\n Denote $\\Theta= \\frac{\\pi}{2n+1}$, $P_1=\\prod\\limits_{k=1}^{n}\\sin{\\frac{k\\pi}{2n+1}},\\; \\; Q_1=\\prod\\limits_{k=n+1}^{2n}\\sin{\\frac{k\\pi}{2n+1}}$ and observe that $P_1>0 .$ Because $(*)\\hspace*{0.5cm} \\ds \\prod\\limits_{k=n+1}^{2n}T_k=\\ds \\prod\\limits_{k=1}^{n}T_{2n+1-k}\\; ,$ we have\r\n$Q_1=\\prod\\limits_{k=1}^{n}\\sin(2n+1-k)\\Theta= \\prod\\limits_{k=1}^{n}\\sin(\\pi-k\\Theta)=P_1\\; .$ Further we try to find $P_1^2=P_1Q_1=\\prod\\limits_{k=1}^{2n}\\sin{k\\theta}.$\r\n Let $z_k=\\cos{(2k\\theta)}+ i\\cdot\\sin{(2k\\theta)}.$ If we select $x=1$ in the equalities $\\ds \\frac{x^{2n+1}-1}{x-1}=\\sum\\limits_{k=0}^{2n}x^k=\\ds \\prod\\limits_{k=1}^{2n}\\left(x-z_k\\right) ,$ we find\r\n\\[ (**)\\; \\; \\; \\; \\begin{array}{c}\\ds \\ds 2n+1=\\ds \\prod\\limits_{k=1}^{2n}2\\left(\\sin{k\\Theta}-i\\cdot\\cos{k\\Theta}\\right)\\sin{k\\Theta}=\\\\ =\\ds 2^{2n}P_1^2 \\prod\\limits_{k=1}^{2n}\\left(\\sin{k\\Theta}-i\\cdot\\cos{k\\Theta}\\right)=2^{2n}P_1^2\\; . \\end{array} \\]\r\nThe last equality may be justified in the following way : \r\n\\[ \\begin{array}{c} \\prod\\limits_{k=1}^{2n}\\left(\\sin{k\\Theta}-i\\cdot\\cos{k\\Theta}\\right)=\\\\ =\\left( \\prod\\limits_{k=1}^{n}\\left(\\sin{k\\Theta}-i\\cdot\\cos{k\\Theta}\\right)\\right)\\cdot \\left( \\prod\\limits_{k=n+1}^{2n}\\left(\\sin{k\\Theta}-i\\cdot\\cos{k\\Theta}\\right)\\right)=\\\\ =\\left( \\prod\\limits_{k=1}^{n}\\left(\\sin{k\\Theta}-i\\cdot\\cos{k\\Theta}\\right)\\right)\\cdot \\left( \\prod\\limits_{k=1}^{n}\\left(\\sin{(2n+1-k)\\Theta}-i\\cdot\\cos{(2n+1-k)\\Theta}\\right)\\right)=\\\\ =\\left( \\prod\\limits_{k=1}^{n}\\left(\\sin{k\\Theta}-i\\cdot\\cos{k\\Theta}\\right)\\right)\\cdot \\left( \\prod\\limits_{k=1}^{n}\\left(\\sin{(\\pi-k\\Theta)}-i\\cdot\\cos{(\\pi-k\\Theta)}\\right)\\right)=\\\\ =\\left( \\prod\\limits_{k=1}^{n}\\left(\\sin{k\\Theta}-i\\cdot\\cos{k\\Theta}\\right)\\right)\\cdot \\left( \\prod\\limits_{k=1}^{n}\\left(\\sin{k\\Theta}+i\\cdot\\cos{k\\Theta}\\right)\\right)=1 \\; . \\end{array} \\]\r\nNow (**) enables us to write $\\ds P_1=\\ds \\frac{\\sqrt{2n+1}}{2^n}\\; .$ \r\nFurther try to find $P_2: =\\prod\\limits_{k=1}^{n}\\cos{k\\Theta}$ in following manner: observe that $P_2>0$ and $P_1P_2=\\ds \\frac{1}{2^n}\\prod\\limits_{k=1}^{n}\\sin{2k\\Theta}$ . Consider further the product $Q_2: =\\ds\\prod\\limits_{k=n+1}^{2n}\\sin{2k\\Theta}$ ... \r\n[b]Remark:[/b] In above hint/solution, equality (*) has an important role .", "Solution_4": "I have another idea.\r\nfor 2,I think you can use the conclusion posted [url=http://www.mathlinks.ro/Forum/viewtopic.php?t=40503]here[/url]\r\nfor1,you can find a solution similar way(but a little different)\r\nsorry for my short post,if who want to know more,i will try to explain again :)", "Solution_5": "Thanks for your solution, flip2004! \r\nI get the idea but I think there is something not allowed if we write:\r\n[quote=\"flip2004\"]\n Let $ z_k=\\cos{(2k\\theta)}+ i\\cdot\\sin{(2k\\theta)}. $ If we select $x=1$ in the equalities $\\ds \\frac{x^{2n+1}-1}{x-1}=\\sum\\limits_{k=0}^{2n}x^k=\\ds \\prod\\limits_{k=1}^{2n}\\left(x-z_k\\right)$,...[/quote]\r\nthanks again.", "Solution_6": "[quote=\"zhaobin\"]I have another idea.\nfor 2,I think you can use the conclusion posted [url=http://www.mathlinks.ro/Forum/viewtopic.php?t=40503]here[/url]\nfor1,you can find a solution similar way(but a little different)\nsorry for my short post,if who want to know more,i will try to explain again :)[/quote]\r\nHi zhaobin, it is possible to share your solution (for 1)?", "Solution_7": "Here are some questions I wish to post:\r\nLet $\\theta=\\frac{\\pi}{2n+1}\\/$ where $n$ is a positive integer. Prove that:\r\n(i) \\[\\sum_{r=1}^{n} cos \\ r \\theta = - \\frac{1}{2} \\] \r\n(ii) \\[\\sum_{r=1}^{n} cos^{2} \\ r \\theta = \\frac{2n-1}{4} \\] \r\n(iii) $r_1,r_2 \\in \\{1,2, \\ldots, n\\}\\/$ \\[\\sum_{r_1 >r_2} cos \\ r_1 \\theta cos \\ r_2 \\theta= \\frac{1-n}{4} \\] \r\n(iv) $\\forall n \\geq k \\geq 1\\/$ (more gerenal case for (i)), \\[\\sum_{r=1}^{n} (cos \\ r \\theta)^{2k-1} = - \\frac{1}{2} \\] \r\n\r\n\r\nI manage to get the solution for the above questions, and I make a conjecture (base on (i) and (ii) above), as follow:\r\nLet $\\theta = \\frac{2 \\pi}{2n+1}, n \\in N\\/$ and $r_1,r_2,\\ldots ,r_k \\in \\{1,2, \\ldots, n\\}\\/$.\r\n\\[\\sum_{r_1 >r_2> \\ldots >r_k} cos \\ r_1 \\theta cos \\ r_2 \\theta \\ldots cos \\ r_k \\theta = (-1)^l \\frac {{n- \\left\\lceil \\frac{k}{2} \\right\\rceil \\choose \\left\\lceil \\frac{k-1}{2} \\right\\rceil }}{2^k} \\]\r\n\r\nwhere $l= \\left\\lceil \\frac{k}{2} \\right\\rceil\\/$\r\n\r\nCan somebody help me on this :? :? :? \r\nThanks a lot!!", "Solution_8": "[quote=\"pipi\"]Thanks for your solution, flip2004! \nI get the idea but I think there is something not allowed if we write:\n[quote=\"flip2004\"]\n Let $ z_k=\\cos{(2k\\theta)}+ i\\cdot\\sin{(2k\\theta)}. $ If we select $x=1$ in the equalities $\\ds \\frac{x^{2n+1}-1}{x-1}=\\sum\\limits_{k=0}^{2n}x^k=\\ds \\prod\\limits_{k=1}^{2n}\\left(x-z_k\\right)$,...[/quote]\nthanks again.[/quote]\r\nsure :) .\r\nfirst,we can now $\\frac{\\sin{(2n+1)x}}{\\sin x}$ is a polynomial of $\\sin x$\r\nlet $P_n(2\\sin x)=\\frac{\\sin{(2n+1)x}}{\\sin x}$\r\nwe can know $P_n(x)$ have 2n roots which are $2\\sin{\\frac{k}{2n+1}}$ or $-2\\sin{\\frac{k}{2n+1}}$,$(k=1,2,\\cdots,n)$\r\nthen find the relation of $P_n(x),P_{n-1}(x),P_{n-2}(x)$\r\nwith induction you will find the answer :)" } { "Tag": [ "function", "trigonometry", "symmetry", "complex numbers", "complex analysis" ], "Problem": "I have been taught that the roots of complex numbers are multivalued, for example $ \\sqrt{3\\plus{}4i}\\equal{}\\pm (2\\plus{}i)$. What to do I have to add two complex roots, for example to compute $ \\sqrt{3\\plus{}4i}\\plus{}\\sqrt{3\\plus{}4i}$? Is the correct way to do it by take any branch of the first square root and add it to any branch of the second square root, so $ \\sqrt{3\\plus{}4i}\\plus{}\\sqrt{3\\plus{}4i}\\equal{}4\\plus{}2i, 0, \\text{ or }\\minus{}4\\minus{}2i$? Or does such situation arise anywhere in mathematics?", "Solution_1": "$ \\sqrt {3 \\plus{} 4i} \\plus{} \\sqrt {3 \\plus{} 4i} \\equal{} 2\\sqrt {3 \\plus{} 4i} \\equal{} \\pm 2(2 \\plus{} i)$, never $ 0$, otherwise you'd be saying $ \\sqrt {3 \\plus{} 4i} \\neq \\sqrt {3 \\plus{} 4i}$. I'd recommend using the principal square root for cases like this one, though (then again, what do I know? XD).", "Solution_2": "$ \\sqrt{3 \\plus{} 4i}$ is not a well-defined expression. You should never have to evaluate it unless the branch is explicitly given.", "Solution_3": "I wish to disagree: Can it not be all of them depending on what branches are chosen? Consider the funtion $ \\displaystyle f(z)\\equal{} \\sqrt{z}\\plus{}\\sqrt{z}$. Cannot $ f(1)\\equal{}0$ if the branches are chosen appropriately? How about $ \\displaystyle g(z)\\equal{}z^{1/3}\\plus{}z^{1/4}$? If $ \\displaystyle g(1)\\equal{}0$ then what is $ \\displaystyle g(\\minus{}1)$? I think $ \\displaystyle \\sqrt{3\\plus{}4i}\\plus{}\\sqrt{3\\plus{}4i}\\equal{}2\\sqrt{3\\plus{}4i}$ when the underlying square root multifunctions are being added symmetrically: first branch of the first expression added to the first branch of second one and second branch of the first with the second of the second expression. That's confusing I know. Never use the principal branch unless you need to use it or the context implies or warrants it: approach roots, logs, inverse trigs, other multifunctions as multivalued quantities until specific values are needed, specific branches are specified, or if the symmetry above is implied, that is if $ \\sqrt{z}\\plus{}\\sqrt{z}\\equal{}2\\sqrt{z}$ is assumed. I think $ \\sqrt{3\\plus{}4i}$ is perfectly well defined. It's $ \\displaystyle \\sqrt{3\\plus{}4i}\\equal{}r^{1/2}e^{i/2(\\Theta\\plus{}2k\\pi)}\\equal{}\\pm r^{1/2}e^{i/2\\Theta}$. If this was a test question, I'd stick to my guns and hope mine is on top the next day . . ." } { "Tag": [ "abstract algebra", "superior algebra", "superior algebra unsolved" ], "Problem": "Let $ G$ be an abelian finite group such that $ n$ divides $ |G|$.\r\nLet $ H\\equal{}\\{ x^n, x\\in G\\}$. There are many ways to prove that $ |H|\\equal{}|G|/n$, but I am trying to find an elementary proof that would not use the notion of quotient or factorization of homomorphism (i.e. $ G/ker (f) \\simeq f(G)$), let alone Cauchy or Sylow.\r\nAny idea?", "Solution_1": "[quote=\"Fleury\"]Let $ G$ be an abelian finite group such that $ n$ divides $ |G|$.\nLet $ H \\equal{} \\{ x^n, x\\in G\\}$. There are many ways to prove that $ |H| \\equal{} |G|/n$.[/quote]\r\nAre you sure of that? Because, if $ G \\equal{} \\{\\minus{}1,1\\} \\times \\{\\minus{}1,1\\}$ and $ n \\equal{} 2$, it appears that $ H \\equal{} \\{(1,1)\\}$ and that $ |H| \\equal{} 1 \\neq \\frac{|G|}{2} \\equal{} 2$.\r\nOr I may have misundertood what you had written...", "Solution_2": "[quote=\"LaChouetteRieuse\"]Are you sure of that? [/quote]\r\nNot anymore :blush:", "Solution_3": "There are some values of $ n$ when your statement is true (although I don;t have an \"elementary solution\"), namely if $ |G|\\equal{}\\prod_{i\\equal{}1}^{r}p_i^{a_i}$ ($ p_i$ primes and $ a_i>0$) and $ n\\equal{}\\prod_{i\\equal{}1}^{r}p_i^{b_i}$ with $ b_i\\equal{}a_i$ or $ b_i\\equal{}0$." } { "Tag": [ "inequalities", "inequalities proposed" ], "Problem": "If $ a,b,c,d \\ge 0$ then prove\r\n$ \\frac {a}{b\\plus{}c}\\plus{}\\frac {b}{c\\plus{}d}\\plus{}\\frac {c}{d\\plus{}a}\\plus{}\\frac {c}{d\\plus{}a} \\ge \\frac {5}{2}$.", "Solution_1": "[quote=\"soruz\"]If $ a,b,c,d \\ge 0$ then prove\n$ \\frac {a}{b \\plus{} c} \\plus{} \\frac {b}{c \\plus{} d} \\plus{} \\frac {c}{d \\plus{} a} \\plus{} \\frac {c}{d \\plus{} a} \\ge \\frac {5}{2}$.[/quote]\r\n\r\nI love inequalities that fail for $ a\\equal{}b\\equal{}c\\equal{}d\\equal{}1$...\r\n\r\nSee http://www.mathlinks.ro/viewtopic.php?t=140344 for the right one (I hope).\r\n\r\n darij", "Solution_2": "If $ a_1,a_2,...,a_n\\ge 0$ then prove\r\n\r\n$ \\frac {a_1}{a_2\\plus{}a_3}\\plus{}\\frac {a_2}{a_3\\plus{}a_4}\\plus{}...\\frac {a_n}{a_n\\plus{}a_n} \\ge \\frac{n}{2}$\r\n\r\nfor $ 0 \\ge a_1 \\ge a_2 ...\\ge a_n$.", "Solution_3": "Actually, the following nice statement holds:\r\nIf $ 1 \\le k \\le {n\\minus{}1}$ is an integer number and $ a_1,a_2,...,a_n\\ge 0$ \r\nsuch that $ a_1 \\le a_2\\le...\\le a_n$ then\r\n\r\n$ \\frac {a_1}{ka{_1}\\plus{}a_{k\\plus{}1}}\\plus{}\\frac {a_2}{ka{_3}\\plus{}a_{k\\plus{}2}}\\plus{}...\\frac {a_n}{ka_n\\plus{}a_k} \\ge \\frac{n}{k\\plus{}1}$", "Solution_4": "If $ a,b,c,d,e,f \\ge 0$ then prove\r\n \r\n$ \\frac {a}{b\\plus{}c}\\plus{}\\frac {b}{c\\plus{}d}\\plus{}\\frac {c}{d\\plus{}e} \\plus{}\\frac{d}{e\\plus{}a}\\plus{}\\frac{f}{a\\plus{}b} \\ge \\frac{5}{2}$.", "Solution_5": "[quote=\"soruz\"]If $ a,b,c,d,e,f \\ge 0$ then prove\n \n$ \\frac {a}{b \\plus{} c} \\plus{} \\frac {b}{c \\plus{} d} \\plus{} \\frac {c}{d \\plus{} e} \\plus{} \\frac {d}{e \\plus{} a} \\plus{} \\frac {f}{a \\plus{} b} \\ge \\frac {5}{2}$.[/quote]\r\nTry Cauchy - Schwarz. :wink:", "Solution_6": "[quote=\"soruz\"]If $ a,b,c,d,e,f \\ge 0$ then prove\n \n$ \\frac {a}{b \\plus{} c} \\plus{} \\frac {b}{c \\plus{} d} \\plus{} \\frac {c}{d \\plus{} e} \\plus{} \\frac {d}{e \\plus{} a} \\plus{} \\frac {f}{a \\plus{} b} \\ge \\frac {5}{2}$.[/quote]\r\n\r\nThis statement is quite as correct as the spelling of the word \"correct\" in your post...\r\n\r\n dg", "Solution_7": "I think the \"correct\" statement should be\r\n\r\n$ \\frac{a}{b\\plus{}c} \\plus{} \\frac{b}{c\\plus{}d} \\plus{} \\frac{c}{d\\plus{}e} \\plus{} \\frac{d}{e\\plus{}a} \\plus{} \\frac{e}{a\\plus{}b} \\geq \\frac{5}{2}$\r\n\r\nIs this right?\r\n\r\n[hide=\"Solution\"]\n\nAssume WLOG $ a\\plus{}b\\plus{}c\\plus{}d\\plus{}e \\equal{} 1$ and let $ f(x)\\equal{}\\frac{1}{x}$ which concaves up for $ x \\in \\mathbb{R}^{\\plus{}}$. By Jensen's Inequality:\n\n$ af(b\\plus{}c) \\plus{} bf(c\\plus{}d) \\plus{} cf(d\\plus{}e) \\plus{}df(e\\plus{}a) \\plus{} ef(a\\plus{}b) \\geq f(ab\\plus{}ac\\plus{}ad\\plus{}ae\\plus{}bc\\plus{}bd\\plus{}be\\plus{}cd\\plus{}ce\\plus{}de) \\equal{} \\frac{(a\\plus{}b\\plus{}c\\plus{}d\\plus{}e)^{2}}{ab\\plus{}ac\\plus{}ad\\plus{}ae\\plus{}bc\\plus{}bd\\plus{}be\\plus{}cd\\plus{}ce\\plus{}de}$\n\nSo we need to prove that: \n\n$ \\frac{(a\\plus{}b\\plus{}c\\plus{}d\\plus{}e)^{2}}{ab\\plus{}ac\\plus{}ad\\plus{}ae\\plus{}bc\\plus{}bd\\plus{}be\\plus{}cd\\plus{}ce\\plus{}de} \\geq \\frac{5}{2} \\iff (a\\minus{}b)^{2} \\plus{} (a\\minus{}c)^{2} \\plus{} (a\\minus{}d)^{2} \\plus{} (a\\minus{}e)^{2} \\plus{} (b\\minus{}c)^{2} \\plus{} (b\\minus{}d)^{2} \\plus{} (b\\minus{}e)^{2} \\plus{}(c\\minus{}d)^{2} \\plus{}(c\\minus{}e)^{2} \\plus{}(d\\minus{}e)^{2} \\geq 0$.[/hide]", "Solution_8": "[quote=\"BanishedTraitor\"]\n\n\n\nSo we need to prove that: \n\n$ \\frac {(a \\plus{} b \\plus{} c \\plus{} d \\plus{} e)^{2}}{ab \\plus{} ac \\plus{} ad \\plus{} ae \\plus{} bc \\plus{} bd \\plus{} be \\plus{} cd \\plus{} ce \\plus{} de} \\geq \\frac {5}{2}$[/quote]\r\nThis is just Cauchy. :wink:" } { "Tag": [ "Gauss", "algebra", "polynomial", "function", "integration", "calculus", "derivative" ], "Problem": "Let A the Gauss ring of integers and $z_1,...,z_n$ in A such that the distance from $z_i$ to $z_1$ is greater than 2 for all $i>1$. Prove that the polynomial $ 1+(x-z_1)...(x-z_n)$ is irreducible in $A[X]$.", "Solution_1": "What is Gauss ring of integers? Is it $a+bi$, $a,b\\in\\mathbb{Z}$?", "Solution_2": "Yes, it is", "Solution_3": "Then it is very standart.\r\nConsider polynomials $f(z)=(z-z_1)(z-z_2)...(z-z_n)$ and $g(z)=(z-z_1)(z-z_2)...(z-z_n)$. We see that $|f(z)-g(z)|=1<|f(z)|+|g(z)|$ for all $z$ lying on the circle $|z-z_1|=1$. So $f(z)$ has the same number of roots inside of $|z-z_1|=1$ as $g(z)$, but $g(z)$ has the only such root $z_1$. \r\n\r\nSuppose the contrary, i.e. $f(z)=a(x)b(x)$. We may WLOG assume $z_1=0$. Then we have $a(0)b(0)=1$, but both $|a(0)|,|b(0)|\\geq 1$ and at least one of these numbers is greater than 1, since one of the polynomials $a$ or $b$ has all roots are greater than 1 at absolute value. Contradiction.", "Solution_4": "Yes, Myth, this was the idea. Probably for you it is standard, but I do not think that proving Rouche's theorem in an oral examination is so easy.", "Solution_5": "And who ask to prove it in an oral examanation? Just use it!", "Solution_6": "This is also my opinion, Myth, but the problem is that sometimes the examinator does not accept high theorems during the examination if you do not know a proof. This is the problem with oral examinations! But anyway, in this case I know a nice proof of this particular case of Rouche's theorem, thus I do not have problems.", "Solution_7": "is there any elementary proof (no complex analysis) of Rouche's theorem ?\r\nHarazi, what is the proof for this particular case you are talking about? I'm very interested. Thanx :)", "Solution_8": "We made in a \"travaux dirig\u00e9\" something about \"indice d'un lacet\" and the teacher has shown to us that if we consider the function $ f(t)=P(re^{it})$ then the \"indice\" of this function, that is $\\int_{0}^{2\\pi}{\\frac{f'(t)}{2\\pi\\cdot (t)}}$ is equal to the number of roots of the polynomial P that lie inside the disk $|z|0}}$ has gaps of size 1 or less. So, suppose we have a gap somewhere, $b_{n}= m$ and $b_{n+1}= m+2$. Thus $m < n\\beta < m+1$ and $m+2 < (n+1)\\beta < m+3$ and $n \\leq m$.\r\n\r\nThis means that $\\frac{m+2}{n+1}< \\beta < \\frac{m+1}{n}$, so if I write $n = m-k+1$ we have $k > 0$ and\r\n\\[\\frac{m+2}{m-k+2}< \\beta < \\frac{m+1}{m-k+1}\\]\r\n\\[\\frac{k}{m-k+2}< \\beta-1 < \\frac{k}{m-k+1}\\]\r\n\\[m-k+1 < \\frac{k}{\\beta-1}< m-k+2 \\]\r\n\\[m+1 < k\\cdot \\frac{\\beta}{\\beta-1}< m+2 \\]\r\nBut $\\frac{\\beta}{\\beta-1}= \\alpha$, so $\\lfloor k \\cdot \\alpha \\rfloor = m+1$ and the two sequences together cover all the integers." } { "Tag": [ "algebra", "polynomial" ], "Problem": "determine k so that f(x) = 2x^2 +(k-1)x - k(k+1) is divisible by x+2 and the value of k is an exact divisor of \r\n\r\n9\r\n15\r\n27\r\n49\r\nnone of these", "Solution_1": "[hide=\"HINT\"]\nVieta's.\n[/hide]", "Solution_2": "[hide=\"Hint\"]\nFactor theorem.\n\nIf $ f(x)$ is a polynomial then $ (x\\minus{}a)$ is a factor iff $ f(a)\\equal{}0$\n[/hide]", "Solution_3": "could be Taylor's????" } { "Tag": [ "function", "combinatorics unsolved", "combinatorics" ], "Problem": "Give two fintes set $X$ and $Y$ such that $|X|=n,|Y=m|$ where $n \\geq m >0$.\r\nFind the number of onto function mapping form $X$ to $Y$ :)", "Solution_1": "By 'onto' you mean surjective?\r\nIn that case, just look to another thread in this section.\r\n\r\nPierre.", "Solution_2": "Anyway ;\r\nlet\r\n$f: X \\rightarrow Y,|X|=x,|Y|=y$ define $A_{i}: =\\{f: X \\rightarrow Y;j\\notin ImY\\},j=1,2,\\dots ,y$\r\nThen $|A_{1}^{c}\\cap A_{2}^{c}\\cap \\dots \\cap A_{y}^{c}|=y^{x}-{y \\choose 1}(y-1)^{x}+\\dots+(-1)^{y}{y \\choose y}(y-y)^{x}=\\sum_{i=0}^{y}(-1)^{i}{y \\choose i}(y-i)^{x}$" } { "Tag": [ "inequalities", "inequalities open" ], "Problem": "let a,b,c and k are possitive real numbers such that $ a \\plus{} b \\plus{} c \\equal{} 3$ . Prove the inequality:\r\n $ \\sqrt {ka^2 \\plus{} b} \\plus{} \\sqrt {kb^2 \\plus{} c} \\plus{} \\sqrt {kc^2 \\plus{} a} \\geq 3\\sqrt {k \\plus{} 1}$", "Solution_1": "It isn't usually trues with all $ k \\ge 0$, so you should write \"Find all the calue of $ k$ for the following ineq still trues\" :maybe:\r\nMy YM: langtukhongnha1012", "Solution_2": "[quote=\"AngelHolder\"]let a,b,c and k are possitive real numbers such that $ a \\plus{} b \\plus{} c \\equal{} 3$ . Prove the inequality:\n $ \\sqrt {ka^2 \\plus{} b} \\plus{} \\sqrt {kb^2 \\plus{} c} \\plus{} \\sqrt {kc^2 \\plus{} a} \\geq 3\\sqrt {k \\plus{} 1}$[/quote]\r\n\r\n\r\n\r\n$ k\\geq \\frac{1}{2}$\r\n\r\nThe inequality holds." } { "Tag": [ "algebra unsolved", "algebra" ], "Problem": "1. A car with tires having radius 33cm was driven on a trip and the odometer indicated that the distance travelled was 640km. Two weeks later, with snow tires installed, the odometer indicated that the distance for the return trip over the same route was 625 km. Find the radius of the snow tires.\r\n\r\nI keep thinking this is $\\frac{640}{625}=\\frac{33}{x}$ But it does not agree with the answer given:\r\n[hide]33.8[/hide]\n2. Bob and Jim, next-door neighbours, use hoses from both houses to fill Bob's swimming pool. They know it takes eighteen hours using both hoses. They also know that Bob's hose, used alone, can fill the pool in six hours less than Jim's hose. How much time is required by each hose alone?\n\nThis one is just plain confusing, I've declared about 5 variables in the first 15 seconds of trying to solve it, the answer is:\n[hide]33.25h with Bob's hose and 39.25h with Jim's hose[/hide]\r\n\r\n3. Prove that $\\frac{\\sqrt{2}+\\sqrt{6}}{\\sqrt{2}+\\sqrt{3}}=2$\r\nI'm really confused on this one, as my calculator tells me its not 2 lol.", "Solution_1": "[b]1[/b]. $t_{1}$ - time of travel of the car with tires having radius $R_{1}= 33$cm. $S_{1}= 640$km - distance of this car .\r\n$t_{2}$ - time of travel of the car with tires having radius $R_{2}= x$cm. $S_{2}= 625$km - distance of this car .\r\n\r\nThen: $t_{1}\\approx \\frac{1}{R_{1}}$, or $t_{1}=\\alpha \\frac{1}{R_{1}}$ and $t_{2}\\approx \\frac{1}{R_{2}}$, or $t_{2}=\\alpha \\frac{1}{R_{2}}$, $\\alpha$ - aspect ratio.\r\n\r\nAnd $S_{1}\\approx t_{1}$, $S_{2}\\approx t_{2}$.\r\n\r\nThen $\\frac{S_{1}}{S_{2}}=\\frac{\\alpha \\frac{1}{R_{1}}}{\\alpha \\frac{1}{R_{2}}}$.\r\n$\\Rightarrow \\frac{S_{1}}{S_{2}}=\\frac{R_{2}}{R_{1}}$\r\n$\\Rightarrow \\frac{640}{625}=\\frac{x}{33}$\r\n$\\Rightarrow x = 33,792$ or $x \\approx 33,8$\r\n\r\n[b]2[/b]$x$h - time, use Bob's hose to fill one swimming pool.\r\n$(x+6)$h - time, use Jim's hose to fill one swimming pool.\r\nFor $x$h Jim's hose to fill $\\frac{x}{x+6}$ part swimming pool.\r\nUse hoses from both houses, then for $x$h fill $(1+\\frac{x}{x+6})$ pools. \"We know it takes eighteen hours using both hoses\".\r\nProportion:\r\n$\\frac{1}{1+\\frac{x}{x+6}}=\\frac{18}{x}$\r\n$18(1+\\frac{x}{x+6})=x$\r\n$18\\frac{x}{x+6}=x-18$\r\n$18x=(x+6)(x-18)$\r\n$x^{2}-30x-108=0$\r\n$x \\approx 33,25$h - Bob's hose, $(x+6) \\approx 39,25$;// Excuse me for my English", "Solution_2": "3)\r\n\r\nMultiply top and bottom by $\\sqrt(3)-\\sqrt(2)$ to get:\r\n$\\frac{(\\sqrt(3)-\\sqrt(2))*(\\sqrt(2)+\\sqrt(6))}{3-2}$\r\n$= (\\sqrt(3)-\\sqrt(2))*(\\sqrt(2)+\\sqrt(6))$\r\n$= \\sqrt(6)+\\sqrt(18)-2-\\sqrt(12)$\r\n$= \\sqrt(6)+3\\sqrt(2)-2-2\\sqrt(3)$.\r\n\r\n\r\nNow, if this is $2$, then $\\sqrt(6)+3\\sqrt(2)-2\\sqrt(3) = 4$,\r\nwhich is not true." } { "Tag": [ "geometry unsolved", "geometry" ], "Problem": "An arbitrary point $B$ is chosen on the line $AC$ and on the segments $AB,BC$ and $CA$ as diameters, circles $k_1,k_2$ and $k$ are constructed. An arbitrary line through the point $B$ intersects $k$ in the points $P$ and $Q$ , and circles $k_1$ and $k_2$ (besides the point $B$ )in the points $R$ and $S$ , respectively. Prove that $PR=QS$.", "Solution_1": "Let AR and CQ meet the circle k at M and N. \r\nSince 1 is a perfect square, then A is not an integer, for ANY n.\n\nUnable to prove or disprove my conjecture. If I'm right, then maybe Peter's Lemma has to be modified.(?) Please help. Thanks.\n\n\n\n\n", "Solution_8": "Peter,\nThe first sentence of your post. You \u201c\u2026will have A odd immediately\u201d(?)\nBut what if t = 4, 9, 16, 25, 36 , 49\u2026or 12, 5, 19 and n is any number. Any! \nThen A will be proper or improper fraction - not an odd as you claim. \n If t = 0 AND n = 1, 3, 5, 7,... or t = 7 AND n = 21, 63, 105, 147, \u2026. or t = 22 AND n = 9, 27, 45, 63, 81\u2026.\nthen in each case we get infinite number of odd A\u2019s \u201cimmediately\u201d and \"m = 2 + 12t\" works. Nothing is always\n You ignore(?) that A is not A(t), but A(t, n). t is parameter, n is a variable (see below)\nTrivial: A can be either a fraction (for any n) or an odd (for certain t\u2019s and certain unlimited n\u2019s). \n \nLet\u2019s subdivide the set of 26 consecutive t\u2019s (from 0 to 25) by two subsets: \u2018bad\u2019 t\u2019s and \u2018good\u2019 t\u2019s.\n\nProposition 1. \u2018Bad\u2019 t\u2019s do not make A an odd. For any n. \n\nHere they are: t = 4, 9, 12, 14, 15, 16, 18, 19, 20, 21, 23, 24, 25.\n\nProposition 2. \u2018Good\u2019 t\u2019s do make A an odd. But not for any n.\n\nGood t\u2019s: t = 0, 1, 2, 3, 5, 6, 7, 8, 10, 11, 13, 17, 22 \n\nBad t\u2019s do not solve the problem A17. We\u2019ll ignore them from now on.\n\nNow we\u2019ll find all unlimited odd A\u2019s (all of them) for first 13 good t\u2019s and and predefined n\u2019s\nWe \u2018ll show that A can not be just any odd but unlimited strictly defined ones.\nIn the table below k runs through all odds: k = 1, 3, 5, 7, \u2026.\n\nIn A(t, n): t is parameter, n is variable (a linear function of odd k\u2019s for each \u2018good\u2019 t). Some examples:\nt = 0 A(0, k) is an odd for any n = k = 1, 3, 5, 7, \u2026\u2026. then A(0, k) = 1, 21, 521, 13 021, 325 521, \u2026\nt = 1 A(1, 3k) is an odd for any n = 3k = 3, 9, 15, 21\u2026.. then A(1, 3k) = 117, 2 823 520 869, \u2026\u2026\u2026 \nt = 7 A(7, 21k) is an odd for any n = 21k = 21, 63, 105, 147,\u2026 then A(7,21k) =\u2026..infinite number of huge odds \nt = 13 A(13, 39k) is an odd for any n = 39k = 39, 117, 195,\u2026 then A(13, 39k) = \u2026.. \u2018\u2019\n\n\u2018good\u2019 t___m=2+12t_____n_______A(t,n) = ((5+12t)^n + 1)/3(2+12t)_____A(t, n), all odds.\n============ ==================================================================\n \n0_________2__________k________A (0, k) = (5^k + 1)/6_____________A(0, k) = 1, 21, 521, 13021, 325521,. \n\t\t\t\t\t\t\t\t\t\t\t\t\t\t\t\t\t\t\t\t\t \n1________14_________ 3k_______A (1, 3k) = (17^3k + 1)/42_________ A(1, 3k) = 117, 2823520869, \u2026.. \n\t\t\t\t\t\t\t\t\t\t\t\t\t\t\t\t\t\t\t\t\t\t\n2________26__________3k_______A(2, 3k) = (29^3k + 1)/78_________ A(2, 3k) = Infinite number of huge odds \n\t\t\t\t\t\t\t\t\t\t\t\t\t\t\t\t\t\t\t\t\t\t \t\n3________38__________9k_______A(3, 9k) = (41^9k + 1)/114________ A(3, 9k) = _______ same\n\n5________62_________15k_______A(5,15k) = (65^15k + 1)/186_______A(5, 15k)________ same \n\t\t\t\t\t\t\t\t\t\t\t\t\t\t\t\t\t\t\t\t\t\t\n6________74__________9k_______A (6, 9k) = (77^9k + 1)/222_______ A(6, 9k)_________ same\n\t\t\t\t\t\t\t\t\t\t\t\t\t\t \n7________86_________ 21k______ A(7, 21k) = (89^21k + 1)/258_____ A(7, 21k)________ same\n\n8________98__________21k______A(8, 21k) = (101^21k + 1)/294_____A(8, 21k)________ same\n\n10______122___________5k______A(10, 5k) = (125^5k + 1)/366______ A(10, 5k)__________\"\n\n11______134__________11k______A(11, 11k) = (137^11k + 1)/402_____A(11, 11k)__________\" \n\n13______158__________39k______A(13, 39k) = (161^39k + 1)/474_____A(13, 39k)_________\u2018\u2019 \n \n17______206__________17k______A(17, 17k) = (209^17k + 1)/618_____A(17, 17k)_________\u2018\u2019 \n\n22______266___________9k_____ A(22, 9k ) = (269^9k + 1)/798______A(22, 9k)___________\u2018\u2019\n\nThere are 13 good and 13 bad t's. Just coincidence.\nThat\u2019s all I can say about that. As an exercise: for 25 < t < 51 find all \u2018good\u2019 t\u2019s and corresponding n\u2019s.", "Solution_9": "all odd prime factors of m are $\\equiv 1 \\pmod{3}$\nwrite $m=2^k \\cdot j$, $k \\geq 0$ and $j\\equiv 1 \\pmod{3}$\ntaking modulo 3, $m\\equiv 2\\pmod{3}$ and n is odd\n$2^k \\cdot j \\equiv 2 (mod 3) \\implies$ k is odd\nbut $k\\geq 3 \\iff 8|m \\implies 8|3^n+1$ creates contradiction\nso $k=1 \\iff 2||m$\nso $m\\equiv 2\\pmod {12}$\n\n$(m+3)^n+1\\equiv 2\\pmod4$ , so A is odd\none example is $(m,n,A)=(2,1,1)$", "Solution_10": "[quote=Peter]Let $m$ and $n$ be natural numbers such that \\[A=\\frac{(m+3)^{n}+1}{3m}\\] is an integer. Prove that $A$ is odd.[/quote]\n\nWhen $m$ is odd ,$ A$ is always odd because both numerator and denominator are odd.\n\n [b]Claim:[/b] When $ m,n$ are both even ,or when $ m$ is divisible by $3$ then $ A$ is never an integer.\n[b]Proof[/b] If $m \\equiv 0 \\mod {3}$ then $$(m+3)^n+1 \\equiv m^n+1 \\equiv 1 \\mod 3 $$ wich contradicts the fact that $ A$ is integer .\nWhen $m \\not\\equiv 0 \\mod 3$ and $n $ even we have that$$(m+3)^{2k}+1 \\equiv m^{2k}+1 \\equiv 2 \\mod 3 $$wich again contradicts that $A$ is integer \n\n[b]Claim:[/b] When $m$ is even , then we need $m=2k $ or $m= 4k$ where $k$ is odd integer for $A $ to be integer. \n[b]Proof:[/b] We must have $$ (m+3)^n+1 \\equiv 3^n+1\\equiv 0 \\mod m$$ or $$ m| 3^n+1$$ since $n$ is odd\n\nBy LTE we can say that $$v_2(m) \\leq v_2 (3^n+1)=v_2(4)+v_2(n)=2 $$\n\n[b]Claim: [/b] When $m=2k$ or $m=4k$ then $v_2(A)=0$\n[b]Proof[/b] $$v_2(A)=v_2 ((m+3)^n+1)-v_2(3m)=v_2(2k+4)-v_2(6k)=0$$ for $ m=2k$ and \n$$v_2(A)=v_2(4k+4)-v_2(12k)=0$$ for $m=4k$\n\nAfter checking all cases we can say that $A$ is always odd. $\\blacksquare$", "Solution_11": "[quote=XbenX]\n[b]Claim: [/b] When $m=2k$ or $m=4k$ then $v_2(A)=0$\n[b]Proof[/b] $$v_2(A)=v_2 ((m+3)^n+1)-v_2(3m)=v_2(2k+4)-v_2(6k)=0$$ for $ m=2k$ and \n$$v_2(A)=v_2(4k+4)-v_2(12k)=0$$ for $m=4k$\n[/quote]\n\nThis part is wrong. Suppose we have $m \\equiv 4 \\text{ (mod }8 \\text{)}$ and $n$ an odd integer, then $(m+3)^n$ is congruent to $7$ (mod $8$). then $v_2(A)$ might be positive, and in that case we need to prove the divisibility relation can't hold, instead of computing $v_2(A)$ more. ", "Solution_12": "Legandre symbol" } { "Tag": [ "permutations", "combinatorics", "IMO", "IMO 1963" ], "Problem": "Five students $ A, B, C, D, E$ took part in a contest. One prediction was that the contestants would finish in the order $ ABCDE$. This prediction was very poor. In fact, no contestant finished in the position predicted, and no two contestants predicted to finish consecutively actually did so. A second prediction had the contestants finishing in the order $ DAECB$. This prediction was better. Exactly two of the contestants finished in the places predicted, and two disjoint pairs of students predicted to finish consecutively actually did so. Determine the order in which the contestants finished.", "Solution_1": "[hide]\nWe use casework on the first place student.\n\nIt is impossible for $ A$ to be first, as this would agree with Prediction 1.\n\nIf $ B$ is first, then $ (CB)$ could not have finished consecutively. So the possible pairs from Prediction 2 are $ (DA)$, $ (AE)$, and $ (EC)$. Only $ (DA)$ and $ (EC)$ are disjoint. We cannot start with $ BEC$, as this would place $ C$ as predicted in Prediction 1. So our order is $ BDAEC$, which has no agreement with Prediction 2. So $ B$ cannot be first.\n\nIf $ C$ is first, then the possible pairs from Prediction 2 are $ (DA)$, $ (AE)$, and $ (CB)$. We cannot use $ (CB)$ because it would place $ B$ as predicted by Prediction 1. This leaves only $ (DA)$ and $ (AE)$, which are not disjoint, so it is impossible for $ C$ to be first.\n\nIf $ D$ is first, we can either use $ (DA)$ or not. If we do, then we must choose either $ (EC)$ or $ (CB)$ as our second pair. If we use $ (EC)$, we cannot start $ DAEC$, as this agrees in four places with Prediction 2. So we have $ DABEC$, which shares $ (AB)$ with Prediction 1, so it is impossible. If we use $ (CB)$, we cannot start $ DACB$, as $ C$ agrees with Prediction 1. So we are left with $ DAECB$, which agrees in five places with Prediction 2.\n\nSo we cannot use $ (DA)$. The possible pairs from Prediction 2 are $ (AE)$, $ (EC)$, and $ (CB)$. Only $ (AE)$ and $ (CB)$ are disjoint. We cannot place $ (AE)$ right after $ D$, as $ (DA)$ is forbidden. So we must have $ DCBAE$, in which $ E$ agrees with Prediction 1. So $ D$ cannot be first.\n\nThus, $ E$ is first. The possible pairs from Prediction 2 are $ (DA)$, $ (EC)$, and $ (CB)$. If we use $ (EC)$, then we must also use $ (DA)$. But $ ECBDA$ and $ ECDAB$ have only zero and one correspondences with Prediction 2, respectively. So we cannot use $ (EC)$ \n\nIf we don't use $ (EC)$, then we must have $ EDACB$ (preventing $ (EC)$ by not placing $ (CB)$ right after $ E$). This clearly satisfies all requirements, so it is the desired order.\n[/hide]", "Solution_2": "Wouldn't that order be wrong, though? B and C finish consecutively, as do D and E.\n\nOr is it that they must finish consecutively in the order that was predicted? So A can't be right before B, but B can be right before A?", "Solution_3": "Why there are no pure combinatorics questions in the early IMOs?", "Solution_4": "[quote=sunjae62700]Why there are no pure combinatorics questions in the early IMOs?[/quote]\n\nDo you want to mean counting??" } { "Tag": [ "linear algebra", "linear algebra unsolved" ], "Problem": "1,Suppose $ A,B\\in F^{n\\times n},AB\\equal{}BA\\equal{}\\mathbf{0},rank(A^2)\\equal{}rank(A)$,show that $ rank(A\\plus{}B)\\equal{}rank(A)\\plus{}rank(B)$.\r\n2,Suppose $ A,B\\in F^{n\\times n},AB\\equal{}BA\\equal{}\\mathbf{0}$.Prove that,there exists a positive integer $ k$ such that $ rank(A^k\\plus{}B^k)\\equal{}rank(A^k)\\plus{}rank(B^k)$.\r\n3,Suppose $ A\\in F^{m\\times n},B\\in F^{n\\times m}$.Prove: $ rank(AB)\\equal{}rank(A)$ if and only if there exists $ C\\in F^{m\\times n}$ such that $ A\\equal{}ABC$. Furthermore ,Prove that if $ (AB)^2\\equal{}AB$ then $ (BA)^2\\equal{}BA$.\r\n\r\nI found these problems difficult for me. Any help will be appreciated.", "Solution_1": "I found on CMJ, David Calan proved the following theorem:\r\n\r\nIf $ A$ and $ B$ are two matrices. Then\r\n\\[ \\text{Rank}(A) \\plus{} \\text{Rank} (B) \\equal{} \\text{Rank}(A \\plus{} B)\r\n\\]\r\nif and only if $ C(A)\\cap C(B) \\equal{} \\{0\\}$ and $ R(A)\\cap R(B) \\equal{} \\{0\\}$, where $ C(X)$ and $ R(X)$ mean the column and row space of $ X$ respectively.\r\n\r\nThe proof is quite elementary (I'll post the proof if anybody need it. :) ).\r\n\r\nThis theorem solves the first problem since form $ AB \\equal{} 0$ implies $ C(B) \\subset \\text{Ker}(A)$ and $ \\text{Rank}(A^2) \\equal{} \\text{Rank}(A)$ implies $ C(A) \\cap \\text{Ker}(A) \\equal{} \\{0\\}$ thus $ C(A)\\cap C(B) \\equal{} \\{0\\}$. Similarly $ R(A)\\cap R(B) \\equal{} \\{0\\}$.", "Solution_2": "Wich issue of CMJ did you find the result of David Calan ? thank", "Solution_3": "For those who have access to the jstor: [url=http://www.jstor.org/pss/2687710]When is rank additive?[/url]", "Solution_4": "For 2/\r\n\r\nThere exist $ q$ positive integer such that \r\n\r\n$ Im(A^k)\\equal{}Im(A^q)$ for any $ k\\geq q$\r\n\r\nWe have also $ Im(A^q) \\cap ker(A^q) \\equal{} 0$ \r\n\r\nSince A,B commute we have $ A^qB^q\\equal{}0$ \r\n\r\n$ Im(B^q)$ is included in $ ker(A^q)$ \r\n\r\nWe obtain \r\n\r\n$ Im(A^q) \\cap Im(B^q)$ is included in $ Im(A^q) \\cap ker(A^q) \\equal{} 0$ \r\n\r\nSo $ C(A^q) \\cap C(B^q) \\equal{} 0$ \r\n\r\nto finish apply the result of beginner \r\n\r\n$ rank(A^q\\plus{}B^q)\\equal{}rank(A^q) \\plus{} rank(B^q)$", "Solution_5": "can some one post a proof of beginner theorem" } { "Tag": [ "IMO" ], "Problem": "Greetings! I was the fourth deadly SIN at the Glasgow IMO.\r\nI am now a conscript. *sob*\r\nSadly, this happens to all male singaporean citizens who aren't exempted on medical grounds. It is a sad fate. So while my foreign peers are out there in university, I'm languishing in an undisclosed location, somewhere within 20km of the coast. If you look at the map of Singapore, you will realise that this is not saying much.\r\n\r\nThe reason why I am online the whole of this sunday is really because I'm on manning duty, which means that I'm sitting at my office computer waiting for a phone call that I hope will never come. (This is ******. ****** is activated. ****** will be at ******)\r\n\r\nMy path to the IMO:\r\n2001: Lose tie-breaker, get sent to SEAMEO-MO instead, first (but vietnam wasn't there)\r\n\r\n2002: Win tiebreaker (notice the pattern?). Get silver. Become extremely depressed a few months later upon getting conscripted. (and upon the realisation of the fact that I'll probably never have so much fun)", "Solution_1": "IMO2002\r\n\r\nZachary Leung (no longer a citizen - who'd want to be at our age? Ans. Females)\r\n21 bronze\r\nCalvin Lin (now a conscript)\r\n10\r\nMeng Dazhe (not a citizen)\r\n20 bronze\r\nKiat Chuan (conscript)\r\n24 silver\r\nJulius Poh (conscript)\r\n25 silver\r\nColin Tan (medically exempted.)\r\n12 HM\r\n\r\nNote that 3/6 are exempt from conscription. Clearly I have been less than cunning.\r\n\r\nWe were 30th. I would like to point out that there was only 1 team <20 pts below us, whereas there were 6 teams within 10 pts (more) of us, and 12 within 20.\r\n\r\nThe only thing I can say to this is - darn!\r\n\r\nBut this year our team's 18th! Makes an old fogey like me feel dull.\r\n*wave flag*" } { "Tag": [], "Problem": "From: http://www.xxhh.net\r\n\r\n\u6211\u662f\u4e00\u4e2a\u8d3c\u4f20\u7edf\u7684\u7537\u4eba\uff08\u5f53\u65f6\u662f\u4f20\u7edf\u7537\u5b69\uff09\uff0c\u4e0a\u5927\u5b66\u4e4b\u524d\u6211\u90fd\u4e0d\u66fe\u79bb\u5f00\u8fc7\u519c\u6751\uff0c\u5373\u4f7f\u4e0a\u4e86\u5927\u5b66\uff0c\u7b2c\u4e00\u4e2a\u5b66\u671f\u8fd8\u662f\u8c61\u4e00\u4e2a\u4e61\u6751\u5c11\u5e74\uff0c\u4e0d\u8bc6\u5f97\u4efb\u4f55\u90fd\u5e02\u98ce\u60c5\uff01\u8bb0\u5f97\u5f53\u65f6\u5927\u90e8\u5206\u540c\u5b66\u5bf9\u67d0\u4e9b\u4eba\u5f00\u5b66\u4e0d\u5230\u4e24\u4e2a\u661f\u671f\u5c31\u5f00\u59cb\u540c\u5c45\u89c1\u602a\u4e0d\u602a\uff0c\u4f46\u8fd9\u4e8b\u53eb\u6211\u60ca\u5947\u4e86\u597d\u957f\u4e00\u6bb5\u65f6\u95f4\uff0c\u56e0\u4e3a\u6211\u548cgf\u62cd\u4e86\u534a\u5e74\u7684\u65f6\u5019\u8fd8\u6ca1\u6709\u7275\u8fc7\u624b\u3002\r\n\u5f00\u59cb\u8fdb\u5165\u6b63\u9898\uff0c\u5728\u4e0a\u5927\u5b66\u4e4b\u524d\uff0c\u6211\u5f53\u7136\u4ece\u672a\u542c\u8bf4\u8fc7\u536b\u751f\u5dfe\uff0c\u56e0\u4e3a\u6211\u4eec\u9ad8\u4e2d\u662f\u6ca1\u4eba\u7528\u90a3\u73a9\u610f\u7684\uff0c\u518d\u8bf4\u4e86\uff0c\u5c31\u662f\u771f\u7684\u6709\u4eba\u7528\uff0c\u6211\u4e5f\u4e0d\u77e5\u9053\u554a\uff0c\u4e0d\u8fc7\u5f53\u65f6\u7684\u786e\u5f88\u5c11\u4eba\u7528\u3002\u4e0a\u4e86\u5927\u5b66\uff0c\u8fd9\u4e2a\u8bcd\u6211\u542c\u7684\u591a\u4e86\u8d77\u6765\uff0c\u4f46\u6211\u5e76\u4e0d\u4e86\u89e3\u5b83\u7684\u771f\u6b63\u7528\u9014\uff0c\u6240\u4ee5\u5bf9\u5b83\u4e5f\u6ca1\u6709\u4ec0\u4e48\u7279\u522b\u7684\uff1b\u5728\u6211\u7684\u8111\u888b\u91cc\u5934\uff0c\u536b\u751f\u5dfe\u5c31\u662f\u9ad8\u7ea7\u7684\u536b\u751f\u7eb8\uff0c\u662f\u6709\u94b1\u4eba\u7528\u7684\u536b\u751f\u7eb8\uff0c\u6211\u6ca1\u94b1\uff0c\u5f53\u7136\u4e0d\u53bb\u4e70\u5566\uff0c\u4f46\u662f\uff0c\u8bf4\u5b9e\u8bdd\uff0c\u6211\u5f53\u65f6\u8fd8\u662f\u5f88\u5411\u5f80\u6709\u4e00\u5929\u53ef\u4ee5\u7528\u4e0a\u536b\u751f\u5dfe\u5730\u3002\r\n\r\n\r\n\u3000\u5728\u8fd9\u79cd\u5411\u5f80\u6301\u7eed\u4e86\u5927\u7ea6\u4e00\u4e2a\u5b66\u671f\u4e4b\u540e\uff0c\u6211\u7ec8\u4e8e\u7b49\u5230\u673a\u4f1a\u4e86\u3002\u5927\u4e00\u7684\u5bd2\u5047\uff0c\u6211\u5728\u5e7f\u5dde\u8bfb\u5927\u5b66\u7684gf\u7559\u5728\u5b66\u6821\u8d5a\u94b1\uff0c\u5f53\u65f6\u62cd\u62d6\u4e0d\u4e45\uff0c\u6709\u4e0d\u7ecf\u5e38\u89c1\u9762\uff0c\u5f53\u7136\u5341\u5206\u60f3\u5ff5\uff0c\u4e8e\u662f\uff0c\u6211\u51b3\u5b9a\u5230\u5e7f\u5dde\u201c\u63a2\u4eb2\u201c\uff1b\u5728\u501f\u5230\u5b66\u751f\u8bc1\u3001\u4e70\u597d\u8f66\u7968\u540e\uff0c\u4e07\u5206\u6fc0\u52a8\u7684\u6211\u7a81\u7136\u8bb0\u8d77\u4e00\u4ef6\u4e8b\uff1a\u536b\u751f\u5dfe\uff1b\u5f53\u65f6\u6211\u6697\u5730\u91cc\u60f3\uff1a\u5e7f\u5dde\u6d88\u8d39\u6c34\u5e73\u591a\u9ad8\u554a\uff0c\u90a3\u91cc\u7684\u4eba\u5e94\u8be5\u4eba\u4eba\u90fd\u662f\u7528\u536b\u751f\u5dfe\u7684\u5427\uff0c\u8981\u662f\u6211\u5e26\u4e86\u4e00\u5377\u536b\u751f\u7eb8\u53bb\u90a3\u4e5f\u592a\u6ca1\u81ea\u5c0a\u4e86\uff0c\u800c\u4e14\uff0c\u4e5f\u7ed9\u6211\u4eec\u5bb6\u4e61\u7684\u5f62\u8c61\u62b9\u7070\u554a\uff0c\u4e0d\u884c\uff0c\u4e0d\u7ba1\u5b83\u6709\u591a\u8d35\uff0c\u6211\u4e00\u5b9a\u8981\u4e70\u4e00\u5377\uff08\u90a3\u65f6\u6211\u8ba4\u4e3a\u536b\u751f\u5dfe\u4e5f\u662f\u4e00\u5377\u4e00\u5377\u7684\uff09\uff0c\u8ba9\u5e7f\u5dde\u7684\u5c0f\u5b50\u770b\u770b\uff0c\u6211\u90a3\u5730\u65b9\u4e00\u6837\u90fd\u7528\u536b\u751f\u5dfe\u3002\u4e3b\u610f\u62ff\u5b9a\uff0c\u6211\u7acb\u5373\u5c31\u53bb\u4e86\u5b66\u6821\u7684\u5c0f\u5356\u90e8\uff0c\u7b14\u8e66\u62a2\u9509\u54a7\u6325?\u30015\u4e2a\u5973\u751f\u4e70\u96f6\u98df\uff0c\u6b63\u5728\u90a3\u91cc\u53fd\u53fd\u55b3\u55b3\uff0c\u6211\u6709\u70b9\u5931\u671b\uff0c\u6211\u53ef\u662f\u7b2c\u4e00\u6b21\u4e70\u8fd9\u4e48\u9ad8\u6863\u7684\u751f\u6d3b\u7528\u54c1\u554a\uff0c\u5f53\u7136\u60f3\u591a\u6b23\u8d4f\u4e00\u4e0b\u522b\u4eba\u7fa1\u6155\u7684\u76ee\u5149\uff0c\u4e0d\u8fc7\u7b97\u4e86\uff0c\u53cd\u6b63\u4e3b\u8981\u662f\u5c55\u793a\u7ed9\u90a3\u4e9b\u5e7f\u5dde\u4eba\u770b\u7684\uff0c\u4e8e\u662f\u6211\u5c31\u6307\u7740\u5305\u88c5\u6700\u534e\u4e3d\u7684\u536b\u751f\u5dfe\u5bf9\u552e\u8d27\u7684\u5c0f\u59d0\u8bf4\uff1a\u7ed9\u6211\u62ff\u4e00\u5377\u90a3\u79cd\u536b\u751f\u5dfe\uff08\u4e3a\u4e86\u800d\u9177\uff0c\u6211\u8fd9\u6b21\u4e70\u4e1c\u897f\u6ca1\u95ee\u4ef7\u94b1\uff09\uff0c\u65c1\u8fb9\u53fd\u53fd\u55b3\u55b3\u7684\u5973\u751f\u7acb\u9a6c\u9759\u4e86\u4e0b\u6765\uff0c\u9f50\u5237\u5237\u7684\u628a\u5934\u8f6c\u5411\u6211\uff0c\u552e\u8d27\u7684\u5c0f\u59d0\u4e5f\u505a\u4e86\u540c\u6837\u7684\u52a8\u4f5c\uff0c\u6211\u5fc3\u91cc\u90a3\u4e2a\u5f97\u610f\u554a\uff0c\u5fc3\u91cc\u60f3\uff1a\u539f\u6765\u800d\u9177\u8fd9\u4e48\u7b80\u5355\u7684\uff01\u4e3a\u4e86\u9177\u5230\u5e95\uff0c\u6211\u6545\u610f\u62ac\u5934\u633a\u80f8\u4e0d\u770b\u5979\u4eec\uff0c\u53ea\u76ef\u7740\u90a3\u5305\u536b\u751f\u5dfe\uff1b\u5927\u7ea65\u79d2\u949f\u4ee5\u540e\uff0c\u90a3\u5c0f\u59d0\u7ec8\u4e8e\u7f13\u8fc7\u52b2\u6765\uff0c\u7a0d\u6709\u4e9b\u8138\u7ea2\u7684\u8bf4\uff1a13\u57576\uff0c\u6211\u5413\u4e86\u4e00\u8df3\uff1a\u8fd9\u536b\u751f\u7eb8\u679c\u7136\u591f\u9ad8\u7ea7\uff0c\u7adf\u7136\u5982\u6b64\u4e4b\u8d35\uff0c\u5fc3\u91cc\u6709\u70b9\u540e\u6094\uff0c\u4f46\u662f\uff0c\u4e3a\u4e86\u4e00\u4e2a\u5b57\uff1a\u9177\uff0c\u6211\u4e00\u72e0\u5fc3\u5c31\u628a\u94b1\u62ff\u51fa\u6765\u4e86,\u7136\u540e\u6211\u81ea\u8c6a\u7684\u8fc8\u8fd9\u5927\u6b65\u8d70\u4e86\u51fa\u53bb\uff1b\u8def\u4e0a\u4eba\u7eb7\u7eb7\u4fa7\u76ee\uff0c\u6211\u5c31\u66f4\u81ea\u8c6a\u4e86\u3002\u5feb\u5230\u5bdd\u5ba4\u7684\u65f6\u5019\uff0c\u6211\u628a\u90a3\u5b9d\u8d1d\u85cf\u4e86\u8d77\u6765\uff0c\u6015\u5bdd\u5ba4\u7684\u90a3\u5e2e\u5144\u5f1f\u7ed9\u6211\u74dc\u5206\u4e86\u3002\r\n\u3000\u3000\u706b\u8f66\u4e0a\u6211\u548c\u4e09\u4e2a\u5728\u6211\u8fd9\u57ce\u5e02\u4e0a\u5b66\u7684\u5e7f\u5dde\u5b66\u751f\u5750\u5728\u4e00\u8d77\uff0c\u4e24\u7537\u4e00\u5973\uff0c\u5176\u4e2d\u6709\u4e00\u5bf9\u60c5\u4fa3\uff1b\u5927\u5bb6\u5728\u8def\u4e0a\u73a9\u7684\u5f88\u5f00\u5fc3\uff1b\u672c\u6765\uff0c\u6211\u662f\u4e0d\u51c6\u5907\u7528\u90a3\u5b9d\u8d35\u7684\u536b\u751f\u5dfe\u5730\uff0c\u4f46\u662f\uff0c\u706b\u8f66\u91cc\u6696\u6c14\u592a\u8db3\u4e86\uff0c\u611f\u89c9\u8138\u4e0a\u6709\u70b9\u7c98\u7c98\u7684\uff0c\u633a\u4e0d\u597d\u53d7\u5730\uff1b\u8fd9\u65f6\uff0c\u540c\u5750\u7684\u51e0\u4e2a\u62ff\u51fa\u4e86\u7eb8\u5dfe\uff0c\u6211\u60f3\uff0c\u53cd\u6b63\u90a3\u5305\u4e0a\u9762\u5199\u7684\u670912\u7247\u561b\uff0c\u7528\u4e00\u7247\u65e0\u59a8\uff0c\u4e0d\u8fc7\u60f3\u5230\u4e00\u7247\u4e00\u5757\u591a\uff0c\u7684\u786e\u5fc3\u75bc\u554a\uff0c\u4f46\u662f\uff0c\u5f53\u65f6\u987e\u4e0d\u4e0a\u90a3\u4e48\u591a\u4e86\uff0c\u4e8e\u662f\uff0c\u6211\u88c5\u505a\u5f88\u968f\u610f\u5730\u6837\u5b50\u62ff\u51fa\u4e86\u90a3\u5305\u7528\u6765\u6491\u95e8\u9762\u5730\u536b\u751f\u5dfe\uff1b\u7acb\u65f6\uff0c\u90a3\u4e24\u7537\u4e00\u5973\u90fd\u77aa\u5927\u773c\u775b\u770b\u7740\u6211\uff0c\u6211\u5fc3\u91cc\u60f3\uff1a\u4f60\u4e2b\u50bb\u4e86\u5427\u4f60\uff0c\u6211\u7528\u7684\u7eb8\u6bd4\u4f60\u7684\u9ad8\u7ea7\uff1b\u7136\u540e\uff0c\u6211\u5f00\u59cb\u4f18\u96c5\u5730\u60f3\u6495\u5f00\u53e3\uff0c\u4f46\u906d\u9047\u5c0f\u5c0f\u5c34\u5c2c--\u6495\u4e0d\u5f00\uff0c\u8fd9\u5f53\u7136\u96be\u4e0d\u5012\u6211\uff0c\u6211\u62ff\u51fa\u6307\u7532\u526a\uff0c\u526a\u4e86\u4e00\u4e2a\u5c0f\u53e3\u51fa\u6765\uff0c\u7136\u540e\u7528\u529b\u987a\u90a3\u5c0f\u53e3\u4e00\u6495\uff0c\u5927\u529f\u544a\u6210\u4e86\uff01\u4e8e\u662f\u6211\u62ff\u4e86\u4e00\u4e2a\u51fa\u6765\uff0c\u4e0d\u7531\u7684\u6697\u81ea\u8d5e\u53f9\uff1a\u8fd9\u4e48\u539a\uff0c\u8fd9\u4e48\u8f6f\uff0c\u602a\u4e0d\u5f97\u8fd9\u4e48\u8d35\u554a\uff1b\u7531\u4e8e\u4e4b\u524d\u6211\u6ca1\u7528\u8fc7\u8fd9\u73a9\u610f\uff0c\u4f46\u53c8\u4e0d\u80fd\u7ec6\u5fc3\u7814\u7a76\uff08\u90a3\u6837\u5c31\u8bc1\u660e\u6211\u662f\u7b2c\u4e00\u6b21\u7528\u8fd9\u79cd\u9ad8\u7ea7\u73a9\u610f\u7684\uff0c\u5176\u5b9e\u5f53\u65f6\u5f88\u5947\u602a\u4e3a\u4ec0\u4e48\u4e00\u9762\u6709\u53cc\u9762\u80f6\u8d34\u5730\uff09\uff0c\u6211\u7528\u6ca1\u6709\u80f6\u8d34\u7684\u90a3\u9762\u64e6\u4e86\u64e6\u8138\u548c\u5634;\u7136\u540e\uff0c\u6211\u60f3\uff0c\u6709\u80f6\u8d34\u7684\u90a3\u9762\u662f\u8ba9\u4f7f\u7528\u8005\u65b9\u4fbf\u5b58\u653e\u5427\uff0c\u6211\u5f88\u4e3a\u81ea\u5df1\u7684\u806a\u660e\u9a84\u50b2\uff0c\u56e0\u6b64\uff0c\u6211\u5c31\u5927\u65b9\u7684\u628a\u5b83\u8d34\u5728\u8f66\u58c1\u4e0a \uff0c\u5fc3\u60f3\uff0c\u6b63\u597d\u9760\u7a97\uff0c\u723d\u3002\r\n\u3000\u5728\u4ed6\u4eec\u60ca\u5947\u548c\u7fa1\u6155\u7684\u773c\u5149\u91cc\uff0c\u6211\u7b2c\u4e00\u6b21\u6765\u5230\u4e86\u5e7f\u5dde\uff0c\u6309\u4e00\u5411\u72ec\u7acb\uff0c\u8fd9\u6b21\u4e5f\u4e0d\u4f8b\u5916\uff0c\u4e0d\u7528gf\u63a5\uff0c\u6211\u81ea\u5df1\u5c31\u6765\u5230\u5979\u5bdd\u5ba4\u697c\u95e8\u53e3\uff0c\u4e0d\u8fc7\u8fd9\u5bdd\u5ba4\u7537\u751f\u514d\u8fdb\uff0c\u597d\u5728\u770b\u5bdd\u5ba4\u7684\u5927\u59d0\u662f\u6211\u8001\u4e61\uff0c\u5979\u597d\u5fc3\u5730\u628a\u6211gf\u53eb\u4e86\u4e0b\u6765\uff0c\u7136\u540e\uff0cgf\u5c31\u53eb\u4e86\u8bb8\u591a\u597d\u670b\u53cb\u4e0b\u6765\u5206\u4eab\u6211\u5e26\u7684\u5bb6\u4e61\u7279\u4ea7\uff0c\u7136\u540e\uff0c\u5c31\u5728\u770b\u5bdd\u5ba4\u5927\u59d0\u7684\u5c0f\u623f\u95f4\u91cc\uff0c\u6211\u5f00\u59cb\u5927\u5305\u5c0f\u5305\u7684\u5f80\u5916\u62ff\u4e1c\u897f\uff0c\u5f53\u6211\u62ff\u5230\u90a3\u5305\u536b\u751f\u5dfe\u65f6\uff0c\u6211\u6545\u610f\u628a\u5b83\u653e\u5230\u4e00\u4e2a\u663e\u773c\u7684\u5730\u65b9\uff0cgf\u5927\u60ca\uff1a\u4f60\u4e70\u8fd9\u4e2a\u5e72\u4ec0\u4e48\uff1f\u6211\u7b54\u5230\uff0c\u62ff\u6765\u7528\u554a\uff0c\u4f17\u4eba\u7686\u7206\u7b11\u2026\u2026", "Solution_1": "\u5f13\u867d\uff01" } { "Tag": [ "topology", "function", "advanced fields", "advanced fields unsolved" ], "Problem": "1. Prove that in $2^{nd}$ countable space a non countable set has an accumulation point.\r\n2. Prove that a space with a closed basis is completely regular.\r\n\r\nThanks!\r\n\r\n[b]edit[/b]: Corrected the statement of the 1st problem.", "Solution_1": "What's the first question really like? :) As it is, it's clearly false: take $\\mathbb R$ with the usual topology. It's second countable, and there are lots of uncountable closed sets.", "Solution_2": "I'm really sorry grobber. :blush: \r\n\r\nAnd now I see it's pretty easy.. :oops: \r\n[hide]\nLet [tex][/tex] be [tex]2^{nd}[/tex] countable topological space. Let [tex]A \\subseteq X[/tex] s.t. [tex]|A| > \\aleph_{0}[/tex]. Let's assume there is no accumulation point of [tex]A[/tex]. Then for any [tex]p \\in A[/tex] we have an open set [tex]G_{p}[/tex] about [tex]p[/tex] s.t. it does not contain any other points from [tex]A[/tex]. Let's define [tex]f: P(A) \\rightarrow \\tau[/tex] like this:\n\n$f(I)= \\bigcup_{\\mu \\in I} G_{\\mu}$ \n\nLet $I_1 \\neq I_2$ be subsets of $A$. Then WLOG, there is $\\mu \\in I_1$ s.t. $\\mu \\notin I_2$. This clearly means that $\\mu \\in f(I_1)$ but $\\mu \\notin f(I_2)$. Therefore, $f(I_1) \\neq f(I_2)$. So $f$ is an injective map, therefore $|\\tau| \\geq 2^{|A|} > 2^{\\aleph_0}$. But $2^{nd}$ countable space can't have topology of cardinality above $2^{\\aleph_0}$! Therefore we got our contradiction.\n$\\blacksquare$\n[/hide]", "Solution_3": "You can prove a lot more: every isolated point of the set is the only point of the set in some properly chosen basis element. This means that we can establish an injection from the set of isolated points of our set to the basis. If the basis is countable, this implies that every set has countably many isolated points, and the first question is weaker than this.\r\n\r\nAs for the second one, what do you mean by closed basis? :? Usually, bases of open sets are considered. Do you mean here that you have some open sets which form a basis for the topology and are also closed?", "Solution_4": "I mean that all the members of the basis are closed and open at once.", "Solution_5": "grobber, I don't understand how are you building your injection from the set of isolated points to the members of a countable basis.. Can you explain please?", "Solution_6": "For every isolated point you consider a basis element which contains only that point (there must be such a basis element, by the definition of an isolated point). \r\n\r\nAs for the other question, if $x$ is a point and $F\\supset x$ is a closed set not containing $x$, then take any basis element $B$ around $x$ and disjoint from $F$, and define the function you're looking for to be $0$ in $B$ and $1$ in the complementary of $B$. It's easy to check that's it's continuous.", "Solution_7": "Thanks grobber! :)" } { "Tag": [ "geometry", "parallelogram", "trigonometry", "quadratics", "trapezoid" ], "Problem": "Let ABCD is a quarilateral , $\\ S_{ABCD}=8$, $\\ AB+AC+CD=8$.Prove that ABCD is a parallelogram and find its diagonal :lol:", "Solution_1": "[quote=\"TRAN THAI HUNG\"]Let ABCD is a quarilateral , $\\ S_{ABCD}=8$, $\\ AB+AC+CD=8$.Prove that ABCD is a parallelogram and find its diagonal :lol:[/quote]\r\nIn the solution below, we use the idea that $ax^{2}+bx+c \\geq 0$ for all $x \\in \\mathbb{R}$ iff $a \\geq 0$ and $b^{2}-4ac \\leq 0 .$\r\n\r\nLet $\\angle BAC = \\alpha$ and $\\angle ACD = \\beta.$ Then we have the relation $\\frac12 (AB)(AC) (\\sin \\alpha)+\\frac12 (CD)(AC)(\\sin \\beta) = S_{ABCD}= 8$\r\n\r\n$\\Rightarrow (\\sin \\beta ){AC}^{2}+[\\sin \\beta (AB-8)-AB \\sin \\alpha] AC+16 =0$ ... (I) (quadratic in $AC$ with real roots.)\r\n\r\n$\\Rightarrow{[(\\sin \\beta (AB-8)-AB \\sin \\alpha)]}^{2}-4 \\cdot 16 \\cdot \\sin \\beta \\geq 0$\r\n\r\n$\\Rightarrow{(\\sin \\beta-\\sin \\alpha)}^{2}{AB}^{2}-16 \\sin \\beta (\\sin \\beta-\\sin \\alpha) AB+64({\\sin}^{2}\\beta-\\sin \\beta ) \\geq 0$\r\n\r\nNow using the result noted in the beginning, we obtain\r\n\r\n${16}^{2}{\\sin}^{2}\\beta{(\\sin \\beta-\\sin \\alpha)}^{2}-4{(\\sin \\beta-\\sin \\alpha)}^{2}\\cdot 64 ({\\sin}^{2}\\beta-\\sin \\beta) \\leq 0$\r\n\r\n$\\Rightarrow{(\\sin \\beta-\\sin \\alpha)}^{2}\\sin \\beta \\leq 0$\r\n\r\n$\\because \\sin \\beta \\neq 0,$ we must have $\\sin \\beta = \\sin \\alpha$ ... (II)\r\n\r\nPlugging this back into equation (I), we get $AC(8-AC) \\sin \\beta =16.$ But note that $AC$ and $AC-8$ are both positive, and so applying $AM-GM,$ we get $AC(8-AC) \\leq 16 \\Rightarrow \\sin \\beta \\geq 1 \\Rightarrow \\beta = 90^{\\circ}= \\alpha.$\r\n\r\nThus, we have $AC(8-AC) = 16 \\Rightarrow{(AC-4)}^{2}= 0 \\Rightarrow AC=4$ (the length of the diagonal.)\r\n\r\nCan someone finish this off by proving that $ABCD$ is really a parallelogram? :)", "Solution_2": "[hide][quote=\"FieryHydra\"][quote=\"TRAN THAI HUNG\"]Let ABCD is a quarilateral , $\\ S_{ABCD}=8$, $\\ AB+AC+CD=8$.Prove that ABCD is a parallelogram and find its diagonal :lol:[/quote]\nIn the solution below, we use the idea that $ax^{2}+bx+c \\geq 0$ for all $x \\in \\mathbb{R}$ iff $a \\geq 0$ and $b^{2}-4ac \\leq 0 .$\n\nLet $\\angle BAC = \\alpha$ and $\\angle ACD = \\beta.$ Then we have the relation $\\frac12 (AB)(AC) (\\sin \\alpha)+\\frac12 (CD)(AC)(\\sin \\beta) = S_{ABCD}= 8$\n\n$\\Rightarrow (\\sin \\beta ){AC}^{2}+[\\sin \\beta (AB-8)-AB \\sin \\alpha] AC+16 =0$ ... (I) (quadratic in $AC$ with real roots.)\n\n$\\Rightarrow{[(\\sin \\beta (AB-8)-AB \\sin \\alpha)]}^{2}-4 \\cdot 16 \\cdot \\sin \\beta \\geq 0$\n\n$\\Rightarrow{(\\sin \\beta-\\sin \\alpha)}^{2}{AB}^{2}-16 \\sin \\beta (\\sin \\beta-\\sin \\alpha) AB+64({\\sin}^{2}\\beta-\\sin \\beta ) \\geq 0$\n\nNow using the result noted in the beginning, we obtain\n\n${16}^{2}{\\sin}^{2}\\beta{(\\sin \\beta-\\sin \\alpha)}^{2}-4{(\\sin \\beta-\\sin \\alpha)}^{2}\\cdot 64 ({\\sin}^{2}\\beta-\\sin \\beta) \\leq 0$\n\n$\\Rightarrow{(\\sin \\beta-\\sin \\alpha)}^{2}\\sin \\beta \\leq 0$\n\n$\\because \\sin \\beta \\neq 0,$ we must have $\\sin \\beta = \\sin \\alpha$ ... (II)\n\nPlugging this back into equation (I), we get $AC(8-AC) \\sin \\beta =16.$ But note that $AC$ and $AC-8$ are both positive, and so applying $AM-GM,$ we get $AC(8-AC) \\leq 16 \\Rightarrow \\sin \\beta \\geq 1 \\Rightarrow \\beta = 90^{\\circ}= \\alpha.$\n\nThus, we have $AC(8-AC) = 16 \\Rightarrow{(AC-4)}^{2}= 0 \\Rightarrow AC=4$ (the length of the diagonal.)\n\nCan someone finish this off by proving that $ABCD$ is really a parallelogram? :)[/quote][/hide]\r\nYou can use the another way for the easier.I will post the solution later :wink:", "Solution_3": "[quote=\"FieryHydra\"]\nLet $\\angle BAC = \\alpha$ and $\\angle ACD = \\beta.$ Then we have the relation $\\frac12 (AB)(AC) (\\sin \\alpha)+\\frac12 (CD)(AC)(\\sin \\beta) = S_{ABCD}= 8$\n[/quote]\r\nFrom this $8=\\frac{AB \\sin \\alpha+CD \\sin \\beta}{2}AC \\le \\frac{AB+CD}{2}AC = \\frac{(8-AC)AC}{2}\\le 8$ and the equality holds only when $AC=4$ and $\\alpha = \\beta =90^\\circ$\r\nso [b]ABCD is trapezoid[/b]. Consider the case $AB=3$, $CD=1$. Hope I'm not wrong but it satisfies all conditions, so its impossible to prove that ABCD is paralelogram. :maybe:", "Solution_4": "[quote=\"delta\"][quote=\"FieryHydra\"]\nLet $\\angle BAC = \\alpha$ and $\\angle ACD = \\beta.$ Then we have the relation $\\frac12 (AB)(AC) (\\sin \\alpha)+\\frac12 (CD)(AC)(\\sin \\beta) = S_{ABCD}= 8$\n[/quote]\nFrom this $8=\\frac{AB \\sin \\alpha+CD \\sin \\beta}{2}AC \\le \\frac{AB+CD}{2}AC = \\frac{(8-AC)AC}{2}\\le 8$ and the equality holds only when $AC=4$ and $\\alpha = \\beta =90^\\circ$\nso [b]ABCD is trapezoid[/b]. Consider the case $AB=3$, $CD=1$. Hope I'm not wrong but it satisfies all conditions, so its impossible to prove that ABCD is paralelogram. :maybe:[/quote]\r\nAhh! I should have seen that! :) Thanks! \r\n\r\nYeah, it seems, $ABCD$ need not be a parallelogram after all.", "Solution_5": "Sorry,that my mistake it's a trapezoid :blush: :wallbash_red:" } { "Tag": [ "number theory proposed", "number theory" ], "Problem": "Let $n$ be a positive integer which has more than $k$ different prime factors. Prove that there exists a convex $n$-gon in the plane such that\r\n\r\n[b]1.[/b] all angles of the $n$-gon are equal;\r\n[b]2.[/b] the lengths of the sides of the $n$-gon are the numbers $1^{k}$, $2^{k}$, ..., $n^{k}$ in some order.\r\n\r\n Darij", "Solution_1": "As I remember there is a generalisation with $P(x)=\\sum_{i=1}^{k}a_{i}x^{i}$\r\nand the polygon with sides $P(1),P(2),...,P(n)$\r\nbut the proof is almost the same :wink:" } { "Tag": [], "Problem": "For a positive integer $n$, let $r(n)$ denote the sum of the remainders when $n$ is divided by $1, 2, \\ldots , n$ respectively. Prove that $r(k) = r(k-1)$ for\r\ninfinitely many positive integers $k$.\r\n\r\nI was just kind of screwing around with this problem and I guessed \r\n\r\n[hide]$r(2^{k})=r(2^{k}-1)$[/hide]\r\n\r\nwhich I don't know is right or not, and if it is I don't know how to prove it... :maybe:", "Solution_1": "[hide] If you let $s_{m}(n)$ be the remainder when $n$ is divided by $m$, we have $r(n) = \\sum_{m=1}^{n}s_{m}(n)$.\n\nNote that for $m > 1$\n\n$s_{m}(n) = s_{m}(n-1)+1$ if $m \\not| n$ and $s_{m}(n) = 0$ if $m | n$.\n\nSo\n\n$r(2^{k}) = \\sum_{m=2}^{2^{k}}s_{m}(2^{k}) = \\sum_{m=1}^{2^{k}-1}[s_{m}(2^{k}-1)+1]-\\sum_{i=0}^{k-1}2^{i}$.\n\nWe add $1$ to every $s_{m}$ term and subtract away the ones for which $s_{m}(2^{k}) = 0$, i.e. all powers of $2$ which were counted in the $2^{k}-1$ summation. This simplifies to\n\n$\\sum_{m=1}^{2^{k}-1}s_{m}(2^{k}-1)+\\sum_{m=1}^{2^{k}-1}1-\\sum_{i=0}^{k-1}2^{i}= r(2^{k}-1)+(2^{k}-1)-(2^{k}-1) = r(2^{k}-1)$.\n\nSo your answer is right :). [/hide]", "Solution_2": "This problem has been posted at least five times on this forum:\r\nhttp://www.mathlinks.ro/viewtopic.php?t=125625\r\nhttp://www.mathlinks.ro/viewtopic.php?t=108343\r\nhttp://www.mathlinks.ro/viewtopic.php?t=37133\r\nhttp://www.mathlinks.ro/viewtopic.php?t=5866\r\n\r\nA follow up question: Prove that $ \\frac{n^2}{10}0.$ If $ k \\ge \\frac{64}{105},$ then\r\n$ a\\plus{}b\\plus{}c \\ge \\sqrt{\\frac{a^2\\plus{}k}{1\\plus{}k}}\\plus{}\\sqrt{\\frac{b^2\\plus{}k}{1\\plus{}k}}\\plus{}\\sqrt{\\frac{c^2\\plus{}k}{1\\plus{}k}}.$", "Solution_1": "[quote=\"can_hang2007\"]Let $ a,$ $ b,$ $ c$ be nonnegative real numbers such that $ a \\plus{} b \\plus{} c \\equal{} ab \\plus{} bc \\plus{} ca > 0.$ If $ k \\ge \\frac {64}{105},$ then\n$ a \\plus{} b \\plus{} c \\ge \\sqrt {\\frac {a^2 \\plus{} k}{1 \\plus{} k}} \\plus{} \\sqrt {\\frac {b^2 \\plus{} k}{1 \\plus{} k}} \\plus{} \\sqrt {\\frac {c^2 \\plus{} k}{1 \\plus{} k}}.$[/quote]\r\n\r\n\r\n\r\n$ k\\geq 2$\r\n\r\n$ (a \\plus{} b \\plus{} c)^2 \\ge 3\\sum {\\frac {a^2 \\plus{} k\\frac {(ab \\plus{} bc \\plus{} ca)^2}{(a \\plus{} b \\plus{} c)^2}}{1 \\plus{} k}}$." } { "Tag": [ "FTW", "AMC", "AIME", "analytic geometry" ], "Problem": "after my failed last tourny... :( i've decided to make another one\r\nhopefully one that'll do better\r\nanyway, here are the rules\r\n\r\nthis will be a 16-player tourny, open to all players\r\nwhen you sign up, plz give me your highest rating, as seeding will be done by this\r\nIt will be a normal game, 10 questions, 12 seconds, time based\r\nwhether it's rated or non-rated, will be up to the players\r\n\r\nplz sign up here and i wish all participants good luck!!!", "Solution_1": "pinkpiglet signs up!!!! highest: 1323 :blush:", "Solution_2": "WHEN WILL THE MADNESS END???? JUST SO YOU KNOW I, MEWTO55555, WILL BOYCOTT [b][u]ALL FUTURE TOURNAMENTS[/b][/u] UNTIL THE EXCESS AMOUNT OF TOURNAMENTS ENDS.", "Solution_3": "1367 highest\r\nWIILL GET HIGHER SOON!", "Solution_4": "[quote=\"mewto55555\"]WHEN WILL THE MADNESS END???? JUST SO YOU KNOW I, MEWTO55555, WILL BOYCOTT [b][u]ALL FUTURE TOURNAMENTS[/b][/u] UNTIL THE EXCESS AMOUNT OF TOURNAMENTS ENDS.[/quote]\r\n\r\n....didnt u start the first tourney...", "Solution_5": "Please ignore this post.", "Solution_6": "[quote=\"ernie\"]Please ignore this post.[/quote]\r\n\r\nu just [i]had[/i] to spam in my topic didn't u?\r\nwell, i'll do that 2 urs to :D \r\nMHAHAHAHAHA (i can't find an evil scientist's face...)", "Solution_7": "OK\r\nthis is the bracket\r\n\r\nROUND 1 \r\ngame 1: person 1 vs. person 16 \r\ngame 2: person 2 vs. person 15 \r\ngame 3: person 3 vs. person 14 \r\ngame 4: person 4 vs. person 13 \r\ngame 5: person 5 vs. person 12 \r\ngame 6: person 6 vs. person 11 \r\ngame 7: person 7 vs. person 10 \r\ngame 8: person 8 vs. person 9 \r\n\r\n\r\nWINNER'S BRACKET \r\nROUND 2 \r\ngame 9: winner of game 1 vs. winner of game 8 \r\ngame 10: winner of game 2 vs. winner of game 7 \r\ngame 11: winner of game 3 vs. winner of game 6 \r\ngame 12: winner of game 4 vs. winner of game 5 \r\n\r\n\r\nROUND 3 \r\ngame 13: winner of game 9 vs. winner of game 12 \r\ngame 14: winner of game 10 vs. winner of game 11 \r\n\r\n\r\nROUND 4 \r\ngame 15: winner of game 13 vs. winner of game 14 \r\n\r\nLOSER'S BRACKET \r\nROUND 2 \r\ngame 16: loser of game 1 vs. loser of game 8 \r\ngame 17: loser of game 2 vs. loser of game 7 \r\ngame 18: loser of game 3 vs. loser of game 6 \r\ngame 19: loser of game 4 vs. loser of game 5 \r\n\r\n\r\nROUND 2b \r\ngame 20: winner of game 16 vs. loser of game 9 \r\ngame 21: winner of game 17 vs. loser of game 10 \r\ngame 22: winner of game 18 vs. loser of game 11 \r\ngame 23: winner of game 19 vs. loser of game 12 \r\n\r\nROUND 3 \r\ngame 24: winner of game 20 vs. winner of game 23 \r\ngame 25: winner of game 21 vs. winner of game 22 \r\n\r\nROUND 3b \r\ngame 26: winner of game 24 vs. loser of game 13 \r\ngame 27: winner of game 25 vs. loser of game 14 \r\n\r\nROUND 4 \r\ngame 28: winner of game 26 vs. winner of game 27 \r\n\r\nROUND 4b \r\ngame 29: winner of game 28 vs. loser of game 15 \r\n\r\n\r\n\r\nCHAMPIONSHIP ROUND \r\ngame 30: winner of game 15 vs. winner of game 29", "Solution_8": "whoo hoo WOOT i join! 1357", "Solution_9": "ooh i wanna join\r\nTOURNEY FEVER :O\r\n\r\nhighest 1404", "Solution_10": "i'm joining 1369", "Solution_11": "ok, \r\nsoooo, pplz participating so far\r\n\r\n#1. Xoangieexo\u2026\u2026.1404\r\n#2. Johnnybeard\u2026\u20261367\r\n#3. Mathblitz\u2026\u2026\u2026.1357\r\n#4. Pinkpiglet\u2026\u2026\u2026.1323", "Solution_12": "you forgot sicarius", "Solution_13": "oops, right\r\nsrry sicarius\r\n\r\n#1. Xoangieexo\u2026\u2026.1404\r\n#2. Sicarius1029\u2026\u20261369\r\n#3. Johnnybeard\u2026\u20261367\r\n#4. Mathblitz\u2026\u2026\u2026.1357\r\n#5. Pinkpiglet\u2026\u2026\u2026.1323\r\n\r\nwe just need 11 more pplz to get started!!\r\ntell all ur friends!!!!", "Solution_14": "um...13someting...", "Solution_15": "changes in bracket\r\nwinners will be decided by the coin toss\r\n\r\nwinner's bracket round 3 and loser's bracket round 2 must be completed july 30th\r\nnext deadline will be after the weekend, during that time, i will not be here because i'm going to acadia(not sure if that's how u spell it)\r\ni will give out instructions and pms on July 31st, before i leave \r\n\r\nROUND 1. \r\n#5. AIME_is_hard\u2026\u2026...1375 vs #9. Mathblitz(winner due to coin toss)\u2026\u2026\u2026..\u2026.1357 (game 1) \r\n#6. AIME15 (winner)\u2026\u2026....\u2026\u2026\u20261370 vs#10. Pinkpiglet\u2026\u2026\u2026\u20261323 (game 2) \r\n#7. Sicarius1029(winner due to ducky's FTW failing)\u2026\u2026..\u20261369 vs#11. Myyellowducky82...1200 (game 3) \r\n#8. Johnnybeard(winner due to coin toss)\u2026\u2026..\u20261367 vs#12. Pacman2812\u2026\u2026\u2026.1200 (game 4) \r\n\r\nWINNER'S BRACKET \r\nROUND 2. \r\nKl2836 vs. Johnnybeard(winner due to kl2836 not responding) (game 5) \r\nBudi713 vs. Sicarius1029(winner due to coin toss) (game 6) \r\nXoangieexo vs. AIME15(winner) (game 7) \r\nBisbmard9(winner due to coin toss) vs. Mathblitz (game 8) \r\n\r\nROUND 3 \r\n[b]Johnnybeard [/b]vs. [b]Bisbmard9 [/b](game 9) \r\n[b]Sicarius1029 [/b]vs. [b]AIME15[/b](game 10) \r\n\r\nROUND 4 \r\nwinner of game 9 vs. winner of game 10 (game 11) \r\n\r\nLOSER'S BRACKET \r\nROUND 2 \r\n[b]AIME_is_hard [/b]vs. [b]Mathblitz [/b](game 12) \r\nPinkpiglet(winner) vs. xoangieexo (game 13) \r\n[b]Myyellowducky82 [/b]vs. [b]Budi713 [/b](game 14) \r\n[b]pacman2812 [/b]vs. [b]Kl2836 [/b](game 15) \r\n\r\nROUND 2a \r\nwinner of game 15 vs. winner of game 14 (game 16) \r\nwinner of game 12 vs. [b]Pinkpiglet [/b](game 17) \r\n\r\nROUND 3. \r\nwinner of game 16 vs. loser of game 9 (game 18) \r\nwinner of game 17 vs. loser of game 10 (game 19) \r\n\r\nROUND 3a \r\nwinner of game 18 vs. winner of game 19 (game 20) \r\n\r\nROUND 4. \r\nwinner of game 20 vs. loser of game 11 (game 21) \r\n\r\nCHAMPIONSHIP ROUND \r\n\r\nWINNER OF GAME 21 vs. WINNER OF GAME 11 \r\n\r\nIF THE WINNER OF THE LOSER'S BRACKET WINS THE CHAMPIONSHIP ROUND, THEN THE ROUND MUST BE PLAYED AGAIN \r\n\r\n[b][i][u]games that can be played right now[/u][/i][/b]\r\n[b]Johnnybeard [/b]vs. [b]Bisbmard9 [/b] (bisbmard9 has responded, if the game does not take place for any reason, and johnnybeard does not respond, then bisbmard wins)\r\n[b]Sicarius1029 [/b]vs. [b]AIME15[/b] (sicarius1029 has responded, if the game does not take place for any reason, and AIME15 does not respond, then sicarius1029 wins)\r\n[b]AIME_is_hard [/b]vs. [b]Mathblitz [/b] (BTW, i haven't seen AIME_is_hard on in a long time, so if this game doesn't get completed by the deadline, mathblitz wins)\r\n[b]Myyellowducky82 [/b]vs. [b]Budi713 [/b]\r\n[b]pacman2812 [/b]vs. [b]Kl2836 [/b]\r\n\r\ni will send out reminders tomorrow as well, when i get on AoPS on july 30, any games not played will be decided by the above criteria and the coin toss", "Solution_16": "Vallon- if you do a coin toss, wouldn't that just defeat the whole purpose of an FTW tourney? Unless you're doing a coin-tossing tourney... :rotfl:", "Solution_17": "ya, to a certain extent, but i'm just trying to get the tourney to keep moving, that way, the people actually participating can play new pplz\r\nanyway, i have told pplz that if they respond, but their opponent doesn't, they get the win, so....\r\nit's not completely coin toss\r\nthe people that play move on, the people that don't play lose", "Solution_18": "[quote=\"vallon22\"]ya, to a certain extent, but i'm just trying to get the tourney to keep moving, that way, the people actually participating can play new pplz\nanyway, i have told pplz that if they respond, but their opponent doesn't, they get the win, so....\nit's not completely coin toss\nthe people that play move on, the people that don't play lose[/quote]\r\n\r\nBut that [b]completely[/b] would defeat the purpose of any math tourney. This is (no offense) like a probablity/social reply tourney.", "Solution_19": "well, i just wanna get this tourney moving\r\nhopefullly, some pplz will actually play\r\n\r\non that note, i have postponed the deadline for the games in winner's bracket round 3 and loser's bracket round 2 until August 1\r\n\r\ngood luck with your games", "Solution_20": "Bah, I need to prepare to a selection test and make homeworks. No time to FTW\r\n\r\nkl(something) wins.", "Solution_21": "ok\r\npacman forfeits so....\r\n\r\nROUND 1. \r\n#5. AIME_is_hard\u2026\u2026...1375 vs #9. Mathblitz(winner due to coin toss)\u2026\u2026\u2026..\u2026.1357 (game 1) \r\n#6. AIME15 (winner)\u2026\u2026....\u2026\u2026\u20261370 vs#10. Pinkpiglet\u2026\u2026\u2026\u20261323 (game 2) \r\n#7. Sicarius1029(winner due to ducky's FTW failing)\u2026\u2026..\u20261369 vs#11. Myyellowducky82...1200 (game 3) \r\n#8. Johnnybeard(winner due to coin toss)\u2026\u2026..\u20261367 vs#12. Pacman2812\u2026\u2026\u2026.1200 (game 4) \r\n\r\nWINNER'S BRACKET \r\nROUND 2. \r\nKl2836 vs. Johnnybeard(winner due to kl2836 not responding) (game 5) \r\nBudi713 vs. Sicarius1029(winner due to coin toss) (game 6) \r\nXoangieexo vs. AIME15(winner) (game 7) \r\nBisbmard9(winner due to coin toss) vs. Mathblitz (game 8) \r\n\r\nROUND 3 \r\n[b]Johnnybeard [/b]vs. [b]Bisbmard9 [/b](game 9) \r\n[b]Sicarius1029 [/b]vs. [b]AIME15[/b](game 10) \r\n\r\nROUND 4 \r\nwinner of game 9 vs. winner of game 10 (game 11) \r\n\r\nLOSER'S BRACKET \r\nROUND 2 \r\n[b]AIME_is_hard [/b]vs. [b]Mathblitz [/b](game 12) \r\nPinkpiglet(winner) vs. xoangieexo (game 13) \r\n[b]Myyellowducky82 [/b]vs. [b]Budi713 [/b](game 14) \r\npacman2812 vs. Kl2836(winner) (game 15) \r\n\r\nROUND 2a \r\n[b]Kl2836 [/b]vs. winner of game 14 (game 16) \r\nwinner of game 12 vs. [b]Pinkpiglet [/b](game 17) \r\n\r\nROUND 3. \r\nwinner of game 16 vs. loser of game 9 (game 18) \r\nwinner of game 17 vs. loser of game 10 (game 19) \r\n\r\nROUND 3a \r\nwinner of game 18 vs. winner of game 19 (game 20) \r\n\r\nROUND 4. \r\nwinner of game 20 vs. loser of game 11 (game 21) \r\n\r\nCHAMPIONSHIP ROUND \r\n\r\nWINNER OF GAME 21 vs. WINNER OF GAME 11 \r\n\r\nIF THE WINNER OF THE LOSER'S BRACKET WINS THE CHAMPIONSHIP ROUND, THEN THE ROUND MUST BE PLAYED AGAIN \r\n\r\n[u][i][b]games that can be played right now [/b][/i][/u]\r\n[b]Johnnybeard [/b]vs. [b]Bisbmard9 [/b](bisbmard9 has responded, if the game does not take place for any reason, and johnnybeard does not respond, then bisbmard wins) \r\n[b]Sicarius1029 [/b]vs. [b]AIME15 [/b](sicarius1029 has responded, if the game does not take place for any reason, and AIME15 does not respond, then sicarius1029 wins) \r\n[b]AIME_is_hard[/b] vs. [b]Mathblitz [/b](BTW, i haven't seen AIME_is_hard on in a long time, so if this game doesn't get completed by the deadline, mathblitz wins) \r\n[b]Myyellowducky82 [/b]vs. [b]Budi713 [/b]", "Solution_22": "I forfeit. (spelling?)", "Solution_23": "[quote=\"myyellowducky82\"]I forfeit. (spelling?)[/quote]\r\n\r\nSpelling is correct. :wink:\r\n\r\nYAY! My 800th post!\r\n\r\nAnd vallon, no offense, but this tourney is in serious need of cancelling.", "Solution_24": "ya, ur prob right\r\nbut right now, hardly any tournies are up and going\r\nhmm...\r\ni'll think of something l8r\r\n\r\nTHIS TOURNEY IS OFFICIALLY CANCELLED :( :( :(", "Solution_25": "since its cancelled, admins (no mods for this forum) please lock this thread?", "Solution_26": "admins hardly come in here\r\nthe easiest way is for pplz to just stop posting\r\ndang, [i]i'm[/i] posting....", "Solution_27": "I'LL JOIN.THOUGH I'LL LOSE IN THE FIRST ROUND. HIGHEST RATING 1321", "Solution_28": "It's canceled, Andrew. At least read the post right before you.", "Solution_29": "This tourney is canceled.\r\n11111111111111111111111\r\n11111111111111111111111\r\n11111111111111111111111\r\n11111111111111111111111\r\n11111111111111111111111\r\n11111111111111111111111\r\n :D :D :D" } { "Tag": [ "trigonometry", "geometry", "trig identities", "Law of Sines", "geometry unsolved" ], "Problem": "I need help, this is the only topic I am really struggling at.\r\nThis may seem rude as its my first post, but if anyone is kind enough! :) \r\n\r\n--------\r\n\r\n\r\n[img]http://i14.tinypic.com/2heatyd.jpg[/img]\r\n\r\nShow that, for triangle RST, t = r sin (alpha + beta) [b]/[/b] sin alpha\r\n\r\n\r\n\r\niv tried using the sine rule,\r\nt [b]/[/b] sin(180 - alpha - beta) [b]=[/b] r [b]/[/b] sin alpha\r\nt = r / sin alpha [b]x[/b] sin(180 - alpha - beta)\r\n\r\nAfter that, i get confused because usually i would expand the sin bracket. ie:\r\n[b]sin(45+45) = sin 45 cos 45 + cos 45 sin 45[/b]\r\n\r\nbut in this case, i have 180, alpha, and beta in one bracket.......\r\n\r\nAm i even taking the right steps?\r\nPlease help.", "Solution_1": "The angle measure of $T=180-(a+b)$\r\n\r\nBy law of sines,:\r\n$\\frac{r}{\\sin a}=\\frac{t}{\\sin (180-(a+b))}$\r\n\r\nFor any number $x$, $\\sin x = \\sin (180-x)$\r\n\r\nSo you get:\r\n$\\frac{r}{\\sin a}=\\frac{t}{\\sin (a+b)}$\r\n$\\implies \\boxed{t=\\frac{r\\sin (a+b)}{\\sin a}}$", "Solution_2": "thanks :)" } { "Tag": [ "function", "special factorizations" ], "Problem": "Please help me. I'm struggling over math.\r\n\r\n\r\n\r\nKindly show the complete solutions.. THANKS ;]", "Solution_1": "ok we can rewrite n(n) as $ n^2$. it is a well-known theorem that the minimum of a parabolic function (or maximum of a negative one) comes when $ x\\equal{}\\minus{}\\frac{b}{2a}$ which in this case is $ n\\equal{}\\boxed{600}$ plugging that in, we get $ c\\equal{}\\boxed{600}$ as well. coooooollll....", "Solution_2": "We see this by completing the square (never liked formulas anyway :) ):\r\n\\begin{align*}\r\nc &= n^2 - 120n + 4200 \\\\\r\n&= n^2 - 120n + 3600 + 600 \\\\\r\n&= (n - 60)^2 + 600.\r\n\\end{align*}\r\nSince something squared is always nonnegative, we easily see that the minimum value of c is 600, since the smallest $ (n-60)^2$ can be is 0." } { "Tag": [ "search", "combinatorics unsolved", "combinatorics" ], "Problem": "A palindrome is a number or word that is the same when read forward and backward, for example, \"176671\" and \"civic.\" Can the number obtained by writing the number from 1 to $ n$ in order ($ n>1$) be a palindrome?", "Solution_1": "This is Russia 1996. It has appeared many times on the forum, and it's easy enough to find with a search for the word \"palindrome.\"\r\nhttp://www.artofproblemsolving.com/Forum/viewtopic.php?t=128508\r\nhttp://www.artofproblemsolving.com/Forum/viewtopic.php?t=19872" } { "Tag": [ "geometry", "circumcircle", "induction", "extremal principle", "combinatorics unsolved", "combinatorics" ], "Problem": "Let $ A_1A_2...A_n$ be a convex polygon. Show that there exists an index $ j$ such that the circum-circle of the triangle $ A_j A_{j \\plus{} 1} A_{j \\plus{} 2}$ covers the polygon (here indices are read modulo n).", "Solution_1": "[hide=\"Hint\"]Take a circle that covers the polygon and try to make it get smaller until it becomes a circumcircle of some triangle.[/hide]", "Solution_2": "I don't think that hint will work for me. I mean.... how will you show that that circle will always become some triangle's circumcircle?", "Solution_3": "i would say try to find a triangle (general) whose circumcircle covers the n gon and then try to create more triangles", "Solution_4": "[quote=\"aakansh92\"]Let $ A_1A_2...A_n$ be a convex polygon. Show that there exists an index $ j$ such that the circum-circle of the triangle $ A_j A_{j \\plus{} 1} A_{j \\plus{} 2}$ covers the polygon (here indices are read modulo n).[/quote]\r\n\r\n[hide=\"Hint to the solution\"]\nProve that the biggest circle possible through any 3 vertices is\ni) :arrow: covering the polygon\nii) :arrow: passing through three consequtive points.\n\nboth can be proved by contradiction method. :) [/hide]", "Solution_5": "Please be more clear and give full solution.\r\n\r\nGaurav, could you do that problem during the postals? If yes, I am eager to look at the solution.", "Solution_6": "Aaknash, were u at the IMOTC last year?", "Solution_7": "Here's my (not well-written :oops: ) solution. Hope you understand the idea.\r\n\r\n[hide]I don't know if the following requires proof. :| \n[b]Lemma 1:[/b] \nSuppose $ S$ is a circle which contains $ Y,Z$ but doesn't contain $ X$.\nThen the portion of the circumference of $ S$ that lies opposite $ X$ wrt $ YZ$ lies outside the circumcircle of $ XYZ$\n [geogebra]8311777ddf6748db7b929f40c8136d68ab2b9775[/geogebra]\n\nSuppose the problem statement is true for all convex $ n$-gon for some $ n\\ge 4$\n(Base case $ 4$ is easy to prove)\nAssume there exists a convex $ n \\plus{} 1$-gon $ A_1,A_2,\\cdots A_{n \\plus{} 1}$ such that the statement isn't true.\n\nBy induction hypothesis on the polygon $ A_1A_2\\cdots A_n$, there exists $ A_j,A_{j \\plus{} 1},A_{j \\plus{} 2}$ such that exactly one vertex of the polygon, $ A_{n \\plus{} 1}$ lies outside their circumcircle.\n\nBy induction hypothesis on the polygon $ A_1A_2\\cdots A_jA_{j \\plus{} 2}\\cdots A_{n \\plus{} 1}$, exactly one vertex of the polygon, $ A_{j \\plus{} 1}$ lies outside a circle $ S$ that has on it's circumference 3 consecutive points from the polygon.\n [geogebra]a1b4053b21d62e5195188b71d634b48bdaf91f91[/geogebra] \nBut none of these three points lie on the same side as $ A_{j \\plus{} 1}$ wrt $ A_jA_{j \\plus{} 2}$. But since $ A_{j \\plus{} 1}$ lies outside $ S$, by lemma 1 such three points can't lie inside the circumcircle of $ A_jA_{j \\plus{} 1}A_{j \\plus{} 2}$. So these 3 points must be $ A_j,A_{j \\plus{} 2},A_{n \\plus{} 1}$. But these are consecutive only if $ n \\equal{} 3$. Which isn't true because $ n\\ge 4$[/hide]\r\nDo reply if you understand or not. :oops:", "Solution_8": "Ah... Sorry... When I replied, I was thinking in another problem.\r\n\r\nAkashnil, I think you should replace three of your $ A_{j \\plus{} 1}$'s by $ A_{j \\plus{} 2}$'s. Anyway, what is wrt?", "Solution_9": "wrt = with relation to, or something similar.", "Solution_10": "wrt=with respect to.\r\n$ A$ is opposite $ B$ wrt $ CD$ means the points $ A$ and $ B$ lie on oposite sides of the line $ CD$.\r\nYes I messed up some of $ A_{j\\plus{}1}$ and $ A_{j\\plus{}2}$s . Sorry. I've edited now. Thanks.\r\n\r\nAnd once again do reply if you understand it now or not.", "Solution_11": "[quote=\"aakansh92\"]Please be more clear and give full solution.\n\n[/quote]\r\nSorry for incomplete post, actually i dont like Latexing much. anyway [hide=\"hare goes the proof\"]\nAmong the finitely many circles through any 3 vertices, there is[b] [u]a maximal one, denote it by $ \\tau$[/u][/b].\n\nas said earlier we prove:\n1. :arrow: $ \\tau$ covers n-gon.\n2. :arrow: $ \\tau$ passes through 3 consequtive vertices.\n\n$ 1.$ Suppose point A' lies outside the maximal circle about $ \\triangle ABC$ where $ A,B,C,A'$\nare vertices of polygon so that they form a convex quadrilateral. Then circumcircle of $ \\triangle A'BC$ has larger radius than $ \\triangle ABC$. :!: CONTRADICTION. :) \n$ 2.$ Let $ A_i,A_j,A_k$ be vertices on the MAXIMAL circle. Let $ A_l$ lie between $ A_j \\& A_k$, not on $ \\tau$. Because of (1), it lies inside $ \\tau$, but then circle about $ \\triangle A_jA_lA_k$ is larger than maximal circumcircle. :!: CONTRADICTION. :) [/hide].\r\nplease verify the solution.", "Solution_12": "[quote=\"Akashnil\"]I don't know if the following requires proof. :| \n[b]Lemma 1:[/b] \nSuppose $ S$ is a circle which contains $ Y,Z$ but doesn't contain $ X$.\nThen the portion of the circumference of $ S$ that lies opposite $ X$ wrt $ YZ$ lies outside the circumcircle of $ XYZ$\n [/quote]\r\nP.S.- \r\nabove lemma can be easily proved since two circles(not identical) can intersect at only two points.\r\nP.P.S.- AKashnil, your proof is really very good. :coolspeak:", "Solution_13": "Yours too :coolspeak:", "Solution_14": "those two solutions are god damn BEAUTIFUL\r\nhats off to you both ith_power and Akashnil\r\nmy solution amounted to atleast 5 whole written pages in Ideas but yours(both) is just beauty", "Solution_15": "That's really wonderfull.Thanks!", "Solution_16": "It is very unfortunate that this problem was taken from the very well known Arthur Engel book's Extremal principle chapter. Infact it is an example problem. Too bad of the problem setters in India.\r\nAnyway, akashnil's solution is superb as it is a different one .\r\nith_power ur solution is superb as u have used the extremal principle well. I also solved it by that way and even in the book they solved it in the same way.\r\n\r\n\r\n\r\n\r\nModerators if this is an unnecessary post sorry but just wanted to say that it was copied thats all.", "Solution_17": "engel does not state consecative", "Solution_18": "Enegel does state consecutive. :D" } { "Tag": [ "calculus", "integration", "floor function", "calculus computations" ], "Problem": "Here's another one that was interesting:\r\n\r\nDetermine whether the improper integral\r\n\r\n\\[ \\int_0^\\infty (\\minus{}1)^{\\lfloor x^2 \\rfloor}dx\r\n\\]\r\n\r\nconverges or diverges.", "Solution_1": "[hide]The question is equivalent to the convergence of the series $ \\sum_{k \\equal{} 0}^{\\infty} ( \\minus{} 1)^k(\\sqrt {k \\plus{} 1} \\minus{} \\sqrt {k})$ (here $ k$ replaces $ \\lfloor x^2\\rfloor$. The series converges by the alternating series test.[/hide]" } { "Tag": [ "LaTeX" ], "Problem": "I want to make my reference goes like this:\r\n\r\nAmacc, D and JXIK, L. 2009, paper compilation, journal of xx, 202-241\r\n\r\nBxxx, L and wxx, D. 2008, xxxx, .\r\n\r\nCxxx, ..\r\n\r\nBasically, I want the references ordered by the alphabetical order of authors' last name.(.Amacc/0 is last name..and then first name. \r\n\r\nwhich bibliography style should I use with \\usepackage{natbib} ?\r\n\r\nI download the APA from CTAN..But when I tried to install it to my TexnicCenter, it fails..No ideas what I should do now..pls help me with the procedure of downloading new bibliography style.", "Solution_1": "[quote=\"weng\"]I download the APA from CTAN..But when I tried to install it to my TexnicCenter, it fails..No ideas what I should do now..pls help me with the procedure of downloading new bibliography style.[/quote]This makes no sense. TexnicCenter is merely an editor so you can't install anything to it. What is \"the APA\"? What does \"it fails\" mean? MiKTeX already comes with APA like styles in it such as apacite and apalike so there's nothing to install. See MiKTeX, Browse Packages for what is installed.\r\n\r\nAlso look at the sort-by-letters styles already included in MiKTeX. These are described [url=http://tug.ctan.org/tex-archive/biblio/bibtex/contrib/sort-by-letters/README]here[/url].", "Solution_2": "when i used \\bibliographystyle{apa}\r\n\r\nthe page of Bibliography just disappear..\r\n\r\nI tried to install apa into MikTek....", "Solution_3": "[quote=\"weng\"]when i used \\bibliographystyle{apa}\n\nthe page of Bibliography just disappear..[/quote]You should know by now that you must give a minimal example to have any chance of analyzing the problem.\n[quote=\"weng\"]I tried to install apa into MikTek....[/quote]Why? It's already installed assuming you have the full version. Again you haven't given any information to work with. What did you do to install it? Didy you use the MiKTeX install procedure in Browse Packages? Did you install it manually? If so, did you refresh the database?", "Solution_4": "Thank you for clarifying those to me!!!!! \r\nI get the problem fixed.." } { "Tag": [ "ARML", "email" ], "Problem": "I pm-ed Paul about ARML but he hasn't replied. Does anyone know who I should contact and what the requirements are?", "Solution_1": "For Iowa ARML, you're looking for James Kirpes. Email is: kirpes.james@iccsd.k12.ia.us. Hm...I'm not entirely sure of requirements. Usually you have to have scored well (placed) in GPML meets. AMC scores are also factored in. I've never had too much trouble getting on a team. If you're a regular on this site, I doubt you will either.", "Solution_2": "Oh, looks like Darren took care of it. I probably should have just given you the email from the beginning, Penguin. <_<;;\r\n\r\nOh well.", "Solution_3": "Sorry to call up an old post, but who in here is going to the Iowa City ARML this June?\r\n\r\nFor those of you that have done it before, how are the three teams divided up?", "Solution_4": "Well, I'm going. :lol: \r\n\r\nThere's a Central Academy team, I'm pretty sure. And then the other two teams are divided according to how Kirpes views their mathematical ability. (based on experience with the team and AMC scores and such)\r\n\r\nI think. Darren or Nick, do you know anything?", "Solution_5": "woohoo! im going. hope to c u guys there :)", "Solution_6": "Yeah, you're pretty much right Paul. CA's got their team. And then Kirpes divides people based on who's good and who's not. More or less.", "Solution_7": "Generally speaking, what is the cutoff for the two teams (based on AMC)? This could be skewed, as many people (like me) who take a lot of time suck at speed-type contests (over 1 hour).", "Solution_8": "I know from central 3 8th graders are going (me being one of them :P ), 1 or 2 freshmen, and 6ish seniors and juniors.", "Solution_9": "I don't think there is a specific cutoff. It's mostly qualitative, rather than quantitative. Don't worry! If you guys aren't on the first team this year, then chances are that a decent performance at ARML will put you on the top team. At least, that's what I think is true.\r\n\r\nDarren, Allendorf, some help here?" } { "Tag": [], "Problem": "Each page number of a 599-page book is printed one time in the book. The first page is page 1 and the last page is page 599. When printing all of the page numbers, how many more 5's are printed than 8's?\r\n\r\nHide your solutions\r\n\r\nBilly", "Solution_1": "[color=indigo]\n[hide]There are 100 more 5's than 8's. First i counted the number of 5's: Between 1-99 there are 20. Between 100-199 there are 20. Same for the 200's, the 300's and the 400's. Then in the 500's there are 120 5's. THat mkes a total of 220 fives. Then the 8's are pretty much the same except they don't get the extra 100 in the 500's so there are 120 8's. :D [/color][/hide]", "Solution_2": "[hide]If you realize, the 5's have 500-599. 8 dont.\nOtherwise, they're the same\nThere are 100 pages in 500s\n>>100<<[/hide]", "Solution_3": "[hide]From 1-599, 8 shows up 9 * 6 times in the ones spot, 9 * 6 times in the 10s spot and 0 times in the hundreds spot. 5 shows up 9 * 6 times in the ones spot, 9 * 6 times in the 10s spot and 100 times in the hundreds spot, so its 100 more. [/hide]", "Solution_4": "so why don't you edit your post, or reenter another to fix it?" } { "Tag": [ "function", "search", "complex analysis", "complex analysis unsolved" ], "Problem": "Suppose that $f,g$ are an analytic functions on $\\mathbb C$. Prove that if $f,g$ have no common roots, then there exist analytic functions $A,B$ whcih $Af+Bg=1$.", "Solution_1": "Leva proved it (on the forum) a while back. I'm sure it's easy to locate using the search function.", "Solution_2": "[url=http://www.artofproblemsolving.com/Forum/viewtopic.php?t=46388]The topic grobber was referring to.[/url]" } { "Tag": [ "trigonometry", "calculus", "calculus computations" ], "Problem": "I need to solve a problem from physics, but first i need to solve these differential equations to find the frequence of two moving rods.\r\n$\\begin{array}{l}\\left\\{ \\begin{array}{l}\\varphi_{1}= \\frac{{\\varphi_{0}}}{2}[\\cos \\omega_{1}t+\\cos \\omega_{2}t] \\\\ \\varphi_{2}= \\frac{{\\varphi_{0}}}{2}[\\cos \\omega_{1}t-\\cos \\omega_{2}t] \\\\ \\end{array}\\right.\\\\ \\end{array}$\r\nwhere\r\n$\\omega_{1}^{2}= \\omega_{0}^{2}+\\omega^{'2}\\\\ \\omega_{2}^{2}= \\omega_{0}^{2}-\\omega^{'2}$", "Solution_1": "So, any idea?", "Solution_2": "Shouldn't this be done using the addition of fazors, in a fazorial graph?", "Solution_3": "What's fazorial graph? I need to solve the differential equations.", "Solution_4": "[quote=\"Nea\"]What's fazorial graph? I need to solve the differential equations.[/quote]\r\nYour original post does not contain any differential equations. It is unclear what you are asking.", "Solution_5": ":blush: \r\nI've erroneously written the previous equations. You arrive to them after some substitutions to these:\r\n$\\begin{array}{l}ml^{2}\\varphi_{1}^{''}+mgl\\varphi_{1}+ca^{2}(\\varphi_{1}+\\varphi_{2}) = 0 \\\\ ml^{2}\\varphi_{2}^{''}+mgl\\varphi_{2}+ca^{2}(\\varphi_{1}+\\varphi_{2}) = 0 \\\\ \\end{array}$\r\nwhere:\r\n$\\begin{array}{l}\\varphi_{1}+\\varphi_{2}= \\varphi \\\\ \\varphi_{1}-\\varphi_{2}= \\varphi \\\\ \\end{array}$\r\nAnyway only these equations, that I wrote now, count. I've arrived at the previous equations because of the initial conditions I had in the problem.", "Solution_6": "I was wondering if you needed $mgl\\varphi_{1}'$ as opposed to $mgl\\varphi_{1}$.\r\nYou wanted to let people know what $m,l,c,a$ are (constants I guess). \r\nI was wondering if something is wrong with the following array because after solving it for $\\varphi$ something is not true about $\\varphi_{2}$\r\n$\\begin{array}{l}\\varphi_{1}+\\varphi_{2}= \\varphi \\\\ \\varphi_{1}-\\varphi_{2}= \\varphi \\\\ \\end{array}$", "Solution_7": "[quote=\"akech\"]I was wondering if you needed $mgl\\varphi_{1}'$ as opposed to $mgl\\varphi_{1}$.\nYou wanted to let people know what $m,l,c,a$ are (constants I guess). \nI was wondering if something is wrong with the following array because after solving it for $\\varphi$ something is not true about $\\varphi_{2}$\n$\\begin{array}{l}\\varphi_{1}+\\varphi_{2}= \\varphi \\\\ \\varphi_{1}-\\varphi_{2}= \\varphi \\\\ \\end{array}$[/quote]\r\n\r\n1. $mgl\\varphi_{1}$ is correct not $msgl\\varphi_{1}'$\r\n2. Yes $m,l,c,a$ are constants\r\n3. About the \"array problem\" you mentioned, the two equations are true after approximations." } { "Tag": [ "Support" ], "Problem": "It seems police practices have gotten out of control in the US with constant racial profiling always looking to pulling over black people to try and catch them ridin' dirty as well as tasering college students for not producing an ID or getting up on time when they want to use the library:\r\n\r\nhttp://helpychalk.blogspot.com/2006/11/ucla-student-repeatedly-tasered-for-not.html\r\n\r\n\r\nMore taser fun here\r\n\r\nhttp://portland.indymedia.org/en/2006/01/332829.shtml\r\n\r\nTasers have been common now for police to use probably because they are claimed not to be dangerous at all.\r\n\r\n\r\nHow brutal are the police in other countries? Would actions such as these be frowned down upon or are they common?", "Solution_1": "Our police are known for their alternating acts of heroism and callousness. Oh, and they have never heard of tasers.", "Solution_2": "Tasers are small weapons that disperse electricity to stun somebody, they are considered a \"safe\" alternative to standard practices and a good way to disarm people.\r\n\r\nIn Florida, it's even legal to taser minors.", "Solution_3": "They're either illegal in some 40 states (USA) or legal in some 40, can't remember which. I know that they are used commonly in prisons to break up fights and stuff. I think as long as they don't get so advanced that they start actually doing damage to people, they are fine, but should by no means be available to the public, even with a liscense (unlike handguns)", "Solution_4": "The media is out to sell stories, not report the truth.\r\n\r\nThe ones that have gotten confused about this are no longer in business, as the public won't support a press or something that isn't what is demanded.\r\n\r\nMost of the police in the USA, as elsewhere, are people just trying to do the best job they can. Some are drawn to the job for poor reasons, such as: they like to have power over others, they are part of a gang that use the police power to further gang goals, etc.\r\n\r\nRacial profiling is used for one reason; it is effective. For most purposes it isn't allowed to be used to directly or indirectly lead to a conviction of a criminal.\r\n\r\nBut let's think about this, what if your loved one was kidnapped by someone who was observed. Wouldn't you want the police to stop those of the same race/height/weight/gender as the witness identified? I would, but this is profiling.\r\n\r\nIf the police are stopping all black men drivers just because they are black, they are not being very productive. This will also clearly show up in their arrest records, so I think the actual occurrances of this, while real, are perhaps overstated. \r\n\r\nLet's be rational, how easy would it be for a defense counsel, looking for any way out for his client, to find out that a traffic officer issued 2 or 3 times as many tickets to purple people, or any other class than exist in the general population or than other officers did. The feds and ACLU would both love to have such a case. I don't believe that such things are still occurring. I will believe that those with the same race as the officer might be treated a bit different, as would an attractive young driver, by an officer who happened to be of the other gender. Also, what paper wouldn't want to run such a story?\r\n\r\nI won't argue that it doesn't occur, but I think the prevelence is far overstated by the press.\r\n\r\nA Taser is a less-lethal method of stopping someone. Wouldn't you want of officer to Tase a kid before he shoots one? I would. It should be legal, but just like shooting -- only in appropriate circumstances.\r\n\r\n(I understand that a Taser fires two darts that penerate the skin and a battery that powers an electric shock that is very painful, ususally causes the equivalent of a short seizure in a person hit by one and is usually not lethal.)\r\n\r\nOur local police force is obtaining them. Several officers asked to be shot themselves so they would know what they were subjecting people to if they decided to use it on a member of the public. Admirable. One was quoting saying that they wouldn't want to do anything they didn't experience themselves. (I don't think he extended this to shooting.)\r\n\r\nI hope this helps.", "Solution_5": "[quote=\"wizard2378\"]\n Several officers asked to be shot themselves so they would know what they were subjecting people to if they decided to use it on a member of the public. Admirable. One was quoting saying that they wouldn't want to do anything they didn't experience themselves. (I don't think he extended this to shooting.)[/quote]\r\n\r\nIn my state, i'm pretty sure it's required for police officers to go through the effects of thier various weapons, so that in court they can justify any action they may have taken with the weapon because they've felt it too. This includes tasers and pepper-spray, but of course not guns.", "Solution_6": "a) What you have provided is very small evidence for making a general statement about police practices in the U.S.\r\n\r\nb) Did you even read this part? [quote]Predictably, the coroner says the taser had nothing to do with his death.[/quote]\r\n\r\nc) Edit: after looking at it again, I change my mind - I think they did step over reach their boundaries and shouldnt' have gone that far." } { "Tag": [], "Problem": "\u039a\u03b1\u03bb\u03bf \u03a6\u03b8\u03b9\u03bd\u03cc\u03c0\u03c9\u03c1\u03bf \u03c3\u0384 \u03cc\u03bb\u03bf\u03c5\u03c2\r\n\r\n\u03a3\u03ba\u03b1\u03c1\u03c9\u03c3\u03b1 \u03bc\u03b9\u03b1 (\u0393\u03b9\u03b1 \u03b5\u03c5\u03ba\u03bf\u03bb\u03b7 \u03c4\u03b7\u03bd \u03c0\u03b7\u03b3\u03b1)\r\n\r\n\u039d\u03b1 \u03b1\u03c0\u03bf\u03b4\u03b5\u03b9\u03c7\u03b8\u03b5\u03af \u03b7 \u03b1\u03bd\u03b9\u03c3\u03cc\u03c4\u03b7\u03c4\u03b1\r\n \r\n$ (\\frac {\\sqrt {a^4 \\plus{} b^4} \\minus{} \\sqrt {b^4 \\plus{} c^4} \\minus{} \\sqrt {c^4 \\plus{} a^4}}{2})^2 \\plus{} \\frac {(|ab| \\minus{} |bc| \\minus{} |ca|)^2}{2}\\geq{c^4}$ \r\n\u03b3\u03b9\u03b1 \u03ba\u03b1\u03b8\u03b5 $ a,b,c$ \u03c3\u03c4o $ R^{*}$", "Solution_1": "[quote=\"xp\"]\u039a\u03b1\u03bb\u03bf \u03a6\u03b8\u03b9\u03bd\u03cc\u03c0\u03c9\u03c1\u03bf \u03c3\u0384 \u03cc\u03bb\u03bf\u03c5\u03c2\n\n\u03a3\u03ba\u03b1\u03c1\u03c9\u03c3\u03b1 \u03bc\u03b9\u03b1 (\u0393\u03b9\u03b1 \u03b5\u03c5\u03ba\u03bf\u03bb\u03b7 \u03c4\u03b7\u03bd \u03c0\u03b7\u03b3\u03b1)\n\n\u039d\u03b1 \u03b1\u03c0\u03bf\u03b4\u03b5\u03b9\u03c7\u03b8\u03b5\u03af \u03b7 \u03b1\u03bd\u03b9\u03c3\u03cc\u03c4\u03b7\u03c4\u03b1\n \n$ (\\frac {\\sqrt {a^4 \\plus{} b^4} \\minus{} \\sqrt {b^4 \\plus{} c^4} \\minus{} \\sqrt {c^4 \\plus{} a^4}}{2})^2 \\plus{} \\frac {(|ab| \\minus{} |bc| \\minus{} |ca|)^2}{2}\\geq{c^4}$ \n\u03b3\u03b9\u03b1 \u03ba\u03b1\u03b8\u03b5 $ a,b,c$ \u03c3\u03c4o $ R^{*}$[/quote]\r\n\r\nCounterexample: $ (a,b,c)\\equal{}(1,0.5,0.45)$", "Solution_2": "\u03c0\u03b1\u03c1\u03b5 \u03c9\u03c2 \u03c0\u03c1\u03bf\u03c5\u03c0\u03cc\u03b8\u03b5\u03c3\u03b7 \u03c4\u03b1 \u03b1,b,c \u03c4\u03b5\u03c4\u03bf\u03b9\u03b1 \u03c9\u03c3\u03c4\u03b5 \u03bf\u03b9 \u03b2\u03b1\u03c3\u03b5\u03b9\u03c2 \u03c4\u03c9\u03bd \u03c4\u03b5\u03c4\u03c1\u03b1\u03b3\u03c9\u03bd\u03c9\u03bd \u03bd\u03b1 \u03b5\u03af\u03bd\u03b1\u03b9 \u03b8\u03b5\u03c4\u03b9\u03ba\u03b5\u03c2.", "Solution_3": "[quote=\"xp\"]\u039a\u03b1\u03bb\u03bf \u03a6\u03b8\u03b9\u03bd\u03cc\u03c0\u03c9\u03c1\u03bf \u03c3\u0384 \u03cc\u03bb\u03bf\u03c5\u03c2\n\n\u03a3\u03ba\u03b1\u03c1\u03c9\u03c3\u03b1 \u03bc\u03b9\u03b1 (\u0393\u03b9\u03b1 \u03b5\u03c5\u03ba\u03bf\u03bb\u03b7 \u03c4\u03b7\u03bd \u03c0\u03b7\u03b3\u03b1)\n\n\u039d\u03b1 \u03b1\u03c0\u03bf\u03b4\u03b5\u03b9\u03c7\u03b8\u03b5\u03af \u03b7 \u03b1\u03bd\u03b9\u03c3\u03cc\u03c4\u03b7\u03c4\u03b1\n \n$ \\left[\\frac {\\sqrt {a^4 \\plus{} b^4} \\minus{} \\sqrt {b^4 \\plus{} c^4} \\minus{} \\sqrt {c^4 \\plus{} a^4}}{2}\\right]^2 \\plus{} \\frac {(|ab| \\minus{} |bc| \\minus{} |ca|)^2}{2}\\geq{c^4}$ \n\u03b3\u03b9\u03b1 \u03ba\u03b1\u03b8\u03b5 $ a,b,c$ \u03c3\u03c4o $ \\mathbb{R^{*}}$[/quote]\r\n\r\n\u0398\u03b1 \u03c7\u03c1\u03b7\u03c3\u03b9\u03bc\u03bf\u03c0\u03bf\u03b9\u03ae\u03c3\u03bf\u03c5\u03bc\u03b5 \u03c4\u03b7\u03bd \u03b1\u03ba\u03cc\u03bb\u03bf\u03c5\u03b8\u03b7 \u03b1\u03bd\u03b9\u03c3\u03cc\u03c4\u03b7\u03c4\u03b1:\r\n\r\n$ \\left(a^{p}_{1} \\minus{} a^{p}_{2} \\minus{} a^{p}_{3}\\right)^{\\frac {1}{p}} \\plus{} \\left(b^{p}_{1} \\minus{} b^{p}_{2} \\minus{} b^{p}_{3}\\right)^{\\frac {1}{p}}\\geq \\left[\\left(a_1 \\plus{} b_1\\right)^{p} \\minus{} \\left(a_2 \\plus{} b_2\\right)^{p} \\minus{} \\left(a_3 \\plus{} b_3\\right)^{p}\\right]^{\\frac {1}{p}}$,\r\n\u03bc\u03b5\r\n$ 0 < p < 1$\r\n\r\n\u039f\u03c0\u03cc\u03c4\u03b5 \u03b3\u03b9\u03b1:\r\n\r\n$ a_1 \\equal{} a^4 \\plus{} b^4,a_2 \\equal{} b^4 \\plus{} c^4,a_3 \\equal{} c^4 \\plus{} a^4\\wedge b_1 \\equal{} 2a^2b^2,b_2 \\equal{} 2b^2c^2,b_3 \\equal{} 2c^2a^2\\wedge p \\equal{} \\frac {1}{2}$ \r\n\r\n\u03ad\u03c7\u03bf\u03c5\u03bc\u03b5 \u03cc\u03c4\u03b9 \r\n\r\n$ \\left(\\sqrt {a^4 \\plus{} b^4} \\minus{} \\sqrt {b^4 \\plus{} c^4} \\minus{} \\sqrt {c^4 \\plus{} a^4}\\right)^{2} \\plus{} \\left[\\sqrt {2}\\left(\\left|ab\\right| \\minus{} \\left|bc\\right| \\minus{} \\left|ca\\right|\\right)\\right]^{2}\\geq \\left(\\sqrt {\\left(a^2 \\plus{} b^2\\right)^{2}} \\minus{} \\sqrt {\\left(b^2 \\plus{} c^2\\right)^{2}} \\minus{} \\sqrt {\\left(c^2 \\plus{} a^2\\right)^{2}}\\right)^{2}$$ (1)$\r\n\r\n\u0394\u03b9\u03b1\u03b9\u03c1\u03ce\u03bd\u03c4\u03b1\u03c2 \u03c4\u03b7\u03bd $ (1)$ \u03bc\u03b5 \u03c4\u03bf $ 4$ \u03c0\u03b1\u03af\u03c1\u03bd\u03bf\u03c5\u03bc\u03b5:\r\n\r\n$ \\left[\\frac {\\sqrt {a^4 \\plus{} b^4} \\minus{} \\sqrt {b^4 \\plus{} c^4} \\minus{} \\sqrt {c^4 \\plus{} a^4}}{2}\\right]^{2} \\plus{} \\frac {\\left(\\left|ab\\right| \\minus{} \\left|bc\\right| \\minus{} \\left|ca\\right|\\right)^{2}}{2}\\geq \\frac {\\left(a^2 \\plus{} b^2 \\minus{} b^2 \\minus{} c^2 \\minus{} c^2 \\minus{} a^2\\right)^{2}}{4} \\equal{} c^{4}$", "Solution_4": "\u03a9\u03c1\u03b1\u03af\u03bf\u03c2 \r\n\u0397 \u03b1\u03bd\u03b9\u03c3\u03cc\u03c4\u03b7\u03c4\u03b1 \u03bb\u03b5\u03b3\u03b5\u03c4\u03b1\u03b9 \u0392elman \u03ba\u03b1\u03b9 \u03b9\u03c3\u03c7\u03cd\u03b5\u03b9 \u03b3\u03b9\u03b1 \u03b8\u03b5\u03c4\u03b9\u03ba\u03ad\u03c2 \u03b2\u03b1\u03c3\u03b5\u03b9\u03c2 .\u0392\u03bb\u03b5\u03c0\u03b5\u03b9\u03c2 \u03c4\u03bf \u03b2\u03b9\u03b2\u03bb\u03b9\u03bf \u03c0\u03bf\u03c5 \u03b5\u03af\u03c7\u03b5 \u03c4\u03b7\u03bd \u0392elman \u03c4\u03bf \u03b5\u03af\u03c7\u03b5 \u03be\u03b5\u03c7\u03ac\u03c3\u03b5\u03b9 \u03b1\u03c5\u03c4\u03bf, \u03b3\u03b9\u03b1 \u03b1\u03c5\u03c4\u03cc \u03ba\u03b1\u03b9 \u03b5\u03bc\u03c6\u03b1\u03bd\u03af\u03c3\u03c4\u03b7\u03ba\u03b5 \u03c4\u03bf \u03c0\u03c1\u03bf\u03b2\u03bb\u03b7\u03bc\u03b1\u03c4\u03b1\u03ba\u03b9......", "Solution_5": "\u0391\u03c2 \u03b4\u03ce\u03c3\u03bf\u03c5\u03bc\u03b5 \u03bb\u03bf\u03b9\u03c0\u03cc\u03bd \u03ba\u03b1\u03b9 \u03c4\u03b7 \u03b3\u03b5\u03bd\u03b9\u03ba\u03b5\u03c5\u03bc\u03ad\u03bd\u03b7 \u03b1\u03bd\u03b9\u03c3\u03cc\u03c4\u03b7\u03c4\u03b1 $ Bellman$.\r\n\r\n\u0388\u03c3\u03c4\u03c9 $ 0 0 and \u03bc(B) > 0 . Let further n be a positive integer and \u03bb=$ \\frac{1}{n}$. Show that there exists a subinterval (c, d) of (0, 1) for which \u03bc(A$ \\cap$ (c, d)) = \u03bb\u03bc(A) and\r\n\u03bc(B $ \\cap$(c, d)) = \u03bb\u03bc(B) . Show further that this is not true if \u03bb is not of the form $ \\frac{1}{n}$.", "Solution_1": "As a general preliminary remark we should additionally assume $ A$ and $ B$ be measurable sets. [quote=\"Mimmzy\"]Show further that this is not true if \u03bb is not of the form $ \\frac {1}{n}$.[/quote]I don't understand, since there is an easy counterexample for every real $ \\lambda\\in(0,1)$:\r\n\r\nThen let $ A\\equal{}(a_1,a_2)$ and $ B\\equal{}(b_1,b_2)$ be two intervals with $ 0